Sunteți pe pagina 1din 612

[

y
y +
x
y
z

yz z + b = 0

z =
xz
E

y
+
=
+
+
b
0
3
z
,33
y +
= 1
( +
x
z
y
3
z
r
r r
x
z =
10 5

+ b = 0
t (n ) =n

3
)
z
,
E
3
=
( +
323x1 a r4ax 2
4ax 2 z
2,31
x
n ) = 0 2 x1 a
10 5
(

x
1

t
=

n
xy
0
x +
8
xz y =
4ax 2 2ax1 x +
z )=
+
=
+
b
0

10 5

4
ax
2
ax
2
2
1x
y =

G
x
y
z
x
,
y =
3

2 2

1
4,
+
8 (2 x1a )(2ax1 ) 16
2 2
x(22ax a=0 )(2ax )2,16
10 5
xy
x xy
1 xyzy = 1 1 2 x1 a5x120a 5=40ax 2 348
y =
y
10 6
=0
2
2
+
+
+G2by = 0 2

4,13
(4ax 2+) +

=
(
)
2
x
a

1
x
y

x =
x
y (2 x1xaz ) =xy(4=ax02 )x41 ax 2 2ax1
48

dx1 t
0 5
x
1

=
y
2
2
2
x
a
4
ax

E x 10 6

1
2
(2x11 a==2 xd1)(
xy y
a 41ax2 ) 16 x 2 a = 0

1yz +
2 x1 a = 4ax
xz
z
=xb4ax

2 + 2
t 2ax
+
+
X1 x1

+
=

G
x
a
0

=
+
t
1
2
z
=
y
y = 1
y2 2 x1za 4ax 2 0 ( =02x )a2 +
x1
x =l2n1
x
x
y
y + 4ax2 x
2
x
a

4
ax

=
1
(
)
2
1
2
x
2
x
a
4
ax

2
1
2
= t
=
z =
0
E
dt
E

dx2
Xx
t
y x(4ax 2 2ax1 3,3
yz

xz
33
0
1 1=2 x1 a 4ax 2 x1 =

ln 1

=
d
+
+
z = 1
t
r
1
x =+

=
y
X
x22 x1 a = 4ax 2
10 52 2 t (n ) = n X
y +
1xexp t y

t
2

=
(
)(
)

16 x a
0

1 2 x a z ) =2ax1
E z
0 2t + 3
E lyn2=x22 x1 a + 4axz2 = 3
2 2xx1 =a X 2 4ax
dt
1
x
=

,33
3
( = ln
= 1
X3
2
1182)2 1 exp t
3
+2x1 a )2 = (,43ax
x (
=0
z
X2
2t=+ 13

r
x + 2t +
10 5

1
x
5

0
l
4
ax
2
ax
n
t(

y
2
1
l
n
)
G xy =
z
E

==ln4
=
z
3
2 x1 a r4axr2
2
2,5 X

,3 +2 ( 1= )(
r r

+ 1b =)0 16 x 22 a 2 = 0 +xb
,31
1
8 3 2xln1a = 22xax
x
1
10 25x21a = 24t4ax
=
0
+
=
y
x
8
+

a
4
ax
1

1
2
0 6 23 2
10 5

2
y
G xy = x1
=
G xy =
(2 x1 a ) = (4ax 2x )
0 dx
4,3
2,5
t

2 = X
2
1
48
r
x
1
r
2
=
y

1
t
+
=
5
0+ b = 0
1 = 23x1 a 14ax 2
10 6
x

dt

X1 x1

2 x a = 4ax
G xy = x1
y =
2 x1 a 4a
0
ln x1 1 x + 2 xy +2 xz= 2+x1 ab+=4ax
0 x2

Eduardo
V.
Chaves
0xz dx W.
xy
t

= t
x x 0x= 2+1
+ 1b = 0
+
y
4ax 2 2ax1
z
y
dx2
X1
t
x

2 x1 a
zX x =x dt
x
xy

x
=
dt
=
G
1

1
x (
xy r 1 y X1exyzp t
y =
X 2 x2
0
2 )(2a2
l

0
r
n
x1 a4ax
2

+ by = xy
x + +b = 0 +
0 + y + xyz2 +
bx =0 xy 2x1xza =4tax
0 2t +13
2
l

d
2
=

X
yt+
y
x z
x3 = X x
+ b1 =20x a
+

x =x1
y
x z x2=
1
(2x1a=)02 =
x
z4ax 2 x2(ax
3
dty
x

X 2 = ln 2

E x
1
t
x
+3
X2
E

ln 3z + b =2x0xz1 a yz 4ax

z2 2 xy0 2t +3y
xz +x yz
+
(2x1 a=)
z
y = 1

bx)(
4
+= 0b z += 0 + (yz2 x+1 aln
+
+
y + y = 1
22
a1
x3 = X
y y +z
=2x01ax
x
r1 rxx

y
=

z =
= 2X y 2ax
y
x
E y
X ln
2 1 z 3x
3E
, x yxy xz xz+==b = 0 42 ax
(z2 x1 a 2)2= 2(4xaxa 22)t 2+
(
2

3
2 x=1 a 1 4ax
t
+
+
=
+
+
b
3
0
3

1
2
,
1
E
x
33x 3(2 x a 3)(2ax ) 16 x2 ayz2 = 0
3 y ( zr n
z
x =+0 z = 1x
1
( )

xz

z
1
1
2
5 x +t

r
0

n
n
+
+
+
=
b

1
( )
4Eax 2 2ax1
0

1
0 5
z )=
=
z

2xyz y yzyz =) = (2 x a y + )2 t= (4ax


)x2n y z2 x1 a z = 4
ax
E

2
4a
2 x1 a

1
2
z =
Eby2=,y30
+
+

xy
= (12x x=1 a 1 )(2axx 1 + )xy 16= x 221a 2,3=108 +x +
3

,33 = 2 x a 4ax 4ax


x=z x1 xy 18 (xz
x 10y5
ax
2

3
y
2
1
z
1 r
2
1
+
+ 10x ++5 b = 0
)2 = (4ax )2 = G x
G(2 xx1yaE= 2

= n
4,3 y = 2 yz y E=x 4 y 2xz1 a z =)x=4ax 2 10=52 x a t+(n4)ax
x5
2

,
xz
4
z

2
xy = 1
1
2 (2 x1 a )(2a
2
10 5y xy+ =1 = 8 1 +,5 1 + + b,z3z4=80
y = 1
,31
2x1ax 04ax62y 0 xy5 =z 1 xy

y 10
yz

+
6
x
8
G x2yEx1=a x1 = 4ax 2 Gz =
1 5
+
+ by = 0
(2 x1 a )2 =
+
0 y dx t x =12 =xy32,=x10a +x14ax 2
0

x
y
x
=
3
z
G x
xy = 1
1(
4,3
z = 1
=xxy+d= 1E 33 1dx1 = t r (n ) y = 2,5

5
x
x

t
x
0
1
X

y =dtt =xz n

2 x1 a = 4
G xyE = 1 1 Gz )

1
yz

x
z
5
0 + b = 0 8 10 6
4ax

x111 a21
+
+
xy = ln 2Xx

0y = 1 =
0 z x2
2

x
x
a
4
ax
+
y

0
l
0
G
n
=
1
2

1
2

x
x
0

= 0 1=y 0 =z
,31 Xx2 = t
yx

dx
xy = 1
t
+
x
8
z
4
ax
2
ax

dx t t
= 1
2
E y 1d1x02xy524=ax
(+ = t12dt3,323ax3x11=XX1xeXx1p1 =t2r 2dt= x1 = X + br = 0r
G xy =X 2 x2 =z = y d1t =
4,
X 2x x2(2x1 aG )(
1
2,5

x 1 ex p t
2 a(n
=)(12ax10)1516
3
(2 x a xy2ax
) x16
0 2t +
x )2 a02 =
0
xy = 1
x3 = X 10 5 E 3z 48 x1ln x2 z)0 =21t+ 23 0 1x2 2 ln x0t2 2 = n ln 1 =

r0=XX
3
rx1 a=ln2), = (24ax 2 )dx 2 t
(620
X 1 t
x = 1

G xy = x1 x3y = 1
3t1+)8 = (4ax22 ) X = ln

b
=
23 (2 x1 a 2

+
x
3

0 dx
x
2
=
d

t
2

+
=
t

t
1
1X 20 lxn5231 = 2 x1 a 4ax 2 3 ln
X1 E

e
tx

2= 1
1
x
y
p
=+2 x1 a 4ax 2 3
1
=0
x =
= G
=
0 21t
x1=a2
x xy y = xz1 x =
dt y
3
x
2,5 2
x41,a34 4=axx24ax
X1 x1
x

l
n
G xy =

=
+
2
2+ =
x+ = X + 2bEx = 0x y +
X 22 2 = +
24 ax
8 3 = X 3 2 22 x=1 a2 x1 4aax
ln1 0x15
0 x2 xy = 01

10 6
Xt 2+1x ln x 2=ty +1 2 E
z 2 y t+

=
3

3 ln
y = 31 1(
dx2 Gt xy = x1 X1 xy t

xy E
0 dxx + t + xz + bx == 0

x1 a 4ax 2
zy = x1

+
x
yz
=
d

t
1
x
1x
1
X1 exp t
+
+ +E b y y
=0 x

r
r
X 2 yx2=

=
x
y
z
(2
x = 1
x
x1 a + 4ax 2
y +
dt
+ b = 0 x y = 1y Ez x=z z 1
0 2t +
X
x

x3 = X G xy = 30 1 1xy ln0x2y

z
l
n

Exy = y1E x + = 3x
E x

x2
= ln yz + b 1 = =0

+
+
X

dx
t

z xz= +1 yzG+ x(z+ zb


y ,
x = 1
1
2x t y2 x z 2t +X31y
z =0
y =

+
x
+
y
l
=
y
n

x y xy = z1 2,
=
dt
3 x1 r= Xr e tz
5 z ) = x +
E x
X 2 x2
Exy = z 1
E
y+b = 01 xp
=

yz

2
3

(x2 = X 2 ,33
z

xz 0+ t + 3 +
xz 20= 1
+ lnbz =
x Gxx =+xy1 = 2 0 5 2,3
3
y = 1
x
1

G
2

=
y + x3 = X
y
x
,5
r
+
x
y

1
y
z
3 t + 1 10 xy5 = 1t=(n0) E
3x
t 10 5

=dy
n
X 2 E= ln 2t +
z =
x
)
E y
z
=
x
x
G
1
=
y = xy1 =
x
z 3
3,33
(
ln 3 2,3
r= 1r
2,G
x1= 4d,3

y
=

5
1
x
t4yt8+
3
z b = 0
1 5
xy = 118 G
xy = 1
r
x +
xy =y =10 X15 01
x +

n
x
(
)

01

t = n
xy
1x0y =5 21 0 E x2
z )=
= d
E z
=bx20x==y X
y
G xy = x + x + +xyy +=xz+xzb+xG
x1
=
2
x
t

2
X
0
d
t

G
(
x
1
t

1
0
+
4
x

2
x
y
z

=
y
,
,
2
d
1
3
,
t
1
5x
z3 = x1
y
18
0 x2 = xdt0+
xyz=341
= 1
x +
xy = 1
8

0 5
=X x
10 5

1 6

y =
x22t +t z
G xy = xyx1+xy y + y yz+yzGb yx=y 00= X1 x1E 2z dt2 d0
4
G
xy xy = xz2,

0
0
= l3nd
+
+
+
=
b
0
,

0
x
1

348
5+ b x = 0
x =
y
x dx1 yt x z
xy x2= 1
3 = X
1 +
1
X 2 x2x +

x
=
x
1
=
y
1

x
y
x
3
0 5
z
dx2 t
X
10 6 E
z
y
0 y2t +
X x = dt
x G
G =xy = xdt =
G xy = x1
E
1
yz

1
x

x
xz
z

=
y
1

23,5 X 3
y 0 yz dx t
0 x2 +xz +0 +yz + + z bl+nz=bx10=xy 0=X 2 1x2
y =
y +
z = t
5
2

y = 1
t
+ 3 10
+
+1 =b y = 0
x
y
z
0
+
y


z X 1
dx2x t y
ln
x
z
x3G=
x y
E
dt
=
Xx1 z x1

y
z
X 3y = x1 =x1
E y(

3,3=33 =
x
y =
d
t
0

0
X

0
l
n z 1= 1
dx1 expt t
X x32,3
(+
33

x
1

ij =

X 0

0
2X
0
X 0

2 3
l
2X

00
l2 3
X 2

X 2 X
0

2
l2

2
2
0
l

2
l

ij =
3
X

2X
l 2
X
0

l 2 3
X
2X

2X

2 3
0

2 3

0
0

ij =
X 2

3
X 2
l2
0
3

X 0

X l
2X
0
2X

2 3
l

l 2 3
0

X 2

X
2
l2
3

l 2
X

2X

0
ij =
l 2 3

X 0

2X

0
2 3
l

X 2

X
2
2
l
3

2
X l
2X
l 2 3

)]

) ( )

] ) ( )

)]

)]

[ [

[ [

( (

)] )]

=
0
ij

() ( )
)]

)]

[(

[
(
)
[ ( )[
[ (
[
[ ( )]
( )]
[
]
(
[)]

(
)]

) ( )[
[(

=
0

ij

0
ij =

0 0
0
ij =
X
2X
0 0
Xl 2 0 3

X 22 X3

0
l
32

X
2
l 2
X 2 20 3
3
X l X 2
l2
3
2lX32
2
X
l
2X

l 2 3

X 0

2X

3
2
0
l ij =
0

X
2

0
3 0

X
2
X 0
ij 2=
X3
l
2X
00 0l 2 2 X

3
X
2
l
X 2 l 2 3
2 XX
0
0
X 2

l 22 X3332
3

X
l

2
X
2
2

2
3
l

0
3

X 2

2

l 2
3
X l
X
2
l2
2X
23 X

2
l 2 3
X ll 2 3
2X

l 2 3

ij

2
2

=
0

X 2
3

2
X l
2X
l 2 3

l
X 2
3
l2

)]

X 0
2X
2 3
l

X 2

X
2
l2
3

2
X l
2X
l 2 3

([ )

(( )
)] [
)] [
(
] [
]
[
(
)]
(
)]
)] [
]
(
(
)]
)]
[
]
(
[
)]
)

[
]
(
( )
( )
( )
)]
( )

)]

(
MECNICA DEL MEDIO
CONTINUO

=
0

ij

)]

ij

)]

=
0

X 0
2X
2 3
l
0
X 2

X
2
l2
3

2
X l
2X
l 2 3

X l
2X

PROBLEMAS RESUELTOS DE

X
2

)]

)]

2
l
X 2
3
l2

)]

II

MECNICA DEL MEDIO CONTINUO

Nomenclature

III

Problemas Resueltos de

Mecanica
del Medio Continuo
EDUARDO WALTER VIEIRA CHAVES

IV

MECNICA DEL MEDIO CONTINUO

Presentacin

fus

i n

s
Sue
lo

-di

Hi

Fl
uid
os

jo
Flu

Mec
. de

Co
nv
ec
ci
n

T rm i

co

Presentacion

ic
ul

dr

ras
Estructu

Vi g

as

Placa
s

s
do
i
l
S
PVCI y Estrategias de Solucin

Mov. Slido Rgido

Ecuaciones Constitutivas

Ecuaciones Fundamentales de MMC


Tensiones
Cinemtica del continuo

Tensores

VI

MECNICA DEL MEDIO CONTINUO

para el Alumno
Guia
1) NO SE MEMORIZA EJERCICIO.
2) Una vez que la teora haya sido estudiada, intentar resolver los ejercicios sin mirar la
solucin. Es importante que el alumno ante un nuevo problema desarrolle la habilidad de
dar la solucin al problema con los conocimientos adquiridos.
3) Tener en cuenta que, en general, un ejercicio es un caso particular de la teora. Es muy
importante saber reconocer cuando estamos ante una aproximacin del caso general.
4) A veces, la solucin de un ejercicio se puede obtener por varios caminos. Una vez
resuelto el ejercicio, intentar verificar si existe otra forma de resolverlo.
5) Cuidado, puede haber erratas, seis crticos...

Contenido

Contenido
ABREVIATURAS ............................................................................................................................................... IX
OPERADORES .................................................................................................................................................... X
UNIDADES (SI) ................................................................................................................................................ XI
NOTACIN .................................................................................................................................................. XIII
FRMULAS TILES ................................................................................................................................... XVII
1 TENSORES ......................................................................................................................... 1
1.1 VECTORES, NOTACIN INDICIAL ..................................................................................................... 1
1.2 OPERACIONES CON TENSORES DE ORDEN SUPERIOR ............................................................... 14
1.3 TRANSPUESTA ...................................................................................................................................... 20
1.3.1 Simetra y Antisimetra ........................................................................................................... 20
1.4 COFACTOR. ADJUNTA. TRAZA. TENSORES PARTICULARES. DETERMINANTE ....................... 26
1.5 DESCOMPOSICIN ADITIVA DE TENSORES .................................................................................. 41
1.6 LEY DE TRANSFORMACIN. INVARIANTES. .................................................................................. 42
1.7 AUTOVALORES, AUTOVECTORES Y TRANSFORMACIONES ORTOGONALES ........................... 49
1.8 REPRESENTACIN ESPECTRAL ........................................................................................................ 58
1.9 TEOREMA DE CAYLEY-HAMILTON ................................................................................................. 63
1.10 TENSORES ISTROPOS Y ANISTROPOS ...................................................................................... 82
1.11 DESCOMPOSICIN POLAR............................................................................................................... 91
1.12 TENSOR ESFRICO Y DESVIADOR ................................................................................................. 91
1.13 OTROS ................................................................................................................................................. 92
1.14 NOTACIN DE VOIGT ..................................................................................................................... 92
1.15 CAMPO DE TENSORES....................................................................................................................100
1.16 TEOREMA CON INTEGRALES ........................................................................................................122
2 CINEMTICA DEL CONTINUO ................................................................................. 153
2.1 DESCRIPCIN DEL MOVIMIENTO, DERIVADA MATERIAL, VELOCIDAD,
ACELERACIN.................................................................................................................................153
2.2 TENSORES DE DEFORMACIN FINITA, DEFORMACIN HOMOGNEA ..............................181
2.3 DESCOMPOSICIN POLAR DEL GRADIENTE DE DEFORMACIN ..........................................225
2.4 DEFORMACIN INFINITESIMAL ....................................................................................................244
3 TENSIONES ................................................................................................................... 257
3.1 FUERZA, TENSORES DE TENSIONES, VECTOR TENSIN..........................................................257
3.2 ECUACIN DE EQUILIBRIO, TENSIONES Y DIRECCIONES PRINCIPALES ..............................268
3.3 OTRAS MEDIDAS DE TENSIN.......................................................................................................278
3.4 MXIMA TENSIN DE CORTE, CRCULO DE MOHR ..................................................................279
3.5 PARTICULARIDADES DEL TENSOR DE TENSIONES ....................................................................286
3.6 ESTADO TENSIONAL EN DOS DIMENSIONES .............................................................................302
3.7 TENSIONES EN COORDENADAS CILNDRICAS Y ESFRICAS ...................................................309

VIII

MECNICA DEL MEDIO CONTINUO: PROBLEMAS RESUELTOS

4 LEYES FUNDAMENTALES DE LA MECNICA DEL MEDIO CONTINUO..........313


4.1 INTRODUCCIN A PROBLEMAS DE FLUJO ................................................................................... 326
4.2 INTRODUCCIN A MOVIMIENTO DE SLIDO RGIDO ............................................................. 334
5 INTRODUCCIN A: ECUACIONES CONSTITUTIVAS, PVCI, Y ESTRATEGIAS DE
SOLUCIN DEL PVCI ..................................................................................................371
6 ELASTICIDAD LINEAL..................................................................................................461
6.1 ELASTICIDAD TRIDIMENSIONAL ................................................................................................... 461
6.2 ELASTICIDAD BIDIMENSIONAL ..................................................................................................... 500
6.3 INTRODUCCIN A ELEMENTOS ESTRUCTURALES 1D ............................................................. 534
6.4 TORSIN ............................................................................................................................................. 549
6.5 ENERGA DE DEFORMACIN PARA ELEMENTOS 1D ............................................................... 565
6 BIBLIOGRAFA .............................................................................................................. 585

Abreviaturas

Abreviaturas
PVCI
PVC
MEF
MEC
MDF
MMC
sii

Problema de Valor de Contorno Inicial


Problema de Valor de Contorno
Mtodo de los Elementos Finitos
Mtodo de los Elementos de Contorno
Mtodo de las Diferencias Finitas
Mecnica del Medio Continuo
si y solo si

Latin
i.e.
et al.
e.g.
etc.
Q.E.D.
v., vs.
viz.

id est
et alii
exempli gratia
et cetera
Quod Erat Demonstrandum
versus
vidilicet

es decir
y otros
por ejemplo
y as sucesivamente
lo que se quera demostrar
versus
a saber

Alfabeto griego
(a)
(b)
(c)
(d)
(e)
(f)
(g)
(h)
(i)
(j)
(k)
(l)
(m)

- alfa
- beta
- ji
- delta
- psilon
- fi
- gamma
- eta
- iota
- fi
- kappa
- lambda
- mu

(n)
(o)
(p)
(q)
(r)
(s)
(t)
(u)
(v)
(w)
(x)
(y)
(z)

- nu
- micron
- pi
- theta
- ro (rho)
- sigma
- tau
- ypsilon
- sigma
- omega
- xi
- psi
- dseta

Operadores

Operadores
=

+
2

Tr ()

() T
() 1

parntesis de MacAuley
norma Euclidiana de
traza de ()
transpuesta de ()

() esf

inversa de ()
inversa de la transpuesta de ()
parte simtrica de ()
parte antisimtrica de ()
parte esfrica de () o parte hidrosttica

() dev

parte desviadora de ()

mdulo de
salto de
producto escalar
determinante de ()
Cofactor de ;
adjunta de ()
traza de ()
doble producto escalar
operador diferencial escalar (Laplaciano)
producto tensorial
gradiente de
divergencia de
producto vectorial
Primer, segundo y tercer invariantes del tensor

() T
() sym
() anti

[[]]

det()
cof ()
adj( )
Tr ()
:
2

grad()
div ()

I , II , III
D
&
Dt
r

1
I
I sym I

Derivada material de
Vector
Vector unitario (versor)
Tensor identidad de segundo orden
Tensor identidad de cuarto orden
Parte simtrica del tensor identidad de cuarto orden

Unidades (SI)

Unidades (SI)
m - metro
kg - kilogramo
s - segundo
K - Kelvin

longitud
masa
tiempo
temperatura

velocidad
aceleracin
energa
fuerza
presin, tensin
conductividad trmica:
frecuencia

m
s
m
s2
J = Nm - Joules
N - Newton
N
Pa 2 - Pascal
m
W
mK
1
Hz Hertz
s

corriente elctrica
cantidad de sustancia
intensidad luminosa

energa, trabajo, calor


potencia
permeabilidad
viscosidad dinmica
flujo de masa
flujo de energa
densidad de masa
densidad de energa

Prefijo
pico
nano
micro
mili
centi
deci

Smbolo

Potencia

10
10 12

m
c
d

10

10 6
10 3
10 2
10

Prefijo
kilo
Mega
Giga
Tera

A - ampere
mol - mol
cd - candela

J = Nm - Joules
J
W Vatio
s
m2
Pa s
kg
m2s
J
m2s
kg
m3
J
m3

Smbolo

Potencia

10
10 3

M
G

10 6

1012

10 9

XII

MECNICA DEL MEDIO CONTINUO: PROBLEMAS RESUELTOS

Constantes Fisicas

Constante de gravitacin Universal de Newton: G = 6.67384 10 11


Velocidad de la luz en el vaco: c = 299 792 458

m
m
300 000 000
s
s

m3
kg s 2

Notacin

Notacion
r r
r r
A( X , t ) a ( X , t ) Aceleracin (configuracin de referencia)

Matriz de transformacin de base

r r
a ( x, t )

Aceleracin (configuracin actual)

B0
B
B

Medio continuo en la configuracin de referencia - t = 0


Medio continuo en la configuracin actual - t
Contorno de B (frontera)

r r
b( x , t )
b

m
s2

N
m3
Tensor izquierdo de deformacin de Cauchy-Green, m 2
tensor de deformacin de Finger
m2

Fuerzas msicas (por unidad de masa)

Tensor de deformacin de Piola

Entropa creada interiormente

m
s2

J
sK
J
Manantial de entropa local por unidad de masa y por
unidad de tiempo
kg s K

Tensor constitutivo elstico


Matriz elstica (notacin de Voigt)
Tensor constitutivo inelstico
Tensor de deformacin de Cauchy
Calor especfico a volumen constante
Calor especfico a presin constante
Cohesin

cc

Concentracin

Tensor derecho de deformacin de Cauchy-Green

DV

Deformacin volumtrica

Tensor velocidad de deformacin o tensor tasa de


deformacin o tensor tasa de deformacin Euleriana o
tensor estiramiento
Diferencial de rea en la configuracin de referencia
m2
Diferencial de rea en la configuracin actual
m2

Ce
[C ]

in

C
c
Cv

Cp

r
dA
r
da

Pa
Pa
Pa

Pa
mol
m3
m3
m3

MECNICA DEL MEDIO CONTINUO: PROBLEMAS RESUELTOS

XIV

Diferencial de volumen
m3
Tensor material de deformacin Green-Lagrange, tensor m 2
de deformacin de Green
m2

dV
E

i , j, k

Tensor de deformacin finita Euleriana o tensor de m 2


deformacin de Almansi
m2
Mdulo de elasticidad longitudinal o mdulo de Young Pa
Base Cartesiana en notacin simblica
Base Cartesiana

Gradiente de deformacin

G
H

Mdulo de elasticidad transversal


Tensor de deformacin de Biot

Entropa total

r
HO

Momento angular

Determinante del Jacobiano

e
E
e i

J ( X , t)
r

Tensor gradiente material de los desplazamientos

j ( x, t )

Tensor gradiente espacial de los desplazamientos

r
J

Tensor de difusividad

Tensor de conductividad trmica

Energa cintica

r
L

Cantidad de movimiento lineal

Tensor gradiente espacial de velocidad

m
M

Masa total
Tensor de tensiones de Mandel
Vector unitario normal a una superficie (configuracin
actual)
Vector unitario normal a una superficie (configuracin
de referencia)

r
r
p = b

Fuerza msicas por unidad de volumen

m
m
Pa
J
K
kgm 2
= Js
s
m3
m3
m
m
m
m
mol
m2s
W
J
=
mK smK
J
kg m
s
m
sm
kg
Pa

N
m3

p
p

Primer tensor de tensiones de Piola-Kirchhoff, tensor


Pa
de tensiones nominales
Presin media
Pa
Presin termodinmica
Pa

r r
q( x , t )

Flujo de calor o vector del flujo no convectivo

Tensor ortogonal

J
m2s

NOTACIN

Q
r
r ( x, t )

XV

R
S

Potencia calorfica
J
Funcin escalar que describe en forma espacial el calor J
generado por las fuentes internas por unidad de masa
kg s
Tensor ortogonal de la descomposicin polar
Segundo tensor de tensiones de Piola-Kirchhoff
Pa

Flujo de entropa

T
r r
t (n) ( x , t , n )
r (N )
t0
r
T ( x, t )
t
t0 t = 0

Tensor de tensiones de Biot


Vector traccin (configuracin actual)

Pa

Temperatura
Tiempo
Tiempo inicial

K
s
s
J
=W
s
J
kg

Tasa de la energa interna

Energa interna especfica

r
U( X , t
r
V ( x, t )

Pa

Pseudo vector tensin (configuracin de referencia)

U&

r r
u( x , t )
r r
u( X , t )

J
kg s m 2
Pa

Vector desplazamiento (Euleriana)


m
Vector desplazamiento (Lagrangiana)
m
Tensor derecho de estiramiento, o tensor de
estiramiento Lagrangiano, o tensor de estiramiento
material
Tensor izquierdo de estiramiento, o tensor de
estiramiento Euleriano, o tensor de estiramiento espacial

r r
r r
V ( X , t ) v ( X , t ) Velocidad (configuracin de referencia)

m
s
m
s
m rad
=
ms
s
J
=W
s
m
m
1
K

r r
v ( x, t )

Velocidad (configuracin actual)

Tensor spin o tensor velocidad de rotacin

w int
r
X
r
x

Potencia tensorial

Coeficiente de expansin trmica

ij
1 , 2 , 3

Delta de Kronecker
Deformaciones principales

Alargamiento unitario

ijk

Smbolo de permutacin, componentes del tensor Levi-Civita

Deformacin
deformaciones)

Tensor de deformacin infinitesimal

Vector posicin coordenada material


Vector posicin coordenada espacial

volumtrica

m
m

(para

pequeas m 3
m3
m
m

MECNICA DEL MEDIO CONTINUO: PROBLEMAS RESUELTOS

XVI

Entropa especfica

J
kg K

Mdulo de deformacin volumtrica

Pa

Difusividad trmica

Estiramiento

Constantes de Lam
Coeficiente de Poisson

Densidad de masa

Densidad de masa de la solucin

Densidad de masa del fluido


r

0 ( x, t )
r

( x, t )

r
N
r
S
m
1 , 2 , 3
r
oct
r
oct
max

Densidad de masa en la configuracin de referencia


Densidad de masa en la configuracin actual
Tensor de tensiones de Cauchy o tensor de tensiones
verdaderas
Componente normal del vector traccin
Componente tangencial del vector traccin
Tensin media
Tensiones principales
Tensin normal octadrica
Tensin tangencial octadrica o tensin de corte
octadrica
Tensin de corte mximo
Tensor de tensiones de Kirchhoff
ngulo de friccin interno
Energa libre de Helmholtz especfica (por unidad de
masa)
Densidad de energa libre de Helmholtz (por unidad de
volumen)

( ) = e

Densidad de energa de deformacin

ngulo de dilatancia
Tensor tasa del tensor de rotacin material

m2
s
m
m
Pa

kg
m3
kg
m3
kg
m3
kg
m3
kg
m3
Pa
Pa
Pa
Pa
Pa
Pa
Pa
Pa
Pa

J
kg

J
m3
J
m3

Formulas
Utiles

Algunas Identidades Trigonomtricas


sin( ) = sin( ) cos( ) cos( ) sin( )
cos( ) = cos( ) sin( ) m sin( ) sin( )
1
cos( ) cos( ) = [cos( + ) + cos( )]
2
1
sin( ) sin( ) = [cos( ) cos( + )]
2
1
sin( ) cos( ) = [sin( + ) + sin( )]
2
1
cos 2 ( ) = [1 + cos(2 )]
2
1
sin 2 ( ) = [1 cos(2 )]
2
+
cos( ) + cos( ) = 2 cos
cos

2 2
+
cos( ) cos( ) = 2 sin
sin

2 2
m
sin( ) sin( ) = 2 sin
cos

2 2

cos 2 ( ) + sin 2 ( ) = 1
sin( )
cos( )
1
sec( ) =
cos( )
1
cos( )
=
cot( ) =
tan( ) sin( )
tan( ) =

sec 2 ( ) + tan 2 ( ) = 1

sin( x)
=1
x 0
x

lim

1 cos( x)
=0
x 0
x

lim

MECNICA DEL MEDIO CONTINUO: PROBLEMAS RESUELTOS

XVIII

Lista de identidades trigonomtricas


http://en.wikipedia.org/wiki/Trigonometric_identity

Algunas Expansin en Serie


f
1 2 f
1 3 f
2
( x a) +
(
x

a
)
+
( x a ) 3 + L (serie de Taylor)
2
3
x
2! x
3! x
n
n

(
1
)
x 2 + L ; ( x < 1) (serie binomial)
(1 + x) n = 1 + nx +
2!
1
1
exp x = 1 + x + x 2 + x 3 + L
2!
3!
1
1
Ln(1 + x) = x x 3 + x 5 L
3!
5!
1 2 1 4
cos( x) = 1 x + x L
2!
4!
1 3 1 5
sin( x) = x x + x L
3!
5!
1
1
cosh( x) = 1 + x 2 + x 4 + L
2!
4!
1 3 1 5
sinh( x) = x + x + x + L
3!
5!
1
1

tan( x) = x + x 3 + x 5 + L x <
3
15
2

f ( x) = f (a ) +

Algunas Derivadas
d
d x
d
1
d
1
(exp x ) = exp x
;
(a ) = Ln(a) a x
;
[Ln( x)] =
;
[log a ( x)] =
dx
dx
dx
x
dx
xLn(a )
d
1 f ( x)
[Ln( f ( x))] =
dx
f ( x) x
donde e exp es el exponencial y Ln es el logaritmo natural, donde se cumple que:
Ln(exp x ) = x

and

d
[sin( x)] = cos( x) ;
dx
1
d
[arcsin( x)] =
dx
1 x2

exp Ln( x ) = x

d
d
[cos( x)] = sin( x) ;
[tan( x)] = sec 2 ( x)
dx
dx
d
d
1
1
;
[arccos(x)] =
;
[arctan(x)] =
2
dx
dx
1 + x2
1 x

Lista de derivadas
http://en.wikipedia.org/wiki/List_of_derivatives

FRMULAS TILES

XIX

Algunas Integrales

exp dx = exp
x

x dx = Ln( x)

f ( x)
exp f ( x ) dx = exp f ( x )
x

Ln( x)dx = xLn( x) x + C

u
= sin 1 + C
a
a u
du
1
u
= tan 1 + C
2
2
a
a +u
a

du

du
2

u a

u
1
sec 1 + C
a
a

Lista de integrales
http://en.wikipedia.org/wiki/List_of_integrals

Solucin de Funciones
Funcin cuadrtica
ax 2 + bx + c = 0

solucin

x=

b b 2 4ac
2a

( a 0)

Regla de Ruffini
http://en.wikipedia.org/wiki/Ruffini%27s_rule

Expresiones relacionadas con el crculo:


Ecuacin del crculo: ( x1 a ) 2 + ( x2 b) 2 = r 2
rea del crculo: A = r 2
Longitud de la circunferencia: C = 2r
x2
r

x1

XX

MECNICA DEL MEDIO CONTINUO: PROBLEMAS RESUELTOS

Expresiones relacionadas con la elipse:


x2
r
x

f2

f1

x1

Ecuacin de la elipse: x = r =
a 2 b2
a2
rea de la elipse: A = ab

Excentricidad: e =

p
1 + e cos

0 < e < 1 , donde a 2 =

p2
se cumple.
(1 e 2 ) 2

1 Tensores
La notacin indicial fue introducida por Einstein (1916, sec. 5),
who later jested to a friend, "I have made a great discovery in
mathematics; I have suppressed the summation sign every
time that the summation must be made over an index which
occurs twice..." (Kollros 1956; Pais 1982, p. 216).
Ref. (Wolfram MathWorld (Einstein Summation))

1.1 Vectores, Notacin Indicial


Ejemplo 1.1

Probar que si a y b son vectores se cumple que:

(ar br ) (ar br ) = (ar ar )(br br ) (ar br )

Solucin:

(ar br ) (ar br )

2
r r 2
r r
= a b = a b sin
r 2 r 2
r 2 r 2
r 2 r
= a b sin 2 = a b 1 cos 2 = a b
2
r 2 r 2
r r
r 2 r 2 r r 2
= a b a b cos = a b a b
r r r r
r r 2
= (a a ) b b a b
r r r 2 r r r 2
donde hemos considerado que a a = a y b b = b .

( ) ( )

Ejemplo 1.2
r

( )

r
a

r
b

cos 2

Probar que: si c = a + b , el mdulo de c puede ser expresado a travs de la siguiente relacin:


r
c =

r
a

r
r r
+ 2 a b cos + b
r

donde es el ngulo que forman los dos vectores a y b .


Solucin:
Partiendo de la definicin del mdulo de un vector se cumple que:
r r
a+b

Universidad de Castilla- La Mancha


Ciudad Real - Espaa

r r
= a+b

) (ar + br ) = ar ar + ar br + br ar + br br
Draft

Por: Eduardo W. V. Chaves (2014)

MECNICA DEL MEDIO CONTINUO: PROBLEMAS RESUELTOS

r r

r r

r r

Teniendo en cuenta que a a = a , b b = b

r r

y que a b = b a (conmutativo), concluimos

que:
r r
a+b

r r r r r r r r
= aa + ab + b a + b b

r
= a
r
= a

r r r
+ 2a b + b

r
r r
+ 2 a b cos + b

r
a

demostracin que a b =

r r
a+b =

r
a

r
r r
+ 2 a b cos + b

r
r r
2 a b cos + b
r

r r
a+b

. Luego es de fcil

.
r r

NOTA: Partiendo de la expresin a + b = a + 2a b + b


valor a + b

r
a

con lo cual demostramos que


r

r
b

podemos concluir que el

ser mximo cuando = 0 resultando que

r
= a

r r r
+ 2a b + b

r
= a

r
r r
+2a b + b

r
r
= a + b

= 0
2

r
b

r
a

r
r r
r
a+b = a + b

Luego para cualquier otro valor de 0 < 180 el valor a + b ser menor que a + b .
r

luego, a + b a + b :

r
b

r
r
r r
r
c = a+b a + b

r r r
c = a+b
r
b

r
a

De forma anloga se puede demostrar que a c + b y b a + c que es la conocida


desigualdad triangular, donde se cumple que:
a<c+b

b<a+c

c<a+b

Universidad de Castilla- La Mancha


Ciudad Real - Espaa

Draft

Por: Eduardo W. V. Chaves (2014)

1 TENSORES

Ejemplo 1.3
Verificar si para las siguientes transformaciones

() = E

1
2

() = E 2

son

transformaciones lineales.
Solucin:

( 1 + 2 ) = E [1 + 2 ] = E 1 + E 2 = (1 ) + ( 2 ) (transformacin lineal)

()
( 1 + 2 ) = ( 1 ) + ( 2 )
( 2 )
( 1 )

1 + 2

1
2

La transformacin () = E 2 se demuestra fcilmente que no es una transformacin lineal


ya que:

( 1 + 2 ) = E[ 1 + 2 ]2 = E [ 12 + 2 1 2 + 22 ] = E12 + E 22 + E 2 1 2
1
1
2
2
= ( 1 ) + ( 2 ) + E 1 2 ( 1 ) + ( 2 )

1
2

1
2

1
2

()
(1 + 2 )

(1 ) + ( 2 )
( 2 )
(1 )
1

1 + 2

Ejemplo 1.4
Considrense los puntos A(1,3,1) , B (2,1,1) , C (0,1,3) y D(1,2,4 ) .

a) Encontrar el rea del paralelogramo definido por AB y AC ;

b) Encontrar el volumen del paraleleppedo definido por: AB , AC y AD ;

Universidad de Castilla- La Mancha


Ciudad Real - Espaa

Draft

Por: Eduardo W. V. Chaves (2014)

MECNICA DEL MEDIO CONTINUO: PROBLEMAS RESUELTOS

c) Encontrar el vector proyeccin del vector AB sobre el vector BC .


Solucin:

a) Primero se calculan los vectores AB y AC :

(
) (
)
r
b = AC = OC OA = (0i + 1j + 3k ) (1i + 3j + 1k ) = 1i 2j + 2k

r
a = AB = OB OA = 2i 1j + 1k 1i + 3j + 1k = 1i 4j + 0k

Utilizando la definicin del producto vectorial se obtiene el producto vectorial:


i
r r
ab= 1

j k
4 0 = ( 8)i 2j + ( 6)k

1 2

El rea del paralelogramo ser igual al mdulo del vector resultante del producto
vectorial:
r r
A = a b = (8) 2 + (2) 2 + ( 6) 2 = 104 (unidades cuadradas)

b) Calculando vector AD :

) (

r
c = AD = OD OA = 1i + 2j + 4k 1i + 3j + 1k = 0i 1j + 3k

Utilizando la definicin:

) (0i 1j + 3k ) ( 8i 2j 6k )

r r r
r r r
V (a, b, c ) = c a b =

= 0 + 2 18 = 16 (unidades cbicas)

c) A continuacin calculamos el vector BC :

) (

BC = OC OB = 0i + 1j + 3k 2i 1j + 1k = 2i + 2j + 2k

Luego el vector proyeccin de AB sobre BC viene dado por:


proj

AB =

BC

BC AB

BC
BC
4
1
42
3

BC

( 2i + 2j + 2k ) (1i 4j + 0k ) ( 2i + 2j + 2k )
( 2i + 2j + 2k ) ( 2i + 2j + 2k )

BC

( 2 8 + 0 ) ( 2i + 2j + 2k )
(4 + 4 + 4 )

proj

BC

AB =

5 5 5
i j k
3
3
3

Ejemplo 1.5
Reescribir en notacin indicial las siguientes expresiones:
a) a1 x1 x 3 + a 2 x 2 x 3 + a 3 x3 x 3
Solucin:

a i xi x 3

(i = 1,2,3)

b) x1 x1 + x2 x2

Universidad de Castilla- La Mancha


Ciudad Real - Espaa

Draft

Por: Eduardo W. V. Chaves (2014)

1 TENSORES

Solucin:

xi x i

(i = 1,2)

a11 x + a12 y + a13 z = b x

c) a 21 x + a 22 y + a 23 z = b y

a 31 x + a 32 y + a 33 z = b z

Solucin:
a1 j x j = b1

a 2 j x j = b2

a 3 j x j = b3

a11 x1 + a12 x 2 + a13 x 3 = b1

a 21 x1 + a 22 x 2 + a 23 x 3 = b2
a x + a x + a x = b
32 2
33 3
3
31 1

ndice
mudo j

ndice

libre
i

a ij x j = bi

Ejemplo 1.6
a) Demostrar que: 3 p v p = v3 ;
b) Demostrar que: 3i A ji = A j 3 ;
c) Obtener el resultado de ij ijk ;
d) Obtener el resultado de i 2 j 3 Aij .
Solucin:
Las componentes de la delta de Kronecker son:
11
ij = 21
31

12 13 1 0 0
22 23 = 0 1 0
32 33 0 0 1

(1.1)

a) La expresin ( 3 p v p ) no tiene ndice libre, luego el resultado es un escalar:

3 p v p = 31v1 + 32 v 2 + 33 v 3 = v3

(1.2)

b) La expresin 3i A ji tiene un ndice libre ( j ), luego el resultado es un vector:

3i A ji = 31 A j1 + 32 A j 2 + 33 A j 3 = A j 3

(1.3)

c) La expresin ij ijk tiene un ndice libre ( k ), luego el resultado es un vector:

ij ijk = 1 j 1 jk

123

1111k

+ 2 j 2 jk + 3 j 3 jk
1
424
3
123
+ 21 21k
+

+ 31 31k
+

12 12 k + 22 22 k

+ 32 32 k

+
+

13 13k

+ 23 23k

(1.4)

+
+ 33 33k

luego, ij ijk = 0 k (vector nulo).


d)

i 2 j 3 Aij = A23

Universidad de Castilla- La Mancha


Ciudad Real - Espaa

Draft

(1.5)

Por: Eduardo W. V. Chaves (2014)

MECNICA DEL MEDIO CONTINUO: PROBLEMAS RESUELTOS

Ejemplo 1.7
Expandir la expresin: Aij x i x j

(i, j = 1,2,3)

Solucin: Los ndices i, j son ndices mudos (indican suma), no hay ndice libre, y como
resultado tenemos un escalar. Expandimos primero el ndice mudo i y a continuacin el
ndice j , resultando as:

expandiendo j

oi

A1 j x1 x j + A2 j x 2 x j + A3 j x 3 x j
Aij x i x j expandiend
1
424
3 1
424
3 1
424
3
A11 x1 x1 A21 x 2 x1 A31 x 3 x1
+
+
+

A12 x1 x 2

A22 x 2 x 2

A32 x 3 x 2

A13 x1 x 3

A23 x 2 x 3

A33 x 3 x 3

Reagrupando los trminos anteriores obtenemos:


Aij x i x j = A11 x1 x1 + A12 x1 x 2 + A13 x1 x3 + A21 x 2 x1 + A22 x 2 x 2 +
A23 x 2 x3 + A31 x3 x1 + A32 x 3 x 2 + A33 x 3 x3

Ejemplo 1.8
Desarrollar las siguientes expresiones y obtener el valor numrico correspondiente:
1) ii jj

Solucin:

2) 1 1

ii jj = ( 11 + 22 + 33 )( 11 + 22 + 33 ) = 3 3 = 9

Solucin:

1 1 = 1 1 = 11 = 1

NOTA: Observar que es incorrecto hacer la siguiente operacin

1 1 = 3 11 = 1 , ya que lo que se reemplaza es el ndice repetido


Ejemplo 1.9
a) Probar que a) ijk pjk = 2 ip ; b) ijk ijk = 6 c) ijk a j a k = 0 i ; d) Obtener el valor numrico
de la siguiente expresin ijk 2 j 3k 1i .
Solucin:
a) Utilizando la expresin: ijk pqk = ip jq iq jp y haciendo q = j , resulta:
ijk pjk = ip

jj

ij

jp

= ip 3 ip = 2 ip

b) Partiendo del resultado anterior, es trivial la siguiente comprobacin ijk ijk = 2 ii = 6 .


c) Observemos que ijk = ikj , es decir, es antisimtrico en jk y observemos que a j a k
resulta un tensor de segundo orden simtrico. Como sabemos el doble producto escalar de un
tensor simtrico y otro antisimtrico es cero luego:
r

ijk a j a k = ijk (a a ) jk = 0 i = (a a ) i = 0 i
d) ijk 2 j 3k 1i = 123 = 1

Universidad de Castilla- La Mancha


Ciudad Real - Espaa

Draft

Por: Eduardo W. V. Chaves (2014)

1 TENSORES

Ejemplo 1.10
Obtener el valor de las siguientes expresiones:
a) ijk i1 j 2 3k
b) ijk pqk = ip jq iq jp para los siguientes casos
b.1) i = 1, j = q = 2, p = 3
b.2) i = q = 1, j = p = 2
c) ( ijk A jp c p A kq c q + i1 )( ist A sa c a A tb c b + i1 )
donde ijk es el smbolo de permutacin y ij es la Delta de Kronecker.
Solucin:
a) ijk i1 j 2 3k = 123 = 1
b.1) 12 k 32 k = 121 321 + 122 322 + 123 323 = 0 (1) + 0 0 + 0 0 = 0
b.2) 12 k 21k = 121 211 + 122 212 + 123 213 = 0 0 + 0 0 + 1 (1) = 1
c) Observemos que la operacin A jp c p = b j resulta un vector y verificamos tambin que
r

ijk A jp c p A kq c q = [( A c) ( A c )]i = (b b) i = 0 i , con lo cual resulta que:


r

( ijk A jp c p A kq c q + i1 )( ist A sa c a A tb c b + i1 ) = (0 i + i1 )(0 i + i1 ) = i1 i1 = 11 = 1

Recordatorio: Smbolo de Permutacin


ijk = jki = kij

ijk = ikj = kji = jik

ijk = 1

ijk = 1

k =3
j =1
i =1

k =2

i=2

j =1
i =1

k =1

i=2

j =1

i =1
i=2
i=3

0
0
0

j=2

0
0
-1

i=3
j =3

0
0
1

0
1

j=2

0
0
0

i=3

j =3

-1

0
-1
0

j=2

1
0
0

j =3

0
0
0
ij 3

0
0
ij 2

0
ij1

Universidad de Castilla- La Mancha


Ciudad Real - Espaa

Draft

Por: Eduardo W. V. Chaves (2014)

MECNICA DEL MEDIO CONTINUO: PROBLEMAS RESUELTOS

NOTA: El tensor de segundo orden ijk wk puede ser fcilmente obtenido como sigue:
ijk wk = ij1w1 + ij 2 w2 + ij 3 w3
0 1 0 0
0 0 1
0 0 0

= w1 0 0 1 + w2 0 0 0 + w3 1 0 0 = w3
0 0 0 w2
1 0 0
0 1 0

w3
0
w1

w2
w1
0

Ejemplo 1.11
r

Escribir en notacin indicial: a) el mdulo del vector a ; b) cos , donde es el ngulo que
r
r
forman los vectores a y b .
Solucin:
r
a

r r
= a a = a i e i a j e j = a i a j ij = a i a i = a j a j
r
luego, tambin cumple que b = b i b i .
2

r r

r r

r
a = ai ai

Por definicin a b = a b cos , donde a b = a i e i b j e j = a i b j ij = a i b i = a j b j .


Teniendo en cuenta que un ndice no puede aparecer ms que dos veces en un trmino de la
expresin, podemos expresar cos como:
r r
a jb j
ab
cos = r r =
ai ai b k b k
a b

Ejemplo 1.12
Demostrar la desigualdad de Schwarz:
r r
r r
ab a b

Desigualdad de Schwarz

(1.6)

Solucin: Consideremos un escalar , y la siguiente operacin:


r
r
a b

r r
r
r r
r r
r
r r
r r
= (a b) (a b) = a a 2 a b b a + b b 0
r
r 2
r r
= a 2 2a b + b

r r
r 2
r 2 2
r r
ab
f ( ) = a 2a b + b 0 , si ahora consideramos el valor de = r 2 podemos
a

obtener que:
2
r r
r r

r r
r
r r ( a b) r 2
2
a
b
a
b

(
)
(
)

f = r 2 = a r 2 2(a b) r 2 + b 0

a
a
a

r
r
r r
r r
r r
r 2
r 2 ( a b) 2
r r (a b) r 2 (a b) 2
( a b) 2
= a
r 4 2 ( a b) r 2 + b = r 2 2 r 2 + b 0
a
a
a
a
r
r
r 2
(a b ) 2
= r 2 + b 0
a

Universidad de Castilla- La Mancha


Ciudad Real - Espaa

Draft

Por: Eduardo W. V. Chaves (2014)

1 TENSORES

r r 2
r 2 (a
b)
b r 2
a

r 2 r 2
r r
a b ( a b) 2

r r
r r
a b ab

Q.E.D.
Solucin alternativa
r r

r r

r r

Teniendo en cuenta que 0 cos 1 la relacin a b = a b cos a b se cumple, luego


r r

r r

concluimos que a b a b .

Ejemplo 1.13

(r r ) (r r )

Escribir la siguiente relacin a b c d sin emplear el producto vectorial.

(r r )

Solucin: Observemos que el producto vectorial a b lo podemos expresar de la siguiente

(r r )

forma: a b = a j e j b k e k = ijk a j b k e i , cuyo resultado ser un vector. De esta forma hemos


utilizado la definicin del smbolo de permutacin. Anlogamente podemos expresar el
r r
r r
producto vectorial c d como c d = nlm c l d m e n , por lo tanto:

)
( )
r
(ar b) (cr dr ) =

( nlm c l d m e n )
= ijk nlm a j b k c l d m e i e n

ijk a j b k e i )

= ijk nlm a j b k c l d m in
= ijk ilm a j b k c l d m

Teniendo
en
cuenta
que
ijk ilm = jki lmi
jki lmi = jl km jm kl = jki ilm , concluimos que:

aplicando

la

relacin

ijk ilm a j b k c l d m = ( jl km jm kl ) a j b k c l d m = a l b m c l d m a m b l c l d m

(r r )

r r

Puesto que el subndice mudo indica el producto escalar: a l c l = (a c ) y b m d m = b d , luego:

(ar br ) (cr dr ) = (ar cr ) (br dr ) (ar dr )(br cr )

Observemos que, cuando c = a y d = b obtenemos que:

(ar br ) (ar br ) = (ar ar ) (br br ) (ar br )(br ar ) = (ar ar ) (br br ) (ar br )

Que es la misma expresin obtenida en el Ejemplo 1.1.


NOTA: Podemos empezar de la ecuacin anterior para demostrar que:
r r
ab

r 2 r 2 r r
r 2 r 2
r r
= a b (a b) 2 = a b a b cos

) = ar
2

r 2
r 2 r 2
b 1 cos 2 = a b sin 2

r r
r r
a b = a b sin
r

r r

r r

Notar que 0 sin 1 , con lo cual podemos demsotrar que a b = a b sin a b ,


luego
r r
r r
ab a b

Universidad de Castilla- La Mancha


Ciudad Real - Espaa

Draft

Por: Eduardo W. V. Chaves (2014)

MECNICA DEL MEDIO CONTINUO: PROBLEMAS RESUELTOS

10

Ejemplo 1.14
Probar que
a) ijk a i a j b k = 0
r r

b) ijk (a k b 3 i1 j 2 + a j b 2 i1 k 3 + a i b1 j 2 k 3 ) = a b
c) Aij A ji es un invariante
Solucin:
a) ijk a i a j b k = ij1a i a j b1 + ij 2 a i a j b 2 + ij 3 a i a j b 3 . Para el trmino ij1a i a j b1 tenemos que:
ij1a i a j b1 = 1 j1 a1 a j b1 + 2 j1a 2 a j b1 + 3 j1 a 3 a j b1
= 111 a1 a1b1 + 211 a 2 a1b1 + 311 a 3 a1b1 +
+ 121 a1a 2 b1 + 221 a 2 a 2 b1 + 321 a 3 a 2 b1 +
+ 131 a1a 3b1 + 231 a 2 a 3 b1 + 331 a 3 a 3 b1
= 321 a 3 a 2 b1 + 231 a 2 a 3 b1 = a 3 a 2 b1 + a 2 a 3b1
=0

Anlogamente para los trminos ij 2 a i a j b 2 = ij 3 a i a j b 3 = 0 . Es interesante observar que


ijk a i a j b k representa el determinante con dos filas iguales:
a1

a2

a3

ijk a i a j b k = a1 a 2 a 3 = 0
b1 b 2

b3

b)
ijk a k b 3 i1 j 2 + ijk a j b 2 i1 k 3 + ijk a i b1 j 2 k 3 =

r r

12 k a k b 3 + 1 j 3 a j b 2 + i 23 a i b1 = a 3b 3 + a 2 b 2 + a1b1 = a i b i = a b

Ejemplo 1.15

(r r ) (r r ) r (r

) r (r

Probar que: a b c d = c d a b d c a b

Solucin: Expresaremos en notacin indicial el segundo miembro de la expresin:

] [

r r r
rr r r
cr d (a
b) d c (a b) = c p d i ijk a j b k d p c i ijk a j b k

[ (

ijk a j b k c p di ijk a j b k c i d p

ni im c m d n np )

pm c m d n

[ (

)]

ijk a j b k (c p di c i d p )

Si utilizamos la propiedad de la delta de Kronecker:


ijk a j b k

)]

ijk a j b k

)c

mdn

ni im np )

pm

y si consideramos que pm ni im np = pil mnl . Reemplazamos en la expresin anterior y


obtenemos:

ijk a j b k c m d n pil mnl

Universidad de Castilla- La Mancha


Ciudad Real - Espaa

Draft

pil [( ijk a j b k ) ( mnl c m d n )]

Por: Eduardo W. V. Chaves (2014)

1 TENSORES

11

(r r )

(r r )

Dado que las componentes de a b son ijk a j b k y las componentes de c d son


mnl c m d n , obtenemos que:

[(r r ) (r r )]

pil [( ijk a j b k ) ( mnl c m d n )] = a b c d

Ejemplo 1.16
r r r

a) Si a , b , c son vectores linealmente independientes y que se cumple que:


r
r
r
r
v = a + b + c

v i = a i + b i + c i 0 i

componente

Probar que los escalares , , son dados por:

ijk v i b j c k
pqr a p b q c r

; =

ijk a i v j c k
pqr a p b q c r

; =

ijk a i b j v k
pqr a p b q c r

b) Dados tres vectores linealmente independientes, demostrar que al intercambiar 2 filas 2


r r r
columnas el signo del determinante a (b c ) cambia.
r

Solucin: a) Haciendo el producto escalar del vector v por el vector ( b c ) obtenemos que:
r r r
r r r
r r r
r r r
v (b c ) = a (b c ) + b (b c ) + c (b c )
14243
14243
=0

Obtenemos entonces el valor de como:

=0

r r r
v (b c )
= r r r
a (b c )

En componentes:

v1
b1

v2
b2

v3
b3

c1

c2

c3

a1
b1

a2
b2

a3
b3

c1

c2

c3

v1
v2

b1
b2

c1
c2

v3

b3

c3

a1
a2

b1
b2

c1
c2

a3

b3

c3

ijk v i b j c k
pqr a p b q c r

Anlogamente podemos obtener los parmetros , , es decir, hacemos el producto escalar


r

del vector v por los vectores a c y a b , respectivamente, i.e.:


r r r
r r r
r r r
r r r
v (a c ) = a (a c ) + b ( a c ) + c ( a c )
14243
14243
=0
=0
r r r
r r r

v
a
c
a
v
c

v (a c )
a (v c)
ijk i j k
jik j i k
= r r r
= r r r =
=
b (a c ) pqr b p a q c r qpr a q b p c r a (b c )
r r r
r r r
r r r
r r r
v (a b ) = a ( a b ) + b ( a b ) + c (a b )
14243
14243
=0
=0
r
r r r
r r
jki a j b k v i a (b v )
v (a b) ijk v i a j b k
= r r r =
=
= r r r
c (a b) pqr c p a q b r qrp a q b r c p a (b c )

NOTA 1: Podemos reestructurar las componentes del vector v de la siguiente forma:

Universidad de Castilla- La Mancha


Ciudad Real - Espaa

Draft

Por: Eduardo W. V. Chaves (2014)

MECNICA DEL MEDIO CONTINUO: PROBLEMAS RESUELTOS

12

v 1 a1

v i = v 2 = a 2
v a
3 3

c 1 a1

c 2 = a 2
c 3 a 3

b1
b2
b3

b1
b2
b3

c 1 z1

c 2 z 2 = B ij z j
c 3 z 3

donde hemos denotado por z1 = , z 2 = , z 3 = . Teniendo en cuenta que:

=z1 =

=z 3 =

v1
v2

b1
b2

c1
c2

ijk v i b j c k
v3
=
a1
pqr a p b q c r

b3

c3

a2

b1
b2

c1
c2

a3

b3

c3

a1
a2

b1
b2

v1
v2

ijk a i b j v k
a3 b 3
=
a1 b 1
pqr a p b q c r

v3

a2

b2

c1
c2

a3

b3

c3

B (1)

; =z 2 =

a1
a2

v1
v2

c1
c2

ijk a i v j c k
a3
=
a1
pqr a p b q c r

v3

c3

a2

b1
b2

c1
c2

a3

b3

c3

B (2)
B

B (3)
B

donde B (i ) es el determinante de la matriz resultante al reemplazar la columna (i) de la


r

matriz B por las componentes del vector v . Con eso, podemos decir que:
Dado v i = B ij z j

zi =

B (i )

Regla de Cramer

NOTA 2: Aunque hemos demostrado para una matriz 3 3 , este procedimiento es vlido
para matrices de n-dimensiones y es conocido en la literatura como Regla de Cramer.
NOTA 3: La solucin ( z i ) solo es posible si B 0 .
NOTA 4: Si v i = 0 i tenemos que B ij z j = 0 i y B (i ) = 0 i , con eso, segn la regla de Cramer
tenemos que:
z i B = B (i ) = 0 i

Notar que, la solucin non-trivial z i 0 i solo es posible si y solo si B = 0 , (ver Ejemplo


1.51).
r

r r r

b) El determinante definido por a (b c ) = [a, b, c ] en notacin indicial queda ijk a i b j c k ,


adems sabiendo que se cumple que:
ijk = jki = kij

ijk = ikj = kji = jik


i

Universidad de Castilla- La Mancha


Ciudad Real - Espaa

Draft

Por: Eduardo W. V. Chaves (2014)

1 TENSORES

r r r

13

r r r

r r r

ijk a i b j c k = [a, b, c ] = ikj a i b j c k = [a, c , b] = jki a i b j c k = [b, c, a]

Luego
a1

a1

a2

a3

ijk a i b j c k = b 1 b 2 b 3 = c 1

c2

c 3 = ikj a i b j c k

b2

b3

a2

a3

c1

c2

c3

b1

b1

b2

b3

c1

c2

c 3 = jki a i b j c k

a1

a2

a3

Ejemplo 1.17
a) Probar las relaciones:

( ) (

r r
r
r r r r r r r r r r r
a (b c ) = (a c ) b a b c = b c c b a
r r
r
r r
r r r
a (b a) = [(a a)1 a a] b
r r
r r
b) Obtener las explcitas componentes del tensor [(a a)1 a a] .

Solucin:

(r r )

a) Representando el producto vectorial b c i = ijk b j c k , luego:

[ar (br cr )]

= rsi a s ( ijk b j c k )
= rsi ijk a s b j c k = rsi jki a s b j c k

= rj sk rk sj a s b j c k
= rj sk a s b j c k rk sj a s b j c k

= asb r c s asb s c r

= ak br c k a jb j c r
r r
r r
= b r (a c ) c r a b
r r r rr r
= b(a c ) c a b r

[(

( )
( )]

Comprobando que:

= (b r c s b s c r )a s
r r r r r
= b c c b a r

) ]

( ) (

r r r
r r r r r r r r r r r
a b c = (a c ) b a b c = b c c b a

Notar que tambin se cumple que:

) (

r r r r r
r r r
r r
r r r r r r r
r
a b c = b c c b a = [(a c )1 c a] b = b a (a b)1 c
r

En el caso particular cuando a = c podemos decir que:

[ar (br ar )]

= (a k a k )b r (a j b j )a r = (a j a j )b p

rp

(a j b p

= (a j a j ) rp (a j jp )a r b p = (a j a j )
r r
r r r
= [(a a)1 a a] b r

rp

jp

)a r

a par b p

b) Teniendo en cuenta la ecuacin anterior podemos obtener que

Universidad de Castilla- La Mancha


Ciudad Real - Espaa

Draft

Por: Eduardo W. V. Chaves (2014)

MECNICA DEL MEDIO CONTINUO: PROBLEMAS RESUELTOS

14

r r
r r
[(a a)1 a a] ij = (a k a k )

ij

ai a j =

(a12

+ a 22

1 0 0 a 1 a 1
1 0 a1 a 2
0 0 1 a1 a 3

a 32 ) 0

a1 a 2
a2a2
a1 a 3

a1 a 3
a1 a 3
a 3 a 3

(a 22 + a 32 ) a1 a 2
a1 a 3

2
2
( a1 + a 3 )
= a1 a 2
a1 a 3
a1 a 3
(a12 + a 22 )
a1 a 3

Ejemplo 1.18
Demostrar la identidad de Jacobi:

r r r r r r r r r r
a b c + b (c a) + c a b = 0
r

(r r )

r r r

(r r ) r

Solucin: A travs del ejercicio anterior demostramos que a b c = (a c ) b a b c , luego,


tambin es vlido que:

(br rar )cr (br cr )arr


) (cr b)ar (cr ar )b

r r r
b (c a) =
r r r
c ab =

Luego, teniendo en cuenta que el producto escalar entre dos vectores es conmutativo, es decir,
r r r r
r r
r r
r r
r r
(a c ) = (c a) , a b = b a , b c = c b , concluimos que:

( ) ( )( ) ( )

(a c )b (a b )c
r r r

r r r

) ( )cr + (br cr )ar = 0r


( )ar (cr ar )br

r r
r r r r
r r r r r
a b c + b (c a) + c a b = b a
r r
c b

1.2 Operaciones con Tensores de Orden Superior


Ejemplo 1.19
Cul es el orden de los tensores representados por sus componentes: v i , ijk , Fijj , ij , C ijkl ,
ij ? Determinar cuntas componentes independientes tiene el tensor C .
Solucin: El orden del tensor viene dado por el nmero de subndices libres, luego:
r r

Tensores de orden uno: v , F


Tensores de segundo orden: ,
Tensor de tercer orden:
Tensor de cuarto orden: C
El nmero de componentes de un tensor viene dado por el mximo valor del rango del
subndice, 3 si ( i = 1,2,3 ), elevado al nmero de subndices libres. Es decir, para el tensor de
cuarto orden, el nmero de ndices libres es 4, luego:
3 4 = (i = 3) ( j = 3) (k = 3) (l = 3) = 81

El tensor de cuarto orden C ijkl tiene 81 componentes independientes.

Universidad de Castilla- La Mancha


Ciudad Real - Espaa

Draft

Por: Eduardo W. V. Chaves (2014)

1 TENSORES

Ejemplo 1.20

(r r ) r (r r ) r

15

(r r ) (r r ) (r r ) r

Demostrar que a) a b c = b c a ; b) a b c d = b c a d
Solucin:
a)

(ar br ) cr = (a e b e ) c e = a e b c = (b c )a e = (br cr )ar (br cr ) ar


r r r r
expresin (a b ) (c d), que resulta un tensor de segundo orden, expresamos
i

jk

b) La
directamente en notacin indicial:

[(ar br ) (cr dr )]

ij

r r r
= (a i b k ) c k d j = a i b k c k d j = b k c k a i d j = (b k c k )(a i d j ) = (b c )(a d) ij
123

escalar

Ejemplo 1.21
Desarrollar y simplificar lo posible la expresin A ij xi x j para los siguientes casos:
a) A ij = A ji
b) A ij = A ji
Solucin:
Expandiendo A ij xi x j obtenemos:
A ij xi x j = A 1 j x1 x j + A 2 j x 2 x j + A 3 j x3 x j =
= A 11 x1 x1 + A 21 x 2 x 1 + A 31 x3 x 1 +

(1.7)

A 12 x1 x 2 + A 22 x 2 x 2 + A 32 x 3 x 2 +
A 13 x1 x 3 + A 23 x 2 x 3 + A 33 x3 x 3

a) A ij = A ji (simetra)
A ij x i x j = A 11 x12 + 2 A 12 x1 x 2 +2 A 13 x1 x 3 + A 22 x 22 + 2 A 23 x 2 x 3 + A 33 x 32

(1.8)

b) A ij = A ji (antisimetra)
A ij xi x j = 0

(1.9)

r
r
r r
A ij xi x j = x A x = A : ( x x )

(1.10)

lo que era de esperar ya que:


r

Si A antisimtrico y ( x x ) resulta simtrico, el doble producto escalar de un tensor


simtrico y uno antisimtrico resulta ser siempre igual a cero.
Ejemplo 1.22
Si las componentes de los tensores de segundo orden y T son representadas
respectivamente por:
5 2 4
ij = 1 2 1
4 3 6
Universidad de Castilla- La Mancha
Ciudad Real - Espaa

Draft

3 1 2
Tij = 4 2 1
1 3 8

(1.11)

Por: Eduardo W. V. Chaves (2014)

MECNICA DEL MEDIO CONTINUO: PROBLEMAS RESUELTOS

16

Obtener T : .
Solucin:
T : = Tij ij

(1.12)

Tij ij = T1 j 1 j + T2 j 2 j + T3 j 3 j
123
123
123
T1111
+

T12 12
+
T13 13

T21 21
+

T31 31
+

T22 22

T32 32

+
T23 23

+
T33 33

(1.13)

luego,
Tij ij = 5 3 + 2 1 + 4 2 + (1) 4 + 2 2 + 1 1 + 4 1 + 3 3 + 6 8 = 87

(1.14)

Ejemplo 1.23
Dadas las componentes del tensor B en el sistema de coordenadas cartesianas:
3 2 4
B ij = 1 5 3
5 7 9

(1.15)

Obtener:
a) C ij = B ik B kj ; b) D ij = B ik B jk ; c) E ij = B ki B kj ; d) C ii , D ii , E ii
Solucin:
C = B B

3 2 4 3 2 4 31 44 54
C ij = B ik B kj = 1 5 3 1 5 3 = 23 48 46
5 7 9 5 7 9 67 108 122

(1.16)

D = B BT

3 2 4 3 2 4
29 25 65

= 1 5 3 1 5 3 = 25 35 67
5 7 9 5 7 9
65 67 155

D ij = B ik B jk

3 2 4 3 2 4 35 46 60
E ij = B ki B kj = 1 5 3 1 5 3 = 46 78 86
5 7 9 5 7 9 60 86 106

(1.17)

E = BT B

(1.18)

Luego:
C ii = C 11 + C 22 + C 33 = 31 + 48 + 122 = 201
D ii = D11 + D 22 + D 33 = 29 + 35 + 155 = 219

(1.19)

E ii = E11 + E 22 + E 33 = 35 + 78 + 106 = 219

NOTA: Verificamos que se cumple que: Tr (B B T ) = Tr (B T B) = B : B

Universidad de Castilla- La Mancha


Ciudad Real - Espaa

Draft

Por: Eduardo W. V. Chaves (2014)

1 TENSORES

17

Ejemplo 1.24
Dadas las componentes cartesianas del tensor de segundo orden B :
1 0 2
B ij = 0 1 2
3 0 3

Obtener: a) B kk

b) B ij B ij

c) B jk B kj

Solucin:
a) B kk = B 11 + B 22 + B 33 = 1 + 1 + 3 = 5
b) B ij B ij = B 1 j B 1 j

123

+ B 2 jB 2 j + B 3 jB 3 j
123
123

B 11B 11
+

B 21B 21
+

+ B 31B 31
+

B 12B 12

+ B 22B 22

+ B 32B 32

+
B 13B 13

+
+ B 23B 23

+
+ B 33B 33

Resultando:
B ij B ij = 1 1 + 0 0 + 2 2 + 0 0 + 1 1 + 2 2 + 3 3 + 0 0 + 3 3 = 28

c) B jk B kj = B 1k B k1 + B 2k B k 2 + B 3k B k 3
123

123

B 11B 11
+

B 12B 21
+
B 13B 31

+ B 22B 22
+
+ B 23B 32

123

B 21B 12
+

B 31B 13
+

+ B 32B 23
+
+ B 33B 33

B jk B kj = B 11B 11 + B 22B 22 + B 33B 33 + 2B 21B 12 + 2B 31B 13 + 2B 32B 23


= 1 1 + 1 1 + 3 3 + 2(0 0) + 2(3 2 ) + 2(0 2 ) = 23

Ejemplo 1.25
Obtener las componentes del tensor D resultante de la siguiente operacin D = A : B , para
los siguientes casos:
a)

b)

con

2 3 2
Aij = 4 1 1
1 1 5

con

7 13 14
Aik Bkj = 11 18 11
16 27 31

2 3 1
Bij = 1 2 1
1 2 5
;

Aik B jk

13 9 17
= 15 9 13
18 12 32

Solucin:
a) A : B = 2 2 + 3 3 + 2 1 + 4 1 + 1 2 + 1 1 + 1 1 + 1 2 + 5 5 = 50

Universidad de Castilla- La Mancha


Ciudad Real - Espaa

Draft

Por: Eduardo W. V. Chaves (2014)

18

MECNICA DEL MEDIO CONTINUO: PROBLEMAS RESUELTOS

b) Teniendo en cuenta la expresin Tr ( A B T ) = Tr ( AT B) = A : B y que Aik B jk = A B T ,


concluimos que A : B = Tr ( A B T ) = 13 + 9 + 32 = 54 .
Ejemplo 1.26
Considrese un tensor de segundo orden T = Tr ( E )1 + ( F : E ) E o en notacin indicial
Tij = E kk ij + ( Fkp E kp ) E ij . Si las componentes de los tensores E y F vienen dadas por:
2 1 4
E ij = 1 5 0
2 0 1

4 3 1
Fij = 2 0 3
2 0 0

a) Obtener las componentes del tensor T . b) Son los tensores T y E coaxiales?


Demustralo.
Solucin:
Obtenemos primero los siguientes escalares:
Tr ( E ) = 2 + 5 + 1 = 8
F : E = 2 4 + 1 3 + 4 1 + 1 2 + 5 0 + 0 3 + 2 2 + 0 0 + 1 0 = 21

Luego
1 0 0
2 1 4 50 21 84

Tij = 80 1 0 + 211 5 0 = 21 113 0


0 0 1
2 0 1 42 0 29

Dos tensores son coaxiales cuando presentan los mismos autovectores o cuando se cumple
que T E = E T :
50 21 84 2 1 4 289
Tik E kj = 21 113 0 1 5 0 = 155
42 0 29 2 0 1 142
2 1 4 50 21 84 289
E ik Tkj = 1 5 0 21 113 0 = 155
2 0 1 42 0 29 142

155 284
586 84
42 197
155 284
586 84
42 197

Con lo cual concluimos que son coaxiales.


Ejemplo 1.27
Obtener el resultado de las siguientes operaciones: I : I , I : I , I : I , I : I , I : I , I : I , I : I ,
I : I , I sym : I sym , I sym : I , I : I sym , donde
I = 11 = I ijkl e i e j e k e l

con

I ijkl = ik jl

(1.20)

I = 11 = I ijkl e i e j e k e l

con

I ijkl = il jk

(1.21)

I ijkl = ij kl

(1.22)

I = 1 1 = I ijkl e i e j e k e l

Universidad de Castilla- La Mancha


Ciudad Real - Espaa

con

Draft

Por: Eduardo W. V. Chaves (2014)

1 TENSORES

19

Solucin:
(I : I ) ijkl = I ijpq I pqkl = ip jq pk ql = ik jl = I ijkl
( I : I ) ijkl = I ijpq I pqkl = iq jp pl qk = ik jl = I ijkl

( I : I ) ijkl = I ijpq I pqkl = ij pq pq kl = qq ij kl = 3I ijkl


( I : I ) ijkl = I ijpq I pqkl = iq jp pk ql = il jk = I ijkl
(I : I ) ijkl = I ijpq I pqkl = ip jq pl qk = il jk = I ijkl
(I : I ) ijkl = I ijpq I pqkl = ip jq pq kl = iq jq kl = ij kl = I ijkl
( I : I ) ijkl = I ijpq I pqkl = iq jp pq kl = iq jq kl = ij kl = I ijkl

Resumiendo lo anterior en notacin tensorial:


I : I = (11) : (11) = 11 = I
I : I = (11) : (11) = 11 = I

I : I = (1 1) : (1 1) = 3(1 1) = 3I
I : I = (11) : (11) = 11 = I
I : I = (11) : (11) = 11 = I
I : I = (11) : (1 1) = 1 1 = I
I : I = (11) : (1 1) = 1 1 = I

I sym : I sym

(
[(
[
(

)(
) (

)
) (
]

1
11 + 11 : 11 + 11
4
1
= 11 : 11 + 11 : 11 + 11 : 11 + (11 : 11)
4
1
= 11 + 11 + 11 + 11
4
1
= 11 + 11
2
= I sym
=

(1 1) : I sym = I : I sym

Universidad de Castilla- La Mancha

(
(

) ( )
) ( )

1
1
1
I + I : I = I : I + I : I = I + I = I =1 1
2
2
2
1
1
1
= I : I + I = I :I + I : I = I + I = I =1 1
2
2
2

I sym : (1 1) = I sym : I =

Ciudad Real - Espaa

1
1
I + I = 11 + 11 , concluimos que:
2
2

Teniendo en cuenta la definicin: I sym =

Draft

Por: Eduardo W. V. Chaves (2014)

MECNICA DEL MEDIO CONTINUO: PROBLEMAS RESUELTOS

20

1.3 Transpuesta
Ejemplo 1.28
Demostrar que la siguiente propiedad es vlida:

A : (B C ) = B T A : C = A C T : B

donde A , B , C son tensores de segundo cualesquiera.


Solucin: Demostraremos esta identidad a travs de sus componentes:

A : (B C ) = A ij e i e j : B lk e l e k C pq e p e q
= A ij B lk C pq e i e j : kp e l e q
= A ij B lk C pq kp il

jq

= A ij B ik C kj

Observemos que cuando trabajamos en notacin indicial el orden no importa, es decir:


A ij B ik C kj = B ik A ij C kj = A ij C kj B ik

Podemos ahora observar que la operacin B ik A ij resultar un tensor de segundo orden cuyas
componentes son (B T A ) kj luego, B ik A ij C kj = (B T A ): C . Anlogamente podemos decir que

A ij C kj B ik = A C T : B .

Ejemplo 1.29
r

Demostrar que, si u , v son vectores y A un tensor de segundo orden, la siguiente relacin es


vlida:
r
r r
r
u AT v = v A u

Solucin:
r
r
u AT v
u i e i A jl e l e j v k e k

r
r
= v A u
= v k e k A jl e j e l u i e i

u i A jl il v k jk
u l A jl v j

= v k kj A jl u i il
= v j A jl u l

Q.E.D.

1.3.1

Simetra y Antisimetra

Ejemplo 1.30
Si es un tensor de segundo orden simtrico y W es un tensor de segundo orden
antisimtrico. Demostrar que : W = 0 .
Solucin:
: W = ij (e i e j ) : Wlk (e l e k ) = ij Wlk il

jk

= ij Wij

Desarrollando

Universidad de Castilla- La Mancha


Ciudad Real - Espaa

Draft

Por: Eduardo W. V. Chaves (2014)

1 TENSORES

21

ij Wij = 1 j W1 j + 2 j W2 j + 3 j W3 j
123
1
424
3
1
424
3
31W31
21W21
11W11
+
+
+
32 W32
22 W22
12 W12
+
+
+
33W33
23W23
13W13

Considerando la propiedad de un tensor simtrico 12 = 21 , 31 = 13 , 32 = 23 y


antisimtrico W11 = W22 = W33 = 0 , w 21 = w 12 , W31 = W13 , W32 = W23 , resultando:
:W =0

Q.E.D.

Ejemplo 1.31
Demostrar que:
r

a) M Q M = M Q sym M ;
b) A : B = A sym : B sym + A anti : B anti ;
r

donde, M es un vector, y Q , A , y B son tensores de segundo orden arbitrarios.


c) Demostrar que si se cumple que ijk T jk = 0 i , T es simtrico, es decir, Tij = T ji .
Solucin:
a)

r
r r
r r
r r
r
M Q M = M Q sym + Q anti M = M Q sym M + M Q anti M
Q.E.D.
r
r
r
r
anti
anti
Ya que el producto: M Q M = Q : M M = 0 , resulta que:
r
r r
r
M Q M = M Q sym M
r
r
NOTA: Podemos hacer la interpretacin geomtrica de M Q anti M = 0 . Notar que la
r r r
r
r r r r
operacin algebraica Q anti M = q (M) resulta un vector, luego M Q anti M = M q (M) = 0 , que
r
r r
implica que M y q (M) son vectores ortogonales. Con eso, concluimos que: la proyeccin de
r
r r
un tensor antisimtrico de segundo grado segn una direccin ( M ) resulta un vector ( q (M) )
r
que es ortogonal a M , ver figura abajo:

r
Q M

r r r
q (M ) M = 0
r
r r
q (M) = Q anti M

r
M

Universidad de Castilla- La Mancha


Ciudad Real - Espaa

Draft

r
M

Por: Eduardo W. V. Chaves (2014)

MECNICA DEL MEDIO CONTINUO: PROBLEMAS RESUELTOS

22

b)
A :B

= ( A sym + A anti ) : (B sym + B anti )


anti
sym
anti
= A sym : B sym + 1
A sym
:B
A anti
:B
: B anti
42
43 + 1
42
43 + A
=0

=0

= A sym : B sym + A anti : B anti

Luego como consecuencia tenemos que:


A : B sym = A sym : B sym

A : B anti = A anti : B anti

c)

Q.E.D.

ijk T jk = ij1 T j1 + ij 2 T j 2 + ij 3 T j 3 = 0 i
= i11 T11 + i 21 T21 + i 31 T31 + i12 T12 + i 22 T22 + i 32 T32 + i13 T13 + i 23 T23 + i 33 T33
= i 21 T21 + i 31 T31 + i12 T12 + i 32 T32 + i13 T13 + i 23 T23 = 0 i

Luego, las componentes del vector resultante quedan:


i =1

1 jk T jk = 132 T32 + 123 T23 = T32 + T23 = 0 T32 = T23

i=2

2 jk T jk = 231 T31 + 213 T13 = T31 T13 = 0 T31 = T13

i=3

3 jk T jk = 321 T21 + 312 T12 = T21 + T12 = 0 T21 = T12

con lo cual demostrando que si ijk T jk = 0 i , T es simtrico, T = T T .


Ejemplo 1.32
Dado un tensor de segundo orden arbitrario A donde se conocen las componentes de su
parte simtrica en el sistema Cartesiano:
A ijsym

4 2 0
= 2 1 0
0 0 3

A N
, donde las componentes del versor N
son N = [1 0 0] .
Obtener N
i

Solucin:
A N
=N
A sym N
con lo cual:
En el Ejemplo 1.31 se ha demostrado que N
4 2 0 1
sym
sym

N A N = N A N = N i A ij N i = [1 0 0] 2 1 0 0 = 4
0 0 3 0

Ejemplo 1.33
Si W es un tensor antisimtrico. a) Demostrar que W W resulta un tensor de segundo
orden simtrico. b) Demostrar tambin que (W T W W) : 1 = 0
Solucin:
a) Si demostramos que (W W ) anti = 0 , demostramos que W W resultar ser simtrico:
(W W) anti =

] [

1
1
1
(W W) (W W ) T = (W W ) W T W T = [(W W) W W ] = 0
2
2
2

donde hemos aplicado la propiedad del tensor antisimtrico W = W T .


Universidad de Castilla- La Mancha
Ciudad Real - Espaa

Draft

Por: Eduardo W. V. Chaves (2014)

1 TENSORES

23

Soluciones alternativas a) Teniendo en cuenta la definicin de un tensor antisimtrico donde se


cumple que W = W T :
W W = W T W = W T W T = (W W ) T

Tambin se puede comprobar a travs de sus componentes:


W12
W13 0
W12
W13
0

( W W ) ij = W12
0
W23 W12
0
W23
W13 W23
0 W13 W23
0
W122 W132
W12 W23
W13 W23

2
2
= W13 W23
W12 W23
W12 W13
2
W12 W23
W12 W13
W132 W23

b) (W T W W ) : 1 = (W pi W pk Wkj ) ij = W pi (W pk Wki ) = W : (W W) = 0 , ya que el doble


producto escalar entre un tensor simtrico (W W ) y uno antisimtrico ( W ) resulta cero.
Ejemplo 1.34
1
2

Sea B un tensor de segundo orden tal que B pq = pqs a s con a i = ijk B jk . Demostrar que B
es un tensor antisimtrico.
Solucin:
1
1
1
B pq = pqs a s = pqs sjk B jk = pqs sjk B jk = pqs jks B jk
2
2
2

Recurriendo a la relacin pqs jks = pj qk pk qj


1
1
pqs jks B jk = ( pj
2
2
1
= (B pq B qp ) = B anti
pq
2

B pq =

qk

pk

1
2

qj )B jk = ( pj qk B jk pk qj B jk )

Solucin Alternativa:
Teniendo en cuenta que B qp = qps a s , y que por definicin se cumple que pqs = qps ,
concluimos que:
B pq = pqs a s = qps a s = B qp

B = B T (antisimtrico)

Ejemplo 1.35
Demostrar que la operacin A anti A sym + A sym A anti resulta un tensor antisimtrico.
Solucin:
Denominando por B = A anti A sym + A sym A anti , y teniendo en cuenta que se cumple que
A anti = (A anti ) T , A sym = (A sym ) T , concluimos que:
B = A anti A sym + A sym A anti = A anti A sym A sym ( A anti ) T = A anti A sym ( A anti A sym ) T

= 2( A anti A sym ) anti

Universidad de Castilla- La Mancha


Ciudad Real - Espaa

Draft

Por: Eduardo W. V. Chaves (2014)

MECNICA DEL MEDIO CONTINUO: PROBLEMAS RESUELTOS

24

Ejemplo 1.36

La relacin n T = T n es vlida siempre? Siendo T un tensor de segundo orden y n un


vector. En el supuesto de que la relacin no sea vlida, para qu caso particular lo sera?
Solucin:

r
n T = n i e i Tkl (e k e l )
= n i Tkl ik e l

r
T n = Tlk (e l e k ) n i e i
= n i Tlk ki e l

= n k Tkl e l

= n k Tlk e l

Con lo que comprobamos que n k Tkl n k Tlk , luego:


r
r
n T T n
r

La relacin n T = T n solo ser vlida cuando el tensor sea simtrico, i.e. n T sym = T sym n .
Ejemplo 1.37
r

Obtener el vector axil w asociado al tensor antisimtrico ( x a ) anti . Expresar w en funcin


r
r
de x y a .
r

Solucin: Sea z un vector arbitrario, se cumple que:


r r
r r r
( x a ) anti z = w z
r
r r
donde w es el vector axil asociado a ( x a ) anti . Teniendo en cuenta que:

r r
r r
1 r r
1 r r r r
( x a ) anti = ( x a ) ( x a ) T = [ x a a x ]
2
2

podemos an decir que:


1 r r r r r r r
[x a a x ] z = w z [xr ar ar xr ] zr = 2wr zr
2
r r
r r r r r
r r r
r
Recordar que, dados tres vectores a , b , c se cumple que: a (b c ) = (b c c b) a ,
r r r r r r
r r
ver Ejemplo 1.17. Luego, se cumple que [x a a x ] z = z ( x a ) . Retomando nuestra

expresin anterior:

[xr ar ar xr ] zr = zr ( xr ar ) = (ar xr ) zr = 2wr zr

con lo cual, concluimos que:


r 1 r r
r r
w = (a x ) es el vector axil asociado al tensor ( x a ) anti
2

Ejemplo 1.38
Consideremos dos tensores antisimtricos W (1) y W ( 2) y sus vectores axil representados,
r
r
respectivamente, por w (1) y w ( 2) . Demostrar que:
r
r
r
r
W (1) W ( 2 ) = ( w ( 2 ) w (1) ) ( w (1) w ( 2) )1
r
r
Tr W (1) W ( 2 ) = 2( w (1) w ( 2 ) )

Universidad de Castilla- La Mancha


Ciudad Real - Espaa

Draft

Por: Eduardo W. V. Chaves (2014)

1 TENSORES

25

Solucin:
Teniendo en cuenta las propiedades de tensor antisimtrico, podemos decir que:
r r
r
W (1) a = w (1) a
r
r r
T
a W (1) = a w (1)
r
r r
a W (1) = a w (1)
r
r r
a W (1) = a w (1)

r r
r
W ( 2) a = w ( 2 ) a

A continuacin hacemos el producto escalar (a W (1) ) (W ( 2) a), obteniendo que:


r

(ar W ) (W
(1)

( 2)

a) = (a w (1) ) ( w ( 2) a)
r

Continuaremos el desarrollo en notacin indicial:


(a i Wij(1) )(W jk(1) a k ) = ( ijk a j w k(1) )( ipq w (p2) a q )

a i (Wij(1) W jk(1) )a k = a j ( ijk ipq wk(1) w (p2 ) )a q = a j ( jp

= a j jp

(1) ( 2 )
kq w k w p

jq

kq

(1) ( 2 )
kp w k w p

jq

]a

(1) ( 2 )
kp ) w k w p

=a

(1) ( 2 )
j wq w j

]a

q
(1) ( 2 )
jq w k w k

]a

En notacin tensorial la expresin anterior queda:

] ar

r
r r r
r
r
r
a W (1) W ( 2 ) a = a ( w ( 2 ) w (1) ) ( w (1) w ( 2 ) )1

con lo cual demostramos que: W (1) W ( 2) = ( w ( 2) w (1) ) ( w (1) w ( 2) )1 .


b)

Tr W (1) W ( 2 )

] [

r
r
r
r
r
r
r
r
= Tr ( w ( 2 ) w (1) ) ( w (1) w ( 2 ) )1 = Tr ( w ( 2 ) w (1) ) Tr ( w (1) w ( 2 ) )1
r
r
r
r
Tr2
[3
1]
= ( w ( 2 ) w (1) ) ( w (1) w ( 2 ) ) 1
=3

r
r
= 2( w (1) w ( 2 ) )

Solucin alternativa
En esta solucin alternativa vamos utilizar las componentes. Para ellos consideremos:
Wij(1)

= W12(1)
W (1)
12

Wij( 2 )

= W12( 2 )
W ( 2 )
12

W12(1)
0
W12(1)
W12( 2 )
0
W12( 2 )

W13(1) 0
(1)
(1)
W23
= w3
0 w2(1)

w3(1)
0
w1(1)

W13( 2 ) 0
(2)
(2)
W23
= w3
0 w2( 2)

w2(1)

w1(1)
0

w3( 2 )
0
w1( 2 )

w2( 2 )

w1( 2 )
0

Con eso podemos obtener que:

[W

(1)

W ( 2) ]ij

Wik(1) Wkj( 2 )

= Wik(1) Wkj( 2 )

= w3(1)
w (1)
2

w3(1) w3( 2 ) w2(1) w2( 2 )

w1(1) w2( 2)
=

w3( 2 ) w1(1)

Universidad de Castilla- La Mancha


Ciudad Real - Espaa

w3(1)
0
w1(1)

w2(1) 0

w1(1) w3( 2 )
0 w2( 2)

w3( 2 )
0
w1( 2 )

w2(1) w1( 2 )

w3( 2 ) w3(1) w1(1) w1( 2 )


w3( 2 ) w2(1)

Draft

w2( 2 )

w1( 2 )
0

w3(1) w1( 2 )

w2( 2) w3(1)

(1) ( 2 )
( 2 ) (1)
w2 w2 w1 w1
Por: Eduardo W. V. Chaves (2014)

MECNICA DEL MEDIO CONTINUO: PROBLEMAS RESUELTOS

26

En el trmino (11) sumamos y restamos el trmino w1( 2) w1(1) , en el trmino (22) sumamos y
restamos el trmino w2( 2) w2(1) y en el trmino (33) sumamos y restamos el trmino w3( 2) w3(1) .
Con lo cual quedamos con:
Wik(1) Wkj( 2 )

w1( 2) w1(1)

= w2( 2) w1(1)
w ( 2) w (1)
3 1

w1( 2 ) w3(1)

w2( 2 ) w3(1) +
w3( 2 ) w3(1)

w1( 2) w2(1)
w2( 2) w2(1)
w3( 2) w2(1)

w1(1) w1( 2 ) w2(1) w2( 2) w3(1) w3( 2)

0
+

w1(1) w1( 2 )

w2(1) w2( 2 )

w3(1) w3( 2 )

w1(1) w1( 2 )

w2(1) w2( 2 )

w3(1) w3( 2 )

Que es igual a:
Wik(1) Wkj( 2 ) = wi( 2) w (j1) ( w1(1) w1( 2) + w2(1) w2( 2) + w3(1) w3( 2) ) ij = wi( 2) w (j1) ( wk(1) wk( 2) ) ij

Dejamos al lector comprobar la traza.


NOTA: La solucin alternativa fue realizada solo como comprobacin. El lector debe dar
preferencia a la solucin tensorial o indicial, porque ni siempre la solucin a travs de
componentes es tan sencilla.

1.4 Cofactor.
Adjunta.
Determinante
Ejemplo 1.39
r

Traza.

Tensores

Particulares.

r r

Demostrar que Tr (a b) = a b .
Solucin:

r r
Tr (a b) = Tr (a i e i ) (b j e j ) = a i b j Tr e i e j = a i b j (e i e j ) = a i b j ij
r r
= aibi = ab

Ejemplo 1.40
1
2

Dado que Tij = Ekk ij + 2 Eij , W = Tij E ij , y P = Tij Tij . Demostrar que:
W = E : E +

[Tr( E )]2

P = 4 2 E : E + (3 + 4 )[Tr ( E )]

Solucin 1: (Notacin indicial)


W=

1
1
1
1
Tij Eij = Ekk ij + 2 Eij Eij = Ekk ij Eij + 2 Eij Eij = Ekk Eii + 2 Eij Eij
2
2
2
2

Como E kk = E ii = Tr (E ) , y Eij Eij = E : E , concluimos que W = E : E +

)(

P = Tij Tij = E kk ij + 2 E ij E qq ij + 2 E ij

[Tr ( E )]2 .

= E kk ij E qq ij + E kk ij 2 E ij + 2 E ij E qq ij + 2 E ij 2 E ij
= 2 E kk ii E qq + 2 E kk E ii + 2 E ii E qq + 4 2 E ij E ij = 3 2 E kk E qq + 4 E kk E ii + 4 2 E ij E ij
= (3 + 4 )E kk E qq + 4 2 E ij E ij

Universidad de Castilla- La Mancha


Ciudad Real - Espaa

Draft

Por: Eduardo W. V. Chaves (2014)

1 TENSORES

27

Con lo cual demostramos que P = 4 2 E : E + (3 + 4 )[Tr ( E )]2 .


Solucin 2: (Notacin tensorial)
En notacin tensorial tenemos que:
1
T = Tr ( E )1 + 2 E , W = T : E , y P = T : T
2

Luego:
1
1
1
T : E = (Tr ( E )1 + 2 E ) : E = (Tr ( E )1 : E + 2 E : E )
2
2
2
1

2
= (Tr ( E ) Tr ( E ) + 2 E : E ) = [Tr ( E )] + E : E
2
2

W=

P = T : T = (Tr ( E )1 + 2 E ) : (Tr ( E )1 + 2 E )
= [Tr ( E )] 1
: 1 + 2 Tr ( E ) 1
: E + 2 Tr ( E ) {
E : 1 + (2 ) 2 E : E
{
{
2

=3

= Tr ( E )

= Tr ( E )

= 3 [Tr ( E )] + 4 [Tr ( E )] + 4 E : E = (3 + 4 )[Tr ( E )] + 4 2 E : E


2

Ejemplo 1.41
Sea un tensor de segundo orden ij que es una funcin del tensor ij , ij = ij ( ij ) , y viene
dado por:
ij = kk ij + 2 ij

Tensorial

= Tr ( )1 + 2

donde , son constantes positivas. Partiendo de la expresin anterior, obtener la expresin


de ij en funcin de ij , es decir, ij = ij ( ij ) . Expresar el resultado en notacin indicial y
tensorial.
Solucin:
Notacin Indicial

Notacin Tensorial

ij = kk ij + 2 ij

= Tr ( )1 + 2

2 ij = ij kk ij

2 = Tr ( )1

ij =

1
ij
kk ij
2
2

Tr ( )1
2
2

Tenemos que obtener la siguiente traza kk , para ello obtenemos la traza de ij :


Notacin Indicial

Notacin Tensorial

ij = kk ij + 2 ij
: 1 = Tr ( )1 : 1 + 2 : 1

ii = kk ii + 2 ii = kk 3 + 2 kk

kk = (3 + 2 ) kk
kk =

Tr ( ) = Tr ( )3 + 2 Tr ( )

1
kk
(3 + 2 )

Universidad de Castilla- La Mancha


Ciudad Real - Espaa

Tr ( ) =

Draft

1
Tr ( )
(3 + 2 )

Por: Eduardo W. V. Chaves (2014)

MECNICA DEL MEDIO CONTINUO: PROBLEMAS RESUELTOS

28

Luego:
Notacin Indicial
ij =
=

Notacin Tensorial

1
ij
kk ij
2
2

Tr ( )1
2
2

1
1
ij
kk ij
2
2 (3 + 2 )

Tr ( )1
2
2 (3 + 2 )

Ejemplo 1.42
Consideremos T un tensor de segundo orden, demostrar las siguientes identidades:

(T ) = (T )
m T

T m

( )

Tr T T

( )

= Tr T m .

Solucin:

(T )

m T

( )

= (T T L T ) = T T T T L T T = T T
T

Q.E.D.

Para la segunda demostracin utilizaremos la propiedad de la traza Tr (T T ) = Tr (T )

( )

Tr T T

( )

= Tr T m

( )

= Tr T m

Q.E.D.

Ejemplo 1.43
Demostrar que: T : 1 = Tr (T ) .
Solucin:
T : 1 = Tij e i e j : kl e k e l
= Tij kl ik jl
= Tij ij = Tii = T jj
= Tr ( T )

Q.E.D.

Ejemplo 1.44
Probar que, si y D son tensores de segundo orden, la siguiente relacin es vlida:
D = Tr ( D )

Solucin: Partimos de la siguiente definicin:


D = ij D ji
= kj D jl ik il = kj D jl
= kj D jl lk
123
( D )

kl

= ( D) kl
= Tr ( D)

lk

lk

= ( D) kk = ( D) ll

Q.E.D.
Una segunda alternativa para la demostracin sera:
D = ij D ji = ij D jk ik
= ( D ) : 1
= Tr ( D )

Universidad de Castilla- La Mancha


Ciudad Real - Espaa

Draft

Q.E.D.
Por: Eduardo W. V. Chaves (2014)

1 TENSORES

29

Ejemplo 1.45
Demostrar que:
det (S ) S =

1
rjk tpq S rt S jp S kq
6

(1.23)

Solucin:
det (S ) = ijk S 1i S 2 j S 3k

(1.24)

pqr det(S ) = ijk S pi S qj S rk

(1.25)

pqr pqr det (S ) = pqr ijk S pi S qj S rk

(1.26)

1
424
3
6

det (S ) =

1
pqr ijk S pi S qj S rk
6

(1.27)

Ejemplo 1.46
Demostrar que: A tpq = rjk A rt A jp A kq
Solucin: Sabemos que:
A = rjk A r1 A j 2 A k 3

A tpq = rjk tpq A r1A j 2 A k 3

(1.28)

La expresin rjk tpq podr ser expresada en funcin de la delta de Kronecker como:
rjk tpq

rt rp rq
= jt jp jq
kt kp kq

(1.29)

= rt jp kq + rp jq kt + rq jt kp rq jp kt jq kp rt kq jt rp

Reemplazando en la expresin anterior (1.29) en la expresin (1.28), y utilizando la propiedad


del operador de sustitucin obtenemos que:
A tpq

= A t1 A p 2 A q 3 + A p1 A q 2 A t 3 + A q1 A t 2 A p 3 A q1 A p 2 A t 3 A t1 A q 2 A p 3 A p1 A t 2 A q 3
= A t1 1 jk A pj A qk + A t 2 2 jk A pj A qk + A t 3 3 jk A pj A qk

= rjk A rt A jp A kq = rjk A tr A pj A qk

Q.E.D.

NOTA: Vamos considerar que C = A B ( C ij = A ik B kj ), luego podemos decir que


C = A B = rjk C r1C j 2 C k 3 = rjk [A B ]r1 [A B ] j 2 [A B ]k 3 = rjk ( A rt B t1 )( A jpB p 2 )( A kqB q 3 )
= rjk A rt A jp A kqB t1B p 2B q 3 = A tpq B t1B p 2B q 3 = A B

Con lo cual hemos demostrado que A B = A B .


Solucin alternativa:

Universidad de Castilla- La Mancha


Ciudad Real - Espaa

Draft

Por: Eduardo W. V. Chaves (2014)

MECNICA DEL MEDIO CONTINUO: PROBLEMAS RESUELTOS

30

Considerando que tpq

1t 1 p 1q 1t 2t 3t
= 2t 2 p 2 q = 1 p 2 p 3 p
3 t 3 p 3 q 1q 2 q 3 q

y la propiedad A B = A B ,

podemos obtener que:


A tpq

1t 2t 3t A11 A12 A13


= rjk tpq A r1A j 2 A k 3 = 1 p 2 p 3 p A 21 A 22 A 23
1q 2 q 3q A 31 A 32 A 33
1t 2t 3t A11

= 1 p 2 p 3 p A 21
1q 2 q 3q A 31

A12
A 22
A 32

A13
A 23
A 33

Notar que 1t A11 + 2t A 21 + 3t A 31 = st A s1 = A t1 , con lo cual:


A tpq = rjk tpq A r1A j 2 A k 3

A t1

= A p1
A q1

At2
A p2
A q2

At3

A p 3 = rjk A tr A pj A qk
A q 3

Ejemplo 1.47
1
6

Demostrar que: A = rjk tpq A rt A jp A kq


Solucin:
Partiendo del problema anterior: A tpq = rjk A rt A jp A kq y multiplicando ambos lados por tpq ,
resulta:
A tpq tpq = rjk tpq A rt A jp A kq

(1.30)

Utilizando la propiedad tpq tpq = tt pp tp tp = tt pp tt = 6 . Luego, la relacin (1.30)


resulta:
A =

1
rjk tpq A rt A jp A kq
6

Q.E.D.

Ejemplo 1.48
Demostrar que la siguiente propiedad es vlida:

r r r
r r r
r r r
r r r
(B a) (b c) (B b) (a c) + (B c ) (a b) = Tr (B) a (b c)

(1.31)

Solucin:
El lado derecho de la ecuacin en notacin indicial queda:

Universidad de Castilla- La Mancha


Ciudad Real - Espaa

Draft

Por: Eduardo W. V. Chaves (2014)

1 TENSORES

31

ijk (B a) i b j c k ijk (B b) i a j c k + ijk (B c ) i a j b k =


= ijk [(B i1a1 + B i 2 a 2 + B i 3 a 3 )b j c k (B i1b1 + B i 2 b 2 + B i 3b 3 )a j c k +
+ (B i1 c 1 + B i 2 c 2 + B i 3 c 3 )a j b k ]
= ijk [(B i1a1b j c k + B i 2 a 2 b j c k + B i 3 a 3b j c k ) (B i1b1 a j c k + B i 2 b 2 a j c k + B i 3b 3 a j c k ) +
+ (B i1 c 1 a j b k + B i 2 c 2 a j b k + B i 3 c 3 a j b k )]
= ijk [B i1 (a1b j c k b1a j c k + c 1a j b k ) + B i 2 (a 2 b j c k b 2 a j c k + c 2 a j b k ) +
+ B i 3 (a 3b j c k b 3 a j c k + c 3 a j b k )]

= (1 jk B 11 + 2 jk B 21 + 3 jk B 31 )(a1b j c k b1a j c k + c 1 a j b k ) +
+ (1 jk B 12 + 2 jk B 22 + 3 jk B 32 )(a 2 b j c k b 2 a j c k + c 2 a j b k ) +

(1.32)

+ (1 jk B 13 + 2 jk B 23 + 3 jk B 33 )(a 3b j c k b 3 a j c k + c 3 a j b k )

Notar que:
a1 a 2
1 jk (a1b j c k b1a j c k + c 1a j b k ) = b1 b 2
c1

c2

a3
b 3 = ijk a i b j c k
c3

2 jk (a1b j c k b1a j c k + c 1a j b k ) = 3 jk (a1b j c k b1a j c k + c 1a j b k ) = 0


Con lo cual la ecuacin (1.32) queda:

r r r
B 11 ijk a i b j c k + B 22 ijk a i b j c k + B 33 ijk a i b j c k = (B 11 + B 22 + B 33 ) ijk a i b j c k = Tr (B) a (b c )

Notar que tambin se cumple que:

r r r
r r r
r r r
r r r
(B T a) (b c ) (B T b) (a c ) + (B T c ) (a b) = Tr (B) a (b c )

ya que Tr (B) = Tr (B T ) . Tambin es vlido que:

r
r r r r
r r r
r
r r r
(B a) (b c ) + a ((B b) c) + a (b (B c )) = Tr (B) a (b c )
r r
r r r
r
r r
r
r r r
[(B a), b, c ] + [a, (B b), c ] + [a, b, (B c )] = I B [a, b, c ]

(1.33)

Ejemplo 1.49
Demostrar que la siguiente propiedad es vlida:

r
r
r
r r r
( A a) ( A b) ( A c ) = det ( A ) a (b c )
r

(1.34)
r

donde A es un tensor de segundo orden no-singular, y a , b y c son vectores linealmente


independientes.
Solucin:
A tensor no-singular det( A ) A 0
r r r
r r r
a , b , c linealmente independientes a b c 0 .

Universidad de Castilla- La Mancha


Ciudad Real - Espaa

Draft

Por: Eduardo W. V. Chaves (2014)

MECNICA DEL MEDIO CONTINUO: PROBLEMAS RESUELTOS

32

(r r )

Escribimos el triple producto escalar en notacin indicial, i.e. a b c = ijk a i b j c k , y


multiplicamos por ambos lados de la igualdad por el determinante del tensor A , resultando:

r r r
a b c A = ijk a i b j c k A

Fue demostrado en el Ejemplo 1.47 que se cumple que A ijk = pqr A pi A qj A rk , con lo cual:

r r r
a b c A = ijk a i b j c k A = pqr A pi A qj A rk a i b j c k = pqr ( A pi a i )( A qj b j )( A rk c k )
r
r
r
= ( A a) ( A b) ( A c )

Ejemplo 1.50
Demostrar que:

r r
r r
det 1 + a b = 3 + 2 a b
r

(1.35)

donde y son escalares, a y b son vectores y 1 es el tensor identidad de segundo orden.


Solucin: Si denotamos por A ij ij + a i b j , el determinante de A viene dado por
A = ijk A i1 A j 2 A k 3 , donde A i1 = i1 + a i b1 , A j 2 =

+ a j b 2 y A k 3 = k 3 + a k b 3 ,

j2

luego podemos decir que:

r r
det 1 + a b = ijk ( i1 + a i b1 )

j2

+ a j b 2 ( k 3 + a k b 3 )

(1.36)

Desarrollando la expresin (1.36) obtenemos que:

r r
det 1 + a b = ijk 3 i1 j 2 k 3 + 2 a k b 3 i1 j 2 + 2 a j b 2 i1 k 3 + 2 a i b 1 j 2 k 3 +
+ a j b 2 a k b 3 i1 + 2 a i a k b 1b 3
2

j2

+ 2 a i a j b1b 2 k 3 + 3 a i a j a k b1b 2 b 3

Observemos que:
3 ijk i1 j 2 k 3 = 3 123 = 3

2 ( ijk a k b 3 i1 j 2 + ijk a j b 2 i1 k 3 + ijk a i b1 j 2 k 3 ) =

r r

2 (12 k a k b 3 + 1 j 3 a j b 2 + i 23 a i b1 ) = 2 (a 3b 3 + a 2 b 2 + a1b1 ) = 2 (a k b k ) = 2 (a b)
ijk a i a k b1b 3 j 2 = i 2 k a i a k b1b 3 = a1a 3b1b 3 a 3 a1b1b 3 = 0
ijk a i a j b1b 2 k 3 = ij 3 a i a j b1b 2 = 123 a1 a 2 b1b 2 213 a 2 a1b1b 2 = 0
ijk a i a j a k b1b 2 b 3 = 0
Notar que no haca falta expandir los trminos ijk a i a k b1b 3 j 2 , ijk a i a j b1b 2 k 3 ,
ijk a i a j a k b1b 2 b 3 , para saber que son iguales a cero, ya que
r

ijk a i a k b1b 3 j 2 = (a a) j b1b 3 j 2 = 0 y anlogamente para los otros trminos. Con lo que
hemos demostrado que:

r r
r r
det 1 + a b = 3 + 2 a b

Para = 1 tenemos que:

Q.E.D.

r r
r r
det 1 + a b = 1 + a b

Anlogamente, se puede demostrar que:

Universidad de Castilla- La Mancha


Ciudad Real - Espaa

Draft

Por: Eduardo W. V. Chaves (2014)

1 TENSORES

33

r r
det a b = 3 ijk a i a j a k b1b 2 b 3 = 0

NOTA: Podemos extrapolar la expresin (1.35) de tal forma que:

det I sym + A B = 3 + 2 A : B

(1.37)

donde I sym es el tensor identidad de cuarto orden simtrico, A y B son tensores de segundo
orden. Notar que det ( I sym ) = (1) 3 + (1) 2 (0)(0 : 0 ) = 1 y det (1 1) = (0) 3 + (0) 2 (1)(1 : 1) = 0 .
Ejemplo 1.51
r

Dado un tensor A , demostrar que existe un vector no nulo n 0 tal que A n = 0 si y solo si
det ( A ) = 0 , Chadwick (1976).
Solucin: Partimos del hecho que det ( A ) A = 0 y tambin escogemos una base arbitraria
r r r
{f , g, h} (linealmente independiente):
r r r
r
r
r
f g h A = ( A f ) ( A g) ( A h)

(ver Ejemplo 1.49)

Por el hecho que det ( A ) A = 0 , eso implica que:

r
r
r
( A f ) ( A g) ( A h) = 0
r
r
r
Con lo cual concluimos que los vectores ( A f ) , ( A g) , ( A h) son linealmente
dependientes. Esto implica que existen escalares no nulos 0 , 0 , 0 tal que:
r
r
r
r
( A f ) + ( A g) + ( A h) = 0
r
r r
r
A f + g + h = 0
r r
A n = 0
r
r r
r
r
donde n = f + g + h 0 .
r
r
r
Ahora escogemos dos vectores k , m que no son linealmente dependientes con n y
r r r
r r r
reemplazamos esta base {k , m, n} en lugar de los vectores {a, b, c} :
r r r
r
r
r
k m h A = ( A k ) [( A m) ( A n)]
r r r
r r r
r r
Considerando que A n = 0 y que k m h 0 , ya que la base {k , m, n} est constituida por

vectores linealmente independientes, obtenemos que:

r r r
k mh A =0
14243
0

A =0

Q.E.D.

Ejemplo 1.52
Sea un tensor de segundo orden arbitrario F . Demostrar que los tensores resultantes
C = F T F y b = F F T son tensores simtricos y semi-definidos positivos. Verificar tambin en que
condiciones C y b son tensores definidos positivos.
Solucin: Para demostrar que los tensores son simtricos, tenemos que demostrar que C = C T
y b = bT :

Universidad de Castilla- La Mancha


Ciudad Real - Espaa

Draft

Por: Eduardo W. V. Chaves (2014)

MECNICA DEL MEDIO CONTINUO: PROBLEMAS RESUELTOS

34

C T = (F T F )T = F T (F T )T = F T F = C
b T = (F F T ) T = (F T )T F T = F F T = b

(simetra)

Con lo cual hemos demostrado que los tensores C = F T F y b = F F T son simtricos.


Para demostrar que los tensores C = F T F y b = F F T son semi-definidos positivos,
partimos de la definicin de un tensor semi-definido positivo, es decir, un tensor A es semir
r
r r
definido positivo si se cumple que x A x 0 , para todo x 0 . Luego:
r
r
r
r
x C x = x (F T F ) x
r
r
= (F x ) (F x )
r 2
= F x 0

r
r
r
r
x b x = x (F F T ) x
r
r
= (F T x ) (F T x )
r 2
= FT x 0

En notacin indicial:
= x i ( Fki Fkj ) x j
= ( Fki x i )( Fkj x j )

x i C ij x j

= Fki x i

x i bij x j

= x i ( Fik F jk ) x j
= ( Fik x i )( F jk x j )
= Fik x i

Con lo cual demostramos que C = F T F y b = F F T son semi-definidos positivos.


r

Observemos que x C x = F x
r

solo ser igual a cero, con x 0 , si F x = 0 , y por

definicin F x = 0 con x 0 , si y solo si det ( F ) = 0 , ver Ejemplo 1.51. Luego, los tensores
C = F T F y b = F F T sern tensores definidos positivos si y solo si det ( F ) 0 .
Ejemplo 1.53

Consideremos los siguientes vectores dX (1) , dX ( 2) , dX (3) , dx (1) , dx ( 2) , dx (3) , en el cual estn
r
r
r
r
relacionados entre si a travs de las transformaciones dx (1) = F dX (1) , dx ( 2) = F dX ( 2) ,
r
r
dx (3) = F dX (3) , donde F es un tensor de segundo orden no-singular y F 1 . a.1) Obtener
la relacin entre los escalares dV y dV0 en funcin de F , sabiendo que
r
r
r
r
r
r
dV = dx (1) (dx ( 2 ) dx (3) ) 0 , dV0 = dX (1) (dX ( 2 ) dX (3) ) 0 . a.2) Obtener la relacin entre
r r
r
r
r
r
r
r
c = dX ( 2 ) dX (3) 0 y c * = dx ( 2 ) dx (3) 0 .
Solucin
a.1) Teniendo en cuenta el enunciado se cumple que:

r
r
r
r
r
r
dV = dx (1) (dx ( 2 ) dx (3) ) = ( F dX (1) ) ( F dX ( 2 ) ) ( F dX (3) )

Recordar que en el Ejemplo 1.49 hemos demostrado que

r
r r r
r
r
a b c A = ( A a) ( A b) ( A c )
r
r
r
r
r
r
dX (1) dX ( 2 ) dX (3) F = ( F dX (1) ) ( F dX ( 2 ) ) ( F dX (3) )

Con eso concluimos que:

r
r
r
r
r
r
dV = dx (1) (dx ( 2 ) dx (3) ) = ( F dX (1) ) ( F dX ( 2 ) ) ( F dX (3) )
r
r
r
= F dX (1) (dX ( 2) dX (3) )

Luego:
dV = F dV0

Universidad de Castilla- La Mancha


Ciudad Real - Espaa

Draft

Por: Eduardo W. V. Chaves (2014)

1 TENSORES

35

a.2) Teniendo en cuenta la ecuacin anterior, podemos obtener que:


dV = F dV 0
r
r
r
r
r
r
r
r
r
dx (1) (dx ( 2 ) dx (3) ) = F dX (1) dX ( 2 ) dX (3) = F ( F 1 dx (1) ) dX ( 2 ) dX (3)
r
r
r
r
r
r
dx (1) (dx ( 2 ) dx (3) ) = dx (1) F F T dX ( 2 ) dX (3)
r
r
r
r
(dx ( 2 ) dx (3) ) = F F T dX ( 2 ) dX (3)
r
r
c * = F F T c

])

NOTA 1: Notar que c * F c . Podemos reescribir la ecuacin anterior como

]
[

r
r
r
r
dx ( 2 ) dx (3) = F F T dX ( 2) dX (3)
r
r
r
r
( F dX ( 2 ) ) ( F dX (3) ) = F F T dX ( 2 ) dX (3)

El tensor F F T es conocido como tensor cofactor de F , i.e. cof ( F ) = F F T , con eso


definimos la inversa de un tensor como:
cof ( F ) = F F T

[F F ]

T T

= [cof ( F )]

F F 1 = [cof ( F )]

1
[cof (F )]T = 1 [adj(F )]
F
F

F 1 =

r
r
dx (1) = F dX (1)

r
dX (1)
r
dX ( 3 )

r
r
r
c = dX ( 2) dX (3)

r
r
r
c * = dx ( 2 ) dx ( 3 )

r
r
c* F c
r
r
c * = [cof ( F )] c

r
r
dx (3) = F dX (3)

dV = F dV0

r
dX ( 2 )
r
r
r
dV0 = dX (1) (dX ( 2 ) dX (3) ) 0

r
r
dx ( 2 ) = F dX ( 2 )
r
dV = dx (1)

(dx ( 2) dx (3) ) 0
r

NOTA 2: Vamos considerar un tensor de segundo orden F = A B , y tres vectores tal que
r r
r r r
r
r r
r
a (b c ) 0 , y a * = B a , b * = B b , c * = B c , luego, aplicando las definiciones anteriores
podemos decir que:

r
r r r
r
r
F a (b c ) = ( F a ) ( F b ) ( F c )

r
r
r
= ( A B a) ( A B b) ( A B c)
r
r
r
= ( A a*) ( A b*) ( A c*)
r
r
r
= A a * (b * c * )
r
r
r
= A (B a ) (B b ) (B c )
r r r
= A B a (b c )

Con lo cual concluimos que: si F = A B se cumple que F = A B = A B .

Universidad de Castilla- La Mancha


Ciudad Real - Espaa

Draft

Por: Eduardo W. V. Chaves (2014)

MECNICA DEL MEDIO CONTINUO: PROBLEMAS RESUELTOS

36

Ejemplo 1.54
Demostrar que si A y B son tensores ortogonales, el tensor resultante de la operacin
C = A B resulta ser otro tensor ortogonal.
Solucin: Por definicin, un tensor ( C ) es ortogonal cuando se cumple que C 1 = C T :
C 1 = ( A B) 1 = B 1 A 1 = B T A T = ( A B) T = C T

Q.E.D.

Ejemplo 1.55
Demostrar que adj( A B) = adj(B) adj( A ) y cof( A B) = [cof( A )] [cof(B)] .
Solucin:
Partiendo de la propia definicin de la inversa podemos decir que:
B 1 A 1 =

[adj(B)] [adj(A )]
B

A B B 1 A 1 = [adj(B)] [adj( A )] = [cof(B)]

A B (A B ) = [adj(B)] [adj( A )] =
1

AB

[cof( A)]T

( [cof(A)] [cof(B)] )

(1.38)

[adj(A B)] = [adj(B)] [adj(A)] = ([cof(A)] [cof(B)])T


A B

adj( A B) = [adj(B)] [adj( A )] = ([cof( A )] [cof(B)])

donde hemos utilizado la propiedad que A B = A B . Adems teniendo en cuenta la


definicin de adjunta y cofactor concluimos que:
adj( A B) = ([cof( A B)]) = ([cof( A )] [cof(B)]) [cof( A B)] = [cof( A )] [cof(B)]
T

(1.39)

Ejemplo 1.56
Demostrar que:
r
r
r r
( A a) ( A b) = [cof( A )] (a b)

(1.40)

Solucin:
Partiendo de la relacin A tpq = rjk A rt A jp A kq (ver Ejemplo 1.46), y multiplicando ambos
lados por a t b p , resultando:
A tpq a t b p = rjk A rt A jp A kq a t b p = rjk ( A rt a t )( A jp b p ) A kq

Multiplicamos ambos lados por A qs1 obtenemos que:


A tpq a t b p A qs1 = rjk ( A rt a t )( A jp b p ) A kq A qs1 = rjk ( A rt a t )( A jp b p ) ks = rjs ( A rt a t )( A jp b p )
1
Notar que A qs
=

[cof ( A )] sq
A

, con lo cual la ecuacin anterior queda:

Universidad de Castilla- La Mancha


Ciudad Real - Espaa

Draft

Por: Eduardo W. V. Chaves (2014)

1 TENSORES

1
A tpq a t b p A qs
= A tpq a t b p

[cof ( A )] sq
A

37

= [cof ( A )] sq tpq a t b p = rjs ( A rt a t )( A jp b p )

r
r r
r
[cof( A )] (a b) = ( A a) ( A b)

Ejemplo 1.57
Demostrar que:

r
r
r
r
r r
r
r
r
r r r
a ( A b) ( A c ) + ( A a) b ( A c ) + ( A a) ( A b) c = Tr ([cof ( A )]) a (b c )

(1.41)
Solucin:

En el ejemplo anterior hemos demostrado que [cof( A )] (a b) = ( A a) ( A b) , luego se


cumplen que:

r r
r
r
r
r
a [cof( A )] (b c ) = a ( A b) ( A c )
r
r
r r
r
r
r r
r
b [cof( A )] (a c ) = b [( A a) ( A c ) ] = ( A a) b ( A c )
r
r
r
r r
r
r
r
r
c [cof( A )] (a b) = c ( A a) ( A b) = ( A a) ( A b) c

Sumando las relaciones anteriores, podemos obtener que:

r r
r
r
r r
r
r r
a [cof( A )] (b c ) b [cof( A )] (a c ) + c [cof( A )] (a b) =
r
r
r
r
r r
r
r
r
= a ( A b) ( A c ) + ( A a) b ( A c ) + ( A a) ( A b) c

Segn el Ejemplo 1.48 se cumple que:

([cof( A)] ar ) (b cr ) ([cof( A)] b) (ar cr ) + ([cof( A)] cr ) (ar b) = Tr ([cof( A)])[rcr (ar b)]
r

r r
= II A [c (a b)]

donde II A = Tr [cof( A )] es el segundo invariante principal de A . Con lo cual demostramos


que:

r
r
r
r
r r
r
r
r
r r r
a ( A b) ( A c ) + ( A a) b ( A c ) + ( A a) ( A b) c = II A a (b c )

NOTA 1: Podemos resumir que:

] [

] [
] (ver Ejemplo 1.48)
r
r
r
r
r r
r
r
r
r r r
a [( A b) ( A c)] + ( A a) [b ( A c )] + ( A a) [( A b) c ] = II [a (b c )]
r
r
r
r r r
( A a) [( A b) ( A c )] = III [a (b c )] (ver Ejemplo 1.49)
r
r r r r
r
r r
r
r r r
( A a) (b c) + a ( A b) c ) + a b ( A c ) = I A a (b c )

Universidad de Castilla- La Mancha


Ciudad Real - Espaa

Draft

(1.42)
(1.43)
(1.44)

Por: Eduardo W. V. Chaves (2014)

MECNICA DEL MEDIO CONTINUO: PROBLEMAS RESUELTOS

38

r r r

donde I A = Tr (A ) , II A = Tr ([cof( A )]) , III A = det (A ) . Utilizando a (b c ) [a, b, c ] como


notacin, las ecuaciones anteriores pueden ser escritas como:
r r
r r r
r
r r
r
r r r
[( A a), b, c ] + [a, ( A b), c] + [a, b, ( A c )] = I A [a, b, c ]
r
r r
r
r
r r
r
r
r r r
[a, ( A b), ( A c )] + [( A a), b, ( A c )] + [( A a), ( A b), c ] = II A [a, b, c]
r
r
r
r r r
[( A a), ( A b), ( A c )] = III A [a, b, c ]
r

r r r

NOTA 2: Si consideramos tres vectores linealmente independientes [a (b c )] [a, b, c ] 0 ,


y tres vectores tal que:
r
r
r
r
f = 1a + 2 b + 3 c
r
r
r
r
g = 1 a + 2 b + 3 c
r
r
r
r
h = 1 a + 2 b + 3 c

r
f
r 1
g = 1
r
h 1

2 3 ar

2 3 b
r
2 3 c

(1.45)

Segn la regla de Cramer, (ver Ejemplo 1.16), se cumple que:

r r r
r r r
r r r
r r r
f (b c ) [ f , b, c ]
[a, f , c ]
[a, b, f ]
;
;
1 = r r r r r r
2 = r r r
3 = r r r
a (b c ) [a, b, c ]
[a, b, c ]
[a, b, c ]
r r r
r r r
r r r
[g, b, c ]
[a, g, c ]
[a, b, g]
;
;
1 = r r r
2 = r r r
3 = r r r
[a, b, c ]
[a, b, c ]
[a, b, c ]
r r r
r r r
r r r
[h, b, c ]
[a, h, c ]
[a, b, h]
;
;
1 = r r r
2 = r r r
3 = r r r
[a, b, c ]
[a, b, c ]
[a, b, c ]
r r r
r r r
Desarrollando el triple producto escalar [ f (g h)] [f , g, h] , podemos obtener que:

1
1

r r r
[ f (g h)] =

2 3
r r r
2 3 [a, b, c ]
2 3

r r r
r r r
r r r
[ f , b, c ] [a, f , c ] [a, b, f ]
r r r
r r r r r r
r r r
r r r
1
[
g
,
b
,
c
]
[
a
,
g
,
c
]
[
a
,
b
,
g
]
[
a
,
b
,
c
]
[
a
, b, c ]
r
=
P

r
r
r r r
r r r
[a, b, c ] r r r
[h, b, c ] [a, h, c ] [a, b, h]

donde
r r r

r r r

r r r

[ f , b, c ] [a, f , c ] [a, b, f ]
2 3
r r r
r r r
r r r
1
2 3 = r r r [gr , br , c ] [a, gr , c ] [a, br , gr ]
r
r
r
r
2 3 [a, b, c ] [h, b, c ] [a, h, c ] [a, b, h]

1
P = 1
1

(1.46)

Para el caso cuando f = A a , g = A b , h = A c , los invariantes principales de P vienen


dados por:

r r
r
r r r
r
r r
1
I P = Tr ( P ) = r r r [ A a, b, c] + [a, A b, c] + [a, b, A c ] = I A
[a, b, c]

Universidad de Castilla- La Mancha


Ciudad Real - Espaa

Draft

Por: Eduardo W. V. Chaves (2014)

1 TENSORES

1
II P = r r r
([a, b, c ]) 2

39

r r
r
r
r r
r r r
r r
r
r r r
r
r r
[a
, c ] [a, b, A b] [ A a, b, c ] [a, b, A a] [ A a, b, c ] [a, A a, c]
r, A b
r r r
r r
r r
r +
r r r
r r
r +
r
[a, A cr , cr ] [a
, b, A c ] [ A c, b, c] [a, b, A c ] [ A b, b, c ] [a, A b, c ]

= II A
III P = III A = det (A )

NOTA 3: Consideremos el sistema Cartesiano donde


r
r
a = a1e 1 + a 2 e 2 + a 3 e 3
a a a a e 1
2
3
r
r 1

b
b
e
b
e
b
e
b
b
b
b
=
=
+
+

1
1 1
2 2
3 3
2
3 e 2
r
cr c c
c 3 e 3
c = c 1e 1 + c 2 e 2 + c 3 e 3
2
1

r
r
r
Consideremos tambin que f = e 1 , g = e 2 , h = e 3 , luego, teniendo en cuenta la ecuacin

anterior y la ecuacin (1.45) podemos concluir que:


r
f
1
r
g = 1
r
h 1

2 3 ar

2 3 b
r
2 3 c

1 0 0 e 1 1
0 1 0 e =

2 1
0 0 1 e 3 1

2 3 a1 a 2 a 3 e 1

2 3 b1 b 2 b 3 e 2
2 3 c 1 c 2 c 3 e 3

luego
1

1
1

2 3 a 1 a 2 a 3 1 0 0
2 3 b1 b 2 b 3 = 0 1 0
2 3 c 1 c 2 c 3 0 0 1

1
1

2 3 a1 a 2 a 3
2 3 = b1 b 2 b 3
2 3 c 1 c 2 c 3

con lo cual podemos obtener la inversa de un tensor. Consideremos el tensor A cuyas


componentes son:
A 11
A ij = A 21
A 31

A 12
A 22
A 32

A 13 a1
A 23 = b1
A 33 c 1

a2
b2
c2

a3
b 3
c 3

r r r
A = [a, b, c ]

Luego, la inversa P = A 1 , (ver ecuacin (1.46)) resulta:


A 1

r r r
r r r
r r r
[ f , b, c ] [a, f , c ] [a, b, f ]
r r r
r r r
r r r
1
= r r r [g, b, c ] [a, g, c ] [a, b, g]
r r r
r r r
[a, b, c ] r r r
[h, b, c ] [a, h, c ] [a, b, h]

b1

c1

0
1
b1
=
A
c1

0
b1

c
1

a1

a2

a3

b2

b3

c2

c3

c1

c2

c3

a1

a2

a3

b2
c2

b3
c3

0
c1

1
c2

0
c3

0
b2

1
b3

a1
0

a2
0

a3
1

c2

c3

c1

c2

c3

Universidad de Castilla- La Mancha


Ciudad Real - Espaa

a 3
b1 b 2 b 3
b2

1 0
0

c2

a1 a 2 a 3

1 b1

b1 b 2 b 3 =

A c1

0 1
0
b1

a1 a 2 a 3
c 1

b1 b 2 b 3

0 0
1

a1

a2

Draft

b3
c3

a2

a3

c2

c3

b3

a1

a3

c3

c1

c3

b2
c2

a1
c1

a2
c2

a3

b2 b3

a1 a 3

b1 b 3

a1 a 2

b1 b 2

a2

Por: Eduardo W. V. Chaves (2014)

MECNICA DEL MEDIO CONTINUO: PROBLEMAS RESUELTOS

40

Teniendo en cuenta que A 1 =

[cof( A )]ij

b
2
c2

b
= 1
c
1
b1

c 1

b3

c3

1
[cof( A )]T = 1 [adj( A )] , podemos concluir que:
A
A

a2

a3

c2

c3

b3

a1

a3

c3

c1

c3

b2

c2

a1

a2

c1

c2

b
a3
2

b2 b3
c2

a
a1 a 3
= 2

c2
b1 b 3

a2
a1 a 2

b1 b 2
b 2

a2

b3

c3

b1

b3

c1

c3

a3

a1

a3

c3

c1

c3

a3
b3

a1
b1

a3
b3

b2

c1 c 2

a1 a 2

c1 c 2

a1 a 2

b1 b 2

b1

Notar que el coeficiente de la matriz anterior, [cof(A )]ij , es obtenido al resolver el


determinante de la matriz resultante al eliminar la fila i th y la columna j th , y cuyo resultado
multiplicamos por (1) i + j , por ejemplo:

[cof(A)]12 = (1)

1+ 2

a1 a 2
b1 b 2
c1

c2

a3
b b3
b3 = 1
c1 c 3
c3

Ejemplo 1.58
Dados los escalares I C , II C , III C en funcin de los escalares I E , II E , III E :
I C = 2I E + 3

II C = 4 I E + 4 II E + 3

III C = 2 I E + 4 II E + 8 III E + 1

(1.47)

Obtener la forma inversa, i.e. obtener I E , II E , III E en funcin de I C , II C , III C .


Solucin:
Las ecuaciones en (1.47) pueden ser reestructuradas como sigue:
I C 2 0 0 I E 3

II C = 4 4 0 II E + 3
III 2 4 8 III 1
E
C

2 0 0 I E I C 3
4 4 0 II = II 3
E C

2 4 8 III E III C 1
1

2 0 0 2 0 0 I E 2 0 0 I C 3

4 4 0 4 4 0 II E = 4 4 0 II C 3
2 4 8 2 4 8 III E 2 4 8 III C 1
1

I E 2 0 0 I C 3



II E = 4 4 0 II C 3
III 2 4 8 III 1
C

donde

Universidad de Castilla- La Mancha


Ciudad Real - Espaa

Draft

Por: Eduardo W. V. Chaves (2014)

1 TENSORES

2 0 0
= 4 4 0
2 4 8

[cof( A )]T

1 0
=

64 4

41

8
0
8

4 4

1
2 4
2

1
2 0

=
2 4
2

1
2 0
8

4 4

4 0
2 8
2 0
2 8

2 0
4 0

0
1
4
1
8

1
8

con lo cual, los escalares I E , II E , III E puede ser obtenidos como:


I E 2 0 0

II E = 4 4 0
III 2 4 8

1
I
3

2
C
1

II C 3 =
III 1 2
1
C
8

2 ( I C 3)
IC 3

1
0 II C 3 = ( 2 I C + II C + 3)

1 III C 1 1

8 ( I C II C + III C 1)
8

0
1
4
1
8

1.5 Descomposicin Aditiva de Tensores


Ejemplo 1.59
Encontrar un tensor de cuarto orden P tal que se cumpla que P : A = A dev , donde A es un
tensor de segundo orden.
Solucin: Teniendo en cuenta la descomposicin aditiva de un tensor en una parte esfrica y
otra desviadora, podemos obtener que:
A = A esf + A dev =

Tr ( A )
1 + A dev
3

A dev = A

Tr ( A )
1
3

Recurriendo a la definicin de los tensores identidades de cuarto rdenes definidos por:


I = 11 = ik jl e i e j e k e l = I ijkl e i e j e k e l

(1.48)

I = 11 = il jk e i e j e k e l = I ijkl e i e j e k e l

(1.49)

I = 1 1 = ij kl e i e j e k e l = I ijkl e i e j e k e l

(1.50)

donde se cumple que:

)(

I : A = ik jl e i e j e k e l : A pq e p e q = ik
= ik jl A kl e i e j = A ij e i e j = A

)(
(

jl A pq

kp lq (e i e j )

I : A = ij kl e i e j e k e l : A pq e p e q = ij kl A pq kp lq e i e j
= ij kl A kl e i e j = A kk ij e i e j = Tr ( A )1

(1.51)
(1.52)

Entonces, podemos decir que:


A dev = A

Tr ( A )
1
1
1

1 = I : A I : A = I I : A = I 1 1 : A
3
3
3
3

Con lo cual, concluimos que:

Universidad de Castilla- La Mancha


Ciudad Real - Espaa

Draft

Por: Eduardo W. V. Chaves (2014)

MECNICA DEL MEDIO CONTINUO: PROBLEMAS RESUELTOS

42

1
P = I 1 1
3

El tensor P es conocido como tensor proyeccin de cuarto orden, Holzapfel (2000).

1.6 Ley de Transformacin. Invariantes.


Ejemplo 1.60
Bajo una transformacin de base e i = a ij e j y considerando que las componentes de un tensor
de segundo orden en esta nueva base vienen dadas por:
Tij = a ik a jl Tkl

Demostrar que:
a) Tii = Tkk = Tr (T ) ; b) Tij T ji = Tkl Tlk ; c) det ( T ) = det ( T )
Solucin:
=j
a) Tij = a ik a jl Tkl i
Tii = a ik a il Tkl = kl Tkl = Tkk = Tll

b) Tij T ji = (a ik a jl Tkl )(a jp a iq T pq ) = a ik a iq a jl a jp Tkl T pq = kq lp Tkl T pq = Tqp T pq = Tkl Tlk


123 123
= kq

= lp

con lo cual hemos demostrado que Tr ( T 2 ) = Tr ( T T ) = Tij T ji


c) det ( Tij ) = det(a ik a jl Tkl ) = det (a ik )det (a jl )det ( Tkl ) = det( Tkl )
1
424
31
424
3
=1

=1

Hemos demostrado que Tkk = Tr ( T ) , Tkl Tlk = Tr ( T T ) y det ( T ) son invariantes.


Ejemplo 1.61
Bajo la transformacin de base e i = a ij e j y de las componentes del tensor de segundo orden
simtrico T en esta nueva base Tij = a ik a jl Tkl . Mostrar que I T , II T , III T son invariantes:
I T = Tr ( T ) = Tii

II T =

1
2
I T Tr ( T 2 )
2

III T = det ( T )

Solucin:
=j
a) Tij = a ik a jl Tkl i
Tii = a ik a il Tkl = kl Tkl = Tkk = Tll

b) Como ya hemos demostrado que I T es un invariante, para demostrar que II T es invariante


es suficiente demostrar que Tr ( T 2 ) = Tr ( T T ) = Tij T ji es un invariante.
Tij T ji = (a ik a jl Tkl )(a jp a iq T pq ) = a ik a iq a jl a jp Tkl T pq = kq lp Tkl T pq = Tqp T pq = Tkl Tlk
123 123
= kq

= lp

con lo cual, demostramos que Tr ( T 2 ) = Tr ( T T ) = Tij T ji es un invariante.


c) det ( Tij ) = det(a ik a jl Tkl ) = det (a ik )det (a jl )det ( Tkl ) = det( Tkl )
1
424
31
424
3
=1

=1

Acabamos de demostrar que Tkk = Tr ( T ) , Tkl Tlk = Tr ( T T ) , det ( T ) son invariantes.

Universidad de Castilla- La Mancha


Ciudad Real - Espaa

Draft

Por: Eduardo W. V. Chaves (2014)

1 TENSORES

43

Ejemplo 1.62
Demostrar que las siguientes relaciones son invariantes:
C12 + C 22 + C 32

C13 + C 23 + C 33

C14 + C 24 + C 34

donde C1 , C 2 , C 3 son los autovalores del tensor de segundo orden C .


Solucin: Cualquier combinacin de los invariantes principales ser un invariante. Intentaremos
expresar las relaciones anteriores en funcin de los invariantes principales.
Consideremos la siguiente relacin:

I C2 = (C1 + C 2 + C 3 ) = C12 + C 22 + C 32 + 2 C1 C 2 + C1 C 3 + C 2 C 3 C12 + C 22 + C 32 = I C2 2 II C


1444424444
3
2

II C

Comprobando que C12 + C 22 + C 32 es un invariante. Anlogamente podemos obtener las dems


relaciones, con lo cual resumimos a continuacin:
C1 + C 2 + C3 = I C
C12 + C 22 + C32 = I C2 2 II C
C13 + C 23 + C33 = I C3 3 II C I C + 3 III C
C14 + C 24 + C34 = I C4 4 II C I C2 + 4 III C I C + 2 II C2
C15 + C 25 + C35 = I C5 5 II C I C3 + 5 III C I C2 + 5 II C2 I C 5 III C II C

Notar que tambin se cumple que:

C1n +1 + C 2n +1 + C3n +1 = C1n + C 2n + C3n I C C1 C 2n 1 + C3n1 C 2 C1n1 + C3n1 C3 C1n 1 + C 2n1

Ejemplo 1.63
Demostrar que, si un tensor de segundo orden simtrico T tiene tres autovalores reales y
distintos, el espacio principal de T est formado por una base ortonormal.
Solucin: Retomando la definicin de autovalores dada por T n ( a ) = a n ( a ) (ndice a no indica
suma), si 1 , 2 , 3 (con 1 2 3 ) son los autovalores del tensor de segundo orden T
luego, se cumple que:
T n (1) = 1n (1)

T n ( 2 ) = 2 n ( 2)

T n (3) = 3 n (3)

(1.53)

Podemos multiplicar la primera expresin por n ( 2) y a la segunda por n (1) resultando:


n ( 2 ) T n (1) = 1n ( 2 ) n (1)

n (1) T n ( 2 ) = 2 n (1) n ( 2 )

(1.54)

Considerando T simtrico, se cumple que n ( 2) T n (1) = n (1) T T n ( 2) = n (1) T n ( 2) luego:


1n ( 2) n (1) = 2 n (1) n ( 2 )

(1.55)

Teniendo en cuenta que n ( 2) n (1) = n (1) n ( 2) , la relacin anterior queda:

( 1 2 ) n (1) n ( 2) = 0

(1.56)

Ya que 1 2 0 , para satisfacer (1.56) se debe cumplir que:


n (1) n ( 2 ) = 0

(1.57)

Anlogamente podemos demostrar que n (1) n (3) = 0 y n ( 2) n (3) = 0 . Con lo que


demostramos que los autovectores son versores ortogonales entre s, constituyendo as una
base ortonormal, donde la matriz de transformacin de base viene dada por:
Universidad de Castilla- La Mancha
Ciudad Real - Espaa

Draft

Por: Eduardo W. V. Chaves (2014)

MECNICA DEL MEDIO CONTINUO: PROBLEMAS RESUELTOS

44

n (1) n 1(1)


A = n ( 2) = n 1( 2)
n (3) n (3)

n (21)
n (22 )
n (23)

n 3(1)

n 3( 2 )
n (33)

(1.58)

NOTA: Si el tensor no es simtrico, los autovectores no necesariamente constituyen una base


ortonormal.
Ejemplo 1.64
Obtener las componentes de la siguiente operacin T = A T A T , donde Tij y a ij son las
componentes de los tensores T y A , respectivamente.
Solucin: La expresin T = A T A T en notacin simblica queda:
(e a e b ) = a rs (e r e s ) T pq (e p e q ) a kl (e l e k ) = a rs T pq a kl sp
T ab

ql

(e r e k )

= a rp T pq a kq (e r e k )

Para obtener las componentes de T es suficiente hacer el doble producto escalar por la base
(e i e j ) , resultando:
(e a e b ) : (e i e j ) = a rp T pq a kq (e r e k ) : (e i e j )
T ab
ai
T ab

bj

= a rp T pq a kq ri

kj

Tij = a ip T pq a jq

Observemos que esta operacin viene representada en forma matricial como:


T = A T AT

Si A es la matriz de transformacin entre bases ortonormales se cumple que A 1 = A T


luego, se cumple que T = A T T A , y la representacin de las componentes se muestran en
la Figura abajo:
T = A T AT

x3

T33

T23

x3

T13

T33

T31
T13

T23

T32

T12

T22

x2

T21

T22

T31

T11

T11

T32

T21

T12

x2

x1

x1

T = AT T A

Figura 1.1: Ley de transformacin de base para tensor de segundo orden.


Universidad de Castilla- La Mancha
Ciudad Real - Espaa

Draft

Por: Eduardo W. V. Chaves (2014)

1 TENSORES

45

Ejemplo 1.65
Consideremos que las componentes de un tensor de segundo orden T , en el sistema de
referencia (x1 , x 2 , x3 ) , estn representadas por:
3 1 0
= Tij = T = 1 3 0
0
0 1

(T )ij

Sabiendo que la matriz de transformacin de coordenadas del sistema (x1 , x 2 , x3 ) al sistema


(x1 , x 2 , x 3 ) viene dada por:

0
2
A=
2

0
2
2
2
2

Obtener las componentes del tensor Tij en el nuevo sistema de coordenadas (x1 , x 2 , x 3 ) .
Solucin: La ley de transformacin para un tensor de segundo orden es:
Tij = aik a jl Tkl

Para que la operacin anterior sea posible en forma matricial:


Tij = ai k Tk l ( al j ) T

Luego:
T = A T AT
0

2
T =
2

0
2
2
2
2

1
0

3
1
0

0 1 3 0 0

0
0
1

1
0

2
2
2
2
0

2
2
2

Efectuando la operacin de matrices obtenemos que:


1 0 0
T = 0 2 0
0 0 4

Universidad de Castilla- La Mancha


Ciudad Real - Espaa

Draft

Por: Eduardo W. V. Chaves (2014)

MECNICA DEL MEDIO CONTINUO: PROBLEMAS RESUELTOS

46

Ejemplo 1.66
Encontrar la matriz de transformacin del sistema ( x, y , z ) al sistema x, y, z , ver Figura 1.2:
z = z
z = z

y = y

x
x

Figura 1.2: Rotacin.


Solucin: Recordar que se cumple que: si tenemos un espacio inicial y sucesivas
transformaciones hasta el espacio final, la ley de transformacin del espacio inicial al espacio
final viene formada por el producto de las transformaciones en el sentido contrario. Es decir,
situamos en el espacio final y seguimos el sentido contrario de las flechas hasta el espacio
inicial, ver figura abajo.
B
r
a

r
a

B 1
r
a

espacio
inicial

A 1

C 1
CBA

C
r
a

espacio
final

A 1B 1 C 1 = (CBA ) 1 siortogonale
s A T B T C T = ( CBA ) T

Podemos observar que la obtencin del sistema x, y, z es una combinacin de rotaciones


mostradas a continuacin:

Universidad de Castilla- La Mancha


Ciudad Real - Espaa

Draft

Por: Eduardo W. V. Chaves (2014)

1 TENSORES

47

Rotacin segn eje z


z = z

del sistema x, y , z al x, y, z
y

cos sin 0
A = sin cos 0
0
0
1

con 0 360

Rotacin segn eje y


del sistema x, y, z al x, y, z

z = z

cos
B = 0
sin

con 0 180

y = y

0 sin
1
0
0 cos

z = z

x
x

Rotacin segn eje z


z = z
z = z

del sistema x, y, z al x, y, z
y = y

cos
C = sin
0

sin
cos
0

0
0
1

con 0 360

Universidad de Castilla- La Mancha


Ciudad Real - Espaa

Draft

Por: Eduardo W. V. Chaves (2014)

MECNICA DEL MEDIO CONTINUO: PROBLEMAS RESUELTOS

48

La matriz de transformacin del sistema ( x, y , z ) para el sistema x, y, z ser dada por:


D = CBA

Resultando:

(sin cos cos + cos sin ) sin cos


(cos cos cos sin sin )

D = ( cos cos sin sin cos ) ( sin cos sin + cos cos ) sin sin

cos sin
sin sin
cos

Los ngulos , , son conocidos como los ngulos de Euler.


Ejemplo 1.67
Si a ij representan las componentes de la matriz de transformacin de base demostrar que las
siguientes ecuaciones se cumplen:
2
2
2
a11
+ a12
+ a13
=1
2
2
2
a 21 + a 22 + a 23 = 1
2
2
2
a 31 + a 32 + a 33 = 1

a11 a 21 + a12 a 22 + a13 a 23 = 0

a 21 a31 + a 22 a 32 + a 23 a 33 = 0
a a + a a + a a = 0
12 32
13 33
11 31

2
2
2
a11
+ a 21
+ a 31
=1
2
2
2
a12 + a 22 + a 32 = 1
2
2
2
a13 + a 23 + a 33 = 1

a11 a12 + a 21 a 22 + a 31 a 32 = 0

a12 a13 + a 22 a 23 + a 32 a 33 = 0
a a + a a + a a = 0
21 23
31 33
11 13

Solucin:
Partimos del principio que la matriz de transformacin de base es una matriz ortogonal, i.e.
a ik a jk = a ki a kj = ij . Con lo cual:

a ik a jk = a i1 a j1 + a i 2 a j 2 + a i 3 a j 3 = ij

(i = 1, j = 1)

(i = 2, j = 2)

(i = 3, j = 3)

(i = 1, j = 2)

(i = 2, j = 3)
(i = 1, j = 3)

2
2
2
a11
+ a12
+ a13
=1
2
2
2
a 21
+ a 22
+ a 23
=1
2
2
2
a31
+ a 32
+ a 33
=1

a11 a 21 + a12 a 22 + a13 a 23 = 0


a 21 a 31 + a 22 a 32 + a 23 a 33 = 0
a11 a 31 + a12 a 32 + a13 a 33 = 0

Solucin Alternativa:
T

AA = 1

a11
a
21
a 31

a12
a 22
a 32

a13 a11
a 23 a12
a 33 a13

a 21
a 22
a 23

a 31 1 0 0
a 32 = 0 1 0
a 33 0 0 1

Al efectuar la multiplicacin de matrices obtenemos que:


2
2
2

+ a12
+ a13
a11

a11 a 21 + a12 a 22 + a13 a 23


a a + a a + a a
12 32
13 33
11 31

a11 a 21 + a12 a 22 + a13 a 23


2
2
2
+ a 22
+ a 23
a 21
a 21 a 31 + a 22 a 32 + a 23 a 33

Universidad de Castilla- La Mancha


Ciudad Real - Espaa

Draft

a11 a 31 + a12 a 32 + a13 a 33 1 0 0

a 21 a 31 + a 22 a 32 + a 23 a 33 = 0 1 0
2
2
2
0 0 1
+ a 32
+ a 33
a 31

Por: Eduardo W. V. Chaves (2014)

1 TENSORES

49

1.7 Autovalores, Autovectores y Transformaciones Ortogonales


Ejemplo 1.68
Demostrar que si Q es un tensor de segundo orden ortogonal propio, y E es un tensor de
segundo orden, los autovalores de E no cambian con la transformacin:
E* = Q E QT

Solucin:
Los autovalores ( i ) del tensor E obtenemos a partir del determinante caracterstico:

(
)
= det (Q E Q 1 )
= det (Q E Q Q 1 Q )
= det [Q (E 1 ) Q ]
(3) det (E 1) det
(Q3)
= det
12Q
1
424

0 = det E * 1

T
T

= det (E 1 )

(
= det (Q
= det (Q

0 = det E *ij ij

ik E kp Q jp

ij

ik E kp Q jp

Q ik Q jp kp

[ (

) ]
)det (Q )

= det Q ik E kp kp Q jp

= (Q ik )det E kp kp

= det E kp kp

jp

Con lo que comprobamos que E y E * tienen los mismos autovalores.


Ejemplo 1.69
Sea A un tensor de segundo orden y Q un tensor ortogonal. Si la ley de transformacin
ortogonal aplicada a A viene dada por A * = Q A Q T , demostrar que A 2 = Q A 2 Q T .
*

Solucin:
A 2 = A* A*

( A 2 ) ij = ( A * A * ) ij = A *ik A *kj

= ( Q A Q T ) (Q A Q T )
= QA Q QA Q
123
T

= (Q ip A pr Q kr )(Q ks A st Q jt )

= Q ip A pr Q kr Q ks A st Q jt
123

=1

= rs

= Q A A QT

= Q ip A pr rs A st Q jt = Q ip A ps A st Q jt

= Q A 2 QT

= Q ip ( A A ) pt Q jt
= (Q A 2 Q T ) ij

Ejemplo 1.70
Dadas las componentes del tensor T :
5 3 3
Tij = 2 6 3
2 2 4

a) Obtener los invariantes principales de T ;


b) Obtener el polinomio caracterstico asociado a T ;
Universidad de Castilla- La Mancha
Ciudad Real - Espaa

Draft

Por: Eduardo W. V. Chaves (2014)

MECNICA DEL MEDIO CONTINUO: PROBLEMAS RESUELTOS

50

c) Si 1 , 2 y 3 son los autovalores de T y 1 = 10 . Obtener 2 y 3 > 2 .


Solucin:
a) Los invariantes principales de T son:
I T = Tr ( T ) = 5 + 6 + 4 = 15 ; II T =

6 3
2 4

5 3

2 4

5 3

= 56 ; III T = det ( T ) = 60

2 6

b) El polinomio caracterstico se obtiene al resolver el determinante:


5
3
2
6
2

3
3

=0

3 2 I T + II T III T = 0 luego:

3 152 + 56 60 = 0

c) En el espacio principal se cumple que:


1 = 10 0
2
Tij = 0
0
0

3 > 2
0
0

donde los invariantes principales son


I T = Tr ( T ) = 1 + 2 + 3 = 15

2 + 3 = 5

III T = det ( T ) = 1 2 3 = 60

23 = 6

Combinando estas dos ecuaciones:


(31) = 3

+
=

(
5
)
6
5
6
0

(2)
3
3
3
3
2 + 3 = 5
3 = 2
23 = 6

Descartamos la solucin (32) = 2 por la imposicin del problema, luego, 3 = 3 . Resumiendo


as:
10 0 0
Tij = 0 2 0
0 0 3

I T = 10 + 2 + 3 = 15

donde se puede comprobar que: II T = 2 3 + 10 3 + 10 2 = 56


III = 10 2 3 = 60
T

Ejemplo 1.71
Determinar los valores principales y las direcciones principales del tensor cartesiano de
segundo orden T , cuyas componentes se representan matricialmente por:

(T )ij

3 1 0
= Tij = T = 1 3 0
0
0 1

Solucin: Buscamos soluciones no triviales para (Tij ij ) n j = 0 i , con la restriccin de que


n j n j = 1 . Como ya hemos visto, la solucin no trivial requiere la condicin:
Tij ij = 0

Explcitamente, la expresin anterior queda:


Universidad de Castilla- La Mancha
Ciudad Real - Espaa

Draft

Por: Eduardo W. V. Chaves (2014)

1 TENSORES

T11
T21

T12
T22

T13
T23

T31

T32

T33

51

3 1
= 1 3
0

0
0

=0

Desarrollando el determinante anterior obtenemos la ecuacin cbica:

(1 ) (3 ) 2 1 = 0

3 7 2 + 14 8 = 0

Podramos haber obtenido directamente la ecuacin caracterstica anterior a travs de los


invariantes:
I T = Tr ( Tij ) = Tii = T11 + T22 + T33 = 7
II T =

T
1
Tii T jj Tij Tij = 22
T32
2

T23
T33

T11

T13

T31

T33

T11

T12

T21

T22

= 14

III T = Tij = ijk Ti1 T j 2 Tk 3 = 8

utilizando la ecuacin caracterstica ser:


3 2 I T + II T III T = 0

3 72 + 14 8 = 0

Resolviendo la ecuacin cbica podemos obtener las tres races reales, puesto que la matriz T
es simtrica:
1 = 1;

2 = 2;

3 = 4

Podemos adems comprobar si los invariantes estn bien calculados utilizando la expresin de
los invariantes en funcin de los autovalores:
I T = 1 + 2 + 3 = 1 + 2 + 4 = 7

II T = 1 2 + 2 3 + 3 1 = 1 2 + 2 4 + 4 1 = 14
III T = 1 2 3 = 8

Con lo que podemos comprobar que los invariantes son los mismos que los obtenidos
anteriormente.
Clculo de las direcciones principales:
Para obtener las direcciones principales, utilizamos la definicin de autovalor-autovector,
donde cada autovalor i est asociado a un autovector n (i ) .
Para 1 = 1
3 1
1

1
3 1
0

0 n1 0
0 n 1 3 1 1

0 n 2 = 1 3 1 0 n 2 = 0
0 1 1 n 3 0
1 1 n 3 0

resultando el siguiente sistema de ecuaciones:


2n1 n 2 = 0

n1 = n 2 = 0
n1 + 2n 2 = 0
0n = 0
3
n i n i = n12 + n 22 + n 32 = 1

Luego, podemos obtener que: 1 = 1

Universidad de Castilla- La Mancha


Ciudad Real - Espaa

n (1) = [ 0 0 1] .

Draft

Por: Eduardo W. V. Chaves (2014)

MECNICA DEL MEDIO CONTINUO: PROBLEMAS RESUELTOS

52

NOTA: Esta solucin podra haberse determinado previamente por la situacin particular que
presentan las componentes del tensor. Al ser los trminos T13 = T23 = T31 = T32 = 0 , T33 = 1 ya
es un valor principal, como consecuencia esta direccin ya es una direccin principal.
Para 2 = 2
3 2
1

1
3 2
0

0 n1 0
n 1 3 2 1
n1 n 2 = 0 n 1 = n 2

n = 1 3 2

0 n 2 = 0 n 1 + n 2 = 0
2
n = 0
0
1 2 n 3 0
1 2 n 3 0
3
0
0

Podemos observar que las dos primeras ecuaciones son linealmente dependientes.
Necesitamos entonces de una ecuacin adicional:
n i n i = n12 + n 22 + n 32 = 1 2n12 = 1 n1 =

1
2

Luego:
2 = 2

1
n (2) =
2

1
2

Para 3 = 4
3 3
1

1
3 3
0

0 n1 0
0 n 1 3 4 1

0 n 2 = 0
0 n 2 = 1 3 4
0
1 4 n 3 0
1 3 n 3 0

n1 n 2 = 0

n 2 = n 2
n1 n 2 = 0
3n = 0
3

n i n i = n12 + n 22 + n 32 = 1 2n12 = 1 n1 =

1
2

Resultando:
3 = 4

1
n (3) =
2

1
2

Podemos entonces resumir que las direcciones principales correspondientes a sus valores
principales son:
1

n (1)

n (2)

n (3)

= [ 0 0 1]

1
2

1
2

1
2

1
2

NOTA: Las componentes utilizadas en este ejemplo son las mismas utilizadas en el Ejemplo
1.65. Adicionalmente, podemos verificar que los autovectores forman la matriz de
transformacin, A , entre el sistema original, (x1 , x 2 , x3 ) , y el espacio principal, (x1 , x 2 , x 3 ) ,
(ver Ejemplo 1.65).

Universidad de Castilla- La Mancha


Ciudad Real - Espaa

Draft

Por: Eduardo W. V. Chaves (2014)

1 TENSORES

53

Ejemplo 1.72
Dado un tensor ortogonal propio Q , a) demostrar que Q tiene un autovalor real e igual a 1 .
b) Demostrar tambin que Q puede ser representado en funcin de un ngulo tal que:
Q = p p + cos (q q + r r ) sin (q r r q )

donde p , q , r , son versores que constituyen una base ortonormal, siendo p la direccin
correspondiente al autovalor = 1 , es decir, p es autovector de Q . c) Obtener los invariantes
r
principales de Q en funcin del ngulo . d) Dado el vector posicin x , determinar el nuevo
r
vector formado por la transformacin ortogonal Q x en el espacio p , q .
Solucin:
a) Teniendo en cuenta la definicin de tensor ortogonal, podemos decir que:
QT Q = 1
QT Q QT = 1 QT
Q T (Q 1) = (Q T 1)
Q T (Q 1) = (Q 1) T

A continuacin obtenemos el determinante de los dos tensores anteriores:

det Q T (Q 1) = det (Q 1) T

[ ]

det Q det[(Q 1)] = det (Q 1)


1
424
3
T

= ( 1) 3 det (Q 1) T
T

] = det[(Q 1)]

= detQ =1

det[(Q 1)] = det[(Q 1)]

donde hemos utilizado las siguientes propiedades del determinante: det[A ] = 3 det[A ] ,
det[ A T ] = det[ A ] , det[A B ] = det[A ]det[B ] . El nico escalar que cumple la expresin anterior
es el cero, luego:
det[(Q 1)] = 0

Teniendo en cuenta la definicin de autovalor, det[(Q 1)] = 0 , concluimos que cuando


= 1 cumple det[(Q 1)] = 0 , luego = 1 es autovalor de Q . Adems, existe una direccin
(autovector) que cumple que Q e 1* = e 1* = e 1* .
b) Vamos considerar que p e 1* , q e *2 , r e *3 constituye una base ortonormal.
e 3
q e *2

e 1* p

Q e 1* = e 1*

e 2
e 1

r e *3

La representacin simblica del tensor en la base e 1* , e *2 , e *3 , viene dada por:

Universidad de Castilla- La Mancha


Ciudad Real - Espaa

Draft

Por: Eduardo W. V. Chaves (2014)

MECNICA DEL MEDIO CONTINUO: PROBLEMAS RESUELTOS

54

Q = Q *ij e *i e *j
* *
* *
* *
= Q11
e1 e 1* + Q12
e1 e *2 + Q13
e1 e *3 +

(1.59)

+ Q *21e *2 e 1* + Q *22 e *2 e *2 + Q *23 e *2 e *3 +


+ Q *31e *3 e 1* + Q *32 e *3 e *2 + Q *33 e *3 e *3

Teniendo en cuenta que e 1* es autovector de Q asociado al autovalor = 1 , se cumple que


Q e 1* = e 1* = e 1* , adems haciendo la proyeccin de Q , dado por (1.59), segn direccin e 1* ,
obtenemos que:
Q e 1* = e 1*

* *
* *
* *
Q e 1* = [ Q11
e1 e 1* + Q12
e1 e *2 + Q13
e1 e *3 +

+ Q *21e *2 e 1* + Q *22 e *2 e *2 + Q *23 e *2 e *3 +


+ Q *31e *3 e 1* + Q *32 e *3 e *2 + Q *33 e *3 e *3

] e 1*

* *
= Q11
e1 + Q *21e *2 + Q *31e *3
*
Con lo cual concluimos que Q11
= 1 , Q *21 = 0 , Q *31 = 0 .

Recordar que dos tensores coaxiales tienen las mismas direcciones principales. Un tensor y su
inversa siempre sern tensores coaxiales, luego si Q 1 = Q T , eso implica que Q T y Q son
coaxiales, y e 1* tambin ser direccin principal de Q T , luego se cumple que:
Q T e 1* = e 1*

* *
Q T e 1* = [ Q11
e1 e 1* + Q *21e 1* e *2 + Q *31e 1* e *3 +
* *
+ Q12
e 2 e 1* + Q *22 e *2 e *2 + Q *32 e *2 e *3 +
* *
+ Q13
e 3 e 1* + Q *23 e *3 e *2 + Q *33 e *3 e *3

] e 1*

* *
* *
* *
= Q11
e1 + Q12
e 2 + Q13
e3
*
*
*
Con lo cual concluimos que Q11
= 1 , Q12
= 0 , Q13
= 0 . Luego, la expresin (1.59) queda:

Q = e 1* e 1* + Q *22 e *2 e *2 + Q *23 e *2 e *3 + Q *32 e *3 e *2 + Q *33 e *3 e *3

(1.60)

En forma de matriz, las componentes de Q en la base e *i vienen dadas por:


x 2*
1 0
*
Q ij = 0 Q *22
0 Q *32

0
Q *23
Q *33

Q *22

Q *32
Q *23

*
Q11
=1

x1*

Q *33
x3*

Universidad de Castilla- La Mancha


Ciudad Real - Espaa

Draft

Por: Eduardo W. V. Chaves (2014)

1 TENSORES

55

Recurrimos una vez ms a la condicin de ortogonalidad Q T Q = Q Q T = 1 , o en funcin de


las componentes en el espacio e *i :
Q *ki Q *kj
1
0
0

= ij

[(Q )
[Q Q

* 2
22
*
*
22 23

1 0
0 Q *
22

0 Q *23

0 1 0
Q *32 0 Q *22
Q *33 0 Q *32

] [Q Q
] [(Q )

+ (Q *32 ) 2
+ Q *32 Q *33

*
*
22 23
* 2
33

0 1 0 0
Q *23 = 0 1 0
Q *33 0 0 1
1 0 0
= 0 1 0

0 0 1

(1.61)

]
]

+ Q *32 Q *33
+ (Q *23 ) 2

El determinante de un tensor ortogonal propio es det (Q) = +1 :


1 0
0 Q *
22

0 Q *32

0
Q *23 = 1
Q *33

Q *22 Q *33 Q *23 Q *32 = 1

(1.62)

Teniendo en cuenta (1.61) y (1.62) tenemos el siguiente conjunto de ecuaciones:


(Q *22 ) 2 + (Q *32 ) 2 = 1
* *
*
*
Q 22 Q 23 + Q 32 Q 33 = 0
* 2
* 2
(Q 33 ) + (Q 23 ) = 1
* *
*
*
Q 22 Q 33 Q 23 Q 32 = 1

cos 2 + sin 2 = 1

cos ( sin ) + sin cos = 0


2
2
cos + sin = 1
cos cos ( sin )(sin ) = 1

Con lo cual hemos demostrado la existencia de un ngulo que cumpla con las condiciones
anteriores.
Q *ij

1 0
= 0 Q *22
0 Q *23

0 1
0
0

*
Q 32 = 0 cos sin
Q *33 0 sin cos

(1.63)

Retomando la expresin (1.60), y teniendo en cuenta (1.63), concluimos que:


Q = e 1* e 1* + (cos ) e *2 e *2 + ( sin )e *2 e *3 + (sin )e *3 e *2 + (cos ) e *3 e *3

= e 1* e 1* + cos e *2 e *2 + e *3 e *3 sin e *2 e *3 e *3 e *2

Considerando que p e 1* , q e *2 , r e *3 , demostramos que:


Q = p p + cos (q q + r r ) sin (q r r q )

Es interesante verificar que la descomposicin aditiva de Q en una parte simtrica y otra


antisimtrica, en el espacio e *i , resulta:
0
0
1

0
= 0 cos
0
0
cos
1444
4244443

sym
Q *ij

[p p +cos (q q +r r )]ij

0
0
0

0
= 0
sin
0 sin
0
1444 4
4244444
3

anti
Q *ij

[sin (q r r q ) ]ij

anti

Observemos que el formato de Q *ij


tiene el mismo formato que presenta un tensor
antisimtrico ( W ) en el espacio definido por su vector axil:

Universidad de Castilla- La Mancha


Ciudad Real - Espaa

Draft

Por: Eduardo W. V. Chaves (2014)

MECNICA DEL MEDIO CONTINUO: PROBLEMAS RESUELTOS

56

Wij*

0 0
= 0 0
0

0

0

donde es el mdulo del vector axil.


c) Teniendo en cuenta (1.63), queda de fcil demostracin que I Q = II Q = 1 + 2 cos , III Q = 1 .
r

d) Representamos el vector posicin x a travs de sus componentes y la base p , q , r :


r
x = pp + qq + rr .
r

Luego, se cumple que: x p = ( pp + qq + rr ) p = p


ver Figura 1.3, se cumple que:

r
x q = q

r
~ = Q xr = p p + cos (q q + r r ) sin (q r r q )
x
= pp + (q cos r sin )q + (r cos + q sin )r

r
x r = r , y tambin,

] [pp + qq + rr ]

e 1* p
q

r
x

e *2
O

r
r
~
x =Q x

r e *3

Figura 1.3

Ejemplo 1.73

Considrense las transformaciones tensoriales p = U p y p = R p , donde R es un tensor


ortogonal de segundo orden y U es un tensor de segundo orden con U U 1 = 1 , i.e. U 1 .
r r
Obtener las leyes de transformacin entre p y p .

Universidad de Castilla- La Mancha


Ciudad Real - Espaa

Draft

Por: Eduardo W. V. Chaves (2014)

1 TENSORES

57

Solucin:
El problema planteado se puede esquematizar a travs de la siguiente figura:
R

r
p

r
p

r
p

?
Teniendo en cuenta que R 1 = R T (tensor ortogonal), es decir, existe la inversa de R y
r
r
considerando p = R p obtenemos que:
r
r
r
r
r
r r
p = R p R 1 p = R 1 R p
R 1 p = 1 p = p
r
r
r
r
Reemplazando p = R 1 p en p = U p , obtenemos que:
r
r
r
r
p = U p
p = U p
r
r
r
r
R 1 p = U p
R 1 p = U p
r
r
r
r
R R 1 p = R U p
U 1 R 1 p = U 1 U p
r
r
r
r r
1 p = R U p
(R U) 1 p = 1 p = p
r
r
r
r
p = (R U) p
p = (R U) 1 p

(1.64)

o en Notacin indicial:
p i = U ij p j

p i = U ij p j

R ij1p j = U ij p j

R ij1p j = U ij p j

R ki R ij1p j = R ki U ij p j

U ki1R ij1p j = U ki1U ij p j

kj p j = R ki U ij p j

(R ki U ij ) p j = kj p j = p k

p k = (R ki U ij )p j

p k = (R ki U ij ) 1 p j

r
p

U 1

r
p

(1.65)

R 1 = R T
(R U)

r
p

(R U) 1 = U 1 R T

Universidad de Castilla- La Mancha


Ciudad Real - Espaa

Draft

Por: Eduardo W. V. Chaves (2014)

MECNICA DEL MEDIO CONTINUO: PROBLEMAS RESUELTOS

58

1.8 Representacin Espectral


Ejemplo 1.74
Sea w un tensor antisimtrico de segundo orden y V un tensor de segundo orden definido
positivo cuya representacin espectral viene dado por:
V=

n ( a ) n ( a )

a =1

Demostrar que el tensor antisimtrico

w puede ser representado por:


3

w = w ab n (a ) n (b)
a ,b =1
a b

Demostrar tambin que se cumple la relacin:


3

w V V w = w ab ( b a ) n ( a ) n (b)
a ,b =1
a b

Solucin:
Es cierto que

a =1

a =1

a ,b =1

w 1 = w n ( a) n (a ) = w n ( a ) n ( a) = (wr n (a ) ) n ( a ) = wb (n (b) n ( a ) ) n ( a)
3

a =1

donde hemos aplicado la propiedad de un tensor antisimtrico w n = w n , donde w es el


vector axil asociado al tensor w . Expandiendo la expresin anterior obtenemos que

w = wb (n (b) n (1) ) n (1) + wb (n (b) n ( 2) ) n ( 2) + wb (n (b) n (3) ) n (3) =

(
+ w (n
+ w (n

= w1 n (1) n (1) n (1) + w2 n ( 2) n (1) n (1) + w3 n (3) n (1) n (1) +


1

)
) n

(
(n

)
) n

(
(n

)
) n

(1)

n ( 2 ) n ( 2 ) + w2 n ( 2 ) n ( 2) n ( 2) + w3 n (3) n ( 2) n ( 2 ) +

(1)

( 3)

( 3)

+ w2

( 2)

( 3)

( 3)

+ w3

( 3)

( 3)

( 3)

Simplificando la expresin anterior resulta que:

w = w2 (n (3) ) n (1) + w3 (n (2) ) n (1) + w1 (n (3) ) n (2) w3 (n (1) ) n ( 2) +

( )

( )

w1 n ( 2) n (3) + w2 n (1) n (3)

Adems teniendo en cuanta que w1 = w 23 = w 32 , w2 = w13 = w 31 , w3 = w12 = w 21 ,


an puede ser expresado por:

w = w 31 n (3) n (1) + w 21 n ( 2) n (1) +


+ w 32 n (3) n ( 2 ) + w12 n (1) n ( 2 ) +
+ w 23 n ( 2 ) n (3) + w13 n (1) n (3)
el cul es exactamente igual a
3

w = w ab n (a ) n (b)
a ,b =1
a b

Universidad de Castilla- La Mancha


Ciudad Real - Espaa

Draft

Por: Eduardo W. V. Chaves (2014)

1 TENSORES

Los trminos

wV

59

y V w pueden ser obtenidos como sigue a continuacin:

3
3

(
)
(
)
a
b
w V = w ab n n b n (b) n (b)
b =1
a ,b =1

ab

b w ab n ( a ) n (b ) n (b ) n (b ) =

a ,b =1
a b

ab

n ( a ) n (b )

a ,b =1
ab

3
3
a
a
a
b
(
)
(
)
(
)
(
)
V w = a n n
w ab n n = a w ab n ( a) n (b)
a ,b =1
a =1
a ,b =1
a b
a b

Luego,

3
3

(
)
(
)
(
)
(
)
a
b
a
b
w V V w = b w ab n n a w ab n n

a ,b =1
a ,b =1

ab
a b

ab ( b

a ) n ( a ) n (b )

a ,b =1
a b

Anlogamente, es posible demostrar que:


3

w V 2 V 2 w = w ab (2b 2a ) n ( a ) n (b)
a ,b =1
a b

Ejemplo 1.75
Dado un tensor definido positivo C , cuyas componentes cartesianas de este tensor vienen
dadas por:
2 0 1
C ij = 0 4 0
1 0 2

Obtener los siguientes tensores: a) C 2 ; b) U = C . c) Comprobar si los tensores C y U son


coaxiales.
Solucin:
Observemos que los tensores C 2 y U = C son tensores coaxiales con el tensor C .
Haciendo la representacin espectral del tensor C :
C=

N
a

(a)

(a )
N

a =1

( a ) son los autovectores del tensor C . Luego,


donde a son los autovalores del tensor C , y N
se cumple que:

Universidad de Castilla- La Mancha


Ciudad Real - Espaa

Draft

Por: Eduardo W. V. Chaves (2014)

MECNICA DEL MEDIO CONTINUO: PROBLEMAS RESUELTOS

60

C2 =

(a) N
(a)
2a N

U= C =

a =1

(a) N
(a )
aN

a =1

Clculo de los autovalores y autovectores del tensor C .


Podemos verificar que segn la estructura de las componentes del tensor C ya conocemos un
auto valor 2 = 4 que est asociado a la direccin N i( 2) = [0 1 0] . Para calcular los restantes
autovalores obtenemos el determinante caracterstico siguiente:
2
1
=0
1
2

( 2 ) 2 = 12

1 = 2 1 = 1
( 2 ) = 1
3 = 2 + 1 = 3

Asociado al autovalor 1 = 1 tenemos el siguiente autovector:


1 N 1(1) 0

=
2 1 N 3(1) 0

2 1
1

1 1 N 1(1) 0
1 1 (1) = 0

N 3

N 1(1) = N 3(1)

con la restriccin N i(1) N i(1) = 1 , resultando que:


(1) N
(1) + N
(1) N
(1) + N
(1) N
(1) = 1
N
1
1
2
2
3
3

(1) N (1) + N
(1) N (1) = 1
N
1
1
1
1

(1) = 1
N
1
2

(1) = N (1) = m 1
N
3
1
2

Asociado al autovalor 3 = 3 tenemos el siguiente autovector:


2 3
1

N 1(3) 0

=
2 3 N (33) 0
1

(3) 0
1 1 N
1
1 1 (3) = 0

N 3

N 1(3) = N 3(3)

(3) + N (3) N (3) = 1


N 1(3) N
1
1
1

1
N 1(3) =
2

con la restriccin N i(3) N i(3) = 1 , resultando que:


(3) N (3) + N (3) N
( 3) + N
(3) N (3) = 1
N
1
1
2
2
3
3

(3) = N (3) = 1
N
3
1
2

Resumiendo tenemos que:


1 = 1
2 = 4
3 = 3

(1) =
N
i

N ( 2 ) = [0
i
(
3) =
N
i

2
2

1 0]

1
1
0

2
2

0 m

transformacin
Matriz
de

A=

1
2
0
1
2

0
1
0

2
0
1
2

Luego se cumple que:


C = A C AT C = AT C A

En el espacio principal tenemos que:

Universidad de Castilla- La Mancha


Ciudad Real - Espaa

Draft

Por: Eduardo W. V. Chaves (2014)

1 TENSORES

1 0 0
Cij = 0 4 0
0 0 3

1
2
Cij = 0

U = C
ij
ij

61

0
16 0
0 9
0

=0
0

0
4
0

0 1 0

0 = 0 2
3 0 0

0
0
3

Observemos que el tensor C es un tensor definido positivo, luego sus autovalores son
positivos. En espacio original tenemos las siguientes componentes:

2
C ij =

1
2
0
1
2

0
1
0

2
0
1
2

1
1 0 0 2
0 16 0 0

0 0 9 1
2

0
1
0

2 5 0 4
0 = 0 16 0
1
4 0 5
2

Observemos que este resultado podra haber sido obtenido fcilmente a travs de la operacin
C 2 = C C , o en componentes:
C ij2

2 0 1 2 0 1 5 0 4
= C ik C kj = 0 4 0 0 4 0 = 0 16 0
1 0 2 1 0 2 4 0 5

Anlogamente:

U ij =

1
2
0
1
2

0
1
0

2
0
1
2

1 0
0 2

0 0

0
0

1
2
0
1
2

0
1
0

1 3 +1
0

2 2
2
0 = 0
1 3 1
0
2 2

3 1

2
0
3 + 1
2

c) Los tensores C y U son coaxiales ya que hemos obtenido los autovalores de U en el


espacio principal de C . Tambin podemos comprobar que son tensores coaxiales porque se
cumple que C U = U C , en componentes.
3 +1
0
2 0 1
2

2
C ik U kj = 0 4 0 0
1 0 2 3 1 0
2

3 +1
0

2
2
U ik C kj = 0
3 1 0
2

3 1
3,098 0 2,098
2
0 = 0
8
0
3 + 1 2,098 0 3,098

3 1
2 0 1 3,098 0 2,098
2
0 0 4 0 = 0
8
0
3 + 1 1 0 2 2,098 0 3,098

Ejemplo 1.76
Sea C un tensor de segundo orden simtrico y R un tensor ortogonal propio. Las
componentes de estos tensores en el sistema Cartesiano vienen dadas por:
Universidad de Castilla- La Mancha
Ciudad Real - Espaa

Draft

Por: Eduardo W. V. Chaves (2014)

MECNICA DEL MEDIO CONTINUO: PROBLEMAS RESUELTOS

62

2 0 1
C ij = 0 4 0
1 0 2

R ij =

0
2
2
2
2

0
2
2
2
2

a) Obtener los siguientes tensores: a.1) C 8 ; a2) U = C .


b) Obtener tambin los invariantes principales de C .
c) Teniendo en cuenta que los tensores b y C estn relacionados entre si a travs de la
siguiente transformacin ortogonal C = R T b R , obtener el tercer invariante principal de b .
Solucin:
0
3280
3281

a) Anlogo al Ejemplo 1.75. Respuesta: C = 0


65536
0
3280
0
3281
8

b)
I C = Tr (C ij ) = C ii = C11 + C 22 + C 33 = 8
II C =

4 0 2 1 2 0
1
C ii C jj C ij C ij =
+
+
= 19 ; III C = C = ijk C i1C j 2 C k 3 = 12
0 2 1 2 0 4
2

c) Teniendo en cuenta las propiedades de determinante, el tercer variante principal de b puede


ser expresado por:
C det (C ) = det (R T b R ) = det (R T )det (b)det (R ) = det (b) = III b = 12
1
424
3
123
= +1

= +1

Ejemplo 1.77
Sea un tensor de segundo orden simtrico S con det (S ) 0 . Considerando que S tiene dos
autovalores iguales, i.e. S 2 = S 3 y S1 S 2 , demostrar que S puede ser representado por:
S = S 1n (1) n (1) + S 2 (1 n (1) n (1) )

donde n (1) es el autovector de S asociado al autovalor S1 , 1 es el tensor identidad de


segundo orden.
Solucin:
Partimos de la representacin espectral de S :
S=

S n

(a)

n ( a )

a =1

= S1n (1) n (1) + S 2 n ( 2 ) n ( 2 ) + S 3 n (3) n (3)

(1.66)

= S1n (1) n (1) + S 2 (n ( 2 ) n ( 2 ) + n (3) n (3) )

Recordar que 1 es un tensor esfrico, con lo cual cualquier direccin es una direccin
principal. Partiendo de este principio adoptamos el espacio principal de S para hacer la
representacin espectral de 1 :

Universidad de Castilla- La Mancha


Ciudad Real - Espaa

Draft

Por: Eduardo W. V. Chaves (2014)

1 TENSORES

1=

(a)

63

n ( a ) = n (1) n (1) + n ( 2 ) n ( 2 ) + n (3) n (3)

(1.67)

a =1

( 2)

( 2)

+ n

( 3)

( 3)

= 1 n

(1)

(1)

Reemplazando lo anterior en (1.66), obtenemos que:


S = S 1n (1) n (1) + S 2 (n ( 2) n ( 2 ) + n (3) n (3) ) = S 1n (1) n (1) + S 2 (1 n (1) n (1) )

1.9 Teorema de Cayley-Hamilton


Ejemplo 1.78
Dado un tensor de segundo orden arbitrario T , demostrar el teorema de Cayley-Hamilton,
que afirma que todo tensor cumple su propia ecuacin caracterstica.
Solucin:
Partimos de la ecuacin caracterstica del tensor: 3 2 I T + II T III T = 0 , cuya ecuacin se
cumple para los autovalores de T , 1 , 2 , 3 , luego:
31 21 I T + 1 II T III T = 0
32 22 I T + 2 II T III T = 0
33 23 I T + 3 II T III T = 0

Reestructurando las ecuaciones anteriores en forma matricial, obtenemos que:


31

0
0

0
32
0

0 21

00
33 0

0
22
0

1
0

0 I T + 0
0
23

0
2
0

0
1 0 0
0 0 0

0 II T 0 1 0 III T = 0 0 0
0 0 1
0 0 0
3

(1.68)

Tij 3 Tij 2 I T + Tij II T III T ij = 0 ij

Notar que en el espacio principal de T se cumple que:


1

Tij = 0
0

0
2
0

0
3

2
Tij = Tik Tkj = 0
0

0
2
0

3
T pj = 0
Tij = Tik Tkp
0

0 1

0 0
3 0
0
2
0

0
2
0

0 1

0 0
3 0

0 21

0 =0
3 0
0
2
0

0 1

0 0
3 0

0
22
0
0
2
0

0
23
31

=0
3 0
0
0

0
32
0

0
33

Teniendo en cuenta la ley de transformacin entre espacios de un tensor de segundo orden, i.e.
Tij = Fik Tkp F pj1 , donde Fij es la matriz de transformacin del espacio original ( Tij ) al espacio
principal ( Tij ). Notar que tambin se cumple que Tij 2 = Fik Tkp2 F pj1 y Tij 3 = Fik Tkp3 F pj1 , (ver
Ejemplo 1.69). Con lo cual la ecuacin en (1.68) puede ser reescrita como:
Universidad de Castilla- La Mancha
Ciudad Real - Espaa

Draft

Por: Eduardo W. V. Chaves (2014)

MECNICA DEL MEDIO CONTINUO: PROBLEMAS RESUELTOS

64

Tij 3 Tij 2 I T + Tij II T III T ij = 0 ij


3
Fik Tkp
F pj1 Fik Tkp2 F pj1 I T + Fik Tkp F pj1 II T III T Fik kp F pj1 = 0 ij

Fik Tkp3 Tkp2 I T + Tkp II T III T kp F pj1 = 0 ij

Fsi1 Fik

3
Tkp

3
sk Tkp

Tst3

Tkp2 I T

Tkp2 I T

Tst2 I T

+ Tkp II T III T kp F pj1 F jt = Fsi1 0 ij F jt = 0 st

+ Tkp II T III T kp pt = Fsi1 0 ij F jt = 0 st

+ Tst II T III T st = 0 st

T T I T + T II T III T 1 = 0

Solucin alternativa:
En el Ejemplo 1.57 (NOTA 1) hemos resumido que:
r r
r r r
r
r r
r
r r r
[( A a), b, c] + [a, ( A b), c ] + [a, b, ( A c )] = I A [a, b, c]
r
r r
r
r
r r
r
r
r r r
[a, ( A b), ( A c )] + [( A a), b, ( A c )] + [( A a), ( A b), c ] = II A [a, b, c]
r
r
r
r r r
[( A a), ( A b), ( A c )] = III A [a, b, c ]
r r r

donde [a, b, c ] a (b c ) 0 se cumple con a 0 , b 0 , c 0 . Ahora si consideramos


r

que el vector a viene dado por a = A f podemos obtener que:


r r
r r r
r
r r
r
r r r
[( A a), b, c] + [a, ( A b), c ] + [a, b, ( A c)] = I A [a, b, c ]
r r r
r
r r
r r
r r r
r
[( A A f ), b, c ] + [( A f ), ( A b), c ] + [( A f ), b, ( A c )] = I A [( A f ), b, c ]
r r r
r
r r
r r
r r r
r
[( A 2 f ), b, c ] + [( A f ), ( A b), c] + [( A f ), b, ( A c)] = I A [( A f ), b, c ]
r r r
r r r
r
r r
r r
r
[( A 2 f ), b, c ] I A [( A f ), b, c ] = [( A f ), ( A b), c ] [( A f ), b, ( A c)]

(1.69)

Teniendo en cuenta la definicin de II A , es cierto que:


r
r
r r
r
r r
r r r
r
r
[f , ( A b), ( A c )] + [( A f ), b, ( A c )] + [( A f ), ( A b), c ] = II A [f , b, c ]
r
r
r r r
r r
r
r r
r
r
[f , ( A b), ( A c )] II A [ f , b, c ] = [( A f ), b, ( A c )] [( A f ), ( A b), c]

Reemplazando la ecuacin anterior en la ecuacin (1.69) podemos obtener que:

r r r
r r r
r
r r
r r
r
[( A 2 f ), b, c ] I A [( A f ), b, c ] = [( A f ), ( A b), c] [( A f ), b, ( A c )]
r r r
r r r
r
r
r r r
r
[( A 2 f ), b, c ] I A [( A f ), b, c] = [f , ( A b), ( A c )] II A [ f , b, c ]
r r r
r r r
r r r
r
r
r
[( A 2 f ), b, c ] I A [( A f ), b, c ] + II A [f , b, c ] [f , ( A b), ( A c )] = 0
r r r
r r r
r r r r
r
r
( A 2 f ) (b c ) I A ( A f ) (b c) + II A f (b c ) f ( A b) ( A c ) = 0
r
r r
r
En el Ejemplo 1.56 hemos demostrado que ( A b) ( A c) = [cof( A )] (b c ) se cumple,

luego, la ecuacin anterior puede ser reescrita como

r r r
r r r
r r r r
r
r
( A 2 f ) (b c ) I A ( A f ) (b c) + II A f (b c ) f ( A b) ( A c ) = 0
r r r
r r r
r r r r
r r
( A 2 f ) (b c ) I A ( A f ) (b c) + II A f (b c) f [cof( A )] (b c) = 0
r
r
r r
r r
( A 2 f ) I A ( A f ) + II A f f [cof( A )] (b c ) = 0
r
r r r r
r r
r
Notar que los vectores ( A 2 f ) , ( A f ) , f 0 , f [cof( A )] no son ortogonales a (b c ) 0 ,

con lo cual podemos concluir que


Universidad de Castilla- La Mancha
Ciudad Real - Espaa

Draft

Por: Eduardo W. V. Chaves (2014)

1 TENSORES

65

r
r
r r
r
( A 2 f ) I A ( A f ) + II A f f [cof( A )] = 0
r
r
r
r r
T
A 2 f I A A f + II A 1 f [cof( A )] f = 0
r r
T
A 2 I A A + II A 1 [cof( A )] f = 0
r
T
A 2 I A A + II A 1 [cof( A )] = 0

Utilizando al definicin A A 1 = [cof(A )]T , la ecuacin anterior queda


r
T
A 2 I A A + II A 1 [cof( A )] = 0
r
A 2 I A A + II A 1 A A 1 = 0

r
A 2 A I A A A + II A 1 A A A 1 A = 0 A
r
A 3 I A A 2 + II A A A 1 = 0

Ejemplo 1.79
Partiendo del teorema de Cayley-Hamilton obtener la inversa de un tensor T en funcin de
potencia de tensores.
Solucin:
El teorema de Cayley-Hamilton afirma que:
T 3 T 2 I T + T II T III T 1 = 0

Haciendo el producto escalar de la expresin anterior por el tensor T 1 obtenemos que:


T 3 T 1 T 2 T 1 I T + T T 1 II T III T 1 T 1 = 0 T 1
T 2 TI T + 1 II T III T T 1 = 0
T 1 =

1
T 2 TI T + 1 II T
III T

Ejemplo 1.80
Dado el tensor T representado por sus componentes en el sistema cartesiano:
5 0 0
T = 0 2 0
0 0 1

Comprobar el teorema de Cayley-Hamilton.


Solucin:
El teorema de Cayley-Hamilton tambin se aplica para las componentes del tensor:
T 3 T 2 I T + T II T III T 1 = 0

donde I T = 5 + 2 + 1 = 8 ; II T = 10 + 2 + 5 = 17 ; III T = 10 , luego:


T

5 3

=0
0

0
23
0

0 125 0 0

0 = 0 8 0
1 0 0 1

Universidad de Castilla- La Mancha


Ciudad Real - Espaa

Draft

; T

5 2

=0
0

0
22
0

0 25 0 0

0 = 0 4 0
1 0 0 1

Por: Eduardo W. V. Chaves (2014)

MECNICA DEL MEDIO CONTINUO: PROBLEMAS RESUELTOS

66

Aplicando el teorema de Cayley-Hamilton verificamos que:


125 0 0
25 0 0
5 0 0
1 0 0
0 8 0 8 0 4 0 + 17 0 2 0 10 0 1 0

0 0 1
0 0 1
0 0 1
0 0 1
1444444444442444444444443
0 0 0
0 0 0

0 0 0

0 0 0
= 0 0 0
0 0 0
0 0 0
= 0 0 0
0 0 0
Q.E.D.

Ejemplo 1.81
Dada una matriz P representada por sus componentes por Pij (i, j = 1,2,3,4) . a) Obtener la
inversa, b) los invariantes, y c) la ecuacin caracterstica. Aplicar los apartados para la matriz
P:
2 3 1
2 1 2
.
1 5 3

1 2 4

1
2
P=
4

1
0
1=
0

Considerar que

0 0 0
1 0 0
0 1 0

0 0 1

Solucin:
Aplicando el Teorema de Cayley-Hamilton se cumple que:
P 4 + P 3 I1 + P 2 I 2 + P I 3 + I 41 = 0

(
P (P (P

P P 3 + P 2 I1 + P I 2 + 1 I 3 + I 41 = 0
2

+ P I1 + 1 I 2 + 1 I 3 + I 41 = 0

P P P P + 1 I 1 + 1 I 2 + 1 I 3 + I 41 = 0
1442443


C1

P P C1 + 1 I 2 + 1 I 3 + I 4 1 = 0
44
3
1442

C2

P C2 + 1 I 3 + I 4 1 = 0

C3 + I 4 1 = 0

donde hemos denotado:

C0 = P

(
= P (C
= P (C

)
+1 I )
+1 I )

C1 = P C0 + 1 I 1

C2

C3

1
2

Podemos sacar la traza de C3 + I 41 = 0 , resultando que:

Universidad de Castilla- La Mancha


Ciudad Real - Espaa

Draft

Por: Eduardo W. V. Chaves (2014)

1 TENSORES

67

Tr (C3 + I 4 1) = Tr (0 )

Tr (C3 ) + Tr ( I 4 1) = Tr (C3 ) + I 4 Tr (1) = Tr (C3 ) + 4 I 4 = 0


I4 =

Tr (C3 )
4

Anlogamente podemos definir:


I3 =

Tr (C2 )
3

I2 =

Tr (C1 )
2

I1 =

Tr (C0 )
1

Con eso obtenemos que:


I1 =

Tr (C0 )
= (1 + 2 + 5 + 4) = 12
1

con eso podemos definir la matriz C1 = P C0 + 1 I 1 :


1
2
C1 =
4

I2 =

2 3 1 1

2 1 2 2
1 5 3 4

1 2 4 3

2 3 1
1
0

2 1 2
12
0
1 5 3

1 2 4
0

Tr (C1 ) (8 13 16 21) (42)


=
=
= 21
2
2
2

A su vez obtenemos C2 = P C1 + 1 I 2
1
2
C2 =
4

I3 =

2 3 1 8
6
14 14
1

2 1 2 8 13
5
7

+
21
0
1 5 3 13
6
16 3


1 2 4 11
2
4
21
0

0 0 0 37 22
15 17

2 5 5
1 0 0 7
=
0 1 0 10 12 14
2


14 11
0 0 1 9
5

Tr (C2 ) ( 37 2 14 + 5) ( 48)
=
=
= 16
3
3
3

A su vez obtenemos C3 = P C2 + 1 I 3
1
2
C3 =
4

I4 =

14 14
0 0 0 8
6

7
1 0 0 8 13
5
=
16 3
0 1 0 13
6


21
0 0 1 11
2
4

2 3 1 37 22
15 17
1

2 1 2 7
2 5 5
+ 16
0
1 5 3 10 12 14
2


1 2 4 9
5
14 11
0

0 0 0 32 0 0 0

1 0 0 0 32 0 0
=
0 1 0 0 0 32 0

0 0 1 0 0 0 32

Tr (C3 ) 4(32)
=
= 32 = det (P )
4
4

Luego, la ecuacin caracterstica queda:


P 4 + P 3 I1 + P 2 I 2 + P I 3 + I 41 = 0

P 4 12P 3 + 21P 2 + 16P 321 = 0

Los coeficientes de la ecuacin caracterstica podran haber sido obtenidos al resolver:


det ( P 1) P 1 = 0

c) La inversa puede ser obtenida partiendo de una de las relaciones obtenida anteriormente:

Universidad de Castilla- La Mancha


Ciudad Real - Espaa

Draft

Por: Eduardo W. V. Chaves (2014)

MECNICA DEL MEDIO CONTINUO: PROBLEMAS RESUELTOS

68

P (C2 + 1 I 3 ) + I 4 1 = 0

P 1 P C2 + 1 I 3 + I 4 P 11 = 0
P 1 =

1
C2 + 1 I 3
I4

P 1 =

Luego:

P 1

C2 + 1 I 3 + I 4 P 1 = 0

1
adj[P ]
det (P )

37 22
15 17
1

2 5 5
1 7
+ 16
=

0
2
(32) 10 12 14

9
14 11 5
0

adj[P ] = C2 + 1 I 3

0 0 0
15 17
21 22

1 0 0 1 7
14
5 5
=
0 1 0 32 10 12 2
2

0 0 1
14 11 21
9

NOTA 1: Este procedimiento que acabamos de realizar, en la literatura, se conoce como


Mtodo de Faddeev-Leverrier.

Notar que la inversa tambin puede ser obtenida a travs del procedimiento utilizado en la
ecuacin (1.46), i.e.:

1
32

2
4
3

0
2
4
3

0
2
4
3

0
2
4
3

0 0 0
2 1 2
1 5 3
1 2 4
1 0 0
2 1 2
1 5 3
1 2 4
0 1 0
2 1 2
1 5 3
1 2 4
0 0 1
2 1 2
1 5 3
1 2 4

1
4
3

1
0
4
3

1
0
4
3

1
0
4
3

2 3 1
0 0 0
1 5 3
1 2 4
2 3 1
1 0 0
1 5 3
1 2 4
2 3 1
0 1 0
1 5 3
1 2 4
2 3 1
0 0 1
1 5 3
1 2 4

Universidad de Castilla- La Mancha


Ciudad Real - Espaa

2
1
3

1
2
0
3

1
2
0
3

1
2
0
3

2 3 1
2 1 2
0 0 0
1 2 4
2 3 1
2 1 2
1 0 0
1 2 4
2 3 1
2 1 2
0 1 0
1 2 4
2 3 1
2 1 2
0 0 1
1 2 4

Draft

2
4
1

1
2
4
0

1
2
4
0

1
2
4
0

2 3 1
2 1 2
1 5 3
0 0 0
2 3 1
2 1 2
1 5 3
1 0 0
2 3 1
2 1 2
1 5 3
0 1 0
2 3 1
2 1 2
1 5 3
0 0 1

= 1
32

21
7
10
9

22 15 17

14

12

14

11 21

Por: Eduardo W. V. Chaves (2014)

1 TENSORES

69

NOTA 2: Tambin podemos obtener los coeficientes del polinomio caracterstico a travs del
siguiente procedimiento. Considerando P 4 P 3 I 1 + P 2 I 2 P I 3 + I 4 1 = 0
El ltimo coeficiente, en el caso I 4 = det(P ) = 32 .

El coeficiente I 3 se obtiene por la suma de los determinantes de las matrices resultantes al


eliminar 1 fila y 1 columna asociados a la diagonal principal, i.e.:
1
2
I3 =
4

2 3 1 1
2 1 2 2
+
1 5 3 4

1 2 4 3

2 3 1 1
2 1 2 2
+
1 5 3 4

1 2 4 3

2 3 1 1
2 1 2 2
+
1 5 3 4

1 2 4 3

2 3 1
2 1 2
1 5 3

1 2 4

2 1 2 1 3 1 1 2 1 1 2 3
= 1 5 3 + 4 5 3 + 2 2 2 + 2 2 1 = 16
1 2 4 3 2 4 3 1 4 4 2 5

El coeficiente I 2 se obtiene por la suma de los determinantes de las matrices resultantes al


eliminar 2 filas y 2 columnas asociados a la diagonal principal, i.e.:
1
2
I2 =
4

2 3 1
2 1 2
+
1 5 3

1 2 4

1
2

2 3 1
2 1 2
+
1 5 3

1 2 4

1
2
+
4

2 3 1 1 2
2 1 2 2 2
+
1 5 3 4 1

1 2 4 3 1

1
2
+
4

2 3 1
2 1 2
1 5 3

1 2 4

1
2

2 3 1
2 1 2
+
1 5 3

1 2 4

3 1
1 2
+
5 3

2 4

5 3 2 2 2 1 1 1 1 3 1 2
=
= 21
+
+
+
+
+
2 4 1 4 1 5 3 4 4 5 2 2

El coeficiente I 1 se obtiene por la suma de los determinantes de las matrices resultantes al


eliminar 3 filas y 3 columnas asociados a la diagonal principal, i.e.:
1
2
I1 =
4

2 3 1 1
2 1 2 2
+
1 5 3 4

1 2 4 3

2 3 1 1
2 1 2 2
+
1 5 3 4

1 2 4 3

= [4] + [1] + [2] + 5 = 12 = Tr (P )

Universidad de Castilla- La Mancha


Ciudad Real - Espaa

2 3 1 1
2 1 2 2
+
1 5 3 4

1 2 4 3

Draft

2 3 1
2 1 2
1 5 3

1 2 4

Por: Eduardo W. V. Chaves (2014)

MECNICA DEL MEDIO CONTINUO: PROBLEMAS RESUELTOS

70

Ejemplo 1.82
Demostrar que si A es un tensor de segundo orden se cumple que:
a) II A =

1
[Tr( A )]2 Tr( A 2 )
2

b) det ( A ) =

1
[Tr (A )]3 + 2 Tr( A 3 ) 3Tr(A ) Tr(A 2 )
6

Solucin:

a) Fue demostrado en el Ejemplo 1.79 que III A A 1 = (A 2 AI A + 1 II A ) , luego, aplicando el


doble producto escalar con el tensor identidad, obtenemos que:

III A A 1 : 1 = A 2 AI A + 1 II A : 1 = A 2 : 1 A : 1 I A + 1 : 1 II A
1

III A Tr ( A ) = Tr ( A ) Tr ( A ) I A + Tr (1) II A = Tr ( A 2 ) ( I A ) 2 + 3 II A

Teniendo en cuenta la inversa de un tensor A 1 =

[cof ( A )]T
III A Tr ( A 1 ) = Tr ( III A A 1 ) = Tr III A

III A

[cof ( A )]T , podemos decir que:


III A

= Tr [cof ( A )]T = Tr ([cof ( A ) ]) = II A

Con lo cual, podemos decir que:


III A Tr ( A 1 ) = II A = Tr ( A 2 ) ( I A ) 2 + 3 II A
II A 3 II A = Tr ( A 2 ) ( I A ) 2
II A =

1
( I A ) 2 Tr ( A 2 )
2

b) Partiendo del teorema de Cayley-Hamilton, que afirma que todo tensor cumple su propia
ecuacin caracterstica:
A 3 A 2 I A + AII A III A 1 = 0

donde I A = [Tr (A )] , II A =

1
[Tr( A )]2 Tr( A 2 ) , III
2

(1.70)

= det (A ) son los invariantes principales

del tensor A . Haciendo el doble producto escalar con el tensor identidad de segundo orden
( 1 ) en la expresin (1.70) obtenemos que:
A 3 : 1 A 2 : 1 I A + A : 1II A III A 1 : 1 = 0 : 1
Tr ( A 3 ) Tr ( A 2 ) I A + Tr ( A ) II A III

[Tr(1)] = [Tr(0)]

1
[Tr(A )]2 Tr(A 2 ) III A 3 = 0
2
1
1
3
Tr ( A 3 ) Tr ( A 2 ) Tr ( A ) + [Tr ( A )] Tr ( A ) Tr ( A 2 ) III A 3 = 0
2
2
1
3
2 Tr ( A 3 ) 3 Tr ( A 2 ) Tr ( A ) + [Tr ( A )] III A 3 = 0
2
Tr ( A 3 ) Tr ( A 2 ) Tr ( A ) + Tr ( A )

Con lo cual obtenemos que:


III

= det ( A ) =

1
[Tr( A )]3 + 2 Tr( A 3 ) 3Tr (A 2 ) Tr( A )
6

o en notacin indicial:

Universidad de Castilla- La Mancha


Ciudad Real - Espaa

Draft

Por: Eduardo W. V. Chaves (2014)

1 TENSORES

III

= det ( A ) =

71

1
A ii A jj A kk + 2A ij A jk A ki 3A ij A ji A kk
6

NOTA: Es interesante destacar que los invariantes principales del tensor A vienen formados
por la combinacin de los tres invariantes fundamentales de un tensor de segundo orden, a saber,
( Tr (A ) , Tr ( A 2 ) y Tr ( A 3 ) ), es decir:
I A = Tr ( A )

1
[ Tr ( A )] 2 Tr ( A 2 )
2
1
=
[Tr ( A )]3 + 2 Tr ( A 3 ) 3 Tr ( A 2 ) Tr ( A )
6

II A =

III A

Ejemplo 1.83

principal de T , III

1
[Tr( T )]2 Tr(T 2 ) es el segundo invariante
2
es el tercer invariante principal, es decir, el determinante de T .

Demostrar que II T = III T Tr ( T 1 ) , donde II T =

Solucin: Fue demostrado en el Ejemplo 1.79 que T 1 =

1
T 2 TI T + 1 II T , luego,
III T

aplicando el doble producto escalar con el tensor identidad, obtenemos que:

1
1
T 2 TI T + 1 II T : 1 =
T 2 : 1 T : 1 I T + 1 : 1 II T
III T
III T

T 1 : 1 =
Tr ( T 1 ) =

1
Tr ( T 2 ) Tr ( T ) I T + Tr (1) II T
III T

III T Tr ( T 1 ) = Tr ( T 2 ) I T2 + 3 II T
14243
= 2 II T

III T Tr ( T ) = II T

Ejemplo 1.84
Demostrar que:
r r
1
(1 + c b) 1 = 1

(c b )
r r
( + c b)

(1.71)

donde c , b son vectores, 1 es el tensor identidad de segundo orden, y y son escalares.


Solucin:

Haciendo que T = (1 + c b) , y teniendo en cuenta la expresin de la inversa obtenida en


el Ejemplo 1.79:
T 1 =

1
T 2 TI T + 1 II T
III T

(1.72)

A continuacin obtenemos T 2 :

Universidad de Castilla- La Mancha


Ciudad Real - Espaa

Draft

Por: Eduardo W. V. Chaves (2014)

MECNICA DEL MEDIO CONTINUO: PROBLEMAS RESUELTOS

72

r r
r r
T 2 = T T = (1 + c b) (1 + c b)
r r
r r
r r r r
= 2 1 1 + 1 (c b) + (c b) 1 + 2 (c b) (c b)
r r r r
r r r r
donde se cumple que (c b) (c b) = (c b)(c b) , ver Ejemplo 1.20. Luego, la expresin

anterior puede ser rescrita como:

r r
r r r r
T 2 = 2 1 + 2 (c b) + 2 (c b)(c b)

y la traza viene dada por:

r r
r r r r
Tr ( T 2 ) = Tr 2 1 + 2 (c b) + 2 (c b)(c b)
r r
r r
r r
= 2 Tr (1) + 2Tr (c b) + 2 (c b) Tr (c b)
r r
r r r r
= 3 2 + 2 (c b) + 2 (c b)(c b)
r r
r r
= 3 2 + 2 (c b) + 2 (c b) 2

A continuacin calculamos los invariantes principales de T

r r
r r
r r
I T = Tr (1 + c b) = Tr (1) + Tr (c b) = 3 + (c b)
r r 2
r r
r r
( I T ) 2 = 3 + (c b) = 9 2 + 6 (c b) + 2 (c b) 2

} {

r r
r r
r r
r r
1
1
2
I T Tr ( T 2 ) = 9 2 + 6 (c b) + 2 (c b) 2 3 2 + 2 (c b) + 2 (c b) 2
2
2
r r
2
= 3 + 2 (c b)
r r
r r
III T = det (1 + c b) = 3 + 2 c b (ver Ejemplo 1.50)

II T =

]}

Luego, la expresin (1.72) queda:


III T T 1 = T 2 I T T + II T 1
r r
r r r r
= 2 1 + 2 (c b) + 2 (c b)(c b)
r r
r r
r r
3 + (c b) (1 + c b) + 3 2 + 2 (c b) 1
r r
r r r r
r r
r r
= 2 1 + 2 (c b) + 2 (c b)(c b) 3 2 1 3 (c b) (c b)1
r r r r
r r
2 (c b)(c b) + 3 2 1 + 2 (c b)1
r r
r r
= 1 2 + (c b)1 (c b)
r r
r r
= ( 2 + c b)1 (c b)
r r
r r
1
T
= ( 3 + 2 c b)1 (c b) = [adj( T ) ] = [cof ( T ) ]

(1.73)

r r

Teniendo en cuenta que T = (1 + c b) , III T = 3 + 2 c b , la expresin anterior


queda:
r r
1 III T
1 (c b)

III T

r r
r r
(c b) 1
(c b)
1 III T
=
1
= 1
r r
III T
III T

( 3 + 2 c b )

T 1 =

(1.74)

o aun:

Universidad de Castilla- La Mancha


Ciudad Real - Espaa

Draft

Por: Eduardo W. V. Chaves (2014)

1 TENSORES

r r
1
(1 + c b) 1 = 1

( ij + c i b j ) 1 =

[[1] + [{c}{b} ] ]
T

[ 1]

ij

73

r r

r r (c b)
( + c b)

Notacin Tensorial

(1.75)

(c b )
( + c k b k ) i j

Notacin Indicial

(1.76)

Notacin matricial

(1.77)

[
{c}{b}T ]
T
( + {c} {b})

NOTA 1: La expresin anterior tambin es vlida para matrices de n-dimensiones.


Para el caso particular cuando = 1 , = 1 , obtenemos que:
r r
r r 1
(c b)
(1 + c b) = 1
r r
1+ c b

(1.78)

NOTA 2: La expresin (1.75) puede ser reescrita como:


r r
1
T 1 = (1 + c b) 1 = 1

r r

r r (c b)
( + c b )

r r
r r
1
[adj( T )]
r ( 2 + c b)1 (c b) =
r
3
2
det ( T )
( + c b )
r r
r r
r r
con lo cual concluimos que adj(1 + c b) = ( 2 + c b)1 (c b) .
=

NOTA 3: Podemos extender la ecuacin (1.75) de tal forma que:


( I sym + A B ) 1 =

I sym

( A B)
( + A : B )

donde ahora tenemos que I sym es el tensor identidad de cuarto orden, A y B son tensores de
segundo orden, y y son escalares. Con eso es de fcil demostracin que ( I sym ) 1 = I sym .
Ejemplo 1.85
r

Teniendo en cuenta que ( 1 + c b) 1 =


r r
1
( A + a b) 1 = A 1

r r

r r (c b) , demostrar que:
( + c b)

r
r
( A 1 a) (b A 1 )
r
r
( + b A 1 a)

(1.79)

donde a , b son vectores, A es un tensor de segundo orden, con det ( A ) 0 ( A 1 ), y ,


son escalares.
Solucin:
r

Observemos que el trmino (A + a b) , puede ser reescrito como:


r r
r r
(A + a b) = A (1 + A 1 a b)

Utilizando la propiedad de la inversa tal que ( A B ) 1 = B 1 A 1 , podemos decir que:

r r
r r 1
r r
(A + a b) 1 = A (1 + A 1 a b) = (1 + A 1 a b) 1 A 1

Universidad de Castilla- La Mancha


Ciudad Real - Espaa

Draft

Por: Eduardo W. V. Chaves (2014)

MECNICA DEL MEDIO CONTINUO: PROBLEMAS RESUELTOS

74

Observemos que el resultado de la operacin A 1 a resulta un vector y lo denotamos por el


r
r
vector c = A 1 a , con lo cual podemos reescribir la expresin anterior como:
r r
r r
(A + a b) 1 = (1 + A 1 a b) 1 A 1
r r
= (1 + c b) 1 A 1

1
r r

1
= 1
r r (c b) A

( + c b )

r r

1
1
= 1 A 1
r r (c b) A

( + c b )
r r

1
1
= A 1
r r c bA

( + c b )
r

1
1 r
1
= A 1
r r ( A a) (b A )

( + c b )
r
r
1

r
= A 1
( A 1 a) (b A 1 )
r

( + b A 1 a)

La expresin anterior en notacin indicial queda


(A ij + a i b j ) 1 =

A ij1

( +

b p A pq1 a q )

( A ik1 a k )(b s A sj1 )

r r

1
Cuidado con la operacin ( A 1 a) b A
(a b) , esta ltima no tiene consistencia, ya
14243

Expresin errnea

r r

que no podemos tener un producto escalar (contraccin) con un escalar (a b) . En notacin


r r

indicial se puede comprobar c b = c i b i = ( A 1 a) i b i = A ik1 a k b i , luego, las expresiones


r r
posibles son ( A 1 a) b = b i A ik1a k = a k A ik1b i = A ik1b i a k = A ik1 a k b i = A ik1a k b i .
1
4
3
r 24
r
bA 1a

1
424
3
r
r
aA T b

1
42r4
3
r
A 1:(ba)

1
424
3
r r
A T :( ab )

1
424
3
r r
A 1:( ab )T

Para el caso particular cuando = 1 , = 1 , recaemos en la frmula de Sherman-Morrison:


r
r
r r 1
( A 1 a) (b A 1 )
1
r
( A + a b) = A
r
1 + b A 1 a

Frmula de Sherman-Morrison

(1.80)

(notacin tensorial)

En notacin matricial la ecuacin anterior queda:

[[ A] + [{a}{b} ] ]
T

= [ A] 1

}{

[ A] 1 {a} {b}T [ A] 1

1 + {b}T [ A] 1 {a}

}
T

Frmula de Sherman-Morrison

(1.81)

(notacin matricial)

NOTA 1: Observar que si (A + a b) = A (1 + A 1 a b) , el determinante viene


definido por:

r r
r r
r r
det (A + a b) = det A (1 + A 1 a b) = det [A ]det (1 + A 1 a b)
r
r
= det [A ]( 3 + 2 b A 1 a)

con lo cual, la expresin (1.79) puede aun ser reescrita como:

Universidad de Castilla- La Mancha


Ciudad Real - Espaa

Draft

Por: Eduardo W. V. Chaves (2014)

1 TENSORES

{ A (

r r
1
( A + a b) 1 =

75

r
r
r
r
+ b A 1 a) A 1 A ( A 1 a) (b A 1 )

]}

con = det (A + a b) = A ( 3 + 2 b A 1 a) .

(1.82)

con eso concluimos que:

r
r
r r
r
r
adj(A + a b ) = A ( 2 + b A 1 a) A 1 A ( A 1 a) (b A 1 )

]}

NOTA 2: Podemos extrapolar la expresin (1.79) de tal forma que:


( D + A B ) 1 =

D 1

( + B : D 1 : A )

[(D

: A ) (B : D 1 )

(1.83)

donde ahora tenemos que D es un tensor de cuarto orden, A y B son tensores de segundo
orden, y , son escalares. Aun podemos decir que:
( D + A B ) 1 =

{ D (

+ B : D 1 : A )D 1 D (D 1 : A ) (B : D 1 )

]}

con = det (D + A B ) = D ( 3 + 2 B : D 1 : A ) .

(1.84)

donde podemos concluir que:


det (D + A B ) = det (D )( 3 + 2 B : D 1 : A )

adj(D + A B ) = D ( 2 + B : D 1 : A )D 1 D (D 1 : A ) (B : D 1 )

Ejemplo 1.86

(1.85)

]}

(1.86)

a) Dado un tensor de segundo orden C = (1 + a b + c d) , demostrar que:


r

r r

[r

r r

r r r

r r r r

1 + a b + c d = 3 + 2 (c d) + 2 (a b) + (a b)(c d) (a d)(b c )

(1.87)
r

donde 1 + a b + c d = det (1 + a b + c d) representa el determinante del


r

tensor C . b) Para el caso particular cuando = 1 , d = a , c = b , demostrar que:


r r
r r
r r
r r
det (1 + a b + b a) = 1 + ( + )(a b) a b

Solucin:
r

(1.88)

Definimos un tensor auxiliar D = 1 + a b y a su vez tenemos que C = (D + c d) .


r

Segn el Ejemplo 1.85, ecuacin (1.82), se cumple que det (D + c d) = D (1 + d D 1 c ) ,


donde:
r r
r r
det (D) D = det (1 + a b ) = 3 + 2 (a b) y
r r
1
(D ) 1 = (1 + a b) 1 = 1

Universidad de Castilla- La Mancha


Ciudad Real - Espaa

r r

r r (a b)
( + a b )

Draft

Por: Eduardo W. V. Chaves (2014)

MECNICA DEL MEDIO CONTINUO: PROBLEMAS RESUELTOS

76

Con eso, podemos decir que:

r
r r
r
det (D + c d) = D (1 + d D 1 c )

r 1
r r
r r r

= 3 + 2 (a b ) (1 + d 1
r (a b) c
r

( + a b )


r r r r
1 r
r r
r

r d (a b) c
= 3 + 2 (a b ) 1 + d 1 c
r

( + a b )

r r
r r
1 r r
r r

r (d a) (b c )
= 3 + 2 (a b ) 1 + d c
r

( + a b )

r r
r r r r
r r

r (a d)(b c )
= 3 + 2 (a b ) 1 + (c d)
r
2 ( + a b )

r r
r r
r
r
r
r
r
r
r
r
Notar que (d a) (b c ) = (a d) (b c ) = (a d)(b c ) .
123 123
escalar

escalar

[
[

r r
r r
r r
r r r r
r r

r (a d)(b c )
det (D + c d) = 3 + 2 (a b) 1 + (c d) 3 + 2 (a b)
r
2

( + a b)

r
r
r
r
r
r
r
r

= 3 + 2 (a b) 1 + (c d) (a d)(b c )

r r
r r
r r r r
r r r r
3
2
2
= + (c d) + (a b) + (a b)(c d) (a d)(b c )

Luego:

r r
r r
r r
r r
r r r r
r r r r
det (1 + a b + c d) = 3 + 2 (c d) + 2 (a b ) + (a b )( c d) (a d)(b c )

Con eso demostramos la ecuacin (1.87).


r

Para el caso particular cuando d = a , c = b , tenemos que:

[
[

]
]

r r
r r
r r
r r
r r r r
r r r r
det (1 + a b + c d) = 3 + 2 (c d) + 2 (a b) (a d)(b c ) (a b)(c d)
r r
r r
r r
r r
r r r r
r r r r
det (1 + a b + b a) = 3 + 2 (b a) + 2 (a b) (a a)(b b) (a b)(b a)
r r
r r r r
r r r r
= 3 + 2 ( + )(a b) (a a)(b b) (a b)(a b)
r r 2 r 2 r 2 r r 2
En el Ejemplo 1.1 hemos demostrado que a b = a b a b . Con eso

( )

demostramos que:
r r
r r
r r
r r
det (1 + a b + b a) = 3 + 2 ( + )(a b) a b

Para el caso particular cuando = 1 , obtenemos que:


r r
r r
r r
r r
det (1 + a b + b a) = 1 + ( + )(a b) a b

Universidad de Castilla- La Mancha


Ciudad Real - Espaa

Draft

Por: Eduardo W. V. Chaves (2014)

1 TENSORES

77

Ejemplo 1.87

a) Obtener la inversa del tensor de segundo orden C = (1 + a b + c d) .


r r
p p
pq

b.1) Dado el tensor D = B + r r

r
r
(B q) (B q)
donde B = B T y B 1 , demostrar que
r
r
q B q

se cumple que:
D 1 = B 1 +

r r r
r
( p q + p B 1 p ) r r
[q q ] r 2 r qr ( B 1 pr )
r r 2
( p q)
( p q)

sym

(1.89)

b.2) Si B es un tensor definido positivo, determinar las condiciones para que D sea un tensor
no-singular.
Solucin:
r

a) Denotando por A = (1 + a b) quedamos con C = ( A + c d) , y teniendo en cuenta


que
r r
1
( A + a b) 1 = A 1

r
r
( A 1 a) (b A 1 )
r
r
( + b A 1 a)

(1.90)

Ver Ejemplo 1.85, ecuacin (1.79). Luego


r r
( A + c d) 1 = A 1

r
r

( A 1 c ) (d A 1 )
r
r
(1 + d A 1 c )

(1.91)

En el Ejemplo 1.84 hemos demostrado que:


r r
1
(1 + c b) 1 = 1

Con eso concluimos que:


A

r r
1
= (1 + a b) 1 = 1

(c b )
r r
( + c b)

(1.92)

(a b)
r r
( + a b )

Adems tenemos que


r r
r r
r r
r 1
r 1
r
r 1 r

(
)
(

)
(
c) r
b

a
b
a
b
A c = 1
r r c = 1 c
r r c = c
r r a

( + a b )
( + a b )
( + a b )

r r
r r
r r
r
r 1
r
r
1 r
a) r
(
)
(

)
1
(
a
b
a
b
d

1
d A = d 1
r = d 1 d
r = d
r r b
r
r

( + a b)
( + a b )
( + a b )

Con eso concluimos que

] [

r r r
r
r r
r r
r r
r
r r r
(1 + a b + c d) 1 = (1) 1 + ( 2 ) (a b) + ( 3) (1) c + ( 2 ) (b c )a (1) d + ( 2 ) (a d)b

(1.93)
donde

(1) =

Universidad de Castilla- La Mancha


Ciudad Real - Espaa

Draft

Por: Eduardo W. V. Chaves (2014)

MECNICA DEL MEDIO CONTINUO: PROBLEMAS RESUELTOS

78

( 2) =
( 3) =

r r

( + a b )

r
r
(1 + d A 1 c )

r
r 1 r r
d A 1 c = (d c )

r r r r

r r (d a )(b c )
( + a b )

b.1) Podemos reescribir el tensor D como:


r r
r
r
r r
r
r
p p
p p
(B q) (B q)
(B q) (B q)
1
1
B
B
B
B
B
(
)
(
)

D = B 1 + 1 r r 1
1
=
+

r
r
r
r
r r
pq
q B q
pq
q Bq
r r
r
r
r
r
r
r

p p
( B 1 p) p ( B 1 B q ) ( B q )
1
1 ( B q ) ( B q )
= B 1 + B r r B

r
r
r r
r
r

= B 1 +
pq
q B q
pq
q Bq

r r
r
1 r

( B p) p q ( B q )
= B 1 +
r
r r
r
pq
q Bq

Denotando por
r
r
a = ( B 1 p )

r r
b=p

r r
c =q

r
r
d = (B q)

= r r
pq

1
= r
r
q Bq

r r
r r
D = B 1 + a b + c d = B C

D 1 = ( B C ) 1 = C 1 B 1
r r
r r
donde C = [1 + a b + c d ] . La inversa del tensor C puede ser obtenido a travs del
apartada (a) con = 1 . Adems tenemos que:

(1) = 1 ,
( 2) =

1
1
1

= r r r
r
r r =
r r = r r
( + a b) (1 + a b) p q (1 + r 1 r ( B 1 pr ) pr ) ( p q + p B 1 p)
pq

r
r 1 r r
d A 1 c = (d c )

r r r r

r r (a d )(b c )
( + a b )

r r
r
r r r
1
= (( B q ) q ) + r r r
(( B 1 p) ( B q ) )( p q )
1 r
( p q + p B p)
r
r r r
r
r ( p ( B T B ) q ) ( p q )
= q B q +
r r r
r
( p q + p B 1 p)

1
1
r
= r
r
r
r r r
1 r
q
B
q

r ( p ( B T B ) q ) ( p q )
1 r
(1 + d A c )
1 + r

r q Bq +
r r r
1 r

q
B
q

(
p
q
p
B
p
)

r r r
1 r
( p q + p B p)
= r
r r r
( p ( B T B ) q ) ( p q )
r r r
r
( p q + p B 1 p )
1
1
( 2 ) (3) = r r r 1 r r
r r r = r
r r r
T
T
( p q + p B p) ( p ( B B ) q ) ( p q ) ( p ( B B ) q ) ( p q )

( 3) =

Universidad de Castilla- La Mancha


Ciudad Real - Espaa

Draft

Por: Eduardo W. V. Chaves (2014)

1 TENSORES

r r

( 2 ) ( 3) (a d ) = r

( p (B

= r
( p ( B T
r r

( 2 ) ( 3) (b c ) = r

( p (B

r r r r

79

r
1
1 r
r r r (( B p) ( B q ))
B) q) ( p q )
r
r
1
1
T
r r r ( p (B B) q ) = r r
( p q)
B) q) ( p q )
r r
1
1
r r r ( p q) = r
r
T
B) q ) ( p q )
( p (B B) q )

( 2 ) (3) (b c )(a d ) = r

( p (B

r r r
1
1 r
r r r (( B p) ( B q ))( p q ) = 1
B) q ) ( p q )

La expresin (1.93) queda:

[
[

] [

r r r
r
r r
r
r r r
C 1 = 1 + ( 2 ) (a b ) + ( 3) c + ( 2 ) (b c )a d + ( 2 ) (a d )b
r r r r
r r
r r
r r r r
C 1 = 1 + ( 2 ) a b + ( 3) c d + ( 3) ( 2 ) (a d ) c b + ( 3) ( 2 ) (b c ) a d +
r r r r r r
+ ( 3) (22 ) (b c )(a d ) a b
r r r r r r
r r
r r r r
C 1 = 1 + ( 2 ) + ( 3) (22 ) (b c )(a d ) a b + ( 3) c d + ( 3) ( 2 ) (a d ) c b +
r r r r
+ ( 3) ( 2 ) (b c ) a d

}[

notar que: { ( 2 ) + (3) (22 ) (b c )(a d )}= ( 2 ) {1 + (3) ( 2 ) (b c )(a d )}= ( 2 ) {1 1} = 0


r r r r

r r r r

r r r r
r r
r r r r
C 1 = 1 + ( 3) c d + ( 3) ( 2 ) (a d ) c b + ( 3) ( 2 ) (b c ) a d

r r r r
r r
r r r r
C 1 = 1 + ( 3) c d + ( 3) ( 2 ) (a d ) c b + ( 3) ( 2 ) (b c ) a d
r r r
r
r
r
( p q + p B 1 p)
1 r r
=1 + r
r r r [q ( B q ) ] + r r [q p ] + L
T
( p q)
( p (B B) q ) ( p q )
r
1
1 r
L+ r
r ( B p) ( B q )
T
( p (B B) q )

Con eso, podemos obtener que:


D 1 = C 1 B 1
r r r
r

r
r
( p q + p B 1 p )
1 r r
= 1 + r
r r r [q ( B q )] + r r [q p ] + L
T
( p q)
( p (B B) q ) ( p q )

r 1
1
1 r
L+ r
r ( B p) ( B q ) B
T
( p (B B) q )

r r r
r
1
r
r
( p q + p B p)
1 r r
1
1
= B 1 + r
r r r [q ( B q ) ] B + r r [q p ] B + L
T
( p q)
( p (B B) q ) ( p q )

r
1
1 r
1
L+ r
r ( B p) ( B q ) B
T
( p (B B) q )

notar que:

{[qr ( B qr )] B } = [qr ( B qr )] B = [q ( B qr ) ]B
r
r
= [q ( B B ) q ) ]
[( B pr ) ( B qr )] B = ( B pr ) ( B B ) qr
1

ij

ik

1
kj

1
kj

= q i ( B kp q p ) B kj1 = q i ( B kp B kj1 q p )

ij

Si ahora consideramos que el tensor B es simtrico, i.e. B = B T , quedamos con:


Universidad de Castilla- La Mancha
Ciudad Real - Espaa

Draft

Por: Eduardo W. V. Chaves (2014)

MECNICA DEL MEDIO CONTINUO: PROBLEMAS RESUELTOS

80

r r r
r
( p q + p B 1 p ) r r
[q q ] + r 1r
r r r r
( p q) ( p q)
( p q)
r r r
1 r
( p q + p B p) r r
[q q ] + r 1r
+
r r r r
( p q) ( p q)
( p q)
r r r
1 r
( p q + p B p) r r
+
[q q ] r 2 r
r r 2
( p q)
( p q)

[qr ( pr B )] + ( pr1qr ) [( B

D 1 = B 1 +
= B 1
= B 1

{[qr ( pr B )] + [( B
1

[qr ( B

p) q ]
r

p) q ]
r

p)]
r

sym

Notar que, debido la simetra de B se cumple que p B 1 = B 1 p = t y B T B = 1 .


b.2) Un tensor ser no-singular si det (D ) 0 . Utilizando la expresin obtenida anteriormente:

r r
r r
D = B 1 + a b + c d

( [

])

r r
r r
r r
r r
det (D) = det B 1 + a b + c d = det ( B )det 1 + a b + c d

Como B es un tensor definido positivo esto implica que det ( B ) > 0 . Luego, la condicin para
r r
r r
que D sea no-singular es que det [1 + a b + c d ] 0 . Utilizando el determinante
obtenido en el Ejemplo 1.86:

r r
r r
r r
r r r r
r r r r
det (1 + a b + c d ) = 3 + 2 (c d ) (a d )( b c ) (a b )( c d )
r r
r r r
r r r
r
r
r r r r r r
donde = 1 , a b = ( B 1 p ) p = p B 1 p , a d = ( B 1 p ) ( B q ) = p q , b c = p q
r r r
r
r
r
r r
r
1 r
c d = q ( B q ) = q B q , (c d ) = r
r q B q = 1 ,
q B q

[(a d )(b c ) (a b )(c d ) ] = r r r


r r r r

r r r r

r r r r
r
r
1
1 r r
r ( p q )( p q ) ( p B p )( q B q )
pq q B q
1

Con eso podemos decir que:

r r
r r
r r r r
r
r
1
1 r r
det 1 + a b + c d = r r r
r ( p q )( p q ) ( p B p )(q B q ) 0
( p q )(q B q )
r r r r
r r
r
r
Luego, las condiciones son: p 0 , q 0 , ( p q ) 0 , i.e. p y q no pueden ser vectores

ortogonales. Otra condicin que hay que cumplir es:


r r r r
r
r r
r
( p q )( p q ) ( p B 1 p )(q B q ) 0
142
4 43
4 144424443
>0

>0

Notar que por el hecho que B sea definido positivo, el trmino (q B q ) > 0 siempre ser
r r
r
r
positivo cualquier que sea q 0 . Lo mismo ocurre con ( p B 1 p ) > 0 , ya que si un tensor es
definido positivo su inversa tambin lo es. Observar tambin que para que D sea definido
r r
r
r r
r
r r
positivo ( det (D ) > 0 ) hay que cumplir ( p q ) 2 > ( p B 1 p )(q B q ) y ( p q ) > 0 . Estas dos
r r

r r

condiciones pueden ser reemplazadas por ( p q ) > ( p B 1 p )(q B q ) .


Ejemplo 1.88
Dado un tensor de segundo orden A = A () y un escalar , demostrar que se cumple que:
dA

dA
A 1
= A Tr
d
d

Universidad de Castilla- La Mancha


Ciudad Real - Espaa

Draft

(1.94)

Por: Eduardo W. V. Chaves (2014)

1 TENSORES

81

Solucin:
En el Ejemplo 1.82 y Ejemplo 1.79, hemos demostrado, respectivamente, que se cumplen:
III

= det ( A ) = A =

1
[Tr( A )]3 + 2 Tr(A 3 ) 3Tr( A 2 ) Tr(A )
6

(1.95)

III A A 1 = A 2 AI A + II A 1

donde I A = Tr (A ) , II A =

(1.96)

1
[Tr ( A )]2 Tr ( A 2 ) .
2

Notar tambin que las siguientes derivadas se cumplen:


d [I A ] d [Tr ( A )] d [A kk ] d [A ik ik ] d [A ik ]
dA
dA
=
=
=
=
ik =
: 1 = Tr
d
d
d
d
d
d
d

d (A 2 )
d Tr ( A 2 )
dA

dA
= Tr
= Tr 2A
= 2 Tr A d
d
d
d

d Tr ( A 3 )
dA

= 3Tr A 2
d
d

Tomando la derivada de (1.95) con respecto a obtenemos que:

}
[

d ( III A ) 1 d
[Tr( A )]3 + 2 Tr(A 3 ) 3Tr( A 2 ) Tr(A )
=
d
d
6
d Tr ( A 3 )
d Tr ( A 2 )
d [Tr ( A )]
1
2 d [Tr ( A )]
Tr ( A ) 3 Tr ( A 2 )
= 3[Tr ( A )]
+2
3

dt
d
d
d
6

1
dA
dA
2 dA
dA
2
Tr ( A ) 3Tr ( A 2 ) Tr
6 Tr A
3[Tr ( A )] Tr + 6 Tr A

d
d
6
d

dA
1
dA
dA

2
Tr ( A ) + [Tr ( A )] Tr ( A 2 ) Tr
Tr A
= Tr A 2

d
d
2
d

o aun
d ( III A )
dA

dA
dA
= Tr A 2
Tr A d I A + II A Tr d

d
d

Haciendo el producto escalar de la ecuacin (1.96) con


III A A 1

(1.97)

dA
, obtenemos que:
d

dA
dA
dA
dA
dA
= A 2 AI A + II A 1
= A2
A
I A + II A
d
d
d
d
d

y sacando la traza con obtenemos que:


dA
dA
dA
2 dA

Tr A 1
A
I A + II A

III A = Tr A
d
d
d
d

dA
dA

dA
= Tr A 2
Tr A
I A + Tr
II A
d
d

(1.98)

Si comparamos las ecuaciones (1.97) y (1.98) concluimos que:


d ( III A )
dA
dA

A 1
= III A Tr A 1
= III A Tr
d
d
d

Universidad de Castilla- La Mancha


Ciudad Real - Espaa

Draft

Q.E.D.

Por: Eduardo W. V. Chaves (2014)

82

MECNICA DEL MEDIO CONTINUO: PROBLEMAS RESUELTOS

1.10 Tensores Istropos y Anistropos


Ejemplo 1.89
Sea el tensor de cuarto orden C , cuyas componentes vienen dados por:
Cijkl = ij kl + ik

+ il

jl

(1.99)

jk

donde ij son las componentes del tensor identidad de segundo orden, y , , son
escalares.
a) Qu tipo de simetra presenta el tensor C ? b) Que condiciones hay que cumplir para que
C sea un tensor simtrico?
Solucin:
El tensor presenta simetra mayor si se cumple que C ijkl = C klij . Teniendo en cuenta (1.99),
concluimos que:
C klij = kl ij + ki lj + kj li = Cijkl

Verificamos ahora si el tensor presenta simetra menor, por ejemplo C ijkl = C ijlk
Cijlk = ij lk + il

jk

+ ik

jl

C ijkl

Se puede comprobar este hecho fcilmente por adoptar i = 2 , j = 1 , k = 1 , l = 2 , con eso:


Cijkl = C 2112 = 21 12 + 21 12 + 22 11 =
Cijlk = C 2121 = 21 21 + 22 11 + 21 12 =

Luego, el tensor C solo ser simtrico (simetra menor y mayor) si = , resultando:


Cijkl = ij kl + ( ik

jl

+ il

jk )

Verifiquemos que ij kl presenta simetra mayor y menor, mientras que los tensores ik jl ,
sym
il jk no son simtricos. Notar tambin que ( ik jl + il jk ) = 2I ijkl
.

Ejemplo 1.90
Sea el tensor de cuarto orden C , cuyas componentes vienen dadas por:
Cijkl = ij kl + ( ik

jl

+ il

jk )

(1.100)

donde y son constantes. Demostrar que C es un tensor istropo.


Solucin: Un tensor de cuarto orden ser istropo si se cumple que C ijkl = C ijkl , donde Cijkl
son las componentes del tensor debido a transformacin de coordenadas. Teniendo en cuenta
que la ley de transformacin de las componentes de un tensor de cuarto orden viene dada por:
C ijkl = a im a jn a kp a lq C mnpq

(1.101)

donde aij es la matriz de transformacin de base, luego:

Universidad de Castilla- La Mancha


Ciudad Real - Espaa

Draft

Por: Eduardo W. V. Chaves (2014)

1 TENSORES

(
+ (a

C ijkl = a im a jn a kp a lq mn pq + mp nq + mq np
= a im a jn a kp a lq mn pq

im a jn a kp a lq

83

)]

mp nq + a im a jn a kp a lq mq np )

= a in a jn a kq a lq + a ip a jq a kp a lq + a iq a jn a kn a lq

= ij kl + ik

jl

+ il jk

(1.102)

= C ijkl

donde hemos utilizado que a ik a jk = ij , o en notacin matricial AA T = 1 , ya que la matriz de


transformacin es una matriz ortogonal. Luego, hemos demostrado que C es istropo, i.e. las
componentes de C no cambian para cualquier cambio de base.
Ejemplo 1.91
Sea C un tensor de cuarto orden simtrico e istropo representado por:

Cijkl = ij kl + ik

jl

+ il

jk

C = 1 1 + 2 I

(notacin indicial)
(notacin tensorial)

donde , son escalares constantes, 1 es el tensor identidad de segundo orden, I es la


parte simtrica del tensor identidad de cuarto orden, es decir, I I sym .
Se pide:
a) Dado un tensor de segundo orden simtrico , obtener que viene dado por la siguiente
operacin = C : . Expresar el resultado en notacin tensorial e indicial.
b) Demostrar que y presentan los mismos autovectores (mismas direcciones
principales).
c) Si son los autovalores (valores principales) del tensor , obtener tambin los
autovalores del tensor .
Solucin: a)
Notacin tensorial:

Notacin indicial
ij = C ijkl kl

= C:

= ij kl + ik

= (1 1 + 2 I ) :

jl

+ il

)]

kl

= ij kl kl + ik jl kl + il jk kl

= 1 1
: + 2 I{
:
{

= ij kk + ij + ji

= Tr ( )1 + 2

= ij kk + 2 ijsym

Tr ( )

jk

sym

( )

= ij kk + 2 ij

donde hemos considerado la simetra del tensor = T .


b) y c) Partiendo de la definicin de autovalor y autovector del tensor :
n = n

Reemplazando el valor de obtenido anteriormente podemos decir que:

Universidad de Castilla- La Mancha


Ciudad Real - Espaa

Draft

Por: Eduardo W. V. Chaves (2014)

MECNICA DEL MEDIO CONTINUO: PROBLEMAS RESUELTOS

84

(Tr()1 + 2 ) n = n
Tr ( )1 n + 2 n = n
Tr ( )n + 2 n = n
2 n = n Tr ( )n = ( Tr ( ) )n
Tr ( )
n
n =
2

n = n

Con lo cual concluimos que y presentan los mismos autovectores (son coaxiales). Y
adems los autovalores de se pueden obtener como:
=

Tr ( )
2

Si denominamos que (1) = 1 , (2) = 2 , (3) = 3 y (1) = 1 , ( 2) = 2 , (3) = 3 . La forma


explcita de la relacin anterior viene dada por:
1 0
0
2

0 0

0
1 0
1

0 =
0 2
2
0 0
3

1 0
donde tambin se cumple que: 0 2
0 0

0
1 0 0
Tr ( )

0 1 0
0
2
0 0 1
3

0
1 0 0
1 0

0 = Tr ( ) 0 1 0 + 2 0 2
0 0 1
0 0
3

0
0
3

Ejemplo 1.92
a) Obtener la inversa del tensor de cuarto orden C = 2 I + 1 1 donde I I sym es el tensor
identidad simtrico de cuarto orden, 1 es el tensor identidad de segundo orden, y > 0 ,
son escalares.
b) Obtener el determinante de C . Adems, si consideramos que =

E
,
(1 + )(1 2 )

E
, que valores deben asumir E y para que el tensor sea definido positivo.
2(1 + )

c) Obtener la expresin inversa de = C : en funcin de > 0 , , donde y son


tensores simtricos de segundo orden.
Solucin:
a) Recurrimos a la ecuacin obtenida en (1.83):
( D + A B ) 1 =

D 1

( + B : D 1 : A )

[(D

: A ) (B : D 1 )

Haciendo que D = I , A = B = 1 , = 2 , = , obtenemos que:


C 1 = ( 2 I + 1 1) 1 =

Universidad de Castilla- La Mancha


Ciudad Real - Espaa

1 1
I
(I 1 : 1) (1 : I 1 )
2
2 ( 2 + 1 : I 1 : 1)

Draft

Por: Eduardo W. V. Chaves (2014)

1 TENSORES

85

Recordar que se cumple que I 1 = I , (I 1 : 1) = I : 1 = 1 . A continuacin obtenemos el valor


del escalar 1 : I 1 : 1 = 1 : I : 1 = 1 : 1 = Tr (1) = 3 . Tambin lo expresamos en notacin indicial:
sym
kl
1 : I 1 : 1 = 1 : I : 1 = ij I ijkl

= ij

1
( ik
2

jl

+ il

jk

) kl

1
( ij ik jl kl + ij il jk kl )
2
1
= ( jj + jj ) = 3
2
=

Resultando que:
C 1 = ( 2 I + 1 1) 1 =

I
(1 1)
2
2 ( 2 + 3 )

Verifiquemos si se cumple que C : C 1 = I sym I :


1

C : C 1 = ( 2 I + 1 1) :
I
(1 1)
2 ( 2 + 3 )
2

C : C 1 = (

2
2
2
I :I
I : (1 1) +
(1 1) : I
(1 1) : (1 1)
2
2 ( 2 + 3 )
2
2 ( 2 + 3 )

Segn el Ejemplo 1.27 se cumplen que I : I = I , I : (1 1) = (1 1) : I = 1 1 , y


(1 1) : (1 1) = 3(1 1) . Con eso obtenemos que:

3 2

2
(1 1) = I
C : C 1 = I +
+

2
(
2
3
)
2
2
(
2
3
)

+
+
1444444
424444444
3
=0

b) Podemos utilizar directamente la expresin (1.37) del Ejemplo 1.51

det I sym + A B = 3 + 2 A : B

Haciendo = 2 , = , A = B = 1 concluimos que:


det ( 2 I + 1 1) = (2 )3 + (2 ) 2 1 : 1 = (2 )3 + (2 ) 2 3 = (2 ) 2 ( 2 + 3)

Para que C sea positivo definido, los autovalores tienen que ser positivos, i.e.:
E
>0
2(1 + )
E
E
E
2 + 3 > 0 2
+3
=
>0
2(1 + )
(1 + )(1 2 ) (1 2 )

>0 =

Denotando por y1 = (1 + ) 0 , y2 = (1 2 ) 0 , podemos concluir que:

Universidad de Castilla- La Mancha


Ciudad Real - Espaa

Draft

Por: Eduardo W. V. Chaves (2014)

MECNICA DEL MEDIO CONTINUO: PROBLEMAS RESUELTOS

86

E
E
=
>0
2(1 + ) 2 y1

2 + 3 =

E > 0

y1 > 0

E < 0
y < 0
1

E
E
=
>0
(1 2 ) y2

E > 0

y 2 > 0

E < 0
y < 0
2

Las condiciones anteriores tienen que cumplir simultneamente. Luego, a travs de la grfica
abajo, podemos concluir que E > 0 y 1 < < 0,5 .
y ( )
y2 = (1 2 ) 0

zona no factible
1

1
( y 2 > 0 E > 0)
( y 1 < 0 E < 0)

E >0

0,5

zona no factible

y1 = (1 + ) 0

( y 1 > 0 E > 0)
( y 2 < 0 E < 0)

c)
= C:

C 1 : = C 1 : C :

C 1 : = I sym : = sym =

= C 1 :
1

=
I
1 1 : =
I:
1 1 :
2 ( 2 + 3 )
2
2 ( 2 + 3 )
2

Tr ( )1
2
2 ( 2 + 3 )

Ver Figura 1.4. Es interesante comparar esta ltima expresin con el Ejemplo 1.41.

Universidad de Castilla- La Mancha


Ciudad Real - Espaa

Draft

Por: Eduardo W. V. Chaves (2014)

1 TENSORES

x3

33

13

23

x1

x3

22
12

21

33

13

32

31

11

87

= C:

23

32
22

31

x2

11

12

21

x2

x1

= C 1 :

C 1

Figura 1.4
Ejemplo 1.93
) un tensor de segundo orden, denominado de tensor acstico elstico, y definido
Sea Q e (N
como:
) =N
Ce N

Q e (N

donde C e es un tensor de cuarto orden simtrico e istropo dado por C e = (1 1) + 2 I ,


e
cuyas componentes son: C ijkl
= ij kl + ( ik jl + il
tensor acstico en funcin de las constantes , .

jk

) . Obtener las componentes del

Solucin:
Utilizando notacin simblica obtenemos que:

( ) (C

) =N
Ce N
= N e
Q e (N
i i

pqrs e p

e q e r e s

) (N e )

C e N e e
= N i C epqrs N j ip sj e q e r = N
p
pqrs s
q
r

) son:
Luego, las componentes de Q e (N

C e N
Q e qr = N
p pqrs s

+ ( + ) N
=N
p
pq rs
pr qs
ps qr
s
N + N N

= pq rsN pN s + (N
p pr qs s
p ps qr s )
N + (N
N + N N )
= N
q r
r q
s qr s
N

es un versor se cumple que N


Ya que N
s s = 1 , resultando que:
N
Q e qr = qr + ( + )N
q r

Universidad de Castilla- La Mancha


Ciudad Real - Espaa

Draft

) = 1 + ( + )N
N

Q e (N

Por: Eduardo W. V. Chaves (2014)

MECNICA DEL MEDIO CONTINUO: PROBLEMAS RESUELTOS

88

Ejemplo 1.94
Sea Q un tensor de segundo orden simtrico y dado por:
) = 1 + ( + )N
N

Q (N
es un versor.
donde , son constante y N
) y determinar las restricciones de e para que exista la
a) Obtener los autovalores de Q (N
) , i.e. Q 1 .
inversa de Q (N

b) Teniendo en cuenta que =

E
E
, =
, determinar los valores posibles de
(1 + )(1 2 )
2(1 + )

) sea un tensor definido positivo.


( E , ) para que Q (N
).
c) Obtener la inversa de Q (N

Solucin:
r

a) Fue demostrado en el Ejemplo 1.50 que, dados dos vectores a y b se cumple que:

r r
r r
det 1 + a b = 3 + 2 a b

Los autovalores se obtienen al resolver el determinante caracterstico det (Q 1) = 0 , donde


i son los autovalores de Q . Luego:

N
1 = 0
det 1 + ( + )N

N
=0
det ( )1 + ( + )N

Haciendo = ( ) y = ( + ) concluimos que:

N
=0
det ( )1 + ( + )N
N
=0
( ) 3 + ( ) 2 ( + )N
{
=1

( ) [( ) + ( + )] = 0
2

( ) 2 [( + 2 ) ] = 0

La ecuacin caracterstica anterior, ecuacin cbica en , tiene las siguientes soluciones:


( ) [( + 2 ) ] = 0
2

solucin

1 =
2
( ) = 0

2 =
[( + 2 ) ] = 0 = ( + 2 )
3

En el espacio principal de Q , las componentes de Q vienen dadas por:

Qij = 0
0

( + 2 )

0
0

Para que haya la inversa de Q , el determinante de Q tiene que ser distinto de cero:
Q = 2 ( + 2 ) 0

+ 2 0

b) Un tensor ser definido positivo si sus autovalores son mayores que cero, luego:

Universidad de Castilla- La Mancha


Ciudad Real - Espaa

Draft

Por: Eduardo W. V. Chaves (2014)

1 TENSORES

89

= 2(1 + ) > 0

E (1 )
E
E
+ 2 =
>0
+2
=

(1 + )(1 2 )
2(1 + ) (2 2 + 1)

(1 + ) 0 1

Verifiquemos que
1
2

(
2
1
)
0

0,5

Denotando por y1 = (1 + ) 0 , y2 = (1 ) 0 , y3 = (2 2 + 1) 0 , podemos reescribir las


condiciones como:

= 2 y > 0
1

+ 2 = Ey2 > 0

y3

E > 0
E < 0

Resumiendo:

E > 0

y1 > 0

E < 0
y < 0
1

y 2 , y3 > 0
E > 0

y 2 , y3 < 0

y2 > 0, y3 < 0

E
<

y2 < 0, y3 > 0

] 1 ; 0,5[ ] 1 ; [

] ; 1[

y ( )

y2 = (1 ) 0

= 1
E<0

E >0

zona no factible

y1 = (1 + ) 0

=1
E >0

= 0,5

y3 = (2 2 + 1) 0

) en el espacio principal de Q (N
) viene dadas por:
c) Las componentes de la inversa de Q (N
Universidad de Castilla- La Mancha
Ciudad Real - Espaa

Draft

Por: Eduardo W. V. Chaves (2014)

MECNICA DEL MEDIO CONTINUO: PROBLEMAS RESUELTOS

90

Qij = 0
0

( + 2 )

inversa

Qij 1 = 0

( + 2 )

) 1 son Q 1 = Q 1 =
Los valores principales de Q (N
1
2

, Q31 =

Q 1 =

2 ( + 2 )

1
. Recordar que un
( + 2 )

tensor y su inversa comparten el mismo espacio principal, i.e. son tensores coaxiales. Adems
) 1 como:
podemos hacer la representacin espectral de Q (N
Q 1 =

1 ( a )
a N

( a ) = Q 1N
(1) N
(1) + Q 1N
(2) N
( 2 ) + Q 1N
( 3) N
( 3)
N
1
2
3

a =1

(1) N
(1) + N
(2) N
( 2 ) ) + Q 1N
( 3) N
(3) = Q 1 (1 N
( 3) N
(3) ) + Q 1N
( 3) N
( 3)
= Q11 (N
3
1
3
N
) + Q 1N
N

= Q 1 (1 N
1

( 3) = N
. Es interesante ver el Ejemplo 1.77. Luego:
donde hemos considerado que N
N
)+
N
) + Q 1N
N
= 1 (1 N
Q 1 = Q11 (1 N
3

1
N

N
( + 2 )

1
1

N
= 1 1 1
N N
N

(
2
)
( + 2 )

+
1

N N
= 1
+

(
2
)

Notar

que

1
+
NN

Ce N
1 N
C e 1 N
,
Q 1 = N

donde

Ce

1
I
(1 1) .
2
2 (2 + 3 )

Ce N
:
Calculamos Qinv = N

C 1 N

1 1 ( + )
ij kl N l
(Qinv ) jk = N
i ijkl l = N i
il jk
2 2 ik jl
2 (2 + 3 )

1 1
+ N N

(Qinv ) jk =
N
(N i ik jl N
l
i il jk l )
i ij kl l
2 2
2 (2 + 3 )
(Qinv ) jk =

1
(N k N j + N l N l
4

(Qinv ) jk =

jk

jk

1
N j N k =

4
2 (2 + 3 )

jk

N j N k
+

4 2 (2 + 3 )

2 +

N j N k
+
4 (2 + 3 )

Luego:
Qinv =

2 +

1
N N
1 +
4
4 (2 + 3 )

Notar que 0 e (2 + 3 ) 0 y que adems son las mismas condiciones para que C 1
(ver Ejemplo 1.92).

Universidad de Castilla- La Mancha


Ciudad Real - Espaa

Draft

Por: Eduardo W. V. Chaves (2014)

1 TENSORES

91

1.11 Descomposicin Polar


Ejemplo 1.95
Considrese un tensor F que tiene inversa ( det ( F ) 0 ), y que puede ser descompuesto
como:
F = Q U = V Q
( a ) , y V tiene los autovalores
Si U tiene los autovalores a asociados a los autovectores N
a
(a )

asociados a los autovectores n , probar que:

a = a
( a ) y n ( a ) .
Obtener tambin la relacin que hay entre los autovectores N

Solucin:
Partiendo de la definicin de autovalor, autovector del tensor U :
(a) = N
(a)
UN
a

(aqu el ndice no indica suma)

Por la definicin de F podemos obtener las siguientes relaciones:


QT F = QT Q U = QT V Q

QT F = U = QT V Q

Reemplazando en la definicin de autovalor, autovector:


(a ) = N
(a)
UN
a
(a ) = N
(a)
QT V Q N
a
(a ) = Q N
(a)
Q QT V Q N
a
123
1

Resultando:
(a) = Q N
(a )
V Q N
a

V n ( a ) = a n ( a )

( a ) . Adems comparando las dos definiciones de


donde hemos considerado que n ( a ) = Q N
autovalor y autovector de los tensores U y V podemos comprobar que tienen los mismos
autovalores y distintos autovectores y que estn relacionados por la transformacin ortogonal
(a) .
n ( a ) = Q N

1.12 Tensor Esfrico y Desviador


Ejemplo 1.96
Considrese un tensor de segundo orden simtrico y su parte desviadora s dev .
a) Obtener el resultado de la operacin s :

s
.

b) Demostrar tambin que los tensores y dev son tensores coaxiales.


Solucin:

Universidad de Castilla- La Mancha


Ciudad Real - Espaa

Draft

Por: Eduardo W. V. Chaves (2014)

MECNICA DEL MEDIO CONTINUO: PROBLEMAS RESUELTOS

92

a) Teniendo la definicin de un tensor desviador = esf + dev = esf + s . Obtenemos que:


s=

I
1 . Luego:
3

1
s ij
ij 1 [I ]
3 [ ] 1 [I ]
s
indicial
1
=
=

ij = ik

3
kl kl 3 kl

jl

1
kl
3

ij

Con lo cual
s ij

s ij
kl

= s ij ik jl kl ij = s ij ik
3

s:

jl

1
1
s ij kl ij = s kl kl s ii = s kl
{
3
3
=0

s
=s

b) Para demostrar que dos tensores son coaxiales, hay que cumplir que:
dev = dev = ( esf ) = esf =
=

I
1
3

I
I
I

1 = 1 = 1 = dev
3
3
3

Con lo cual demostramos que los tensores y dev son coaxiales, es decir, tienen las mismas
direcciones principales.

1.13 Otros
Ejemplo 1.97
1

Considere J = [det (b )] 2 = ( III b ) 2 , donde b es un tensor de segundo orden simtrico, b = b T .


Obtener la derivada de J y de Ln(J ) con respecto a b .
Solucin:
1

( III b ) 2
J
= 1 ( III ) 1 III b = 1 ( III ) 1 III b T = 1 ( III ) 1 b 1 = 1 J b 1
=
2
2
b
b
b
b 2
b
b
2
b
2
2
2

1

Ln III b 2
[Ln(J )]

=
b
b

1 III b 1 1
= b
2 III b b
2

1.14 Notacin de Voigt


Ejemplo 1.98
a) Escribir la relacin = C : en notacin de Voigt, donde C = 1 1 + 2 I es un tensor
de cuarto orden, y los tensores simtricos y estn estructurados segn notacin de Voigt
como:

Universidad de Castilla- La Mancha


Ciudad Real - Espaa

Draft

Por: Eduardo W. V. Chaves (2014)

1 TENSORES

11

22

{ } = 33 ;
12
23

13

93

11

22

{ } = 33
212
2 23

213

b) Escribir la ecuacin = C 1 : en notacin de Voigt, donde el tensor C 1 (ver Ejemplo


1.90) viene dado por:
C 1 =

I
1 1
2
2 ( 2 + 3 )

Solucin:
Podemos escribir la relacin = (1 1 + 2 I ) : en notacin indicial:

ij = ij kl + 2 ik
2

jl

+ il

jk

kl

= ij kl + ik

jl

+ il

jk

)]

kl

El tensor identidad de segundo orden en notacin de Voigt queda:


1
1

1 0 0
1

Voigt
ij = 0 1 0 {} =
0
0 0 1
0

0

Luego el trmino (1 1)ij = ij kl en notacin de Voigt queda:

I ijkl = ij kl

1
1
1
1


1
1
Voigt

I = [1 1 1 0 0 0] =
0
0
0
0


0
0

[]

El tensor identidad de cuarto orden, I ijkl =

I ijkl

I1111
I
2211
I 3311
Voigt
[I ] =
I1211
I 2311

I1311

1 1 0 0 0
1 1 0 0 0
1 1 0 0 0
T
= {}{}
0 0 0 0 0
0 0 0 0 0

0 0 0 0 0

1
ik jl + il jk , en notacin de Voigt:
2

I1122
I 2222

I1133
I 2233

I1112
I 2212

I1123
I 2223

I 3322
I1222

I 3333
I1233

I 3312
I1212

I 3323
I1223

I 2322
I1322

I 2333
I1333

I 2312
I1312

I 2323
I1323

I1113 1

I 2213 0
I 3313 0
=
I1213 0
I 2313 0

I1313 0

0 0 0 0 0
1 0 0 0 0
0 1 0 0 0

0 0 12 0 0
0 0 0 12 0

0 0 0 0 12

Con eso podemos decir que C = 1 1 + 2 I y en notacin de Voigt queda:

Universidad de Castilla- La Mancha


Ciudad Real - Espaa

Draft

Por: Eduardo W. V. Chaves (2014)

MECNICA DEL MEDIO CONTINUO: PROBLEMAS RESUELTOS

94

1
1

1
[C ] =
0
0

1 1 0 0 0
1
0

1 1 0 0 0

0
1 1 0 0 0
+ 2
0 0 0 0 0
0
0
0 0 0 0 0

0 0 0 0 0
0

0 0 0 0 0 + 2

1 0 0 0 0
+ 2


0 1 0 0 0

+ 2
=

0 0 12 0 0 0
0
0
1

0 0 0 2 0
0
0
0

1
0 0 0 0 2 0
0
0

0
0

0
0

0
0
0

0
0
0

0
0

Luego:

= (1 1 + 2 I ) :

Voigt

0 0 0 11
11 + 2

+ 2

0 0 0 22
22
33

+ 2 0 0 0 33

=
0 0 212
0
0
12 0
23 0
0
0
0 0 2 23

0
0
0 0 213
13 0
14444444444
4244444444444
3

{ } = [C ]{ }

b)
= C 1 :

1
I
1 1 : =
I:
1 1 :
=
2 ( 2 + 3 )
2
2 ( 2 + 3 )
2

Tr ( )1
=
2
2 ( 2 + 3 )
ij =

1
ij
kk ij
2
2 ( 2 + 3 )

Notar que:

11
(11 + 22 + 33 ) 11 =
( 22 + 33 )
11
2
2 ( 2 + 3 )
2 ( 2 + 3 )
( 2 + 3 )

22
(11 + 22 + 33 ) 22 =
(11 + 33 )
=
22
2
2 ( 2 + 3 )
2 ( 2 + 3 )
( 2 + 3 )

11 =
22

33
(11 + 22 + 33 ) 33 =
(11 + 22 )
33
2
2 ( 2 + 3 )
2 ( 2 + 3 )
( 2 + 3 )

1
1
1

=
12
212 = 12
( kk ) 12 =
{ 2 12

2
2 ( 2 + 3 )
=0

33 =
12

2 23 =
213 =

23

13
Reestructurando las ecuaciones anteriores en notacin de Voigt, obtenemos:

Universidad de Castilla- La Mancha


Ciudad Real - Espaa

Draft

Por: Eduardo W. V. Chaves (2014)

1 TENSORES

( 2 + 3 )

11

2 ( 2 + 3 )

22
33 2 ( 2 + 3 )

=
212
0
2 23


0
213

95

2 ( 2 + 3 )
2 ( 2 + 3 )
+

( 2 + 3 )
2 ( 2 + 3 )

( 2 + 3 )
2 ( 2 + 3 )

0
1

{ } = [C ]1 { }

0 11

22
0
33

0 12
23

0 13

Ejemplo 1.99
r

Considrese T ( x , t ) un tensor simtrico de segundo orden, el cual es funcin de la posicin


r
( x ) y del tiempo (t ) . Considrese tambin que las componentes del tensor segn la direccin
x3 son iguales a cero, i.e. T13 = T23 = T33 = 0 .
r

NOTA: Definimos T ( x , t ) como un campo tensorial, i.e. el valor de T ( x , t ) depende de la


posicin y del tiempo. Si el tensor es independiente de una direccin para todo el dominio
r
r
( x ) , e.g. si T ( x , t ) es independiente de la direccin x3 , ver Figura 1.5, el problema puede ser
considerado como bidimensional (estado plano) simplificando bastante las ecuaciones.
2D

x2

T
Tij = 11
T12

x2

T12
T22

T22
T22

T12
T12

T12

T11

T11

T11

x1
T12
T22

x3

x1

Figura 1.5: Problema bidimensional (2D).


, T12 , tras un cambio de base en el plano x1 x 2 tal
a) Obtener las componentes T11 , T22
como se indica en la Figura 1.6;

b) Obtener tambin el valor de correspondiente a las direccin principales de T .


c) Obtener los valores de Tij , (i, j = 1,2) , cuando T11 = 1 , T22 = 2 , T12 = 4 y = 45 .
Obtener tambin los valores principales y direcciones principales de T ;

Universidad de Castilla- La Mancha


Ciudad Real - Espaa

Draft

Por: Eduardo W. V. Chaves (2014)

MECNICA DEL MEDIO CONTINUO: PROBLEMAS RESUELTOS

96

y T12 , para
d) Dibujar una grfica mostrando la relacin entre y las componentes T11 , T22
una variacin de 0 hasta 360 .

OBS.: Utilizar notacin de Voigt, y expresar los resultados en funcin de 2 .


a11
a ij = a 21
0

x2
x1

x 2

a12
a 22
0

0 cos sin 0
0 = sin cos 0
1 0
0
1

x1

Figura 1.6: Matriz de transformacin para problema bidimensional (2D).


Solucin:
a) Podemos utilizar directamente la ley de transformacin en la notacin de Voigt
{T } = [M] {T } , donde
T11
T
22
T
{T } = 33 ;
T12
T23


T13

a11 2

2
a 21
2

[M] = a 31
a 21 a11
a a
31 21
a 31 a11

a12 2

a13 2

a 22
a 32 2
a 22 a12
a 32 a 22
a 32 a12

a 23
a 33 2
a13 a 23
a 33 a 23
a 33 a13

T11
T
22
T
{T } = 33
T12
T23

T13

2a11 a12

2a12 a13

2a 21 a 22
2a 31 a 32
(a11 a 22 + a12 a 21 )
(a 31 a 22 + a 32 a 21 )
(a 31 a12 + a 32 a11 )

2a 22 a 23
2a 32 a 33
(a13 a 22 + a12 a 23 )
(a 33 a 22 + a 32 a 23 )
(a 33 a12 + a 32 a13 )

2a 21 a 23

2a 31 a 33

(a13 a 21 + a11 a 23 )
(a 33 a 21 + a 31 a 23 )
(a 33 a11 + a 31 a13 )
2a11 a13

En este caso particular la matriz de transformacin [M] tras eliminar filas y columnas
asociadas con la direccin x3 queda:
T11 a11
T = a 2
22 21
T12 a 21 a11

a12

a 22

a 22 a12

T11

2a 21 a 22
T22
a11 a 22 + a12 a 21 T12

2a11 a12

(1.103)

La matriz de transformacin ( a ij ) en el plano viene dada en funcin de un nico parmetro:


:
a11 a12
aij = a21 a22
a31 a32
Universidad de Castilla- La Mancha
Ciudad Real - Espaa

a13 cos
a23 = sin
a33 0
Draft

sin
cos
0

0
0
1

(1.104)

Por: Eduardo W. V. Chaves (2014)

1 TENSORES

97

T = A T AT

x2

T22

x 2

T22

T12

T11

T12

T11

T11

T11

T12

T12

x1

T22

x1

T22

x1

T = AT T A

Figura 1.7: Matriz de transformacin para problema bidimensional (2D).


Reemplazando los valores de los coeficientes en la ecuacin (1.103) obtenemos que:
2
T11 cos

T =
2
22 sin
T12 sin cos

sin 2
cos 2
cos sin

2 cos sin T11

2 sin cos T22


cos 2 sin 2 T12

Tomando partido de las siguientes relaciones trigonomtricas,


cos 2 sin 2 = cos 2 , sin 2 =

(1.105)
2 cos sin = sin 2 ,

1 cos 2
1 + cos 2
, cos 2 =
, obtenemos que:
2
2

1 + cos 2

T11

1
cos
2

T =

22
2

T12
2
sin

1 cos 2

sin 2
2

T11
1 + cos 2

sin 2 T22
2

T12
sin 2
cos 2

Explcitamente las componentes vienen dadas por:

1 + cos 2
1 cos 2
T11 +
T22 + T12 sin 2
T11 =
2
2

1 cos 2
1 + cos 2

=
T11
T22 T12 sin 2
T22
2
2

sin 2
sin 2
T12 =
T11 +
T22 + T12 cos 2

Reestructurando la expresin anterior an podemos decir que:

Universidad de Castilla- La Mancha


Ciudad Real - Espaa

Draft

Por: Eduardo W. V. Chaves (2014)

MECNICA DEL MEDIO CONTINUO: PROBLEMAS RESUELTOS

98

T11 + T22 T11 T22


+
cos 2 + T12 sin 2
T11 =
2
2

T + T22 T11 T22


= 11

cos 2 T12 sin 2


T22
2
2

T T22
T12 = 11
sin 2 + T12 cos 2

(1.106)

b) Recordemos que las direcciones principales se caracterizan por la ausencia de las


componentes tangenciales, es decir, Tij = 0 para i j . Si queremos encontrar las direcciones
principales en el caso plano hacemos que T12 = 0 , obteniendo as:
T T22
T12 = 11
sin 2 + T12 cos 2 = 0
2

sin 2
cos 2

2 T12
T11 T22

tan(2 ) =

T11 T22

sin 2 = T12 cos 2


2

2 T12
T11 T22

Con lo cual:

2 T12

T11 T22

1
2

= arctan

(1.107)

Para encontrar los valores principales (autovalores), debemos resolver la siguiente ecuacin
caracterstica:
T11 T
T12

T12
=0
T22 T

T 2 T ( T11 + T22 ) + T11 T22 T122 = 0

cuya solucin viene dada por:


T(1, 2 )

=
=

[ ( T11 + T22 )]

[ (T11 + T22 )]2


2(1)

T11 + T22

[(T11 + T22 )]2

4(1) T11 T22 T122

4 T11 T22 T122


4

Reestructurando la ecuacin anterior obtenemos que:


T(1, 2 )

T + T22
T T22
= 11
11

2
2

(1.108)

+ T122

c) Para obtener las componentes Tij , (i, j = 1,2) aplicamos directamente (1.106), donde
T11 = 1 , T22 = 2 , T12 = 4 y = 45 , i.e.:

1+ 2 1 2
T11 = 2 + 2 cos 90 4 sin 90 = 2,5

1+ 2 1 2
=

cos 90 +4 sin 90 = 5,5


T22
2 2

1 2
T12 =
sin 90 4 cos 90 = 0,5
2

Universidad de Castilla- La Mancha


Ciudad Real - Espaa

Draft

Por: Eduardo W. V. Chaves (2014)

1 TENSORES

99

El ngulo correspondiente a la direccin principal viene dado por:

2 T12 2 (4)
=
1 2
T11 T22

1
2

= arctan

( = 41.4375 )

Las valores principales de T ( x , t ) pueden ser obtenidos como sigue:


T(1, 2 )

T + T22
T T22
= 11
11

2
2

+ T122

T1 = 5.5311

T2 = 2.5311

d) Teniendo en cuenta la ecuacin (1.106) y variando de 0 hasta 360 podemos los


, T12 , y cuyos valores estn ilustrados en la siguiente grfica:
valores de T11 , T22
x1
T1

= 41.437

T2

x1
= 131.437
2

Componentes

T22

T1 = 5.5311

T22

T12

T11
0
0

50

100

-2

45

T12

-4

150

200

250

350

T11

x1

T2 = 2.5311

300

= 86.437

-6

TS max = 4.0311

Ejemplo 1.100
Obtener los valores principales y direcciones principales de la parte simtrica del tensor T ,
cuyas componentes en el sistema cartesiano vienen dadas por:
5 1
Tij =

3 4

(i, j = 1,2)

Solucin:
La parte simtrica del tensor viene dada por:

Universidad de Castilla- La Mancha


Ciudad Real - Espaa

Draft

Por: Eduardo W. V. Chaves (2014)

MECNICA DEL MEDIO CONTINUO: PROBLEMAS RESUELTOS

100

Tijsym =

5 2
1
Tij + T ji =

2
2 4

Valores principales:
5

=0

2 9 + 16 = 0

La solucin de la ecuacin cuadrtica viene dada por:


(1, 2 ) =

9 (9) 2 4 (1) (16)


2 1

1 T1 = 6,5615

2 T2 = 2,4385

Podemos dibujar el crculo de Mohr del tensor T sym :


TSsym

(T11sym , T12sym )
2

TII = 2,4385

TI = 6,5615

T Nsym

En el caso plano, la direccin principal se puede obtener directamente a travs de la relacin:


tan(2) =

2 T12sym
T11sym

sym
T22

22
=4
54

= 37,982

1.15 Campo de Tensores.


Ejemplo 1.101
Encuentre el gradiente de la funcin f ( x1 , x2 ) = sin( x1 ) + exp x1x2 en el punto (0;1).
Solucin:
Por definicin el gradiente de una funcin escalar viene definido de la forma:
xr f =

donde:

f
f
e1 +
e2
x1
x 2

f
f
= cos( x1 ) + x 2 exp x1x2 ;
= x1 exp x1x2
x1
x 2

xr f ( x1 , x 2 ) = cos( x1 ) + x 2 exp x1x2 e 1 + x1 exp x1 x2 e 2


xr f ( x1 = 0, x 2 = 1) = [2] e 1 + [0] e 2 = 2e 1

Universidad de Castilla- La Mancha


Ciudad Real - Espaa

Draft

Por: Eduardo W. V. Chaves (2014)

1 TENSORES

Ejemplo 1.102

101

Supongamos que v y son respectivamente vector y escalar, y dos veces diferenciables


continuamente. Usando notacin indicial, demostrar que:
r

a) xr ( xr v ) = 0
b) xr ( xr ) = xr 2
c) xr ( ) = xr + xr
r

d) xr (v ) = xr v + xr v

e) xr ( A B) = xr A : B + A ( xr B)

(donde A y B son tensores de segundo orden)

Solucin:
a) Considerando que
r
r
xr v = ijk v k , j e i

(1.109)

()
el
xl

(1.110)

xr () =

luego
r
r

xr ( xr v ) =
( ijk v k , j e i ) e l =
( ijk v k , j il ) =
( ljk v k , j ) = ljk v k , jl
x l
x l
x l

(1.111)

Observemos que ljk es un tensor antisimtrico en lj y vk , jl es simtrico con lj , luego:


ljk v k , jl = 0

(1.112)

b)
xr ( xr ) =

, j

( ,i e i ) e j =
( ,i ij ) =
= , jj =
x j
x j
x j
x j

x j

2
2
=
= xr
x 2
j

(1.113)

c)

[ xr ( )]i

= ( ) ,i = ,i + ,i = [ xr ]i + [ xr ]i

(1.114)

d) El resultado de la operacin xr (v ) resulta un escalar, luego:


r
r
r
xr (v ) = (v i ) ,i = ,i vi + vi ,i = ( xr ) v + ( xr v )

e) Considerando que ( A B) ij = A ik B kj , [ xr ( A B)]i = ( A B) ij , j = ( A ik B kj ) , j , luego


( A ik B kj ) , j = A ik , j B kj + A ik B kj , j = [ xr A : B ]i + [A ( xr B)]i

Ejemplo 1.103
Probar la identidad:

r
r r
r
xr (a + b) = xr a + xr b

Solucin:

Universidad de Castilla- La Mancha


Ciudad Real - Espaa

Draft

Por: Eduardo W. V. Chaves (2014)

MECNICA DEL MEDIO CONTINUO: PROBLEMAS RESUELTOS

102

Considerando que a = a j e j y b = b k e k y xr = e i
(a j e j + b k e k )
x i

e i

a j
x i

e j e i +

r r

podemos expresar xr (a + b) como:


x i

r
r
b k
a
b
e k e i = i + i = xr a + xr b
x i
x i x i

Solucin Alternativa: Desarrollando directamente en notacin indicial:

Q.E.D.

r
r r
r
xr (a + b) = (a i + b i ), i = a i , i +b i , i = xr a + xr b

Ejemplo 1.104
r

Obtener las componentes de ( xr a) b .


Solucin:
r

Considerando: a = a j e j ; b = b k e k y xr = e i

( i = 1,2,3 ) podemos decir que:


x i

a j
a j
r r (a j e j )
a j

( xr a) b =
e j e i (b k e k ) = b k ik
e j = b k
e j
e i (b k e k ) =
x i

x
x
x

i
k
i

Expandiendo el ndice mudo k :

bk

a j
x k

= b1

a j

+ b2

x1

a j
x 2

+ b3

a j
x 3

a 1
a
a
+ b2 1 + b3 1
j = 1 b1
x1
x 2
x 3

a 2
a 2
a

+ b2
+ b3 2
j = 2 b1
x1
x 2
x 3

j = 3 b a 3 + b a 3 + b a 3
1
2
3

x 3
x1
x 2

Ejemplo 1.105
Probar que la siguiente relacin es vlida:
r
r
q 1
1 r
xr = xr q 2 q xr T
T
T T
r r
r
donde q( x , t ) es un campo vectorial arbitrario y T ( x , t ) un campo escalar.

Solucin:
r
r
q
qi qi
1
1
1
1 r
=
= q i ,i 2 q i T,i = xr q 2 q xr T (escalar)
T
T
T
T x i T T , i T
r
x

Ejemplo 1.106
Demostrar que:
r

a) rot (a) xr (a) = ( xr a) + ( xr a)


r

r r

r r

(1.115)
r

b) xr (a b) = ( xr b)a ( xr a)b + ( xr a) b ( xr b) a

Universidad de Castilla- La Mancha


Ciudad Real - Espaa

Draft

(1.116)

Por: Eduardo W. V. Chaves (2014)

1 TENSORES

103

2
c) xr ( xr a) = xr ( xr a) xr a

(1.117)

2
d) xr ( xr ) = xr + ( xr ) ( xr )

(1.118)

Solucin:

a) El resultado de la operacin xr (a) ser un vector, cuyas componentes vienen dadas


por:

[r

r
x

r
(a) i

= ijk (a k ) , j

= ijk ( , j a k + a k , j )
= ijk a k , j ijk , j a k
(1.119)
r
r
r
r
= ( x a) i ijk ( x ) j a k
r
r
r
= ( xr a) i ( xr a) i
r
r
r
r
r
r
con lo que comprobamos la identidad: rot (a) = xr (a) = ( xr a) + ( xr a) .
r r
r r
Las componentes del producto vectorial (a b) vienen dadas por (a b) k = kij a i b j . Luego:

[r

r r
(a b) l = lpk ( kij a i b j ) , p = kij lpk (a i , p b j + a i b j , p )

r
x

(1.120)

b) Considerando que kij = ijk , el resultado de ijk lpk = il jp ip jl y reemplazando en la


expresin anterior obtenemos que:

[r

r
x

r r
(a b) l = kij lpk (a i , p b j + a i b j , p ) = ( il
= il

jp a i , p b j

jp

ip jl a i , p b j + il

ip jl )(a i , p b j + a i b j , p )

jp a i b j , p

ip jl a i b j , p

(1.121)

= a l , p b p a p , p b l + a l b p, p a p b l , p

Podemos observar que

[( br ) ar ] = a b
r
x

l, p

[( ar ) br ] = a
r
x

l, pb p

[(

r
x

a)b]l
r r

= a p, p b l ,

[(

r
x

b)a]l
r r

= al b p, p ,

.
r

c) Las componentes del producto vectorial ( xr a) vienen dadas por ( xr a) i = ijk a k , j .


123
ci

Luego:

[r

r
r
( xr a)

r
x

= qli c i ,l = qli ( ijk a k , j ) ,l = qli ijk a k , jl

(1.122)

Considerando que qli ijk = qli jki = qj lk qk lj , la expresin anterior queda:

[r

r
x

r
r
( xr a)

= qli ijk a k , jl = ( qj lk qk lj )a k , jl = qj lk a k , jl qk lj a k , jl
= a k , kq a q ,ll

(1.123)

Podemos observar que [ xr ( xr a)]q = a k , kq y xr 2 a q = a q ,ll .


r

d)
xr ( xr ) = ( ,i ) ,i = ,ii + ,i ,i = xr + ( xr ) ( xr )
2

(1.124)

donde y son funciones escalares.

Universidad de Castilla- La Mancha


Ciudad Real - Espaa

Draft

Por: Eduardo W. V. Chaves (2014)

MECNICA DEL MEDIO CONTINUO: PROBLEMAS RESUELTOS

104

Otra identidad interesante que origina de la anterior es:


xr ( xr ) = xr + ( xr ) ( xr )
2

(1.125)

xr ( xr ) = xr + ( xr ) ( xr )
2

Restando las dos identidades anteriores obtenemos que:


xr ( xr ) xr ( xr ) = xr xr
2

(1.126)

xr ( xr xr ) = xr xr
2

Ejemplo 1.107

a) Probar que xr ( xr v ) = 0 y que xr ( xr ) = 0 , donde es un campo escalar, y v es


un campo vectorial;
r

[r

b) Demostrar que xr ( xr v ) v = ( xr v )( xr v ) + xr ( xr v ) v ( xr v ) ( xr v ) ;
r

r r

c) Teniendo en cuenta que = xr v , demostrar que xr ( xr 2 v ) = xr 2 ( xr v ) = xr 2 .


Solucin:

Considerando: xr v = ijk v k , j e i

( xr
r

v =
ijk v k , j e i e l = ijk
v k , j il = ijk
v k , j = ijk v k , ji
x l
x l
x i
r
La segunda derivada de v es simtrica en ij , i.e. v k , ji = v k ,ij , mientras que ijk es antisimtrico
xr

( )

( )

en ij , i.e., ijk = jik , luego:


ijk v k , ji = ij1v1, ji + ij 2 v 2, ji + ij 3 v3, ji = 0
Observar que ij1v1, ji es el doble producto escalar de un tensor simtrico ( v1, ji = v1,ij ) con un
antisimtrico ( ij1 = ji1 ), cuyo resultado es cero.
Anlogamente demostramos que:

r
r
xr ( xr ) = ijk , kj e i = 0 i e i = 0

NOTA: El rotacional del gradiente de un escalar resulta ser igual al vector nulo, y la
divergencia del rotacional de un vector resulta ser igual a cero.
r

b) Denominamos por = xr v , con eso, quedamos con

[(

) ]

r
r
r r
xr xr v v

r
r r
= xr ( v )

Recurrimos a la identidad (1.116), luego, se cumple que:

r
r r
r r
r r
r r
r r
xr ( v ) = ( xr v ) ( xr )v + ( xr ) v ( xr v )
r
r
r
Notar que el trmino xr = xr ( xr v ) = 0 , que fue demostrado en el apartado a).

Luego, concluimos que:

r
r r
r r
r r
r r
xr ( v ) = ( xr v ) + ( xr ) v ( xr v )
r
r
r r
r
r r
r
r
= ( xr v )( xr v ) + xr ( xr v ) v ( xr v ) ( xr v )

c) Recurriendo a la identidad (1.117) podemos decir que:

Universidad de Castilla- La Mancha


Ciudad Real - Espaa

Draft

Por: Eduardo W. V. Chaves (2014)

1 TENSORES

105

r
r
r r
r
r r
2r
xr v = xr ( xr v ) xr ( xr v ) = xr ( xr v ) xr

Aplicando el rotacional a la expresin anterior obtenemos que:

r
r
r
r
r
r
2r
xr ( xr v ) = xr [ xr ( xr v )] xr ( xr )
14442r 444
3
=0

donde hemos tenido en cuenta que el rotacional del gradiente de un escalar resulta el vector
r
r
r
nulo. Recurrimos una vez ms la identidad (1.117) para expresar el trmino xr ( xr ) ,
resultando:

r
r
r
r
r
r
r
2r
2 r
2 r
xr ( xr v ) = xr ( xr ) = xr ( xr ) + xr = xr xr ( xr v ) + xr
144244
3
=0
r
r
2
= xr ( xr v )

Ejemplo 1.108
Demostrar que:
r

a) xr (a b) = ( xr a) b + a ( xr b) rot (a) b + a rot (b)


Solucin:
r

(1.127)

La operacin xr (a b) resulta un escalar:


r
r
r r
r r r r
xr (a b) = ( ijk a j b k ) ,i = ijk a j ,i b k + ijk b k ,i a j = ( xr a) b + a ( xr b)
1
1
23
r 2r3
r r
( a) k

( b) j

Ejemplo 1.109
a) Si T es un tensor de segundo orden, obtener la representacin simblica en la base
r
r
r
r
Cartesiana de: a.1) ( xr T ) , a.2) ( xr T )T , a.3) ( xr T T ) , y a.4) ( xr T T )T . a.5)
r
Considerando c un vector constante, demostrar que:

r
r
r
r r r
T
xr ( T c) = ( xr T ) c = c xr T

b) Obtener la notacin simblica de xr ( xr )T .


r
u
F = r + 1 , demostrar que
x

r
r
xr ( xr F )T = 0 , c.2)
r
r
xr F T = 0 . c.3) Obtener la expresin explcita de las componentes de xr F .

c.1) Consideremos el tensor

Solucin:
r

a.1) ( xr T ) =

Tqj

e p Tqj (e q e j ) =
e p e q e j = Tqj , p ipq e i e j = ipq Tqj , p e i e j
x p
x p

a.2) ( xr T )T = ipq Tqj , p e j e i = jpq Tqi , p e i e j


r

a.3) ( xr T T ) =

T jq

e p T jq (e q e j ) =
e p e q e j = ipq T jq , p e i e j
x p
x p

a.4) ( xr T T )T = jpq Tiq , p e i e j


Universidad de Castilla- La Mancha
Ciudad Real - Espaa

Draft

Por: Eduardo W. V. Chaves (2014)

MECNICA DEL MEDIO CONTINUO: PROBLEMAS RESUELTOS

106

donde hemos tenido en cuenta la definicin e j e k = ijk e i .


r

a.5) Haciendo que a = T c = ( Tqj c j )e q = aq e q , podemos obtener que:


r
r
aq
a
r
r

ipq e i = ipq q e i = ipq aq, p e i
xr ( T c ) = xr a =
e p aq e q =
x p
x p
x p
ipq aq , p e i = ipq ( Tqj c j ), p e i = ipq Tqj , p c j e i + ipq Tqj c j , p e i = ipq Tqj , p c j e i
{
=0

donde hemos tenido en cuenta que c es constante, i.e. c j , p =

c j
x p

= 0 jp .

Notar que ipq Tqj , p son las componentes de ( xr T )ij (vase (a.1)), luego

r
r
r
r
r
r r
xr ( T c ) = ipq Tqj , p c j e i = ( xr T )ij c j e i = ( xr T ) c i e i = c ( xr T )T i e i
r
r
( xr T ) c = ipq Tqj , p e i e j c k e k = ipq Tqj , p c k e i jk = ipq Tqj , p c j e i

b) En el apartado hemos demostrado que ( xr ) = ipq qj , p e i e j se cumple, luego


r
r
qj , p

e s ipq qj , p e j e i = ipq
e s e j e i = ipq tsj qj , ps e t e i
xr ( xr )T =
xs
xs
= ( iqp )( tjs ) qj , ps e t e i = iqp tjs qj , ps e t e i = qpi jst qj , ps e t e i

Notar que:
r
r
qj , p

e s ipq qj , p e i e j = ipq
e s e i e j = ipq qj , ps tsi e t e j
xr ( xr ) =
xs
xs
= its ipq qj , ps e t e j = ( tp sq tq sp ) qj , ps e t e j

= ( tp sq qj , ps tq sp qj , ps )e t e j = ( sj , ts tj , ss )e t e j
r
r
r
r
r
r
u
r r u r r
u r r
r
r
r
c.1) Notar que x F = x r + 1 = x r + x (1) = x r = x J , donde
x

x
x
r
u
u
hemos considerado que J r . Si consideramos que qj = J qj = q = u q, j , y reemplazamos
x j
x
r
r
en xr ( xr )T , podemos obtener que:
r
r
xr ( xr F )T = iqp tjs J qj , ps e t e i = iqp tjs uq , jps e t e i

Notar que uq , jps = uq , pjs = uq , psj , i.e. es simtrico en js , y el tensor tjs = tsj es antisimtrico
r

en js , luego tjsuq , jps = 0tqp , y xr ( xr F )T = 0ti e t e i = 0 .


Solucin alternativa:
Teniendo en cuenta que

it ij is
iqp tjs = qt qj qs = it qj ps + ij qs pt + is pj qt is qj pt qs pj it ps qt ij
pt pj ps
concluimos que
Universidad de Castilla- La Mancha
Ciudad Real - Espaa

Draft

Por: Eduardo W. V. Chaves (2014)

1 TENSORES

107

iqp tjs Fqj , ps = ( it qj ps + ij qs pt + is pj qt is qj pt qs pj it ps qt ij )uq , jps


= it qj psuq , jps + ij qs ptuq , jps + is pj qtuq , jps is qj ptuq , jps qs pj it uq , jps ps qt ijuq , jps
= it u j , jss + us ,its + ut , ppi u j , jti it us , pps ut ,ipp = 0ti

Notar que it u j , jss = it u p , pss = it u p ,ssp = it us , pps , u s ,its = u j ,itj = u j , jti , ut , ppi = ut ,ipp .
r

c.2) Expresando xr J T en notacin indicial podemos obtener que:


r
J qj

xr J T =
e p J qj (e j e q ) =
e p e j e q = J qj , p ipj e i e q
x p
x p
= ipj J qj , p e i e q = ipj uq , jp e i e q = 0 ip e i e q

Notar que uq , jp = uq , pj es simtrico en jp mientras que ipj = ijp es antisimtrico en jp .


r

c.3) Expresando xr J en notacin indicial obtenemos (vase (a.1)):


r
xr J = ipq J qj , p e i e j = ipquq , jp e i e j

Expandiendo el trmino ipquq , jp podemos obtener que:


ipquq, jp = ip1u1, jp

1
424
3

+ ip 2u2, jp
1
424
3

+ ip 3u3, jp
1
424
3

i11u1, j1 + i12u2, j1 + i13u3, j1


+

i 21u1, j 2 + i 22u2, j 2 + i 23u3, j 2


+
i 31u1, j 3

+
+ i 32u2, j 3

+
+ ip 3u3, j 3

luego
u3,12 u2,13 u3, 22 u2, 23 u3,32 u2,33 J 31, 2 J 21,3
r


r
( x J )ij = u1,13 u3,11 u1, 23 u3, 21 u1,33 u3,31 = J 11,3 J 31,1
u2,11 u1,12 u2, 21 u1, 22 u2,31 u1,32 J 21,1 J 11, 2

J 32, 2 J 22,3 J 33, 2 J 23,3

J 12,3 J 32,1 J 13,3 J 33,1


J 22,1 J 12, 2 J 23,1 J 13, 2

Notar que
J 13, 2 J 12,3 J 23, 2 J 22,3 J 33, 2 J 32,3 u1,32 u1, 23 u 2,32 u 2, 23 u3,32 u3, 23
r

T
( xr J ) ij = J 11,3 J 13,1 J 21,3 J 23,1 J 31,3 J 33,1 = u1,13 u1,31 u 2,13 u 2,31 u3,13 u3, 31
J 12,1 J 11, 2 J 22,1 J 21, 2 J 32,1 J 31, 2 u1, 21 u1,12 u 2, 21 u 2,12 u3, 21 u3,12

= 0ij
r
u
Notar que, si J = r podemos concluir que tambin se cumple que
x
r
r
r
r
1
1r
1r
= ( J + J T ) xr ( xr )T = xr ( xr J )T + xr ( xr J T )T = 0
1
424
3
2
2 1442443 2
=0

Ejemplo 1.110

=0

Dados los vectores a y v demostrar que

r
r r
r
r
( xr v ) a = xr v ( xr v )T

] ar

Solucin:
Universidad de Castilla- La Mancha
Ciudad Real - Espaa

Draft

Por: Eduardo W. V. Chaves (2014)

MECNICA DEL MEDIO CONTINUO: PROBLEMAS RESUELTOS

108

[r

Si consideramos que ( xr v ) i = ijk vk , j , luego ( xr v ) a s = sip ijk vk , j a p . Notar tambin


que la relacin sip ijk = psi jki = pj sk pk sj se cumple. Con eso obtenemos que

[(r

r
x

r r
v ) a s = sip ijk vk , j a p = ( pj sk pk sj )vk , j a p = ( pj sk vk , j pk sj vk , j )a p
r
r
r
= (vs , p v p , s )a p = xr v ( xr v )T a

{[

Solucin alternativa:

r
r
r
r
= xr v , luego xr v ( xr v )T = 2( xr v ) anti = 2 l anti . Notar que el vector
r 1 r
r
r r
r
axil asociado con el tensor antisimtrico ( xr v ) anti = (v xr ) anti es el vector = ( xr v )
2
r anti r r r
r
(ver Ejemplo 1.37). Recordar que la propiedad ( x v ) a = a se cumple, con eso
l

Si consideramos que

concluimos que:
r r r
r
( xr v ) anti a = a

rT r 1 r
r r
1 r r
x v ( xr v ) a = ( xr v ) a
2
2
r
r
r r
r vr ( r vr )T a
= ( xr v ) a
x
x

Ejemplo 1.111
r

r r

Consideremos un campo vectorial u = u( x ) . A travs de sus componentes demostrar que: a)


r
2r
xr u = xr ( xr u)
r
r
xr ( xr u) = 0 .

cuando xr ( xr u) = 0 , b) que xr 2u = xr ( xr u)

cuando

Solucin:
Hemos demostrado en el Ejemplo 1.106 que lo siguiente es cierto:
r
r
r
r
2r
xr ( xr a) = xr ( xr a) xr a
indicial
ilq qjk a k , jl = a j , ji ai , jj
Con lo cual

r
r
r r
r
2r
xr ( xr u) xr u = xr ( xr u) xr ( xr u)

indicial

ui , jj = u j , ji ilq qjk uk , jl

Con eso es fcil verificar que:


a)

r
r
r r
r
2r
xr ( xr u) xr u = xr ( xr u) xr ( xr u)
1442r 44
3

r
2r
xr u = xr ( xr u)

=0

Componentes:
ui , jj = u j , ji

ui ,11 + ui , 22 + ui ,33 = u1,1i + u 2, 2i + u3,3i

u1,11 + u1, 22 + u1,33 = u1,11 + u 2, 21 + u3,31

u 2,11 + u 2, 22 + u2,33 = u1,12 + u2, 22 + u3,32


u + u + u = u + u + u
3, 22
3,33
1,13
2 , 23
3,33
3,11

u1, 22 + u1,33 = u 2, 21 + u3,31

u 2,11 + u 2,33 = u1,12 + u3,32


u + u
3, 22 = u1,13 + u 2 , 23
3,11

(1.128)

Notar que en el sistema Cartesiano tenemos que:


r
u = ui e i = u1e 1 + u2 e 2 + u3e 3

r
r
r
r
u u
u u
u
u
( xr u) rot (u) = (rot (u) )i e i = 3 2 e 1 + 1 3 e 2 + 2 1 e 3
x2 x3
x3 x1
x1 x2
142
142
142
4 r43
4
4 r43
4
4 r43
4
= (rot (u) )1

Universidad de Castilla- La Mancha


Ciudad Real - Espaa

= (rot (u) )2

Draft

= (rot (u) )3

Por: Eduardo W. V. Chaves (2014)

1 TENSORES

109

r
r
r
r
r
r
r
r
r (rot (u) )3 (rot (u) )2
(rot (u) )1 (rot (u) )3
(rot (u) )2 (rot(u) )1
r
r

e1 +
e 2 +
e 3

x ( x u) =

x
x
x
x
x
x

2
3
3
1
1
2

r
r
(rot (u) )3 (rot (u) )2 u 2 u1 u1 u3

x x

x3
2 1 x2 x3 x3 x1
x2r

r
(rot (u) )1 (rot (u) )3 u3 u 2 u 2 u1
r
r
r

xr ( xr u) i =

=
x3 x2 x3 x1 x1 x2
x1

x3r

(rot (u) ) (rot (u


) )1
2
u1 u3 u3 u 2

x2
x1

x1 x3 x1 x2 x2 x3

u 2,12 u1, 22 u1,33 + u3,13

= u3, 23 u 2,33 u 2,11 + u1, 21


u u u + u
3,11
3, 22
2, 32
1,31
r
r
r
r
Si estamos considerando que xr ( xr u) = 0 , luego:

u 2,12 u1, 22 u1,33 + u3,13 0 u 2,12 + u3,13 = u1, 22 + u1,33


r
r
r


xr ( xr u) i = u3, 23 u 2,33 u 2,11 + u1, 21 = 0 u3, 23 + u1, 21 = u 2,33 + u 2,11
u u u + u 0 u + u
1,31
3,11
3, 22
2, 32
2 ,32 = u3,11 + u3, 22
1,31

que son las mismas condiciones presentadas en las ecuaciones en (1.128).


b)

r
r
r
r
r
2r
xr ( xr u) xr u = xr ( xr u) xr ( xr u)
142r 43

r
r
r
r
2u = xr ( xr u)

=0

Componentes
u1,11 + u1, 22 + u1,33 = (u2,12 u1, 22 u1,33 + u3,13 )

ui , jj = ilq qjk uk , jl

u2,11 + u2, 22 + u2,33 = (u3, 23 u2,33 u2,11 + u1, 21 )


u + u + u = (u u u + u )
3, 33
1,31
3,11
3, 22
2 , 32
3,11 3, 22
r
r
Si consideramos que xr ( xr u) = 0 , podemos obtener que:

[ xr ( xr ur )] i = u1,1i + u2,2i + u3,3i = 0i

u1,11 + u2, 21 + u3,31 = 0

u1,12 + u2, 22 + u3,32 = 0


u + u + u = 0
2 , 23
3, 33
1,13

(1.129)

u3,31 + u2, 21 = u1,11

u1,12 + u3,32 = u 2, 22
u + u
2 , 23 = u3, 33
1,13

Si reemplazamos las ecuaciones anteriores dentro de (1.129) demostramos que la igualdades se


cumplen.
Ejemplo 1.112

Sean un campo tensorial de segundo orden, y a un campo vectorial. Demostrar las


siguientes identidades:
a)
b)

r
r
r
xr (a ) = : [( xr a) T ] + a ( xr )
r
r r r
r
xr ( a) = a [ xr T ] [ xr a]

(1.130)
(1.131)

donde es el tensor Levi-Civita (pseudo-tensor de tercer orden).

Universidad de Castilla- La Mancha


Ciudad Real - Espaa

Draft

Por: Eduardo W. V. Chaves (2014)

MECNICA DEL MEDIO CONTINUO: PROBLEMAS RESUELTOS

110

Solucin:a)

r
a = a i e i jk e j e k = jk a i pij e p e k

xr (a ) =
jk a i pij e p e k e q =
jk a i pij e p
x q
x q
r
xr (a ) = jk a i pij ,k e p = ( pij jk , k a i + pij jk a i ,k )e p

kq

jk a i pij e p
x k

Notar que pij jk , k ai = pij ( xr ) j ai = pij (a)i ( xr ) j = [a ( xr )]p


r

y pij jk ai ,k = pij jk ( xr a) ik = pij ( xr a) ik jk = pij ( xr a) T

] = { : [(
ij

r
x a)

T ]}p

con lo cual demostramos la ecuacin en (1.130).


r

Notar que, si a = x la ecuacin (1.130) queda:

r
r
r
r
r
xr ( x ) = : ( xr x ) T + x ( xr ) = : (1 T ) + x ( xr ) = : T + x ( xr )

b)

r
a = jk e j e k ai e i = jk ai pki e j e p

xr ( a) =
jk ai pkie j e p e q =
jk ai pki e j
xq
xq
r
xr ( a) = jk ai pki e j = ( pki jk , p ai + pki jk ai , p )e j

pq

jk ai pki e j
x p

,p

Notar que

[r

pki jk , p ai = ipk jk , p ai = xr T

pki jk ai , p = kip ai , p jk = kpi ai , p jk

{ [

]}

r r
= a xr T j
r
r
r
r
= ( xr a) k jk = xr a

ij ai

Ejemplo 1.113
r

{ [

]}

r r

Sea v un campo vectorial que es funcin de x , i.e. v = v ( x ) , donde sus componentes vienen
dadas por:
v1 = x1 5 x 2 + 2 x3

v 2 = 5 x1 + x 2 3 x3
v = 2 x + 3 x + x
1
2
3
3
r
r
a) Obtener el gradiente de v ; b) Obtener xr v : 1 ; c) Hacer la descomposicin aditiva del
r
tensor xr v a travs de su parte simtrica y otra antisimtrica; d) Obtener el vector axil
r
asociado al tensor antisimtrico ( xr v ) anti .

Solucin: a)

r
r v
xr v = r
x

v1

x1
v

r
v
componente

s ( xr v ) ij = i = 2
x j
x
1
v3
x1

Universidad de Castilla- La Mancha


Ciudad Real - Espaa

Draft

v1
x 2
v 2
x 2
v 3
x 2

v1

x3 1 5 2
v 2
= 5
1 3
x3

1
v3 2 3
x3

Por: Eduardo W. V. Chaves (2014)

1 TENSORES

111

b) xr v : 1 = Tr ( xr v ) = 1 + 1 + 1 = 3 . Observar que la traza del gradiente de un vector es igual a


r

la divergencia del vector, i.e.: xr v = vi ,i = v1,1 + v 2, 2 + v 3,3 =


r

c) xr v = ( xr v ) sym + ( xr v ) anti =

v1 v 2 v3
+
+
= 3.
x1 x 2 x3

] [

r
r
r
r
1
1
( xr v ) + ( xr v ) T + ( xr v ) ( xr v ) T
2 44424443 1
2 44424443
1
r
=( xr v ) sym

r
=( xr v ) anti

Luego, las componentes de ( xr v ) sym y ( xr v ) anti vienen dadas respectivamente por:


r
( xr v ) ijsym

1 0 0
1 vi v j

=
+
= 0 1 0
2 x j x i
0 0 1

r
( xr v ) ijanti

0 5 2
1 v i v j
0 3
=

= 5
2 x j xi
2 3
0

d) Recordar que

r
(W) ij ( xr v ) anti

ij

vianti
,j

1 v
v
= 2 1
2 x1 x 2
1 v
v
3 1
2 x1 x 3

1 v1 v 2

2 x 2 x1

0
1 v3 v 2

2 x 2 x 3

1 v1 v3

2 x 3 x1
1 v 2 v 3

2 x3 x 2

(1.132)

w3 w2
w1
0
0
w1
r
donde w1 , w2 , w3 son las componentes del vector axil w correspondiente al tensor
r
antisimtrico W ( xr v ) anti , luego para el problema propuesto:
0
= W21
W31

W12
0
W32

W13 0
W23 = W12
0 W13

W12
0
W23

W13 0
W23 = w3
0 w2

w1 = 3

w2 = 2
w = 5
3

w2 0 5 2

0
w1 = 5
0 3
w1
0 2 3
0
r
El vector axil en la base Cartesiana queda: w = 3e 1 + 2e 2 + 5e 3 .
0
w
3
w2

w3

Solucin Alternativa d) Recurriendo al Ejemplo 1.36 donde hemos demostrado que


r

1 r r
(a x )
2

es el vector axil asociado al tensor antisimtrico ( x a ) anti . Luego, el vector axil asociado al

[r

tensor antisimtrico ( xr v ) anti = (v ) ( xr )


e 1
r 1
w=
2 x1
v1
=

e 2

x 2
v2

anti

e 3
v

1 v
= 3 2
x3 2 x 2 x3
v3

es el vector w =

1 rr r
x v .
2

v
v
v
v
e 1 3 1 e 2 + 2 1
x1 x 2

x1 x 3


e 3

1
(3 (3) )e 1 ((2) (2) )e 2 + (5 (5))e 3 = 3e 1 + 2e 2 + 5e 3
2

Universidad de Castilla- La Mancha


Ciudad Real - Espaa

Draft

Por: Eduardo W. V. Chaves (2014)

MECNICA DEL MEDIO CONTINUO: PROBLEMAS RESUELTOS

112

Ejemplo 1.114
l

Sea un tensor de segundo orden definido por


r
y W = ( xr v ) anti demostrar que

r
r
= xr v . Teniendo en cuenta que D = ( xr v ) sym

r
r
W D + D W = 2(D W) anti = ( xr v xr v ) anti = ( l l ) anti

Solucin:
En el Ejemplo 1.34 hemos demostrado que, dado un tensor de segundo orden arbitrario
se cumple que
l

anti

+l

sym

sym

anti

= 2( l

anti

sym anti

Luego, se cumple que W D + D W = 2(D W ) anti . Teniendo en cuenta la definicin de


simetra y antisimetra, D =

1
l +l
2

W D + D W = 2(D W ) anti =

] , W = 12 [l l ] , podemos concluir que:

2
(l + l T ) (l l T )
4

anti

1
T
T
T
T anti
l l + l l l l l l
2
anti
1
1
T
T
T
l l l l
= 1
+
3 2 l l l l
2 4442444

=0

1
T
l l (l l )
2

OBS.: Notar que el tensor resultante


(l l

l )T

= l l

l T

anti

1
2( l l ) anti
2
l

T anti

anti

r
r
= ( l l ) anti = ( xr v xr v ) anti

l es simtrico, ya que:

l .

Ejemplo 1.115
Consideremos un escalar J = F det(F ) y un tensor de segundo orden arbitrario definido
por

r
dF
= xr v = F& F 1 , donde F&
representa la derivada temporal de F . Demostrar que
dt

se cumple que:
r
d(J ) &
J = J ( xr v )
dt

(1.133)

Solucin:
En el Ejemplo 1.87 hemos demostrado que, dado un tensor de segundo orden arbitrario
A = A () , se cumple que

dA

dA
= A Tr
A 1 . Haciendo A = F y = t podemos obtener

d
d

que:
dF
dt

( )

r
r
dJ
dF
= F Tr
F 1 = J Tr F& F 1 = J Tr F& F 1 = J Tr l = J Tr( xr v ) = J ( xr v )
dt
dt

Solucin alternativa:
En el Ejemplo 1.46 hemos demostrado que dado un tensor de segundo orden F la relacin
F tpq = rjk Frt F jp Fkq se cumple, y si aplicamos la derivada material podemos obtener que:

Universidad de Castilla- La Mancha


Ciudad Real - Espaa

Draft

Por: Eduardo W. V. Chaves (2014)

1 TENSORES

113

DF
D
tpq =
( rjk Frt F jp Fkq ) = rjk F&rt F jp Fkq + rjk Frt F& jp Fkq + rjk Frt F jp F&kq
Dt
Dt

(1.134)

Segn el enunciado del problema tenemos que l = F& F 1 F& = l F , con lo cual las
siguientes relaciones se cumplen F&rt = l rs Fst , F& jp = l js Fsp y F&kq = l ks Fsq , y la ecuacin en
(1.134) puede ser reescrita como:
DF
tpq = rjk F&rt F jp Fkq + rjk Frt F& jp Fkq + rjk Frt F jp F&kq
Dt
= rjk l rs Fst F jp Fkq + rjk Frt l js Fsp Fkq + rjk Frt F jp l ks Fsq

Multiplicamos ambos lados de la igualdad por ut v p w q , resultando:


DF
Dt

tpq ut v p w q = rjk l rs Fst F jp Fkq ut v p w q + rjk Frt l js Fsp Fkq ut v p w q + rjk Frt F jp l ks Fsq u t v p w q
= rjk ( l rs Fst ut )( F jp v p )( Fkq w q ) + rjk ( Frt ut )( l js Fsp v p )( Fkq w q ) + rjk ( Frt ut )( F jp v p )( l ks Fsq w q )
= rjk ( l rs a s )(b j )(c k ) + rjk (a r )( l js b s )(c k ) + rjk (a r )(b j )( l ks c s )

donde hemos denotado por a s = Fst ut , b j = F jp v p , c s = Fsq w q . La ecuacin anterior en


notacin tensorial queda:

] [

r r r r
DF r r r
r r r r
r
r r r
u ( v w ) = ( l a) (b c ) + a ( l b) c + a b ( l c ) = Tr ( l ) a (b c )
Dt

donde hemos usado la propiedad de traza (ver Ejemplo 1.48). La ecuacin anterior aun puede
ser reescrita como:

DF r r r
r r r
r
r
r
r r r
u ( v w ) = Tr ( l ) a (b c ) = Tr ( l ){( F u) [( F v ) ( F w )]} = Tr ( l ) F u ( v w )
Dt

donde hemos aplicado la propiedad de determinante (ver Ejemplo 1.49), con lo cual
concluimos que

DF
= Tr ( l ) F .
Dt

Ejemplo 1.116
r

Considrese un campo vectorial representado por su campo vector unitario b ( x ) , ver Figura
r
r
1.8. Obtener un tensor proyeccin de segundo orden P tal que se cumpla que p = P u ,
r
r
r
donde u es un vector arbitrario y p es ortogonal al campo definido por el versor b ( x ) .

r
b ( x )

Figura 1.8: Campo vectorial


Universidad de Castilla- La Mancha
Ciudad Real - Espaa

Draft

Por: Eduardo W. V. Chaves (2014)

MECNICA DEL MEDIO CONTINUO: PROBLEMAS RESUELTOS

114

Solucin:
El problema planteado se puede apreciar en la figura abajo:
r
u

r r
r
ub = p = P u

r r
a = u // b
r
b ( x )

Luego, a travs de suma de vectores se cumple que: u = a + p . Adems el vector a puede ser
r r
r
r
obtenido a travs de la proyeccin de u segn la direccin b : a = a b = (u b ) b . Con eso
podemos decir que:

r r r
p=ua
r
r
r
r
= u (u b ) b = u (u b ) b
r
r
= 1 u (b b ) u
r
= 1 (b b ) u
r
= P u

p i = ui ai
= u i (u k b k )b i
= u u b b
k

= ( ik

ik

b k b i )u k

= Pik u k

Con lo cual concluimos que el tensor proyeccin de segundo orden viene dado por:
P = 1 b b

Este mismo resultado podra haber sido obtenido a travs del producto vectorial. Dibujando
el problema planteado en otra perspectiva para mejor visualizacin, tenemos que:
r
u b

r
b ( x )

r
u
r
b (u b )

Teniendo en cuenta que a (b a) = [(a a)1 a a] b , ver Ejemplo 1.16, podemos decir
r

r r

r
r
r r
que: b (u b ) = (b b )1 b b u = 1 b b u = p .

Luego, podemos representar un vector como sigue:

r r r
r
r
u = u // b + ub = (b b ) u + 1 (b b ) u

r
r
r
r
donde u// b = (b b ) u es el vector paralelo a direccin de b , y ub = 1 (b b ) u es el
vector perpendicular.

Universidad de Castilla- La Mancha


Ciudad Real - Espaa

Draft

Por: Eduardo W. V. Chaves (2014)

1 TENSORES

Ejemplo 1.117

115

r r

Dado un campo vectorial v ( x ) , demostrar que se cumple la siguiente relacin:


r
r r 1
r
r
xr v v = xr (v 2 ) v ( xr v )
2
r
r
donde v = v es el mdulo de v .

Solucin:

r r
r
r
1 r 2
1
1
1
x (v ) i = [ xr (v v )]i = (v k v k ) ,i = (v k ,i v k + v k v k ,i ) = v k v k ,i = (v xr v ) i .
2
2
2
2
r
r
En un punto del campo vectorial v vamos considerar un plano normal a v y recordar que la
r
proyeccin de un tensor de segundo orden segn una direccin ( v ) resulta un vector el cual
r
no necesariamente tiene la misma direccin que ( v ), con eso vamos representar los siguientes
r r
r
r
vectores xr v v y v xr v :

Notar que

r r
xr v v

r
r
( xr v )

r
v

r r
cv
r
r
r
c ( xr v )

r
r
v xr v

r
r r
r
c = v ( xr v )

Verifiquemos que a travs de suma de vectores se cumple que:

r r r r
r
xr v v + c = v xr v
r r
r
r r
c = v xr v xr v v
r r
r r
r
c = v xr v v xr v T
r r
r
r
r
r
c = v ( xr v xr v T ) = v 2( xr v ) anti

Si consideramos que w es el vector axil asociado al tensor antisimtrico ( xr v ) anti se cumple


r
r r r
r
r
r r
que: ( xr v ) anti v = w v v ( xr v ) anti = v w . Adems se cumple tambin que
r
r
r
r
rot (v ) xr v = 2 w . Luego,
r
r r
r
r
r r
r
c = v 2( xr v ) anti = v 2w = v ( xr v )

(1.135)

con eso concluimos que:


r r r r
r
xr v v + c = v xr v

Universidad de Castilla- La Mancha


Ciudad Real - Espaa

r r r
r r
xr v v = v xr v c

Draft

r r 1
r r
r
xr v v = xr (v 2 ) v ( xr v )
2

Por: Eduardo W. V. Chaves (2014)

MECNICA DEL MEDIO CONTINUO: PROBLEMAS RESUELTOS

116

Es interesante observar que cuando ( xr v ) resulta ser un tensor simtrico, ( xr v ) = ( xr v ) sym ,


r

r r

se cumple que ( xr v ) anti = 0 , c = 0 , ( xr v ) = 0 , xr v v = v xr v y tiene la misma


r
direccin que v .
r

En el caso que se cumpla ( xr v ) = ( xr v ) anti tenemos que: c = v 2( xr v ) anti = 2v ( xr v ) , ver


r
r
r r
r
expresin (1.135). Con eso se cumple tambin que v xr v = xr v v , y adems, v es
r
r
perpendicular al vector ( xr v ) , ver Figura 1.9.
r
r
( xr v ) = ( xr v ) anti

r r
xr v v

r r
cv
r
r
r
c ( xr v )

r
v
r
r
( xr v )

r
r
v xr v

r
r r
r
r
r
c = v ( xr v ) = 2v ( xr v )

Figura 1.9
Solucin Alternativa:
r r
r
r
r
r
r
r
r
xr v v = (( xr v ) sym + ( xr v ) anti ) v = ( xr v ) sym v + ( xr v ) anti v
r
r
r
r
r
r
r
r
= ( xr v ) sym v + ( xr v ) anti v + (( xr v ) anti v ( xr v ) anti v )
r
r
r
r
r
r
= (( xr v ) sym v ( xr v ) anti v ) + 2( xr v ) anti v
r
r
r
r
r
r
r
1
= (( xr v ) + ( xr v )T ) (( xr v ) ( xr v )T ) v + 2( xr v ) anti v
2
r
r
r
r r
r
r
r
1
= (2( xr v )T ) v + 2( xr v ) anti v = v ( xr v ) + 2( xr v ) anti v
2
r
r
1 r 2 r
= x (v ) v ( xr v )
2
r
r
r
r
r
r
r
r
r
Recordar que ( xr v anti )T = ( xr v ) anti , luego 2( xr v ) anti v = v 2( xr v ) anti = v ( xr v ) .

Ejemplo 1.118
r r

Considrese un campo vectorial estacionario u( x ) . Obtener las componentes del diferencial


r
r r
total du . Considerando que u( x ) representa el campo de desplazamientos y es independiente
de la componente x3 , hacer la representacin grfica del campo de desplazamiento en un
elemento diferencial de rea dx1 dx 2 .
Solucin: Segn la definicin de diferencial total y de gradiente se cumple que:

Universidad de Castilla- La Mancha


Ciudad Real - Espaa

Draft

Por: Eduardo W. V. Chaves (2014)

1 TENSORES

r r
u( x )
r
x

x2

r
dx

117

r r
r
u( x + dx )
r r r
r r r
du u( x + dx ) u( x )
r
r r
du = xr u dx

r
r
x + dx

x1
x3

Luego, las componentes vienen dadas por:

du i =

u i
dx j
x j

u1

du1 x1
du = u 2
2 x
du 3 1
u 3
x1

u1
x 2
u 2
x 2
u 3
x 2

u1
u
u
u1
dx1 + 1 dx 2 + 1 dx3
du1 =

x1
x 2
x3
x3 dx
1

u 2
u
u
u
dx 2 du 2 = 2 dx1 + 2 dx 2 + 2 dx3

x3
x1
x 2
x3


u 3 dx3
u 3
u 3
u 3
=
+
+
d
dx
dx
dx3
u

3
1
2

x3

x1
x 2
x3

con
du1 = u1 ( x1 + dx1 , x 2 + dx 2 , x3 + dx3 ) u1 ( x1 , x 2 , x3 )

du 2 = u 2 ( x1 + dx1 , x 2 + dx 2 , x3 + dx3 ) u 2 ( x1 , x 2 , x3 )
du = u ( x + dx , x + dx , x + dx ) u ( x , x , x )
3
1
1
2
2
3
3
3
1
2
3
3

Para el caso plano, es decir, cuando el campo es independiente de x3 , el campo de


desplazamientos en el elemento diferencial de rea viene definido por:
u1
u1

du1 = u1 ( x1 + dx1 , x 2 + dx 2 ) u1 ( x1 , x 2 ) = x dx1 + x dx 2

2
1

du = u ( x + dx , x + dx ) u ( x , x ) = u 2 dx + u 2 dx
2
1
2
1
1
2
2
2
1
2
2
x1
x 2

o an:
u1
u1

u1 ( x1 + dx1 , x 2 + dx 2 ) = u1 ( x1 , x 2 ) + x dx1 + x dx 2

2
1

u
u

u ( x + dx , x + dx ) = u ( x , x ) + 2 dx + 2 dx
2
1
1
2
2
2
1
2
2 1
x1
x 2

Observemos que la expresin anterior es equivalente a la expansin en serie de Taylor


teniendo en cuenta solo hasta trminos lineales. La representacin del campo de
desplazamiento en el elemento diferencial de rea se muestra en la Figura 1.10.

Universidad de Castilla- La Mancha


Ciudad Real - Espaa

Draft

Por: Eduardo W. V. Chaves (2014)

MECNICA DEL MEDIO CONTINUO: PROBLEMAS RESUELTOS

118

u 2
dx 2
x 2

u2 +

u2 +

u 2
u
dx1 + 2 dx 2
x1
x 2

( x1 , x 2 + dx 2 )

( x1 + dx1 , x 2 + dx 2 )

u1
dx 2
x 2

u1 +

u1 +

r
du

dx 2

u1
u
dx1 + 1 dx 2
x1
x 2

u1 +

(u1 )

( x1 + dx1 , x 2 )

( x1 , x 2 )

x2

u1
dx1
x1

(u 2 )

u2 +
dx1

x1

u 2
dx1
x1

144444444444444444424444444444444444443

=
644444444444444444474444444444444444448
x 2 ,u 2
u2 +

u1
dx2
x2

u 2
dx2
x2

dx 2

O
u2

O
u1
u1 +

dx 2
A
O

dx1

u 2
dx1
x1

dx1

u1
dx1
x1

x1 ,u1

Figura 1.10
Universidad de Castilla- La Mancha
Ciudad Real - Espaa

Draft

Por: Eduardo W. V. Chaves (2014)

1 TENSORES

Ejemplo 1.119

119

Dado un campo tensorial de segundo orden, T ( x ) . Demostrar que si no hay una fuente de
r
r
r
T ( x ) se cumple que la divergencia de T ( x ) es igual a cero, i.e. xr T = 0 . Para la

demostracin considerar el campo tensorial en un elemento diferencial de volumen


dV = dx1 dx 2 dx 3 en el sistema Cartesiano.
Solucin:

Primero vamos establecer el campo T ( x ) en el diferencial de volumen. Para ello, partimos de


r
la definicin del diferencial de T ( x ) que viene definido a travs del gradiente como:
r
r
r
dT T ( x + dx ) T ( x )
r
r
r
r
r
r
r
r
r
T ( x + dx ) T ( x ) = xr T dx T ( x + dx ) = T ( x ) + xr T dx
r
dT = x T dx

En componente la expresin anterior queda:

r
r
r
Tij ( x + dx ) = Tij ( x ) + Tij ,k dx k
r
= Tij ( x ) + Tij ,1 dx1 + Tij , 2 dx 2 + Tij ,3 dx3
Tij
Tij
r Tij
dx1 +
dx 2 +
dx3
= Tij ( x ) +
x1
x 2
x 3

La representacin de las componentes del campo Tij ( x + dx ) se pueden apreciar en la Figura


r

1.11. Observar que en la cara normal a x1 + dx1 actan las componentes Ti1 ( x ) +

Ti1
dx1 , ya
x1

que segn nuestra convencin el primer ndice indica la direccin haca donde apunta y el
segundo ndice indica el plano normal.
r

Una vez establecido el campo de Tij ( x + dx ) en el elemento diferencial de volumen,


r
r
aplicamos el balance total de las componentes del campo Tij ( x + dx ) segn las direcciones x1 ,
x 2 , x3 .
r

Balance total de Tij ( x + dx ) en dV segn direccin x1 es igual a cero (no hay fuente):

T
T
T
T11 + 11 dx1 dx 2 dx3 + T13 + 13 dx 3 dx1 dx 2 + T12 + 12 dx 2 dx1 dx3 T11 dx 2 dx3
x1
x3
x 2

T13 dx1 dx 2 T12 dx1 dx3 = 0

Simplificando la expresin anterior obtenemos que:


T
T11
T
dx1 dx 2 dx3 + 13 dx3 dx1 dx 2 + 12 dx 2 dx1 dx3 = 0
x 2
x1
x3

T11 T12 T13


+
+
=0
x1
x 2
x3

Anlogamente segn las direcciones x 2 y x3 vamos obtener, respectivamente, que:


T21 T22 T23
+
+
=0
x1
x 2
x3

Universidad de Castilla- La Mancha


Ciudad Real - Espaa

Draft

T31 T32 T33


+
+
=0
x1
x 2
x3

Por: Eduardo W. V. Chaves (2014)

MECNICA DEL MEDIO CONTINUO: PROBLEMAS RESUELTOS

120

x3

Cara oculta

T11
T33 +

Cara oculta

T33
dx3
x3
T23 +

T13 +

T13
dx3
x3

T21
T23
dx3
x3

T32 +

T12

T22

T31 +

T31
dx1
x1

T32
T11 +

T12 +
T21 +

T11
dx1
x1

T21
dx1
x1

T31
dx 3

T32
dx2
x2

T12
dx2
x2

T22 +

T22
dx2
x2

x2

dx1

T13
T23

x1

Cara oculta

T33
dx 2

Figura 1.11: Componentes del campo tensorial en un elemento diferencial de volumen.


Luego, tenemos el siguiente conjunto de ecuaciones que hay que cumplir simultneamente:
T11 T12 T13
+
+
=0

x
x
x
1
2
3

T21 T22 T23


+
+
=0

x
x
x
1
2
3

T31 T32 T33


+
+
=0

x1
x 2
x3

T11,1 + T12, 2 + T13,3 = 0

T21,1 + T22, 2 + T23,3 = 0

T31,1 + T32, 2 + T33,3 = 0

T1 j , j = 0

T2 j , j = 0

T3 j , j = 0

Tij , j = 0 i

Demostrando as que en la ausencia de fuente la divergencia es igual a cero:


Tij , j = 0 i

( xr T ) i = 0 i

tensorial

r
xr T = 0

NOTA 1: Si tenemos un campo tensorial, el orden de la fuente (o sumidero) es de un orden


menor que el campo tensorial, e.g. si el campo tensorial es un vector, la fuente de este campo
tensorial ser un escalar.
NOTA 2: Si la divergencia de un campo tensorial es positiva indica que tenemos una fuente
del campo tensorial, caso contrario, si la divergencia es negativa tenemos un sumidero.

Universidad de Castilla- La Mancha


Ciudad Real - Espaa

Draft

Por: Eduardo W. V. Chaves (2014)

1 TENSORES

121

Ejemplo 1.120
Demostrar que:

[( xr T ) ur ] ar = [ xr ( T ar )] ur

(1.136)

r
r r r
donde T = T (x ) es un campo tensorial de segundo orden, u = u( x ) es un campo vectorial, y
r
r
a es un vector arbitrario (independiente de ( x ) ).

Solucin:

r r

Observar que la operacin [( xr T ) u] a resulta un vector, que en notacin indicial queda:

{[( xr T ) ur ] ar}i = [( xr T ) ur ]ik (ar ) k


r

= ( xr T ) ikp u p a k = Tik , p u p a k = Tik , p u p a k

(1.137)

El trmino [ xr (T a )] u en notacin indicial queda:


r
r
r
gradiente
( T a ) i = Tik ak

[ xr ( T a )] ij = ( T a ) i , j = ( Tik ak ), j
r
[ xr ( T a )] ij = ( Tik ak ), j = Tik , j ak + Tik ak , j = Tik , j ak
{

r
r
[ xr ( T a )] ij = a ( xr T T )

=0 k , j

ij

r
r
+ [ T ( xr a )]ij = a ( xr T T )
123

ij

(1.138)

= Tik , j ak

=0

r
r
donde hemos considerado que a es independiente de ( x ) . Haciendo el producto escalar de la
r
ecuacin anterior con u obtenemos que:
{[ xr ( T ar )] ur}i = [ xr ( T ar )]ij u j = Tik , j a k u j = Tik , p u p a k
(1.139)

Si comparamos (1.137) con (1.139) demostramos (1.136).


r

r r

Notar que, si a = a (x ) es dependiente de x y segn la ecuacin (1.138) obtenemos que:

[ xr ( T ar )]ij = ( Tik ak ), j = Tik , j ak + Tik ak , j


r
r
r
[ xr ( T a )] ij = [a ( xr T T )] ij + [ T ( xr a )] ij
Ejemplo 1.121

Demostrar que si el mdulo de un vector, = (t ) , es constante en el tiempo eso implica que


r
r
d
es ortogonal a
para todo tiempo t .
dt

r r

Solucin: Partimos de la definicin del mdulo de un vector: = , con lo cual:


2

( ) = d (r r ) = d (r ) r + r d (r ) = 2r d (r ) = 0

r
d

dt

dt

dt

dt

dt

r
r d

dt

NOTA: Un caso particular de este ejemplo es el movimiento circular donde:


r
v

r
x = constante
r
r dx

r
x
r dx
dt
v=

dt

r
x

Universidad de Castilla- La Mancha


Ciudad Real - Espaa

Draft

Por: Eduardo W. V. Chaves (2014)

MECNICA DEL MEDIO CONTINUO: PROBLEMAS RESUELTOS

122

1.16 Teoremas con Integrales


Ejemplo 1.122

Comprobar el Teorema de la divergencia (Teorema de Gauss) para el campo vectorial F


cuyas componentes Cartesianas viene dadas por Fi = xi + ( x32 x 3 ) i 3 . Considerar la frontera
definida por el cilindro x12 + x 22 1 , 0 x3 1 .
Solucin:
El Teorema de la divergencia afirma que:

r
r
xr F dV = F n dS

donde n es la normal a la superficie y apunta hacia fuera.


x3

x12 + x 22 1

S (2)

n ( 2 )

r
r

r
r =1
h =1

S (1)

n (1)

r
x

x2
n (3)

x1

S ( 3)

Figura 1.12.
r

Clculo de xr F dV :
V

r
xr F = Fi ,i = xi + ( x 32 x3 ) i 3

,i

= xi ,i + ( x32 x3 ) ,i i 3 = ii + ( x32 x3 ) ,3

= 3 + (2 x3 1) = 2 x3 + 2

Luego:

xr

r
F dV = (2 x3 + 2) dV =

x3 =1

A x =(02 x

+ 2)dx3 dA = 3 dA = 3(r 2 ) = 3
A

donde A viene definido por el crculo x12 + x 22 1 .


r

Clculo de F n dS
S

Universidad de Castilla- La Mancha


Ciudad Real - Espaa

Draft

Por: Eduardo W. V. Chaves (2014)

1 TENSORES

123

Separamos la frontera en tres superficies: S (1) , S ( 2) , S (3) , ver Figura 1.12. Luego,
r

F n dS = F n

(1)

dS (1) +

S ( 1)

F n

( 2)

dS ( 2 ) +

S ( 2)

F n

( 3)

dS (3)

S ( 3)

r
F

explcitas de
son:
F1 = x1 + ( x32 x3 ) 13 = x1 ,
F2 = x 2 ,
r
2
2
F3 = x3 + ( x3 x 3 ) 33 = x3 . La representacin de F en la base Cartesiana viene dada por:
r
F = x1e 1 + x 2 e 2 + x32 e 3 . Las normales correspondientes a cada superficie vienen definidas a
continuacin:
Las

componentes

r
n (1) // r

n (1) =

x12

x 22

( x1 e 1 + x 2 e 2 ) ; n ( 2 ) = e 3 ; n (3) = e 3

En la superficie S (1) se cumple que:

r
F n (1) dS (1) =

S ( 1)

+ x 2 e 2 + x 32 e 3 )

x 22

( x e
1

1
x12

S ( 1)

( 1)

x12
x12

x 22

dS (1) =

1dS

(1)

x 22

( x1 e 1 + x 2 e 2 )dS (1)

= 2rh = 2

(1)

donde hemos considerado el rea del cilindro ( 2rh = 2 ).


En la superficie S ( 2) se cumple que x3 = 1 :
r

F n

( 2)

( x e

dS ( 2 ) =

S (2)

+ x 2 e 2 + 1e 3 ) (e 3 ) dS ( 2 ) =

S (2)

1dS

(2)

= r 2 =

S (2)

donde hemos considerado el rea del crculo ( r 2 = ).


En la superficie S (3) se cumple que x3 = 0 :
r

F n

( 3)

dS (3) =

( 3)

Con lo cual:

( x e
1

+ x 2 e 2 + 0e 3 ) (e 3 )dS (3) =

( 3)

r
F n dS =

r
F n (1) dS (1) +

S (1 )

0dS

( 3)

=0

(3)

r
F n ( 2 ) dS ( 2 ) +

S (2)

F n

( 3)

dS (3) = 3

S ( 3)

Luego, comprobando as el Teorema de la divergencia: xr F dV = F n dS = 3 .


V

Ejemplo 1.123
Sea un dominio de rea delimitado por el contorno como muestra en la Figura 1.13.
Considrese tambin que m es un campo tensorial de segundo orden y un campo escalar.
Demostrar que se cumple la siguiente relacin:

[m :

r
r
x ( x

[m

)]d = [( xr ) m] n d [( xr m) xr xr ]d

ij

, ij ] d = ( , i m ij )n j d [m ij , j , i ] d

Universidad de Castilla- La Mancha


Ciudad Real - Espaa

Draft

Por: Eduardo W. V. Chaves (2014)

MECNICA DEL MEDIO CONTINUO: PROBLEMAS RESUELTOS

124

x2

x1

Figura 1.13.

Solucin: Se puede aplicar directamente la definicin de integracin por partes para la


demostracin. Pero partiremos de la definicin del teorema de la divergencia. Luego dado un
r
tensor v se cumple que:

r
x

v d = v n d indicial
v j , j d = v j n j d

r
r
Pero si consideramos que el tensor v es el resultante de la operacin v = xr m y lo
equivalente en notacin indicial v j = , i m ij y reemplazndolo en la expresin anterior

obtenemos que:

j, j

d = v j n j d

[ ,

[ ,

ij

[ ,

ij

m ij

,j

dV = , i m ij n j d

m ij + , i m ij , j d = , i m ij n j d

m ij d = , i m ij n j d

[ ,

m ij , j d

Lo equivalente en notacin tensorial:

[m :

r
r
x ( x

)]d = [( xr ) m] n d [ xr ( xr m)]d

Q.E.D.
NOTA: Si consideramos ahora un dominio de volumen V delimitado por una superficie S
r
con normal n y sea N un vector y T un escalar tambin se cumple que:

N T,
i

ij

dV = N i T , i n j dS N i , j T , i dV

r
r
r
N xr ( xr T )dV = ( xr T N ) n dS xr T xr NdV

donde hemos aplicado directamente la definicin de integracin por partes.

Universidad de Castilla- La Mancha


Ciudad Real - Espaa

Draft

Por: Eduardo W. V. Chaves (2014)

1 TENSORES

Ejemplo 1.124

125

Si un campo vectorial se define como: b = xr v , probar que:

b n
i

d S = , i ijk v k , j dV = , i b i dV

donde es una funcin nicamente de x , i.e., = ( x ) .


r

Solucin1: Si b = xr v , luego b i = ijk v k , j . Reemplazando en la integral de superficie anterior


resulta:

b n
i

dS = ijk v k , j n i dS

Aplicando el teorema de la divergencia de Gauss, resulta:

b n
i

dS = ijk v k , j n i dS

= ( ijk v k , j ), i dV

= ( ijk , i v k , j + ijk v k , ji ) dV
V

= (, i ijk v k , j + ijk v k , ji ) dV = , i b i dV
1
424
3
1
424
3
V

bi

Q.E.D.

Solucin 2:

b n
i

dS = (b i ), i dV = (, i b i + b i , i ) dV

como b i = ijk v k , j b i ,i = ijk v k , ji = ijk v k ,ij = 0

b n
i

dS = , i b i dV = , i ijk v k , j dV

Ejemplo 1.125
Sea un dominio de volumen V delimitado por la superficie S . a) Demostrar que:
r

( x n + n x) dS = 2V 1
S

donde n es el versor normal exterior a la superficie S . b) Demostrar tambin que:

r
x

) x dV

r
= ( n ) x dS dV

ik , k

x j dV = ik n k x j dS ij dV

y
r

x (

r
x

) dV

r
= x ( n ) dS T dV

x
i

jk , k

dV = x i jk n k dS ji dV
S

donde es un tensor de segundo orden arbitrario.

Universidad de Castilla- La Mancha


Ciudad Real - Espaa

Draft

Por: Eduardo W. V. Chaves (2014)

MECNICA DEL MEDIO CONTINUO: PROBLEMAS RESUELTOS

126

r
dS = n dS

x2

dS

B
r
x

x1
x3

Solucin:
a) Teniendo en cuenta solo el primer trmino del integrando, podemos decir que:
r

( x n ) dS = ( x 1 n ) dS = ( x 1) n dS
S

Aplicando el teorema de la divergencia obtenemos que:


r

( x n ) dS = ( x 1) n dS =
S

r
x

( x 1) dV

Seguiremos el desarrollo en notacin indicial:

x n
i

dS = x i

dS = (

jk n k

jk

x i ) , k dV = (

jk , k

xi +

jk

x i , k ) dV

Teniendo en cuenta que jk ,k = 0 j , xi ,k = ik , concluimos que:

x n
i

( x n ) dS = V 1

dS = ji dV = ji dV = ji V

= V1

(1.140)

Anlogamente, concluimos que (n x ) dS = V 1 . Con lo cual es cierto que:


S

( x n + n x) dS = 2V 1
S

b) Verifiquemos que se cumple que


( x j ik ) , k = x j , k ik + x j ik ,k
{
=

x j ik ,k = ( x j ik ) ,k ij

jk

r
r
( xr ) x = xr ( x )

Con eso podemos decir que:

Universidad de Castilla- La Mancha


Ciudad Real - Espaa

Draft

Por: Eduardo W. V. Chaves (2014)

1 TENSORES

r
x

r
x

127

) x dV = xr ( x ) dV dV
V

ik , k

ik , k

dV = x j ik n k dS ij dV

r
= ( n ) x dS dV

dV = ( x j ik ) ,k dV ij dV

) x dV = ( x ) n dS dV
S

x
j

= ( ik n k ) x j dS ij dV

donde hemos aplicado el teorema de la divergencia a la primera integral del lado derecho de la
igualdad.
Teniendo en cuenta que

[( xr ) xr ]T = [ xr ( xr ) ]T

r
r T
x ( xr ) = [ xr ( x )] T

En notacin indicial xi jk ,k = ( xi jk ) ,k ji .
Con eso podemos decir que:
r

x (

r
x

x (

) dV = [ xr ( x )]T

dV T dV

jk , k

dV

jk , k

r
x

) dV = ( x ) n dS T
S

dV = ( xi jk )n k dS ji dV

r
= x ( n ) dS T dV

dV = ( xi jk ) , k dV ji dV

= xi ( jk n k ) dS ji dV

NOTA: Si obtenemos la traza de la ecuacin (1.140) podemos tambin concluir que:

x n
i

( x n ) : 1 dS = ( x n ) dS = V 1 : 1

dS = ji jiV = iiV

(1.141)

Si estamos en el espacio tridimensional (3D) la traza ii = 3 , si estamos en el plano (2D)


tenemos que ii = 2 . Con eso podemos concluir que:

x n
i

( x n ) dS = 3V

dS = 3V

(3D)

( x n ) d = 2 A

x n d = 2 A

i i

(2D)

x2

x1

A : rea del dominio

Figura 1.14: Plano 2D.


Universidad de Castilla- La Mancha
Ciudad Real - Espaa

Draft

Por: Eduardo W. V. Chaves (2014)

MECNICA DEL MEDIO CONTINUO: PROBLEMAS RESUELTOS

128

Ejemplo 1.126
Sea un escalar que viene dado por:
GM
a

= r

donde G y M son escalares y constantes, y a es el mdulo del vector a 0 . a) Obtener el


r r
gradiente de . b) Obtener el gradiente de para el caso particular cuando a = x y dibujar el
campo xr en el espacio Cartesiano.
Solucin:
GM
,i = r
a
x ,i

( xr ),i r

= GM 1 ( ar ) ,i

ar 2
,i

(1.142)

Notar que:
1
1
r r 1
r
1 r r 2 r r
1 r r 2
2
( a ) ,i = ( a a ) = (a a ) ( a a ) ,i = ( a a ) ( a k a k ) ,i
2

,i 2
1

r r
1 r r 2
1
(a a ) ( a k ,i a k + a k a k ,i ) = (a a ) 2 ( a k ,i a k ) = r ( a k , i a k )
a
2

o en notacin tensorial:
r
r
1 r
xr ( a ) = r (a xr a )
a

(1.143)

Luego, la expresin (1.142) queda:


1 1
1 r
GM
,i = GM r 2 ( a ) ,i = GM r 2 r (ak ,i ak ) = r 3 (ak ,i ak )
a a
a
x ,i
a

r
GM r
= r 3 (a xr a ) i
a

( xr ),i r

(1.144)

r
a
a

Adems, teniendo en cuenta que el versor segn la direccin de a viene dado por a = r , an
podemos decir que:
r
r
r
GM
r
r
( xr ),i = GM
r 3 (a x a ) i = r 2 (a x a ) i

(1.145)

a
a
r r
b) Para el caso particular cuando a = x tenemos que:

r
r
1
1
1
( x ) ,i = r ( x k ,i x k ) = r ( ki x k ) = r ( xi ) donde r = x = x12 + x 22 + x32
x
x
x

o en notacin tensorial:
r
r
1 r
1 r
1 r
xr ( x ) = r ( x xr x ) = r ( x 1) = r ( x ) = x
x
x
x

Con lo cual

Universidad de Castilla- La Mancha


Ciudad Real - Espaa

Draft

Por: Eduardo W. V. Chaves (2014)

1 TENSORES

GM
,i = r
x
x i

( xr )i r

129

= GM 1

xr 2
,i

r
( x ) = GM ( xr )
i
,i
r3

(1.146)

o en notacin tensorial:
GM
xr = xr r
x

GM r GM
=
x = r 2 x
xr 3
x

(1.147)

Observar que el campo vectorial xr es radial, i.e. es normal a las superficies de las esferas
r

definidas por x y disminuye con x = r 2 , (ver Figura 1.15).


x3

Esferas
x

xr

x = 1

r
x

r
b

x1

x2
xr

xr

Figura 1.15
La ecuacin (1.147) tambin puede ser reescrita como:
r
GM GM GM (r )
= b =
r = (r )r
r =
= 2 r =
r r
r
r r

(1.148)

GM
representa el potencial gravitacional que tiene la siguiente
x

NOTA: Este ejemplo = r


r

propiedad b = xr , ver Figura 1.15, donde G = 6,67384 10 11

m3
kg s 2

es la constante

gravitacional, M es la masa total del planeta. Verificamos las unidades:

Universidad de Castilla- La Mancha


Ciudad Real - Espaa

Draft

Por: Eduardo W. V. Chaves (2014)

MECNICA DEL MEDIO CONTINUO: PROBLEMAS RESUELTOS

130

[] = GM
r =

m 3 kg kg m m N m J
( Unidad de energa por unidad de masa )
= 2
=
=
kg
kg
kg s 2 m
s kg
(energa especfica)

[br ]= [ ] = xr = mJkg = mN kgm = skgmkg = sm (Unidad(unidad


de fuerza por unidad de masa)
de aceleracin)
r
x

Es interesante comprobar tambin que xr b = xr [ xr ] = 0 , ver Ejemplo 1.107.


r

GM

Podemos obtener b en la superficie de la Tierra a travs de b = xr = r

x , donde la

masa total de la Tierra es M 5,98 10 24 kg y el radio aproximado R 6,37 10 6 m ,


resultando
r
GM
GM
b = r 2 x = 2 x 9,82 x
R
x
r

su mdulo denotamos por g = b 9,82

m
.
s2

Adoptando por x el sistema que tiene su origen en el centro del cuerpo de masa M , e
r
r
invocando la ley de Newton ( F = ma ), podemos obtener la fuerza que est sometido un
r
cuerpo de masa ( m ) que se encuentra bajo la influencia del campo gravitacional b = xr :
r
r
r
GMm
F = ma = mb = r 2 x
x

(1.149)

Podemos expresar la relacin anterior en un sistema genrico tal y como se indica en la Figura
1.16.
x 2

x1

M
x3

r
x (M )
x2

x3

r
x

r
F ( Mm )

r
F ( mM )

r
x (m )

r
r r
x ( M ) + x = x ( m)
r r
r
x = x (m) x (M )
x1

Figura 1.16

Universidad de Castilla- La Mancha


Ciudad Real - Espaa

Draft

Por: Eduardo W. V. Chaves (2014)

1 TENSORES

131

Luego, para el sistema x la fuerza viene dada por:


r
F ( mM ) =

GMm
r ( m) r ( M )
x x

r
r
( x ( m) x ( M ) )
r
r
x ( m) x ( M )

Ley de gravitacin universal de


Newton

(1.150)

donde utilizamos la nomenclatura F (mM ) para indicar que es la fuerza en m debido a la


influencia de M . Observar tambin que en M tenemos la misma fuerza en mdulo y
r
direccin, pero de sentido contrario F (Mm ) .
Ejemplo 1.127
Considerando que =

r
1
donde r = x = x12 + x 22 + x32 , se pide:
r

a) Demostrar que:

r r
2 2 2
xr xr ( x 0) 2 2 + 2 + 2 = 0 Ecuacin de Laplace
(1.151)
x1
x 2
x3
r r
para r 0 . Utilizamos la nomenclatura xr ( x 0) para indicar que el origen no est

incluido.
b) Dada una superficie cerrada S que contiene el origen, demostrar que:

( ) n dS = 4
r
x

(1.152)

donde n es el versor normal a la superficie.


Solucin:
Fue obtenido en el Ejemplo 1.126 que
GM
xr = xr r
x

GM r GM
=
x = r 2 x
xr 3
x

(1.153)

Haciendo que GM = 1 obtenemos que:


1 1 r 1
xr = xr r = r 3 x = r 2 x
x x
x

(1.154)

1 r
1
( xr ) i = r 3 x = r 3 xi

x
x
i

(1.155)

o en notacin indicial:

Calculando la divergencia de la relacin anterior quedamos con:


x
xr xr = ,ii = r 3i
x

= x i ,i x 1 = x i ,i x 3 ( xr )
i
i
,
i
r 3
r 3
r
xr 3

x 4
x
x

,i

,i
r

1
x

(1.156)

En el Ejemplo 1.126 hemos demostrado que xr ( x ) = r ( x ) y adems teniendo en cuenta


que xi ,i = ii = 3 , podemos decir que:
Universidad de Castilla- La Mancha
Ciudad Real - Espaa

Draft

Por: Eduardo W. V. Chaves (2014)

MECNICA DEL MEDIO CONTINUO: PROBLEMAS RESUELTOS

132

xr

( xr ) =

3 r
3
r 4 ( x ) ,i

x
i
r3
x

3 x
3
= r 3 xi r 4 ri
x
x
x

x
x
3
3
= r 3 + ri 5i
x
x
r 2
3x
3
= r 3 + r 5 =0
x
x

(1.157)

c) Adoptamos una esfera arbitraria de radio r cuya rea de la superficie es 4r 2 . Luego:

n dS = 1 x n dS = 1

x
r 2
r 2
2

x S
x
S x

1
1
= 2 ( rea) = 2 (4r 2 ) = 4
r
r

1
( ) n dS = r
r
x

dS
S

(1.158)

Observar que x n = 1 ya que para la esfera se cumple que x // n .


Es interesante verificar que a travs del teorema de la divergencia hay que cumplir que:

[ ]dV = ( ) n dS
r
x

r
x

r
x

,ii dV

= ,i ni dS
S

(1.159)

Hemos demostrado anteriormente que xr xr ( x 0) = 0 , pero eso solo es vlido para

r r
todo x 0 (no est incluido el origen). Es decir, teniendo en cuenta el resultado (1.158), y
r r
para que (1.159) tenga consistencia, en x = 0 tenemos una fuente (manantial o sumidero) e
igual a ( 4 ). Con eso es muy intuitivo concluir que cualquier superficie cerrada que no
contenga el origen se cumple que xr n dS = 0 , (ver Parker (2003)).

Ejemplo 1.128
a) Demostrar que:

( ) n dS = 4GM (r )
S

(1.160)

GM
es el potencial gravitacional, y M (r ) es la masa total contenida en la
r
esfera de radio r , donde la superficie de contorno de la esfera denotamos por S .

donde =

b) Considerando un planeta que tiene forma de esfera de radio r = a , obtener la masa total del
planeta en funcin de la densidad de masa, donde la densidad de masa es funcin del radio, i.e.
= (r ) .
c) Obtener el potencial gravitacional para r < a y r a . En este apartado considerar la
densidad de masa uniforme en el planeta = 0 .

Universidad de Castilla- La Mancha


Ciudad Real - Espaa

Draft

Por: Eduardo W. V. Chaves (2014)

1 TENSORES

133

Solucin:
a) En el Ejemplo 1.127 hemos demostrado que:

( ) n dS = r n dS = 4
1

(1.161)

Multiplicamos ambos lados de la igualdad por GM (r ) obtenemos que:


1
GM (r ) n dS = 4GM ( r )
r
S

GM ( r )
n dS = 4GM (r )
r

[ ] n dS = 4GM (r )

(1.162)

b)
Planeta esfrico

r=a

(r )

La masa total viene definida por:

M = (r )dV

(1.163)

Notar que V = 43 r 3 dV = 43 3r 2 dr = 4r 2 dr . Con lo cual:

M = (r )dV =
V

r =a

(r )4r

dr

(1.164)

r =0

c) Recordar que en el Ejemplo 1.126 (ver ecuacin (1.148)) hemos obtenido que
r
GM GM GM (r )
= b =
r = (r )r
= 2 r =
r =
r r
r
r r

(1.165)

Utilizando la ecuacin (1.162) podemos decir que:

[ ] n dS = 4GM (r )
S

r
b n dS = (r ) r n dS = (r ) dS = (r )(4r 2 ) = 4GM (r )
123

(r )r 2 = GM (r )

Universidad de Castilla- La Mancha


Ciudad Real - Espaa

=1

(r ) =

(1.166)

GM (r )

Draft

r2
Por: Eduardo W. V. Chaves (2014)

MECNICA DEL MEDIO CONTINUO: PROBLEMAS RESUELTOS

134

donde M (r ) = V 0 = 43 r 3 0 . Luego:

(r ) =

GM (r )
r

4G 0
r
3

d (r ) 4G 0
=
r
dr
3

d ( r ) =

4G 0
rdr
3

(1.167)

Integrando la ecuacin anterior obtenemos que:

d =

4G 0
rdr
3

(r ) =

4G 0 r 2
+C
3
2

(1) (r ) =

2G 0 2
r +C
3

(1.168)

donde hemos denotado que (r ) = (1) (r ) para r < a . Para valores de r a el potencial
gravitacional viene dado por

GM 4Ga 3 0
=
= (2)
r
3r

(1.169)

ra

donde M es la masa total del planeta cuyo valor es M = V 0 = 43 a 3 0 . Notar que el potencial
tiene que ser continuo en r = a , (ver Parker (2003)), con lo cual:

(1) (r = a) = ( 2) (r = a)
4Ga 3 0
2G 0 2

a +C =

3a
2Ga 0 2Ga 3 0 4 3 2GM 3 3MG
C =
=
=
=
a
a
34
a
4
2a

(1.170)

Con lo cual la ecuacin (1.168) queda

(1) (r ) =

2G 0 2
2G 0 2 3MG MG 2 3MG MG r 2
3
r +C =
r
= 3r
= 2 2
3
3
2a
2a
2
2a
2a 2a

(1.171)

resumimos:

(r ) =

(r ) =

Universidad de Castilla- La Mancha


Ciudad Real - Espaa

MG r 2
3


para
2
2
2
2a 2a
MG
para
ra
r

Draft

r<a

(1.172)

Por: Eduardo W. V. Chaves (2014)

1 TENSORES

135

(r )
superficie del planeta
a
r

MG
a

punto de inflexin
3MG
2a

Figura 1.17: Potencial gravitacional vs. radio.

Figura 1.18: Potencial gravitacional (Ref. Wikipedia: Gravitational potential).


Ejemplo 1.129
a) Demostrar que la rbita de un planeta se realiza en un plano. b) Demostrar las Leyes de
Kepler, (Johannes Kepler (1571-1630):
b.1) Primera Ley de Kepler (1609): La rbita de cada planeta es una elipse, teniendo el Sol en unos
de los focos de la elipse;
b.2) Segunda Ley de Kepler (1609): El vector posicin Sol=>Planeta en movimiento describe un
rea a una tasa constante;
b.3) Tercera Ley de Kepler (1618): Si T (periodo orbital) representa el tiempo necesario para que
un planeta realice una vez su rbita elptica, cuyo eje mayor es 2a , se cumple que T 2 = a 3 ,
donde es una constante.
Universidad de Castilla- La Mancha
Ciudad Real - Espaa

Draft

Por: Eduardo W. V. Chaves (2014)

MECNICA DEL MEDIO CONTINUO: PROBLEMAS RESUELTOS

136

Recordatorio: Expresiones relacionados con la elipse


x2
r
x

f2

f1

x1

Ecuacin de la elipse: x = r =
Excentricidad: e =

a 2 b2
a2

p
1 + e cos
;

0 < e < 1 , donde se cumple que a 2 =

p2
.
(1 e 2 ) 2

rea de la elipse: A = ab
Solucin:
M - masa del Sol
m - masa del planeta
r
x
x = r
x

r
r dx
v=
,
dt
r
a // x

x2
x3

r
x
Sol

r r r
c = xv

r
dx r
=v
dt

r
r
r d 2 x dv
a= 2 =
dt
dt

r r
F // a

t=0

r
h

x1

Figura 1.19: rbita del planeta.


a) Para demostrar que la rbita se realiza en un plano tenemos que demostrar que la normal
r
r
formada por el plano x y v no cambia con el tiempo, es decir, si el vector c no vara con el
tiempo, i.e. es una constante.
En el Ejemplo 1.126 en la ecuacin (1.149) hemos demostrado que:

Universidad de Castilla- La Mancha


Ciudad Real - Espaa

Draft

Por: Eduardo W. V. Chaves (2014)

1 TENSORES

137

r
r
r
r
GMm
GM
;
F = ma = mb = r 2 x
a = r 2 x
(1.173)
x
x
r r r
Obtenemos la tasa del vector c = x v :
r
r r d r r r r r r
dc d r r
d r
= ( x v ) = ( x ) v + x (v ) = v12
3v + 1
x2
3
a =0
r
r
dt dt
dt
dt
=0
=0
r r r
Con lo cual hemos demostrado que el vector c = x v no vara con el tiempo, implicando

que la rbita se realiza en un plano.


b.1) Primera Ley de Kepler
Ya que la rbita del planeta se realiza en un plano, adoptamos como dicho plano el x1 x 2 ,
r
luego el vector c tiene misma direccin que x3 , ver Figura 1.19.
r

Expresamos c en funcin de x .
r
r
d( x )
r dx d r
r dx

= ( x x) =
v=
x + x
dt dt
dt
dt

y
r
r
d( x )
r d( x )
r r r
r
r
r 2
r 2
x
d
dx
dx
= x
3
= x x
c = x v = ( x x )
x + x
x 2
x + x x
1
dt
r
dt
dt
dt
dt

=0

GM

Teniendo en cuenta que a = r


r
r r r
que v , i.e. (a c ) // v :

r r
x , calculamos el vector a c que tiene misma direccin

r r GM r 2
dx
dx
dx
dx

a c = r 2 x x x
= GM x x
= GM ( x ) x ( x x )
x

dt
dt
dt
dt

dx
= GM
dt
r r r
r r r r r r
donde hemos utilizando la propiedad a b c = (a c )b (a b)c , ver Ejemplo 1.16. Notar
dx
dx
2
siempre se cumple, y x x = x = 1 .
tambin que se cumple que x = 0 ya que x
dt
dt
Teniendo en cuenta que GM es una constante, tambin se cumple que:

r r
dx d (GM x )
a c = GM
=
dt
dt

Ya que el vector c no vara con el tiempo, tambin se cumple que:


r r
r
r r dv r d ( v c )
ac =
c =
dt
dt

Luego:

r r
d (v c ) d (GM x )
=
dt
dt

Integrando en el tiempo la ecuacin anterior obtenemos que:


r
r r
v c = GM x + h

Universidad de Castilla- La Mancha


Ciudad Real - Espaa

Draft

Por: Eduardo W. V. Chaves (2014)

MECNICA DEL MEDIO CONTINUO: PROBLEMAS RESUELTOS

138

donde h es el vector constante de integracin y no depende del tiempo. Notar que h se


r r
encuentra en el plano x1 x 2 , ya que (v c ) y x tambin se encuentran en el plano x1 x 2 ,
ver Figura 1.19. Calculamos entonces:
r
r
x h = x h cos = h cos
r

donde hemos denotado por h = h . Con lo cual:


r
c2 = c

r r
r r r r r r
= c c = ( x v ) c = (v c ) x
r
r
r
r r
r
r
= GM x + h ( x x ) = x GM x x + x h x = x GM + x h cos
r
= x (GM + h cos ) = r (GM + h cos )
r
donde hemos considerado que r = x . Luego, obtenemos la siguiente ecuacin de la elipse:
2

c2
p
c
GM
r=
=
=
(GM + h cos ) (GM + h cos ) 1 + e cos
GM
2

donde hemos considerado que:


p=

c2
GM

e=

h
GM

(1.174)

b.2) Segunda Ley de Kepler


r 1 r
A = x S
2

S 0

r 1 r
r
dA = x ds
2

x2

S
A
r
x
x1

La tasa de dA queda:

r
r
r
r
r
r
r 1 r D ( ds ) 1 D ( x )
r 1 r r 1r
D (dA) 1 D ( x ds ) 1 D ( x )
=
=
ds + x
=
ds + x v = c (constante)
2
2 Dt
2
2 1Dt
2
Dt
Dt
Dt
42r 43 2
=0

y su mdulo:
r
D (dA) D(dA) 1 r 1
=
= c = c
Dt
Dt
2
2

Universidad de Castilla- La Mancha


Ciudad Real - Espaa

Draft

Por: Eduardo W. V. Chaves (2014)

1 TENSORES

139

NOTA: Como consecuencia de la segunda ley tenemos que si las reas de dos sectores son
iguales el tiempo necesario para recoger sus trayectos sern iguales. Luego, segn la Figura
1.20 si las reas de los sectores OCD y EFO son iguales los tiempos recogido de C D y
E F tambin sern iguales. Como consecuencia, el planeta cuando est ms cerca del Sol
tendr mayor velocidad que cuando est ms alejado.
sector EFO

sector OCD
E
D

A
C

Figura 1.20: rbita del planeta.


b.3) Tercera Ley de Kepler
Si T es el tiempo total para una rbita completa (periodo orbital), obtenemos que:
T

D (dA)
1
1
A=
dt =
c dt = cT
2
Dt
2
0
0

Teniendo en cuenta el rea de la elipse: A = ab , concluimos que


T=

2ab
c

T2 =

1
cT = ab , con lo cual:
2

4 2 a 2 b 2
c2

(1.175)

A travs de las relaciones de la elipse se cumple que:


e=

a 2 b2
a2

b2 = a 2 a 2e2

y teniendo en cuenta que a 2 =

b 2 = a 2 (1 e 2 )

p2
p
a=
(1 e 2 )a = p en la relacin anterior,
2 2
2
(1 e )
(1 e )

podemos decir que:


b 2 = a 2 (1 e 2 )

b 2 = ap

p=

b2
a

Con lo cual la ecuacin (1.175) puede ser reescrita como:


T2 =

4 2 a 2 b 2 4 2 a 2 ab 2 4 2 a 3 p 4 2 3
=
=
=
a = a3
GM
c2
c 2a
c2

donde hemos tenido en cuenta que

Universidad de Castilla- La Mancha


Ciudad Real - Espaa

(1.176)

p
1
=
, ver ecuacin (1.174).
2
GM
c

Draft

Por: Eduardo W. V. Chaves (2014)

MECNICA DEL MEDIO CONTINUO: PROBLEMAS RESUELTOS

140

NOTA COMPLEMENTARIA 1

Propiedades Geomtricas de Curvas


Vamos considerar una curva descrita por la funcin y = y ( x) , ver Figura 1.21.

y ( x)

y
x
s 2 = x 2 + y 2
y

Figura 1.21
Primera derivada:

dy ( x)
y y, x (tangente a la curva en el punto x )
dx

Segunda derivada:

d 2 y ( x)
y y, xx
dx 2

Longitud de arco infinitesimal ds :


Segn la Figura 1.21 podemos obtener que:
(x 2 + y 2 )
x 2
y
x = 1 +
s = x + y = (x + y ) 2 =
2
x
x
x
2

Luego, definimos la longitud de arco infinitesimal como:


2
2

y
dy
ds = lim 1 + x = 1 + dx 1 + ( y ) 2 dx = 1 + ( y ) 2
x 0

x
dx

ds
= 1 + ( y ) 2
dx

1
2

dx

1
2

Curvatura
La curvatura mide la rapidez que la direccin s cambia con respecto al cambio de la longitud
de arco s , donde s es el vector unitario (versor) segn la direccin ( y ) . Luego, definimos el
vector curvatura como:
r

ds
ds

curvatura

r
ds
= =
ds

donde ( x) es la curvatura de la curva en el punto x .


Universidad de Castilla- La Mancha
Ciudad Real - Espaa

Draft

Por: Eduardo W. V. Chaves (2014)

1 TENSORES

141

Vamos considerar que existe un ngulo tal que tan( ) =

dy
y y si derivamos con
dx

respecto a x podemos obtener que:


d dy d
[tan( )] = d [tan( )] d = sec 2 ( ) d = 1 + tan 2 ( ) d
=
dx dx dx
d
dx
dx
dx

d2y
dx

y = 1 + tan 2 ( )

d
d
= 1 + ( y ) 2

dx
dx

] ddx = 1 + dy

dx

y
d
=
dx [1 + ( y ) 2 ]

Con lo cual la curvatura puede ser obtenida como sigue:


=

donde se cumple que

d d dx
y
=
=
ds
dx ds [1 + ( y) 2 ]

dx
=
ds

1
1
[1 + ( y) 2 ] 2

1
1
[1 + ( y) 2 ] 2

[1 + ( y) 2 ] 2

Notar que la curvatura de la circunferencia es constante, luego:


=

d
=
ds

ds = d

ds = d

integrando

2r = 2

ds = d
=

2 1
=
2r r

(1.177)

donde ( 2r ) es la longitud de la circunferencia de radio r .


Si consideramos la Figura 1.22 podemos concluir que la curvatura de la circunferencia de radio
r (1) es mayor do que la de radio r ( 2 ) :
r (1) < r ( 2 )

1
r

(1)

>

r ( 2)

(1) > ( 2 )

r ( 2)

r (1)

Figura 1.22
Universidad de Castilla- La Mancha
Ciudad Real - Espaa

Draft

Por: Eduardo W. V. Chaves (2014)

MECNICA DEL MEDIO CONTINUO: PROBLEMAS RESUELTOS

142

Una ecuacin interesante que podemos obtener de la ecuacin de curvatura (ver ecuacin
(1.177)), es:

ds = d = B A B _ A
A

B _ A

Area = ds = B A B _ A

Area

B
B

Figura 1.23
Por ejemplo, vamos considerar una circunferencia de radio r , y la variacin de ngulo de A a
B puede ser obtenido como sigue:
1
Area = ds = [(2r ) ]
4
1
1

= [(2r ) ] =
4
r
2
= B A B _ A

B _ A =

Figura 1.24

Universidad de Castilla- La Mancha


Ciudad Real - Espaa

Draft

Por: Eduardo W. V. Chaves (2014)

1 TENSORES

143

Vector Curvatura
ds
ds

donde s es el versor (tangente a la curva), y si utilizamos la propiedad del vector unitario,


podemos concluir que:
s = 1

= s s = 1

d (s s ) d (1)
=
=0
ds
ds
d (s ) d (s )
ds

s + s
= 2 s = 0
ds
ds
ds

y ( x)

s = 0

s
ds
ds

ds
ds

Figura 1.25
Relaciones geomtricas entres incrementos de coordenadas
r

r
La matriz de transformacin del sistema x al sistema n - s (cuyo sistema denotamos por x ),
ver Figura 1.26, viene dada por:

cos
aij =
sin

sin n 1 n 2
=

cos n 2 n 1

Luego se cumple que:


T

dxi = aij dx j

dxi = a ji dx j

n 2 0 n 1
dx n
1 = 1
=
dx2 n 2 n 1 ds n 2

n 2 0 n 2 ds

=
n 1 ds n 1ds

Con lo cual podemos obtener que:


dx2
n 1 ds
= dx
n 2 1
ds

Universidad de Castilla- La Mancha


Ciudad Real - Espaa

Draft

n i = 3ij

dx j
ds

Por: Eduardo W. V. Chaves (2014)

MECNICA DEL MEDIO CONTINUO: PROBLEMAS RESUELTOS

144

donde kij es el smbolo de permutacin.

x2

x2

x2

n i = 3ij

x1

ds

dx2

(i, j = 1,2)

ds

dx2
n 1 ds
n i = =

n 2 dx1
ds

dx j

x1

kij - smbolo de permutacin


ds - longitud de arco

dx1

x1

Figura 1.26
NOTA COMPLEMENTARIA 2

Centro Geomtrico (Centroide - C.Geo)


r

Dado un volumen V = dV delimitado por la superficie S , y los sistemas x y x , (ver Figura


V

1.27). A travs de suma de vectores podemos obtener que:


r r r
x = x + x

Al integrar en el volumen obtenemos:

r r
r
r
r
xdV = ( x + x )dV = xdV + x dV

El centroide del volumen ( x (V ) ) es el punto donde se cumple que:

r
r
x dV = 0

r r
x = x (V )

Luego, el centroide del volumen puede calculado como sigue:


r

r
r
r
r
r
xdV = x (V ) dV + x dV = x (V ) dV = x (V ) dV

V
12r3
=0

Universidad de Castilla- La Mancha


Ciudad Real - Espaa

xdV xdV

r
x (V ) = V

dV

=V

Draft

Por: Eduardo W. V. Chaves (2014)

1 TENSORES

145

x3
x2

dV

x3

r
x

x2
r
x

x1
V

r
x (V )

x1

Figura 1.27
Las componentes del centroide de volumen ( x1(V ) , x 2(V ) , x3(V ) ) son obtenidas como sigue:

x dV

x dV

x1(V )

x 2(V )

dV

x3(V )

dV

x dV
3

Centroide de volumen

dV

(1.178)

donde:

x dV

es el primer momento de volumen con respecto al eje x1 ;

x dV
2

es el primer momento de volumen con respecto al eje x 2 ;

x dV

es el primer momento de volumen con respecto al eje x 3 .

Notar que si el volumen est formado por V = V ( A) + V ( B ) tambin se cumple que:


r

xdV xdV

r
x (V ) = V

dV

=V

r
r
Vx (V ) = xdV =

r
r
Vx (V ) = xdV

xdV = xdV

V ( A ) +V ( B )

( A)

V ( A)

xdV

(B)

r A
r B
= V ( A) x (V ) + V ( B ) x (V )

V (B)

donde hemos utilizado la definicin:


r

xdV

r A
( A)
x (V ) = V

dV

( A)

V ( A)

Universidad de Castilla- La Mancha


Ciudad Real - Espaa

xdV

( A)

r B
(B)
x (V ) = V

dV

(B)

( B)

V (B)

Draft

Por: Eduardo W. V. Chaves (2014)

MECNICA DEL MEDIO CONTINUO: PROBLEMAS RESUELTOS

146

De forma anloga a la obtencin de las ecuaciones en (1.178) que definen el centroide de


volumen, tambin poder definir el centroide de rea:

x dA
1

x1( A)

x dA

x dA

x 2( A)

dA

x3( A)

dA

dA

Centroide de rea

(1.179)

Centroide de lnea

(1.180)

y el centroide de lnea:

x dL
1

x1( L )

x dL

x dL

x 2( L )

dL

x3( L )

dL

dL

Centro del Campo Escalar dentro de un Dominio


r

Si tenemos un campo escalar ( = ( x ) ), podemos obtener que:


r

x = x (V _ ) + x

r (V _ )

xdV = x

Integrando

r
dV + x dV

El centro del campo escalar = ( x ) dentro de un volumen V viene definido por:


r

x dV = 0

Con lo cual podemos obtener:


r

r
r
xdV = x (V _ ) dV

xdV xdV

r
x (V _ ) = V

dV

=V

V ( )

cuyas componentes son:

x dV
1

x1(V _ )

dV

x dV
2

x 2(V _ )

dV

x dV
3

x 3(V _ )

dV

Notar que, si el campo escalar es contante dentro del volumen, el centro del campo escalar y el
centroide de volumen coinciden:
r

xdV

r
x (V _ ) = V

dV

xdV
V

dV

xdV

=V

dV

r
= x (V )

De forma anloga podemos definir el centro del campo escalar delimitado por un rea:
r

r
x (A _) =

xdA
A

dA
A

y el centro del campo escalar definido un una lnea:

Universidad de Castilla- La Mancha


Ciudad Real - Espaa

Draft

Por: Eduardo W. V. Chaves (2014)

1 TENSORES

147

r
x (L _) =

xdL
L

dL
L

Si el campo escalar representa la densidad de masa ( ) el centro del campo escalar se


denomina de Centro de Masa (C.M.):
r

xdV

r
x (V _ ) = V

dV

Centro de masa en el volumen

(1.181)

Centro de un Campo Vectorial dentro de un Dominio


Consideremos que el cuerpo de volumen V est bajo la accin de un campo vectorial
r r r
r
b = b( x ) , y de un campo escalar ( x ) , (ver Figura 1.28).
r r
b( x )

x3
x2

dV

x3

r
x

x2
r
x

x1

( x)

r (br )

x1

Figura 1.28
Podemos obtener que:
r r r
r r r
r
r r
r r
x = x (b ) + x x b = x (b ) b + x b
r r r
r r
r r
x bdV = x (b ) bdV + x bdV

El centro del campo vectorial se cumple cuando:


r

x bdV = 0

Universidad de Castilla- La Mancha


Ciudad Real - Espaa

Draft

(1.182)

Por: Eduardo W. V. Chaves (2014)

MECNICA DEL MEDIO CONTINUO: PROBLEMAS RESUELTOS

148

Luego
r r

(b )
(b )
(b )
x bdV = x bdV = x bdV = x F

donde hemos definido

r r r
r r
x (b ) F = x bdV

r
r
F = bdV

(1.183)

Notar que F es la fuerza resultante y est situada en el punto x (b ) .


r

Si el campo arbitrario b es uniforme en el dominio V podemos obtener que:

r r r (br ) r
r
xdV b = x
b dV

V
V

r
r

r r r
r

r r r

dV x (V _ ) x (b ) b = 0
dV x (V _ ) b = dV x (b ) b

V
r (V _ ) r (br ) r
x
x
=0
r (V _ ) r (br )
x
=x
r r

(b )
x bdV = x bdV

y si adems el campo escalar es uniforme tenemos que:


r
r r
r
x ( ) = x (b ) = x
r
Si el campo escalar = es la densidad de masa, y b representa el campo gravitacional en las

proximidades de la superficie de la Tierra, la ecuacin (1.183) queda:


0
Fi = b i dV =
= 0
gdV mg
V

donde m = dV es la masa total del cuerpo.


V

Centro de Rotacin Nula


Podemos decir que:

r r r
r rr
r
r r rr
x = x () + x
x b = x () b + x b
r
rr
r r
r r
x bdV = x ( ) bdV + x bdV

El centro del campo vectorial de rotacin nula se cumple cuando:


r

x bdV = 0

(1.184)

Con lo cual:
r

V
1

42
4
3
r

()
()
x bdV = x bdV = x bdV

rr
r r
x () F =

Universidad de Castilla- La Mancha


Ciudad Real - Espaa

Draft

Por: Eduardo W. V. Chaves (2014)

1 TENSORES

149

donde es el torque que el campo b produce en el cuerpo y viene dado por:


r r
r
= x bdV

(1.185)

Si el campo escalar representa la densidad de masa ( = ), y b representa el campo


r r
gravitacional, x ( ) se denomina de Centro de Gravedad (G). Notar que, si fijamos el cuerpo en su
centro de gravedad y que est libre para girar, cualquier posicin (rotacin alrededor del centro
de gravedad) que el cuerpo se encuentre el momento resultante continuar siendo cero, y no
tendr tendencia a girar.
r

Si es uniforme x (V _ ) = x (V )
r

Si b es uniforme x (V _ ) = x (b )
r

Si b y son uniformes x (V _ ) = x (b ) = x (V )
r r
b( x )

x2

x3

x3

x2

x1

x3

x1

r (br )

x3

r
x (V )

x2

r
x (V _ )

x 2

r
( x)

x1

x1

Figura 1.29

A continuacin vamos obtener el torque del campo (b x ) :


r r r
x = x + x
r
r r
r r
r r
r
r
x (b x ) = x (b x ) + x (b x )

Integrando sobre el volumen del cuerpo sta ltima ecuacin podemos obtener que:

r
r r
r r
r r
r
r
x (b x )dV = x (b x )dV + x (b x )dV

(1.186)

El centro del campo vectorial (b x ) de rotacin nula se cumple cuando:

r r
r
r
x (b x )dV = 0

Universidad de Castilla- La Mancha


Ciudad Real - Espaa

Draft

(1.187)
Por: Eduardo W. V. Chaves (2014)

MECNICA DEL MEDIO CONTINUO: PROBLEMAS RESUELTOS

150

Con eso la ecuacin (1.186) queda:

r
r
r r
r r
r r
r r
r
r r
r
r
x (b x )dV = x (b x )dV = x (b x )dV = x x ( ) F = x

donde hemos utilizado la ecuacin (1.185). En el Ejemplo 1.17 hemos demostrado que
r r r
r r
r r r
x

(
b

x
)
dV
=
[(
x

x
)
1

x
x ] b , con lo cual la ecuacin anterior queda:

r r
r r
r
x (b x )dV = x

r r
r r
r
r
r r
r r
{ [( x x )1 x x ] b }dV = x { j O b }dV = x

donde hemos definido el tensor de segundo-orden:


r r
r r
j O = ( x x )1 x x

j O ij = x k x k ij xi x j

cuyas componentes son:


j O ij = x k x k ij xi x j =

( x12

x 22

1 0 0 x1 x1
1 0 x1 x 2
0 0 1 x1 x3

x1 x 2
x2 x2

x32 ) 0

x 2 x3

x1 x3
x 2 x3
x3 x3

( x 22 + x32 )
x1 x 2
x1 x3

2
2
= x1 x 2
( x1 + x3 )
x 2 x3
x x
x 2 x3
( x12 + x 22 )
1 3

(1.188)

Si = (densidad de masa) y b representa un campo vectorial uniforme, podemos obtener


que:

r r
r
{ jO b }dV = x

r r

jO dV b = x r

r r r
IO b = x

donde I O es el tensor de inercia de masa y viene dado por:

2
2
( x 2 + x3 )dV
V
= x1 x 2 dV
V
x1 x3 dV

I O( ) = jO dV

I O( ) ij

cuya unidad en el SI es [I O( ) ] =

kg
m3

x1 x 2 dV
V

( x + x )dV
x x dV

2
1

2
3

2 3

x1 x3 dV

V
x 2 x3 dV
V

( x12 + x 22 )dV

m 2 m 3 = kg m 2 .

De forma anloga, podemos definir el tensor de inercia de rea:

2
2
x1 x 2 dA
x1 x3 dA
( x 2 + x3 )dA

A
A
A
( A)
2
2

( x1 + x3 )dA
I O ij = x1 x 2 dA
x 2 x3 dA

A
A

A
x1 x3 dA
( x12 + x 22 )dA
x 2 x3 dA

A
A
A
r r
r r r
Note que si consideramos el torque (b x ) y el vector x = x + x podemos obtener:

Universidad de Castilla- La Mancha


Ciudad Real - Espaa

Draft

Por: Eduardo W. V. Chaves (2014)

1 TENSORES

151

r r
r
r
r r
r r
x (b x )dV = { [( x x )1 x x ] b }dV

r r
r r
r r
r r
r
= { { [( x + x ) ( x + x )]1 [( x + x ) ( x + x )] } b }dV

r r
r r
r r
r r
{ { [( x x ) + ( x x ) + ( x x ) + ( x x )]1
=
r r
r r
r
r
r
r
r
[( x x ) + ( x x ) + ( x x ) + ( x x ) } b }dV
V
r r
r r
r
r
r r
r
r
= { [( x x )1 x x ] b }dV + { [( x x )1 x x ] b }dV +

r r
r
r r
r
r
r
r r
+ { [( x x )1 x x ] b }dV + { [( x x )1 x x ] b }dV

(1.189)
r

Notar que, si el campo b es uniforme:


r r
r r
r
r

r r
r r
{ [( x x )1 x x ] b }dV = [( x x )1 x x ] dV b

V
r r
r
r

r r
= ( x x )1 dV b + x x dV b

r
r r r
r
r
= x dV x 1 b + x x dV b

y
r r
r
r r
r
r
r

r
r
{ [( x x )1 x x ] b }dV = [( x x )1 x x ] dV b

V
r r
r
r
r

r
= ( x x )1 dV b + x x dV b

V
r
r r
r
r
r
= x x dV 1 b + x dV x b

r
r
r
r r
Como estamos considerando que el campo b es uniforme se cumple que x (V _ ) = x (b ) = x ,
r
r
y tambin la ecuacin x dV = 0 . Con eso la ecuacin (1.189) queda:

r r
r
r
r r
r r
x (b x )dV = { [( x x )1 x x ] b }dV

r r
r r
r
r
r r
r
r
= { [( x x )1 x x ] b }dV + { [( x x )1 x x ] b }dV

(1.190)

Si consideramos que:

Universidad de Castilla- La Mancha


Ciudad Real - Espaa

Draft

Por: Eduardo W. V. Chaves (2014)

MECNICA DEL MEDIO CONTINUO: PROBLEMAS RESUELTOS

152

r r
r
r
r
r r
r r
x (b x )dV = { [( x x )1 x x ] b }dV = { j O b }dV

r
r r
r r
r r
r
r
x (b x )dV = { [( x x )1 x x ] b }dV = { j O b }dV = j O

r r
r
r
r
r r
r
r
x (b x )dV = { [( x x )1 x x ] b }dV = { jO b }dV

{ b }dV

(1.191)

La ecuacin (1.190) puede ser reescrita como:

r r
r r
r
r
r
r r
r r
r r
r
r
{ [( x x )1 x x ] b }dV = { [( x x )1 x x ] b }dV + { [( x x )1 x x ] b }dV

r
r
r
{ j O b }dV = { jO b }dV + { j O b }dV

r
r
r
r
{ j O b }dV { jO b }dV { j O b }dV = 0

r r

[ j O jO j O ] dV b = 0

(1.192)
r

Notar que la ecuacin anterior debe cumplir para cualquier campo vectorial uniforme b ,
luego:

[j

jO j O ] dV = 0

(1.193)

j O dV = jO dV + j O dV
V

donde las componentes (ver ecuacin (1.188)) de j O , jO , y jO vienen dadas por:


( x 22 + x32 )
( x 2 2 + x3 2 )
x1 x 2
x1 x3
x1 x 2
x1 x3

2
2
2
2
( j O ) ij = x1 x 2
( x1 + x3 )
x 2 x3 , ( jO ) ij = x1 x 2
( x1 + x 3 )
x 2 x3 ,
x x
x x
x 2 x3
( x12 + x 22 )
x 2 x3
( x1 2 + x 22 )
1 3
1 3

( x 22 + x 32 )
x1 x 2
x1 x 3

2
2
( jO ) ij = x1 x 2
( x1 + x3 )
x 2 x3
x x
x 2 x3
( x12 + x 22 )
1 3

Universidad de Castilla- La Mancha


Ciudad Real - Espaa

Draft

Por: Eduardo W. V. Chaves (2014)

2 Cinemtica del Continuo


2.1 Descripcin
del
Movimiento,
Velocidad, Aceleracin

Derivada

Material,

Ejemplo 2.1
Un continuo viene definido por un cuadrado de lado b , y est sometido a movimiento de
slido rgido definido por una rotacin antihoraria por un ngulo de 30 . Encontrar las
ecuaciones de movimiento. Obtener tambin la nueva posicin de la partcula D .
r

Nota: Considerar los sistemas x y X superpuestos.


X 2 , x2
x2

D
30
b

x1

D
B
30

A = A

X 1 , x1

Solucin: Aplicamos las ecuaciones de movimiento de un slido rgido x = c + Q X = Q X ,


r

con c = 0 . Las componentes de Q son las mismas que las componentes de la matriz de
r
r
transformacin del sistema x para el sistema x , i.e.:
cos sin 0
Q ij = sin cos 0
0
0
1

Luego, las partculas vienen gobernadas por las ecuaciones de movimiento:


x1 cos 30 sin 30 0 X 1


x 2 = sin 30 cos 30 0 X 2
x 0
0
1 X 3
3

La partcula que inicialmente estaba en D ( X 1 = 0 , X 2 = b , X 3 = 0 ) mueve para la siguiente


posicin:
Universidad de Castilla- La Mancha
Ciudad Real - Espaa

Draft

Por: Eduardo W. V. Chaves (2014)

MECNICA DEL MEDIO CONTINUO: PROBLEMAS RESUELTOS

154

x1D cos 30 sin 30 0 0 b sin 30


D


x 2 = sin 30 cos 30 0 b = b cos 30
x D 0

0
1 0
0

Ejemplo 2.2
Un movimiento del medio continuo en la descripcin material viene dado por:
x1 = exp t X 1 exp t X 2

x 2 = exp t X 1 + exp t X 2

x3 = X 3

para t > 0 . Encontrar las componentes de la velocidad y aceleracin en coordenadas


espaciales y materiales.
Solucin:
Velocidad:
V1 = exp t X 1 + exp t X 2
r r
r r
Dx ( X , t ) componentes
V ( X , t) =

V2 = exp t X 1 exp t X 2
Dt
V = 0
3

(2.1)

A1 = exp t X 1 exp t X 2

t
t
A2 = exp X 1 + exp X 2
A = 0
3

(2.2)

Aceleracin:

Para encontrar las componentes de la velocidad y la aceleracin reemplazamos las


ecuaciones del movimiento:
Velocidad (descripcin espacial)
v1 = x 2

v 2 = x1

v3 = 0

a2 = x2 = v1

a3 = 0

Aceleracin (descripcin espacial)


a1 = x1 = v2

(2.3)

Ejemplo 2.3
El campo de velocidad de un fluido viene dado por:
r
v = x1e 1 + x2 e 2 + x3e 3

(2.4)

r
T ( x , t ) = 3 x 2 + x3 t

(2.5)

y el campo de temperatura es:

Encontrar la tasa de cambio en el tiempo de la temperatura.


Solucin:

La tasa de cambio de una propiedad viene dada por la derivada material T ( x , t ) :


r
r
T T
T
T
DT T ( x , t ) T ( x , t )
+
+
=
vj =
v1 +
v2 +
v3
x j
t x1
x 2
x3
t
Dt
DT
= x 3 + (0 x1 + 3 x 2 + tx 3 ) = x 3 + (3 x 2 + tx 3 )
Dt
Universidad de Castilla- La Mancha
Ciudad Real - Espaa

Draft

(2.6)
(2.7)

Por: Eduardo W. V. Chaves (2014)

2 CINEMTICA DEL CONTINUO

155

Ejemplo 2.4
Dado el movimiento:
xi = X i + 0,2tX 2 1i

(2.8)

y el campo de temperatura (estacionario):


r
T ( x ) = 2 x1 + x 22

(2.9)

a) Encontrar el campo de temperatura en la descripcin material;


b) Encontrar la tasa de cambio de la temperatura para una partcula que en la configuracin
de referencia ocupaba la posicin (0,1,0) .
Solucin:
Segn las ecuaciones del movimiento tenemos que:
x1 = X 1 + 0,2tX 2 11 = X 1 + 0,2tX 2
x 2 = X 2 + 0,2tX 2 12 = X 2
x3 = X 3 + 0,2tX 2 13 = X 3

Luego:

r
r
r r
T ( x ( X , t )) = 2 x1 ( X , t ) + x 2 ( X , t )

= 2( X 1 + 0,2tX 2 ) + ( X 2 )
r
= 2 X 1 + ( X 2 + 0,4t ) X 2 = T ( X , t )
2

b) La derivada material de la temperatura viene dada por:

r
DT ( X , t ) & r
T ( X , t ) = 0,4 X 2
Dt

Para la partcula ( X 1 = 0; X 2 = 1; X 3 = 0) tenemos que:


T& (( X 1 = 0; X 2 = 1; X 3 = 0), t ) = 0,4 X 2 = 0,4

Ejemplo 2.5

r r

Determinar el campo de velocidad V ( X , t ) en la descripcin material y el campo de


r r
aceleracin A( X , t ) de una partcula en el tiempo t en funcin de la tasa de cambio de los
r r
desplazamientos U ( X , t )
Solucin:
r r
r&
D r r
V ( X , t) =
U ( X , t) = U
Dt
r r
r&
D r r
D2 r r
&r&
A( X , t ) =
V ( X , t ) = V == 2 U ( X , t ) = U
Dt
Dt

(2.10)

r r& &r&
A =V =U

(2.11)

Ejemplo 2.6
Considrense las siguientes ecuaciones del movimiento en la descripcin Lagrangiana:
r
2
x1 ( X , t ) = X 2 t 2 + X 1
x1 1 t
r


Forma Matricial
x 2 = 0 1
x 2 ( X , t ) = X 3 t + X 2
r

x 0 0
3
x 3 ( X , t ) = X 3

Universidad de Castilla- La Mancha


Ciudad Real - Espaa

Draft

0 X 1

t X 2
1 X 3

(2.12)

Por: Eduardo W. V. Chaves (2014)

MECNICA DEL MEDIO CONTINUO: PROBLEMAS RESUELTOS

156

Es este un movimiento posible? Si as es, encontrar los campos de desplazamiento,


velocidad y aceleracin en la descripcin Lagrangiana y Euleriana. Considrese un partcula
P , que en el tiempo t = 0 ocupaba la posicin X 1 = 2, X 2 = 1, X 3 = 3 , encontrar la
velocidad de P en los tiempos t = 1s y t = 2 s .
Solucin:
El movimiento es posible si J 0 . Verificamos que el movimiento es posible:

xi
J=
X j

x1
X 1
x 2
=
X 1
x3
X 1

x1
X 2
x 2
X 2
x3
X 2

x1
X 3 1 t 2
x 2
=0 1
X 3
0 0
x3
X 3

0
t =1 0
1
r

El campo vectorial de desplazamiento viene dado por la definicin u = x X . Utilizando


las ecuaciones del movimiento (2.12) obtenemos que:
u1

u 2

u 3

(Xrr , t ) = x ( Xrr, t ) X
(Xr , t ) = x ( Xr , t ) X
(X , t ) = x ( X , t ) X
1

= X 2t 2

= X 3t

=0

(2.13)

que son las componentes del desplazamiento en la descripcin Lagrangiana. La velocidad y


la aceleracin vienen dadas por:

( )

du 1 X , t
d
=
X 2t 2 = 2 X 2t
V1 =
dt
dt

du 2 X , t
d
= ( X 3t ) = X 3
V 2 =
dt
dt

du 3 X , t
d
= (X 2t ) = 0
V3 =
dt
dt

( )

dV1

A1 = dt = 2 X 2

dV 2

=0
A2 =
dt

dV 3

A3 = dt = 0

( )

(2.14)

La forma inversa de (2.77) nos proporcionan las ecuaciones del movimiento en la


descripcin Euleriana:
X 1 1 t 2

1
X 2 = 0
X 0
0
3

r
t 3 x1 X 1 ( x , t ) = x1 t 2 x 2 + t 3 x 3
r

t x 2 X 2 ( x , t ) = x 2 tx 3
r
1 x 3 X 3 ( x , t ) = x 3

(2.15)

Luego, los campos de desplazamiento, velocidad y aceleracin en la descripcin Euleriana


se obtienen al reemplazar las ecuaciones (2.15) en las expresiones (2.13) y (2.14), i.e.:

(Xr ( xr , t ), t ) = X ( xr , t )t = ( x tx )t = u ( xr , t )
(Xr ( xr , t ), t ) = X ( xr , t )t = x t = u ( xr , t )
(X ( xr , t ), t ) = u ( xr , t ) = 0
r r
r
r
V (X ( x , t ), t ) = 2 X ( x , t )t = 2( x tx )t = v ( x , t )
r r

r
r
V (X ( x , t ), t ) = X ( x , t ) = x = v ( x , t )
r r
r

V (X ( x , t ), t ) = v ( x , t ) = 0
u1

u 2

u 3

Ciudad Real - Espaa

(2.16)

Universidad de Castilla- La Mancha

(2.17)

Draft

Por: Eduardo W. V. Chaves (2014)

2 CINEMTICA DEL CONTINUO

A1

A2

A3

(Xr ( xr , t ), t ) = 2 X ( xr , t ) = 2( x
(Xr ( xr , t ), t ) = a ( xr , t ) = 0
(X ( xr , t ), t ) = a ( xr , t ) = 0
r

157

r
tx 3 ) = a1 ( x , t )

(2.18)

Teniendo en cuenta la descripcin Lagrangiana de la velocidad dada por (2.14), la velocidad


de la partcula P ( X 1 = 2, X 2 = 1, X 3 = 3 ) en el tiempo t = 1s viene dada por:
r
r
r
V1 ( X , t ) = 2 X 2 t = 2 m / s ; V2 ( X , t ) = X 3 = 3m / s ; V3 ( X , t ) = 0

Observemos que en el tiempo t = 1s la partcula P ocupa una nueva posicin definida por:
x1 = X 2 t 2 + X 1 = 3 ;

x 2 = X 3t + X 2 = 4 ;

x3 = X 3 = 3

Luego, la velocidad de la partcula P tambin puede ser obtenida por (2.17), i.e.:
r
v1 ( x , t ) = 2( x 2 tx 3 )t = 2( 4 1 3) 1 = 2m / s
r

v 2 ( x , t ) = x 3 = 3m / s
v ( xr , t ) = 0
3

Observemos que, la velocidad de la partcula es la misma sea utilizando la descripcin


Lagrangiana o la Euleriana, ya que la velocidad es una propiedad intrnseca de la partcula.
La velocidad de la partcula P en el tiempo t = 2 s viene dada por:
V1

V 2

V3

(Xrr , t ) = 2 X t = 2 2 1 = 4m / s
(Xr , t ) = X = 3m / s
(X , t ) = 0
2

En el tiempo t = 2 s la nueva posicin de la partcula P queda definida por:


r
x1 ( X , t ) = X 2 t 2 + X 1 = 6
r

x2 ( X , t ) = X 3t + X 2 = 7
r

x 3 ( X , t ) = X 3 = 3

Como podemos verificar en la figura abajo, la descripcin Lagrangiana del movimiento


r r
x ( X , t ) describe la trayectoria de la partcula P .
NOTA: Notar que la velocidad espacial no se puede obtener a travs de
r r
DX ( x , t ) r r r
= 0 v ( x , t ) . Podemos verificar este hecho con el ejemplo propuesto:
Dt
r
r
r
r
r
DX i ( x , t ) X i ( x , t ) X i ( x , t )
X i ( x , t )
X i ( x , t )
r
r
r
v1 ( x , t ) +
v 2 ( x, t ) +
v3 ( x , t )
=
+
Dt
t
x 2
x 3
x1

Luego:
r
r
r
r
r
r
DX 1 ( x , t ) X 1 ( x , t ) X 1 ( x , t ) r
X 1 ( x , t )
X 1 ( x , t ) r
=
+
v1 ( x , t ) +
v2 ( x , t ) +
v3 ( x , t )
t
x2
x3
Dt
x1

) [

= 2tx2 + 3t 2 x3 + 1 2( x2 tx3 )t t 2 x3 + t 3 0 = 0

Universidad de Castilla- La Mancha


Ciudad Real - Espaa

Draft

Por: Eduardo W. V. Chaves (2014)

MECNICA DEL MEDIO CONTINUO: PROBLEMAS RESUELTOS

158

r
r
r
r
r
r
DX 2 ( x , t ) X 2 ( x , t ) X 2 ( x , t ) r
X ( x , t ) r
X ( x , t )
v1 ( x , t ) + 2
v2 ( x , t ) + 2
v3 ( x , t )
+
=
Dt
x3
x2
t
x1

= ( x3 ) + [0 2( x2 tx3 )t + 1 x3 t 0] = 0
r
r
r
r
r
r
DX 3 ( x , t ) X 3 ( x , t ) X 3 ( x , t ) r
X ( x , t ) r
X ( x , t )
v1 ( x , t ) + 3
v2 ( x , t ) + 3
v3 ( x , t )
+
=
x3
Dt
x2
t
x1

= (0 ) + [0 2( x2 tx3 )t + 0 x3 + 1 0] = 0
r r r
Adems, teniendo en cuenta que u = x X , se cumple que:
r r
r r
r r
r r
Dx ( X , t ) D r r
Du( X , t ) r& r
v ( X , t) =
=
u( X , t ) X ( x , t ) =
u( X , t )
Dt
Dt
Dt

Adems, tambin se cumple que:


r r
r r
r r
r& r
r r
Du( x , t ) u( x , t ) u( x , t ) r r
v ( x , t ) = u( x , t )
=
+
r v ( x, t )
Dt
t
x
Trayectoria de la partcula P

r
viP ( x , t = 1s) = [2;3;0]

r
Vi P ( X P , t = 1s) = [2;3;0]
t0

X iP = [2;1;3]

t = 1s
Partcula P

r
Vi P ( X P , t = 2s ) = [4;3;0]

xiP = [3;4;3]
Partcula P

t = 2s

xiP = [6;7;3]

r
viP ( x , t = 2s ) = [4;3;0]

Ejemplo 2.7
El campo de velocidad de un medio continuo, expresado en forma Euleriana es el
siguiente:
v1 =

x1
1+ t

; v2 =

2 x2
1+ t

; v3 =

3 x3
1+ t

(2.19)
r

a) Determinar la relacin entre las coordenadas espaciales y materiales xi = xi ( X , t ) ;


b) Obtener las componentes de la aceleracin cuando se utiliza la descripcin espacial del
movimiento.
c) Obtener las componentes de la aceleracin cuando se utiliza la descripcin Lagrangiana
del movimiento.
Solucin:

Universidad de Castilla- La Mancha


Ciudad Real - Espaa

Draft

Por: Eduardo W. V. Chaves (2014)

2 CINEMTICA DEL CONTINUO

a) Considerando que vi =

dxi
dt
v1 =

dx1 =

159

dx1
x
dx
dt
= 1 1 =
dt 1 + t
x1 1 + t

1 + t dt Lnx

(2.20)

= Ln(1 + t ) + Ln(C1 )

x1 = C1 (1 + t )

(2.21)

La condicin inicial t = 0 x1 = X 1 implica que C1 = X 1


x1 = X 1 (1 + t )

(2.22)

dx 2 2 x 2
dx 2 2dt
=

=
dt
1+ t
x2
1+ t
1
2
dx 2 =
dt Lnx 2 = 2Ln(1 + t ) + LnC 2
x2
1+ t
v2 =

x2 = C 2 (1 + t )

(2.23)
(2.24)

para t = 0 x 2 = X 2 C 2 = X 2
x2 = X 2 (1 + t ) 2

v3 =
1

dx3 =

(2.25)

dx3 3 x3
dx
3dt
=
3 =
dt 1 + t
x3 1 + t

1 + t dt Lnx

= 3Ln(1 + t ) + LnC3

(2.26)

x3 = C3 (1 + t )3

(2.27)

y para t = 0 x3 = X 3 C3 = X 3
x3 = X 3 (1 + t ) 3

(2.28)

Las ecuaciones del movimiento:


(2.29)
x1 = X 1 (1 + t ) ;
x2 = X 2 (1 + t ) 2
;
x3 = X 3 (1 + t )3
r r
b) Conocido v ( x , t ) en la descripcin espacial (Euleriana), podemos aplicar la derivada
material:

r r
r r
r r
r r
v ( x , t )
+ v ( x , t ) v ( x , t )
a ( x, t ) =
t
v
v
ai = i + (vi , k )vk = i + (vi ,1v1 + vi , 2v2 + vi ,3v3 )
t
t

(2.30)
(2.31)

luego,
1
x

+ 1
+ 0 + 0 = 0
(1 + t )
1 + t 1 + t

2x2
2x 2
2 x2

a2 =
+ 0 + 2
+ 0 =
2
2
1+ t 1+ t
(1 + t )
(1 + t )

a1 =

a3 =

x1

3x 3
6 x3

+ 0 + 0 + 3
=
1 + t 1 + t (1 + t ) 2
(1 + t )

3 x3

Universidad de Castilla- La Mancha


Ciudad Real - Espaa

(2.32)

Draft

Por: Eduardo W. V. Chaves (2014)

MECNICA DEL MEDIO CONTINUO: PROBLEMAS RESUELTOS

160

c) La velocidad en la descripcin Lagrangiana viene dada por:


V1 = X 1

V2 = 2 X 2 (1 + t )

V3 = 3 X 3 (1 + t ) 2

dV2
= 2X2
dt

a3 =

(2.33)

luego,
a1 =

dV1
=0
dt

a2 =

dV3
= 6 X 3 (1 + t )
dt

(2.34)

Ejemplo 2.8
Respecto a un conjunto de ejes materiales X i y espaciales xi superpuestos, el campo de
desplazamientos de un cuerpo continuo viene dado por:
x1 = X 1

x2 = X 2 + AX 3

x3 = X 3 + AX 2

(2.35)

en las que A es constante. Hallar las componentes del vector desplazamiento en las formas
material y espacial.
Solucin:
Vector desplazamiento:
r r r
u= xX

u1 = x1 X 1 = 0

u 2 = x 2 X 2 = X 2 + AX 3 X 2 = AX 3
u = x X = X + AX X = AX
3
3
3
2
3
2
3

(2.36)

Las ecuaciones del movimiento inverso son obtenidas a continuacin:


x1 1
x = 0
2
x 3 0

0 X1
A X 2
A 1 X 3
0
1

1
det 0
0

0
A = 1 A 2
A 1

0
1

(2.37)

la inversa:
1
0

0
1 A
A 1
0

1 A 2
1
=
0
1 A2
0

1 A
A 1
0

(2.38)

luego,
X1
X = 1
2 1 A2
X 3

1 A2

0
0

1 A
A 1
0

x1
x
2
x3

X 1 = x1

( x2 Ax3 )
X2 =
1 A2

1
X 3 = 1 A2 ( x3 Ax2 )

(2.39)

Componentes del vector desplazamientos en coordenadas espaciales:

u1 = x1 X 1 = 0

A( x3 Ax 2 )
1

( x 2 Ax3 ) =
u 2 = x 2 X 2 = x 2
2
1 A2
1 A

A( x 2 Ax3 )
1
( x3 Ax 2 ) =
u1 = x3 X 3 = x3
2
1 A2
1 A

Universidad de Castilla- La Mancha


Ciudad Real - Espaa

Draft

(2.40)

Por: Eduardo W. V. Chaves (2014)

2 CINEMTICA DEL CONTINUO

161

Ejemplo 2.9
Considrese las ecuaciones del movimiento:
x1 = X 1

x2 = X 2 + X 3t

x3 = X 3 + X 3 t

(2.41)

Determinar las velocidades de las partculas que pasan por el punto (0,1,2) en los tiempos
t1 = 0 s y t 2 = 1 s
Solucin:
El campo de velocidad viene dado por:

r r
r r
Dx ( X , t )
V ( X ,t) =
Dt

(2.42)

en componentes:
V1 = 0

V2 = X 3

V3 = X 3

(2.43)

r r
Para t = 0 s tenemos que x = X , luego, ( X 1 = 0, X 2 = 1, X 3 = 2)

V1 = 0

V2 = 2

V3 = 2

(2.44)

Para t = 1 s , la partcula que est pasando por ( x1 = 0, x 2 = 1, x3 = 2) en la configuracin de


referencia ocupaba la posicin:
x1 = 0 = X 1

x 2 = 1 = X 2 + X 3 ( X 1 = 0; X 2 = 0; X 3 = 1)
x3 = 2 = X 3 + X 3

(2.45)

luego,
V1 = 0

V2 = 1

V3 = 1

(2.46)

Ejemplo 2.10
Dado un sistema de referencia e i , el movimiento de una partcula del medio continuo est
definido por las siguientes ecuaciones:

ct
ct

+ X 2 cos
x1 = X 1 sin 2
2
2
2

X
X
X
X
+
+
2
2
1
1

ct
ct
+ X 2 sin
x 2 = X 1 cos 2
2
2
X +X2
2
1
X1 + X 2
x3 = X 3

(2.47)

donde c es una constante.


Determinar las componentes de la velocidad en coordenadas materiales y espaciales.
Solucin:
Las componentes de la velocidad en la descripcin material (Lagrangiana) son:

Universidad de Castilla- La Mancha


Ciudad Real - Espaa

Draft

Por: Eduardo W. V. Chaves (2014)

MECNICA DEL MEDIO CONTINUO: PROBLEMAS RESUELTOS

162

r
V1 ( X , t ) =

ct
X 1 cos 2
2
2
2
X 1 + X 2
X1 + X 2

ct
X 2 sin
X2 +X2

2
1

r
V2 ( X , t ) =

ct

sin
X

1
2
2
2
2

X 1 + X 2
X1 + X 2

ct
+ X 2 cos

X2 +X2
2

r
V3 ( X , t ) = 0

(2.48)

Teniendo en consideracin (2.47), podemos notar que se cumple la siguiente relacin:


x12 + x22 = X 12 + X 22

(2.49)

Luego, las componentes de la velocidad en la descripcin espacial (Euleriana) son:


r
cx
v1 ( x , t ) = 2 2 2
x1 + x2

r
v2 ( x , t ) =

c x1
+ x22

x12

r
v3 ( x , t ) = 0

(2.50)

Las ecuaciones inversas del movimiento son:


ct
ct
cos

sin 2
2
x 2 + x 2 0
x1 + x 2

1
2

X1
x1

X = cos c t sin c t 0 x
2
2 x2 + x2
x2 + x2

2
2
1
X 3 1

x
0
0
1 3

(2.51)

Ejemplo 2.11
El campo de velocidad tiene las siguientes componentes:
v1 = x1

v2 =

x2
2t + 3

v3 = 0

(2.52)

en la descripcin Euleriana. Encontrar las ecuaciones paramtricas de la trayectoria de la


partcula que en la configuracin de referencia estaba en ( X 1 , X 2 , X 3 ).
Solucin:
Para encontrar la trayectoria debemos resolver el sistema:
dx1
= x1
dt

dx2
x
= 2
dt
2t + 3

dx3
=0
dt

(2.53)

con las condiciones iniciales


x1 (t = 0) = X 1

x2 (t = 0) = X 2
x (t = 0) = X
3
3
x1

X1
x2

X2

dx1
= dt
x1 0

x
Ln 1 = t
X1

dx 2
dt
=
x2
2t + 3
0

x
Ln 2
X2

(2.54)

x1 = X 1 exp t

( )

= Ln 2t + 3 Ln 3

x2 = X 2

2
t +1
3

(2.55)

x3 = X 3
Universidad de Castilla- La Mancha
Ciudad Real - Espaa

Draft

Por: Eduardo W. V. Chaves (2014)

2 CINEMTICA DEL CONTINUO

163

Luego, las ecuaciones del movimiento vienen dadas por:


x1 = X 1exp t

; x2 = X 2

2
t + 1 ; x3 = X 3
3

(2.56)

Ejemplo 2.12
Considrense las siguientes ecuaciones del movimiento:
x1 = X 1

x2 = 2 t X 3 + X 2

x3 = X 3
(2.57)
r
y una cantidad fsica representada por el campo escalar q ( x , t ) en la descripcin Euleriana:
r
q ( x , t ) = 2 x1 + x 2 x3 + 1
(2.58)
;

a) Obtener la descripcin Lagrangiana de esta cantidad fsica;


b) Obtener la velocidad en las descripciones Lagrangiana y Euleriana;
c) Obtener la tasa de cambio de la cantidad fsica en cuestin.
d) Obtener la tasa de cambio local de q en el punto espacial (1,3,2) .
Solucin:
r

r r

a) La descripcin Lagrangiana es inmediato, q ( x , t ) = q ( x ( X , t ), t ) = Q( X , t ) , es decir,


r
reemplazamos las ecuaciones del movimiento (2.57) en la expresin de la variable q ( x , t )
dada por (2.58):
r
Q( X , t ) = 2 X 1 + X 2 + ( 2t 1) X 3 + 1

(2.59)

r r
r r
Dx ( X , t )
V ( X , t) =
Dt

(2.60)

b) La velocidad

Descripcin Lagrangiana
V1 = 0

V2 = 2 X 3

V3 = 0

(2.61)

Las ecuaciones del movimiento inversa:


x1 = X 1

x2 = 2 t X 3 + X 2
x = X
3
3

X 1 = x1

X 2 = x 2 2 t x3
X = x
3
3

Luego, la descripcin Euleriana de la velocidad viene dada por:


v1 = 0

v2 = 2 x3

v3 = 0

(2.62)

c) La tasa de cambio de la variable viene dada por la derivada material


r
D
Q( X , t ) = 2 X 3
Q& =
Dt

o
q& =

r
r
q( x , t )
+ xr q v
t4
1
42
3

(2.63)

(2.64)

= 0 ( estacionario )

Universidad de Castilla- La Mancha


Ciudad Real - Espaa

Draft

Por: Eduardo W. V. Chaves (2014)

MECNICA DEL MEDIO CONTINUO: PROBLEMAS RESUELTOS

164

q
q
q
q& = 0 + q, i vi = 0 +
v1 +
v2 +
v3 = [(2)(0) + (1)(2 x3 ) + ( 1)(0)] = 2 x3
x 2
x3
x1

(2.65)

Podramos haber obtenido este resultado partiendo de que Q& = 2X 3 y reemplazando


X 3 = x3 , obteniendo:
r r
r
q& ( x , t ) = Q& ( X ( x , t ), t )

r
q& ( x , t ) = 2 x3

(2.66)
r

d) Observemos que el campo de la cantidad fsica en cuestin es estacionario, i.e. q = q ( x ) ,


luego la tasa local

r
q ( x )
= 0 para cualquier punto espacial.
t

Ejemplo 2.13
Dado el campo de desplazamientos (descripcin Lagrangiana):
u1 = ktX 2

u2 = 0

u3 = 0

y el campo de la temperatura (descripcin Euleriana):


r
T ( x , t ) = ( x1 + x 2 ) t

a) Encontrar la tasa de cambio de la temperatura para una partcula que en el tiempo t = 1s


est pasando por el punto (1,1,1) .
Solucin:

r
r
r
r
T
T
x
dT
(
X
, t)

Podemos aplicar las dos definiciones: T& ( x , t ) =


T& ( X , t ) =
+ r
t
t x t

A travs de la relacin u i = xi X i podemos obtener las ecuaciones del movimiento:


u1 = x1 X 1

x1 = X 1 + ktX 2

u 2 = x2 X 2

x2 = X 2

u 3 = x3 X 3

x3 = X 3

El campo de temperatura en la descripcin material queda:

r
r r
T ( x ( X , t ), t ) = ( x1 + x 2 ) t = (( X 1 + ktX 2 ) + ( X 2 ) ) t = X 1t + kX 2 t 2 + X 2 t = T ( X , t )

Luego, la derivada material viene dada por:

r
T& ( X , t ) = X 1 + 2kX 2 t + X 2

Si queremos encontrar la tasa de la temperatura para una partcula que est pasando por el
punto x1 = 1, x 2 = 1, x3 = 1 en t = 1s , tenemos dos posibilidades. 1) encontrar la posicin de
la partcula en la configuracin de referencia y reemplazar en la ecuacin anterior. 2)
obtener la expresin de la tasa de la temperatura en la descripcin espacial, para esto
r r
necesitamos de las ecuaciones de movimiento X ( x , t ) :
x1 = X 1 + ktX 2

x2 = X 2
x = X
3
3

X 1 = x1 ktx 2

X 2 = x2
X = x
3
3

r r
r
T& ( X ( x , t ), t ) = X 1 + 2kX 2 t + X 2 = ( x1 ktx 2 ) + 2kt ( x 2 ) + ( x 2 ) = T& ( x , t )
r
Simplificando tenemos que T& ( x , t ) = x1 + ktx 2 + x 2 . Luego:
Universidad de Castilla- La Mancha
Ciudad Real - Espaa

Draft

Por: Eduardo W. V. Chaves (2014)

2 CINEMTICA DEL CONTINUO

165

T& ( x1 = 1, x 2 = 1, x3 = 1, t = 1) = (1 k ) + 2k + 1 = k + 2

Solucin Alternativa:
r
T x1 T x2 T x3
r
T T x T T xi

+ r
=
+
= ( x1 + x2 ) +
+
+
T& ( x , t ) =

t x t
t xi t
x1 t x2 t x3 t
= ( x1 + x2 ) + (tkX 2 + t (0) + (0)(0) ) = x1 + x2 + tkX 2

Observemos que x 2 = X 2 , luego:


r
T& ( x , t ) = x1 + x 2 + tkx 2

Ejemplo 2.14
Considrese las siguientes ecuaciones del movimiento:
x1 = X 1

x2 = X 2 +

t
X3
2

x3 = X 3 +

t
X2
2

(2.67)

a) Para que valores de t > 0 (tiempo) este movimiento es posible y que tenga sentido
fsico?
b) Determinar las componentes de la velocidad en la descripcin Lagrangiana y Euleriana;
c) Obtener la ecuacin de la trayectoria.
Solucin:
a) Obteniendo el determinante del Jacobiano:
1 0 0
xi
t2
= 0 1 2t = 1
J=F =
4
X j
0 2t 1

(2.68)

con lo que el movimiento es posible para t < 2 s , ya que:


J =1

t2
>0t <2 s
4

(2.69)

b) Velocidad en la descripcin Lagrangiana:

V = 0
1

D
t

X2 + X3 =
V2 =
Dt
2

t
D

V3 =
X3 + X2 =
Dt
2

X3
2

(2.70)

X2
2

La inversa del tensor F viene dada por:

Universidad de Castilla- La Mancha


Ciudad Real - Espaa

Draft

Por: Eduardo W. V. Chaves (2014)

MECNICA DEL MEDIO CONTINUO: PROBLEMAS RESUELTOS

166

1
1
adj( F ) =
[cof (F )]T Fij1
F
F

F 1 =

Fij1

J
1
= 0
J
0

0
1
2t

1
t
2
1 0
= t
J 2
0

t
2

1
0

t
2

0 1
1 0

1
0

0 1
1 0

0 2t

t
2

0
0
1

0
1
0

1
t
2
0

t
2
0

(2.71)

2t
1
r

Las ecuaciones inversas del movimiento X = F 1 x :


x1 1 0 0 X 1
x = 0 1 t X
2 2
2
x3 0 2t 1 X 3

J
X1
X = 1 0
2 J
0
X 3

0
1
2t

0 x1

2t x 2
1 x3

(2.72)

Reemplazando X i en la expresin de la velocidad, obtenemos la velocidad en la


descripcin espacial:
t
x2
2 x tx
2
= 3 22
v1 = 0 ; v 2 =
2
t
4t
2
2
x3

t
x3
2 x tx
2
= 2 23
; v3 =
2
t
4t
2
2
x2

(2.73)

c) La trayectoria se obtiene eliminando t de las ecuaciones del movimiento (2.67):


x1 = X 1

( x3 X 3 ) X 3 = ( x 2 X 2 ) X 2

Ejemplo 2.15

x3 =

X2
X2
x2 2 + X 3
X3
X3

(2.74)

La velocidad en un punto x de un fluido estacionario viene dada por:


b 2 ( x12 x 22 )
b 2 x1 x 2
r
1 + 2U
v =U
e
e 2 + Ve 3
( x12 + x 22 ) 2
( x12 + x 22 ) 2

donde U y V son constantes.


r

(2.75)
r

Mostrar que xr v = 0 y encontrar la aceleracin de la partcula en x .


Solucin:
r
v
x ( x 2 3x 2 )
x ( x 2 3x 2 )
v v
xr v = vi ,i = 1 + 2 + 3 = 2Ub 2 1 21 2 23 + 2Ub 2 1 21 2 23 = 0
x1 x2 x3
( x1 + x2 )
( x1 + x2 )

La aceleracin:

r
r v
r r
r r
a=
+ xr v v = xr v v
t

Las componentes del gradiente espacial de la velocidad vienen dadas por:

Universidad de Castilla- La Mancha


Ciudad Real - Espaa

Draft

Por: Eduardo W. V. Chaves (2014)

2 CINEMTICA DEL CONTINUO

167

x1 (3 x 22 x12 ) x 2 (3 x12 x 22 ) 0
r

2Ub
( xr v ) ij = 2
x 2 (3 x12 x 22 ) x1 (3 x 22 x12 ) 0
2 3
( x1 + x 2 )
0
0
0

r r
Las componentes de la aceleracin vienen dadas por a i = ( xr v ) ij (v ) j :
2

2 x1U 2 b 4
2
2 3
( x1 + x 2 )

a i = 2 x 2U 2 b 4
2
2 3
( x1 + x 2 )

Ejemplo 2.16

Dada una funcin escalar en la configuracin de referencia ( X , t ) . Obtener la relacin


r
r
entre el gradiente material de ( X , t ) , es decir, Xr ( X , t ) , y el gradiente espacial de
r
r
( x , t ) , es decir, xr ( x , t ) .
Solucin:
Recordemos que una variable que est en la configuracin de referencia la podemos
expresar en la configuracin actual a travs de las ecuaciones del movimiento, es decir,
r
r r
r
( X , t ) = ( X ( x , t ), t ) = ( x , t ) . Luego, partiendo de la definicin del gradiente de un
escalar podemos obtener que:
r
r r
r
r
r
r

(
X
,
t
)

(
X
( x , t ), t ) x ( x , t )
Xr ( X , t ) =
r
=

r
=
r
r F = xr ( x , t ) F
x
x
X
X

Y la forma inversa:

r
r
r
r r
r
r

(
x
,
t
)

(
x
(
X
,
t
),
t
)

(
X
, t)
xr ( x , t ) =
r
r=
r F 1 = Xr ( X , t ) F 1
r =
x
x
X
X

Ejemplo 2.17
Para el siguiente campo de velocidades:
v1 = 0 ;
v2 = 0 ;
v3 = f ( x1 , x 2 ) x3
a) Hallar las trayectorias de las partculas;
b) Determinar la densidad de masa ( ), sabiendo que en t = 0 , = f ( x1 , x 2 ) .

Solucin:
dx1
= v1 = 0 x1 (t ) = C1 para t = 0 x1 = X 1 x1 (t = 0) = C1 = X 1 ;
dt
dx 2
= v 2 = 0 x 2 (t ) = C 2 para t = 0 x 2 = X 2 x 2 (t = 0) = C 2 = X 2
dt
dx3
dx3
= v3 = f ( x1 , x 2 ) x3 = f (C1 , C 2 ) x3
=
dt
x3

f (C , C
1

2 ) dt

Ln( x 3 ) = f (C1 , C 2 )t + k

haciendo k = Ln(C 3 ) obtenemos que:

Universidad de Castilla- La Mancha


Ciudad Real - Espaa

Draft

Por: Eduardo W. V. Chaves (2014)

MECNICA DEL MEDIO CONTINUO: PROBLEMAS RESUELTOS

168

Ln( x3 ) Ln(C 3 ) = f ( X 1 , X 2 )t
x
ln 3
C3

x
= f ( X 1 , X 2 )t 3 = exp f ( X 1 , X 2 )t x3 = C 3 exp f ( X 1 , X 2 )t
C3

para t = 0 x3 = X 3 x3 (t = 0) = C 3 = X 3
Resumiendo:
x1 = X 1

x2 = X 2

x 3 = X 3 exp f ( X 1 , X 2 )t

(2.76)

Densidad de masa:

con

1 0
Fij = 0 1

Fij =

0
0

xi
X j

= exp f ( X 1 , X 2 )t

? ? exp f ( X 1 , X 2 )t

Los valores ( ? ) no son necesarios para obtener el determinante. Luego:

0
F

f (X1, X 2 )
exp f ( X 1 , X 2 )t

Observemos que segn el enunciado, t = 0 , = f ( x1 , x 2 ) , y segn expresiones (2.76)


concluimos que 0 = f ( X 1 , X 2 ) .
Ejemplo 2.18
Calcular la derivada material

D
para la propiedad cuando esta est descrita en las
Dt

siguientes coordenadas:
r

Materiales: ( X , t ) = X 1t 2 ;

Espaciales: ( x , t ) =

Solucin:

x1t 2
.
(1 + t )

a) Derivada material de ( X , t ) = X 1t 2 :
r
r
D
( X , t ) & ( X , t ) = 2 X 1t
Dt
r

b) Derivada material de ( x , t ) =

x1t 2
:
(1 + t )

r
r
r
r
r ( x , t ) ( x , t )
( x , t )
D
r
( x, t ) =
+ x v =
+
vi
t
t
xi
Dt
r
r
r
r
( x , t ) ( x , t )
( x , t )
( x , t )
v1 +
v2 +
v3
=
+
t
x 2
x3

x1
r

x t 2 ( x , t )
= 1 +
v1 + 0 + 0
t (1 + t ) x1

Universidad de Castilla- La Mancha


Ciudad Real - Espaa

Draft

Por: Eduardo W. V. Chaves (2014)

2 CINEMTICA DEL CONTINUO

169

Necesitamos conocer la velocidad v1 . Partimos del principio de que una propiedad es algo
intrnsico a la partcula luego:
r

( X , t ) = X 1t 2

r r

( X ( x, t ), t ) = ( x, t ) =

x1t 2
(1 + t )

X1 =

x1
(1 + t )

La velocidad queda:

r
D
X 1t 2 = 2 X 1t e 1
v( X , t) =
Dt

x
r
v ( x , t ) = 2 1 t e 1
(1 + t )

Retomando a la derivada material tenemos que:


r
2

x t2 t2
r
x t ( x , t ) 2 x1 t
D
1 2 +
( x, t ) = 1 +
v1 =
X1

t (1 + t ) x1
Dt
(1 + t ) (1 + t ) (1 + t )

2x t
2 x1 t
x
x t2 t2
= 1 1 2 +
2 1 t =
(1 + t ) (1 + t ) (1 + t ) (1 + t ) (1 + t )

Tambin podramos haber obtenido este mismo resultado partiendo que se cumple que
r
r
D
( X , t ) & ( X , t ) = 2 X 1t y adems conociendo las ecuaciones del movimiento
Dt

X1 =

x1
, obtenemos que:
(1 + t )

r
r
D
( X , t ) & ( X , t ) = 2 X 1t
Dt

r r
r r
x
r
D
( X ( x, t ), t ) & ( X ( x, t ), t ) & ( x, t ) = 2 1 t
(1 + t )
Dt

Ejemplo 2.19
Sean las siguientes ecuaciones del movimiento en la descripcin Lagrangiana:
2
x1 = X 1t 2 + 2 X 2 t + X 1
x1 t + 1 2t
forma


Matricial
2
t +1
x 2 = 2t 2
x 2 = 2 X 1 t + X 2 t + X 2
x 0
x = 1 X t + X
0
3
3
3 2 3

0 X 1

0 X 2
1
t + 1 X 3
2

(2.77)

Encontrar las componentes del vector desplazamiento en la descripcin Lagrangiana y


Euleriana.
Solucin:
r

El vector desplazamiento viene dado por u = x X . Reemplazando las ecuaciones del


movimiento (2.77) obtenemos:
u1 = x1 X 1 = X 1t 2 + 2 X 2 t

2
u 2 = x 2 X 2 = 2 X 1t + X 2 t

1
u 3 = x 3 X 3 = 2 X 3 t

que son las componentes del vector desplazamiento en la descripcin Lagrangiana


(material).
Para obtener la descripcin Euleriana debemos obtener las ecuaciones inversas de
(2.77), resultando:

Universidad de Castilla- La Mancha


Ciudad Real - Espaa

Draft

Por: Eduardo W. V. Chaves (2014)

MECNICA DEL MEDIO CONTINUO: PROBLEMAS RESUELTOS

170

2tx 2 x1 (1 + t )

X1 = 3
3t 1 t t 2
x
0
1
2 x1t 2 x 2 (1 + t 2 )
0
x2 X 2 =
3t 3 1 t t 2
3t 3 1 t t 2 x 3

2 x3
1
(t + 2)
X 3 =
2
(t + 2)

r r r
Podemos utilizar, una vez ms, la definicin u = x X , pero ahora reemplazamos las

(1 + t )
2t
X1

1

2
(1 + t 2 )
2t
X 2 = 3
2
t
t
t

3
1

X
3
0
0

coordenadas materiales, obteniendo as las componentes del vector desplazamiento en


coordenadas espaciales (descripcin Euleriana):
2tx 2 x1 (1 + t )

u1 = x1 X 1 = x1 3
3t 1 t t 2

2 x1t 2 x 2 (1 + t 2 )
x
X
x
=

u
2
2
2
2
3t 3 1 t t 2

2 x3
u 3 = x 3 X 3 = x 3
(t + 2)

Ejemplo 2.20
Las
siguientes
ecuaciones
describen el movimiento de las
partculas de un cuerpo (medio
continuo):

X 2 , x2

Configuracin de referencia

x1 = X 1 + 0,2 X 2 t

x2 = X 2
x = X
3
3

B
1

O
D

X 1 , x1

X 3 , x3

Figura 2.1: Configuracin de referencia t = 0 .


En t = 0 , este cuerpo tiene forma de cubo de lado unitario con un vrtice en el origen,
punto O, como se indica en la Figura 2.1. Determinar la configuracin del cuerpo en el
instante t = 2 s .
Solucin:
Para obtener la configuracin actual del cuerpo para el instante t = 2 s analizaremos
independientemente el movimiento de las partculas. La partcula que ocupa posicin O
(origen) en t = 0 tiene coordenadas materiales:
X1 = 0

X2 = 0

X3 = 0

Sustituyendo en la expresin del movimiento:


x1 = 0

x i ( X 1 = 0, X 2 = 0, X 3 = 0, t ) x 2 = 0
x = 0
3
Universidad de Castilla- La Mancha
Ciudad Real - Espaa

Draft

Por: Eduardo W. V. Chaves (2014)

2 CINEMTICA DEL CONTINUO

171

Concluyendo que la partcula del origen no cambia de posicin durante el movimiento.


Las partculas que ocupan la lnea OA en la configuracin inicial tienen como coordenadas
de referencia: ( X 1 , X 2 = 0, X 3 = 0) . En coordenadas espaciales:
x1 = X 1 + 0,2 X 2 t = X 1

x2 = X 2 = 0

x3 = X 3 = 0

Es decir, todas las partculas que estn en la lnea OA no se mueven durante el


movimiento. Anlogamente, podemos verificar que la recta ( X 1 , X 2 = 0, X 3 = 1) en la
configuracin de referencia ( X 1 , X 2 = 0, X 3 = 1) no se mueve:
x1 = X 1 + 0,2 0 2 = X 1

x2 = X 2 = 0

x3 = X 3 = 0

Las partculas que estn en la lnea CB ( X 1 , X 2 = 1, X 3 = 0) en el tiempo t = 2 s ocuparn


las posiciones:
x1 = X 1 + 0,2 1 2 = X 1 + 0,4

x2 = X 2 = 1
x = X = 0
3
3

Luego, todas las partculas que estn en la lnea CB se desplazarn 0,4 segn la direccin
x1 .
Las partculas que estn en la lnea OC en t = 0 , tras el movimiento ocuparn las
posiciones:
x1 = X 1 + 0,2 X 2 t = 0 + 0,2 2 X 2 = 0,4 X 2

x2 = X 2
x = X = 0
3
3

Siguiendo el mismo procedimiento para las partculas restantes, se obtiene la configuracin


final ( t = 2 s ) del cuerpo representada por la Figura 2.2.
x2
0,4

0,4

C
E

C 1

Configuracin del cuerpo


en t = 2 s

E 1
A=A

O
1

G=G

x1

x3

Figura 2.2: Configuracin actual t .

Universidad de Castilla- La Mancha


Ciudad Real - Espaa

Draft

Por: Eduardo W. V. Chaves (2014)

MECNICA DEL MEDIO CONTINUO: PROBLEMAS RESUELTOS

172

Ejemplo 2.21
Considrense las ecuaciones del movimiento dadas por:
x1 = X 1 + t 2 X 2
x1 1
Forma


Matricial
2
x 2 = t 2
x 2 = t X 1 + X 2
x 0
x = X
3
3
3

0 X 1

0 X 2
1 X 3

t2
1
0

1) Determinar la trayectoria de una partcula Q que originalmente, en t 0 , estaba en


X i = (1,2,1) ;
2) Considerando la configuracin actual t = 0,5s . Determinar las componentes de la
velocidad y de la aceleracin de una partcula P que originalmente estaba en
X i = (16
; 4 ;1) ;
15 15
3) Obtener las ecuaciones del movimiento en la descripcin Euleriana;
4) Obtener las componentes de la velocidad y la aceleracin de una partcula que en el
tiempo ( t = 0,5s ) pasa por el punto xi = (1,0,1) .
NOTA: Considerar el sistema internacional de unidades.
Solucin:
1)Utilizando las ecuaciones del movimiento y reemplazando las coordenadas materiales del
punto X i = (1,2,1) , obtenemos:
x 1 = 1 + 2t 2

x2 = 2 + t 2

x3 = 1

Las ecuaciones anteriores representan la trayectoria de la partcula. Para obtener la ecuacin


de la partcula, eliminamos el tiempo de las ecuaciones de movimiento, obteniendo as:
x1 2 x 2 = 3

x3 = 1

Lo cual indica que la partcula se mueve en lnea recta segn la ecuacin ( x1 2 x 2 = 3) en


el plano x3 = 1 , ver figura siguiente.
X 3 , x3

trayectoria de la partcula

( x1 2 x 2 = 3)

x3 = 1

X 2 , x2

X 1 , x1

2) Las componentes de la velocidad y de la aceleracin de la partcula P vienen dadas


respectivamente por:

Universidad de Castilla- La Mancha


Ciudad Real - Espaa

Draft

Por: Eduardo W. V. Chaves (2014)

2 CINEMTICA DEL CONTINUO

173

V1 = 2tX 2
r
r r
Dx componente s
V ( X , t) =

V 2 = 2tX 1
Dt
V = 0
3
A1 = 2 X 2
r
r r
Dv componente s

A2 = 2 X 1
A( X , t ) =
Dt
A = 0
3

Luego, para la partcula situada originalmente en X i = (16


; 4 ;1) , en t = 0,5s tiene:
15 15
4
4
V1 = 2 0,5 15
= 15
m/s

16
16
V 2 = 2 0,5 (15 ) = 15 m / s
V = 0
3

8
4
A1 = 2 15
= 15
m / s2

2
16
32
A2 = 2 (15 ) = 15 m / s
A = 0
3

3) Invirtiendo las ecuaciones del movimiento resulta:


x1 = X 1 + t 2 X 2 X 1 = x1 t 2 X 2

2
2
x2 = t X 1 + X 2 X 2 = x2 t X 1
x = X X = x
3
3
3
3

x1 t 2 x 2
=
X
1
1 t4

x t 2 x1
X 2 = 2
1 t4

X 3 = x3

4) Podemos obtener la velocidad y la aceleracin de la partcula que en el tiempo ( t = 0,5s )


pasa por el punto xi = (1,0,1) simplemente obteniendo la expresin de la velocidad y
aceleracin en coordenadas espaciales:
Velocidad:

x 2 t 2 x1
=
2
v
t
1
1 t4

V1 = 2tX 2

x1 t 2 x 2

= 0,5 s
sustituyen do
=

2
=
V
X
t
2
t

v
t
2
2
1
4
(1, 01)
X1 , X 2
x
1 t
V = 0

3
v 3 = 0

v1 = 15 m / s

16

v 2 = m / s
15

v 3 = 0

Aceleracin:

x 2 t 2 x1
=
2
a
1
1 t4

A1 = 2 X 2

x1 t 2 x 2

=0,5 s
sustituyen do
=

=
2
t

A
X
2
a
2
2
1
4
(1, 01)
X1 , X 2
x
1 t
A = 0

3
a 3 = 0

2
a1 = 15 m / s

32

m / s2
a 2 =
15

a 3 = 0

Podemos obtener la posicin inicial de esta partcula utilizando las ecuaciones inversas del
movimiento obtenidas en el apartado 3, xi (1,0,1) , resultando:

Universidad de Castilla- La Mancha


Ciudad Real - Espaa

Draft

Por: Eduardo W. V. Chaves (2014)

MECNICA DEL MEDIO CONTINUO: PROBLEMAS RESUELTOS

174

x1 t 2 x 2 1 (0,5 2 )(0) 16
=
=
=
X
1
15
1 t4
1 (0,5) 4

x 2 t 2 x1 0 (0,5 2 )(1)
4

=
=
X 2 =
4
4
15
1 t
1 (0,5)

X = x = 1
3
3

Podemos verificar que es la misma partcula P referida en el apartado 2. Es lgico que


hayamos encontrado las mismas velocidades y aceleraciones utilizando las coordenadas
materiales o espaciales, ya que la velocidad y aceleracin son propiedades intrnsecas de una
partcula.
Ejemplo 2.22
La aceleracin de una partcula en un medio continuo est descrita por:
r
r
r r
r r
D v v
=
a ( x , t) =
+ xr v v
Dt t

Demostrar que la aceleracin tambin se puede escribir como:


r
r
v2
Dv v
=
+ xr
Dt t
2

r
r
v2
r
r v
r
v ( xr v ) =
+

r
r
v rot v

Solucin:
Para demostrar la relacin anterior es suficiente demostrar por identificacin de trminos:
v2
r r
xr v v = xr
2

r
r
r
v ( xr v )

En notacin simblica:
v2
xr
2

r
r

r 1


v ( xr v ) = e i
e r (v s e s )
v j v j (v i e i )

2 x i
x r

Utilizado la definicin del operador de permutacin (Captulo 1) podemos expresar el


producto vectorial como:
v2
xr
2

r
r

v
r
1

v ( xr v ) = e i
v j v j (vi e i ) rst s e t

2 x i
x r

v j
v s
1
e k
= e i 2v j
rst itk v i
2
x r
x i

En el captulo 1 del libro se demostr que rst itk = rst kit = rk si ri sk , luego:
v2
xr
2

r
v j
r
r
v ( xr v ) = v j

x i

v j
=vj
x i
v j
=vj
x i

Universidad de Castilla- La Mancha


Ciudad Real - Espaa

Draft

v s
e k
x r

v
v
e i rk si v i s ri sk v i s
x r
x r

v
v
e i v s s v i k e k
x i
x k
e i ( rk si ri sk )v i

e k

Por: Eduardo W. V. Chaves (2014)

2 CINEMTICA DEL CONTINUO

175

r
v j
v2 r
r
v
v
xr v ( xr v ) = v j
e i vs s e k + vi k e k

x
xi
2
i
k

v j
v
v
= sj vs
e i vs s ik e i + vi k e k
xi
xk
xi
= vs

r r
v e
v s
v
v
e i vs s e i + vi k e k = k k vi = xr v v
xi
xi
xi
xi

Q.E.D.

NOTA: Es interesante que el lector compare ste desarrollo con el Ejemplo 1.117
(captulo 1).
Ejemplo 2.23

r r

Consideremos las ecuaciones de movimiento x ( X , t ) y el campo de temperatura T ( x , t )


dado por:
x1 = X 1 (1 + t )

x 2 = X 2 (1 + t )
x = X
3
3

r
T ( x ) = x12 + x 22

Encontrar la tasa de cambio de la temperatura de la partcula P en el tiempo t = 1s , dicha


partcula P estaba en el punto ( X 1 = 3, X 2 = 1, X 3 = 0) en el tiempo t = 0 .
Solucin 1:
En esta primera solucin obtenemos la derivada material de la temperatura Lagrangiana,
r
para ello tenemos que obtener la temperatura en la descripcin Lagrangiana T ( X , t )
(Temperatura Lagrangiana):
r
T ( x ) = x12 + x 22

sustituyen do
las ecuaciones del movimiento

r
T ( X , t ) = X 12 (1 + t ) 2 + X 22 (1 + t ) 2

La derivada material de la temperatura Lagrangiana viene dada por:

r
r
DT dT ( X , t )
&
T ( X , t)
=
= 2 X 12 (1 + t ) + 2 X 22 (1 + t )
Dt
dt

Sustituyendo t = 1s , ( X 1 = 3, X 2 = 1, X 3 = 0) , en la ecuacin anterior obtenemos que:


r
T& ( X , t ) = 2 X 12 (1 + t ) + 2 X 22 (1 + t ) = 2(3) 2 (1 + 1) + 2(1) 2 (1 + 1) = 40

Solucin 2:
En esta solucin alternativa, utilizamos directamente la definicin de derivada material de
r

una propiedad Euleriana, i.e. T& ( x , t ) =

r
r
r
DT T ( x ) T ( x )
vk ( x, t ) .
=
+
Dt
t
x k

De las ecuaciones del movimiento obtenemos:


r
v1 ( X , t ) = X 1
x1 = X 1 (1 + t )
r

velocidad
x 2 = X 2 (1 + t ) v 2 ( X , t ) = X 2
r
x = X

3
3
v 3 ( X , t ) = 0

Universidad de Castilla- La Mancha


Ciudad Real - Espaa

Draft

Por: Eduardo W. V. Chaves (2014)

MECNICA DEL MEDIO CONTINUO: PROBLEMAS RESUELTOS

176

Las ecuaciones del movimiento en la descripcin Euleriana vienen dadas por:


x1

X 1 = (1 + t )

x1 = X 1 (1 + t )
x2

movimiento inverso
x2 = X 2 (1 + t ) X 2 =
(1 + t )
x = X

3
3
X 3 = x3

Luego, es posible obtener la velocidad Euleriana como:


r r
r
r
x1

V1 ( X ( x , t ), t ) = X 1 ( x , t ) = (1 + t ) = v1 ( x , t )

r r

x2
r
r
= v 2 ( x, t )
V 2 = ( X ( x , t ), t ) = X 2 ( x , t ) =
(1 + t )

V3 = v 3 ( xr , t ) = 0

r
Con eso, la derivada material de la temperatura Euleriana, T ( x , t ) , viene dada por:
r
r
T
T
T ( x )
T
DT ( x , t ) & r

T ( x, t ) =
+
v1 +
v2 +
v3
t3
x 2
x 3
Dt
x1
12
= 0 (Campo estacionar io)

x
x
r
T& ( x , t ) = 2 x1 1 + 2 x 2 2 + 0
1+ t
1+ t

2x 2 2x 2
r
2
T& ( x , t ) = 1 + 2 =
( x12 + x 22 )
1+ t 1+ t 1+ t

La posicin de la partcula P en el tiempo t = 1s viene dada por:


x1 = X 1 (1 + t ) = 3(1 + 1) = 6

x 2 = X 2 (1 + t ) = 1(1 + 1) = 2
x = X = 0
3
3

Luego, sustituyendo las coordenadas espaciales en la expresin de la derivada material de la


temperatura obtenemos que:
r
2
2
( x12 + x 22 ) =
T& ( x , t ) = T& ( x1 = 6, x 2 = 2, x 3 = 0, t = 1) =
(6 2 + 2 2 ) = 40
1+ t
1+1
r
Alternativamente, la expresin T& ( x , t ) puede tambin ser obtenido como sigue:
r
T& ( X , t ) = 2 X 12 (1 + t ) + 2 X 22 (1 + t )
2
2
r r
x
x
r 2
r 2
T& ( X ( x , t ), t ) = 2[X 1 ( x , t )] (1 + t ) + 2[X 2 ( x , t )] (1 + t ) = 2 1 (1 + t ) + 2 2 (1 + t )
(1 + t )
(1 + t )
r
2
=
( x12 + x 22 ) = T& ( x , t )
(1 + t )

Ejemplo 2.24
Sea el movimiento:
x i = X i (1 + t )

t > 0

Universidad de Castilla- La Mancha


Ciudad Real - Espaa

Draft

Por: Eduardo W. V. Chaves (2014)

2 CINEMTICA DEL CONTINUO

177

Determinar el campo de velocidad en la descripcin espacial.


Solucin:
La velocidad ser obtenida a travs de la derivada temporal de las ecuaciones del
movimiento:
Vi = x& i =

d
[X i (1 + t )] = X i
dt

(2.78)

Para hallar la velocidad en la descripcin espacial tenemos que obtener la inversa de las
ecuaciones del movimiento y reemplazar en la ecuacin (2.78):
xi

x i = X i (1 + t ) X i = (1 + t )

v = X ( xr , t ) = x i
i
i
1+ t

Ejemplo 2.25

Considrese el campo de temperatura T ( x ) en la descripcin espacial y las ecuaciones del


movimiento siguientes:
T = 2( x12 + x 22 )

i {1,2}
xi = X i (1 + t )

Encuentre en t = 1s la tasa de cambio de temperatura de una partcula que ocupaba la


posicin (1,1) en la configuracin de referencia.
NOTA: Podemos observar que el campo de temperatura es un campo estacionario, es
r
decir T = T ( x ) .
Solucin 1:
En esta primera solucin obtendremos la ecuacin de la temperatura en la descripcin
material:
r
T ( x ) = 2( x12 + x 22 )

reemplazan do las ecuaciones


del movimiento

r
T ( X , t ) = 2 X 2 (1 + t ) 2 + X 2 (1 + t ) 2
1
2

La derivada material viene dada por:

r
r
DT dT ( X , t )
&
T ( X , t) =
=
= 2 2 X 12 (1 + t ) + 2 X 22 (1 + t )
Dt
dt

Reemplazando t = 1s y las coordenadas materiales ( X 1 = 1; X 2 = 1) obtenemos:


T& ( X 1 = 1; X 2 = 1; t = 1) = 16

Solucin 2:
En esta segunda solucin usaremos directamente la definicin de derivada material para
propiedades descritas en coordenadas espaciales:
r
T ( x ) = 2( x12 + x12 )

Universidad de Castilla- La Mancha


Ciudad Real - Espaa

Draft

x i = (1 + t ) X i

i {1,2}

Por: Eduardo W. V. Chaves (2014)

MECNICA DEL MEDIO CONTINUO: PROBLEMAS RESUELTOS

178

r
r
r
DT T ( x ) T ( x ) x k
&
T ( x, t ) =
=
+
Dt
t
x k t
r
r
T ( x )
dado que T ( x ) no es funcin del tiempo
=0:
t

i {1,2}

r
T x1
T x 2
T& ( x , t ) = 0 +
+
x1 {
t
x 2 {
t
V1 = X 1

r
x
x
T& ( x , t ) = 0 + 4 x1 1 + 4 x2 2
1+ t
1+ t

V2 = X 2

r
4 x2 4 x2
T& ( x , t ) = 1 + 2
1+ t 1+ t

La partcula que en la configuracin de referencia ocupaba la posicin (1,1) , en el tiempo


t = 1s estar en la posicin xi = (1 + t ) X i = 2 X i , es decir, ( x1 = 2; x 2 = 2 ):
T& ( x1 = 2; x 2 = 2; t = 1) =

4( 2) 2
1+1

4( 2) 2
1+1

= 16

Ejemplo 2.26
Dadas las siguientes ecuaciones del movimiento:
x1 = X 1exp t + X 3 (exp t 1)

x2 = X 2 + X 3 (exp t exp t )

x3 = X 3

Determinar las componentes de la velocidad y de la aceleracin en coordenadas materiales


y espaciales.
Solucin:
Primero obtenemos la inversa de la ecuacin del movimiento:
x1 = X 1 exp t + X 3 (exp t 1)

t
t
x 2 = X 2 + X 3 (exp exp )
x = X X = x
3
3
3
3

x1 X 1 exp t = x 3 (exp t 1)

x 2 X 2 = x 3 (exp t exp t )
x = X X = x
3
3
3
3

resultando:
X 1 = x1 exp t exp t (exp t 1)

2t
t
X 2 = x 2 x 3 (exp 1)exp
X = x
3
3

(2.79)

o
t
x1 exp

x2 = 0
x 0
3

t
X 1
X 1 exp


1 (exp t exp t ) X 2 inversa
X 2 = 0
X 0
X
0
1
3
3

(exp t 1)

exp t (exp t 1) x1

1 (exp 2 t 1)exp t x 2
x
0
1
3
0

a) La velocidad en la descripcin material viene dada a travs de sus componentes:


V1 = X 1 exp t + X 3 exp t
r

D
V 2 = X 3 exp t + X 3 exp t = X 3 (exp t + exp t )
V i=
x j ( X , t)
Dt
V = 0
3

Universidad de Castilla- La Mancha


Ciudad Real - Espaa

Draft

(2.80)

Por: Eduardo W. V. Chaves (2014)

2 CINEMTICA DEL CONTINUO

179

b) La aceleracin en la descripcin material viene dada por:


A1 = X 1exp t + X 3 exp t
r
r

DV i ( X , t )
Ai ( X , t ) =

A2 = X 3 (exp t exp t )
Dt
A = 0
3

(2.81)

Para obtener la velocidad y la aceleracin en la descripcin espacial es suficiente sustituir en


las ecuaciones (2.80) y (2.81) los valores de X 1 , X 2 , X 3 , dados por la ecuacin (2.79),
resultando:
v1 = x1 + x 3

t
t
v 2 = x 3 (exp + exp )
v = 0
3

Velocidad en la
descripci n espacia l

a1 = x1 + x 3

t
t
a 2 = x 3 (exp exp )
a = 0
3
Aceleraci n en la
descripci n espacial

Ejemplo 2.27
El movimiento de un medio continuo viene definido por las siguientes ecuaciones:
x1 = 12 ( X 1 + X 2 )exp t + 12 ( X 1 X 2 )exp t

t
t
1
1
x 2 = 2 ( X 1 + X 2 )exp 2 ( X 1 X 2 )exp
x = X
3
3
0 t constante

Expresar las componentes de la velocidad en la descripcin material y espacial.


Solucin:
Las componentes de la velocidad utilizando la descripcin material son:
r

Dx1 ( X , t ) 1
1
= ( X 1 + X 2 )exp t ( X 1 X 2 )exp t
V1 =
Dt
2
2

r
Dx 2 ( X , t ) 1
1

= ( X 1 + X 2 )exp t + ( X 1 X 2 )exp t
V 2 =
Dt
2
2

V3 = 0

(2.82)

Para expresar las componentes de la velocidad en la descripcin espacial tenemos que


obtener la inversa de las ecuaciones de movimiento, xi = x i ( X 1 , X 2 , X 3 ) resultando:
(exp t

x1
t
(exp
x
=
2
x
3

+ exp t )
2
exp t )
2
0

(exp t exp t )
2
(exp t + exp t )
2
0

(exp 2t + 1)exp t
X1
1
inversa
X 2 = (exp 2t 1)exp t
X 2
0
3

Universidad de Castilla- La Mancha


Ciudad Real - Espaa

Draft

0
X 1

0 X 2


1 X 3

(exp 2t 1)exp t
(exp 2 t + 1)exp t
0

0 x1

0 x2
2 x 3

Por: Eduardo W. V. Chaves (2014)

MECNICA DEL MEDIO CONTINUO: PROBLEMAS RESUELTOS

180

Para obtener la velocidad en la descripcin espacial es suficiente reemplazar las ecuaciones


anteriores en las expresiones de la velocidad (2.82), resultando:
v1 = x 2

v 2 = x1
v = 0
3

Ejemplo 2.28
Dado el movimiento:
x i = ( X 1 + ktX 2 ) i1 + X 2 i 2 + X 3 i 3

i {1,2,3}

y el campo de temperatura T = x1 + x 2 . Encontrar la tasa de cambio de T para la partcula


que en la configuracin actual est situada en el punto (1,1,1) .
Solucin:
Explcitamente las ecuaciones de movimiento son:
x1 = X 1 + ktX 2

x2 = X 2

x3 = X 3

Reemplazando xi en la expresin de la temperatura, se obtiene la temperatura en la


configuracin material:
r
r
T ( x ) = x1 + x 2 T ( X , t ) = X 1 + ktX 2 + X 2

La derivada material de la temperatura viene dada por:


r
DT D ( X 1 + ktX 2 + X 2 )
1,1,1)
T& ( X , t ) =
= kX 2 = k x 2 (
T& = k
=
Dt
Dt

Solucin alternativa:
La derivada material para una propiedad expresada en la descripcin espacial viene dada
por:
DT T T x k
T& ( x1 , x 2 , x 3 , t ) =
=
+
Dt
t x k t

Considerando T = x1 + x 2 , obtenemos:
T T x1 T x 2 T x 3
+
+
+
T& ( x1 , x 2 , x 3 , t ) =
t x1 t
x 2 {
t
x 3 {
t
{
{
=0

=0

=0

=0

T& ( x1 , x2 , x3 , t ) = kX 2

Hallando la inversa de las ecuaciones del movimiento:


x1 = X 1 + ktX 2
X 1 = x1 ktx 2

inversa
X 2 = x 2
x2 = X 2
x = X
X = x
3
3
3
3
T& ( x1 , x 2 , x 3 , t ) = kX 2 = kx 2

Para la partcula que en la configuracin actual pasa por el punto (1,1,1) :


T& ( x1 = 1, x 2 = 1, x 3 = 1, t ) = k

Ejemplo 2.29
Dado un campo de velocidad estacionario. Se pregunta: Las velocidades de las partculas
son constantes? Justificar la respuesta. En caso negativo, en situacin se cumple.
Universidad de Castilla- La Mancha
Ciudad Real - Espaa

Draft

Por: Eduardo W. V. Chaves (2014)

2 CINEMTICA DEL CONTINUO

Solucin:

181

Un campo ( x , t ) se dice que es estacionario si la tasa local no vara con el tiempo:


r
( x , t )
=0
t

= ( x ) Campo estacionario

(2.83)

Un campo de velocidad estacionario (movimiento estacionario) se indica en la Figura 2.3.


Luego, como se verifica en la Figura 2.3, la representacin del campo para los tiempos t1 y
t 2 no cambia. Pero, eso no implica que las velocidades de las partculas no estn
r
cambiando a lo largo del tiempo. Fijemos nuestra atencin en un punto espacial fijo x * . En
r
r
el tiempo t1 la partcula Q est pasando por x * y tiene velocidad v * , consideremos
tambin una partcula P que est pasando por otro punto y que tiene velocidad tal que
r
r
r
v P (t1 ) v * . Para un tiempo t 2 la partcula P est pasando ahora por el punto x * y si el
r
r
campo es estacionario, la velocidad de la partcula P en x * tiene que ser v * , i.e.
r
r
v P (t 2 ) = v * . Esto se puede verificar fcilmente a travs de la derivada material de la
velocidad (que est asociada siempre con la misma partcula):
r r
r r
r r r
r r r
r r
Dv ( x , t ) r r
v ( x , t )
a ( x, t ) =
+ xr v v ( x , t ) = xr v v ( x ) = a ( x )
Dt
t4
1
42
3
r

(2.84)

= 0 (Estaciona rio)

Para que la aceleracin de la partcula sea igual a cero, adems de campo de velocidad
r
estacionario, el campo de velocidad tambin tiene que ser homogneo, i.e. xr v = 0 .
Verifiquemos tambin que, aunque la velocidad espacial sea independiente del tiempo la
material no necesariamente lo ser, ya que:
r r r r r
r r
v ( x ) = v ( x ( X , t )) = v ( X , t )

(2.85)

2.2 Tensores de Deformacin Finita, Deformacin Homognea


Ejemplo 2.30
Una barra (considerada como un slido de una dimensin) sufre un estiramiento uniforme
de todos sus puntos dado por:
= exp at

donde a = ctte .
r

(2.86)
r r

a) Obtener las ecuaciones del movimiento x = x ( X , t ) ;


b) Obtener las componentes del tensor velocidad de deformacin D .

Universidad de Castilla- La Mancha


Ciudad Real - Espaa

Draft

Por: Eduardo W. V. Chaves (2014)

MECNICA DEL MEDIO CONTINUO: PROBLEMAS RESUELTOS

182

t1

r r
v ( x)

r r
r
r
v ( x * , t1 ) = v * = v Q

Partcula - Q

Partcula - P
r
r
v P v*

r
x*

t2

r r
v ( x)

r r
r
r
v ( x * , t2 ) = v * = v P

Partcula - P
r
x*

Figura 2.3: Campo estacionario.

x1

= exp at

Figura 2.4.
Solucin:
Haciendo el planteamiento en 1D :
=

ds dx
=
= exp at
dS dX

Universidad de Castilla- La Mancha


Ciudad Real - Espaa

dx = exp at dX

Draft

dx = exp

at

dX

(2.87)

Por: Eduardo W. V. Chaves (2014)

2 CINEMTICA DEL CONTINUO

183

x1 = exp at X 1 + C

(2.88)

x = exp 0 X 1 + C X = X + C C = 0

(2.89)

para t = 0 x = X , luego
Obtenemos as las ecuaciones del movimiento:
x1 = exp at X 1

x2 = X 2

x3 = X 3

(2.90)

El campo de velocidad:
v1 =

dx1
= a X 1 exp at = a x1
dt

v2 = 0

v3 = 0

(2.91)

Tensor velocidad de deformacin:


v j
1 v
D ij = i +
2 x j xi

a 0 0
Dij = 0 0 0
0 0 0

(2.92)

Ejemplo 2.31
Considrese las ecuaciones del movimiento dada por las siguientes expresiones:
x1 = X 1 + 2 X 3

x2 = X 2 2 X 3

x3 = X 3 2 X 1 + 2 X 2

Determinar las componentes del tensor de deformacin de Green-Lagrange ( E ).


Solucin 1:
Campo de desplazamiento
u1 = x1 X 1 = 2 X 3

u 2 = x 2 X 2 = 2 X 3
u = x X = 2 X + 2 X
3
3
1
2
3

Partiendo de las componentes del tensor de deformacin de Green-Lagrange en funcin


del gradiente material de los desplazamientos:
Eij =

1 ui u j uk uk
+
+
2 X j X i X i X j

u
= i
X j

sym

1 u uk
+ k
2 X i X j

1 ui u j 1 uk uk
+

=
+
2 X j X i 2 X i X j

donde el gradiente material de los desplazamientos viene dado por:


u1

X 1
u i u 2
=
X j X 1

u 3
X 1

u1
X 2
u 2
X 2
u 3
X 2

u1

X 3 0 0 2
u 2
= 0 0 2

X 3

u 3 2 2 0
X 3

Verifiquemos que el gradiente material de los desplazamientos es un tensor antisimtrico.


Es decir, la parte simtrica es el tensor nulo. Slo queda el trmino:

Universidad de Castilla- La Mancha


Ciudad Real - Espaa

Draft

Por: Eduardo W. V. Chaves (2014)

MECNICA DEL MEDIO CONTINUO: PROBLEMAS RESUELTOS

184

0 0 2 T 0 0 2 2 2 0

1 u u k 1
0 0 2 = 2 2 0
0
0
2
=

E ij = k

2 X i X j 2
2 2 0 2 2 0 0
0 4

Solucin 2:
Podemos aplicar directamente la definicin:
E ij =

1
1
C ij ij = Fki Fkj ij
2
2

donde:
x1

X 1
xi x 2
Fij =
=
X j X 1

x3
X 1

x1
X 2
x 2
X 2
x3
X 2

x1

X 3 1 0 2
x 2
= 0 1 2
X 3

x3 2 2 1
X 3

Luego:
1 0 2 T 1 0 2 1 0 0 2 2 0

1
E ij = 0 1 2 0 1 2 0 1 0 = 2 2 0
2

2 2 1 2 2 1 0 0 1 0
0 4

Ejemplo 2.32
Consideremos una transformacin homognea definida por las siguientes ecuaciones:
x1 = X 1 + 2 X 2 + X 3

x2 = 2X 2

x3 = X 1 + 2 X 3

(2.93)

Demostrar que para una transformacin homognea, vectores paralelos en la configuracin


de referencia, siguen paralelos tras la deformacin.
Para la demostracin considere dos partculas A y B cuyos vectores posicin en la
configuracin de referencia son:
r
X A = e 1 + e 2

r
X B = 2e 1 + 2e 2 + e 3

(2.94)

Solucin:
El vector que une las dos partculas en la configuracin de referencia viene dado por:
r r r
V = B A = e 1 + e 2 + e 3

(2.95)

El gradiente de deformacin:
1 2 1
xi
Fij =
= 0 2 0
X j
1 0 2

(2.96)

Podemos obtener los vectores posicin de las partculas en la configuracin actual:


r
r
dx = F dX

homognea
Transforma
cin

r
r
x =FX

(2.97)

luego,
Universidad de Castilla- La Mancha
Ciudad Real - Espaa

Draft

Por: Eduardo W. V. Chaves (2014)

2 CINEMTICA DEL CONTINUO

xiA

1 2 1 1 3
= 0 2 0 1 = 2
1 0 2 0 1

xiB

185

1 2 1 2 7
= 0 2 0 2 = 4
1 0 2 1 4

(2.98)

y el vector que une estos dos puntos es:


r r
r
(2.99)
v = x B x A = 4e 1 + 2e 2 + 3e 3
r
luego cualquier vector paralelo a V , por ejemplo 2e 1 + 2e 2 + 2e 3 , despus de la
r
transformacin: 8e 1 + 4e 2 + 6e 3 , es paralelo a v .

Ejemplo 2.33
Considere una base ortonormal cartesiana e i y considere una deformacin de corte puro
representada por la deformacin homognea:
r r
x = X + k t X 2e 1

(2.100)

o explcitamente:
x1 = X 1 + k t X 2

x2 = X 2

x3 = X 3

(2.101)

Obtener la forma geomtrica en la configuracin actual de la Figura 2.5 representada por


un rectngulo en la configuracin de referencia.
X2
B

X1

Figura 2.5
Solucin:
El gradiente de deformacin:
1 k t 0
xi
Fij =
= 0 1 0
X j
0 0 1

(2.102)
r

Verificamos que se trata de un caso de deformacin homognea, x = F X + c con c = 0 .


Determinante del Jacobiano:
J = F =1

(2.103)

Verificamos que para este caso no hay dilatancia.


Para la lnea BC , que tiene como coordenadas en la configuracin de referencia ( X 1 , X 2 ,0)
tenemos que:
Universidad de Castilla- La Mancha
Ciudad Real - Espaa

Draft

Por: Eduardo W. V. Chaves (2014)

MECNICA DEL MEDIO CONTINUO: PROBLEMAS RESUELTOS

186

x1 = X 1 + k t X 2

x2 = X 2

x3 = 0

(2.104)

Para la lnea OA , de coordenadas ( X 1 ,0,0) :


x1 = X 1

x2 = 0

x3 = 0

(2.105)

luego no se mueve. La deformacin final se puede apreciar en la Figura 2.6.


x2
B

x1

Figura 2.6
Ejemplo 2.34
Sean las ecuaciones del movimiento:
x1 = X 1 +

2
X2
2

2
X1 + X 2
2

x2 =

x3 = X 3

(2.106)

Se pide:
a) Probar que esta deformacin es un ejemplo de transformacin homognea;
r
b) Determinar las componentes del campo de desplazamientos u en coordenadas
materiales y espaciales;
c) Determinar en la configuracin actual la figura geomtrica formada por las partculas que
en la configuracin de referencia formaban un crculo:
X 12 + X 22 = 2

X3 = 0

d) Obtener las componentes del tensor derecho de deformacin de Cauchy-Green ( C ) y


del tensor de deformacin de Green-Lagrange ( E ).
e) Obtener los valores principales de los tensores C y E .
Solucin:
a) Una transformacin homognea si es el del tipo:
xi = Fij X j

(2.107)

donde

xi
Fij =
=
X j

Universidad de Castilla- La Mancha


Ciudad Real - Espaa

1
2
2
0

Draft

2
2
1
0

0
1

(2.108)

Por: Eduardo W. V. Chaves (2014)

2 CINEMTICA DEL CONTINUO

x1
x =
2
x3

2
2

1
2
2
0

1
0

187

0
X1

0 X 2
1 X 3

(2.109)

Comprobando que es una transformacin homognea. Su forma inversa se obtiene las


ecuaciones del movimiento en coordenadas espaciales:
X1 2
X = 2
2
X 3 0

2
2
0

0 x1

0 x 2
1 x3

X 1 = 2 x1 2 x 2

X 2 = 2 x1 + 2 x 2
X = x
3
3

(2.110)

b) El campo de desplazamientos viene dado por:

r r r
u= x X

componente
s

2
2
X 2 X1 =
X2
u1 = x1 X 1 = X 1 +
2
2

2
2
X1 + X 2 X 2 =
X1
u 2 = x 2 X 2 =
2
2

u 3 = x 3 X 3 = 0

En coordenadas espaciales queda:

(2.111)

u1 = x1 X 1 = x1 2 x1 2 x 2 = x1 + 2 x 2

u 2 = x 2 X 2 = x 2 2 x1 + 2 x 2 = 2 x1 x 2

(2.112)

u 3 = x 3 X 3 = x3 x3 = 0

c) Dada la ecuacin de las partculas que en la configuracin de referencia formaba un


crculo:
X 12 + X 22 = 2

X3 = 0

(2.113)

En la configuracin actual queda:

(2 x

2 x2

) + (
2

2 x1 + 2 x 2

(2.114)

=2

Desarrollando obtenemos que:


3x12 + 3 x 22 4 2 x1 x 2 = 1 (ecuacin de una elipse)

(2.115)

Ver Figura 2.7. Es interesante que el lector verifique las posiciones de las partculas
P ( X 1 = 0; X 2 = 2 ; X 3 = 0) y Q( X 1 = 2 ; X 2 = 2 ; X 3 = 0) en la configuracin deformada.

Universidad de Castilla- La Mancha


Ciudad Real - Espaa

Draft

Por: Eduardo W. V. Chaves (2014)

MECNICA DEL MEDIO CONTINUO: PROBLEMAS RESUELTOS

188

2
1,5
Conf. de Referencia
Conf. Deformada

1
0,5

x2

0
-2

-1

-0,5
-1
-1,5
-2
x1

Figura 2.7: Curva material


d) El tensor derecho de deformacin de Cauchy-Green y el tensor de deformacin de
Green-Lagrange vienen dados, respectivamente, por:
C = FT F

E=

1
(C 1)
2

(2.116)

Luego las componentes de C son:

C ij =

1
2
2
0

2
2
1
0

0
1

1
2
2
0

2
2
1
0

0 =

3
2
2
0

2
3
2
0

(2.117)

Obteniendo los autovalores de (2.117). Ya conocemos un autovalor C 3 = 1 . Para obtener


los otros autovalores es suficiente resolver:
3
C
2
2

2
3
E
2

=0

3+ 2 2
2,91421
C1 =
1

2
C 2 3C + = 0
4
32 2

0,08579
C 2 =
2

(2.118)

Resumiendo:
C1 =

3
3
+ 2 ; C 2 = 2 ; C3 = 1
2
2

Universidad de Castilla- La Mancha


Ciudad Real - Espaa

Draft

(2.119)

Por: Eduardo W. V. Chaves (2014)

2 CINEMTICA DEL CONTINUO

189

Las componentes del tensor E son:

1
E ij = C ij ij
2

1
=
2

3
2

2
3
2
0

2
0

0
1
1
0
0

0 0 1 0 = 2 2
4

0
1 0 0 1

2 2
1
0

0
0

(2.120)

Los valores principales de E son obtenidos por:


1
E
4
2
2

2
2

=0

1
E
4

1+ 2 2
0,95711
E1 =
7
E

4
E2
=0
2 16
1 2 2

0,45711
E 2 =
4

(2.121)

Luego, los valores principales de E son:


E1 =

1+ 2 2
4

; E2 =

1 2 2
4

(2.122)

; E3 = 0

Solucin Alternativa:
Recordar que los tensores C y E son coaxiales, es decir, podemos trabajar en el espacio
principal para obtener los autovalores de E :

E ij =

1
C ij ij
2

3 + 2 2

2
1
= 0
2

0
32 2
2
0

1 + 2 2


0
1 0 0 4

0 0 1 0 = 0

1 0 0 1 0

0
1 2 2
4
0

0
0

Ejemplo 2.35
Consideremos las siguientes ecuaciones del movimiento:
1
;
(2.123)
X1 + X 2
x3 = X 3
2
r
a) Obtener el campo de desplazamiento ( u ) en las descripciones Lagrangiana y Euleriana;
x1 = X 1 +

1
X2
2

x2 =

b) Determinar la curva material en la configuracin actual de una circunferencia material


definido en la configuracin de referencia como:
X 12 + X 22 = 2

X3 = 0

c) Obtener las componentes del tensor derecho de deformacin de Cauchy-Green y el


tensor de deformacin de Green-Lagrange;
d) Obtener los estiramientos principales.
Solucin:
El gradiente de deformacin viene dado por:
2 1 0
xi
1
Fij =
= 1 2 0
X j 2
0 0 2

Universidad de Castilla- La Mancha


Ciudad Real - Espaa

Draft

J = F = 0.75

Por: Eduardo W. V. Chaves (2014)

MECNICA DEL MEDIO CONTINUO: PROBLEMAS RESUELTOS

190

y combinando con las ecuaciones del movimiento dada en (2.123), obtenemos que:
x1
2 1 0 X 1
x = 1 1 2 0 X
2 2
2
x3
0 0 2 X 3

x i = Fij X j

Con eso, podemos verificar que el ejemplo propuesto es un caso de deformacin


r r
homognea con c = 0 . La forma inversa de la ecuacin anterior es:
4
2

X 1 = 3 x1 3 x 2
X1
4 2 0 x1

2
4

X = 1 2 4 0 x

(2.124)
X 2 = x1 + x 2
2 3
2
3
3

X 3
0
0 3 x3
X 3 = x3

r r r
El campo de desplazamiento viene definido por u = x X , con lo cual las componentes

del desplazamiento Lagrangiano resulta ser:


r
1
1

u1 ( X , t ) = x1 X 1 = X 1 + 2 X 2 X 1 = 2 X 2

r
1
1

u i = xi X i u 2 ( X , t ) = x 2 X 2 = X 1 + X 2 X 2 = X 1
2
2

r
u 3 ( X , t ) = x3 X 3 = 0

(2.125)

Las componentes del desplazamiento Euleriana pueden ser obtenidas por reemplazar las
ecuaciones del movimiento Euleriano (2.124) en (2.125), luego:
r r
r

1
u
X
(
( x, t ), t ) = X 2 ( x , t ) =
1

r r
r
1

u 2 ( X ( x , t ), t ) = X 1 ( x , t ) =
2

r r
u ( X ( x , t ), t ) = u ( xr , t ) = 0
3
3

r
1 2
4
x1 + x 2 = u1 ( x , t )

2 3
3
r
1 2
4
x1 + x 2 = u 2 ( x , t )

2 3
3

(2.126)

Las partculas que pertenecen a la circunferencia X 12 + X 22 = 2 en la configuracin de


referencia, formarn una nueva curva en la configuracin actual y que viene definida por:
2

2
4
4
2
X 12 + X 22 = 2 x1 x 2 + x1 + x 2 = 2 20 x12 32 x1 x 2 + 20 x 22 = 18
3
3
3
3

el cual es la ecuacin de una elipse (Figura 2.8 muestra la curva material en las diferentes
configuraciones).
Las componentes de C y E pueden ser obtenidas a travs de las definiciones C = F T F
y E=

1
(C 1) :
2
Cij = Fki Fkj

Universidad de Castilla- La Mancha


Ciudad Real - Espaa

2 1 0 2 1 0 1,25 1 0
1
Cij = 1 2 0 1 2 0 = 1 1,25 0
4
0 0 2 0 0 2 0
0 1

Draft

Por: Eduardo W. V. Chaves (2014)

2 CINEMTICA DEL CONTINUO

1
Eij = Cij ij
2

191

1,25 1 0 1 0 0 0,125 0.5 0

1
Eij = 1 1,25 0 0 1 0 = 0.5 0,125 0
2
0 1 0 0 1 0
0
0
0

En el espacio principal de C sus componentes vienen dadas por:


21

C ij = 0
0

0
22
0

0
23

C ij = 0
0

0
3

0
2
0

donde i son los estiramientos principales. A continuacin, calculamos los autovalores de


C:
1,25 C
1
C1 = 2,25
= 0 C 2 2,5C + 0,5625 = 0
1
1,25 C
C 2 = 0,25

21

C ij = 0
0

0
22
0

0 2,25
0
0

0= 0
0,25 0
23 0
0
1

0
0

0
2
0

0 1,5 0 0

0 = 0 0,5 0
3 0
0 1

2.0

curva material
1.5
Reference Conf.
Current Conf.

1.0
0.5
x2

0.0
-2

-1

-0.5
-1.0
-1.5
-2.0
x1

Figura 2.8: Curva material.

Universidad de Castilla- La Mancha


Ciudad Real - Espaa

Draft

Por: Eduardo W. V. Chaves (2014)

MECNICA DEL MEDIO CONTINUO: PROBLEMAS RESUELTOS

192

Ejemplo 2.36
Probar que

[(detF ) F T ] = 0
r

Xr

(2.127)

r
r
dA .
Datos: Relacin de Nanson da = J F T dA da = da n = J F T N

Solucin:
Considerando la relacin de Nanson en notacin indicial da n i = J Fki1N k dA , donde
J = detF . Podemos integrar en toda la superficie:
1
ki N k dA

n da = J F
i

(2.128)

S0

Notar que dado una funcin escalar f se cumple que:

f da =

f ,i dV =

f
dV
xi

Haciendo f = 1 , obtenemos que:

da = 0 i

Retomando la ecuacin (2.128), y aplicando el teorema de la divergencia de Gauss para la


segunda integral, obtenemos que:

n da = 0 = J F
i

1
ki N k dA =

1
ki
k

dV0 =

V0

S0

(J F ),

Xr

V0

J Fki1 dV0 = 0 i
X k

(2.129)

[(detF ) F T ] dV0 = 0
r

V0

Luego si es vlido para todo el volumen, tiene que ser vlido localmente:
Xr

Ejemplo 2.37

[(detF ) F T ] = 0
r

r r

sym

(2.130)

r r

sym

Demostrar que a) E& = F T Xr u& ( X , t)


y b) D = xr u& ( x , t) , donde E es el tensor de
deformacin de Green-Lagrange y D es el tensor velocidad de deformacin.
Solucin:
a)

) [

] [

1
D
D 1 T
1
E&
E=
F F 1 = F& T F + F T F& = ( F T F& ) T + ( F T F& ) = F T F&

2
Dt
Dt 2
2

sym

Notar que:

r r
u& ( X , t )
D xi ( X , t )
Dxi ( X , t )
&
[
F&ij =
=
=
= Xr u& ( X , t ) ij
u i ( X , t )] = i
Dt X j X j
Dt
X j
X j

con lo cual, demostramos que:

Universidad de Castilla- La Mancha


Ciudad Real - Espaa

Draft

Por: Eduardo W. V. Chaves (2014)

2 CINEMTICA DEL CONTINUO

D
E&
E = F T F&
Dt

sym

193

r r sym
= F T Xr u& ( X , t )

b)

] [

r r sym
r
r T
r r
1
1
T
sym
l + ( l ) = xr v + ( xr v ) = ( xr v ( x , t ))
= xr u& ( x , t )
2
2
r r
r r
donde hemos considerado que v ( x , t ) = u& ( x , t ) .
D=l

sym

Ejemplo 2.38
Considrese el siguiente campo de velocidad:
v1 = 5 x2 + 2 x3

v2 = 5 x1 3 x3

v3 = 2 x1 + 3 x2

Demostrar que dicho movimiento corresponde a un movimiento de slido rgido.


Solucin:
En primero vamos obtener el gradiente espacial de la velocidad
vienen dadas por:
v1

x1
r
vi ( x, t ) v 2
l ij =
=
x j
x
1

v3
x1

v1
x 2
v 2
x 2
v3
x 2

(l ), cuyas componentes

v1

x3 0 5 2
v 2
= 5
0 3

x3

0
v3 2 3
x3

(2.131)

Recordar que ( l ) podemos descomponer en una parte simtrica ( D ) y otra antisimtrica


( W ). Tambin podemos verificar que l = D + W = W . Ya que D = 0 , el movimiento es de
slido rgido.
Ejemplo 2.39
Consideremos el siguiente campo de velocidad:
v1 = 3 x2 + 1x3

v2 = 3x1 5 x3

v3 = 1x1 + 5 x2

Demostrar que el movimiento correspondiente se trata de un movimiento de slido rgido.


Solucin: Calculamos las componentes del gradiente espacial de velocidad
v1

x1
r
vi ( x , t ) v 2
=
l ij =
x j
x
1

v3
x1

v1
x 2
v 2
x 2
v 3
x 2

(l ):

v1

x3 0 3 1
v 2
= 3
0 5 = l ijanti

x3

0
v3 1 5
x3

Teniendo en cuenta que l puede ser descompuesto de forma aditiva en una parte
simtrica ( l sym D ) y una antisimtrica ( l anti W ), i.e. l = D + W , podemos que
concluir que D = 0 , el cual caracteriza un movimiento de slido rgido.
Ejemplo 2.40
El campo de desplazamientos de un cuerpo viene descrito por las siguientes ecuaciones:
u1 = 3 X 12 + X 2
Universidad de Castilla- La Mancha
Ciudad Real - Espaa

u 2 = 2 X 22 + X 3
Draft

u3 = 4 X 32 + X 1
Por: Eduardo W. V. Chaves (2014)

MECNICA DEL MEDIO CONTINUO: PROBLEMAS RESUELTOS

194

Determinar el vector dx (configuracin actual) cuyo vector en la configuracin de


r
referencia estaba representado por dX y pasaba por el punto P(1,1,1) .
X 3 , x3

r
dX

dX 1
dX k = dX 2
dX 3

X 2 , x2
X 1 , x1

Solucin:

Para determinar el vector dx necesitamos obtener el gradiente de deformacin F . Las


componentes del gradiente de deformacin material pueden obtenerse utilizando
directamente la ecuacin:
ui
Fij = ij +
X j

1
1 + 6 X 1

1 + 4X 2
Fij = 0
1
0

1 + 8 X 3
0
1

Las componentes del gradiente de deformacin en el punto P(1,1,1) son:


Fij

7 1 0
= 0 5 1
1 0 9
r

Una vez obtenido el gradiente de deformacin F , las componentes del vector dx vienen
dadas por:
dx i = Fij dX j

dx1 7 1 0 dX 1 7 dX 1 + dX 2
dx = 0 5 1 dX = 5dX + dX
2
3
2
2
dx 3 1 0 9 dX 3 dX 1 + 9 dX 3

Ejemplo 2.41
Dadas las componentes del campo de desplazamientos siguientes:
u1 = 2 X 12 + X 1 X 2

2
u 2 = X 2
u = 0
3

para X 1 0; X 2 0

Se pide:
a) Encontrar el vector en la configuracin de referencia cuyo vector en la configuracin

actual es dx i = (OP ) i = (1,0,0) ;


b) Encontrar el estiramiento de un elemento de lnea que en la configuracin actual es el

vector dx i = (OP ) i = (1,0,0) y que pasa por el punto P(1,0,0) .


Solucin:
Universidad de Castilla- La Mancha
Ciudad Real - Espaa

Draft

Por: Eduardo W. V. Chaves (2014)

2 CINEMTICA DEL CONTINUO

195

a) Dadas las componentes del desplazamiento podemos obtener las componentes del
movimiento segn la ecuacin:
u i = xi X i
x1 = X 1 + 2 X 12 + X 1 X 2

do
sustituyen

x2 = X 2 + X 22
valores de u1 ,u 2 ,u3
x = X
3
3

x1 = u1 + X 1

x2 = u 2 + X 2
x = u + X
3
3
3

Podemos verificar que no se trata de una deformacin homognea, ya que una recta en la
configuracin de referencia no sigue siendo una recta en la configuracin deformada.
Como ejemplo consideremos que unas partculas que ocupan una recta en la configuracin
de referencia, tras la deformada estas partcula ya no formarn una recta en la configuracin
actual, ver Figura 2.9.
2,5

x2

1,5

1
0,5

Conf. Actual

Conf. Referencia

0
0

0,5

1,5

2,5

3,5

4,5

x1

Figura 2.9: Deformacin de la recta (1,1).


Reemplazando el punto P ( x1 = 1, x 2 = 0, x3 = 0) en las ecuaciones del movimiento anterior,
resulta:
1 = X 1 + 2 X 12 + X 1 X 2

2
0 = X 2 + X 2
0 = X
3

Podemos decir de inmediato que X 3 = 0 y X 22 = X 2 , debido a la restriccin del problema


X 2 0 , luego la nica solucin posible es X 2 = 0 . Sustituyendo los valores de X 3 = 0 y
X 2 = 0 en la primera ecuacin resulta:
X 1 = 1
resolviend o

1 = X 1 + 2 X 12
X 1 = 12

Debido a la restriccin X 1 0 , la nica solucin posible es X 1 = 12 . As:


( X 1 = 12 ;
Universidad de Castilla- La Mancha
Ciudad Real - Espaa

X 2 = 0;

Draft

X 3 = 0)
Por: Eduardo W. V. Chaves (2014)

MECNICA DEL MEDIO CONTINUO: PROBLEMAS RESUELTOS

196

Es decir, la partcula que en la configuracin actual ocupa ( x1 = 1, x 2 = 0, x 3 = 0) , en la


configuracin de referencia ocupaba el lugar ( X 1 = 12 ; X 2 = 0; X 3 = 0;) .
r

Calculemos el incremento en esta direccin dX , segn la ecuacin dx j = F jk dX k y su


forma inversa:
dX 1
dx1
dX = F 1 dx
jk
2
2
dX 3
dx 3

Para lo cual debemos calcular antes las componentes del gradiente de deformacin material
F jk . Podemos hacerlo directamente partiendo de la definicin:

F jk

x1

X 1
x
= 2
X
1
x 3
X 1

x1
X 2
x 2
X 2
x 3
X 2

x1

X 3 (1 + 4 X + X )
X1
1
2
x 2
=
0
1 + 2X 2
X 3

0
0
x 3
X 3

0
0
1

Para el punto P( X 1 = 12 ; X 2 = 0; X 3 = 0;) obtenemos las componentes del gradiente de


deformacin:
F jk

3 12 0

= 0 1 0
0 0 1

Calculamos la inversa ( F jk ) 1 F jk1 . Por definicin:


T

F jk1

0 0
1
1 0,5 0
1
1
1

=
adj( F jk ) = 0,5 3 0 = 0
3
0
3
3
F jk
0
0
0
3
0 3

Luego:
dX 1
dx1
1 0,5 0 1 13

dX = F 1 dx = 1 0
3
0 0 = 0
jk
2
2

3
dX 3
dx 3
0
0
3 0 0

b) El estiramiento viene dado por:


=

Universidad de Castilla- La Mancha


Ciudad Real - Espaa

r
dx
r
dX

Draft

12 + 0 + 0

(13 )2 + 0 + 0

=3

Por: Eduardo W. V. Chaves (2014)

2 CINEMTICA DEL CONTINUO

197

0,1

x2

0,08
0,06

Conf. Actual

0,04

Conf. Referencia

0,02
0
0

0,5

1,5

2,5

3,5

x1

Figura 2.10: Deformacin de la recta OT .


Ejemplo 2.42
Utilizar

la

DF
D
[det (F )] = ij cof Fij
Dt
Dt

( )

definicin

para

obtener

la

expresin

D
[det (F )] = Jv i ,i .
Dt

Solucin: Considerando que Fij =

x i
, luego:
X j

x
x
D
[det (F )] = D i cof Fij = D i cof Fij = D (v i )cof Fij
Dt
Dt X j
X j Dt
X j
r
o an considerando que v i ( x ( X , t ), t ) , podemos decir que:

( )

( )

( )

D
[det (F )] = vi x k cof Fij
x k X j
Dt

( )

Y considerando la definicin del cofactor: [cof (Fij )]T = (Fij )1 det (Fij ) , obtenemos que:
D
[det (F )] = vi xk Fij
Dt
xk X j

( )

( )

det Fij =

vi
Fkj F ji
x k

( )

( )

det Fij =

vi
v
ki det Fij = i det Fij
x k
xi

( )

( )

= Jvi ,i

Solucin alternativa: ver Ejemplo 1.115 en el captulo 1.


Ejemplo 2.43

Dado el diferencial dx , hallar su derivada material.


Solucin:
r
r
r
r
r
r r
D r D
D
D
dx =
( F dX ) =
( F ) dX + F
( dX ) = l F
d3
X = l dx xr v dx
1
2
r
Dt
Dt
Dt
Dt
1
424
3
dx
r
0

Las componentes vienen dadas por:


v
D r
dx = v i , k dx k = i dx k

x k
Dt i

Universidad de Castilla- La Mancha


Ciudad Real - Espaa

Draft

Por: Eduardo W. V. Chaves (2014)

MECNICA DEL MEDIO CONTINUO: PROBLEMAS RESUELTOS

198

Ejemplo 2.44
Considerando las ecuaciones del movimiento:
x1 = X 1 + 4 X 1 X 2

x2 = X 2 + X 22

x3 = X 3 + X 32

Encontrar el tensor de deformacin de Green-Lagrange E .


Solucin:
El tensor de deformacin de Green-Lagrange viene dado por:
1 T
( F F 1)
2

E=

Eij =

1
( Fki Fkj ij )
2

(2.132)

Considerando las ecuaciones del movimiento podemos obtener las componentes del
gradiente de deformacin material F :
x1

X 1
x k x 2
Fkj =
=
X j X 1

x 3
X 1

Fki Fkj

x1

X 3 (1 + 4 X )
4X1
2
x 2
0
1 + 2X 2
=
X 3

0
0
x 3
X 3

x1
X 2
x 2
X 2
x 3
X 2

0
1 + 2 X 3

0
0 (1 + 4 X 2 )
4X1
(1 + 4 X 2 )

= 4X1
1 + 2X 2
0
0
1 + 2X 2

0
0
1 + 2 X 3
0
0
2
(1 + 4 X 2 )

(1 + 4 X 2 ) 4 X 1
0

2
2
0
= (1 + 4 X 2 ) 4 X 1 ( 4 X 1 ) + (1 + 2 X 2 )

X
0
0
(
1
2
)
+
3

0
0
1 + 2 X 3

Reemplazando la relacin anterior en la ecuacin (2.132) obtenemos que las componentes


del tensor de deformacin de Green-Lagrange vienen dadas por:
(1 + 4 X 2 ) 2 1

(1 + 4 X 2 ) 4 X 1
0

1
2
2
0
E ij = (1 + 4 X 2 ) 4 X 1 ( 4 X 1 ) + (1 + 2 X 2 ) 1

2
2

0
0
(
1
+
2
)

1
X
3

Ejemplo 2.45
Obtener los invariantes principales de E en funcin de los invariantes principales de C y
b.
Solucin:
Los invariantes principales de E son:
I E = Tr ( E )

II E =

1 2
I E Tr ( E 2 )
2

III E = det ( E )

1
2

Considerando que E = (C 1) , resulta que:


Primer Invariante:
1
1
1
1
I E = Tr ( E ) = (C 1) = Tr (C 1) = [Tr (C ) Tr (1) ] = (I C 3)
2
2
2
2

Universidad de Castilla- La Mancha


Ciudad Real - Espaa

Draft

Por: Eduardo W. V. Chaves (2014)

2 CINEMTICA DEL CONTINUO

1 2
I E Tr ( E 2 )
2

Segundo Invariante: II E =

donde

I E2

199

1
1

= (I C 3) = I C2 6 I C + 9 ,
4
2

[ ( )

1
1
1
1

Tr ( E ) = Tr (C 1) = Tr (C 1) 2 = Tr C 2 2C + 1 = Tr C 2 2 Tr (C ) + Tr (1 )
4
4
4
2

1
= Tr C 2 2 I C + 3
4
2

[ ( )

Para obtener Tr (C 2 ) , adoptaremos el espacio de las direcciones principales donde se


cumple que:
C12

C C = C 2 = 0
0

0 Tr C 2 = C12 + C 22 + C 32
C 32

0
C 22

( )

Pero considerando la siguiente relacin:

I C2 = (C1 + C 2 + C 3 ) = C12 + C 22 + C 32 + 2 C1 C 2 + C1 C 3 + C 2 C 3
1444424444
3
2

II C

C12

C 22

C 32

I C2

2 II C

Luego:
Tr ( E 2 ) =

1 2
I C 2 II C 2 I C + 3
4

Con lo cual concluimos que el segundo invariante viene dado por:


II E =

) (

1 1 2
1
1
I C 6 I C + 9 I C2 2 II C 2 I C + 3 = ( 2 I C + II C + 3)

2 4
4
4

Tercer Invariante:
3

III E

1
1
= det ( E ) = det (C 1) = det [(C 1)]
2
2

Trabajando en el espacio principal de C se cumple que:


C1 1
0
det (C 1) = 0
C2 1
0

0
0

= (C1 1)(C 2 1)(C 3 1)

C3 1

= C1C 2 C 3 C1C 2 C1 C 3 C 2 C 3 + C1 + C 2 + C 3 1 = III C II C + I C 1

luego III E =

1
( III C II C + I C 1) . Resumiendo:
8

1
(I C 3 )
2
1
INVERSA
II E = ( 2 I C + II C + 3)

4
1
III E = ( III C II C + I C 1)
8

IE =

Universidad de Castilla- La Mancha


Ciudad Real - Espaa

Draft

I C = 2I E + 3
II C = 4 II E + 4 I E + 3
III C = 8 III E + 4 II E + 2 I E + 1

Por: Eduardo W. V. Chaves (2014)

MECNICA DEL MEDIO CONTINUO: PROBLEMAS RESUELTOS

200

Ejemplo 2.46
Sea = (I C , II C , III C ) una funcin de valor-escalar, donde I C , II C , III C son los
invariantes principales del tensor derecho de deformacin de Cauchy-Green C . Obtener la
derivada de con respecto a C y con respecto a b . Comprobar que la siguiente igualdad
es vlida F ,C F T = ,b b .
Solucin:
Utilizando la regla de la cadena podemos obtener que:

,C =

(I C , II C , III C ) I C
II C
III C
=
+
+
I C C II C C
III C C
C

(2.133)

Considerando las derivadas parciales de los invariantes vistas en el captulo 1, podemos


decir que:
I C
II C
III C
=1 ,
= IC 1 C T = IC 1 C ,
= III C C T = III C C 1 , luego:
C
C
C

,C =

,C

(I C 1 C ) + III C C 1
1+
I C
II C
III C

I C 1
III C C 1
+
=
C+
I
I
I
I
I
I
I
I

C
C
C
C

(2.134)

Tambin es vlido que:


,b =
I b 1
III b b 1
+
b+
II b
III b
I b II b

(2.135)

Haciendo una contraccin por la izquierda con F y por la derecha por F T en la relacin
(2.134) obtenemos que:

F ,C F T =
I C F 1 F T
F C F T +
III C F C 1 F T
+
I
I
I
I
I
I
I
I

C
C
C
C

(2.136)
Y considerando las siguientes relaciones:
F 1 F T = F F T = b
C = F T F F C F T = F F T F F T = b b = b2

Y considerando la relacin C 1 = F 1 b 1 F concluimos que:


C 1 = F 1 b 1 F F C 1 F T = F F 1 b 1 F F T = b 1 b

Luego la expresin (2.136) puede ser rescrita como:


F ,C F T =
I C b
b +
III C b 1 b
+
II C
III C
I C II C

+
F ,C F T =
I C 1
b+
III C b 1 b
II C
III C
I C II C

Tambin es vlido que:

Universidad de Castilla- La Mancha


Ciudad Real - Espaa

Draft

Por: Eduardo W. V. Chaves (2014)

2 CINEMTICA DEL CONTINUO

201

F ,C F T =
I b 1
B+
III b b 1 b
+
II b
III b

I b II b
F ,C F T = ,b b Q.E.D.

Verificando la expresin (2.135) podemos concluir que la relacin ,b b = b ,b es vlida,


indicando que los tensores ,b y b son coaxiales.
Ejemplo 2.47
Demostrar que el tensor de deformacin de Green-Lagrange ( E ) y el tensor derecho de
deformacin de Cauchy-Green ( C ) son tensores coaxiales.
Solucin:
Dos tensores son coaxiales cuando presentan las mismas direcciones principales. Tambin
se puede demostrar que son coaxiales cuando se cumpla la relacin:
C E = E C

Partiendo de la definicin C = 1 + 2 E concluimos que:


C E = (1 + 2 E ) E = 1 E + 2 E E = E 1 + 2 E E = E (1 + 2 E ) = E C

Con lo cual se demuestra que los tensores E y C son tensores coaxiales.


Ejemplo 2.48

Obtener la relacin E& = F T D F , partiendo de la definicin (ds ) 2 (dS ) 2 = dX 2 E dX .


Obtener tambin la relacin entre

D
(ds ) 2 y D .
Dt

Solucin:

Tomando la derivada material de la relacin (ds ) 2 (dS ) 2 = dX 2 E dX :

D
(ds ) 2 (dS ) 2
Dt

El trmino

r
D
D r
dX 2 E dX
(ds ) 2
=
Dt
Dt
r&
r
r
r
r
r&
D r r
[dx dx ] = 2d{
=
X E dX + 2dX E& dX + 2dX E d{
X
Dt
=0
=0
r
r
r D r
&
= 2 dx
[dx ] = 2dX E dX
Dt
=

D r
[dx ] , puede expresarse de la siguiente forma:
Dt

r
D
D r
[
]
d
d
x
F
X

=
Dt
Dt r

& dX
F
=

= l F dX

D
D x k
[dx k ] =

Dt X i
Dt
D x k

Indicial


=
Dt X i

= k dX i

X i

dX i

D x k
dX i =

dX i
DX i t

Con lo cual podemos decir que:

Universidad de Castilla- La Mancha


Ciudad Real - Espaa

Draft

Por: Eduardo W. V. Chaves (2014)

MECNICA DEL MEDIO CONTINUO: PROBLEMAS RESUELTOS

202

r
r
2dX E& dX

r D r
[dx ]
= 2 dx
r
r Dt
= 2 dx l F dX
r
r
= 2 F dX l F dX
r
r
= 2 dX F T l F dX

Podemos descomponer de forma aditiva el tensor gradiente espacial de velocidad ( l ) en


una parte simtrica ( D ) y otra antisimtrica ( W ):
r
r
2dX E& dX

r
r
= 2 dX F T l F dX
r
r
= 2dX F T (D + W ) F dX
r
r
r
r
= 2 dX F T D F dX + 2 dX F T W F dX
r
r
= 2 dX F T D F dX
r
r
r
r
r
r
Observemos que dX F T W F dX = dx W dx = W : (dx dx ) = 0 , ya que el tensor W
r
r
es antisimtrico y (dx dx ) un tensor simtrico. Con lo que concluimos que:

E& = F T D F

D
(ds ) 2 y D queda:
Dt
r
r
r
r
D
(ds ) 2 = 2dX F T D F dX = 2dx D dx
Dt

Con lo cual la relacin entre

Ejemplo 2.49
Obtener la tasa del determinante del Jacobiano ( J& ) en funcin de la tasa del tensor de
deformacin de Green-Lagrange ( E& ) y tambin en funcin de la tasa del tensor derecho
de deformacin de Cauchy-Green ( C& ).
Solucin:
Considerando que J& = J Tr (D ) , donde D es el tensor tasa de deformacin, y est
relacionado con E& por D = F T E& F 1 , luego:

) (

J& = J Tr (D) = J Tr F T E& F 1 = J F T E& F 1 : 1

En notacin indicial queda:


J&

= J Fki1 E& kp F pj1 ij

= J Fki1 F pi1 E& kp

= J F 1 F T : E&

= J C 1 : E&
J
= C 1 : C&
2

An podemos expresar J& en funcin de F& , para ello consideremos la siguiente relacin

1
E& kp = F&sk Fsp + Fsk F&sp . Luego J& an puede se expresado por:
2

1
J 1 1 &
J& = J Fki1 F pi1 E& kp = J Fki1 F pi1 F&sk Fsp + Fsk F&sp =
Fki F pi Fsk Fsp + Fki1 F pi1 Fsk F&sp
2
2
J
J 1 &
= si Fki1 F&sk + si F pi1 F&sp =
Fks Fsk + F ps1 F&sp = JFts1 F&st = JF T : F&
2
2

Resumiendo, podemos expresar la tasa del determinante del Jacobiano como:


Universidad de Castilla- La Mancha
Ciudad Real - Espaa

Draft

Por: Eduardo W. V. Chaves (2014)

2 CINEMTICA DEL CONTINUO

203

J 1 &
C :C
= JF T : F&
2
J
= J Tr (C 1 E& ) = Tr (C 1 C& ) = J Tr ( F& F 1 )
2

J& = J Tr (D) = J C 1 : E&

Ejemplo 2.50
Las componentes del campo de desplazamiento de un medio continuo son:
u 1 = 0,1X 22

u2 = 0

u3 = 0

Se pide:
a) Es una deformacin posible en un cuerpo continuamente deformable? Justifique su
respuesta;
b) Determinar el tensor derecho de deformacin de Cauchy-Green;

c) Encontrar los vectores deformados relativos a los vectores materiales b = 0,01e 1 y


r
c = 0,015 e 2 , los cuales pasaban por el punto P (1,1,0) en la configuracin de referencia;
r

d) Determinar los estiramientos de estos vectores, b y c , en el punto P (1,1,0) ;


e) Determinar el cambio sufrido por el ngulo comprendido entre los dos vectores.
Solucin:
a) Para que un movimiento sea posible, el determinante del Jacobiano tiene que ser
positivo. El gradiente de deformacin material viene dado por:
1 0 0 0 0,2 X 2
u i
= 0 1 0 + 0
Fij = ij +
0
X j
0 0 1 0
0

0 1 0,2 X 2
1
0 = 0
0
0 0

0
0
1

Calculando el determinante: Fij = J = 1 > 0 . Luego es un movimiento posible.


b) El tensor derecho de deformacin de Cauchy-Green viene definido por C = F T F ,
luego las componentes viene dadas por:
1
C ij = 0,2 X 2
0

0 0 1 0,2 X 2
1 0 0
1
0 1 0
0

0 1
0 = 0,2 X 2
1 0

0,2 X 2
0
2
2
0,2 X 2 + 1 0
0
1

c) El vector b = 0,01e 1 en el punto P(1,1,0) se deforma segn al criterio:


r
b = F

b
P

b1 1 0,2 1 0 0,01 0,01


b = 0
1
0 0 = 0
2
b 3 0
0
1 0 0
r

y el vector c = 0,015e 2 en la configuracin actual queda:


c 1 1 0,2 1 0 0 0,003
c = 0
1
0 0,015 = 0,015
2
c 3 0
0
1 0 0

Universidad de Castilla- La Mancha


Ciudad Real - Espaa

Draft

Por: Eduardo W. V. Chaves (2014)

MECNICA DEL MEDIO CONTINUO: PROBLEMAS RESUELTOS

204

d) Para obtener el estiramiento, utilizamos directamente la ecuacin:


r
b
r
b

br =

0,01 2
=1
0,01

El estiramiento del vector c viene dado por:


cr

r
c
r
c

0,003 2 + 0,015 2
= 1,0198 1,02
0,015

Solucin Alternativa: Teniendo en cuenta que M = M C M y evaluando C en el punto


P obtenemos que:
1
C ij ( X 1 = 1, X 2 = 1, X 3 = 0) = 0,2 X 2
0

0
0,2
+ 1 0
0
1
0,2 X 2
2

X 22

1 0,2 0
= 0,2 1,04 0
0
0 1

Luego aplicando b = b C b y c = c C c , podemos obtener que:


b

0, 2 0 1
1

= [1 0 0] 0,2 1,04 0 0 = 1
0
0
1 0

0, 2 0 0
1

= [0 1 0] 0,2 1,04 0 1 = 1,04


0
0
1 0

b = 1

c = 1,0198
r

e) En la configuracin actual el ngulo que forman los vectores, b y c , puede obtenerse


segn la relacin:
r r
b c
cos = r r
b c

cos =

(0,01e 1 + 0e 2 + 0e 3 ) (0,003e 1 + 0,015 e 2 + 0e 3 )


0,01

0,003 + 0,015

0,00003
= 0,196116135
0,01 0,000234

= arccos(0,196116135) 78,69

Estos dos vectores, en la configuracin de referencia, formaban un ngulo de 90 , luego el


cambio de ngulo ser:
= 90 78,69 = 11,3

Solucin Alternativa: Dadas dos direcciones en la configuracin de referencia representadas


por sus versores M y N , el ngulo formado por estos versores en la configuracin actual
(tras la deformada) viene dado por:
cos =

M C N
M C N
=
M N
M C M N C N

Haciendo M = b , N = c se cumple que:

Universidad de Castilla- La Mancha


Ciudad Real - Espaa

Draft

Por: Eduardo W. V. Chaves (2014)

2 CINEMTICA DEL CONTINUO

205

0, 2 0 0
1

b C c = [1 0 0] 0,2 1,04 0 1 = 0,2


0
0
1 0

Luego cos =

0,2
b C c
b C c
=
=
= 0,196116135
b c
1 1,04
b C b c C c

Ejemplo 2.51
Obtener una expresin de la densidad de masa en funcin del tercer invariante del tensor
de deformacin de Green 0 = 0 ( III C ) .
Solucin:
Partiendo de la ecuacin:

0 ( X ) = ( x, t ) J ( x, t )
y considerando que el tercer invariante viene dado por III C = det (C ) = J 2 , obtenemos que:
J = III C , luego:

0 =

(2.137)

III C

Ejemplo 2.52
En un cierto instante, el campo de desplazamientos de un medio continuo es:
u1 = (a1 1) X 1

u 2 = (a 2 1) X 2 + a1X 1

u 3 = (a 3 1) X 3

donde es una constante. Determinar a1 , a 2 y a 3 sabiendo que el slido es


incompresible, que un segmento paralelo al eje X 3 no se alarga y que el rea de un
elemento situado en el plano X 1 X 3 no se ha modificado.
Solucin:
r

Partiendo de la definicin del campo de desplazamientos ( u = x X ):


u1 = x1 X 1 = (a1 1) X 1

x1 = a1 X 1

u 2 = x 2 X 2 = (a 2 1) X 2 + a1X 1
u 3 = x 3 X 3 = (a 3 1) X 3

x 2 = a 2 X 2 + a1X 1

x3 = a3 X 3

Luego las ecuaciones del movimiento son:


x1 = a1 X 1

x 2 = a 2 X 2 + a1X 1
x = a X
3 3
3

0
x1 a1

x 2 = a1 a 2
x 0
0
3

0 X 1

0 X 2 (deformacin homognea)
a 3 X 3

Pudiendo sacar la informacin del determinante F = a1 a 2 a 3 > 0 .


Con la condicin de incompresibilidad dV = F dV0 F J = 1 , con lo cual obtenemos la
siguiente relacin:
a1 a 2 a 3 = 1

= [0 0 1] ) no se alarga conlleva a que el


Que un segmento paralelo al eje X 3 ( M
i
estiramiento segn esta direccin es unitario M = 1 :

Universidad de Castilla- La Mancha


Ciudad Real - Espaa

Draft

Por: Eduardo W. V. Chaves (2014)

MECNICA DEL MEDIO CONTINUO: PROBLEMAS RESUELTOS

206

E M
= 1 + 2E = 1
M = 1 + 2M
33

E 33 = 0

Las componentes del tensor de deformacin de Green-Lagrange ( E =


a1
1
E ij = 0
2
0

a1
a2
0

0 a1
0 a1
a 3 0

1 T
F F 1 ):
2

a12 + a12 2 1 a1 a 2
0 1 0 0
0

2
a1 a 2
a2 1
0 0 1 0 =
0
2
a 3 0 0 1
a 32 1
0
0

0
a2
0

Luego:
E 33 = a 32 1 = 0

a 3 = 1

rea en el plano X 1 X 3 no se ha modificado


0
x1 a1

x 2 = a1 a 2
x 0
0
3

0 X 1

0 X 2
a 3 X 3

con N i(1) = [1 0 0] y N i(3) = [0 0 1] obtenemos que:


n (i1)

0
a1

= a1 a 2
0
0

0 1 a1


0 0 = a1
a 3 0 0

n i(3)

0
a1

= a1 a 2
0
0

0 0 0

0 0 = 0
a 3 1 a 3

Luego el rea en la configuracin actual:


e 1
r (1) r (3)
n n = a1
0

e 2 e 3
a1 0 = a1e 1 a1 a 3 e 2 + 0e 3
0

a3

y su mdulo no se modifica N (1) N (3) = n (1) n (3) = 1 :


r
r
n (1) n (3) = 1 = (a1 ) 2 + (a1 a 3 ) 2 a12 a 32 2 + a12 a 32 = 1

Hemos obtenido anteriormente que a 32 = 1 , con lo cual obtenemos que:


a12 a 32 2 + a12 a 32 = 1

a12 2 + a12 = 1 a12 =

1
1
a1 =
2
(1 + )
(1 + 2 )

Con lo cual concluimos que:


a1 =

1
(1 + )
2

a 2 = (1 + 2 )

a3 = 1

Ejemplo 2.53
El slido de la Figura 2.11 sufre una deformacin uniforme (homognea).
Se pide:
a) Obtener la expresin general de la descripcin material del campo de desplazamientos
r r
U ( X , t ) en funcin del tensor gradiente material de los desplazamientos J .

Universidad de Castilla- La Mancha


Ciudad Real - Espaa

Draft

Por: Eduardo W. V. Chaves (2014)

2 CINEMTICA DEL CONTINUO

207

b) Obtener dicha expresin sabiendo que, adems, se cumple las siguientes condiciones de
contorno:
r
r
u 2 ( X , t ) = u3 ( X , t ) = 0

X 1 , X 2 , X 3

u1 ( X 1 = 0, X 2 , X 3 , t ) = 0
u1 ( X 1 = L, X 2 , X 3 , t ) =

c) Justificar los valores posibles (positivos y negativos) que puede tomar .


d) Calcular los tensores material y espacial de deformacin y el de deformacin
infinitesimales;
x3

x1

x2

Figura 2.11:

Solucin:

Una deformacin uniforme viene caracterizada por F ( X , t ) = F (t ) . Adems sabemos que:


r
r
uniforme
F ( X , t ) = 1 + J ( X , t ) Deformaci
n
F (t ) = 1 + J (t )

donde J es el gradiente material de los desplazamientos y para una deformacin uniforme


no es dependiente de la posicin, con lo cual podemos decir que:
r r
r
r r
u( X , t )
J (t ) =
r

J (t ) dX = du( X , t )
X
r
donde c (t ) es una constante de integracin. Luego:
r r
r r
u( X , t ) = J (t ) X + c (t )

r r
r r
u( X , t ) = J (t ) X + c (t )

En componentes:
u1 J 11 X 1 + J 12 X 2 + J 13 X 3 c1


u 2 = J 21 X 1 + J 22 X 2 + J 23 X 3 + c 2
u J X + J X + J X c
32 2
33 3
3
3 31 1

b) De las condiciones del apartado b) podemos decir que:


r

condicin 1) u 2 ( X , t ) = u 3 ( X , t ) = 0

X 1 , X 2 , X 3 :

u1 J 11 X 1 + J 12 X 2 + J 13 X 3 c1


J 21 = 0; J 22 = 0; J 23 = 0, c 2 = 0
u 2 = 0 = J 21 X 1 + J 22 X 2 + J 23 X 3 + c 2
u = 0 J X + J X + J X c J 31 = 0; J 32 = 0; J 33 = 0, c3 = 0
32 2
33 3
3
31 1
3

Universidad de Castilla- La Mancha


Ciudad Real - Espaa

Draft

Por: Eduardo W. V. Chaves (2014)

MECNICA DEL MEDIO CONTINUO: PROBLEMAS RESUELTOS

208

condicin 2) u1 ( X 1 = 0, X 2 , X 3 , t ) = 0 :
u1 = 0 J 11 X 1 + J 12 X 2 + J 13 X 3 c1



0
u2 =
+ 0 {J 12 = 0; J 13 = 0, c1 = 0
u
0
0
3

condicin 3) u1 ( X 1 = L, X 2 , X 3 , t ) =
u1 J 11 L 0



u 2 = 0 + 0 J 11 =
L
u = 0 0
3

Con lo cual podemos decir que las componentes del gradiente material de los
desplazamientos son:

L
J ij = 0

0 0
0 0

0 0

Y adems el campo de desplazamientos:

X1

L
r r
r
r r
u( X , t ) = J (t ) X + c (t ) componente

s u i ( X , t ) = 0

c) Para que el movimiento sea posible y tenga significado fsico hay que cumplir que
F >0:

1 + L
componentes
F (t ) = 1 + J (t )
Fij = 0

0 0

1 0 F = 1 + > 0 > L

L
0 1

d) Tensor material de deformacin (Tensor de deformacin de Green-Lagrange):

E=

1 T
F F 1
2

componente

1 2
+
2
L 2 L
0
E ij =

0 0

0 0
0 0

Tensor espacial de deformacin (Tensor de deformacin de Almansi):

e=

1
1 F FT
2

componente

1 2
1 0 0
+
2
L 2 L

eij =
2
0 0 0

1 + 0 0 0
L

Tensor de deformacin infinitesimal:

Universidad de Castilla- La Mancha


Ciudad Real - Espaa

Draft

Por: Eduardo W. V. Chaves (2014)

2 CINEMTICA DEL CONTINUO

L
ij = 0

209

0 0
0 0

0 0

Ejemplo 2.54
Sobre el tetraedro de la Figura 2.12, (ver Oliver (2000), se produce una deformacin
uniforme ( F = ctte ) con las siguientes consecuencias:
1. Los puntos O , A y B no se mueven;
2. El volumen del slido pasa a ser " p" veces el volumen inicial;
3. La longitud del segmento AC pasa a ser

p
2

veces la inicial;

4. El ngulo AOC pasa a ser de 45 .


Se pide:
a)
b)
c)

Justificar por qu no puede utilizar la teora de deformacin infinitesimal;


Obtener el tensor gradiente de deformacin, los posibles valores de " p" y el campo
de desplazamiento en su forma material y espacial;
Dibujar el slido deformado
x3

x2

A
x1

Figura 2.12.

Solucin:
a) el ngulo AOC pasa de 90 a 45 por lo que, evidentemente, no se trata de una
pequea deformacin, ya que en el caso de pequeas deformaciones << 1 , y en este
problema tenemos que <<

0,7854
4

b) Estamos en un caso de deformacin homognea. Luego, las ecuaciones del movimiento


viene dadas por:
r r
r
x = F (t ) X + c (t )

Universidad de Castilla- La Mancha


Ciudad Real - Espaa

Draft

Por: Eduardo W. V. Chaves (2014)

MECNICA DEL MEDIO CONTINUO: PROBLEMAS RESUELTOS

210

x1 F11

x 2 = F21
x F
3 31

F13 X 1 c1

F23 X 2 + c 2
F33 X 3 c3

F12
F22
F32

El punto O( X 1 = 0, X 2 = 0, X 3 = 0) no se mueve:
0 F11

0 = F21
0 F
31

F12
F22
F32

F13 0 c1

F23 0 + c 2
F33 0 c3

c1 0

c 2 = 0
c 0
3

El punto A( X 1 = a, X 2 = 0, X 3 = 0) no se mueve:
a F11

0 = F21
0 F
31

F13 a

F23 0
F33 0

F12
F22
F32

a aF11

0 = aF21
0 aF
31

F11 = 1

F21 = 0
F = 0
31

El punto B( X 1 = 0, X 2 = a, X 3 = 0) no se mueve:
0 1 F12

a = 0 F22
0 0 F
32

F13 0

F23 a
F33 0

0 aF12


a = aF22
0 aF
32

F12 = 0

F22 = 1
F = 0
32

Agrupando las informaciones anteriores:


1 0 F13
Fij = 0 1 F23
0 0 F33

F = F33

El volumen del slido pasa a ser " p" veces el volumen inicial. La relacin del diferencial de volumen
en la configuracin de referencia y actual viene dada por:
dV = F dV0

dV = F dV

V final = F Vinicial = F33Vinicial

donde hemos tenido en cuenta que la deformacin es homognea. Con lo cual, concluimos
que F33 = p
(La longitud del segmento AC pasa a ser

p
2

veces la inicial). Como estamos con deformacin

homognea, una recta en la configuracin de referencia seguir siendo una recta en la


configuracin deformada.
El punto C ( X 1 = 0, X 2 = 0, X 3 = a ) se desplaza como:
x1C 1 0 F13 0
C

x 2 = 0 1 F23 0
x C 0 0 p a

3

x1C aF13
C

x 2 = aF23
x C ap

La longitud del segmento AC en la configuracin de referencia es L AC = a 2 . El vector


que une los puntos A A( x1 = a, x 2 = 0, x3 = 0) , C ( x1 = aF13 , x 2 = aF23 , x3 = ap) en la
configuracin deformada viene dado por:
AC = (aF13 a )e 1 + (aF23 )e 2 + (ap)e 3

Su mdulo:
Universidad de Castilla- La Mancha
Ciudad Real - Espaa

Draft

Por: Eduardo W. V. Chaves (2014)

2 CINEMTICA DEL CONTINUO

211

AC = l AC = (a ( F13 1)) 2 + (aF23 ) 2 + (ap) 2 = a ( F13 1) 2 + ( F23 ) 2 + ( p) 2

Utilizando la informacin proporcionada por el problema: l AC =


l AC =
a ( F13 1) 2 + ( F23 ) 2 + ( p ) 2 =

p
2
p
2

p
2

L AC . Con lo cual:

L AC
a 2

( F13 1) 2 + ( F23 ) 2 + ( p ) 2 = p

Resultando:
( F13 1)2 + ( F23 )2 + p 2 = p 2

( F13 1) 2 + ( F23 ) 2 = 0

F13 = 1

F23 = 0

Resultando:
1 0
Fij = 0 1
0 0

1
0
p

El ngulo AOC pasa a ser de 45 .


dX i(1) = [1 0 0]

dX i( 2 )

= [0 0 1]

dxi(1) = Fij dX (j1)

dx1(1) 1 0 1 1 1
(1)

dx 2 = 0 1 0 0 = 0
dx (1) 0 0 p 0 0

3

dx1( 2) 1
( 2)

dxi( 2 ) = Fij dX (j 2 )

dx 2 = 0
dx ( 2) 0
3
r
r
dx (1) dx ( 2 )

cos( AOC ) = cos(45 ) = r (1) r ( 2 ) =


dx
dx

0 1 0 1

1 0 0 = 0
0 p 1 p
2
2

donde dx (1) = 1 , dx ( 2) = 1 + p 2 , dx (1) dx ( 2) = 1 . Luego:


1
1+ p

2
2

p = 1

Como el determinante del Jacobiano tiene que ser mayor que cero F = p > 0 , eso implica
que p = 1 :
1 0 1
Fij = 0 1 0
0 0 1

Las ecuaciones del movimiento quedan:

Universidad de Castilla- La Mancha


Ciudad Real - Espaa

Draft

Por: Eduardo W. V. Chaves (2014)

MECNICA DEL MEDIO CONTINUO: PROBLEMAS RESUELTOS

212

x1 1 0 1 X 1 X 1 + X 3


x 2 = 0 1 0 X 2 = X 2
x 0 0 1 X X

3
3
3

El campo de desplazamientos material queda:


r r
r r
u( X , t ) = x X

u1 X 1 + X 3 X 1 X 3


u 2 = X 2 X 2 = 0
u X
X 0
3
3
3

El campo de desplazamientos espacial:


u1 x3

u 2 = 0
u 0
3

c)
x3
C

x1C aF13 a
C

x 2 = aF23 = 0
x C ap a

3

B = B

x2

A = A
x1

Ejemplo 2.55
Considrense las siguientes ecuaciones del movimiento:
x1 = X 1

x 2 = X 2 X 3

x 3 = X 3 + X 2

Se pide:
a) El gradiente de deformacin, el tensor derecho de deformacin de Cauchy-Green, el
tensor izquierdo de deformacin de Cauchy-Green, el tensor de deformacin de GreenLagrange y el tensor de deformacin de Almansi. Verificar si se trata de un caso de
deformacin homognea.
b) El tensor derecho de estiramiento, el tensor de rotacin de la descomposicin polar y la
base principal del tensor izquierdo de deformacin de Cauchy-Green de la descomposicin
polar.
c) La longitud final de un elemento de longitud inicial 2 que se encuentra en la direccin
X 3 , y la distorsin angular de un ngulo que inicialmente es de 30 y est en el plano
X1 X 2 .
Universidad de Castilla- La Mancha
Ciudad Real - Espaa

Draft

Por: Eduardo W. V. Chaves (2014)

2 CINEMTICA DEL CONTINUO

213

d) Obtener el tensor de deformacin considerando el caso de pequeas deformaciones.


Solucin:
a) Gradiente de deformacin ( F )
x1

X 1
x
x
Fij = i = 2
X j
X
1
x3
X 1
r
r
En general tenemos que dx = F dX , y

x1
X 2
x 2
X 2
x3
X 2

x1

X 3 1 0
0
x 2
= 0 1

X 3

x3 0 1
X 3

si estamos en un caso de deformacin homognea


r r
r
(caso particular del movimiento) se cumple que x = F X + c , hecho que se puede
r r
comprobar a travs de las ecuaciones del movimiento en forma matricial donde c = 0 :
0 X1
x1 1 0
x = 0 1 X
2
2
x3 0 1 X 3

Tensor derecho de deformacin de Cauchy-Green ( C = F T F ):


0 1 0
0 1
0
0
1 0

2
0
C ij = Fki Fkj = 0 1 0 1 = 0 1 +
0 1 0 1 0
0
1 + 2

Tensor izquierdo de deformacin de Cauchy-Green ( b = F F T )


0 1 0
0 1
0
0
1 0

2
0
bij = Fik F jk = 0 1 0 1 = 0 1 +
0 1 0 1 0
0
1 + 2

1
2

El tensor de deformacin de Green-Lagrange ( E = (C 1) ) y el tensor de deformacin


1
2

de Almansi ( e = (1 b 1 ) ) vienen definidos a travs de sus componentes por:


1
0
0 1 0 0
0 0
1

1
1
2
E ij = (C ij ij ) = 0 1 +
0 0 1 0 = 0 2
2
2
2
0
0 0
0
1 + 2 0 0 1

0
1 0 0 1


1
1
1
1
eij = ( ij bij ) = 0 1 0 0

2
2
1+ 2

0
0
1

0
0

0
0
0

1
0 = 0
2 1 + 2

0
0
2
1 +

0
0
2

1+ 2

Se puede verificar el resultado a travs de la relacin E = F T e F :

Universidad de Castilla- La Mancha


Ciudad Real - Espaa

Draft

Por: Eduardo W. V. Chaves (2014)

MECNICA DEL MEDIO CONTINUO: PROBLEMAS RESUELTOS

214

0
0 0
1 0
2
1

E ij = 0 1 0
1+ 2
2
0 1
0
0

0 1 0
0
0 0

0 1 = 0 2
0

2
0 0
2 0 1

1+ 2

0
0
2

b) Segn el formato de las componentes cartesianas de C , podemos verificar que ele


espacio original ya es el espacio principal de C , es decir, las direcciones principales son
N i(1) = [1 0 0] , N i(1) = [0 1 0] , N i(1) = [0 0 1] . Por definicin, el tensor derecho de
estiramiento viene dado por U = C , y sus componentes:
1

U ij = 0
0

0 inversa
U ij1
1+ 2

1+
0

= 0

0
1
1+ 2
0

1+ 2

A travs de la descomposicin polar por la derecha ( F = R U R = F U 1 ), luego:

R ij = Fik U kj1

1
0
0

= 0 1 0
0 1

0
1
1+ 2
0

1
0
=
1+ 2

1+ 2

1+ 2

0
0

1
1

Observemos que segn el formato de las componentes cartesianas de b , tenemos como


direcciones principales [1 0 0] , [0 1 0] , [0 0 1] , pero esta no es la base principal del
tensor b de la descomposicin polar. Como hay dos autovalores iguales correspondientes a
las direcciones [0 1 0] , [0 0 1] , cualquier direccin en el plano x 2 x 3 ser una
direccin principal.
X 2 , x2

Cualquier direccin en el plano


x 2 x 3 es una direccin
principal de b

n (1) -direccin principal nica,


asociada al autovalor b1 = 1 .
X 1 , x1

X 3 , x3

Figura 2.13: Espacio principal de b .

Universidad de Castilla- La Mancha


Ciudad Real - Espaa

Draft

Por: Eduardo W. V. Chaves (2014)

2 CINEMTICA DEL CONTINUO

215

Recordar que la descomposicin polar es nica, es decir, slo habr una nica base
principal de b para la descomposicin polar. A travs de la relacin n ( a ) = R N ( a )
podemos encontrar la base de b de la descomposicin polar:
n (i 2 )

n (i 3)

= R N

(2)

( 3)
= R N

1+ 2

1
=
0
1+ 2 0

1+ 2

=
0
1+ 2 0

0 0

1
1 =
2
1 0 1 +

0
1

0 0

1
0 =
2
1 1 1 +

0
1

0
1


0

(a ) :
Verifiquemos adems que se cumple la relacin R = n ( a ) N
a =1

R ij = n i(1) N (j1) + n i( 2 ) N (j2 ) + n i(3) N (j3)


1
1
= 0[1 0 0] +
1+ 2
0
1 0 0
1
= 0 0 0 +
1+ 2
0 0 0

0
1
1 [0 1 0] +

1+ 2

0 0
0 1

0
1
0 +
1+ 2
0

0
[0 0 1]

1

0 0 0
1
0 0 =

1 + 2
0 0 1

1+ 2

0
0

c) A travs de la relacin de estiramiento segn una direccin M , M =

0
1

r
dx
ds
, y
r =
dS
dX

teniendo en cuenta que el estiramiento no depende de la integral de lnea (deformacin


homognea), se cumple que:

L final = ds = M dS = M dS = M Linicial

El estiramiento segn la direccin X 3 viene definido por:


X = C 33 = 1 + 2 E 33 = 1 + 2
3

Luego:
2

L final = M dX 2 = 1 + 2 ( Linicial ) = 2 1 + 2
0

Como se trata de un caso de deformacin homognea, una recta en la configuracin de


referencia sigue sendo una recta en la configuracin actual, ver Figura 2.14.

Universidad de Castilla- La Mancha


Ciudad Real - Espaa

Draft

Por: Eduardo W. V. Chaves (2014)

MECNICA DEL MEDIO CONTINUO: PROBLEMAS RESUELTOS

216

X 3 , x3
A

A
Linicial = 2

L final

X 2 , x2

x1A 1 0 0 X 1A
A
A
x 2 = 0 1 X 2
x3A 0 1 X 3A


1 0 0 0 0
= 0 1 0 = 2
0 1 2 2

X 1 , x1

Figura 2.14.
Segn la Figura 2.14, podemos comprobar que:
( Linicial ) 2 = 2 2 + (2 ) 2 = 4(1 + 2 )

Linicial = 2 1 + 2

Para obtener el ngulo en la configuracin actual que forman dos versores, podemos
utilizar la expresin:
cos =

E N
cos + 2 M
M N

(2.138)

donde es el ngulo que forman los versores M y N en la configuracin de referencia, y


es el ngulo que estos versores forman en la configuracin actual.
Teniendo en cuenta que el tensor de deformacin de Green-Lagrange es independiente de
r
X , adoptaremos dos versores en el plano X 1 X 2 que forman un ngulo de = 30 :
N i = [1 0 0] , M i = [cos 30 sin 30 0] . Con estos datos tenemos que:
0 cos 30
0 0
1

2
E N = [1 0 0] 0
M
0 sin 30 = 0

2
0 0 2 0

Los estiramientos:
2M

0
0 1
1

= M C M = [1 0 0] 0 1 +
0 0 = 1
0
0
1 + 2 0

2N

0
1

= N C N = [cos 30 sin 30 0] 0 1 + 2
0
0

M = 1

cos 30
0 sin 30
1 + 2 0
0

= cos 2 30 + (1 + 2 ) sin 2 30 = 1 + 2 sin 2 30

Luego, N = 1 + 2 sin 2 30 . Resultando que:

Universidad de Castilla- La Mancha


Ciudad Real - Espaa

Draft

Por: Eduardo W. V. Chaves (2014)

2 CINEMTICA DEL CONTINUO

217

E N
cos + 2 M
cos 30
=
M N
1 + 2 sin 2 30

cos =

Como se trata de un caso de deformacin homognea, podemos adoptar dos rectas en la


configuracin de referencia y obtener estas dos rectas en la configuracin actual y obtener
el ngulo que forman. Por ejemplo, adoptando las rectas OB = [cos 30 0 0] , y
OC = [cos 30 sin 30 0] . Segn las ecuaciones del movimiento, el punto O no se mueve.
A continuacin obtenemos las nuevas posiciones de los puntos B y C , ver Figura 2.15:

x1B 1 0
0 X 1B 1 0
0 cos 30 cos 30
B
B

x 2 = 0 1 X 2 = 0 1 0 = 0
x B 0 1 X B 0 1 0 0

3
3
x1C 1 0
0 X 1C 1 0
0 cos 30 cos 30
C
C

x 2 = 0 1 X 2 = 0 1 sin 30 = sin 30
x C 0 1 X C 0 1 0 sin 30

3
3
X 3 , x3

sin 30

sin 30

X 2 , x2

cos 30

30

B = B
X 1 , x1

Figura 2.15.

A continuacin obtenemos el ngulo que forman los nuevos vectores O B y O C :


OB OC = OB OC cos
cos2 30 = cos 2 30 cos2 30 + sin 2 30 + 2 sin 2 30 cos

cos =

cos 30
1 + 2 sin 2 30

0 0 0
d) ij = 0 0 0
0 0 0

Ejemplo 2.56
Un movimiento de cuerpo rgido est caracterizado por presentar la siguiente ecuacin de
movimiento:
r
r r
x = c(t ) + Q(t ) X

Universidad de Castilla- La Mancha


Ciudad Real - Espaa

Draft

(2.139)
Por: Eduardo W. V. Chaves (2014)

MECNICA DEL MEDIO CONTINUO: PROBLEMAS RESUELTOS

218

Encontrar la velocidad y aceleracin en funcin de que es el vector asociado con el


& QT .
tensor antisimtrico = Q
Solucin:
r D r r& r& & r
v=
x = x = c + Q X
Dt

& QT Q
& = Q , la relacin anterior puede an ser escrita
Considerando que = Q
como:
r
r r
v = c& + Q X

r r
r r
v = c& + ( x c )

Utilizando la propiedad que para un tensor antisimtrico el producto escalar de ste con
r r r
r
r
un vector a podr ser representado por a = a , donde es el vector asociado al
tensor antisimtrico . Luego:
r r
r r
r r
r r
v = c& + ( x c ) = c& + ( x c )

(2.140)

La aceleracin vendr dada por:


r r &r& &r& && r
a = v& = x
=c + Q X

&& = & Q + Q
& la expresin anterior podr ser representada por:
Considerando que Q
r
r
r
r
r
r &r&
& ) X = &cr& + & Q X + Q
& X = &cr& + & Q X + Q X
a=c
+ (& Q + Q
r
r r
r r
= &c& + & ( x c ) + ( x c )

Una vez ms utilizando la propiedad a = a , podemos decir que:


r &r& r&
r r
r
r
r r
a =c
+ ( x c) + [ ( x c)]

(2.141)

Para un movimiento de slido rgido con c = 0 , la velocidad viene dada por v = x , y el


tensor tasa de deformacin D viene dado por:
D ij =
=

1 vi v j
+
2 x j xi

1
ipq p qj
2

1 ( ipq p x q ) ( jpq p x q ) 1
x
x
=
= ipq p q + jpq p q
+
2
2
x j
xi
x j
xi

1
1
+ jpq p qi = ipj p + jpi p = ipj p ipj p = 0 ij
2
2

Una vez ms hemos demostrado que D = 0 para un movimiento de slido rgido.


Ejemplo 2.57
r

Dado un sistema de coordenadas x fijo en el espacio y un sistema mvil x * caracterizado


r
nicamente por rotacin, ver Figura 2.16. Demostrar que la tasa de un vector b puede ser
representado por:
r
r
r
r
Db
Db
r r Db
T

b
=
+

b
Dt

Dt

fijo Dt mvil

mvil

Universidad de Castilla- La Mancha


Ciudad Real - Espaa

Draft

(2.142)

Por: Eduardo W. V. Chaves (2014)

2 CINEMTICA DEL CONTINUO

219

r
r
r
Db
r Db

donde
representa la tasa de b con respecto al sistema fijo x ,

Dt mvil
Dt fijo
r
representa la tasa de b con respecto al sistema que est rotando con una velocidad angular

r
b

x3

x2*

x3*

x1*
x2

x1

Figura 2.16
Solucin:
A travs de ley de transformacin de las componentes de los tensores, las siguientes
relaciones son vlidas:

r
r
Componente s b * = A b

r
r
b = A T b*

donde A es la matriz de transformacin del sistema x al sistema x * .


r

La tasa de b = A T b * resulta:

r
r
r&
D r r& D
b b =
A T b * = A& T b * + A T b *
Dt
Dt

(2.143)

Haciendo una analoga con la tasa de un tensor ortogonal, ver captulo 2 en Chaves (2007),
podemos decir que = A& A T A& T = A T T , donde T es un tensor antisimtrico
r
r
y representa el tensor tasa de rotacin del sistema x * con respecto al sistema x . Pudiendo
as expresar (2.143) como:
r
r&
r
r&
r
r&
D r r&
b b = A& T b * + A T b = A T T b * + A T b * = A T T b * + b *
(2.144)

Dt
r
r r
r
Recurriendo a la propiedad del tensor antisimtrico tal que T b * = b * , donde es el

r r
vector axil asociado al tensor antisimtrico T , es decir, = (t ) es la velocidad angular
r
del sistema mvil x * . Resultando que (2.144) an puede ser escrito como:
r&
r
r&
r
r&
r
b = A T T b * + b * = A T * b * + b * (componentes)

r&

r&

(2.145)
r

Notar que el trmino A b representa las componentes de b en el sistema x * , y notar


r&

r&

tambin que A b b * , luego:

r * r
r
A b& = b& * + r * b * (componentes)

Universidad de Castilla- La Mancha


Ciudad Real - Espaa

Draft

(2.146)

Por: Eduardo W. V. Chaves (2014)

MECNICA DEL MEDIO CONTINUO: PROBLEMAS RESUELTOS

220

Tambin podemos expresar la ecuacin anterior en notacin tensorial:


r
r
Db
Db
r r

+ b (notacin tensorial)
Dt = Dt

fijo
mvil

(2.147)

Ejemplo 2.58
r

a) Un slido gira con una velocidad angular constante = 3 e 3 . Se pide:


a.1) Obtener las componentes de la velocidad en la descripcin espacial y material;
a.2) Obtener la aceleracin en la descripcin espacial (Euleriana);
a.3) Si 3 = 3rad / s encontrar la posicin, velocidad y aceleracin en el tiempo t = 2,5s de
una partcula que en la configuracin de referencia ocupaba la posicin (1,1,0) .
b) Teniendo en cuenta el resultado del Ejemplo 1.126 donde hemos obtenido el vector
r

GM
x

fuerzas msicas b = r x donde g = b es la aceleracin de la gravedad debido al


campo gravitacional. Si consideramos la Tierra como una esfera que gira alrededor de su eje
r
con velocidad angular = 3 e 3 , obtener la aceleracin de la gravedad ( g ) a nivel del mar
en funcin de la latitud .
Solucin:
X 3 , x3
r
= 3 e 3

r r
r = r e r = re r

3
r
r

e 3
e r

r
x

X 2 , x2

X 1 , x1

Figura 2.17.
r r

a.1) A travs del ejercicio anterior podemos decir que v ( x , t ) = x , o en notacin


indicial:
vi = ijk j x k = i1k
{1 x k + i 2 k
{2 x k + i 3k 3 x k = i 3k 3 x k
=0

=0

= i 313 x1 + i 32 3 x 2 + i 33 3 x3 = i 31 3 x1 + i 32 3 x 2
{
=0

Luego:
v1 = 1323 x2 = 3 x2

Universidad de Castilla- La Mancha


Ciudad Real - Espaa

v2 = 2313 x1 = 3 x1

v3 = 0

Draft

Por: Eduardo W. V. Chaves (2014)

(2.148)

2 CINEMTICA DEL CONTINUO

221

Solamente por verificacin, recordar que el vector vorticidad ( ) es igual al rotacional del
r
r
r r
campo de velocidad, xr v = rot v = y es igual a 2 veces el vector velocidad angular
r
r
r
(vector axil) = 2 = 2 w :
e 1
r
r

xr v =
x1
v1

e 2

x 2
v2

e 3
v
v

= 3 2
x3 x 2 x3
v3
r
= ( 3 ( 3 ) )e 3 = 2 3 e 3 =

v
e 1 3 1
x1 x3

1 r
2

v
v
e 2 + 2 1
x1 x 2

r 1 r
1
2
2
r r
r r r
Observar que el campo v ( x , t ) es estacionario, es decir, v = v ( x ) .
r

e 3

1
2

La velocidad angular viene definida por = xr v = rot v = = ( 2 3 e 3 ) = 3 e 3 .

Para un movimiento de slido rgido las ecuaciones del movimiento vienen gobernadas por
r
r
x = Q(t ) X , donde las componentes del tensor ortogonal viene dada por las componentes
r
r
de la matriz de transformacin del sistema x al sistema x , luego:
x1 cos (t ) sin (t ) 0 X 1 cos (t ) X 1 sin (t ) X 2


x 2 = sin (t ) cos (t ) 0 X 2 = sin (t ) X 1 + cos (t ) X 2
x 0

0
1 X 3
X3
3

Teniendo en cuenta que =

d(t )
y al integrar obtenemos que:
dt

d(t ) = dt

(t ) = t

Pudiendo as reescribir las ecuaciones del movimiento como:


x1 cos (t ) sin (t ) 0 X 1 X 1 cos(t ) X 2 sin(t )


x 2 = sin (t ) cos (t ) 0 X 2 = X 1 sin(t ) + X 2 cos(t )
x 0

0
1 X 3
X3
3

(2.149)

Para obtener la expresin de la velocidad en la descripcin material (Lagrangiana),


reemplazamos las ecuaciones del movimiento (2.149) en las expresiones (2.148):
r
v1 ( X , t ) = 3 ( X 1 sin(t ) + X 2 cos(t ))
r

v 2 ( X , t ) = 3 ( X 1 cos(t ) X 2 sin(t ))
r

v3 ( X , t ) = 0

(2.150)

a.2) La aceleracin Euleriana obtenemos a travs de la definicin de derivada material de la


r r
velocidad v ( x , t ) :
r r
r r
r r
r r
r r r
v ( x , t ) v ( x , t ) x ( X , t )
a ( x, t ) =
+
= xr v v ( x , t )
r
t4
x
t
1
42
3
r
0

donde las componentes del gradiente espacial de la velocidad vienen dadas por:

Universidad de Castilla- La Mancha


Ciudad Real - Espaa

Draft

Por: Eduardo W. V. Chaves (2014)

MECNICA DEL MEDIO CONTINUO: PROBLEMAS RESUELTOS

222

r r
r
v ( x , t )

r = ( xr v )ij
x ij

v1

x1
v
= 2
x
1

v3
x1

v1
x 2
v 2
x 2
v 3
x 2

v1

x 3 0
v 2
= 3
x 3

v 3 0
x 3

0
0 (antisimtrico)
0

0
0

Comprobando que realmente se trata de un movimiento de slido rgido. Luego, las


componentes de la aceleracin Euleriana vienen dadas por:
0 3 x2 32 x1


0 3 x1 = 32 x2
0 0 0

0 3
r
r r r
r
ai ( x , t ) = [ x v v ( x , t ) ]i = 3
0
0
0
r r
Podemos expresar la aceleracin a ( x , t ) = 32 x1 e 1

32 x 2 e 2 en el sistema cilndrico, ver

Figura 2.17. Recordar del captulo 1 de los apuntes que se cumplen que:
x1 = r cos ,

x1 = r cos ,

e 1 = e r cos e sin ,

e 2 = e r sin + e cos . Luego, la

aceleracin en el sistema cilndrico queda:


r
a = 32 x1e 1 32 x2e 2 = 32 ( r cos )(e r cos e sin ) 32 ( r sin )(e r sin + e cos )
r v
= 32 r (cos 2 + sin 2 )e r = 32 re r = 32 r = actpe
v
Esta ltima conocida como la aceleracin centrpeta ( actpe ).

a.3) La partcula que en la configuracin de referencia ocupaba la posicin (1,1,0) describe


una trayectoria circular de radio r = 2 en el plano x1 x 2 , ver Figura 2.18.
Partcula P en t = 2,5s

X 2 , x2

r r
v P ( x , t = 2,5)

1
r
x

r r
v ( X , t = 0)

Partcula P

r
X

X 1 , x1

Trayectoria de la partcula P
Figura 2.18.
r

En la configuracin de referencia ( t = 0 ) se cumple que X = x . Para la partcula P


tenemos que:
r
v1P ( x , t = 0) = 3 x 2 = 3 X 2 = (3)(1) = 3
32 X 1 9
P r
r

; a iP ( x , t = 0) = 32 X 2 = 9
v 2 ( x , t = 0) = 3 x1 = 3 X 1 = (3)(1) = 3
P
0 0
v3 = 0

Universidad de Castilla- La Mancha


Ciudad Real - Espaa

Draft

Por: Eduardo W. V. Chaves (2014)

2 CINEMTICA DEL CONTINUO

223

Para el tiempo t = 2,5s la posicin, velocidad, y aceleracin de la partcula P vienen dadas


por:
x1P X 1 cos(t ) X 2 sin(t ) cos(3 2,5) sin(3 2,5) 0,59136
P


x 2 = X 1 sin(t ) + X 2 cos(t ) = sin(3 2,5) + cos(3 2,5) = 1,28464
x P

X3
0
0

r
v1P ( x , t = 2,5) = 3 x 2 = (3)(1,28464) = 3,85391
P r
v 2 ( x , t = 2,5) = 3 x1 = (3)(0,59136) = 1,77409
P
v 3 = 0
32 x1 5,322
r

a iP ( x , t = 2,5) = 32 x 2 = 11,562
0 0

b) Para una partcula situada en la superficie de la Tierra, debido a la rotacin se sentir


como si estuviera siendo proyecta hacia fuera segn direccin de r , ver figura abajo. Hay
que tener en cuenta que la fuerza real es la Centrpeta debido a la aceleracin centrpeta.
Por conveniencia, adoptamos una fuerza ficticia, fuerza centrfuga, que sera la aparente
causa de esta proyeccin hacia afuera. Asociada a esta fuerza tenemos la aceleracin
v
v
centrfuga ( a ctfu ) que es igual pero de sentido contrario a la aceleracin centrpeta ( a ctpe ).
x3 , z
x3 , z

3
r

x3

r
b

v
a ctfu

x2 , y

x1 , x

La aceleracin de la gravedad para una latitud definida por viene dada por:
r
v
g = a ctfu + b

Recordar
v
a ctfu

r
+b =

que
v
a ctfu

dados

dos

r
r
b cos + b

v
+ 2 a ctfu

particular tenemos que, b = g ,

vectores
2

se

cumple

que

, ver Ejemplo 1.02. Para este caso en

r
v
v
a ctpe = a ctfu = 32 r = 32 r . Verificar tambin que

r = R cos y que cos = cos( ) = cos . Con eso, obtenemos que:


Universidad de Castilla- La Mancha
Ciudad Real - Espaa

Draft

Por: Eduardo W. V. Chaves (2014)

MECNICA DEL MEDIO CONTINUO: PROBLEMAS RESUELTOS

224

r
v
g = actfu + b =

v
actpe

r
r
v
2 actpe b cos + b

= (32 r ) 2 2(32 r ) g cos + g 2

= (32 R cos ) 2 2(32 R cos ) g cos + g 2

Resultando
g = g 2 2 g 32 R cos 2 + 34 R 2 cos 2

Observar que en los polos ( = 90 ) se cumple que g Pol = g y en la lnea del ecuador se
cumple que g Ecu = g 2 2 g 32 R + 34 R 2 = ( g 32 R ) 2 = g 32 R .
Ejemplo 2.59
Considrese una barra sometida a sucesivos desplazamientos como se indica la Figura abajo

B0

L0

L0

L(1)

L( 2 )

L(f1)

L L( 2 )

L(1)
L
( 2)

Demostrar que la deformacin Ingenieril (deformacin de Cauchy) no es aditiva para


incremento sucesivos de deformacin, es decir, (1) + ( 2) .
Solucin: La deformacin de Cauchy fue definida como:
C =

L L L 0
=
= 1
L0
L0

Luego, la deformacin total sufrida por el cuerpo, es decir, de la configuracin B0 hasta la


configuracin B es
L( 2 ) L0 L( 2 )
=
1
L0
L0

C =

En la configuracin B la deformacin Ingenieril queda:


C(1) =

L(1) L0 L(1)
=
1
L0
L0

En la configuracin B teniendo en cuenta solo el incremento de desplazamiento u ( 2) ,


tenemos que:
C( 2) =

L( 2) L(1) L( 2)
= (1) 1
L(1)
L

Luego

Universidad de Castilla- La Mancha


Ciudad Real - Espaa

Draft

Por: Eduardo W. V. Chaves (2014)

2 CINEMTICA DEL CONTINUO

225

L(1)
L( 2)

L( 2 )
C(1) + C( 2 ) =
1 + (1) 1
1 = C
L
L0
L0

Un requerimiento esencial de toda deformacin es que pueda caracterizar los


desplazamientos reales, en el caso la longitud final:
L0

L(1)

C(1) dx =
1dx = L(1) L0 = L(1)
L0

0
0
(1)
( 2)
L + L = L
L1
L1
(2)
L

(C2 ) dx = (1) 1 dx = L( 2 ) L(1) = L( 2 )


L

0
0

L0

L0

L0

C dx

L
=
1 dx = L L0 = L
L

0 0

Deformacin de Green-Lagrange
Observemos que la deformacin de Green-Lagrange en la configuracin B viene dado
por:
L2 L20

G =

Podramos
E=E

(1)

+F

haber
(1)T

obtenido

2 L20

esta

misma

1 2
1
2

expresin

utilizando

E F , donde para el caso uniaxial tenemos que


( 2)

(1)

E ( 2) G( 2) , F (1) (1) =

la

E G , E

relacin
(1)

G(1) ,

T
L(1)
. Luego, la ecauccn E = E (1) + F (1) E ( 2) F (1) , en una
L0

dimensin queda:
G =

G(1)

+ (1)G( 2)(1)

2
( 2)2
2
2
2
L(1) 1 L( 2) 2 L(1) L L0
1 L(1)
= L L0
1 +
(1) 1
=
=
2 L0
2 L20
2 L20
L0 2 L
L0

2.3 Descomposicin Polar del Gradiente de Deformacin


Ejemplo 2.60
Considrense las componentes cartesianas del gradiente de deformacin dadas por:
5 3 3
Fij = 2 6 3
2 2 4
Obtener los tensores derecho ( U ) e izquierdo ( V ) de estiramiento y el tensor de rotacin
( R ) de la descomposicin polar.

Solucin:
Antes de obtener los tensores U , V , R , vamos analizar el tensor F .
Para que tenga sentido fsico el determinante de F tiene que ser mayor que cero,
det ( F ) = 60 > 0 . Los autovalores y autovectores F se pueden obtener como:
F11 = 10 asociado al autovector m i(1) = [0,6396021491; 0,6396021491; 0,4264014327]
F22 = 3 asociado al autovector m i( 2 ) = [ 0,5570860145; 0,7427813527; 0,3713906764]
F33 = 2 asociado al autovector m i(3) = [ 0,4082482905; 0,4082482905; 0,8164965809]
Universidad de Castilla- La Mancha
Ciudad Real - Espaa

Draft

Por: Eduardo W. V. Chaves (2014)

MECNICA DEL MEDIO CONTINUO: PROBLEMAS RESUELTOS

226

Se puede verificar fcilmente que la base constituida por estos autovectores no forma una
ortonormal, es decir, m i(1) m i( 2) 0 , m i(1) m i(3) 0 , m i( 2) m i(3) 0 . Verificamos tambin que si
B es la matriz que contiene los autovectores de F :
m i(1) 0,6396021491;
0,6396021491;
0,4264014327
( 2)
B = m i = 0,5570860145; 0,7427813527; 0,3713906764
m (3) 0,4082482905; 0,4082482905; 0,8164965809

podemos hallar que det (B ) = 0,905 1 , y que B 1 B T . Pero se cumple que:


10 0 0
5 2 2

B 0 3 0 B = 3 6 2 = FijT
0 0 2
3 3 4

5 2 2
10 0 0

1
y
B 3 6 2B = 0 3 0
3 3 4
0 0 2
Tensor derecho de deformacin de Cauchy-Green, C = F T F (tensor definido positivo):
33 31 29
C ij = Fki Fkj = 31 49 35
29 35 34
1

Los autovalores y autovectores del tensor C son:

(1) = [0,6861511933; 0,7023576528 0,1894472683]


N
i
autovector

= 3,770098
C 22

N i( 2 ) = [0,5105143234; 0,2793856273; 0,8132215099]
(3) = [ 0,518239; 0,65470405; 0,550264423]
= 102,955163
C 33
autovector

N
i
= 9,274739
C11

autovector

Dichos autovectores constituye una base ortonormal luego, se cumple que AC1 = ACT , y
det (AC ) = 1 , donde:
N i(1) 0,6861511933 0,7023576528 0,1894472683

AC = N i( 2) = 0,5105143234 0,2793856273 0,8132215099


N
(3) 0,518239
0,65470405
0,550264423
i

cumpliendo que:

C11

A 0
0

0
0
0
33 31 29
33 31 29
C11

0 AC = 31 49 35 = C ij ; AC 31 49 35 AC = 0 C 22
0
29 35 34
29 35 34
0


0 C 33
0 C 33
En el espacio principal de C obtenemos las componentes del tensor derecho de
estiramiento U como:

0 C11
0
0 3,0454455
0
0

1 0

U = U ij = 0 2 0 = 0
C 22
0 =
0
1,9416741
0

0 0 3 0

0
C 33
0
0
10,1466824

T
C

C 22

y su inversa:

1

0
0
0

3,0454455

1
1

U 1 = Uij1
0 =
0
0

1,9416741
2

1
1
0
0

0
10,1466824
3
Pudiendo as obtener las componentes del tensor U en el espacio original a travs de la ley
1

1
= 0

de transformacin:

Universidad de Castilla- La Mancha


Ciudad Real - Espaa

Draft

Por: Eduardo W. V. Chaves (2014)

2 CINEMTICA DEL CONTINUO

227

4,66496626 2,25196988 2,48328843


A U AC = 2,25196988 6,00314487 2,80907159 = U ij
2,48328843 2,80907159 4,46569091
T
C

y
0,31528844 0,05134777 0,14302659
A U AC = 2,25196988 0,24442627 0,12519889 = U ij1
0,14302659 0,12519889 0,38221833
1

T
C

Luego, el tensor de rotacin de la descomposicin polar viene dado por la expresin


R = F U 1 , que resulta en un tensor ortogonal propio det (R ) = 1 .
0,10094326 0,05592536
0,9933191

= 0,10658955 0,98826538 0,10940847


R ij =
0,04422505 0,11463858 0,9924224
Tensor izquierdo de deformacin de Cauchy-Green, b = F F T (tensor definido positivo):
43 37 28
bij = Fik F jk = 37 49 28
28 28 24

Fik U kj1

Los autovalores y autovectores del tensor b son:

n i(1) = [0,6212637156 0,7465251613 0,238183919]


= 3,770098
b22
autovector

n i( 2 ) = [0,4898263742 0,1327190337 0,8616587383]
= 102,955163
b33
autovector

n (i 3) = [ 0,611638389 0,6519860747 0,448121233]
Observemos que son los mismos autovalores del tensor C , pero con distintos
autovectores. Los autovectores del tensor b tambin constituye una base ortonormal
luego, se cumple que Ab1 = AbT , y det (Ab ) = 1 , donde:
= 9,274739
b11

autovector

n (i1) 0,6212637156 0,7465251613


0,238183919
( 2)
Ab = n i = 0,4898263742 0,1327190337 0,8616587383
n (3) 0,611638389 0,6519860747 0,448121233

cumpliendo que:

b11

A 0
0

0
0
0
b11
43 37 28
43 37 28

0 Ab = 37 49 28 = bij ; Ab 37 49 28 Ab = 0 b22
0
0
28 28 24
28 28 24


0 b33
0 b33
Ya que los tensores C y b tienen los mismos autovalores se cumple que Uij = Vij , es decir,
T
b

b22

que tienen las mismas componentes en sus respectivos espacios principales. Y como
consecuencia Uij1 = Vij 1 . Luego se cumple que:
Pudiendo as obtener las componentes del tensor U en el espacio original a travs de la ley
de transformacin:
5,3720129 2,76007379 2,41222612
A V Ab = A U Ab = 2,76007379 6,04463857 2,20098553 = Vij
2,41222612 2,20098553 3,6519622
T
b

T
b

y
0,28717424 0,07950684 0,14176921
A V Ab = A U Ab = 0,07950684 0,23396031 0,08848799 = Vij1
0,14176921 0,08848799 0,42079849
T
b

T
b

Universidad de Castilla- La Mancha


Ciudad Real - Espaa

Draft

Por: Eduardo W. V. Chaves (2014)

MECNICA DEL MEDIO CONTINUO: PROBLEMAS RESUELTOS

228

El tensor de rotacin de la descomposicin polar ya obtenido anteriormente tiene ser el


mismo si utilizamos la expresin R = V 1 F .
Tambin podramos haber obtenido los tensores U , V , R a travs de su representacin
espectral. Es decir, si conocemos los estiramientos principales i y los autovectores de C ,
(i ) , y los autovectores de b , n (i ) , es de fcil demostracin que:
N

U=
V=
R=

F=

(a ) N
(a) = N
(1) N
(1) + N
(2) N
(2) + N
( 3) N
( 3)
N
1
2
3

n ( a ) n ( a ) = 1 n (1) n (1) + 2 n ( 2 ) n ( 2 ) + 3 n (3) n (3)

a =1
3

a =1
3

(a)

a =1
3

( a ) = n (1) N
(1) + n ( 2) N
( 2 ) + n (3) N
( 3)
N

( a ) = n (1) N
(1) + n ( 2 ) N
( 2 ) + n (3) N
( 3)
n ( a ) N
1
2
3

a =1

(a) N
(a) =
a R N

a =1

n ( a ) n ( a ) R

a =1

(a ) N
( a ) = n ( a ) n ( a ) R
= R a N
a

a =1
a =1

= R U = V R
3

Como podemos verificar la representacin espectral de los tensores R y F no viene


presentada en el sentido estricto de la representacin espectral, es decir, autovalor y
autovector del tensor.
Ejemplo 2.61
El tensor gradiente de deformacin en un punto del cuerpo viene dado a travs de la
siguiente combinacin lineal de didicas:
F = 0,2e 1 e 1 0,1e 1 e 2 + 0,3e 2 e 1 + 0,4e 2 e 2 + 0,1e 3 e 3

donde e i

(i = 1,2,3) representa la base cartesiana. Se pide:

a) Determinar los tensores de deformacin b y C ;


b) Determinar los autovalores y autovectores de b y C ;
c) Escribir F en su representacin espectral en funcin de los autovalores de C ( C a ) y
3

( a ) , siendo los estiramientos principales, n


verificar si se cumple que F = a n ( a ) N
a
a =1

los autovectores de C ;
los autovectores de b , y N

d) Obtener la representacin espectral y las componentes: del tensor de rotacin ( R ) de la


descomposicin polar; y de los tensores de estiramientos U y V ;
Solucin
Las componentes del gradiente de deformacin en forma de matriz vienen dadas por:
F = Fij e i e j = 0,2e 1 e 1 0,1e 1 e 2 + 0,3e 2 e 1 + 0,4e 2 e 2 + 0,1e 3 e 3

0,2 0,1 0
Fij = 0,3 0,4 0
0
0
0,1
Universidad de Castilla- La Mancha
Ciudad Real - Espaa

Draft

Por: Eduardo W. V. Chaves (2014)

2 CINEMTICA DEL CONTINUO

229

a) Las componentes del tensor izquierdo de deformacin de Cauchy-Green, b = F F T ,


vienen dadas por:
T

0,2 0,1 0 0,2 0,1 0


0,05 0,02 0

bij = Fik F jk = 0,3 0,4 0 0,3 0,4 0 = 0,02 0,25 0


0
0
0
0,1 0
0
0,1
0
0,01

(2.151)

Las componentes del tensor derecho de deformacin de Cauchy-Green, C = F T F ,


vienen dadas por:
0,2 0,1 0
C ij = Fki Fkj = 0,3 0,4 0
0
0
0,1

0
0,2 0,1 0 0,13 0,1
0,3 0,4 0 = 0,1 0,17
0

0
0
0,1 0
0 0,01

(2.152)

b) Determinar los autovalores y autovectores de b y C ;


=C N
(a)
C N
(a)

C C1 = 0

donde el ndice (a ) no indica suma. Observemos que ya conocemos un autovalor de C ,


C (3) = 0,01 , ver componentes de C (2.152). Luego, el determinante caracterstico queda:
0,13 C

0,1

0,1

0,17 C

=0

(0,13 C )(0,17 C ) 0,01 = 0

La solucin de la ecuacin cuadrtica es:


C (1) = 0,25198

C ( 2) = 0,04802

Luego:
0,633399
(1)

C (1) = 0,25198 N i = 0,77334

0
0
( 3)

C (3) = 0,01 N i = 0
1

C ( 2 ) = 0,04802

0,77334
N ( 2 ) = 0,63399
i

Anlogamente para obtener los autovalores y autovectores del tensor b :


b n = b( a ) n ( a )

donde el ndice (a ) no indica suma.. Luego


b(1) = 0,25198

b(3) = 0,01

n i(3)

0,098538
= 0,995133

0
0
= 0
1

n i(1)

b( 2) = 0,04802

n i( 2)

0,995133
= 0,098538

Observemos que era de esperar que los tensores C y b presentan los mismos autovalores:

Universidad de Castilla- La Mancha


Ciudad Real - Espaa

Draft

Por: Eduardo W. V. Chaves (2014)

MECNICA DEL MEDIO CONTINUO: PROBLEMAS RESUELTOS

230

0
0
0,252

0,048 0
C ij = 0
0
0
0,01

0
0
0,252

0,048 0
bij = 0
0
0
0,01

Adems la representacin espectral de los tensores C y b viene dadas respectivamente


por:
C=

(a ) N
(a)
2a N

a =1

b=

n
2
a

(a)

n ( a )

a =1

donde a son los estiramientos principales. Considerando que 2a = C a son los autovalores
de C y de b , los estiramientos principales son:
(1) = 0,25198 0,501976

( 2) = 0,04802 0,219134

(3) = 0,01 = 0,1

( a ) . Calculemos las componentes de


c) Para verificar si se cumple F = a n ( a ) N
a =1

( a ) , con los resultados obtenidos anteriormente. Resultando:


a n ( a ) N

ij
a =1
3

3
( a ) = n (1) N (1) + n ( 2 ) N ( 2 ) + n (3) N (3)
a n ( a ) N
1 i
j
2 i
j
3 i
j

a =1
ij

0,76958 0,6309 0
0,06247 0,0762 0

= 0,50197 0,0762 0,06247 0 + 0,219134 0,6309 0,76958 0 +


0
0
0
0
0
0
0 0 0
0,2 0,1 0

+ 0,10 0 0
= 0,3 0,4 0
0 0 1
0
0
0,1

Luego, resulta ser cierto.


d)
R=

(a)

(a )

componentes

a =1

0,832 0,554 0
(R )ij = 0,554 0,832 0
0
0
1

Que puede ser verificado con:


0,76958 0,6309 0 0,06247 0,0762 0 0
R ij = 0,0762 0,06247 0 + 0,6309 0,76958 0 + 0

0
0
0 0
0
0 0
0,333 0,139
3
(a)
(a)

aN N
U=
componentes (U)ij 0,139 0,388
a =1
0
0

V=

a n ( a ) n ( a )

componentes

a =1

Universidad de Castilla- La Mancha


Ciudad Real - Espaa

0 0 0,832 0,5547 0
0 0 = 0,5547
0,832
0
0 1 0
0
1
0
0
0,1

0,222 0,028 0
(V )ij 0,028 0,5 0
0
0
0,1

Draft

Por: Eduardo W. V. Chaves (2014)

2 CINEMTICA DEL CONTINUO

231

Ejemplo 2.62
Para un movimiento dado (deformacin de corte):
x1 = X 1 + kX 2

x2 = X 2

x3 = X 3

donde k es una constante. Encontrar los tensores F (Gradiente de deformacin),


C (Tensor derecho de deformacin de Cauchy-Green), b (Tensor izquierdo de
deformacin de Cauchy-Green), E (Tensor de deformacin de Green-Lagrange), U
(Tensor derecho de estiramiento), V (Tensor izquierdo de estiramiento) y R (Tensor de
rotacin de la descomposicin polar).
Solucin:
Tensor gradiente de deformacin:
x1

X 1
xi x 2
Fij =
=
X j X 1

x3
X 1

x1
X 2
x 2
X 2
x3
X 2

x1

X 3 1 k 0
x 2
= 0 1 0

X 3

x3 0 0 1
X 3

Tensor derecho de deformacin de Cauchy-Green ( C = F T F ), cuyas componentes son:


k
1 0 0 1 k 0 1

C ij = Fki Fkj = k 1 0 0 1 0 = k 1 + k 2
0 0 1 0 0 1 0
0

0
0
1

Tensor izquierdo de deformacin de Cauchy-Green ( b = F F T ), cuyas componentes son:


2
1 k 0 1 0 0 1 + k

bij = Fik F jk = 0 1 0 k 1 0 = k
0 0 1 0 0 1 0

Tensor material de deformacin de Green-Lagrange, E =

1 0
0 1

1
(C 1) , cuyas componentes
2

son:
1
k
1
E ij = k 1 + k 2
2
0
0

0 1 0 0
0 k
1

0 0 1 0 = k k 2
2
0 0
1 0 0 1

0
0
0

Verifiquemos que slo hay deformacin en el plano x1 x 2 .


Teniendo en cuenta la descomposicin polar F = R U = V R , y que:
C = ( V R )T ( V R ) = R T V T V R = R T V V R = R T V 2 R = R T b R

Por simplicidad vamos trabajar slo en el plano x1 x 2 , con lo cual representaremos las
componentes del tensor de rotacin como:
cos sin c s
R ij =

=
sin cos s c

(i, j = 1,2)

donde, se cumple que cos 2 + sin 2 = c 2 + s 2 = 1 . La relacin C = R T b R queda:

Universidad de Castilla- La Mancha


Ciudad Real - Espaa

Draft

Por: Eduardo W. V. Chaves (2014)

MECNICA DEL MEDIO CONTINUO: PROBLEMAS RESUELTOS

232

k c s 1 + k 2
1
k 1 + k 2 = s c

( sck 2 s 2 k + c 2 k )
k c s (c 2 + c 2 k 2 + 2 sck + s 2 )
=


1 s c ( sck 2 s 2 k + c 2 k )
(c 2 + s 2 k 2 2sck + s 2 )

k
c.
2
k
Ahora partiendo de la relacin ( sck 2 s 2 k + c 2 k ) = k y considerando que s =
c
2

De la relacin (c 2 + c 2 k 2 + 2 sck + s 2 ) = 1 (c 2 k 2 + 2 sck + 1) = 1 obtenemos que s =

obtenemos que:
c=

1
2

k
+1
4

s=

k +4

k
2
2

k
+1
4

k
k2 + 4

Luego:
2

2
k +4
k
R ij =
2
k +4
0

k
2

k +4
2
k2 + 4
0

De la descomposicin polar F = R U = V R , podemos obtener que U = R T F , y que


V = F R T , cuyas componentes son:
2

2
k +4
k
U ij = R ki Fkj =
2
k +4
0

Vij = Fik R jk

k
k2 + 4
2
k2 + 4
0

2
1 k 0 k + 4
k
= 0 1 0
2
0 0 1 k + 4
0

0
2
1 k 0 k + 4
k

0 0 1 0 =
2
0 0 1 k + 4
1
0

k
k2 + 4
2
k2 + 4
0

2
2+k
0
2
k +4
k

0 =
2
k +4
1
0

k
2

k +4
2+ k2
k2 + 4
0

k
k2 + 4
2
k2 + 4
0

Ejemplo 2.63
Un paraleleppedo deformable de dimensiones 2 2 1 se encuentra en su configuracin
de referencia en la posicin que indica la Figura 2.19. Este cuerpo se somete a una
deformacin:
r r
x ( X , t ) = exp X 2t e 1 + tX 12 e 2 + X 3 e 3

(2.153)

siendo ( X 1 , X 2 , X 3 ) las coordenadas materiales y t el tiempo. Para este cuerpo se pide:


r

a) Obtener las componentes del gradiente de deformacin F , en todo punto X e instante


t.

Universidad de Castilla- La Mancha


Ciudad Real - Espaa

Draft

Por: Eduardo W. V. Chaves (2014)

2 CINEMTICA DEL CONTINUO

233

b) Lo mismo para el tensor derecho de deformacin de Cauchy-Green C . Cules son los


estiramientos principales?
c) Obtener tambin las componentes correspondientes al tensor derecho de estiramiento
U y al tensor de rotacin R . Comprobar que este ltimo es un tensor ortogonal propio.
d) Cul es el volumen del paraleleppedo deformado en el instante t = 1s ?
X2

X3

X1

Figura 2.19.
Solucin:
a) Segn Ec. (2.153), las componentes del vector posicin son x1 = exp X 2t , x 2 = tX 12 ,
x3 = X 3 , luego las componentes del gradiente de deformacin F vienen dadas por:
0
xi
Fij =
= 2tX 1
X j
0

0
1

t exp X 2t
0
0

b) El tensor derecho de deformacin de Cauchy-Green C viene definido por C = F T F ,


con componentes C ij = Fki Fkj :
0

C ij = t exp X 2t

0 0

0 2tX 1
1 0

2tX 1
0
0

0 4t 2 X 12

0 = 0
1 0

t exp X 2t
0
0

0
2

t exp
0

2 X 2t

0
1

Observemos que este espacio es el espacio de las direcciones principales de C . Si i son


los estiramientos principales se cumple la siguiente relacin:
C = U2 =

2
a

(a ) N
(a)
N

U=

a =1

(a) N
(a)
N

a =1

Como estamos en el espacio principales, podemos obtener los estiramientos principales:


1 = + 4t 2 X 12

2 = + t 2 exp 2 X 2t

3 = + 1

(a ) N
( a ) es por definicin un tensor definido positivo,
Ya que el tensor U = a N
a =1

implicando que sus valores principales son positivos, luego:


1 = 2tX 1

Universidad de Castilla- La Mancha


Ciudad Real - Espaa

2 = t exp X 2t

Draft

3 = 1

Por: Eduardo W. V. Chaves (2014)

MECNICA DEL MEDIO CONTINUO: PROBLEMAS RESUELTOS

234

2tX 1
U ij = 0
0

c)

0
0
1

0
t exp
0

X 2t

U ij1

1
2tX
1

= 0

0
1
t exp X 2t
0

Segn la descomposicin polar F = R U R = F U 1 , con eso podemos obtener las


componentes del tensor ortogonal propio R :
0

R ij = 2tX 1
0

t exp

0 2tX 1

0 0

1
0

X 2t

0
0

0
1
t exp X 2t
0

0
0 1 0
0 = 1 0 0

0 0 1
1

Verificamos que debe cumplir la ortogonalidad R R 1 = R R T = 1 :


R ik R jk

0 1 0 0 1 0 1 0 0
= 1 0 0 1 0 0 = 0 1 0
0 0 1 0 0 1 0 0 1

y propio ya que det (R ) = 1 .


d) Para calcular el volumen final utilizaremos la relacin dV = JdV0 , donde J = F es el
determinante del Jacobiano y viene dado por:
t exp X 2t

0
J = 2tX 1
0

0
0 = 2t 2 X 1exp X 2t
1

0
0

Para el tiempo t = 1s tenemos que J = 2 X 1exp X 2 . Luego el volumen en el tiempo t = 1s


vendr dado por:

dV =

JdV0 =

V0

(2 X exp )dX
1

X2

3 dX 2 dX 1

= 4 exp 2 1 25,556

X 1 =0 X 2 = 0 X 3 = 0

Obs.: No se puede utilizar la expresin V = JV0 porque no se trata de un caso de


deformacin homognea.
Ejemplo 2.64
Un cuerpo continuo experimenta la deformacin:
x1 = X 1

x 2 = X 2 + kX 3

x3 = X 3 + kX 2

donde k es una constante.


a) Determinar el gradiente de deformacin ( F ); el tensor derecho de deformacin de
Cauchy-Green ( C ); el tensor de deformacin de Green-Lagrange ( E ).
b) Calcular el campo de desplazamiento, la longitud al cuadrado (dx) 2 de los lados OA y
OB , y de la diagonal OC , despus de la deformacin en el pequeo rectngulo indicado en

la figura abajo.

Universidad de Castilla- La Mancha


Ciudad Real - Espaa

Draft

Por: Eduardo W. V. Chaves (2014)

2 CINEMTICA DEL CONTINUO

235

X3

dX 2

dX 3
X1

X2

c) Considrese ahora un cuadrado como la figura abajo


x3

C
23

B
B

x2

c.1) Determinar los estiramientos segn las direcciones OC y BA ; c.2) el ngulo 23 en la


configuracin actual en funcin de k .
c.3) Aplicar la descomposicin polar del tensor F , es decir, determinar U y R
Solucin:
a) C = F T F . Las componentes del gradiente de deformacin son:
1 0 0
xi
= 0 1 k
Fij =
X j
0 k 1
0
0
1 0 0 1 0 0 1

2
C ij = Fki Fkj = 0 1 k 0 1 k = 0 1 + k
2k
0 k 1 0 k 1 0
2k
1 + k 2

Tensor material de deformacin de Green-Lagrange, E =

1
(C 1) , cuyas componentes
2

son :
1
0
0
0 1 0 0
0 0
1
1

2
2
2k
2 k 0 1 0 = 0 k
E ij = 0 1 + k
2
2
0 2k k 2
2k
1 + k 2 0 0 1
0
r r r
b.1) Campo de desplazamientos, u = x X , cuyas componentes son:

Universidad de Castilla- La Mancha


Ciudad Real - Espaa

Draft

Por: Eduardo W. V. Chaves (2014)

MECNICA DEL MEDIO CONTINUO: PROBLEMAS RESUELTOS

236

u1 = x1 X 1 = 0 ; u 2 = x 2 X 2 = kX 3
; u 3 = x 3 X 3 = kX 2
r
b.2) Clculo de (dx ) 2 = dx 2
r
r
r
r
r
r
(dxr )2 = dxr dxr = F dX F dX = dX F T F dX = dX C dX

Explcitamente:

(dx )

= [dX 1

0
0 dX 1
1

2
dX 3 ] 0 1 + k
2k dX 2
0
2k
1 + k 2 dX 3

dX 2

= (dX 1 ) 2 + (dX 2 ) 2 (1 + k 2 ) + (dX 3 ) 2 (1 + k 2 ) + 4k (dX 2 )(dX 3 )

Luego, para la diagonal OC tenemos que [0 dX 2 dX 3 ] , resultando que:

(dx )2 = (dX 2 ) 2 (1 + k 2 ) + (dX 3 ) 2 (1 + k 2 ) + 4k (dX 2 )(dX 3 )


Para el lado OA tenemos que [0 dX 2 0] , resultando:

(dx )2 = (dX 2 ) 2 (1 + k 2 )
Para el lado OB tenemos que [0 0 dX 3 ] , resultando que:

(dx )2 = (dX 3 ) 2 (1 + k 2 )
(configuracin de referencia) viene dado por el la
c) El estiramiento segn una direccin N
C N
.
expresin ( N )2 = N

c.1) Estiramiento segn direccin OC : N i = 0

( )

OC

= 0

1
2

0
1
1
2
0 1 + k
2
0
2k

Estiramiento segn direccin BA : N i = 0

( )

BA

= 0

1
2

1
2

0
1
1
2
0 1 + k
2
0
2k

1
, con lo cual:
2

0
2k

1 + k 2

1
= (1 + k ) 2

2
1

1
, con lo cual:
2

0
0

2k
= (1 k ) 2

2
1 + k 2 1

c.2) Variacin del ngulo. Podemos utilizar directamente la expresin:


cos =

Universidad de Castilla- La Mancha


Ciudad Real - Espaa

C N

C N

M
M
=
M N
C M
N
C N

Draft

Por: Eduardo W. V. Chaves (2014)

2 CINEMTICA DEL CONTINUO

237

= [0 0 1] , y segn direccin OA es
donde el versor segn direccin OB es M
i
N i = [0 1 0] . Con eso obtenemos que:

( )

0
0 0
1

2
= [0 0 1] 0 1 + k
2k 0 = 1 + k 2
0
2k
1 + k 2 1

( )

0
0 0
1

2
= [0 1 0] 0 1 + k
2k 1 = 1 + k 2
0
2k
1 + k 2 0

OB

OA

0
0 0
1

Mi C ij N j = [0 0 1] 0 1 + k
2k 1 = 2k
0
2k
1 + k 2 0

Resultando que:
cos 23 =

C N

2k
M
=
M N
1+ k2

c.3) Descomposicin Polar F = R U = V R , donde:


C = U2 =

(a ) N
(a )
aN

U= C =

a =1

(a) N
(a)
aN

a =1

Clculo de los valores principales de C . Verifiquemos que por el formato de las


componentes del tensor C , slo hay deformacin segn el plano x 2 x 3 . Adems ya
conocemos un autovalor 1 = 1 asociado a la direccin Ni(1) = [1 0 0] . Simplificando as el
determinando caracterstico como:
(1 + k 2 )

2k

2k

(1 + k )

) (

2 2 1 + k 2 + 1 k 2

=0

) (

2 2 1 + k 2 + 1 2k 2 + k 4 = 0

=0

Las races son: 2 = 1 + k 2 + 2k = (1 + k ) 2

3 = 1 + k 2 2k = (1 k ) 2

Luego, en el espacio principal de C , tenemos que:


0
1

C ij = 0 (1 + k ) 2
0
0

(1 k ) 2
0
0

Las direcciones principales son 2 Ni( 2) = 0

1
( 3)
, 3 N i = 0
2

1
2

1
.
2

Luego, la matriz de transformacin entre el espacio original y el espacio principal queda:

1
a ij = A = 0

Universidad de Castilla- La Mancha


Ciudad Real - Espaa

Draft

0
1
2
1
2

0
1
2
1

2
Por: Eduardo W. V. Chaves (2014)

MECNICA DEL MEDIO CONTINUO: PROBLEMAS RESUELTOS

238

Es decir, se debe cumplir que:


C = A C AT
0
1
0 (1 + k ) 2

0
0

0 1
0 = 0

(1 k ) 2
0

0 1
0
1
0 1+ k2
2
2k
1 0

0
1
2
1
2

0 1
2k 0

1 + k 2
0

0
1
2
1

0
1
2
1
2

Luego, en el espacio principal de C , tenemos que:


0
1

C ij = 0 (1 + k ) 2
0
0

(1 k ) 2
0
0

+ 1
0

U ij = 0
+ (1 + k ) 2
0
0

2
+ (1 k )
0
0

0
0
1

U ij = 0 (1 + k )
0
0
0
(1 k )

La inversa en el espacio principal:

Uij1

1
= 0

0
0

(1 k )

0
1
(1 + k )
0

Las componentes del tensor U en el espacio original vienen dadas por:


U 1 = A T U 1 A

1
U ij = 0

0
1
2
1
2

0
1
2
1

1
0

0
1
(1 + k )
0

0 1
0 0

1
0
(1 k )

0
1
2
1
2



0 1
1
= 0
2
1
0
2

0
1
(1 k 2 )
k
(1 k 2 )

0
k
(1 k 2 )

1
(1 k 2 )

De la descomposicin polar obtenemos que F = R U R = F U 1

1 0 0 1

R ij = 0 1 k 0
0 k 1
0

Universidad de Castilla- La Mancha


Ciudad Real - Espaa

0
1
(1 k 2 )
k
(1 k 2 )

Draft

0 1 0 0
k
= 0 1 0
2
(1 k )

1 0 0 1
(1 k 2 )

Por: Eduardo W. V. Chaves (2014)

2 CINEMTICA DEL CONTINUO

239

Ejemplo 2.65
Dada la siguiente ley de movimiento:
x1 = 1 X 1

x 2 = 3 X 3
x = X
2 2
3

Si pide:
a) Encontrar el volumen deformado para un cubo unitario;
b) Encontrar el rea deformada de un cuadrado unitario en el plano X 1 X 2 , y dibujar el
rea deformada;
c) Aplicar la Descomposicin Polar y obtener los tensores U , V y R
Solucin:
a)
x1 1

x2 = 0
x 0
3

0 X 1

3 X 2
0 X 3

0
0
2

1
Fij = 0
0

0
3 (deformacin Homognea)
0

0
0
2

El determinante de F viene dado por F J = 1 2 3 , y el volumen deformado:


dV = F dV0 integrando
V final = F Vinicial = 1 2 3

b) Aplicando la relacin de Nanson y teniendo en cuenta que estamos en el caso particular


de deformacin homognea:
r
r
r
r
da = JF T dA integrando
a final = JF T Ainicial

donde
e 1
r
Ainicial = 1

e 2
0

e 3
0 = e 3 ; Fij1 =
0

2 3
1
0
1 2 3
0

0
0
1 2

0 1
1 3 = 0

0
0

0
0
1
3

1
2

Con lo cual el vector rea deformada queda:


1

a1
1

a 2 = 1 2 3 0
a

3
0

0
0
1
2

0
0 0
1

0 = 1 2

3
1 0
0

Su mdulo queda:
r
a final = ( 1 2 ) 2 = 1 2

Universidad de Castilla- La Mancha


Ciudad Real - Espaa

Draft

Por: Eduardo W. V. Chaves (2014)

MECNICA DEL MEDIO CONTINUO: PROBLEMAS RESUELTOS

240

X 3 , x3
B (0,0, 2 )

r
a final = 1 2

C (1 ,0, 2 )
O (0,0,0)

B (0,1,0)

r
Ainicial = 1

C (1,1,0)

A(1,0,0)

X 2 , x2

A( 1 ,0,0)
X 1 , x1

donde los puntos A(1,0,0) , B(0,1,0) y C (1,1,0) se desplazan segn la ley del movimiento:
x1A 1
A
x2 = 0
x A 0
3
x1C 1
C
x2 = 0
xC 0
3

0
0
2

0 1 1

3 0 = 0
0 0 0

x1B 1
B
x2 = 0
x B 0
3

0
0
2

0 1 1

3 1 = 0
0 0 2

0
0
2

c) Segn la definicin de la descomposicin polar


U= C = F

1
C ij = 0
0
1
bij = 0
0

0 0 0

3 1 = 0
0 0 2

F y
0
0
3
0
0
2

V = b = F F

0 1
2 0
0 0
0 1
3 0
0 0

0
0
2
0
0
3

F = R U = V R

donde

0 21

3 = 0
0 0
0 21

2 = 0
0 0

0
22
0
0
23
0

0
23
0

0
22

1
U ij = 0
0

1
Vij = 0
0

0
2
0
0
3
0

0
0
3
0
0
2

Verifiquemos que el espacio original coincide con el espacio principal de C . Verifiquemos


tambin que C y b tienen los mismos autovalores pero direcciones principales distintas.
Para obtener el tensor de rotacin de la descomposicin polar R = F U 1 = V 1 F , con lo
cual:
1
R ij = 0
0

Universidad de Castilla- La Mancha


Ciudad Real - Espaa

0
0
2

0 1
3 0

0
0

Draft

0
1
2
0

0
1 0 0
0 = 0 0 1


1 0 1 0
3

Por: Eduardo W. V. Chaves (2014)

2 CINEMTICA DEL CONTINUO

241

jemplo 2.66
Determinar para la deformacin homognea:
x1 = 3 X 1

x2 = 2 X 2

x3 = 3 X 3 X 2

el elipsoide de deformacin material que resulta de la deformacin de una esfera material


X 12 + X 22 + X 32 = 1 (ver Figura 2.20). Probar que este elipsoide en el espacio principal del
tensor izquierdo de estiramiento V tiene la forma:
x1 2
21

x 2 2
22

x3 2
23

=1

donde 1 , 2 , 3 son los estiramientos principales.

X 2 , x2

X 3 , x3

Superficie material
(Siempre constituida por las
mismas partculas)

X 1 , x1

Figura 2.20: Esfera material.


Solucin:
La ley del movimiento y de su inversa vienen dadas por:
x1 3

x2 = 0
x 0
3

0
2
1

0 X 1
X1


0 X 2 inversa
X 2 =
X
3 X 3
3

3
3

0
1
2
3
6

0
0

0
x1

0 x2

3 x3

Las ecuaciones del movimiento en la descripcin espacial viene dada por:


X1 =

3
x1
3

X2 =

x2
2

X3 =

3
3
x2 +
x3
6
3

Reemplazando en la ecuacin de la esfera:


2

X 12

X 22

X 32

=1

3 x2 2 3
3

x +
+
x +
x
=1
3 1 2 6 2 3 3

Tras la simplificacin de la expresin anterior obtenemos que:


Universidad de Castilla- La Mancha
Ciudad Real - Espaa

Draft

Por: Eduardo W. V. Chaves (2014)

MECNICA DEL MEDIO CONTINUO: PROBLEMAS RESUELTOS

242

x12 + x 22 + x32 + x 2 x3 = 3

Que es la ecuacin de un elipsoide. Tenemos ahora que representar la ecuacin de este


elipsoide en el espacio principal del tensor derecho de estiramiento V . Recordemos que el
tensor V y el tensor b son coaxiales (tienen las mismas direcciones principales), y adems
se cumple que:
V = b = F FT

A continuacin obtenemos las componentes del tensor b , y sus autovalores y autovectores.


3

bij = 0
0

0
2
1

0 3 0

0 0
2

3 0 1

0
0
0
3

0 = 0
5
3
0 3
3
3

Verificamos que ya conocemos un autovalor y autovector b1 = 3 , n i(1) = [1 0 0] . Luego,


las otras direcciones principales estarn el plano x 2 x 3 . Obteniendo los dems
autovalores y autovectores

n (i 2 ) = 0
b2 = 6 autovector

2
2

n (i 3) = 0
b3 = 2 autovector

2
2

2
2

Resultando as que:

1
3 0 0

bij = 0 6 0 Matriz
deTrasnfor
macin
aij = 0

0 0 2

1 = 3
0

Vij = 0
2 = 6
0
0

0
2
2
2
2

0
2
2
2

0
3 = 2

Luego, aplicando la ley de transformacin del sistema x1 , x 2 , x3 al sistema x1 , x 2 , x3 ,


obtenemos que:

x1 1

x2 = 0
x
3
0

2
2
2
2

0
2
2
2

x1

x 2
x
3

x1 = x1

2
2

x 2 +
x3
x2 =
2
2

2
2
x 2 +
x3
x3 =
2
2

Con lo cual, la ecuacin del elipsoide en el espacio principal de V viene representada por:

Universidad de Castilla- La Mancha


Ciudad Real - Espaa

Draft

Por: Eduardo W. V. Chaves (2014)

2 CINEMTICA DEL CONTINUO

243

x12 + x 22 + x32 + x 2 x3 = 3

(x1 )

2
2
2
2
2 2
2
+
x +
x3 +
x 2 +
x3 +
x 2 +
x3
x 2 +
x 3 = 3
2 2

2
2
2
2

2
2

Simplificando la expresin anterior obtenemos que:


x1 2 x 2 2 x3 2
x1 2
x 2 2
x3 2
x1 2 x 2 2 x 3 2
+
+
=
+
+
= 2 + 2 + 2 =1
3
6
2
2
3
( 3 ) 2 ( 6 ) 2 ( 2 ) 2 1
X 3 , x3

X 2 , x2

x3

3 = 2
2 = 6

1 = 3

x 2

x1

X 1 , x1

Figura 2.21: Elipsoide material (configuracin deformada).

x2
x1

x3

x 2

X2

X1

x2
x1

F
x3

X3

x1

x3

Figura 2.22: Descomposicin polar por la izquierda.

Universidad de Castilla- La Mancha


Ciudad Real - Espaa

Draft

Por: Eduardo W. V. Chaves (2014)

MECNICA DEL MEDIO CONTINUO: PROBLEMAS RESUELTOS

244

2.4 Deformacin Infinitesimal


Ejemplo 2.67
Dadas las ecuaciones del movimiento
x1 = X 1 + 4 X 1 X 2t

x2 = X 2 + X 22t

x3 = X 3 + X 32t

(2.154)

a) Encontrar el campo de velocidad;


b) Encontrar el campo de deformacin infinitesimal;
c) Para el tiempo t = 1 s , obtener el tensor de deformacin infinitesimal.
Solucin:
a) Velocidad:
V1 = 4 X 1 X 2
r
r r

dx
V 2 = X 22
V ( X , t) =
dt

2
V3 = X 3

(2.155)

r
A1 = 0
r r
dV

A( X , t ) =
A2 = 0
dt
A = 0
3

(2.156)

b) Aceleracin:

c) Campo de desplazamientos:
u1 = x1 X 1 = X 1 + 4 X 1 X 2 X 1 = 4 X 1 X 2

2
2
u 2 = x2 X 2 = X 2 + X 2 X 2 = X 2

2
2
u3 = x3 X 3 = X 3 + X 3 X 3 = X 3

(2.157)

Luego, las componentes del tensor de deformacin infinitesimal vienen dadas por:
ij =
u1

X 1
u i u 2
=
x j X 1

u 3
X 1

u1
X 2
u 2
X 2
u 3
X 2

1 u i u j
+
2 x j xi

u1

X 3 4 X
2
u 2
0
=
X 3

u 3 0
X 3

(2.158)

4X1
2X 2
0

0
0
2 X 3

(2.159)

luego:
4 X 2
ij = 2 X 1
0

Universidad de Castilla- La Mancha


Ciudad Real - Espaa

2 X1
2X 2
0

Draft

0
0
2 X 3

(2.160)

Por: Eduardo W. V. Chaves (2014)

2 CINEMTICA DEL CONTINUO

245

Ejemplo 2.68
Consideren el siguiente tensor de deformacin infinitesimal:

0
2
X
23
l
X X
22 3
l

0
0

X
X
ij = 0 2 2 3

X2
0 3
l2

(2.161)

y el tensor de rotacin infinitesimal:

0
0

ij = 0
0

2
2
0 2 X 2 X 3
2l

2l 2

X 22 X 32

(2.162)

Hallar las componentes del campo de desplazamientos.


Solucin:
El gradiente de los desplazamientos viene relacionado con el tensor de deformacin
infinitesimal y el tensor spin como:
u i , j = ij + ij

ij =

1
ui, j + ui, j
2

; ij =

(2.163)

2
3X 3
2 X 2 X 3

(2.164)

1
ui, j ui, j
2

luego:
0

u i , j = 2 0
2l

2X 2 X 3

0 X 22 + X 32

X 22

u1
=0
u1 = 0
x1
u2

= 2 (2 X 2 X 3 )
x2 2l

u3

= 2 (2 X 2 X 3 )
x3
2l

u = 2l (2 X
2

2X3

)x2 u2 = 2 [X 22 X 3 + C1 ( X 3 )]

u = 2l (2 X
3

(2.165)
2l

2X3

(2.166)

)x3 u3 = 2 [X 32 X 2 + C2 ( X 2 )]
2l

Para determinar la constante C1 ( X 3 ) del resultado (2.166) derivamos con respecto a X 3 :

C ( X )
C ( X )
u 2

= 2 X 22 + 1 3 = 2 X 22 3 X 32 1 3 = 3 X 32
X 3 2l
X 3 2l
X 3

(2.167)

C1 ( X 3 ) = X 33

Anlogamente hacemos para determinar la constante C2 ( X 2 ) :

Universidad de Castilla- La Mancha


Ciudad Real - Espaa

Draft

Por: Eduardo W. V. Chaves (2014)

MECNICA DEL MEDIO CONTINUO: PROBLEMAS RESUELTOS

246

u 3

= 2
X 2
2l

2 C 2 ( X 2 )
C 2 ( X 2 )

2
2
= X 22
X 3 +
= 2 X2 + X3
X

2l
2
2

(2.168)

X3
C2 ( X 2 ) = 2
3

Luego, el campo de desplazamientos viene dado por:


u1 = 0 ; u 2 =

2l 2

[X

2
2 X3

X 33

; u3 =

X 23
2
+
X
X
3 2

3
2l 2

(2.169)

Ejemplo 2.69
Demostrar que, para el caso de pequeas deformaciones, la tasa del tensor de deformacin
infinitesimal ( & ) es igual al tensor tasa de deformacin ( D ).
Solucin:
Consideremos la relacin entre la tasa del tensor material de deformacin de GreenLagrange ( E& ) y el tensor tasa de deformacin ( D ):
E& = F T D F

(2.170)

Para el caso de pequeas deformaciones se cumple que F 1 , y adems se cumple


tambin que E& e& & luego:
E& = & = D

(2.171)

Ejemplo 2.70
Dado el movimiento
x1 = X 1

x2 = X 2 + X 1 exp 2t 1

x3 = X 3 + X 1 exp 3t 1

(2.172)

Encontrar el tensor tasa de deformacin ( D ) y compararlo con la tasa del tensor


infinitesimal de deformacin ( & ).
Solucin:
Por definicin el tensor tasa de deformacin ( D ) es la parte simtrica del tensor gradiente
espacial de la velocidad:
1
(l + l T )
r
r
y ( x , t ) = sym u
2
r
l = xv

D=

(2.173)

El tensor infinitesimal de deformacin por definicin es igual la parte simtrica del


gradiente de los desplazamientos:
r
D
= (u) sym &
(2.174)
Dt
r r r
El campo de desplazamientos viene dado por u = x X . Considerando las ecuaciones del

movimiento dadas, las componentes del campo de desplazamiento quedan:


u1 = x1 X 1 = X 1 X 1 = 0

2t
2t
u 2 = x 2 X 2 = X 2 + X 1 exp 1 X 2 = X 1 exp 1

3t
3t
u 3 = x 3 X 3 = X 3 + X 1 exp 1 X 3 = X 1 exp 1

(
(

Universidad de Castilla- La Mancha


Ciudad Real - Espaa

Draft

)
)

(
(

)
)

Por: Eduardo W. V. Chaves (2014)

2 CINEMTICA DEL CONTINUO

247

r
r Du
El campo de velocidades viene definido por v =
. Luego, las componentes del campo
Dt

de velocidades, en coordenadas materiales, son:


V1 = 0

2t
V2 = X 1 (2exp )

3t
V3 = X 1 ( 3exp )

(2.175)

Teniendo en cuenta las ecuaciones inversas del movimiento:


x1 = X 1

2t
x 2 = X 2 + X 1 (exp 1)

3t
x3 = X 3 + X 1 (exp 1)

X 1 = x1

2t
X 2 = x 2 x1 (exp 1)

3t
X 3 = x3 x1 (exp 1)

(2.176)

podemos obtener el campo de velocidades en coordenadas espaciales:


v1 = 0

v2 = 2 x1exp 2t

v3 = 3 x1exp 3t

(2.177)

Las componentes del tensor gradiente espacial de la velocidad ( l ) vienen dadas por:
0

vi
r
= 2exp 2t
( l ) ij = ( x v ) ij =
x j
3t
3exp

0 0
0 0
0 0

0
0 0
0
1
1

2t
(D) ij = ( l ij + l ji ) =
0 0 + 2exp 2t
2exp
2
2
3exp 3t 0 0 3exp 3t

0
exp 2t exp 3t

0
0
= exp 2t

3 exp 3t
0
0

Tambin obtenemos el tensor spin W = l

Wij =

1
l ij l ji
2

(2.178)
T
0 0
0 0

0 0

(2.179)

anti

= exp 2t
3 exp 3t
2

exp 2t
0
0

exp 3t
2

(2.180)

Tensor de deformacin Infinitesimal ( )


Conocido el campo de desplazamiento:
u1 = 0

2t
u 2 = x1 (exp 1)

3t
u 3 = x1 (exp 1)

(2.181)

Las componentes del gradiente de desplazamientos vienen dadas por:

Universidad de Castilla- La Mancha


Ciudad Real - Espaa

Draft

Por: Eduardo W. V. Chaves (2014)

MECNICA DEL MEDIO CONTINUO: PROBLEMAS RESUELTOS

248

0
0 0

r
u i
2t
(u)ij = = exp 1 0 0
x j
exp 3t 1 0 0
r
Podemos decomponer (u) en una parte simtrica y una antisimtrica:
r
r
r
(u)ij = symu ij + anti u ij = ( )ij + ()ij

(
(

) (

)
)

(2.182)

(2.183)

La parte simtrica:

r
sym u ij

0
0 0
0
0
1

2t
2t
exp 1 0 0 + exp 1 0
=
2
exp 3t 1 0 0 exp 3t 1 0

0
exp 2t 1 exp 3t 1

1
0
0
= exp 2t 1
= ij
2
3t

0
0
exp 1

(
(

)
)

(
(

)
)

0
0
0

(2.184)

Tambin proporcionamos el tensor spin infinitesimal:

( )ij

0
(exp 2t 1) (exp 3t 1)0

1
0
0
= (exp 2t 1)

2
3t

(exp
1
)
0
0

(2.185)

Luego, la tasa de :

(& )ij


exp 2t 1 exp 3t 1
0

D
( )ij = D 1 exp 2t 1
0
0
=

Dt
Dt 2

exp 3t 1
0
0

0
exp 2t exp 3t

0
0
= exp 2t

3 exp 3t
0
0

(2.186)

Con lo que concluimos que:


D = &

(2.187)

Ejemplo 2.71
Consideremos un cuerpo material bajo el rgimen de pequeas deformaciones, el cual est
sometido al siguiente campo de desplazamientos:
u1 = (2 x1 + 7 x 2 ) 10 3

; u 2 = (10 x 2 x1 ) 10 3

; u 3 = x3 10 3

a) Encontrar el tensor de deformaciones infinitesimales, y el tensor spin infinitesimal;


b) Encontrar los invariantes principales del tensor de deformacin infinitesimal, y las
deformaciones principales;
c) Dibujar el crculo de Mohr en deformaciones, y obtener la deformacin tangencial
mxima;
Universidad de Castilla- La Mancha
Ciudad Real - Espaa

Draft

Por: Eduardo W. V. Chaves (2014)

2 CINEMTICA DEL CONTINUO

249

d) Encontrar la deformacin volumtrica lineal y el tensor de deformacin infinitesimal


desviador.
Solucin
a) El gradiente de los desplazamientos:

( u)ij

u1

x1
u i u 2
=
=
x j x1

u 3
x1

u1
x 2
u 2
x 2
u 3
x 2

u 1

x 3 2
7
0
u 2
= 1 10 0 10 3

x 3

0
1
u 3 0
x 3

Tensor spin infinitesimal:

ij =

anti

u ij

u j
1 u
= i
2 x j x i

0 4 0

= 4 0 0 10 3


0 0 0

Tensor de deformacin infinitesimal:

ij =

sym

u ij

u j
1 u
= i +
2 x j xi

3 0
2

= 3 10 0 10 3

0
1

b) Las deformaciones principales (autovalores) se obtienen al resolver el determinante


caracterstico:
3
0
2
3
10
0 10 3 = 0

0
0
1

donde = 10 3 . Al desarrollar el determinante anterior obtenemos la ecuacin


caracterstica 3 I 2 + II III = 0 , donde los invariantes principales de vienen
definidos por I = Tr ( ) , II =

1
[Tr()]2 Tr ( 2 ) , III = det() , ver captulo 1. Luego,
2

para el problema propuesto los invariantes son:


I = Tr ( ) = (2 10 + 1) 10 3 = 11 10 3
2
3
0 2
3
0 2
3
0

1
2
2
II = [Tr ( )] Tr ( ) = 3 10 0 + 3 10 0 + 3 10 0 10 6 = 1 10 6
2
0
0
1
0
0 1
0
0
1

III = det ( ) = 11 10 9

Resultando en la siguiente ecuacin caracterstica:


3 I 2 + II III = 0
3 + 11 10 3 2 + 11 10 6 11 10 9 = 0

Al resolver la ecuacin anterior obtenemos los autovalores de (deformaciones


principales). Pero, si nos fijamos en el formato de las componentes de verificamos que
33 = 1 10 3 ya es una deformacin principal y que est asociada a la direccin

Universidad de Castilla- La Mancha


Ciudad Real - Espaa

Draft

Por: Eduardo W. V. Chaves (2014)

MECNICA DEL MEDIO CONTINUO: PROBLEMAS RESUELTOS

250

n i = [0 0 1] . Luego, para obtener los dems autovalores es suficiente resolver el

siguiente sistema:
3
2
3
3
10 = 0

10

+ 12 10 + 11 10

=0

2 + 12 10 3 + 11 10 6 = 0
1 = 1,0 10 3

2 = 11,0 10 3

c) Para dibujar el crculo de Mohr en deformaciones, ver Apndice A, tenemos que


reestructurar las deformaciones principales tal que I II III , i.e.:
I = 1,0 10 3

II = 1,0 10 3

III = 11,0 10 3

La deformacin tangencial mxima viene dada por:


III
1
max = S max = I
= 6 10 3
2
2

El crculo de Mohr en deformaciones se puede apreciar en la figura abajo.


S (10 3 )
S max = 12 max = 6

II = 1
III = 11

N (10 3 )

I = 1

d) La deformacin volumtrica lineal - V :


V = I = Tr ( ) = 12 10 3

Haciendo la descomposicin aditiva de en una parte esfrica y otra desviadora,


= esf + dev , donde la parte esfrica viene dada por:
ijesf

0
4 0
Tr ( )

=
ij = 0 4 0 10 3
3
0
0 4

Y, la parte desviadora por:


ijdev

= ij

ijesf

2
3
0 4 0
0
2 3 0

3
= 3 10 0 0 4 0 10 = 3 6 0 10 3
0
0 0 4
0
0 0
0 4

Universidad de Castilla- La Mancha


Ciudad Real - Espaa

Draft

Por: Eduardo W. V. Chaves (2014)

2 CINEMTICA DEL CONTINUO

251

Ejemplo 2.72
En un punto de un slido el gradiente de los desplazamientos viene representado por sus
componentes como:
4 1 4
r
(u) ij = 1 4 2 10 3
4 0
6

(2.188)

Determinar:
a)

Las componentes del tensor infinitesimal de deformacin y rotacin;

b) Las componentes de la parte esfrica y desviadora


deformacin;
c)

del tensor infinitesimal de

Los invariantes principales de : I , II , III ;

d) Los autovalores y autovectores del tensor de deformacin.


Solucin:
a) El tensor infinitesimal de deformacin ( ) viene dado por la parte simtrica del
gradiente de los desplazamientos:

r 1
r
r
= sym u = (u) + (u) T
2

(2.189)

Luego:
4 1 4 4
1 4
8 0 0
4 0 0

1
1

3
3
ij = 1 4 2 + 1 4 0 10 = 0 8 2 10 = 0 4 1 10 3
2
2
0 2 12
0 1 6
6 4 2 6
4 0
r
El tensor spin infinitesimal = anti u
4 1 4 4
1 4
0 2 8
0 1 4

1
1

3
3
ij = 1 4 2 1 4 0 10 = 2 0
2 10 = 1 0
1 10 3
2
2
8 2 0
4 1 0
6 4 2 6
4 0

b) Descomponiendo de forma aditiva el tensor en una parte esfrica y una parte


desviadora:
= esf + dev

(2.190)

donde la parte esfrica viene dada por:

esf

Tr ( )
6 10 3
=
1=
1 = 2 10 31
3
3

ijesf

2 0 0
= 0 2 0 10 3
0 0 2

(2.191)

La parte desviadora viene dada por:


ijdev

4 0 0
2 0 0
2 0 0

3
3
= 0 4 1 10 0 2 0 10 = 0 6 1 10 3
0 1 6
0 0 2
0 1 4

(2.192)

c) Los invariantes principales del tensor son:

Universidad de Castilla- La Mancha


Ciudad Real - Espaa

Draft

Por: Eduardo W. V. Chaves (2014)

MECNICA DEL MEDIO CONTINUO: PROBLEMAS RESUELTOS

252

I = Tr ( ) = 6 10 3
4 1 4 0 4 0
10 6 = 17 10 6
+
+
II =

1
6
0
6
0

(2.193)

III = (4 (4) 6 4 ) 10 9 = 100 10 9

d) Teniendo en cuenta las componentes del tensor de deformacin:


4 0 0
ij = 0 4 1 10 3
0 1 6

(2.194)

Ya verificamos que 1 = 4 10 3 es un autovalor y la direccin [ 1,0,0] es el autovector


asociado a 1 . Para encontrar los dems autovalores hay que obtener la solucin del
determinante caracterstico:
4

=0

(2.195)

(4 )(6 ) 1 = 0
2 2 25 = 0
=

2
1 = 6,0990
b b 2 4ac 2 (2) 4 1 (25) 2 4 + 4 25
=
=
= 1 26
2a
2 1
2
2 = 4,099

Luego:
1 = 4 10 3 ;

2 = 6,0990 10 3 ;

3 = 4,099 10 3

(2.196)

Reestructurando las deformaciones:


I = 6,0990 10 3 ;

II = 4 10 3 ;

III = 4,099 10 3

(2.197)

Ejemplo 2.73
Encontrar el tensor de deformacin infinitesimal y el tensor de rotacin infinitesimal para
el siguiente campo de desplazamiento:
u 1 = x12

u 2 = x1 x 2

u3 = 0

Solucin:
Tensor de deformacin Infinitesimal
En el rgimen de pequeas deformaciones, el tensor de deformacin viene dado por:
E ijL eijL ij =

1 u i u j
+
2 x j
x i

Tenemos que hallar el gradiente del desplazamiento:


u 1

x1
u j u 2
=
x k x1

u 3
x1

Universidad de Castilla- La Mancha


Ciudad Real - Espaa

u1
x 2
u 2
x 2
u 3
x 2

Draft

u1

x 3 2 x
1
u 2
x
=
2
x 3
0
u 3
x 3

0
x1
0

0
0
0

Por: Eduardo W. V. Chaves (2014)

2 CINEMTICA DEL CONTINUO

253

Con eso podemos obtener:


E ijL

eijL

ij

u j
1 u
= i +
2 x j
x i

2 x

1 1
= x2
2
0

0 2 x1
0 + 0
0 0

0
x1
0

x2
x1
0

x2
2

0 2 x1

0 = x 2

0 2
0

x1
0

Tensor de rotacin infinitesimal:


2 x1
1 ui u j 1

= x2
ij =
2 x j xi 2
0

0
x1
0

0 2 x1
0 0
0 0

x2
x1
0

0
0

x
0 = 2
2
0 0

x2
2
0
0

Ejemplo 2.74
En la Figura 2.23 se muestra la transformacin que experimenta el cuadrado ABCD de
lado unitario.
X 2 , x2
x2

D
C

D
1

45

A = A

X 1 , x1

B
x1

Figura 2.23: Cuerpo sometido a una rotacin.


Se pide:
a) Plantear las ecuaciones del movimiento;
b) Es vlida la teora de pequeas deformaciones? Probar y Justificar;
c) Es vlida la teora de deformacin finita (grandes deformaciones)? Probar.
Solucin:
La ley de transformacin entre los sistemas x x viene dada por:

x1
x1 cos sin 0 x1

= 45
x 2 = sin cos 0 x 2 x 2 =
x
x 0
0
1 x3
3
3

Universidad de Castilla- La Mancha


Ciudad Real - Espaa

Draft

2
2
2
2
0

2
2
2
2
0

0
x1

0 x 2

1 x 3

Por: Eduardo W. V. Chaves (2014)

MECNICA DEL MEDIO CONTINUO: PROBLEMAS RESUELTOS

254

Si consideramos los sistemas materiales y espaciales superpuestos, las ecuaciones de


movimiento quedan definidas por la inversa de la expresin anterior, es decir, x x :
2

x1 2
2

x 2 =
x 2
0
3

2
2
2
2
0

0
X 1

0 X 2

1 X 3

2
2
X1 +
X2
x1 =
2
2

2
2
X1 +
X2
x2 =
2
2

x3 = 0

Por ejemplo, el punto C en la configuracin de referencia tiene coordenadas materiales


X 1C = 1 , X 2C = 1 . Tras el movimiento: x1C =

2
2
2
2
(1) +
(1) +
(1) = 2 , x 2C =
(1) = 0
2
2
2
2

Campo de desplazamientos:

u1 = x1 X 1 =

u 2 = x 2 X 2 =

2
2
2
2
X1
X 2 X 1 = X 1
X2
1
2
2
2
2

2
2
2
2
X1 +
X2 X2 =
X 1 + X 2
1
2
2
2

Gradiente material de los desplazamientos:


u1

X 1
u i u 2
=
X j X 1

u 3
X 1

u1
X 2
u 2
X 2
u 3
X 2

u1 2
1

X 3 2
u 2 2
=
X 3 2

u 3
X 3 0

1 0
2

0
0

2
2

El tensor de deformacin infinitesimal viene definido como = symu =

r
r
1
(u) + (u)T ,
2

con o obtenemos:
2
1

ij = 0
0

1 0 0 ij
2
0
0

Como para un movimiento de slido rgido el tensor de deformacin tiene que ser cero, es
decir, = 0 (tensor de deformacin infinitesimal), E = 0 (tensor de deformacin de
Green-Lagrange), e = 0 (tensor de deformacin de Almansi). Calculando las componentes
del tensor de deformacin de Green-Lagrange:
u j u k u k
1 u
+
E ij = i +
2 X j X i X i X j

Universidad de Castilla- La Mancha


Ciudad Real - Espaa

Draft

0 0 0

= 0 0 0


0 0 0

Por: Eduardo W. V. Chaves (2014)

2 CINEMTICA DEL CONTINUO

255

Ejemplo 2.75
Un rectngulo de base y altura b se gira en sentido antihorario 30 . Tras el giro el
rectngulo sufre una deformacin de tal forma que la base mantiene su longitud inicial y la
altura se dobla. Calcular el gradiente de deformacin, el tensor derecho de deformacin de
Cauchy-Green, y el tensor de deformacin de Green-Lagrange.
Solucin:
X 2 , x2
C

x2
D

2b

30

x1

30

A = A

X 1 , x1

Figura 2.24: Cuerpo sometido rotacin/deformacin.


Fijaros que podemos hacer la descomposicin del movimiento por: primero una
deformacin y a continuacin una rotacin:
El movimiento de deformacin
viene gobernado por el tensor
derecho de estiramiento de la
descomposicin polar:

X 2 , x2

2b

1 0 0
U ij = 0 2 0
0 0 1

B B
A = A

X 1 , x1

donde hemos aplicado la definicin


del estiramiento. Notar que son los
propios estiramientos principales. A
continuacin
aplicamos
una
rotacin, donde las componentes
del tensor R son las mismas que la
matriz de transformacin del
r
r
sistema x al sistema x :

cos sin 0
R ij = sin cos 0
0
0
1
Universidad de Castilla- La Mancha
Ciudad Real - Espaa

Draft

Por: Eduardo W. V. Chaves (2014)

MECNICA DEL MEDIO CONTINUO: PROBLEMAS RESUELTOS

256

Luego, aplicando la descomposicin polar por la derecha F = R U :


cos sin 0 1 0 0 cos 2 sin 0
Fij = R ik U kj = sin cos 0 0 2 0 = sin 2 cos 0
0
0
1 0 0 1 0
0
1

Para el problema propuesto, tenemos que:


cos 30 2 sin 30 0
Fij = sin 30 2 cos 30 0
0
1
0
r

Como se trata de un caso de deformacin homognea se cumple x = F X + c , en este


r r
caso con c = 0 . Por ejemplo, para una partcula que en la configuracin de referencia
ocupaba el punto D , en la configuracin actual estar segn:
x1D cos 30 2 sin 30 0 X 1D cos 30 2 sin 30 0 0 2b sin 30
D


D
x 2 = sin 30 2 cos 30 0 X 2 = sin 30 2 cos 30 0 b = 2b cos 30

x D 0
0
1 X 3D 0
0
1 0
0

hecho que se puede comprobar fcilmente a travs de la Figura 2.24.


A travs de la definicin del tensor derecho de deformacin de Cauchy-Green, C = F T F ,
podemos obtener las componentes cartesianas:
cos sin 0 cos sin 0 1 0 0
C ij = Fki Fkj = sin cos 0 sin cos 0 = 0 4 0
0
0
1 0
0
1 0 0 1

1
2

El tensor de deformacin de Green-Lagrange, E = (C + 1) , y sus componentes


cartesianas quedan:
1 0 0 1 0 0 0 0 0



1
E ij = 0 4 0 0 1 0 = 0 1,5 0
2
0 0 1 0 0 1 0 0 0

Observemos que el espacio original coincide con el espacio principal de deformacin.


Tambin podamos haber obtenido las componentes de C y E a travs de sus
3

1
a =1 2

(a) N
(a) , E =
representaciones espectrales: C = 2a N
(2a 1)N (a ) N ( a) , donde
a =1

a son los estiramientos principales.

Universidad de Castilla- La Mancha


Ciudad Real - Espaa

Draft

Por: Eduardo W. V. Chaves (2014)

3 Tensiones
3.1 Fuerza, Tensor de Tensiones, Vector Tensin
Ejemplo 3.1
Ignorando la curvatura de la superficie de la tierra, el campo gravitacional puede suponerse
de la forma como se muestra en la Figura 3.1, donde g es la aceleracin de la gravedad.
Obtener la fuerza resultante que acta en el cuerpo B .

x3

x2

x1

Figura 3.1: Campo gravitacional.


Solucin:
Todos los cuerpos situados en este campo se encontrarn sometidos a la fuerza:
0
r
b i ( x , t ) = 0
g

La fuerza msica que acta sobre el cuerpo es: Fi =


V

r
0

b i ( x , t ) dV =

g dV

kg m
kg m
dV = 2 = N ( Newton )
3 2
s
m s

Podemos verificar la unidad de F : [F] =


V

Universidad de Castilla- La Mancha


Ciudad Real - Espaa

Draft

Por: Eduardo W. V. Chaves (2014)

MECNICA DEL MEDIO CONTINUO: PROBLEMAS RESUELTOS

258

Ejemplo 3.2
Las componentes del tensor de tensiones en el punto P son:
8 4 1
ij = 4 3 0,5 Pa
1 0,5 2

x3

C (0,0,5)

a) Calcular el vector tensin en el punto P


segn la direccin del plano ABC ,
como se indica en la Figura 3.2.

b) Obtener el vector tensin normal ( N )

B (0,2,0)

r
y el vector tensin tangencial ( S ),

ver Apndice A en Chaves(2007, 3 edicin).

x2

A(3,0,0)

x1

Figura 3.2: Plano ABC .


Solucin:
En primer lugar, deberemos obtener la direccin normal a este plano, para ello escogemos
dos vectores pertenecientes al plano y hacemos el producto vectorial entre ellos:

BA = OA OB = 3e 1 2e 2 + 0e 3

BC = OC OB = 0e 1 2e 2 + 5e 3

El vector normal al plano ABC viene dado a travs del producto vectorial de los vectores
definidos anteriormente:
e 1 e 2 e 3

r
n = BC BA = 0 2 5 = 10e 1 + 15e 2 + 6e 3
3 2 0
r
r
n 10
15
6

El versor asociado a n ser: n = r = e 1 + e 2 + e 3


19
19
n 19

Utilizando la ecuacin t i(n) = ij n j , podemos obtener las componentes del vector tensin
de la forma:
t1
8 4 1 10
t1
26
1
t = 1 4 3 0,5 15 Pa

t 2 = 8 Pa
2 19

19
t 3
1 0,5 2 6
t 3
29,5
r
b) El vector tensin t (n) asociado a la direccin n puede descomponerse en una componente
r
r
normal N y en otra tangencial S tal como se indica en la Figura 3.3. La suma vectorial

de estos vectores resulta:


donde N

r
r
r
r
t (n ) = N + S

t (n) = N n + S s
r
r
y S son los mdulos de N y de S , respectivamente.

Como visto en el Apndice A, N puede ser obtenido a travs de las siguientes relaciones

Universidad de Castilla- La Mancha


Ciudad Real - Espaa

Draft

Por: Eduardo W. V. Chaves (2014)

3 TENSIONES

259

r
N = t (n) n = ( n ) n = n n = : (n n )

= t i(n) n i = ( ij n j )n i = n i ij n j = ij (n i n j )
N

= t i n i

10
1
= 2 [26 8 29,5] 15 1,54 Pa
19
6

r
t (n)

x3

r
S

r
N
s

e 3
e 2

e 1

x2

x1

Figura 3.3: Componentes normal y tangencial del vector tensin.


La componente tangencial viene dada por:
r

S = t (n) s = ( n ) s = s n = : (n n ) = t i(n) s i = ( ij n j )s i = s i ij n j = ij (s i n j )

La componente tangencial tambin puede ser obtenida a travs del teorema de Pitgoras:
r
t (n)

= 2N + 2S

2S = t i(n) t i(n) 2N

donde

t i(n) t i(n)

26
1
= 2 [26 8 29,5] 8 4,46
19
29,5

Resultando que:

S = t i(n) t i(n) 2N = 4,46 2,3716 2,0884 Pa

Ejemplo 3.3
El estado tensional en un punto del continuo viene representado a travs de las
componentes del tensor de tensiones de Cauchy como:
2 1 0
ij = 1 2 0 Pa
0 0 2

Universidad de Castilla- La Mancha


Ciudad Real - Espaa

Draft

Por: Eduardo W. V. Chaves (2014)

MECNICA DEL MEDIO CONTINUO: PROBLEMAS RESUELTOS

260

a) Obtener las componentes de en un nuevo sistema x1 , x 2 , x3 , donde la matriz de


transformacin viene dada por a ij , ver Figura 3.4.
b) Obtener los invariantes principales de ;
c) Obtener los autovalores y autovectores de . Verificar tambin si los autovectores
forman una matriz de transformacin de base entre el sistema original y el principal;
d) Obtener la representacin grfica del tensor de tensiones de Cauchy, i.e. el crculo de
Mohr en tensiones (ver Apndice A en Chaves (2007));
e) Obtener la parte esfrica ( esf ) y la parte desviadora ( dev ) del tensor . Tambin,
obtener los invariantes principales de dev y los autovalores de dev ;
f) Obtener la tensin normal octadrica ( oct
N ) y la tensin tangencial octadrico (o tambin conocida
oct
como tensin de corte octadrica) ( S ).
x3

x 2

3 0 4
1
a ij = A = 0 5 0
5
4 0 3

x3

x1

donde

e 2

e 3

1
e

e 3
e 1

e 2

a11 = cos 1
x2

a12 = cos 1
a13 = cos 1
M

x1

Figura 3.4: Matriz de transformacin


Solucin:
a) Como hemos visto en el captulo 1 (Chaves (2007)), la ley de transformacin de las
componentes del tensor de segundo orden viene dada por:
ij = aik a jl kl

11

12
13

12
22
23

Forma
Matricial

= A AT
T


13
2 0,6 0
3 0 4 1 1 0 3 0 4
1

23 = 2 0 5 0 2 2 0 0 5 0 = 0,6 2 0,8
5
0 0,8 2
4 0 3 0 0 2 4 0 3
33

donde estas componentes se pueden apreciar en la Figura 3.5.


b) Los invariantes principales del tensor de tensiones de Cauchy stress tensor son obtenido
a travs de las expresiones:
I = Tr ( ) = ii = 11 + 22 + 33

] (

1
1
2
2
13
223
( Tr ) 2 Tr ( 2 ) = ii jj ij ij = 11 22 + 11 33 + 33 22 12
2
2
1
III = det ( ) = ijk i1 j 2 k 3 = ii jj kk 3 ii jk jk + 2 ij jk ki
6
2
2
= 11 22 33 + 212 23 13 11 223 22 13
33 12
II =

Universidad de Castilla- La Mancha


Ciudad Real - Espaa

Draft

Por: Eduardo W. V. Chaves (2014)

3 TENSIONES

261

Reemplazando los valores del problema propuesto obtenemos que:


I = 6

II =

2 0
0 2

2 0

0 2

2 1
1 2

= 11

III = 6

c) Los valores principales (autovalores) ( i ) y direcciones principales ( n (i ) ) son obtenidos a


travs del siguiente sistema de ecuaciones:
1
0 n1 0
2
1
2
0 n 2 = 0

0
0
2 n 3 0

Para obtener las soluciones no triviales de n (i ) , tenemos que resolver el siguiente


determinante caracterstico:
ij ij

2
1
= 1
2
0

0
0

=0

Pero, si nos fijamos en las componentes del tensor de tensiones de Cauchy, verificamos
que ya conocemos un autovalor y autovector ya que las componentes tangenciales segn
direccin x3 son iguales a cero, luego:
1 = 2 Direccin
principal
n1(1) = n (21) = 0 , n3(1) = 1

Para obtener los autovalores restantes, es suficiente resolver el determinante:


2

= (2 ) 1 = 0
2

2 = 1

3 = 3

Expresando as las componentes del tensor de tensiones de Cauchy en el espacio principal:


2 0 0
ij = 0 1 0 Pa
0 0 3

Direccin principal asociada al autovalor 2 = 1 :


1
0 n1( 2 ) 0
2 1
( 2)
( 2)
( 2 ) n1 + n 2 = 0

1
2 1
0 n 2 = 0 ( 2 )
n1( 2 ) = n (22 )

( 2)

n
+
n
=
0
2
0
0
2 1 n 3( 2 ) 0 1
2

Con n 3( 2 ) = 0 y utilizando la restriccin n1( 2 ) + n (22 ) = 1 obtenemos que:


n1( 2 ) = n (22 ) =

1
2

, luego n i( 2 ) =

Direccin principal asociada al autovalor 2 = 3 :


1
0 n1( 3) 0
2 3
( 3)
( 3)
1
n ( 3) = 0 n1 + n 2 = 0 n ( 3) = n ( 3)

2
3
0

1
2

2 n ( 3) n ( 3) = 0
2
0
0
2 3 n 3( 3) 0 1
2

Con n 3(3) = 0 y utilizando la restriccin n1(3) + n (23) = 1 obtenemos que:


Universidad de Castilla- La Mancha
Ciudad Real - Espaa

Draft

Por: Eduardo W. V. Chaves (2014)

MECNICA DEL MEDIO CONTINUO: PROBLEMAS RESUELTOS

262

n1( 3) = n (23) =

luego n i(3) =

1
2

Como hemos visto en el captulo 1, los autovectores del tensor constituye una matriz de
transformacin, B , del espacio original al espacio principal, i.e. = B B T . Luego, hay
que cumplir que:

0
0
1 = 2
0
2 = 1
0 =

0
0
3 = 3

2
1

2
1

x3
x3

0
1

2 1 0
1
0 1 2 0

2
0 0 2 1

0
1
2
1
2

x2
x2

P
x1

x1

= A AT

x3
33
13
13
11

x3

23

33

23

23
12

22
12

23

13
x2

13

22

x2

12

12

11
x1
x1

= AT A
Figura 3.5: Ley de transformacin de base.

Universidad de Castilla- La Mancha


Ciudad Real - Espaa

Draft

Por: Eduardo W. V. Chaves (2014)

3 TENSIONES

263

d) La representacin grfica de un tensor de segundo orden simtrico, i.e. el crculo de


Mohr, puede ser obtenido tal y como se describe en el Apndice A en Chaves (2007). Para
ello, debemos reestructurar los autovalores de tal forma que I > II > III , resultando:
I = 3

II = 2

III = 1

Las tres circunferencias son definidas por:


Crculo 1

Crculo 2

Crculo 3

1
( II + III ) = 1,5
2
1
(centro)C 2 = ( I + III ) = 2,0
2
1
(centro)C 3 = ( I + II ) = 2,5
2

(centro)C1 =

;
;

1
( II III ) = 0,5
2
1
(radio) R2 = ( I III ) = 1,0
2
1
(radio) R3 = ( I II ) = 0,5
2
(radio) R1 =

Entonces, el crculo de Mohr en tensiones viene representado en la Figura 3.6.


S

S max = 1
R2
R1
C3
III = 1

C1

R3

II = 2

I = 3 = N max

Figura 3.6: Crculo de Mohr en tensiones.


e) Como definido en al captulo 1, un tensor de segundo orden puede ser descompuesto de
forma aditiva en una parte esfrica y otra desviadora:
Notacin Tensorial

Notacin Indicial

= esf + dev

1
ij = ijesf + ijdev = kk ij + ijdev
3
dev
= m ij + ij

= m 1 + dev

(3.1)

La representacin esquemtica de estas componentes se puede apreciar en la Figura 3.7. El


valor de m viene dado por:
m =

11 + 22 + 33 1 + 2 + 3 1
I
1
6
=
= kk = Tr ( ) = = = 2
3
3
3
3
3 3

Universidad de Castilla- La Mancha


Ciudad Real - Espaa

Draft

Por: Eduardo W. V. Chaves (2014)

MECNICA DEL MEDIO CONTINUO: PROBLEMAS RESUELTOS

264

x3
33
23

13
13

23
12

11

22
12

x2

14414444442444444443
x3

x3

dev
33

23

13
13

m
x2

23
12

dev
22
12

dev
11

x1

x2

x1

esf

dev

Figura 3.7: Parte esfrica y desviadora de .


Luego, la parte esfrica queda definida por:
ijesf

2 0 0
= m ij = 2 ij = 0 2 0
0 0 2

Y, la parte desviadora por:


ijdev

11 12 13 m 0
= 12 22 23 0 m
13 23 33 0
0
13 (211 22 33 )

1
(2 22
=
12
3

13

0
0
m
12
11 33 )
23

23

11 22 )
13

1
3

(2 33

Luego,
ijdev

Universidad de Castilla- La Mancha


Ciudad Real - Espaa

1
0 0 1 0
2 2

= 1
22
0 = 1 0 0
0
0
2 2 0 0 0

Draft

Por: Eduardo W. V. Chaves (2014)

3 TENSIONES

265

Los tensores y dev son coaxiales (ver captulo 1 en Chaves (2007), i.e., presentan las
mismas direcciones principales. Luego, podemos obtener los autovalores de dev
fcilmente si operamos en el espacio principal de :
ijdev

1
= 0
0

0
2
0

m
0

3 0
0
0

0 0 0 0
0 = 0 1 0
m 0 0 1

0
m
0

Los invariantes de dev vienen dados por:


I dev = Tr ( dev ) = 0

II dev = 1

III dev = 0

Tradicionalmente, los invariantes del tensor de tensiones desviador vienen denotados por:
J1 = I dev = 0
J 2 = II
J 3 = III

dev

dev

1 2
I 3 II
3
1
=
2 I 3 9 I II + 27 III
27
=

f) Las tensiones normal y tangencial octadricas vienen dadas por:


oct
N =

oct
S oct =

1
(1 + 2 + 3 ) = 1 ii = I = m
3
3
3

1
2
2 I 2 6 II =
J2 =
3
3

( ) + ( ) + ( )
dev 2
1

dev 2
2

dev 2
3

Reemplazando los valores del problema propuesto obtenemos que:


oct
N = m = 6

oct =

2
2
J2 =
3
3

Ejemplo 3.4
Las componentes del tensor de tensiones en un punto P son:
1 2 3
ij = 2 4 6 MPa
3 6 1

Obtener:

(3.2)

a) El vector traccin t en P para un plano normal al eje x1 ;


r

b) El vector traccin t en P para un plano cuyo vector normal es (1,1,2) ;


r

c) El vector traccin t en P para un plano paralelo al plano 2 x1 2 x 2 x3 = 0 ;


d) Las tensiones principales en P ;
e) Las direcciones principales de en P .

Universidad de Castilla- La Mancha


Ciudad Real - Espaa

Draft

Por: Eduardo W. V. Chaves (2014)

MECNICA DEL MEDIO CONTINUO: PROBLEMAS RESUELTOS

266

Solucin:
a) El vector normal al plano es (1,0,0) . Luego el vector traccin viene dado por:
1 2 3 1 1
= 2 4 6 0 = 2
3 6 1 0 3

t i(n)

(3.3)

b) El vector unitario (versor) asociado a la direccin (1,1,2) es:


1
1
1
n i =
6
2

(3.4)

luego,

t i(n)

1 2 3 1
5
1
1

=
2 4 6 1 =
10
6
6
3 6 1 2
1

(3.5)

c)
2
1 2 3 2
5
1
1
1
)

(n
n i = 2 t i = 2 4 6 2 = 10
3
3
3
1
3 6 1 1
7

(3.6)

d) Resolviendo el determinante caracterstico


1
2
2
4
3

3
6

=0

(3.7)

obtenemos que:
1 = 10 ; 2 = 0 ; 3 = 4

(3.8)

e) Las tensiones principales correspondientes son:


Para 1 = 10
3
9n1 + 2n 2 + 3n 3 = 0


(1)
2n1 6n 2 + 6n 3 = 0 n i = 6
3n + 6n 9n = 0
5
2
3
1

(3.9)

Anlogamente:
n i( 2 )

2
1

( 3)
= 1 ; n i = 2
0
3

(3.10)

Normalizacin de las direcciones principales:


3
n (1)
n i(1) = r i = 1 6
70
n (1)
5

Universidad de Castilla- La Mancha


Ciudad Real - Espaa

2
n ( 2)
n i( 2 ) = r i = 1 1
5
n ( 2)
0

Draft

1
n ( 3)
n i(3) = r i = 1 2
14
n ( 3)
3

Por: Eduardo W. V. Chaves (2014)

3 TENSIONES

267

Ejemplo 3.5
r

Probar que S = t (n) (1 n n ) , donde t (n) es el vector traccin asociado al plano cuya

normal es n y S es la tensin tangencial asociada a este plano.


Solucin 1:

r
r
r
r
r
r
S = t (n) t (n) n n = t (n) t (n) n n = t (n) (1 n n )

Solucin 2:
Podemos resolver el problema anterior utilizando slo las componentes de la ecuacin
r
r
S = t (n) [ : (n n )]n :

S i = t i(n) (n k n l kl ) n i = t i(n ) n i n k t (kn) = t (kn )

n i n k t (kn) = t (kn) ( ik n i n k )

ik

o en forma compacta:
r
r
S = t (n) (1 n n )

Ejemplo 3.6
El estado de tensin en un punto P del medio continuo se da esquemticamente por:
x3

1
22

4
1

x2

x1

Determinar el valor de la componente 22 del tensor de tensiones para que exista al menos
un plano que pase por P que est libre de tensiones;
Determinar la direccin de dicho plano.
Solucin:
r

Buscamos un plano cuya direccin es n tal que t (n ) = 0 . Podemos relacionar el tensor de


tensiones con el vector tensin segn expresin:
r
t (n) = n

luego:
t1(n ) 0 1
(n )
t 2 = 1 22
t (n ) 4 1
3

4 n1 0

1 n 2 = 0
0 n3 0

Resultando en el siguiente sistema de ecuaciones:


Universidad de Castilla- La Mancha
Ciudad Real - Espaa

Draft

Por: Eduardo W. V. Chaves (2014)

MECNICA DEL MEDIO CONTINUO: PROBLEMAS RESUELTOS

268

n2 + 4n3 = 0 n3 = 4 n 2

n1 + 22n 2 + n3 = 0

1
4n1 + n2 = 0 n1 = n 2
4

Combinando las ecuaciones anteriores obtenemos que:


n1 + 22 n 2 + n 3 = 0

1
1
n 2 + 22 n 2 n 2 = 0
4
4

22 n 2 = 0
2

r r
1
1
Luego, para n 0 , tenemos que: 22 = 0 22 = .

Para determinar la direccin del plano partimos de la restriccin: n i n i = 1 , luego:


n i n i = 1 n12 + n 22 + n 32 = 1
2

1
1
n 2 + n 22 + n 2 = 1
4
4

n2 =

2 2
3

n1 = n 3 =
r

2
6

Obteniendo as la direccin de la normal al plano, cuando se cumple t (n ) = 0 :


1
2
4
n i =
6
1

3.2 Ecuacin de
Principales

Equilibro,

Tensiones

Direcciones

Ejemplo 3.7
El campo de tensin de un medio continuo viene representado por:
1
ij = 0
2 x2

0
1
4 x1

2 x2
4 x1
1

unidades de tensin

(3.11)

donde xi son las coordenadas cartesianas.


a) Despreciando las fuerzas msicas, est el cuerpo en equilibrio?
b) Determinar el vector tensin que acta en un punto ( x1 = 1, x2 = 2, x3 = 3) segn el
plano x1 + x 2 + x3 = 6 ;
c) Determinar la proyeccin del vector tensin segn la direccin normal y tangencial al
plano x1 + x 2 + x3 = 6 ;
Solucin:
Ecuacin de equilibrio:

r r
xr + {
b = 0

r
=0

ij , j = 0 i

(3.12)

expandiendo,

Universidad de Castilla- La Mancha


Ciudad Real - Espaa

Draft

Por: Eduardo W. V. Chaves (2014)

3 TENSIONES

i1,1 + i 2, 2 + i 3,3 = 0 i

269

11,1 + 12, 2 + 13,3 = 0

21,1 + 22, 2 + 23,3 = 0

31,1 + 32, 2 + 33,3 = 0

(3.13)

ya que:
11
=0
x1
21
=0
x 2
31
=0
x3

12
=0
x2
22
=0
x2
32
=0
x2

13
=0
x3
23
=0
x3
33
=0
x3

(3.14)

b) El versor normal al plano x1 + x 2 + x3 = 6 es:

1
1
1
n i =
3
1

(3.15)

r
t (n) = n

(3.16)

1 0 4
ij ( x1 = 1, x 2 = 2, x3 = 3) = 0 1 4
4 4 1

(3.17)

El vector tensin t (n) :

luego,

t i(n)

1 0 4
1
5
1 1

= 0 1 4
1 =
5
3
3
4 4 1
1
9

(3.18)

c) Componente normal
1
r (n )
1
1
N = t n =
[5 5 9] 1 = 1 (5 + 5 + 9) = 19
3
3
3
3
1

(3.19)

Componente tangencial
r r
2S = 2N + t (n) t (n)

(3.20)

5
r (n ) r (n )
1
t t =
[5 5 9] 5 1 = 131
3
3
3
9

(3.21)

luego
2

32
19 131
2S = +
=
3
9
3

(3.22)

Ejemplo 3.8

Universidad de Castilla- La Mancha


Ciudad Real - Espaa

Draft

Por: Eduardo W. V. Chaves (2014)

MECNICA DEL MEDIO CONTINUO: PROBLEMAS RESUELTOS

270

Dado un cuerpo en equilibrio esttico, donde el campo del tensor de tensiones de Cauchy
viene representado a travs de las siguientes componentes cartesianas:
11 = 6 x13 + x 22

; 12 = x 32

22 = 12 x13 + 60
33 = 18 x 23 + 6 x33

; 23 = x 2
; 31 = x12

Determinar el vector de fuerzas msicas (por unidad de volumen) en el punto


( x1 = 2; x 2 = 4; x3 = 2 ).
Solucin:
Ecuacin de equilibrio:
r r
xr + b = 0

(3.23)

11 12 13

+
+
+ b1 = 0 b1 = 11 12 13

x 2
x3
x1
x 2
x3
x1

21 22
+
+ 23 + b 2 = 0 b 2 = 21 22 23

x 2
x3
x1
x 2
x3
x1
31 32 33
31 32 33
x + x + x + b 3 = 0 b 3 = x x x
2
3
1
2
3
1

(3.24)

b1 = 18 x12 0 0

b 2 = 0 0 0
b = 2 x 1 18 x 2
1
2
3

18 x12

0
bi =

2 x 1 18 x 2
1
2

(3.25)

Para el punto x1 = 2; x 2 = 4; x3 = 2 obtenemos que:


72
b i = 0
77

(Fuerza por unidad de volumen)

(3.26)

Ejemplo 3.9
El campo del tensor de tensiones de Cauchy viene representado por sus componentes
como:
x12 x 2

ij = k (a 2 x 22 ) x1

donde k y a son constantes.

1 3
( x 2 3a 2 x 2 )
0
3

0
2ax32
(a 2 x 22 ) x1

(3.27)

Encontrar el campo de fuerzas msicas b (por unidad de masa) necesario para que el
campo de tensin est en equilibrio.
Solucin:

Universidad de Castilla- La Mancha


Ciudad Real - Espaa

Draft

Por: Eduardo W. V. Chaves (2014)

3 TENSIONES

271

11 12 13
+
+
+ b1 = 0 b1 = 2 x1 x 2 k + 2 x1 x 2 k = 0

x
x
x

1
2
3

k
21 22 23
+
+
+ b 2 = 0 b 2 = k (a 2 x 22 ) (3 x 22 3a 2 ) = 0

3
x 2
x 3
x1
31 32 33
x + x + x + b 3 = 0 b 3 = 4kax3
2
3
1

(3.28)

Luego:
0
4kax3
0
bi =

1

(Fuerza por unidad de masa)

Ejemplo 3.10

(3.29)

Suponga que las fuerzas msicas son b = ge 3 , donde g es una constante. Considere el
siguiente tensor de tensiones:
x2
ij = x3
0

x3
0

0
x 2
p

x2

(3.30)

Encontrar p tal que cumpla con las ecuaciones de equilibrio. Considerar una constante
y el campo homogneo de densidad de masa, es decir, no depende del vector posicin.
Solucin:
Ecuacin de equilibrio:
r r
xr + b = 0

11 12 13
+
+
+ b1 = 0

x
x
x

1
2
3

21 22 23
+
+
+ b2 = 0

x 2
x 3
x1
31 32 33
x + x + x + b 3 = 0
2
3
1

33 (p )
p
= b3
=
=
x 3
x3
x3

(3.31)

0 + 0 + 0 + b = 0 b = 0
1
1

0 + 0 + 0 + b 2 = 0 b 2 = 0

0 + 33 + b 3 = 0
x3

g
p
=1 +

x3

dp = 1 +
dx
3

(3.32)

(3.33)

g
g

p = 1 +
x
x 3 p = 1 +

Verificacin:

+ 1 +
g = + + g g = 0

Universidad de Castilla- La Mancha


Ciudad Real - Espaa

Draft

(3.34)

Por: Eduardo W. V. Chaves (2014)

MECNICA DEL MEDIO CONTINUO: PROBLEMAS RESUELTOS

272

Ejemplo 3.11
Muestre que para el siguiente campo de tensin:
11 = x 22 + ( x12 x 22 ) ; 12 = 2x1 x 2
;
2
2
2
2
2
22 = x1 + ( x 2 x1 ) ; 33 = ( x1 + x 2 )

23 = 13 = 0

Satisface las ecuaciones de equilibrio con fuerzas msicas iguales a cero.


Solucin:
Ecuaciones de equilibrio:
ij , j + b i = 0 i
{
=0i

ij , j = 0 i

(i , j = 1,2,3)

i1,1 + i 2, 2 + i 3,3 = 0 i

11 12 13
+
+
=0

x
x
x

1
2
3


22 23
+
=0
21 +
x 2
x 3
x1
31 32 33
=0
+
+

x 3
x 2
x1

i = 1 11,1 + 12 , 2 + 13,3 = 0

i = 2 21,1 + 22 , 2 + 23, 3 = 0
i = 3 31,1 + 32 , 2 + 33,3 = 0

Las ecuaciones de equilibrio quedan:


11,1 + 12 , 2 + 31,3 = 2 x1 2x1 = 0

12 ,1 + 22 , 2 + 23, 3 = 2 x 2 + 2x 2 = 0

13,1 + 23, 2 + 33,3 = 0

Con lo cual se comprueba que el cuerpo est en equilibrio.


Ejemplo 3.12
Considrese el siguiente campo de tensiones:
x1 + x 2
r
ij ( x ) = 12
0

12
x1 2 x 2
0

0
0
x 2

Considerando el medio en equilibrio, encontrar 12 , sabiendo que es funcin de x1 , x 2 , i.e.


12 ( x1 , x 2 ) . Se sabe tambin que el medio est libre de fuerzas msicas y que el vector
r
tensin en el plano x1 = 1 viene dado por: t (n) = (1 + x 2 )e 1 + (5 x 2 )e 2 .
Solucin:
Como el cuerpo est en equilibrio debe satisfacer las ecuaciones de equilibrio:
ij , j + b i = 0 i
{

ij , j = 0 i

(i, j = 1,2,3)

=0 i

i1,1 + i 2 , 2 + i 3,3 = 0 i

Resultando:
Universidad de Castilla- La Mancha
Ciudad Real - Espaa

Draft

Por: Eduardo W. V. Chaves (2014)

3 TENSIONES

273

11 12 13

+
+
= 1 + 12 + 0 = 0

x 2
x 3
x 2
x1

22 23 12
21 +
+
=
2+0=0
x 2
x 3
x1
x1
31 32 33
=0+0+0=0
+
+

x 3
x 2
x1
r
Como dato del problema tenemos que cuando x1 = 1 , t (n) = (1 + x 2 )e 1 + (5 x 2 )e 2 , luego:

12
1 + x 2

ij ( x1 = 1, x 2 ) = 12 1 2 x 2
0
0
t

(n )

12
1 + x 2

= ij ( x1 = 1, x 2 )n j = 12 1 2 x 2
0
0

0
0
x 2

0 1 1 + x 2
0 0 = 5 x 2
x 2 0 0

(3.35)

t (n) = ij ( x1 = 1, x 2 ) n j

1 + x2
12 ( x1 = 1, x 2 ) 0 1
1 + x2
1 + x 2

12 ( x1 = 1, x 2 )
1 2 x2
0 0 = 12 ( x1 = 1, x 2 ) = 5 x 2
0

x 2 0
0
0
0

A travs de las ecuaciones de equilibrio:


12
=2
x1

12

= 2x1

12 ( x1 , x 2 ) = 2 x1 + C ( x 2 )

A travs de la condicin de contorno dada por (3.35) podemos obtener la constante de


integracin:
12 ( x1 = 1, x 2 ) = 5 x 2 = 2 + C ( x 2 )

C ( x2 ) = 3 x2

Luego:
12 ( x1 , x 2 ) = 2 x1 x 2 + 3

Ejemplo 3.13
Obtener las ecuaciones de equilibrio (en notacin ingenieril), partiendo de un elemento
r
diferencial ( dx ), donde la variacin de las tensiones de punto a punto (campo de tensiones)
es la que se muestra en la Figura 3.8.
Solucin:
Para obtener las ecuaciones de equilibro partiremos de que la suma de las fuerzas que
actan en el diferencial sea cero. Haciendo el equilibrio de fuerzas segn direccin x :

=0

xy

x
dx dydz x dydz + xy +
dy dxdz
x
y

xy dxdz + xz + xz dz dxdy xz dxdy = 0


z

b x dxdydz + x +

Universidad de Castilla- La Mancha


Ciudad Real - Espaa

Draft

Por: Eduardo W. V. Chaves (2014)

MECNICA DEL MEDIO CONTINUO: PROBLEMAS RESUELTOS

274

Simplificando la ecuacin anterior resulta:

b x dxdydz +

xy
x

dxdydz +
dxdydz + xz dxdydz = 0
x
y
z
x xy xz
+
+
=0
x
y
z

b x +

Cara oculta

z +

Cara oculta

z
dz
z
yz +

xz +

yz

xz
dz
z

xy

xz

dz

yz +

bz

xz +

yz
y

y +

bx

yz

xy +

x + x dx
x

xy +

xy
x

dz

dy

by

xz
dx
x

xy

xy
y

y
y

dy

dy

dx

dx

xz

yz

Cara oculta

dy

Figura 3.8: Tensiones en un elemento diferencial.


Resultante de fuerzas segn direccin y :

Fy = 0

yz

dy dxdz y dxdz + yz +
dz dxdy
z
y

xy

yz dxdy + xy +
dx dydz xy dydz = 0
x

b y dxdydz + y +

Simplificando la ecuacin anterior resulta:

b y +

Universidad de Castilla- La Mancha


Ciudad Real - Espaa

xy
x

Draft

y
y

yz
x z

=0

Por: Eduardo W. V. Chaves (2014)

3 TENSIONES

Resultante de fuerzas segn direccin z :

275

Fz = 0

dz dxdy z dxdy + xz + xz dx dzdy


z
x

yz

xz dzdy + yz +
dy dxdz yz dxdz = 0
y

b z dxdydz + z +

Simplificando la ecuacin anterior resulta:

b z +

xz yz z
+
+
=0
x
y
z

Luego, las ecuaciones de equilibrio son:


x xy xz
+
+
+ b x = 0

y
z
x
xy y yz
+
+
+ b y = 0

y
x z
x

z
xz + yz +
+ b z = 0
x
y
z

Ejemplo 3.14
Dado un medio continuo donde se conoce el estado tensional en un punto y que viene
representado a travs de las componentes del tensor de tensiones de Cauchy:
1 1 0
ij = 1 1 0 Pa
0 0 2

Se pide:
a) Encontrar las tensiones principales y las direcciones donde se producen.
Solucin:
Para obtener las tensiones principales i = i y direcciones principales n (i ) debemos
resolver el siguiente sistema de ecuaciones:
1
1
1
1

0
0

0 n1 0
0 n 2 = 0
2 n 3 0

(3.36)

La obtencin de soluciones no triviales de n (i ) , es equivalente a resolver:


ij ij = 0

Pero si nos fijamos en el formado de la matriz que contiene las componentes del tensor de
tensiones, podemos notar que ya tenemos una solucin principal, ya que las componentes
tangenciales en la direccin 3 son cero, luego:
direccin
1 = 2
n1(1) = n (21) = 0 , n3(1) = 1

Universidad de Castilla- La Mancha


Ciudad Real - Espaa

Draft

Por: Eduardo W. V. Chaves (2014)

MECNICA DEL MEDIO CONTINUO: PROBLEMAS RESUELTOS

276

Pero obtener las otras dos direcciones es suficiente con resolver:


1
1
= (2 ) = 0
1
1

Podemos fcilmente verificar que las races de la ecuacin anterior son:


2 = 2 , 3 = 0

Expresamos las componentes del tensor de tensiones en este nuevo sistema como:
2 0 0
ij = 0 2 0 Pa
0 0 0

b) Direcciones principales
b.1) Para obtener la direccin principal asociada a la solucin 2 = 2 , sustituimos esta
solucin en la ecuacin (3.36):
1
0 n1( 2 ) 0
1 2

1
1 2
0 n (22 ) = 0

0
0
2 2 n (32 ) 0

n1( 2 ) + n (22 ) = 0
( 2)
n1 n (22 ) = 0
2

Resolviendo el sistema obtenemos n 3( 2) = 0 , n1( 2) = n (22) y utilizando n1( 2 ) + n (22 ) = 1 resulta:


n1( 2 ) = n (22 ) =

1
2

1
. n ( 2 ) =

1
2

b.2) Para la solucin 3 = 0 , obtenemos que:


n1( 3) + n (23) = 0

n1( 3) + n (23) = 0
(3)
2n 3 = 0

1
0 n1( 3) 0
1 0

1
1 0
0 n (23) = 0

0
0
2 0 n 3( 3) 0

Resolviendo el sistema obtenemos n (33) = 0 , n1(3) = n (23) y utilizando n1(3) + n (23) = 1 ,


resulta: n1(3) =

1
1 (3) 1
, n (23) =
. ni =
2
2
2

1
2

0 .

Como hemos visto, los autovectores constituyen una matriz de transformacin, A , entre
los dos sistemas, es decir, = A A T , As:

0
0
1 = 2
0
0 =
2 = 2

0
0
3 = 0

Universidad de Castilla- La Mancha


Ciudad Real - Espaa

1
2
1
2

1
2
1

Draft

0
1

1 1 0
1
0 1 1 0
2

0 0 2 1
0

0
1
2
1

Por: Eduardo W. V. Chaves (2014)

3 TENSIONES

277

Ejemplo 3.15
Una presa prismtica est sometida a una presin ejercida por el agua. La presa tiene
espesor b y altura h , ver Figura 3.9. Obtener las restricciones de las componentes
cartesianas del tensor de tensiones de Cauchy en las caras BC , OB y AC .
x2

a - densidad de masa del agua


g - aceleracin de la gravedad
C

a
a g (h x 2 )
h

x1

Figura 3.9.
Solucin:
La cara BC tiene como normal n i( BC ) = [0 1 0] . Teniendo en cuenta que en esta cara no
hay vector traccin, concluimos que:
t i( BC )

= 0 i = ij n j

11

21
31

12
22
32

13 0 12 0
23 1 = 22 = 0
33 0 32 0

Lo que es lo mismo que i 2 = 0 y debido a la simetra 2i = 0 .


La cara OB tiene como normal n (i BC ) = [ 1 0 0] . Teniendo en cuenta que en esta cara el
vector traccin tiene como componentes t i(OB ) = [ a g (h x 2 ) 0 0] , concluimos que:
t i(OB )

a g ( h x 2 )
= n
=
0
ij j

11

21
31

12
22
32

13 1 11 a g (h x 2 )

0
23 0 = 21 =

0
33 0 31

Lo que es lo mismo que i1 = a g (h x 2 ) i1 .


La cara AC tiene como normal n (i BC ) = [1 0 0] . Teniendo en cuenta que en esta cara no
hay vector traccin, concluimos que:
t i( AC )

= 0 i = ij n j

11

21
31

12
22
32

13 1 11 0
23 0 = 21 = 0
33 0 31 0

Lo que es lo mismo que i1 = 0 y debido a la simetra 1i = 0 .

Universidad de Castilla- La Mancha


Ciudad Real - Espaa

Draft

Por: Eduardo W. V. Chaves (2014)

MECNICA DEL MEDIO CONTINUO: PROBLEMAS RESUELTOS

278

3.3 Otras Medidas de Tensin


Ejemplo 3.16
Demostrar que se cumplen las siguientes relaciones:
P = J dev F T + J m F T

S = JF 1 dev F T + J m C 1

donde P y S son el primer y segundo tensor de tensiones de Piola-Kirchhoff,


respectivamente, C es el tensor derecho de deformacin de Cauchy-Green, F es el
gradiente de deformacin, J es el determinante del Jacobiano, y m es la tensin media
del tensor de tensiones de Cauchy. Demostrar tambin que se cumplen las siguientes
relaciones:
P : F = S : C = 3J m

Solucin:
Teniendo en cuenta que P = J F T , y la descomposicin de como = esf + dev ,
podemos obtener que:
P = J ( dev + m 1) F T = J dev F T + J m 1 F T = J dev F T + J m F T

Consideremos ahora la definicin del segundo tensor de tensiones de Piola-Kirchhoff


S = JF 1 F T , y teniendo en cuenta la descomposicin aditiva de como
= esf + dev , obtenemos:
S = JF 1 F T = JF 1 ( dev + m 1) F T = JF 1 dev F T + JF 1 m 1 F T
= JF 1 dev F T + J m C 1

Aplicando en doble producto escalar entre los tensores S y C , obtenemos que:


S : C = JF 1 dev F T : C + J m C 1 : C

donde el trmino JF 1 dev F T : C queda:


JF 1 dev F T : C

C
= ( JF 1 dev F T ) : {

( JF 1 dev F T ) ij ( F T F ) ij

1
= J Fip1 dev
pk F jk Fqi Fqj

F T F

= J qp qk dev
pk
= J dev
pk pk
dev
=J
:1
1
424
3

Tr (

dev

=0

) =0

Luego:
S : C = J m C 1 : C = J m Tr (C 1 C ) = J m Tr (1) = 3 J m

Ahora, haciendo el doble producto escalar entre P y F , obtenemos que:


P : F = J dev F T : F + J m F T : F

Analizando el trmino J dev F T : F , concluimos que:

Universidad de Castilla- La Mancha


Ciudad Real - Espaa

Draft

Por: Eduardo W. V. Chaves (2014)

3 TENSIONES

J dev F T : F

279

= ( J dev F T ) ij ( F ) ij
= J ikdev F jk1 Fij = J ikdev ik
dev
=J
:1 = 0
1
424
3

Tr ( dev )=0

Luego,
P : F = J m F T : F = J m Tr ( F T F T ) = J m Tr (1) = 3 J m

3.4 Mxima Tensin de Corte, Crculo de Mohr


Ejemplo 3.17
Cul es la tensin de corte mxima cuyo estado tensional en un punto es el siguiente?
x2

30 MPa

20 MPa
x1

x3

Figura 3.10.

Solucin:
Como los ejes xi son ejes
principales, dibujamos el crculo
de Mohr con las tensiones
principales
I = 30 MPa ,
II = 20MPa y III = 0 .

max (MPa)

20

30

N (MPa)

Figura 3.11.

max =

Universidad de Castilla- La Mancha


Ciudad Real - Espaa

30 0
2

Draft

= 15 MPa

(3.37)

Por: Eduardo W. V. Chaves (2014)

MECNICA DEL MEDIO CONTINUO: PROBLEMAS RESUELTOS

280

Ejemplo 3.18
Dado el estado tensional en un punto representado por el elemento infinitesimal mostrado
en la Figura 3.12. Se pide:
x2

a) Dibujar el crculo de Mohr;


b) Obtener la tensin normal mxima,

20 MPa

e indicar el plano en la que se produce;


c) Obtener la tensin tangencial mxima.

5 MPa
x1

10 MPa

x3

Figura 3.12:
Solucin:
S
max = 15

N max = 10MPa
S max =

20

10 (20)
= 15MPa
2

N (MPa)

10

Ejemplo 3.19
Determinar para que valores de * son posibles
los siguientes estados tensionales en planos que
pasen por P :

Caso a) N = 4

=2

Caso b) N = 4

=1

Caso c) N = 7

=0

Figura 3.13.
Solucin
Para que los pares de valores ( N ; ) sean factibles, tienen que pertenecer a la zona en gris
en el crculo de Mohr, ver Figura 3.14, o pertenecer a las circunferencias.

Universidad de Castilla- La Mancha


Ciudad Real - Espaa

Draft

Por: Eduardo W. V. Chaves (2014)

3 TENSIONES

281

Figura 3.14: Crculo de Mohr.

Caso a)
Caso b)

Caso c)

1
2

Figura 3.15: Crculo de Mohr.

Universidad de Castilla- La Mancha


Ciudad Real - Espaa

Draft

Por: Eduardo W. V. Chaves (2014)

MECNICA DEL MEDIO CONTINUO: PROBLEMAS RESUELTOS

282

Caso a) :En este caso el par ( N = 4; = 2) pertenece al crculo formado por la tensiones
principales 2 y 6 , luego * puede ser cualquiera, ver Figura 3.16.

Caso a)

2
1

Figura 3.16: Crculo de Mohr.


Caso b) En este caso podemos decir que la solucin es:
*( 2) * *(1)

(3.38)

donde *( 2) , *(1) estn sealados en la Figura 3.17.

Casos lmites

( 4,1)

( 4,1)

1
2

* ( x ) 6

* ( 2)

* (1) 6

Figura 3.17: Crculo de Mohr.

Universidad de Castilla- La Mancha


Ciudad Real - Espaa

Draft

Por: Eduardo W. V. Chaves (2014)

3 TENSIONES

283

Partiendo de la ecuacin de la circunferencia:


( x xC ) 2 + ( y y C ) 2 = R 2

Para el caso *(1) , tenemos: x = 4; x C =

( *(1) + 2)
2

(3.39)

y = 1; y C = 0; R =

( *(1) 2)
2

Reemplazando estos valores en la ecuacin de la circunferencia, resulta:


( x xC ) 2 + ( y y C ) 2 = R 2
2

( * 2)
( * + 2)
4 (1)
+ (1 0)2 = (1)

2
2

(3.40)

*(1) = 4,5

Para el caso

*( 2 )

, tenemos: x = 4; xC =

(6 + *( 2 ) )
2

; y = 1; y C = 0; R =

(6 *( 2 ) )
2

reemplazando estos valores en la ecuacin de la circunferencia, resulta:


( x xC ) 2 + ( y y C ) 2 = R 2
2

(6 *( 2 ) )
(6 + *( 2) )
4
+ (1 0)2 =

2
2

(3.41)

*( 2 ) = 3,5

luego:
(3.42)

3,5 * 4,5

Caso c) En este caso la nica solucin posible es que N sea una tensin principal, luego
(3.43)

* = 7

* = 7

Figura 3.18: Crculo de Mohr.

Universidad de Castilla- La Mancha


Ciudad Real - Espaa

Draft

Por: Eduardo W. V. Chaves (2014)

MECNICA DEL MEDIO CONTINUO: PROBLEMAS RESUELTOS

284

Ejemplo 3.20
Obtener la mxima tensin normal y tangencial (de corte) y dibujar el crculo de Mohr
correspondiente para los siguientes estados tensionales:
a)
a)

0
ij = 0
0 0 0

2 0 0
b) ij = 0 0
0
0

(3.44)

Solucin: a) Valores principales. Si verificamos el formato de las componentes del tensor de


tensiones de Cauchy, ya podemos decir que un valor principal es (3) = 0 . Luego, es
suficiente obtener solo los dos otros autovalores:


= ( ) 2 2 = 0 = = 0

(3.45)

(1) = 0
( ) 2 2 = 0 2 2 + 2 2 = 0 (2 + ) = 0
( 2) = 2

(3.46)

N max = 2
max =

max =

Figura 3.19:

b)

N max =
max =

(2) 3
=
2
2

Figura 3.20.
Universidad de Castilla- La Mancha
Ciudad Real - Espaa

Draft

Por: Eduardo W. V. Chaves (2014)

3 TENSIONES

285

Ejemplo 3.21
Hacer la representacin del crculo de Mohr para los siguientes casos:
1) Caso unidimensional, estado de carga de traccin
2) Caso unidimensional, estado de carga de compresin
3) Caso bidimensional, estado de carga de traccin
4) Caso triaxial
5) estado de corte puro
Solucin:
1) Caso unidimensional, estado de carga de traccin

I
0

0 0
0 0
0 0

2) Compresin uniaxial

x
0
0
0
II

0
0

0
0
0

II

3) Caso biaxial
II

I
II
I
0

Universidad de Castilla- La Mancha


Ciudad Real - Espaa

0
II
0

0
0
0

Draft

Por: Eduardo W. V. Chaves (2014)

MECNICA DEL MEDIO CONTINUO: PROBLEMAS RESUELTOS

286

4) Caso triaxial

III

I
0

II

0
II
0

0
0
III

III

II

5) Corte puro
S
0 0
0 0

0 0 0

3.5 Particularidades del Tensor de Tensiones


Ejemplo 3.22
Las componentes del tensor de tensiones de Cauchy en un punto P vienen dadas por:
5 6 7
ij = 6 8 9 GPa
7 9 2

(3.47)

a) Calcular la tensin media;


b) Calcular la parte volumtrica y desviadora del tensor .
Solucin:

5+8+2
m = kk =
=5
3
3

ij =

ijesf

ijdev

Universidad de Castilla- La Mancha


Ciudad Real - Espaa

ijdev

esf
ij

= ij

Draft

m
= 0
0

esf
ij

0
m
0

0 5 0 0
0 = 0 5 0
m 0 0 5

(3.48)

0 6 7
= 6 3 9
7 9 3

(3.49)

ijdev

Por: Eduardo W. V. Chaves (2014)

3 TENSIONES

287

Ejemplo 3.23
Considere las componentes del tensor de tensiones:
5 3 2
ij = 3 1 0
2 0 3

unidades de
tensin

(3.50)

dadas en el sistema constituido por la base (e 1 , e 2 , e 3 ) .


Dada la ley de transformacin de base entre los sistemas x y x' por:
x'1

x' 2

x '3

x1

3
5

4
5

x2

3
5

x3

4
5

donde el sistema x' est constituido por la base (e '1 , e ' 2 , e ' 3 ) .
r

a) Obtener el vector tensin t ( e'2 ) segn el plano cuya normal es e ' 2 , expresado segn el
sistema cartesiano (e '1 , e ' 2 , e ' 3 ) con el siguiente formato:

r
t (e'2 ) = ( )e 1 + ( )e 2 + ( )e 3

(3.51)

b) Obtener la parte esfrica y desviadora del tensor de tensiones.


Solucin:
a) Como definimos, la primera fila de la matriz de transformacin est formada por los
cosenos directores del eje x'1 con x1 , x2 y x3 , luego:
3 0 4
1
A = 0 5 0
5
4 0 3

(3.52)

y la ley de transformacin para las componentes de tensor de segundo orden:


(3.53)

' = A A T

luego:

53 0 45 5 3 2 53

ij = 0 1 0 3 1 0 0
4 0 3 2 0 3 4
5
5
5

t i( e' 2 )

9
1
= 5
5
12

Universidad de Castilla- La Mancha


Ciudad Real - Espaa

9 9 2
1
1 0 = 9 5 12
5
2 12 31
0 35
0

r
9
12
t (e'2 ) = e 1 + (1)e 2 + e 3
5

5

Draft

(3.54)

(3.55)

Por: Eduardo W. V. Chaves (2014)

MECNICA DEL MEDIO CONTINUO: PROBLEMAS RESUELTOS

288

ya que:

11

21
31

12
22
32

t 1 ( e'1 )
13
( e ' )
23 = t 2 1
( e ' )
33 t 3 1

t1

( e ' 2 )
( e ' 2 )

t2
( e ' )
t3 2

t1

( e '3 )

( e '3 )

t2

( e '3 )
t3

(3.56)

b)
I
ij + ijdev
3
I = 5 +1+ 3 = 9

ij = ijesf + ijdev =

esf
ij

ijdev

ij ijesf

(3.57)
(3.58)

3 0 0
= 0 3 0
0 0 3

(3.59)

2 2 3 2
5 3 3

= 3 1 3
0 = 3 2 0
2
0 3 3 2 0 0

(3.60)

Ejemplo 3.24
El estado de tensin en un medio continuo (cuerpo) est dado por el tensor de tensiones
de Cauchy:
0
ij = Cx 3
0

Cx 3
0
Cx1

0
Cx1
0

donde C es una constante. Considrese que el cuerpo est libre de fuerzas msicas.
a) Probar si el cuerpo est en equilibrio;
b) Calcular el vector tensin en el punto P (4,4,7) segn un plano cuya normal viene dada
2
3

2
3

1
3

por n = e 1 + e 2 e 3 .
c) Representar los crculos de Mohr del estado de tensin del punto P .
Solucin:
a) Para que el medio continuo est en equilibrio hay que cumplir las ecuaciones de
equilibrio:
r r
+ b = 0 ; ij,j + b i = 0 i

0
ij = Cx 3
0

Cx 3
0

Cx1

(3.61)

0
Cx1
0

(3.62)

Para el problema propuesto b i = 0 i , luego:


ij,j

i = 1 0 + 0 + 0 = 0
i1 i 2 i 3

=
+
+
i = 2 0 + 0 + 0 = 0
=
x j
x1
x 2
x3
i = 3 0 + 0 + 0 = 0

ij

Universidad de Castilla- La Mancha


Ciudad Real - Espaa

Draft

(3.63)

Por: Eduardo W. V. Chaves (2014)

3 TENSIONES

289

ij,j = 0 i luego el cuerpo est en equilibrio.

b) El vector tensin viene dado por:


r
(n )
t (n) = n ; t i = ij n j

0
ij ( x1 = 4; x 2 = 4; x3 = 7) = Cx3
0

Cx3
0
Cx1

(3.64)

0 0
Cx1 = 7C
0 0

7C
0
4C

0
4C
0

2
1
n j = 2
3
1

(3.65)

(3.66)

Resultando que:
0
r (n )
t i = ij n j = 7C
0

7C
0
4C

0 2
14C
1 1

4C 2 = 18C
3
3
8C
0 1

(3.67)

c)
0
0 7

ij = C 7 0 4
0 4 0

(3.68)

Los autovalores (tensiones principales) vienen dados por:


0 0 0
0 7

C 7 0 4 0 0 = 0
0 4 0 0 0

C
0
0 7

C 7 0 4 C 0

0 4 0

C
0

0 =0

(3.69)

Considerando que = C , obtenemos:


0
7

C 7 4 = C 3
0 4
3 + 16 + 49 = 0

0
7
7 4 = 0

0 4

2 + 65 = 0

= 65

(3.70)
(3.71)

Con eso, obtenemos que = C 65 .

Universidad de Castilla- La Mancha


Ciudad Real - Espaa

Draft

Por: Eduardo W. V. Chaves (2014)

MECNICA DEL MEDIO CONTINUO: PROBLEMAS RESUELTOS

290

Resultando as un estado de corte puro, el crculo de Mohr viene representado por:


S

C 65

III = C 65

I = C 65

Ejemplo 3.25
El estado tensional en un punto del cuerpo viene dado por las componentes del tensor de
tensiones de Cauchy representado en el sistema cartesiano como:
a) Obtener la tensin desviadora;
b) Determinar las tensiones principales ( I , II , III ) y las direcciones principales;
c) Dibujar el crculo de Mohr;
d) Obtener la mxima tensin de corte;
e) Encontrar el vector tensin en un plano que pasa por el punto dado cuya normal a este
plano es n = 0,75e 1 + 0,25e 2

6
e3 ;
4

f) Obtener tambin la tensin normal y tangencial en este plano.


x3

r
t ( e 3 ) = 8e 1

r
t ( e 2 ) = 6e 1

e 3
e 1

e 2

x2

r
t ( e1 ) = 6e 2 + 8e 3

x1

Figura 3.21:
Solucin: Segn la Figura 3.21 podemos obtener las componentes del tensor de tensiones de
Cauchy como:

Universidad de Castilla- La Mancha


Ciudad Real - Espaa

Draft

Por: Eduardo W. V. Chaves (2014)

3 TENSIONES

291

0 6 8
ij = 6 0 0
8 0 0

a)
ij = ijesf + ijdev

La parte esfrica ijesf =

I
ij = 0 ij ya que I = 0 . Luego, la parte desviadora viene dada por:
3
ijdev

ijesf

0 6 8
ij = 6 0 0
8 0 0

b) Los autovalores pueden ser determinados por el determinante caracterstico:


6
6
8

8
0 =0

3 + 100 = 0

2 + 100 = 0

Las soluciones son 1 = 0 , 2 = 10 , 3 = 10 , que son las tensiones principales. Las direcciones
principales quedan:
1 = 0 autovalor
n i(1) = [0 0,8 0,6]

2 = 10 autovalor
n i( 2 ) = [ 0,707 0,424 0,566]
3 = 10 autovalor
n i(3) = [0,707 0,424 0,566]

I = 10 , II = 0 , III = 10

c) El crculo de Mohr viene dibujado en la figura abajo:


S
max = 10

III = 10

II = 0

I = 10

d) En el crculo de Mohr se puede obtener directamente la tensin de corte mxima:


max = 10

Universidad de Castilla- La Mancha


Ciudad Real - Espaa

Draft

Por: Eduardo W. V. Chaves (2014)

MECNICA DEL MEDIO CONTINUO: PROBLEMAS RESUELTOS

292

e) Teniendo en cuenta que t i (n) = ij n j , podemos obtener las componentes del vector tensin
6
e3 :
4

en el plano de normal n = 0,75e 1 + 0,25e 2

t 1 (n ) 0 6 8 0,75 3,39898
(n )

t 2 = 6 0 0 0,25 4,5
t (n ) 8 0 0

6
6
3
4

f)
r
t (n)

r
S

r
N
s

El mdulo de N se puede obtener a travs de la proyeccin N = t (n) n = t i (n) n i ,


luego:
r
N

0,75
(n )

= t i n i [ 3,39898 4,5 6] 0,25 5,09847


6

El vector N viene dado por:


r
r
N = N n = 3,82385e 1 1,27462e 2 + 3,12216e 3
r
r
r
Adems como se cumple que t (n) = N + S , podemos obtener el vector tangencial a este

plano como:

r
r
r
S = t (n) N ( 3,39898 + 3,82385)e 1 + (4,5 + 1,27462 )e 2 + (6 3,12216 )e 3
(0,42487 )e 1 + (5,77462 )e 2 + (2,87784 )e 3

y su mdulo:
r
S

(0,42487)2 + (5,77462)2 + (2,87784)2

41,808713 = 6,465966
r

OBS.: Tambin podramos haber utilizado la expresin S

r
obtener el mdulo de S .

Universidad de Castilla- La Mancha


Ciudad Real - Espaa

Draft

r r
r
= t (n) t (n) N

para

Por: Eduardo W. V. Chaves (2014)

3 TENSIONES

293

Ejemplo 3.26
El campo del tensor de tensiones de Cauchy de un medio continuo viene representado por:
3 x1
r
ij ( x ) = 21

31

5 x 22
3x 2
32

2 x3
0

a) Obtener las fuerzas msicas (por unidad de volumen) para que el medio continuo est en
equilibrio.
b) Para un punto particular ( x1 = 1, x 2 = 1, x3 = 0 ). Se pide:
b.1) Dibujar el Crculo de Mohr en tensiones. Obtener la tensin tangencial y normal
mximas.
1

b.2) Obtener el vector tensin en el plano definido por la normal n i =

b.2.1) Obtener la componente normal y tangencial en este plano.


Solucin:
a) Debido a la simetra del tensor de tensiones de Cauchy tenemos que:
3 x1
r 2
ij ( x ) = 5 x 2
0

5 x 22
3x 2
2 x3

2 x3
0

+ 12, 2 + 13,3 = b1
3 + 10 x 2 + 0 = b1
r r componentes 11,1

r
x + b = 0
21,1 + 22, 2 + 23,3 = b1 0 + 3 + 2 = b 2
0 + 0 + 0 = b

31,1 + 32, 2 + 33,3 = b1

con lo cual obtenemos que:


10 x 2 3
b i = 5

(Fuerza por unidad de volumen)

(3.72)

b) Para el punto en particular ( x1 = 1, x 2 = 1, x3 = 0 ) tenemos que:


3 5 0
ij = 5 3 0
0 0 0

donde podemos verificar que 3 = 0 es un valor principal. Para obtener los otros
autovectores es suficiente con resolver:
3
5
=0
5
3

(3 ) = (5) 2

3 = 5

1 = 8

2 = 2

Reestructurando los autovalores:


I = 8 , II = 0 , III = 2

Universidad de Castilla- La Mancha


Ciudad Real - Espaa

Draft

Por: Eduardo W. V. Chaves (2014)

MECNICA DEL MEDIO CONTINUO: PROBLEMAS RESUELTOS

294

b.1) El crculo de Mohr viene dibujado en la figura abajo:


S
max = 5

II = 0

III = 2

I = 8

En el crculo de Mohr se puede obtener directamente la tensin de corte mxima max = 5 y la


tensin normal mxima N max = I = 10 .

e) Teniendo en cuenta que t i (n) = ij n j , podemos obtener las componentes del vector tensin
en el plano de normal n =

1
1
1
e1 +
e2 +
e 3 (vector unitario):
3
3
3

t 1 (n )
3 5 0 1
8
(n ) 1
1

5 3 0 1 =
8
t 2 =
3
3
t (n )
0 0 0 1
0
3

b.2) La tensin normal:


N = ti

(n )

n i =

1
[8 8 0] 1 = 16
3
3
1

Para la componente tangencial podemos aplicar directamente S


r
donde t (n)

r r
r
= t (n) t (n) N

8
r (n ) r (n )
1 1
(n ) (n )
[8 8 0] 8 = 128 . Luego:
= t t = ti ti =
3
3 3
0

r
S

r r
r
= t (n) t (n) N

128 16
128
=
3 3
9

S =

128
3

Ejemplo 3.27
El estado tensional en un punto del cuerpo viene dado por las componentes del tensor de
tensiones de Cauchy segn su parte esfrica y desviadora, respectivamente:
ijesf

1 0 0
= 0 1 0
0 0 1

ijdev

0 6 8
= 6 0 0
8 0 0

Se pide:
Universidad de Castilla- La Mancha
Ciudad Real - Espaa

Draft

Por: Eduardo W. V. Chaves (2014)

3 TENSIONES

295

a) Obtener las componentes del tensor de tensiones de Cauchy;


b) Determinar las tensiones principales ( I , II , III ) y las direcciones principales.
c) Obtener la mxima tensin de corte;
d) Dibujar el crculo de Mohr para: d.1) el tensor de tensiones de Cauchy ( ij ), d.2) Parte
esfrica ( ijesf ) y; d.3) parte desviadora ( ijdev );
Solucin:
a)

ij =

ijesf

1 0 0 0 6 8 1 6 8
= 0 1 0 + 6 0 0 = 6 1 0
0 0 1 8 0 0 8 0 1

ijdev

En el Ejemplo 3.25 hemos obtenido los valores principales del tensor ijdev que es el
mismo del problema propuesto. Como el tensor y su parte desviadora tienen las mismas
direcciones principales, podemos obtener de forma automtica las tensiones principales:
ij =

ijesf

ijdev

0 9 0 0
1 0 0 10 0

0 = 0 1 0
= 0 1 0 + 0 0
0 0 1 0 0 10 0 0 11

Las direcciones principales son las mismas del tensor del Ejemplo 3.25.
d) Crculo de Mohr
S

S
max = 10

+
dev
III = 10

dev
II = 0

dev
= 10
I

dev
N

I = II = III = 1

Parte desviadora

Parte esfrica

14444444444444442444444444444444
3

S
max = 10

III = 9

Universidad de Castilla- La Mancha


Ciudad Real - Espaa

II = 1

Draft

I = 11

Por: Eduardo W. V. Chaves (2014)

MECNICA DEL MEDIO CONTINUO: PROBLEMAS RESUELTOS

296

Ejemplo 3.28
En un punto P del medio continuo el tensor de tensiones de Cauchy viene
representado por sus componentes cartesianas por:
1 1 0
ij = 1 1 0 MPa ,
0 0 2

Se pide:
a) Determinar las tensiones principales y las direcciones principales en el punto P ;
b) Obtener la mxima tensin de corte;
c) Dibujar el crculo de Mohr para: c.1) el tensor de tensiones de Cauchy ( ij ), c.2) Parte
esfrica ( ijesf ) y; c.3) parte desviadora ( ijdev );
d)

i.) Encontrar el vector tensin en un plano que pasa por el punto dado cuya
r
direccin normal a este plano es n = 1,0e 1 + 1,0e 2 + 0e 3 ;
ii.) Obtener tambin la tensin normal y tangencial en este plano.

f) Obtener los autovalores y autovectores de la parte desviadora del tensor de tensiones de


Cauchy ( dev ).
Solucin:
a) Ver Ejemplo 3.14. Los autovalores son I = 2 , II = 2 , III = 0
b) y c)
ijdev

= ij

ijesf

Universidad de Castilla- La Mancha


Ciudad Real - Espaa

1 0 0
1 0 0
2 0 0
2
4

= 0 2 0 0 1 0 = 0 1 0
3
3
0 0 1
0 0 2
0 0 0

Draft

Por: Eduardo W. V. Chaves (2014)

3 TENSIONES

297

max = 1

N
III = 1,333

I , II = 0,667

I = II = III = 1,333

Parte desviadora

Parte esfrica

14444444444444442444444444444444
3
S

max = 1

III = 0

I , II = 2
r

d) El vector tensin se obtiene a partir de t (n) = n , normalizando el vector obtenemos


r
n
n

que: n = r =

1
1
e1 +
e 2 + 0e 3 . Vector tensin:
2
2

t 1(n ) 1 1 0
2
1
(n )
1 1

t 2 = 1 1 0
1 = 2 2
t (n ) 0 0 2 2 0
0

Ejemplo 3.29
Las componentes de un estado de tensin en un punto P son:
0
0
29

ij = 0 26 6 Pa
0
6
9

Descompnganse las componentes del tensor de tensiones en una parte esfrica y otra
desviadora, y determnense los valores de las tensiones principales del tensor desviador.
Solucin:
Considerando la descomposicin aditiva del tensor de tensiones en una parte esfrica y
desviadora:

Universidad de Castilla- La Mancha


Ciudad Real - Espaa

Draft

Por: Eduardo W. V. Chaves (2014)

MECNICA DEL MEDIO CONTINUO: PROBLEMAS RESUELTOS

298

ij = ijdev + ijesf

La parte desviadora viene dada por


ijdev

11 m
= 12
13

12
22 m
23

13
23
33

siendo la tensin media dada por:


m =

( 29 26 + 9)
1
ii =
=4
3
3

Resultando as:
ijdev

0
0 25
0
0
29 4

= 0
26 4
6 = 0 30 6 Pa
0
6
9 4 0
6
5

Las componentes del tensor hidrosttico son:


ijhid

esf
ij

4 0 0
= 0 4 0 Pa
0 0 4

Para comprobar las operaciones anteriores, la siguiente relacin tiene que verificarse:
ij =

ijdev

ijesf

0
0 4 0 0 29
0
0
25

= 0 30 6 + 0 4 0 = 0 26 6 Pa
0
6
5 0 0 4 0
6
9

Obteniendo la ecuacin caracterstica del tensor de tensiones desviador:


ijdev ij = 0
3 J 2 J 3 = 0

Con la solucin de la ecuacin cbica anterior obtenemos las tensiones principales del
tensor desviador:
1dev = 25 Pa
dev
2 = 6 Pa
dev = 31Pa
3

Ejemplo 3.30
Descomponer el tensor de tensiones de Cauchy dado por sus componentes:
12
ij = 21
31

4
9
32

0
2 MPa
3

en su parte esfrica y desviadora.


Obtener los invariantes del tensor desviador
Obtener tambin la tensin normal octadrica, y la tensin media en este punto.
Solucin:

Universidad de Castilla- La Mancha


Ciudad Real - Espaa

Draft

Por: Eduardo W. V. Chaves (2014)

3 TENSIONES

299

Debido a la simetra del tensor de tensiones de Cauchy:


0
12 4

ij = 4 9 2 MPa
0 2 3

I 12 + 9 + 3 24
=
=
= 8.
3
3
3

Tensin media m = oct =

La parte esfrica y desviadora del tensor de tensiones son:


ijesf

8 0 0
= 0 8 0
0 0 8

ijdev

= ij

ijesf

0 8 0 0 4 4
0
12 4

9 2 0 8 0 = 4 1 2
=4
0 2 3 0 0 8 0 2 5

Los invariantes principales del tensor desviador son:


I dev J1 = 4 + 1 5 = 0 , como era de esperar, ya que la traza de cualquier tensor desviador

es cero.
II dev =

2 5

0 5

4 4
4 1

= 41 = J 2

o bien utilizando la definicin: J 2 =

) (

1 2
1
I 3 II = 24 2 3 151 = 41
3
3

III dev J 3 = det ( dev ) = 44

Ejemplo 3.31
El estado tensional en un punto est dado por el tensor de tensin:
a b
ij = a c
b c

donde a , b , c son constantes y es un valor de tensin. Determinar las constantes a ,


b , c de tal manera que el vector tensin se anule en un plano octadrico.
Solucin:
Un plano octadrico tiene el siguiente versor: n i =
r

1
3

[1

1 1] . El vector tensin en este

plano viene definido por t (n) = n , en componentes:

t 1(n ) a b
1
+ a + b 0 a + b = 1
(n )
1 1

1 =
a + + c = 0 a + c = 1
t 2 = a c

3
t (n ) b c 3 1
b + c + 0 b + c = 1

resolviendo el sistema anterior obtenemos que, b =

1
1
1
, c=
, a=
2
2
2

Ejemplo 3.32
En un punto P del medio continuo el tensor de tensiones de Cauchy viene
representado por sus componentes cartesianas por:
Universidad de Castilla- La Mancha
Ciudad Real - Espaa

Draft

Por: Eduardo W. V. Chaves (2014)

MECNICA DEL MEDIO CONTINUO: PROBLEMAS RESUELTOS

300

57
ij = 21
31

0
50
32

24
0 MPa
43

a) Determinar las tensiones principales y las direcciones principales en el punto P ;


b) Obtener la tensin tangencial mxima y la tensin normal mxima;
c) Dibujar el crculo de Mohr del estado tensional correspondiente;
r
d) Obtener el vector tensin t (n) en el plano octadrico del espacio de Haigh-Westergaard.
Obtener tambin la tensin normal octadrica y la tensin tangencial octadrica.
Solucin:
Teniendo en cuenta que la simetra del tensor de tensiones de Cauchy:
57 0 24
ij = 0 50 0 MPa
24 0 43

Verificamos que la tensin 22 = 50 ya es una tensin principal y est asociada al


autovector n ( 2) = [0 1 0] . Para encontrar las otras tensiones principales resolvemos el
siguiente sistema:
57
24
=0
24
43

1 = 25
2 100 + 1875 = 0
3 = 75

Utilizando la definicin de autovalor-autovector, podemos obtener los siguientes


autovectores:
Asociado al autovalor 1 = 25

n (1) = [m 0,6 0 0,8]

Asociado al autovalor 3 = 75

n (3) = [ 0,8 0 0,6]

Circulo de Mohr en tensiones:


Reestructurando tal que I > II > III :
I = 75 , II = 50 , III = 25

b, c) El crculo de Mohr viene dibujado en la figura abajo:


S

max =

max = 25

III = 25

Universidad de Castilla- La Mancha


Ciudad Real - Espaa

II = 50

Draft

75 25
= 25
2

I = 75 = N max

Por: Eduardo W. V. Chaves (2014)

3 TENSIONES

301

d) El espacio de Haigh-Westergaard est formado por las tensiones principales luego, el


r
vector tensor en este espacio viene dado por t (n) = n , cuya normal del plano octadrico
1
tiene como componentes n i =

r
t (n) = n

1
:
3

1
3

componente

t 1(n) 75 0 0
1
75
(n)
1 1

t 2 = 0 50 0
1 = 3 50
t (n) 0 0 25 3 1
25

Su mdulo viene dado por:


r
t (n)

1
8750
75 2 + 50 2 + 25 2 =
3
3

r
t (n) = 54,00617

La tensin normal octadrica viene dada por oct = t (n) n :


oct

1
[75 50 25] 1 = 50
=
3 3
1
1

Podramos haber aplicado directamente la definicin de tensin normal octadrica:


oct =

I
75 + 50 + 25
= m =
= 50
3
3

La tensin tangencial octadrica se puede obtener a travs del teorema de Pitgoras:


oct =

r
t (n)

oct =

8750
50 2 = 20,4124
3

Tambin podramos haber aplicado la definicin:


oct =

1
1
2 I 2 6 II =
2 150 2 6 6875 = 20,41241
3
3

donde I = 150 , II = 75 50 + 75 25 + 50 25 = 6875 .


e) Componentes de la parte esfrica del tensor:
ijesf

50 0 0
Tr ( )
=
ij = m ij = 0 50 0
3
0 0 50

ijesf

57 0 24 50 0 0 7 0 24
= 0 50 0 0 50 0 = 0 0 0
24 0 43 0 0 50 24 0 7

Su parte desviadora:
ijdev

= ij

f) Teniendo en cuenta que el tensor y su parte desviadora son coaxiales, es decir, presentan
las mismas direcciones principales, podemos utilizar el espacio principal para obtener los
valores principales del tensor desviador:
ijdev

= ij

ijesf

Universidad de Castilla- La Mancha


Ciudad Real - Espaa

0
75 0 0 50 0 0 25 0

= 0 50 0 0 50 0 = 0 0
0
0 0 25 0 0 50 0 0 25

Draft

Por: Eduardo W. V. Chaves (2014)

MECNICA DEL MEDIO CONTINUO: PROBLEMAS RESUELTOS

302

3.6 Estado Tensional en Dos Dimensiones


Ejemplo 3.33
Considere el siguiente estado de tensin:
5

4
2
y

6
x

Figura 3.22:
Obtener el estado de tensin en este punto ij .
Solucin:
En el estado de tensn plano ij (i, j = 1,2) se necesitan dos planos para definir
completamente el estado tensional en el punto:
x
ij =
xy

xy
y

(3.73)

Segn la Figura 3.22 verificamos que:


5

x = 4
xy = 2

xy = 2
y

y = 6
x

Figura 3.23:
Universidad de Castilla- La Mancha
Ciudad Real - Espaa

Draft

Por: Eduardo W. V. Chaves (2014)

3 TENSIONES

303

Luego:
4 2
ij =

2 6

(3.74)

Ejemplo 3.34
Considrese un material compuesto, constituido por matriz y fibras segn direccin de 45
tal como se indica en la Figura 3.24. Este material compuesto puede romper si la tensin de
corte a lo largo de la fibra supera el valor de 3,8 10 6 Pa ( N / m 2 ) .
Para una tensin normal x = 2,8 10 6 Pa , determnese el valor mximo de y para que el
material no rompa.
y

45

45

Figura 3.24: Material compuesto (matriz-fibra).


Solucin: Este es un ejemplo tpico de transformacin de coordenadas. Es decir, tenemos
que considerar la tensin de corte mxima segn la direccin = 45 . Para ello realizamos
la transformacin de coordenadas siguiente:
xy ( ) =
xy ( = 45 )

x y

sin 2 + xy cos 2
2
y 4,8 10 6 Pa
6
2,8 10 y
=
sin( 90 ) = 3,8 10 6 Pa
2

(compresin)

Ver Ejemplo 1.99 del captulo 1.

Universidad de Castilla- La Mancha


Ciudad Real - Espaa

Draft

Por: Eduardo W. V. Chaves (2014)

MECNICA DEL MEDIO CONTINUO: PROBLEMAS RESUELTOS

304

Ejemplo 3.35
Las tensiones que actan en dos planos que pasan por el punto P estn indicadas en la
Figura 3.25. Determnese el valor de la tensin de corte en el plano a a y las tensiones
principales en este punto.
y

80 Pa
45
x

60

60 Pa

a
b

Figura 3.25: Estados tensionales en un punto, segn los planos a y b .


Solucin:
Para obtener el estado de tensin en un punto, en el caso de dos dimensiones,
determinamos las tensiones: x , y , xy , como se indica en la Figura 3.26.
y

xy
80 Pa

xy
xy

45

xy

45

60

80 Pa

60
xy
60 Pa

60 Pa
b

a)

b)

Figura 3.26: Estados tensionales en un punto, segn los planos a y b .


Segn la Figura 3.26, podemos determinar directamente x y xy descomponiendo el
vector tensin 60 Pa , ver Figura 3.26(b), i.e.:
x = 60 cos( 30 ) = 51,962 Pa
xy = 60 cos( 60 ) = 30 Pa

Para determinar la componente y , emplearemos las ecuaciones:

Universidad de Castilla- La Mancha


Ciudad Real - Espaa

Draft

Por: Eduardo W. V. Chaves (2014)

3 TENSIONES

x ( ) N =
xy S ( ) =

x + y
2
x y
2

x y
2

305

cos 2 + xy sin 2

sin 2 xy cos 2

Reemplazando los valores numricos en las expresiones anteriores:


( = 45 ) =
( = 45 ) =

51,962 + y
2
51,962 y
2

51,962 y
2

cos( 90 ) + 30 sin( 90 ) = 80 Pa

sin( 90 ) 30 cos( 90 )

La primera ecuacin nos proporciona el valor de y :


y = 48,038 Pa

Una vez determinado y , podemos determinar (= 45) :


( = 45 ) = 1,96 Pa

Las tensiones principales pueden determinarse a travs de las componentes x , y , xy , tal


como se indica en las ecuaciones:
(1, 2 ) =

(1, 2 )

x + y
2

x y

2

+ 2xy

51,962 + 48,038
51,962 48,038
=

2
2

Universidad de Castilla- La Mancha


Ciudad Real - Espaa

Draft

= 80,1Pa
+ 30 2 1
2 = 19,9 Pa

Por: Eduardo W. V. Chaves (2014)

MECNICA DEL MEDIO CONTINUO: PROBLEMAS RESUELTOS

306

Ejemplo 3.36
Dado el estado de tensiones x = 1Pa , xy = 4 Pa y y = 2 Pa . Obtener una grfica de
ngulo-tensiones ( x , y , xy ), siendo el ngulo de giro de la cua dada en la Figura
3.27.
y

y = 2 Pa

xy = 4 Pa

xy = 4 Pa
x

x = 1Pa

xy
y

Figura 3.27: Estado tensional en un punto.


Solucin:
Calculemos los distintos valores de x , y , xy utilizando las ecuaciones:
x =

x + y

xy =

2
x y

2
x + y

y =

x y
2

cos 2 + xy sin 2

sin 2 + xy cos 2

y x
2

cos 2 xy sin 2

Podemos calcular el ngulo correspondiente a la direccin principal a travs de la ecuacin:


tan 2 =

2 xy
x y

2 ( 4)
= 8 ( = 41,437 )
1 2

y las tensiones principales:


1, 2 =

x + y
2

x y

2

+ 2xy

1 = 5,5311P

2 = 2,5311Pa

Considerando las leyes de transformacin, podemos obtener los distintos valores de


x , y , xy para distintos valores de . Haciendo variar de 0 hasta 360 podemos
representar las tensiones x , y , xy en funcin del ngulo, ver Figura 3.28. Podemos
observar que cuando = 41,437 la tensin tangencial es cero ( xy = 0 ) y las tensiones
principales I = 5,5311Pa y II = 2,5311Pa .

Universidad de Castilla- La Mancha


Ciudad Real - Espaa

Draft

Por: Eduardo W. V. Chaves (2014)

3 TENSIONES

307

x
1

= 41,437

x
= 131,437
2

Tensiones

1 = 5,5311

xy

x
0
0

50

100

-2

-4

200

250

2 = 2,5311

300

350

45

xy

150

= 86,437

-6

max = 4,0311

Figura 3.28: Tensiones en funcin del ngulo .


Ejemplo 3.37
a) Dado un campo de tensiones ij (i , j = 1,2) , y los siguientes valores:
m11 =

t
2

11 x 3 dx 3

m12 =

t
2

t
2

12 x 3 dx3

m 22 =

t
2

t
2

22 x3 dx 3

t
2

y dado un nuevo sistema x1 x 2 x 3 formado por una rotacin alrededor de x3 de un


ngulo , obtener la ley de transformacin de mij (i, j = 1,2) para este nuevo sistema.
Solucin:
Debido a la simetra de ij = ji , concluimos que m12 = m21 . La matriz de transformacin
del sistema x1 x 2 x 3 al sistema x1 x 2 x 3 viene dada por:
cos
aij = sin
0

Universidad de Castilla- La Mancha


Ciudad Real - Espaa

sin
cos
0

0
0
1

Draft

2D

cos
A=
sin

sin
cos

Por: Eduardo W. V. Chaves (2014)

MECNICA DEL MEDIO CONTINUO: PROBLEMAS RESUELTOS

308

Vamos utilizar la notacin de Voigt, luego:



m11

{m } = m22 =
m
12

t
2


11

22 x3 dx3 =

12

t
2

t
2


11

22 x3 dx3 =

12

t
2

t
2

t
2

11
[M] 22 x3dx3 = [M]

12

t
2

11

22 x3 dx3

12

t
2

con eso, podemos concluir que:



m11
m11

{m } = m 22 = [M]m 22 = [M]{m}
m
m
12
12

(3.75)

donde la matriz [M] es la matriz de transformacin para un tensor de segundo orden


cuando ste est en la notacin de Voigt, ver Ejemplo 1.99, y viene dada por:
a11 2

[M] = a 212
a a
21 11

a12

a 22

a 22 a12

cos 2

2
2a 21 a 22
= sin
a11 a 22 + a12 a 21 sin cos

sin 2

2a11 a12

cos
2

cos sin

2 cos sin

2 sin cos
cos 2 sin 2

Adems considerando que [M]1 = [N ]T , obtenemos {m} = [N ]T {m }, donde


a11 2
[N ] = a 212
2a a
21 11

a12

a 22

2a 22 a12

cos 2

2
a 21 a 22
= sin
a11 a 22 + a12 a 21 2 sin cos

sin 2

a11 a12

cos 2
2 cos sin

sin cos
cos 2 sin 2
cos sin

El mismo resultado (3.75) podra haber sido obtenido si considerbamos mij como un
tensor de segundo orden en el plano, y a travs de la ley de transformacin de un tensor de
segundo orden obtenemos que:
mij = aik a jl mkl
m
11

m12

(i, j = 1,2)

cos
m12
=
sin
m 22

sin m11
cos m12

m = Am A T

m12 cos
m22 sin

sin
cos

(3.76)

x3 = x3
x2

x3 =

t
2
x2

x3 =

Universidad de Castilla- La Mancha


Ciudad Real - Espaa

t
2

Draft

x1

x1

Por: Eduardo W. V. Chaves (2014)

3 TENSIONES

309

3.7 Tensiones en Coordenadas Cilndricas y Esfricas


Ejemplo 3.38
Demustrese que un cilindro cerrado de pared delgada de radio interno r y espesor t
sujeto a una presin interna p , ver Figura 3.29, tiene como estado tensional:
r = 0

pr
t

z =

pr
2t

NOTA: Las expresiones anteriores slo son vlidas para un cilindro de pared delgada.
x

Figura 3.29: Cilindro cerrado bajo presin.


Solucin:
Una vez adoptados los ejes de referencia de la Figura 3.29, planteamos el equilibrio de
fuerzas segn las direcciones z , y y r .
Equilibrio de fuerzas segn direccin z :

=0

p ( r ) = z ( 2 r )t

pA = p (r 2 )

pr
z =
2t

Equilibrio de fuerzas segn direccin y :

Fy = 0

2 ( Lt ) = p ( 2 rL )
=

pr
t

pA = p ( 2rL )
r

Universidad de Castilla- La Mancha


Ciudad Real - Espaa

Draft

Por: Eduardo W. V. Chaves (2014)

MECNICA DEL MEDIO CONTINUO: PROBLEMAS RESUELTOS

310

Equilibrio de fuerzas segn direccin r :


Podemos verificar que en la pared interna del cilindro la tensin radial ( r ) es igual a la
presin ( r = p ) y en la pared externa est libre de presin r = 0 , luego:
p r 0

para el caso

r = p

r <<
r
>> 1
t
r << z

r = 0

luego, r 0 cuando comparado con z , .

Experimentalmente se ha observado que la relacin para despreciar r es


situacin:
z =

pr
= 5p
2t

pr
= 10 p
t

r
10 . En esta
t

r = p

Podemos verificar tambin que la tensin es ms grande, > z , es decir, que para un
material homogneo, un cilindro rompera segn direccin de z , como se indica en la
figura siguiente:
Pero si el material estuviera
constituido
por
un
material
heterogneo, como por ejemplo,
matriz y fibras en la direccin de ,
la forma de rotura ya no estara tan
definida.

Observemos tambin que las


tensiones obtenidas anteriormente
para el cilindro de pared delgada no
sern vlidas si

r
< 10 . El error
t

cometido ya ser significativo.

Universidad de Castilla- La Mancha


Ciudad Real - Espaa

Draft

Por: Eduardo W. V. Chaves (2014)

3 TENSIONES

311

Ejemplo 3.39
Demustrese que una esfera cerrada de pared delgada de radio interno r y espesor t ,
sometida a una presin interna p , ver Figura 3.30, presenta el estado tensional siguiente:
r = 0

pr
2t

pr
2t

NOTA: Las expresiones anteriores slo son vlidas para una esfera de pared delgada.
x3 , z

e r

x3

x1

e
x2

x2 , y

x1 , x

Figura 3.30: Esfera sometida a presin interna.


Solucin:
Considerando los ejes adoptados en la Figura 3.30, planteamos el equilibrio de fuerzas
segn direccin x , y , r .
Equilibrio de fuerzas segn direccin x :
y

e r

pA = p ( r )

Universidad de Castilla- La Mancha


Ciudad Real - Espaa

=0

p ( r 2 ) + ( 2 r )t = 0
2

2r

pr
2t

Draft

Por: Eduardo W. V. Chaves (2014)

MECNICA DEL MEDIO CONTINUO: PROBLEMAS RESUELTOS

312

Equilibro de fuerzas segn direccin y :

y
pA = p ( r 2 )

e e

e r

=0

p ( r 2 ) + ( 2 r )t = 0
=

pr
2t

2r

Equilibrio de fuerzas segn direccin r :


Podemos verificar que en la pared interna de la esfera la tensin radial es igual a la presin
( r = p ) y que la pared externa est libre de presin ( r = 0 ), luego:
p r 0

Para el caso

r = p

r = 0

r <<
r
luego r 0 comparado con , .
>> 1
t
r <<

Experimentalmente se ha observado que la relacin para despreciar r es


situacin:

r
10 , en esta
t

pr

= 2t = 5 p

pr

= 5p
=
2t

r = p

Universidad de Castilla- La Mancha


Ciudad Real - Espaa

Draft

Por: Eduardo W. V. Chaves (2014)

4 Leyes Fundamentales de
la Mecnica del Medio
Continuo
Ejemplo 4.1
Demostrar el teorema del transporte de Reynolds partiendo de la siguiente expresin:
D
D
dV =
JdV 0
Dt V
Dt V

(4.1)

donde V es el volumen en la configuracin actual, V0 es el volumen en la configuracin de


referencia, J es el determinante del Jacobiano y es un campo escalar que describe una
cantidad fsica de una partcula en el espacio por unidad de volumen en un instante de
tiempo t .
Solucin:
r
r
D
DJ
D
D
D
JdV 0 = J
+
+ J xr v dV 0 =
+ xr v dV
dV 0 = J
Dt V
Dt
Dt
Dt
Dt

V0
V0
V
0

(4.2)
Ejemplo 4.2
r

Si PijL ( x , t ) representa alguna propiedad escalar, vectorial o tensorial cualquiera por unidad
de masa de un medio continuo, probar que la siguiente expresin es siempre vlida:
r
DPijL ( x , t )
r
D
PijL ( x , t ) dV =
dV
Dt V
Dt
V

(4.3)

Solucin:
Partiendo del Teorema del transporte de Reynolds:
D
r
r
r v p
D
( x , t )dV = ( x , t ) + ( x , t )
dV
Dt V
x p
Dt
V

Haciendo que = PijL y reemplazando en la ecuacin anterior, resulta:

Universidad de Castilla- La Mancha


Ciudad Real - Espaa

Draft

Por: Eduardo W. V. Chaves (2014)

MECNICA DEL MEDIO CONTINUO: PROBLEMAS RESUELTOS

314

D
Dt

D
v p
( PijK ) + PijK
dV
x p
Dt
V
D
v
D
=
+ PijK k dV
PijK + PijK
x k
Dt
Dt
V
v

D
D
=
+ k
PijK + PijK
x k
Dt
Dt
1
V
4243

PijK dV =

dV

=0
ecuacin de continuida d

Con lo que concluimos que:


DP
D
PijK dV = ijK dV
Dt V
Dt
V

Q.E.D.

Ejemplo 4.3
Probar que la siguiente relacin es vlida:
r
r r

( v ) + xr ( v v )
(4.4)
t
r
r r
r r
donde ( x , t ) es la densidad de masa, a ( x , t ) es la aceleracin Euleriana, y v ( x , t ) es la
r

a=

velocidad Euleriana.
Solucin:
Partiendo del teorema del transporte de Reynolds:
r
D

dV =
dV + (v n ) dS
Dt V
t
V
S

y considerando que = v obtenemos que:


r
r
r
r
( v )
D
v dV =
dV + v (v n ) dS
Dt V
t
V
S

en notacin indicial queda:


( v i )
D
vi dV =
dV + v i (v k n k ) dS
t
Dt V
V
S

v dV =
1
Dt
23
i

= ai

( v i )
dV + ( v i v k )n k dS
t
S

Aplicando el teorema de la divergencia para la integral de superficie obtenemos que:


( v i )
( v i )

+ ( v i v k ) ,k dV
dV + ( vi v k ) ,k dV
ai dV =
ai dV =
V

En notacin tensorial:

r
r r
( v )
V a dV = V t + xr ( v v ) dV

r
r ( v )
r r
a=
+ xr ( v v )
t

Q.E.D.

Ejemplo 4.4
Para un campo de velocidad dado por:
vi =

xi
1+ t

para t 0

a) Encontrar la densidad de masa de una partcula en funcin del tiempo;


Universidad de Castilla- La Mancha
Ciudad Real - Espaa

Draft

Por: Eduardo W. V. Chaves (2014)

4 LEYES FUNDAMENTALES DE LA MECNICA DEL MEDIO CONTINUO

315

b) Probar que para el movimiento dado se cumple que x1 x 2 x3 = 0 X 1 X 2 X 3 .


Solucin:
a Por la conservacin de masa sabemos que:
v
v
D
D d
+ k =0

=
= k
x k

Dt

Dt

Utilizando el campo de velocidad dado hallamos que:


v i
ii
1 x i
x i

luego

1 + t x i

d
3
=
dt
1+ t

1+ t
d

x k

dt

3
1+ t

3dt
1+ t

Integrando la relacin anterior obtenemos que:


d
3dt
= 1 + t Ln = 3 Ln(1 + t ) + C1
La constante de integracin C1 se obtiene con la condicin inicial t = 0 donde se cumple
que la densidad de masa es igual a la densidad de masa de referencia ( = 0 ):
Ln 0 = 3 Ln(1 + 0) + C1 C1 = Ln 0
1
0
0
+ ln 0 = Ln

Ln = 3 Ln(1 + t ) + Ln 0 = Ln

=
3
3
3

(1 + t )

(1 + t )

(1 + t )

b) Utilizando la definicin de velocidad y aplicando el campo de velocidad obtenemos:


vi =

dx i
x
= i
dt 1 + t

dx i
dt
=
xi 1 + t

Integrando la ecuacin anterior resulta:

dx i
dt
=
xi
1+ t

Integrando

Lnx i = Ln(1 + t ) + C1

(4.5)

Aplicando la condicin inicial, donde se cumple que: t = 0 xi = X i , luego:


Ln X i = Ln(1 + 0) + C1

C1 = Ln X i

Reemplazando el valor de C1 en la ecuacin (4.5) obtenemos:


Ln x i = Ln(1 + t ) + Ln X i

con lo que concluimos que:

Ln( x i ) = Ln[ X i (1 + t )]

xi = X i (1 + t )

Expandiendo la relacin anterior:

y considerando que =

x1
x1 = X 1 (1 + t ) (1 + t ) =
X
1

x2
x 2 = X 2 (1 + t ) (1 + t ) =
X
2

x
x 3 = X 3 (1 + t ) (1 + t ) = 3

X3

(ver apartado 1 de este ejemplo), obtenemos:

(1 + t )3
(1
12
+3
t )(1
12
+3
t )(1
12
+3
t) = 0
x1
X1

x2
X2

Universidad de Castilla- La Mancha


Ciudad Real - Espaa

x3
X3

x1 x 2 x 3 = 0 X 1 X 2 X 3
Q.E.D.

Draft

Por: Eduardo W. V. Chaves (2014)

MECNICA DEL MEDIO CONTINUO: PROBLEMAS RESUELTOS

316

Ejemplo 4.5

Considrese un medio continuo y una propiedad ( x , t ) asignada por la densidad, i.e.


unidad de la propiedad por unidad de volumen. Obtener la tasa de esta propiedad de tal
forma que venga descrita por una integral de volumen de control conjuntamente con una
integral de superficie de control.
Solucin:
Recordar que la tasa de una propiedad est relacionada siempre con las mismas partculas.
A travs de la Derivada Material podemos obtener la tasa de una propiedad cuando sta
r
est en la descripcin espacial ( x , t ) . Luego, la variacin total de la propiedad en volumen
V que est delimitado por la superficie S viene dada por:
r
r
r
D
D
D
D

( x, t )dV =
( x, t ) + ( x, t ) (dV )
( dV ) = dV
Dt V
Dt
Dt
Dt

V
V

r
r
r
D

( x, t ) + ( x, t ) xr v dV
= dV
Dt

(4.6)

r
r
r
D
= ( x , t ) + ( x , t ) xr v dV
Dt

Podemos aplicar la definicin de derivada material a la ecuacin anterior, resultando que:


r
r
r
r
D
D
( x, t )dV = ( x, t ) + ( x, t ) xr v dV
Dt V
Dt

V
r
r
r
r
( x , t ) r r

= ( x, t ) +
r v ( x , t ) + ( x , t ) xr v dV
x
t

V
r
r
r
r
( x , t ) r

= ( x , t ) dV +
r v + ( x , t ) xr v dV
x
t

V
V

(4.7)

r
r

= ( x , t ) dV + [ xr (v ) ]dV
t

V
V

Podemos aplicar el teorema de la divergencia a la segunda integral del lado derecho de la


igualdad y obtener que:
flujo de a travs

de la superficie S
6
4
4744
8
r
r
r
( x , t )
D
( x, t )dV =
dV + ({
v ) n dS
Dt V

t
1
4
2
4
3
V
S flujo de

(4.8)

local

r
( x , t )
como el trmino
es local, la integral de volumen de la derecha de la igualdad es un
t

volumen de control y la integral de superficie es una superficie de control ya que las


r
r
variables (v ) estn en las descripcin Euleriana. El trmino (v ) representa el flujo de la
propiedad que atraviesa la superficie S , ver Figura 4.1. Cuando no hay fuente o

sumidero de la propiedad se cumple que

r
D
( x, t )dV = 0 .
Dt V

NOTA: Verificar que cuando la propiedad es la densidad de masa ( = ) la ecuacin (4.8)


se denomina ecuacin de continuidad de masa:
Universidad de Castilla- La Mancha
Ciudad Real - Espaa

Draft

Por: Eduardo W. V. Chaves (2014)

4 LEYES FUNDAMENTALES DE LA MECNICA DEL MEDIO CONTINUO

r
r
r
r
D
D
( x, t )dV = ( x, t ) + ( x, t ) xr v
Dt V
Dt
V

r
r

r
dV = t ( x , t ) + x ( v ) dV = 0

317

(4.9)

Si la ecuacin anterior es vlida para todo el volumen tambin lo ser localmente, i.e.:
r
r
r
D
( x, t ) + ( x, t ) xr v = 0 Ecuacin de continuidad de masa
Dt

(4.10)

r
r

( x, t ) + xr ( v ) = 0
t

Ecuacin de continuidad de masa

volumen de control
S

r
(v )

r
r
q n = [(v ) n ] n

r
( x , t )
t

r
x

(4.11)

superficie de control

Figura 4.1: Volumen y superficie de control.


Ejemplo 4.6
Demostrar que
r
r

+ ( xr ) v = ( ) + xr ( v )
t
t

+ , i vi = ( ) + ( vi ) ,i

t
t

(4.12)
r

donde es un campo escalar, es el campo de densidad de masa, y v es el campo de


velocidad.
Solucin:

+ , i ( v i ) + ( v i ) ,i
+
( ) + (v i ) ,i =
t
t
t

=
+ , i v i +
+ ( v i ) , i =
+ , i v i
t
t
t

donde hemos considerado la ecuacin de continuidad de masa

Universidad de Castilla- La Mancha


Ciudad Real - Espaa

Draft

+ ( vi ) ,i = 0 .
t

Por: Eduardo W. V. Chaves (2014)

MECNICA DEL MEDIO CONTINUO: PROBLEMAS RESUELTOS

318

Volumen material

Volumen de control

Superficie de control

t=0
v0

XP

Partcula P

X*

Volumen de control

Volumen material

Superficie de control

t1

r
v( x * , t1 )

xP

Partcula Q

x*

Volumen material

Volumen de control

Superficie de control

t2

r
v( x * , t 2 )
xP

Partcula R

x*

Figura 4.2: Volumen material vs. volumen y superficie de control.


Ejemplo 4.7
Las componentes del campo del tensor de tensiones de un medio continuo en equilibrio
vienen dadas por:
11 = x12 ;

22 = x 22 ;

12 = 21 = 2 x1 x 2 ;

33 = x12 + x 22
23 = 32 = 31 = 13 = 0

Encontrar la fuerza msica (por unidad de volumen) que acta en el continuo.


Solucin: Aplicando las ecuaciones de equilibrio podemos obtener que:
r r
xr + b = 0
11 12 13
+
+ b1 = 0
+

x 2
x 3
x1
21 22 23
+
+ b2 = 0
+

x 2
x 3
x1
31 32 33
+ b3 = 0
+
+

x1
x 3
x 2

2 x1 + 2 x1 + b 1 = 0

2 x 2 + 2 x 2 + b 2 = 0
b = 0
3

Con lo que concluimos que para satisfacer las ecuaciones de equilibrio hay que cumplir que:
Universidad de Castilla- La Mancha
Ciudad Real - Espaa

Draft

Por: Eduardo W. V. Chaves (2014)

4 LEYES FUNDAMENTALES DE LA MECNICA DEL MEDIO CONTINUO

4 x1 = b 1 b 1 = 4 x1

4 x 2 = b 2 b 2 = 4 x 2

b 3 = 0

319

b = 4( x1 e 1 + x 2 e 2 )

Ejemplo 4.8
Sea el movimiento del continuo dado por las siguientes ecuaciones:
x1 = X 1 + tX 3

x 2 = X 2 + tX 3
x = X t ( X + X )
3
1
2
3
donde es una constante. Encontrar la densidad de masa en la configuracin actual ( )
en funcin de la densidad de masa en la configuracin de referencia ( 0 ) .
Solucin:
Queremos encontrar una funcin densidad de masa tal que: = ( 0 ) . Sabemos que la
ecuacin de continuidad de masa en la forma Lagrangiana viene dada por 0 = J , donde
el determinante del Jacobiano puede ser obtenido directamente de las ecuaciones de
movimiento:

x i
J= F =
X j

x1
X 1
x 2
=
X 1
x 3
X 1

x1
X 2
x 2
X 2
x 3
X 2

Resultando as:

x1
X 3
1
x 2
= 0
X 3
t
x 3
X 3

0
J

t
t = 1 + 2( t ) 2

0
1
t

0
1 + 2( t ) 2

Ejemplo 4.9
Dado el campo de velocidad:
v1 = x1 x3

v 2 = x 22 t

v3 = x 2 x 3t

y el campo de tensiones:
x 2 x1
ij = x 2 x3
0

x 2 x3
x 22

0
x 2
x32

x2

donde es una constante. Encontrar las fuerzas msicas (por unidad de volumen) de tal
forma que cumpla con el principio de la conservacin del momento lineal.
Solucin:
Del principio de la conservacin del momento lineal obtenemos las ecuaciones del
movimiento:
r
r
r
xr + b = v& = a

b = a xr

El campo de aceleracin:

Universidad de Castilla- La Mancha


Ciudad Real - Espaa

Draft

Por: Eduardo W. V. Chaves (2014)

MECNICA DEL MEDIO CONTINUO: PROBLEMAS RESUELTOS

320

r r
r r
r v ( x , t ) v ( x , t ) r r
a=
+
r v ( x, t )
t
x

ai =

v i v i
+
vj
t
x j

donde
0
v i 2
= x2
t
x 2 x3

x3
v i
= 0
x j
0

0
2x2 t
x3t

x1
0
x 2 t

Luego:
0 x3
v i v i
ai =
v j = x 22 + 0
+
t x j
x 2 x 3 0

x1 x1 x 3 0 x1 x 32 + x1 x 2 x 3 t

0 x 22 t = x 22 +
2 x 23 t

x 2 t x 2 x 3 t x 2 x 3 x 3 x 22 t 2 + x 22 x 3 t 2

0
2x2t
x3t

x1 x 32 + x1 x 2 x 3 t

2
3
=
x2 + 2x2 t

x 2 x 3 + x 3 x 22 t 2 + x 22 x 3 t 2

La divergencia del tensor de tensiones de Cauchy viene dada por:


11 12 13
+
+
= ( x 2 x3 )

x 2
x 3
x1

22 23
ij , j = 21 +
+
= (2 x 2 )

x 2
x 3
x1
31 32 33
+
+
= ( 2 x 3 1)

x 2
x 3
x1
r
r
Con lo cual, las fuerzas msicas, b = a xr , quedan:

b i = a i ij , j

ij , j

x 2 x3
= 2 x 2
2 x 3 1

x1 x 32 + x1 x 2 x 3 t
x 2 x3

2
3
x2 + 2x2 t
b i =
2x2
x 2 x 3 + x 3 x 22 t 2 + x 22 x 3 t 2
2 x 3 1

Ejemplo 4.10
Teniendo en cuenta el Principio de la conservacin del momento angular, demostrar que:

[r

( x (a b) (a b) x dV =

[ ( xr tr

r
r
t * x dS

r r

r r

donde: x - vector posicin; ( x , t ) -densidad de masa; a ( x , t ) -aceleracin; b( x , t ) -fuerzas


r r
msicas (por unidad de masa); t * ( x , t ) - vector traccin prescrito (fuerza de superficie) en la
superficie S .
Solucin:
El principio de la conservacin del momento angular establece que:

r
r
r
r
r
r r
r
D
( x t * )dS + ( x b)dV =
( x v )dV = ( x a )dV
Dt V
V
V

Universidad de Castilla- La Mancha


Ciudad Real - Espaa

Draft

Por: Eduardo W. V. Chaves (2014)

4 LEYES FUNDAMENTALES DE LA MECNICA DEL MEDIO CONTINUO

321

A continuacin hacemos el producto vectorial de la expresin anterior con un vector


r
r
arbitrario z , independiente de x , resultando:
r
r
r
r
z ( x a )dV = z

r
r r
r
r
( x t * )dS + z ( x b)dV

r
r
r
z ( x a )dV =

r
r r
r
r
r
z ( x t * )dS + z ( x b)dV

Se ha demostrado en el captulo 1 que dados tres vectores a , b , c , se cumple que


r r
r r r r r
r
a (b c ) = (b c c b) a . Luego, la expresin anterior puede ser reescrita como:

r
r r r
r
r
r
r r r
r r
r r
r
( x a a x ) z dV = ( x t * t * x ) z dS + ( x b b x ) z dV

r
r r r r r
r r r r r
r r
r r
( x a a x ) z dV ( x b b x ) z dV = ( x t * t * x ) z dS

r
r
r r
r r
r r
r r
r r
x (a b) (a b) x z dV = ( x t * t * x ) z dS

r
r

r r r
r
r r
r r
r
r
x (a b) (a b) x dV z = ( x t * t * x ) dS z
V

r
Como el vector z es arbitrario, concluimos que:
r
r
r r
r r
r
r r
r
x (a b) (a b) x dV = ( x t * t * x ) dS

Ejemplo 4.11
1) Teniendo en cuenta la definicin del tensor tensin media ( ) de un medio continuo:

V = dV
V

Y partiendo del principio de que el continuo est en equilibrio esttico, demostrar que se
cumple que:
=

1
2V

[x b + b x ] dV + 2V ( x t
r

r*

r
r
+ t * x ) dS

2) Teniendo en cuenta ahora que el volumen del continuo viene dado por V = V (1) V ( 2) ,
ver Figura 4.3, y que dicho continuo est sometido a una presin p (1) en la superficie S (1) ,
y una presin p ( 2) en la superficie S ( 2) . Considerando que el continuo est libre de fuerzas
msicas, verificar que se cumple la relacin:
=

(V

(1)

1
( p (1)V (1) p ( 2)V ( 2 ) )1
V ( 2) )

Solucin:

Teniendo en cuenta la ecuacin de equilibrio xr + b = a = 0 (principio de la


conservacin del momento lineal). Para todo el continuo hay que cumplir que:

Universidad de Castilla- La Mancha


Ciudad Real - Espaa

Draft

Por: Eduardo W. V. Chaves (2014)

MECNICA DEL MEDIO CONTINUO: PROBLEMAS RESUELTOS

322

x [
r

r
x

+ b]
r

r
dV = 0

r
x

r
r
r
dV + x b dV = 0

(4.13)

n (1)
S (1)

p (1)

V (1)
S (2)

V ( 2)

n ( 2 )
p (2)

Figura 4.3
En el captulo 1, ver Ejemplo 1.125, hemos demostrado que se cumplen las siguientes
relaciones:
r

r*

( ) x dV = ( n ) x dS dV = t

x ( ) dV = x ( n ) dS

r
x dS dV

(4.14)

r r
dV = x t * dS T dV

(4.15)

donde hemos tenido en cuenta que el vector tensin prescrito, t * = n . Reemplazando


(4.15) en la expresin (4.13), obtenemos que:
r

r
x

r
r
r
dV + x b dV = 0

r*

xt
S

r
r
r
dS T dV + x b dV = 0

(4.16)

r
r r
r
T dV = x t * dS + x b dV

Tambin se cumple que:

r r
r
r
dV = t * x dS + b x dV

(4.17)

Observemos que los tensores resultantes de las operaciones x t * , x b , no son


tensores simtricos. Esto quiere decir que en la expresin (4.13) no ha tenido en cuenta el

Universidad de Castilla- La Mancha


Ciudad Real - Espaa

Draft

Por: Eduardo W. V. Chaves (2014)

4 LEYES FUNDAMENTALES DE LA MECNICA DEL MEDIO CONTINUO

323

principio de la conservacin del momento angular (simetra del tensor de tensiones de


Cauchy). Para garantizar que sea simtrico, hacemos que:
r r
r
1 r r

r
r
+ T
1 r
dV = t * x dS + b x dV + x t * dS + x b dV
2
2 S
2 S

V
V
V

r r r r
1
1 r r* r* r
sym dV =
x b + b x dV +
x t + t x dS
2V
2S
V

(4.18)

Teniendo en cuenta la definicin del tensor tensin media, concluimos que:


r r
r

1 r r
r
r
+ T
1 r
dV = t * x dS + b x dV + x t * dS + x b dV
2
2 S

2 S
V
V
V

r r r r
1
1 r r* r* r
x b + b x dV +
sym dV =
x t + t x dS
2
2
V
V
S

(4.19)

r r r r
1
1 r r* r* r
x b + b x dV +
V =
x t + t x dS
2V
2S

1
2V

[r

x b + b x dV +

1
2V

[ xr tr

r
r
+ t * x dS

Si adems el cuerpo est libre de fuerzas msicas, la expresin anterior se resume a:


=

1
2V

[ xr tr

r
r
+ t * x dS

(4.20)

Para el caso particular de la Figura 4.3 tenemos que V = V (1) V ( 2) , S = S (1) + S ( 2) ,


r (1)
r ( 2)
t * = p (1) n (1) , t * = p ( 2) n ( 2) . En este caso, la expresin (4.20) queda:
=

2(V

(1)

r r

r
r
r
r r
r
1
x t * + t * x dS (1) + x t * + t * x dS ( 2 )
( 2)
V ) S (1)

S ( 2)

r
r
r
1
(1) r
(1) + n (1) x dS (1) + p ( 2 ) x n ( 2) + n ( 2 ) x dS ( 2 )

x
p
n

2(V (1) V ( 2 ) ) S (1)

S (2)

r (1) (1) r
r ( 2) (2) r
1
(1)
(1)
( 2)
( 2)
x
x
x
x
n
n
n
n

p
dS
p
dS

2(V (1) V ( 2 ) )

S ( 1)
S ( 2)

Hemos demostrado en el captulo 1 que se cumple ( x n + n x ) dS = 2V 1 , donde n es


S

el versor normal exterior a la superficie S (ver Ejemplo 1.125). Para este ejemplo n ( 2) es
normal interior a la superficie
S (2) ,
debido a eso tenemos que

[x n
r

( 2)

r
+ n ( 2 ) x dS ( 2 ) = 2V ( 2 ) 1 , resultando que:

S (2)

Universidad de Castilla- La Mancha


Ciudad Real - Espaa

Draft

Por: Eduardo W. V. Chaves (2014)

MECNICA DEL MEDIO CONTINUO: PROBLEMAS RESUELTOS

324

r
r
r
r
(1)
(1)
(1)
(1)
( 2)
( 2)
(2)
( 2)

x
x
x
x
p

n
+
n

dS
+
p

n
+
n

dS

2(V (1) V ( 2 ) )

S (1 )
S (2)
1
1
=
p (1) 2V (1) 1 p ( 2) 2V ( 2 ) 1 = (1)
p (1)V (1) p ( 2 )V ( 2 ) 1
(1)
( 2)
( 2)
2(V V )
(V V )

Ejemplo 4.12
Dado el campo de velocidad:
v1 = ax1 bx 2

v 2 = bx1 ax 2

v 3 = c x12 + x 22

donde a , b y c son constantes.


a) Comprobar si se cumple la ecuacin de continuidad de masa;
b) Es un movimiento isocrico?, es decir, es un medio incompresible?
Solucin:
Ecuacin de continuidad de masa:
r
D
+ ( xr v ) = 0
Dt

donde xr v = v i ,i = v1,1 + v 2, 2 + v 3,3 = a a + 0 = 0 . El movimiento es isocrico, ya que


r
xr v = 0 .
Ejemplo 4.13

Partiendo de la ecuacin de energa u& = : D xr q + r , demostrar que la ecuacin de


energa puede ser escrita como:

r r
r
r
D
1 2
u + v = xr (v ) + b v xr q + r
Dt
2

(4.21)

D
1 2

u + v = (v j ji ) ,i + b i v i q i ,i + r
Dt
2

r r
r
r

1 2
1 2 r
u + v + xr u + v v = xr (v ) + b v xr q + r
t
2
2

1
1
u + v 2 + u + v 2 vi = (v j ji ) ,i + b i vi qi ,i + r
t
2
2 ,i

(4.22)

r r
r
r

1 2
1 2 r

u + v + xr u + v v = xr (v ) + b v xr q + r
2
2
t


1 2
1 2
u + 2 v + u + 2 v vi = (v j ji ) ,i + b i vi qi ,i + r
,i

(4.23)

donde es la densidad de masa, u es la energa interna especfica, v es el mdulo de la


r

r r

velocidad ( v 2 = v = v v ), es el tenor de tensiones de Cauchy, b es la fuerza msica


r
(por unidad de masa), q es vector flujo de calor, r es la fuente de calor.
2

Universidad de Castilla- La Mancha


Ciudad Real - Espaa

Draft

Por: Eduardo W. V. Chaves (2014)

4 LEYES FUNDAMENTALES DE LA MECNICA DEL MEDIO CONTINUO

325

Solucin:
Teniendo en cuenta la ecuacin de energa:
r

u& = : D xr q + r

u& = ijDij qi ,i + r

donde D es el tensor tasa de deformacin el cual es igual a la parte simtrica del gradiente
r
espacial de la velocidad ( D = ( xr v ) sym l sym ). Notar tambin que : D = : l sym = : l
ya que el doble producto escalar entre un tensor simtrico ( = T ) y un antisimtrico
( l anti ) resultar ser cero, i.e. : l anti = 0 , luego
r
ij D ij = ij ( l ) ij = ij ( xr v ) ij = ij vi , j

Notar que ( ij vi ) , j = ij , j vi + ij vi , j ij vi , j = ( ij vi ) , j ij , j vi = ijD ij , con eso la ecuacin


de energa puede ser reescrita como:
r

u& = : D xr q + r
r
r
r
u& = xr (v ) ( xr ) v xr q + r

u& = ij D ij qi ,i + r
u& = ( ij vi ) , j ij , j vi qi ,i + r

Teniendo en cuenta las ecuaciones de movimiento podemos obtener que:


r
&r& = vr&
xr + b = u
r
r
xr = v& b
r r
r
r r
( xr ) v = v& v b v

&&i = v&i
ij , j + b i = u
ij , j = v&i b i
ij , j vi = v&i vi b i vi

Con lo cual la ecuacin de energa queda:


r

u& = xr (v ) ( xr ) v xr q + r

r r
r
r
r r
u& = xr (v ) ( v& v b v ) xr q + r
r r
r
r r
r
u& + v& v = xr (v ) + b v xr q + r

Notar que

u& = ijD ij qi ,i + r
u& = ( ij vi ) , j ( v&i vi b i vi ) qi ,i + r
u& + v&i vi = ( ij vi ) , j + b i vi qi ,i + r

r r
r r
r r
r r
D r r
1 D r r
1 D 2
(v v ) = (v& v ) + (v v& ) = 2(v& v ) (v& v ) =
(v v ) =
(v ) . Luego, la
Dt
2 Dt
2 Dt

ecuacin de energa puede tambin ser reescrita como

r r
r
r r
r
u& + v& v = xr (v ) + b v xr q + r
r r
r
r
1 D 2

(v ) = xr (v ) + b v xr q + r
u& +
2 Dt

r r
r
r
1 2
D

u + v = xr (v ) + b v xr q + r
2
Dt

cuya ecuacin en notacin indicial queda


u& + v&i vi = ( ij vi ) , j + b i vi qi ,i + r
Du 1 D 2

+
(v ) = ( ij vi ) , j + b i vi qi ,i + r
Dt 2 Dt

D
1 2

u + v = ( ij vi ) , j + b i vi qi ,i + r
Dt
2

demostrando as la ecuacin (4.21). La ecuacin (4.22) puede ser fcilmente demostrada si


consideramos la derivada material

Universidad de Castilla- La Mancha


Ciudad Real - Espaa

r
D

+ ( xr ) v , i.e.:
& =
Dt
t

Draft

Por: Eduardo W. V. Chaves (2014)

326

MECNICA DEL MEDIO CONTINUO: PROBLEMAS RESUELTOS

r r
r
r
D
1 2
u + v = xr (v ) + b v xr q + r
Dt
2
r r
r
r

1
1 r

u + v 2 + xr u + v 2 v = xr (v ) + b v xr q + r
2
2
t

En el Ejemplo 4.6 hemos demostrado que

r
r

+ ( xr ) v = ( ) + xr ( v ) se
t
t

cumple, y si consideramos que = u + v 2 demostramos as le ecuacin (4.23).

1
2

4.1 Introduccin a Problemas de Flujo


Ejemplo 4.14
Obtener la ecuacin de energa para un movimiento de slido rgido. Considrese que el
r
flujo de calor viene dado por q = K (T ) xr T , donde K (T ) es un tensor de segundo orden
denominado de tensor de conductividad trmica (propiedad del material), y considrese
u
, donde c es el calor especfico (propiedad del material). Proporcionar
T
tambin la unidad del tensor K en el SI.

tambin que c =

Solucin: Para un movimiento de cuerpo rgido la potencia tensional es igual a cero, luego, la
ecuacin de energa se resume a:
r
r
u T
=
:3
D xr q + r = xr q + r
1
2
T t
=0
r
T
c
= xr q + r
t
T
c
= xr [ K (T ) xr T ] + r
t

u& =

Resultando as que:
xr [K (T ) xr T ] + r = c

T
t

La ecuacin anterior se conoce como ecuacin de flujo de calor que se aplica a problema de
conduccin trmica.
DT T
.
NOTA 1: Cuando no hay transporte de masa se cumple T&
=
Dt

Teniendo en cuenta las siguientes unidades: [q] =


r

J
W
T K
= 2 , xr T = r = , podemos
2
x m
m s m

verificar que para que haya compatibilidad de unidades hay que cumplir que:

[qr ]

[K ] [ xr T ]

W
W K
J
J
m 2 s = m 2 = s m K = m K m

W
J
.
=
s
m
K
m
K

Luego, concluimos que [K ] =

Universidad de Castilla- La Mancha


Ciudad Real - Espaa

Draft

Por: Eduardo W. V. Chaves (2014)

4 LEYES FUNDAMENTALES DE LA MECNICA DEL MEDIO CONTINUO

327

NOTA 2: Tambin es interesante destacar que debido a que la potencia tensional es nula,
podemos desacoplar el problema mecnico del problema trmico, es decir, podemos
analizar los dos problemas separadamente.
Ejemplo 4.15
Consideremos un problema de conduccin trmica, ver Ejemplo 4.14, donde tenemos una
pared de espesor h en la cual la temperatura en la cara exterior ( x1 = 0 ) es igual a 38 C y
la temperatura en la cara interior ( x1 = h ) es igual a 21 C , ver Figura 4.4. Obtener el flujo
de calor para el caso definido por: problema estacionario, el campo de temperatura segn
las direcciones x 2 y x3 es homogneo, no hay fuente interna de calor, y el material es
homogneo e istropo.
x2
T ( A) = 38 C

Datos:
h = 0,04m
T ( B ) = 21 C

(Exterior)

K = 0,19

W
mK

(Interior)
x1

r
q
h

Figura 4.4
OBS.: Cuando decimos que un problema es estacionario, nos referimos que el campo de la
incgnita es estacionario. En el caso de la ecuacin de flujo de calor, la incgnita es la
temperatura.
Solucin:
Como hemos visto en el Ejemplo 4.14 la ecuacin de gobierno para este problema es la
ecuacin xr [K xr T ] + r = c

T
. Si consideramos el problema estacionario, tenemos
t

T
= 0 . Si no hay fuente interna de calor esto implica que r = 0 . Con estas
t
simplificaciones la ecuacin de flujo de calor se resume a xr [K xr T ] = 0 , adems si el
r
material es homogneo, el tensor con las propiedades trmicas K no vara con x
resultando
que
xr [K xr T ] = K : xr [ xr T ] = 0
o
en
notacin
indicial

que

[K

ij T , j

],

= K ij ,i T , j + K ij T , ji = K ij T , ji = 0 . Expandiendo esta ltima expresin, obtenemos


123
=0

que:

Universidad de Castilla- La Mancha


Ciudad Real - Espaa

Draft

Por: Eduardo W. V. Chaves (2014)

MECNICA DEL MEDIO CONTINUO: PROBLEMAS RESUELTOS

328

K 11

2T
2T
2T
2T
2T
2T
+
K
+
K
+
K
+
K
K
+
+
12
13
21
22
23
x 2 x1
x3 x1
x1x 2
x3 x 2
x12
x 22
+ K 31

2T
2T
2T
+ K 33 2 = 0
+ K 32
x 2 x3
x1x3
x3

(4.24)

Si el campo de temperatura segn las direcciones x 2 y x3 es homogneo, eso implica que


las componentes del gradiente de temperatura segn estas direcciones son iguales a cero, i.e.
T
T
=
= 0 . Para un material istropo, las componentes del tensor de conductividad
x 2 x3

trmica, ver Captulo 5 en Chaves (2007), vienen dadas por:


K 0 0
K ij = 0 K 0
0 0 K

Con estas consideraciones la expresin (4.24) se reduce a:


K 11

2T
=0
x12

11 =K
K

Integrando una vez la expresin K


K

2T
=0
x12

2T
=0
x12

(4.25)

2T
= 0 obtenemos que:
x12

integrando

T
+ q1 = 0
x1

q1 = K

dT
dx1

Que es la ecuacin de flujo de calor de Fourier. Observar que para este caso q1 es una
constate, es decir, es independiente de x1 . Integrando una vez ms:

dT =

q1
dx1
K

T ( x1 ) =

q1
x1 + C
K

Aplicando la condicin de contorno, x1 = 0 T = T ( A) , obteniendo as la constante de


integracin C = T ( A) . Resultando que T ( x1 ) =

q1
x1 + T ( A) . Adems para x1 = h tenemos
K

que
T ( x1 = h) = T ( B ) =

q1
h + T ( A)
K

q1 = K

(T ( B ) T ( A) )
h

Notar que en este caso (unidimensional) el gradiente de temperatura es la pendiente de la


recta definida por la temperatura, y la temperatura vara linealmente en la pared, ver Figura
4.4.
Reemplazando los datos del problema, ver Figura 4.4, obtenemos el flujo:
q1 = K

(T ( B ) T ( A) )
W
J
W (21 38)( K )
= 0,19
= 80,75 2 = 80,75 2

h
m
m s
mK 0,04(m)

Observar que la transformacin de temperatura de grados Celsius para Kelvin viene dada
por K = C + 273,15 , luego la diferencia de temperatura sea en grados Celsius o en Kelvin
ser la misma. Observar tambin que el flujo de calor va en el sentido de mayor
temperatura hacia la regin de menor temperatura.

Universidad de Castilla- La Mancha


Ciudad Real - Espaa

Draft

Por: Eduardo W. V. Chaves (2014)

4 LEYES FUNDAMENTALES DE LA MECNICA DEL MEDIO CONTINUO

329

NOTA: Supongamos que ahora tengamos dos paredes con propiedades distintas tal y
como se muestra en la Figura 4.5.
T ( A)
T (B )

K (1)

K ( 2)

T (C )

r
q
x1

h (1)

h (2)

Figura 4.5
(T ( B ) T ( A) )
sigue siendo vlida. Tambin es
h (1)
(T (C ) T ( B ) )
vlido para el material 2 : q1 = K ( 2)
. Para obtener el flujo tenemos que
h (2)
aplicar la compatibilidad de temperatura en la cara B , es decir:

Observemos que la expresin q1 = K (1)

(T ( B _ 1) T ( A) )
h (1)
(T (C ) T ( B _ 2 ) )
q1 = K ( 2)
h ( 2)

q1 = K (1)

q1 h (1)

T ( B _ 1) = T ( A)

T ( B _ 2) = T (C ) +

K (1)
q1 h ( 2)
K ( 2)

T ( B _ 1) = T ( B _ 2 )
T ( A)

q1 h (1)
K (1)

= T (C ) +

q1 h ( 2 )
K ( 2)

Resultando que:
q1 =

(T (C ) T ( A) )
h (1) h ( 2 )

K (1) + K ( 2 )

Ejemplo 4.16
Considrese un problema de conduccin trmica estacionario sin fuente interna de calor y
r
r
donde el flujo de calor viene gobernado por la ley de Fourier ( q = K ( x ) xr T ), donde el
r
campo de tensor de conductividad trmica viene representado por K ( x ) , y es un tensor de
segundo orden arbitrario (no necesariamente simtrico).
a) Demostrar que el tensor de conductividad trmica es un tensor semi-definido positivo;
r

b) Verificar en que situacin la parte antisimtrica de K ( x ) no afecta en el resultado del


problema de conduccin de calor. Considerar la potencial tensional igual a cero.

Universidad de Castilla- La Mancha


Ciudad Real - Espaa

Draft

Por: Eduardo W. V. Chaves (2014)

MECNICA DEL MEDIO CONTINUO: PROBLEMAS RESUELTOS

330

c) Que formato presenta K si el material es istropo.


Solucin:
a) Partimos de la desigualdad de conduccin de calor que hay que cumplir siempre:
r
q xr T 0

q i T,i 0

r
(K ( x ) xr T ) xr T 0
r
xr T K ( x ) xr T 0

( K ij T, j )T,i 0
T,i K ij T, j 0

Recordar que un tensor arbitrario A es semi-definido positivo si se cumple que


r
r
r r
r
x A x 0 para todo x 0 . Demostrando as que K ( x ) es un tensor semi-definido
r
positivo. Luego, como consecuencia los autovalores de K ( x ) sern todos valores reales
r
mayores o iguales a cero, i.e. K 1 0 , K 2 0 , K 3 0 . Tambin recordar que, ya que K ( x )
r
no es simtrico, el espacio principal de K ( x ) no constituye una base ortonormal.
Es interesante constatar que la parte antisimtrica no afecta en la desigualdad de
conduccin de calor, ya que:

r
xr T K ( x ) xr T 0

xr T K sym + K anti xr T 0
xr T K

sym

xr T + xr T K

anti

xr T 0

xr T K sym xr T + K anti : ( xr T xr T ) 0

Note que K anti : ( xr T xr T ) = 0 ya que el doble producto escalar entre un tensor


antisimtrico ( K anti ) y otro simtrico ( xr T xr T ) resulta ser igual a cero, luego:
r
0 xr T K ( x ) xr T = xr T K sym xr T 0
r

Es decir, siempre se cumplir la desigualdad de conduccin de calor, sea K ( x ) simtrico o


no.
b) Para el problema planteado, la nica ecuacin de gobierno que queda es la ecuacin de
energa:

r
r
Du
u& = : D xr q + r = xr q
Dt

donde u es la energa interna especfica, : D es la potencial tensional, y r es la fuente


interna de calor por unidad de volumen. Luego:

u& = qi ,i = (K ijT, j ),i = K ij ,iT, j + K ijT, ji


= ( xr K T ) ( xr T ) + K : xr ( xrT )

= ( xr K T ) ( xr T ) + K sym + K anti : xr ( xr T )
= ( xr K T ) ( xr T ) + K sym : xr ( xrT ) + K anti : xr ( xrT )
= ( xr K T ) ( xr T ) + K sym : xr ( xrT )

donde hemos considerado la simetra de

[ xr ( xr T )]ij = T,ij = T, ji .

Si el material es

r
homogneo eso implica que el campo de K no depende de ( x ) con lo cual el trmino
K ij ,i = 0 j . En esta situacin quedamos con:

u& = K sym : xr ( xr T )
Universidad de Castilla- La Mancha
Ciudad Real - Espaa

Draft

Por: Eduardo W. V. Chaves (2014)

4 LEYES FUNDAMENTALES DE LA MECNICA DEL MEDIO CONTINUO

331

Luego, cuando el material es homogneo la parte antisimtrica de K no afecta en el


resultado.
c) Un material istropo tiene la caracterstica que sus propiedades en un punto no cambian
si hacemos un cambio de base. Un tensor de segundo orden istropo es un tensor esfrico,
ver captulo 1, luego el tensor K tiene que ser del tipo: K = K1 , donde K es una constante.
En forma de matriz:
1 0 0
K ij = K 0 1 0
0 0 1

Ejemplo 4.17
Supongamos un medio continuo donde en un punto material hay dos cantidades fsicas por
unidad de volumen c s y c f tal que c = c f + c s , ver Figura 4.6, y se cumple tambin que
r r
r
v = v f + v s . Considerando que es un proceso isotrmico, un medio incompresible, que la
propiedad c s no afecta en la velocidad del material f y que el campo c f es homogneo, y
no hay fuente del material f . Demostrar que:
r
c s
Q s xr (v f c s ) + xr (D xr c s ) =
t

Ecuacin ConveccinDifusin

(4.26)

donde el flujo de la propiedad s viene dado por q ( D ) = D xr c s .


Volumen de control

r
vf

cs
r
v

dV

r
vs

Figura 4.6: Medio heterogneo.


Solucin:
Partiendo de la ecuacin de continuidad de esta cantidad fsica:
Q=

+ xr
t

(v )

Q=

r
r
(c f + c s )
+ r (c f + c s )(v f + v s )
x
t

(4.27)

con Q = Q s + Q f . Luego:

Universidad de Castilla- La Mancha


Ciudad Real - Espaa

Draft

Por: Eduardo W. V. Chaves (2014)

MECNICA DEL MEDIO CONTINUO: PROBLEMAS RESUELTOS

332

r
r
(c f + c s )
+ r (c f + c s )(v f + v s )
t
x
f
s
r
r
r
(c + c ) f r f
Qs + Q f =
+ r c v + c f v s + csv f + csv s
t
x
f
s
r
r
r
r
c
c
Qs + Q f =
+
+ xr c f v f + c f v s + c s v f + c s v s
t
t
f
c
r c s
r
r
r
+ xr (c s v f ) + xr c f v s + c s v s
Qs + Q f =
+ xr (c f v f ) +
t
t
Qs + Q f =

(4.28)

Si suponemos que para el material ( f ) no hay fuente, luego

r
c f
+ xr (c f v f ) = 0 , y
t

Q f = 0 , que es la ecuacin de continuidad de la cantidad c f . Con eso quedamos con:


Qs =

Qs =

r
r
r
c s
+ xr (c s v f ) + xr c f v s + c s v s
t

r
r
r
c s
+ xr (c s v f ) + xr (c s v s ) + xr (c f v s )
t

(4.29)
(4.30)

r
r
r
r
c s
(4.31)
+ xr (c s v f ) + xr (c s v s ) + xr c f v s + c f xr v s
t
r
Si la cantidad fsica c f no cambia con x , campo homogneo, luego el gradiente
r
r
xr c f = 0 . Si adems, para el medio ( s ) lo consideramos incompresible xr v s = 0 . Estas
simplificaciones nos indican que el material ( s ) no afecta en las propiedades y ni en el
campo de velocidad del material ( f ). Lgicamente si la cantidad del material ( s ) es

Qs =

significativa esta aproximacin ya no ser vlida. Con estas aproximaciones quedamos con:
r
r
r
r
c s
c s
(4.32)
+ xr (c s v f ) + xr q ( D )
+ xr (c s v f ) + xr (c s v s ) =
t
t
r
r
Observemos que el trmino (c s v s ) q ( D ) representa el flujo debido a la concentracin del
r
r
material ( s ), trmino difusivo. El trmino (c s v f ) q (C ) est relacionado con transporte de
r
masa, trmino convectivo. Teniendo en cuenta que q ( D ) = D xr c s la expresin (4.32)
Qs =

queda:
r
r
c s
+ xr (c s v f ) + xr q ( D )
t
r
c s
Qs =
+ xr (c s v f ) + xr (D xr c s )
t
r
c s
Q s xr (c s v f ) + xr (D xr c s ) =
t

Qs =

(4.33)

Demostrando as (4.26).

Universidad de Castilla- La Mancha


Ciudad Real - Espaa

Draft

Por: Eduardo W. V. Chaves (2014)

4 LEYES FUNDAMENTALES DE LA MECNICA DEL MEDIO CONTINUO

333

Ejemplo 4.18
Considrese un tanque lleno de agua
con concentracin de sedimentos, ver
Figura 4.7. La concentracin de
sedimentos
(densidad
de
concentracin) viene dada por

( kx t )

3 , por unidad de
c( x3 , t ) = C t exp
volumen, donde C y k son constantes

positivas. Se pide: a) Obtener la masa


total de sedimentos en el tanque; b)
Obtener el flujo de sedimentos
sabiendo que ste es solo funcin de
r r
x3 y del tiempo t , i.e. q = q( x3 , t ) .

x3

x2

x1
a

Figura 4.7: Tanque con sedimentos.


Solucin:
Para obtener la masa total tenemos que resolver la integral:

h b a

M = c dV =

C t exp

( kx3t )

dx1 dx 2 dx3 = ab C t exp

0 0 0

( kx3t )
C
= ab
exp

( kx3t )

dx3

C abC
C
= ab
exp ( kht ) + =
exp ( kht ) 1
k
k
k

Para obtener el flujo, podemos aplicar la ecuacin de continuidad de la concentracin:


Q=

r
c s
+ xr q
t

r
c s
xr q = qi ,i =
t

(4.34)

donde hemos tenido en cuenta que no hay fuente de sedimentos Q = 0 . Para este problema
el flujo no es dependiente de x 2 y x1 . Con estas condiciones q1,1 = q 2, 2 = 0 . Luego:
q i ,i = q1,1 + q 2, 2 + q 3,3 =

donde

q1 q 2 q 3
c s
=
+
+
t
x1 x 2 x 3

q 3
c s
=
x 3
t

c s
( kx3t )
( kx3t )
( kx3t )
=
C t exp
= C exp
C t k x 3 exp
t
t

(4.35)

y reemplazando en la

expresin (4.35) obtenemos que:

dq 3
c s
( kx3t )
( kx3t )
=
= C exp
+ C t k x 3 exp
t
dx 3

dq 3 =
q3 =

[ C exp

+ C t k x 3 exp

( kx3t )

]dx

C k x3 t
C
C
( kx3t )
( kx3t )
( kx3t )
exp
exp

exp
+ K3
kt
kt
kt

Universidad de Castilla- La Mancha


Ciudad Real - Espaa

( kx3t )

Draft

Por: Eduardo W. V. Chaves (2014)

MECNICA DEL MEDIO CONTINUO: PROBLEMAS RESUELTOS

334

q 3 = C x 3 exp

( kx3t )

+ K3
{
=0

El vector flujo viene dado por q = C x3 exp

( kx3t )

e 3 .

4.2 Introduccin a Movimiento de Slido Rgido


Ejemplo 4.19
Obtener el momento lineal y el momento angular para un slido sometido a un
movimiento de slido rgido.
x3

Slido rgido

Bt

C.M .

r
x

x3

x2

r
v
x1

x2
x1

Figura 4.8
Solucin:
Para un movimiento de slido rgido todas las partculas estarn sometidas a una misma
r
velocidad v . En el captulo 2 obtuvimos que la velocidad para un medio continuo
sometido a un movimiento de cuerpo rgido viene dada por:
r r r
r r
v = c& + ( x c )

donde es el vector axil (velocidad angular) asociado al tensor antisimtrico W (tensor


spin).
Momento lineal:
r
r r
r
r
r r
r r
r r
L = v dV = c& + ( x c ) dV = c& dV + x dV c dV

En notacin indicial:
r& r
r r

(
x
+

c)

dV = c i dV + ijk j x k dV ijk j c k dV

= c i dV + ijk j x k dV ijk j c k dV
V

= mc i + ijk j m x k ijk j c k m = m c i + ijk j ( x k c k )

donde notamos que:

x dV = x dV = (m x ) , siendo

m la masa total y

r
x k el vector posicin del centro de masa (C.M.). As obtenemos que:
r
r r
r r
L = m c& + ( x c )
(Momento lineal de slido rgido)
r
= mv

Universidad de Castilla- La Mancha


Ciudad Real - Espaa

Draft

(4.36)

Por: Eduardo W. V. Chaves (2014)

4 LEYES FUNDAMENTALES DE LA MECNICA DEL MEDIO CONTINUO

335

donde v = c& + ( x c ) es la velocidad del centro de masa.


Momento angular:
r
r r
r
r
r
r r
H O = ( x v ) dV = x c& + ( x c ) dV

)]

r r
r
r r
r
r r
= x c& dV + x ( x ) dV x ( c ) dV

(4.37)

r
r
r r
r
r r
= x dV c& + x ( x ) dV x dV ( c )
V

V
r r r
Del captulo 1 obtuvimos que: dados tres vectores a , b , c la siguiente relacin se cumple
r r
r r
r
r r r
r r r
r
r r
r r r
r r
a (b c ) = (a c )b (a b)c . Luego, a (b a) = [(a a) 1 (a a) ] b , si a = c . Con
r
r r
r r r
r r r
es x ( x ) dV = [( x x ) ( x ) x ] dV y en notacin indicial:

[x

x ] dV = [x
V

x k i x p p

x k p

pi

= xk xk

pi

x p p x i dV = x k x k

pi

x p x i p dV

x p x i dV p = I O ip p

En notacin tensorial queda:


r

r r

x ( x ) dV = [( x x ) 1 ( x x )] dV = I O

r r
r r
r
donde I O = [( x x ) 1 ( x x )] dV es el tensor de inercia con respecto al sistema Ox .
V

Como podemos observar I O es un pseudo-tensor de segundo orden y simtrico. Es un


pseudo-tensor porque depende del sistema de referencia adoptado. Aunque sea un pseudotensor, por comodidad nos vamos a referir simplemente por tensor de inercia, cuyas
componentes son I O ij = [x k x k ij xi x j ] dV , y explcitamente:
V

I O 11 = [( x1 x1 + x 2 x 2 + x 3 x 3 ) 11 x1 x1 ] dV = x 22 + x 32 dV

I O 22 = x12 + x 32 dV

I O 33 = x12 + x 22 dV

I O 12 = [( x1 x1 + x 2 x 2 + x 3 x 3 ) 12 x1 x 2 ] dV = [x1 x 2 ] dV = I O 12

I O 13 = [x1 x 3 ] dV = I O 13

I O 23 = [x 2 x 3 ] dV = I O 23

donde I O11 , I O 22 , I O 33 , son conocidos como momentos de inercia, y I O12 , I O13 , I O 23 ,


son conocidos como productos de inercia en la mecnica clsica. Luego:

I Oij

2
2
x2 + x3 dV
V
= [x1 x 2 ] dV
V
[x1 x3 ] dV
V

[x1 x 2 ] dV

[x1 x3 ] dV
I O 11
V

[x 2 x3 ] dV = I O12
V
I O13
x12 + x22 dV

[ + ] dV
[x x ] dV [
x12

x32

2 3

I O 12
I O 22

I O 23

I O13

I O 23
I O 33

(4.38)
Universidad de Castilla- La Mancha
Ciudad Real - Espaa

Draft

Por: Eduardo W. V. Chaves (2014)

MECNICA DEL MEDIO CONTINUO: PROBLEMAS RESUELTOS

336

Retomando la ecuacin (4.37) podemos decir que:

r
r
r
r r
r
r r
H O = x dV c& + x ( x ) dV x dV ( c )
V

V
r r&
r
r r&
r
r r
r r
r
= m x c + I O m x ( c ) = m x c ( c ) + I O
r r r
Sumando y restando el trmino m x x en la relacin anterior obtenemos:
r r r r
r r r
r r
r
r
r
r r
r
H O = m x c& c + I O = m x c& + ( x c ) m x ( x ) + I O
r r
r r
r r r
r r
r r
r r
r
r
= m x v m ( x x) 1 ( x x) + I O = m x v + m ( x x) ( x x) 1 + I O
r r
r
= m x v + I
r r r
= m x v + HG

{[

(4.39)
Definiendo as el Momento angular con respecto al punto O :
r
r r r
Momento angular con respecto al sistema
HO = m x v + HG
(4.40)
que pasa por O
r
r
r
r
Verifiquemos que cuando c = 0 tenemos que H O = I O . Notar tambin que en la
ecuacin (4.39) hemos definido el tensor de inercia con respecto al sistema que pasa por el
r r
r r
centro de masa: I = I O + m[( x x ) ( x x ) 1] . Por medio de esta ecuacin podemos
calcular el tensor de inercia en cualquier sistema si conocemos el tensor de inercia que pasa
por el centro de masa, i.e.: IO ij = Iij m[ xi x j ( x12 + x22 + x32 ) ij ] . Explcitamente estas
componentes vienen dadas por:
IO11 = I11 + m( x22 + x32 ) ; IO12 = I12 m( x1 x2 )
Teorema
IO 22 = I22 + m( x12 + x32 ) ; IO 23 = I23 m( x2 x3 )
IO 33 = I33 +

m( x12

x22 )

IO13 = I13 m( x1 x3 )

de

los

ejes

paralelos

(4.41)

Observemos que las expresiones anteriores constituyen el teorema de los ejes paralelos
(Teorema de Steiner) de la Mecnica Clsica, y en forma matricial queda:
I O ij

I11

= I12
I
13

I12
I 22
I 23

x 22 + x 32
I13

I 23 + m x1 x 2
x x
I33
1 3

x1 x 2
x12

x 32

x 2 x3

x1 x 3
Teorema
x 2 x3
paralelos
x12 + x 22

de

los

ejes

(4.42)

NOTA: Si tenemos dos slidos B (1) y B ( 2) podemos decir que


r
HO =

(x

(1)

r
v ) dV (1) +

V (1 )

(x

( 2) r

v ) dV ( 2 )

V (2)

r
r
r
r
r
r
= m x (1) v (1) + I (1) (1) + m ( 2 ) x ( 2 ) v ( 2 ) + I ( 2 ) ( 2 )
r
r
r
r
r
r
= m (1) x (1) v (1) + m ( 2 ) x ( 2 ) v ( 2 ) + I (1) (1) + I ( 2 ) ( 2 )
r
r
r
= m ( sys ) x ( sys ) v ( sys ) + I ( sys ) ( sys )
(1)

donde (1) y ( 2 ) representan las propiedades de los slidos B (1) y B ( 2) respectivamente.


Si estos dos cuerpo estn atados tendrn las mismas velocidades angulares
r
r
r
r
(1) = ( 2 ) = ( sys ) = , entonces, se cumple que:
r
HO = m

r
r
r
x ( sys ) v ( sys ) + [ I (1) + I ( 2 ) ]
r
r
r
y si el sistema Ox est en el centro de masa del sistema ( v (sys ) = 0 ) con lo cual:
r
r
r
H O = I ( sys ) = [ I (1) + I ( 2 ) ]

Universidad de Castilla- La Mancha


Ciudad Real - Espaa

( sys )

Draft

Por: Eduardo W. V. Chaves (2014)

4 LEYES FUNDAMENTALES DE LA MECNICA DEL MEDIO CONTINUO

337

Ejemplo 4.20
Considrese un paraleleppedo de dimensiones a b c , ver Figura 4.9, cuyo campo de
r
densidad masa, ( x ) , es homogneo. Obtener el tensor de inercial con respecto al sistema
que pasa por el centro de gravedad.
x3

x2

x1

Figura 4.9: Paraleleppedo.


Solucin:
Utilizaremos la ecuacin (4.38):

2
2
x2 + x3 dV
V
= [x1 x 2 ] dV
V
[x1 x3 ] dV
V

I Oij

[x1 x3 ] dV
I O 11
V

[x 2 x3 ] dV = I O12
V
I O13
x12 + x22 dV

[x1 x 2 ] dV

[ + ] dV
[x x ] dV [
x12

x32

2 3

I O 12
I O 22

I O 23

I O13

I O 23
I O 33

Notar que la densidad de masa para un material homogneo no vara con x , y adems se
cumple que m = dV = dV = V = abc . El momento de inercia I O11 queda:
V

I O11 = x 22 + x32 dV =

c
2

b
2

a
2

[x

2
2

+ x32 dx1dx 2 dx3 =

c b a
2
2
2

Anlogamente podemos obtener que I O 22 =

abc 2
m
(b + c 2 ) = (b 2 + c 2 )
12
12

m 2
(a + c 2 )
12

I O 33 =

m 2
(a + b 2 ) .
12

Dejamos al lector demostrar que I O12 = I O 13 = I O 23 = 0 . Recordar que el tensor de inercia


nos dar la informacin de cmo la masa est distribuida segn los ejes adoptados, y fijar
que la masa est distribuida de forma igual con respecto al plano x1 x2 , luego

[x x ] dV = 0 . Resultando que, los ejes adoptados son ejes principales de inercia:


1 2

I Oij

m 2
2
0
0

12 (b + c )

m 2

=
0
(a + c 2 )
0
12

m 2

2
0
0
(a + b )

12

Universidad de Castilla- La Mancha


Ciudad Real - Espaa

Draft

Por: Eduardo W. V. Chaves (2014)

MECNICA DEL MEDIO CONTINUO: PROBLEMAS RESUELTOS

338

NOTA: Una relacin de tensores de inercia para varios slidos se puede encontrar en
Wikipedia http://en.wikipedia.org/wiki/List_of_moments_of_inertia
Ejemplo 4.21
Considrese tres barras de longitud a y masa m , (ver Figura 4.10). Obtener el tensor de
r
inercia de todo el sistema con respecto al sistema de coordenadas en Ox .
x2

r
r
x (1) = 0
r
a
a
x ( 2 ) = e 1 + e 2
2
2
r ( 3) a
a
e 1 + e 3
x =
2
2

barra 2

a
2

x2

a
2

a
2

x1
x2

r
x ( 2)

a
2

r
x ( 3)

a
2

x1

barra 1

x1

a
2

x3
x3

barra 3

Figura 4.10: Sistema compuesto por tres barras.


r

Dato: Tensor de inercia de la barra para el sistema Ox viene dado por:


x2
a
2

a
2

I gij

g
x1

0 0 0
ma 2
=
0 1 0
12
0 0 1

x3

Universidad de Castilla- La Mancha


Ciudad Real - Espaa

Draft

Por: Eduardo W. V. Chaves (2014)

4 LEYES FUNDAMENTALES DE LA MECNICA DEL MEDIO CONTINUO

339

Solucin:
)
(1)
( 2)
( 3)
r
r
r
r
I O( sys
x = I Ox + I Ox + I Ox

Utilizaremos

IO ij = Iij m xi x j ( x12 + x22 + x32 ) ij

I O ij

I11

= I12
I
13

I12
I 22
I 23

(ver

Ejemplo

4.19

(NOTA

1)),

donde

(Teorema de Steiner):

x 22 + x 32
I13

I 23 + m x1 x 2
x x
I33
1 3

x1 x 2
x12 + x 32
x 2 x3

x1 x 3

x 2 x3
x12 + x 22

(4.43)

Barra 1 - I O(1X)r
Vector position del centro de masa: ( x1(1) = 0, x2(1) = 0, x3(1) = 0)
(1)
r)
(I Ox
ij

0 0 0
ma 2
=
0 1 0
12
0 0 1

Barra 2 - I O( 2Xr)
a
2

Vector posicin del centro de masa: ( x1( 2 ) = , x 2( 2 ) =

a ( 2)
, x 3 = 0)
2

Utilizando la ecuacin (4.43) podemos obtener que:


I11

(I O( 2xr) ) ij = I12
I
13

I12
I 22
I 23

x 22 + x 32
I13

I 23 + m x1 x 2
x1 x 3
I 33

x1 x 2
x12 + x 32
x 2 x3

a 2
2
+ 0
2
1 0 0
2
ma
+ m a a
0
0
0
=

12
2 2
0 0 1

x1 x 3

x 2 x3
x12 + x 22

a a

2 2
2
a
2
+0
2
x 2 x3

4 3 0
ma 2
x 2 x3
3 3 0
=

12

0
0 7
2
2

a
a


+
2
2
0

Barra 3 - I O(3X)r
Vector posicin del centro de masa: ( x1( 2 ) =
I11

(I O( 2xr) ) ij = I12
I
13

I12
I 22
I 23

x 22 + x 32
I13

I 23 + m x1 x 2
x1 x 3
I 33

a ( 2)
a
, x 2 = 0, x 3( 2 ) = )
2
2

x1 x 2
x12 + x 32
x 2 x3

x1 x 3

x 2 x3
x12 + x 22

2 a 2
0
(0 ) +
2

1
0
0

2
2
ma 2
a
a
+ m
0
1
0
0
+
=

12
2
2

0 0 0
a a
x2 x3


2 2

Universidad de Castilla- La Mancha


Ciudad Real - Espaa

Draft

a a


2 2
4 0 3
ma 2
0 7 0
x 2 x3 =

12

3 0 3
2

+0
2

Por: Eduardo W. V. Chaves (2014)

MECNICA DEL MEDIO CONTINUO: PROBLEMAS RESUELTOS

340

Luego, podemos calcular


)
(1)
( 2)
( 3)
r
r
r
r
(I O( sys
x ) ij = (I Ox ) ij + (I Ox ) ij + (I Ox ) ij

0 0 0
2
ma 2
+ ma
=
0
1
0
12
12
0 0 1

4 3 0
4 0 3
8 3 3
2
2
3 3 0 + ma 0 7 0 = ma 3 11 0

12

12
0
3 0 3
3
0 7
0 11

Ejemplo 4.22
Obtener el principio de la conservacin del momento lineal y del momento angular para un
slido sometido a un movimiento de cuerpo rgido.
Solucin: Para un movimiento de cuerpo rgido, ver Figura 4.8, todas las partculas estarn
r
sometidas a una misma velocidad v . Partiendo de la definicin de principio de la
conservacin del momento lineal:

r
r
r
r
D
t * dS + b dV =
v dV = L&
Dt V
V

Recurrimos ahora la expresin del momento lineal para un movimiento de slido rgido
r
r
obtenido en el ejemplo anterior como L = m v , luego:
r
r
r
r
r
D
dS + b dV =
v dV = L& = m v& = m a
Dt V
S
14442V 444
3
r
F
r*

Resultando:

F = ma

Consideremos ahora el principio de la conservacin del momento angular:

r
r&
r r
r
r
r
D
D r
( x t * )dS + ( x b)dV =
( x v )dV =
HO HO
Dt V
Dt
S
1
444442V44444
3
r
MO

Resultando:
r

r&
= H O , tambin es vlido que

r&
= HG

donde la expresin del momento angular H O para un movimiento de slido rgido fue
obtenida en el ejemplo anterior. Los conjuntos de ecuaciones anteriores,

F = m a

r
r&
M G = H G , nos informan que los dos sistemas presentados en la Figura 4.11 son

equivalentes:
NOTA: Cuando estamos tratando de un movimiento de slido rgido, las ecuaciones de
gobierno del problema se resumen a:
r

F = ma

Universidad de Castilla- La Mancha


Ciudad Real - Espaa

r&
= HO

Draft

Ecuaciones de gobierno para


movimiento de slido rgido

(4.44)

Por: Eduardo W. V. Chaves (2014)

4 LEYES FUNDAMENTALES DE LA MECNICA DEL MEDIO CONTINUO

r&
HG

r
F( 2 )

r
F( n )

341

r
ma

r
F(1)

G - centro de masa

Figura 4.11
Ejemplo 4.23
Consideremos una viga con las siguientes condiciones de carga y de apoyo:
P

B
L
2

HA

L
2

VA

VB

Figura 4.12: Viga isosttica.


Obtener las reacciones de apoyo VA , VB , H A .
Solucin:
Aunque en la viga haya deformacin (rgimen de pequeas deformaciones) y tensin, para
efecto de clculo de las reacciones de apoyo de una viga isosttica podemos considerar que
de un cuerpo rgido se tratara, y las ecuaciones necesarias, ver ecuaciones (4.44), son:

r
r r
F=ma =0

r
r
r&
MA = HA =0

F
F
F

=0

=0

=0

M
M
M

F
F

=0

=0

=0

pudiendo ahora determinar V A = V B + P sin =

= H A + P cos = 0 H A = P cos

= V A + V B P sin = 0 V A = V B + P sin

= V B L P sin

L
P sin
= 0 VB =
2
2

P sin
. Notar que tenamos 3 ecuaciones
2

y tres incgnitas. Si tenemos un sistema donde hay ms incgnitas que ecuaciones, sistema
hiperesttico, este procedimiento ya no es vlido ya que las reacciones dependern de la
deformada de la viga y sta a su vez depender de la rigidez de la viga.

Universidad de Castilla- La Mancha


Ciudad Real - Espaa

Draft

Por: Eduardo W. V. Chaves (2014)

MECNICA DEL MEDIO CONTINUO: PROBLEMAS RESUELTOS

342

Ejemplo 4.24
Obtener la energa cintica para un slido sometido a un movimiento de cuerpo rgido, ver
Figura 4.8.
Solucin:
Para un movimiento de cuerpo rgido todas las partculas estarn sometidas a una misma
r r r
r r
velocidad v = c& + ( x c ) . Luego la energa cintica viene dada por:

][

r r
1
1
(v v )dV =
cr& + r ( xr cr ) cr& + r ( xr cr ) dV
2V
2V

r
r r r
Haciendo la siguiente suma de vectores x = x + x , donde x es el vector posicin del
r
centro de masa, y x es el vector posicin de las partculas del slido con respecto al

K (t ) =

sistema que pasa por el centro de masa.


K(t ) =

][

r r
r r
r r& r
r
r
cr& +

((
)
)
((
)
) dV

c
c

x
x
x
x

1
2V

[(

] [(

r r
r r
r r
r r
r r
r r
1
c& + ( x c ) + ( x ) c& + ( x c) + ( x ) dV
2V

r r& r
r r
Notar que v = c + ( x c) es la velocidad del centro de masa. Con lo cual la ecuacin

anterior queda:
1

2V

K (t ) =

K (t ) =

{ [vr + (r xr )] [vr + (r xr )] }dV

r r
r r r
r r r
r r
r r
1
1
1
1
v v dV +
v ( x ) dV +
( x ) v dV +
( x ) ( x ) dV
2V
2V
2V
2V

Simplificando la ecuacin anterior, podemos obtener que:


K (t ) =

r r
r r r
r r
r r
1
1
v v dV + v ( x ) dV + ( x ) ( x ) dV
2V
2V
V

A continuacin analizaremos separadamente el trmino


1)

r r
1 r
1
v v dV = v
2
2V

dV = 2 mv

r r

x ) = 0
2) v ( x ) dV = v x dV = v ( m {

r
=0

Observemos que el sistema x pasa por el centro de masa luego, el vector posicin del
r
centro de masa para el sistema x es el vector nulo.
r

3) ( x ) ( x ) dV
V

[( x ) ( x )] dV =

ijk j x k

ipq p x q dV = ( jp kq jq kp ) j x k p x q dV
V

= j ( jp

kq x k

x q jq

kp x k

x q ) p dV

= j (

jp x k

x k x p x j ) p dV

= j (

Universidad de Castilla- La Mancha


Ciudad Real - Espaa

jp x k

x k x j x p ) dV p = j I jp p

Draft

Por: Eduardo W. V. Chaves (2014)

4 LEYES FUNDAMENTALES DE LA MECNICA DEL MEDIO CONTINUO

343

En notacin tensorial queda:


r

[( x ) ( x )] dV = [( x x ) 1 ( x x )] dV = I

donde I es el tensor de inercia respecto al sistema que pasa por el centro de masa, (ver
Ejemplo 4.19).
Resultando as que la expresin de la energa cintica para un movimiento de slido rgido:
K (t ) =

r r
r r r
r r
r r
1
1
1
v v dV + 2 v ( x ) dV + ( x ) ( x ) dV
2V
2
2V
1V44424443

=0

r
1
1r
mv 2 + I
2
2

1
2

1r
2

K(t ) = mv 2 + I

Energa cintica para un


movimiento de slido rgido

(4.45)

Representando las componentes del tensor de inercia como:

2
2
x 2 + x3 dV
V
Iij = [x1 x 2 ] dV
V
[x1 x3 ] dV
V

[x1 x 2 ] dV

[x1 x3 ] dV
I11
V

[x 2 x 3 ] dV = I12
V
I13
x1 2 + x 2 2 dV

[x + x ] dV
[x x ] dV
[
1

2 3

I13

I 23
I 33

I12

I 22
I 23

La forma explcita de la energa cintica queda:


1
1
1
1
K(t ) = mv 2 + k I kj j = mv 2 + [1
2
2
2
2
=

I 11

3 ] I12
I
13

I 12

I 22
I 23

I13 1

I 23 2
I 33 3

1
1
mv 2 + I11 12 + I 22 22 + I 33 32 2 I12 1 2 2 I13 1 3 2 I 23 2 3
2
2

1
2

K(t ) = mv 2 +

1
I1112 + I 22 22 + I 33 32 2 I12 1 2 2 I13 13 2 I 23 2 3
2

(4.46)

Ejemplo 4.25
Considrese el pseudo-tensor de inercia, I O , con respecto al sistema ortonormal Ox1 x 2 x 3 ,
ver Figura 4.13. a) Hacer la interpretacin fsica del tensor de inercia. b) Dado otro sistema
ortonormal, representado por Ox1* x 2* x 3* , obtener las componentes del tensor de inercia en
este nuevo sistema. c) Demostrar que el tensor de inercia es un tensor definido positivo.
Para un slido en movimiento, en que situaciones la tasa del tensor de inercia,

DI O &
IO ,
Dt

es cero?

Universidad de Castilla- La Mancha


Ciudad Real - Espaa

Draft

Por: Eduardo W. V. Chaves (2014)

MECNICA DEL MEDIO CONTINUO: PROBLEMAS RESUELTOS

344

x3

x2*

x3*

x1*
O

x2

x1

Figura 4.13
Solucin:
El pseudo-tensor de inercia depende del sistema de coordenadas adoptado. Y por
definicin viene dado por:

r r
r r
I O = [( x x ) 1 ( x x )] dV

I O ij = x k x k ij x i x j dV

En componentes

2
2
x 2 + x3 dV
V
I ij = [x1 x 2 ] dV
V
[x1 x3 ] dV
V

[x1 x 2 ] dV

[x1 x 3 ] dV

V
[x 2 x3 ] dV
V

x12 + x 22 dV
V

[x + x ]dV
[x x ] dV [
2
1

2
3

2 3

a) El tensor de inercia nos dar informacin de cmo la masa del slido est distribuida con
respecto al sistema adoptado.
El trmino [x1 x 2 ] dV nos indica como la masa del slido est distribuida con respecto al
V

plano x1 x 2 . Luego, Si el material es homogneo, i.e. campo de densidad de masa


uniforme, y x1 x 2 es un plano de simetra, es decir si la masa est distribuida por igual

con respecto al plano x1 x 2 , el trmino [x1 x 2 ] dV es igual a cero. Con eso, concluimos
V

tambin que si los planos x1 x 2 , x1 x 3 , x 2 x3 , son planos de simetra, la matriz de


inercia ser una matriz diagonal.
Como ejemplo de ilustracin, vamos considerar un estudiante atado a un disco y con los
brazos estirados donde en cada mano sujeta un peso (ver Figura 4.14 sistema inicial). El
r
disco est girando con un velocidad angular constante (i ) y el tensor de inercia segn el
r
sistema x viene dado por I O(i ) . Si consideramos un sistema sin disipacin de energa, qu
pasar al sistema cuando el estudiante mueva sus brazos haca dentro?, ver Figura 4.14
sistema final. Como estamos tratando con un sistema conservativo, el momento angular se
conserva, luego
r
r
H O( i ) = H O( f )
r
r
I (Oi ) ( i ) = I O( f ) ( f )

Universidad de Castilla- La Mancha


Ciudad Real - Espaa

Draft

Por: Eduardo W. V. Chaves (2014)

4 LEYES FUNDAMENTALES DE LA MECNICA DEL MEDIO CONTINUO

345

Ya que para el sistema final la masa est ms concentrada segn el eje de rotacin, la
r
r
desigualdad I O( f ) < I (Oi ) se cumple, y como consecuencia ( f ) > (i ) .

x3

Sistema inicial

x3

Sistema final

r
(i )

r
( f )

I O( f )

I O(i )

Figura 4.14:
b) Vamos suponer que los sistemas dados estn relacionados por la ley de transformacin
xi* = Aij x j , donde Aij es la matriz ortogonal de transformacin de base, luego, se cumple

que xi = A ji x *j . Pudiendo as expresar I O ij de la siguiente manera:

I O ij = x k x k ij x i x j dV = ( x k* x k* )Aip
V

pq

{[

= Aip ( x k* x k* )

pq

x *p x q* A jq dV = Aip ( x k* x k* )
V

]}

A jq Aip x *p A jq x q* dV

pq

x *p x q* dV A jq

= Aip I *O ij A jq

Notar que x k x k = A ks x s* A kt x t* = x *s x t* A ks A kt = x *s x t* st = x *s x s* = x t* x t* = x k* x k* .

Abusando un poco de la notacin, utilizamos tambin notacin tensorial, pero hay que
tener en cuenta que estamos trabajando con las componentes del tensor, y no haciendo una
transformacin ortogonal.

r r
r r
r r
r
r
I O = [( x x ) 1 ( x x )] dV = ( x * x * )A T 1 A (A T x * A T x * ) dV

r r
r
r
= ( x * x * )A T 1 A (A T x * x * A ) dV
V

= AT
V

{ [( x * x * )1 ( x * x * )]} A dV = A T [( x * x * )1 ( x * x * )] dV A
r

= A T I *O A
Universidad de Castilla- La Mancha
Ciudad Real - Espaa

Draft

Por: Eduardo W. V. Chaves (2014)

MECNICA DEL MEDIO CONTINUO: PROBLEMAS RESUELTOS

346

I O = A T I *O A

I O ij = A

*
ip I O ij

A jq

Componentes del tensor de inercia tras un


cambio de base (rotacin)

(4.47)

Luego, es vlido que I *O = A I O A T , que son las nuevas componentes del tensor de

inercia en el sistema x1* x 2* x3* . Verifiquemos que es la misma ley de transformacin de las
componentes de un tensor de segundo orden, donde A es la matriz de transformacin del
sistema x1 x 2 x3 al sistema x1* x 2* x3* .
c) Un tensor definido positivo, por definicin, sus autovalores son mayores que cero.
Partiremos de la expresin de la energa cintica obtenida en el Ejemplo 4.24:
1
2

K(t ) = mv 2 +

1
I1112 + I 22 22 + I 33 32 2 I12 1 2 2 I13 13 2 I 23 2 3
2

La energa cintica es un escalar y siempre positivo, solo en dos situaciones la energa


cintica ser igual a cero, cuando no haya masa o cuando el cuerpo est en reposo. Vamos
adoptar un sistema tal que el origen est situado en el centro de masa y los ejes adoptados
son ejes de simetra (ejes principales de inercia) y que el cuerpo est girando alrededor del
origen, centro de masa. En esta situacin la expresin de la energa cintica se resume a:
1
K(t ) = [1
2

0 1
I1 0
1

3 ] 0 I 2 0 2 = I1 12 + I 2 22 + I 3 32 > 0
2
0
0 I 3 3
144
42444
3

Autovalores del
tensor de inercia

1
2

Si adems tenemos un movimiento tal que 2 = 3 = 0 , nos quedamos con K(t ) = I112 ,

luego, la nica forma que la energa cintica sea siempre positiva es que I1 > 0 .
Anlogamente, podemos concluir que I 2 > 0 , I 3 > 0 . Con eso concluimos que el tensor
de inercia es un tensor definido positivo.
d) Como el tensor de inercia depende del sistema adoptado, en las siguientes situaciones el
tensor de inercia para un slido en movimiento no cambia con el tiempo:
1) Si el sistema adoptado est unido al slido.
2) Si el slido est girando alrededor de un eje de simetra, por ejemplo si un cilindro est
girando alrededor del eje prismtico, luego, durante el movimiento la distribucin de masa,
con respecto a los ejes adoptados, no cambia con el tiempo:
r

sistema de referencia fijo en el


espacio

Figura 4.15:
Universidad de Castilla- La Mancha
Ciudad Real - Espaa

Draft

Por: Eduardo W. V. Chaves (2014)

4 LEYES FUNDAMENTALES DE LA MECNICA DEL MEDIO CONTINUO

347

Ejemplo 4.26
Considrese un cilindro homogneo de radio r y altura h = 3r con masa total igual a m .
Encontrar el tensor de inercia en el sistema Ox1 x 2 x3 . El sistema Ox1 x 2 x3 viene dado por
una rotacin del sistema Ox1x 2 x3 de 45 a lo largo del eje x1 . Los sistemas Gx1 x 2 x3 y
Ox1x 2 x3 tienen las mismas orientaciones.
x3

r
x3
x3

x1

x2

r
rG

h = 3r

x2
45

x2

x1, x1

Figura 4.16
Datos: Para el sistema de referencia Gx1 x 2 x3 se conoce el tensor de inercia y viene dado
por:

I G ij

1
2
2
0
12 m(3r + h )

1
m(3r 2 + h 2 )
=
0
12

0
0

mr 2 2 0 0
0 2 0
0 =

0
0
1
1 2

mr
2

Solucin:
Primero obtenemos el tensor de inercia en el sistema Ox1x 2 x3 a travs del teorema de
Steiner, ver ecuacin (4.41) del Ejemplo 4.19. Despus aplicamos una rotacin al tensor
segn ecuacin (4.47) del Ejemplo 4.25.
A travs de las ecuaciones (4.42):
I11

I O ij = I12
I
13

I12
I 22
I 23

x 22 + x32
I13

I 23 + m x1 x 2
x x
I33
1 3

x1 x 2
x12

x32

x2 x3

x1 x3

x2 x3
x12 + x 22

(4.48)

donde ( x1 , x 2 , x 3 ) son las coordenadas del centro de masa con respecto al sistema Ox1x 2 x3 ,
3
2

y considerando el vector rG = x1e 1 + x 2 e 2 + x 3 e 3 = 0e 1 + re 2 + re 3 , podemos obtener


que:
Universidad de Castilla- La Mancha
Ciudad Real - Espaa

Draft

Por: Eduardo W. V. Chaves (2014)

MECNICA DEL MEDIO CONTINUO: PROBLEMAS RESUELTOS

348

2
3 2
r + ( 2 r )

2 0 0

mr 2

I O ij =
0 2 0 + m
0

2
0 0 1

( r )( 3 r )

2

0
3 2
2
0 + ( 2 r )

( r )( 2 r )

0
34 0
mr 2

=
0
0 13 6

4
0 6 6

02 + r 2

Teniendo en cuenta la matriz de transformacin entre los sistemas Ox1x 2 x3 y Ox1 x 2 x3 :


0
0
1

A = 0 cos 45 sin 45
0 sin 45 cos 45

y aplicando la relacin (4.47) obtenemos que:


IO ij = A I O A = Aip I O ij A jq
T

Ejemplo 4.27

0
34 0
mr 2
0
7 7
=

8
0 7 31

Teniendo en cuenta el momento angular H O = m x v + I = m x v + H G , encontrar la


tasa del momento angular de tal forma que no tenga la necesidad de calcular en cada
instante de tiempo el tensor de inercia.
r
- velocidad angular del slido

r
HG

r
- velocidad angular del sistema x *

x3

x2*

x3*

r
HO

x1*

x3
G

r
x
O

x2

x1

x 2

G - centro de masas

x1

Figura 4.17

Universidad de Castilla- La Mancha


Ciudad Real - Espaa

Draft

Por: Eduardo W. V. Chaves (2014)

4 LEYES FUNDAMENTALES DE LA MECNICA DEL MEDIO CONTINUO

349

Solucin:
Recurriendo a la derivada material podemos decir que:

r
r
r r r
r r D r
r Dv r&
r&
D r
D
D
Dx r
HO HO =
m x v + HG =
m xv +
HG = m
v +m x
+ HG
Dt
Dt
Dt
Dt
Dt
Dt
r r
r r r&
= m v{
v + m x a + HG
r

[ ]

=0

Luego, obtenemos que:


r r r&
r&
D r
HO HO = m x a + HG
Dt

(4.49)
r

donde a es la aceleracin del centro de masa. A continuacin analizamos el trmino H& G .


Adoptamos un sistema mvil x1 x 2 x3 pero con orientacin fija y siempre paralelo al sistema
r
fijo en el espacio x1 x 2 x3 , ver Figura 4.17. Expresando las componentes de I y en el
sistema x1 x 2 x3 , obtenemos que:
r
r
H G = I

r&
r&
D r
& r

H G H G = I + I
Dt
r
Notar que como el slido gira con respecto al sistema x , con lo cual la distribucin de
r
masa, con respecto al sistema x , cambia, y a su vez el tensor de inercia tambin cambia.
tasa

Luego, a cada instante de tiempo tenemos que calcular el tensor de inercia. Este
procedimiento es muy costoso. Para solventar este problema adoptamos un nuevo sistema
r
x * , que tambin tiene origen en el centro de masa, ver Figura 4.17. A travs de ley de
transformacin de las componentes de los tensores, las siguientes relaciones son vlidas:
r
r
r
r
H G* = A H G
H G = A T H G*
;
r
r
r
r
Componente s * = A ; = A T *
*
T
;
I O = A T I O* A
I O = A I O A
r
r
donde A es la matriz de transformacin del sistema x al sistema x * .
r
r
La tasa de H G = A T H G* resulta:

r&
r
r
r&
D r
D
A T H G* = A& T H G* + A T H G*
H G H G =
Dt
Dt

(4.50)

Haciendo una analoga con la tasa de un tensor ortogonal, ver captulo 2 en Chaves (2007),
podemos decir que = A& A T A& T = A T T , donde T es un tensor antisimtrico
r
r
y representa el tensor tasa de rotacin del sistema x * con respecto al sistema x . Pudiendo
as expresar (4.37) como:
r&
r
r&
H G = A T T H G* + A T H G*
(componentes)
r
r&
= A T T H G* + H G*

(4.51)

r
r r
r
Recurriendo a la propiedad del tensor antisimtrico tal que T H G* = H G* , donde
r r
es el vector axil asociado al tensor antisimtrico T , es decir, = (t ) es la velocidad
r
angular del sistema mvil x * (ver NOTA 4). Resultando que (4.51) an puede ser escrito
como:

Universidad de Castilla- La Mancha


Ciudad Real - Espaa

Draft

Por: Eduardo W. V. Chaves (2014)

MECNICA DEL MEDIO CONTINUO: PROBLEMAS RESUELTOS

350

r&
r
r&
H G = A T T H G* + H G*

(componentes)
r
r
r
&
= A T * H G* + H G*

(4.52)

donde
r*
r&
D * r * DI * r *
H G* =
I =
+ I * D
Dt
Dt
Dt

Para que el trmino


1)

DI *
sea igual a cero, podemos tener dos posibilidades:
Dt

r r
r
DI *
= 0 si el sistema x * est unido al slido. En este caso se cumple que = , es
Dt

decir, la velocidad del sistema mvil es igual a la velocidad angular del slido.
2)

DI *
= 0 si el slido gira alrededor de un eje prismtico, ver Figura 4.15 en el Ejemplo
Dt

4.25.
NOTA 1: La ecuacin (4.52) puede ser reescrita como:
r&
r
r&
r
H G = A T * H G* + H G*

(componentes) (4.53)
r&
r*
r& *
r * r * r& *
T r*

A H G = A A H G + H G = H G + H G

r&
r&
r
Notar que el trmino A H G representa las componentes de H G en el sistema x * , y notar
r
r
tambin que A H& G H& G* , luego:

r * r
r
A H& = H& * + r * H * (componentes)
G
G
G

(4.54)

Tambin podemos expresar la ecuacin anterior en notacin tensorial:


r
DH G

Dt

r
DH G

r r

=
Dt + H G

r
f

(notacin tensorial)

(4.55)

r
r
DH G

representa la tasa de H G con respecto al sistema fijo,

Dt

r
r
r
representa la tasa de H G con respecto al que est rotando con una velocidad angular .

r
DH G

donde
Dt

NOTA 2: La ecuacin (4.55) es vlida para cualquier vector, i.e. la tasa de cambio de un
r
r
r
vector b respecto a un sistema fijo x es igual a la tasa de cambio del vector b con
r
respecto al sistema mvil x * ms el producto vectorial entre la velocidad angular del
r
r
sistema mvil ( asociado al tensor antisimtrico T ) y el vector b :
r
r
r
r
Db
Db
r r Db
T

b
=
+

b
Dt

Dt

fijo Dt mvil

mvil

Universidad de Castilla- La Mancha


Ciudad Real - Espaa

Draft

(4.56)

Por: Eduardo W. V. Chaves (2014)

4 LEYES FUNDAMENTALES DE LA MECNICA DEL MEDIO CONTINUO

351

r
Db
D
D
D
. Notar
Adoptaremos la siguiente nomenclatura ,

Dt fijo Dt f Dt mvil Dt r

r r D
D
D
tambin que
.
=
=
+
1
2r3 Dt
Dt f Dt r
r
=0

r
b

x3

x2*

x3*

x1*
x 2

x1

Figura 4.18
NOTA 3: Notar que la ecuacin (4.56) es la tasa convectiva, (ver Captulo sobre
Objetividad de Tensores en Chaves (2007) 3 edicin), la cual viene definida por
C
r r
a = a& + l

a , donde

C
r r
= D + W , luego a = a& + l

a = a& + (D + W)T a . Recordar del

1
2

Captulo 2 (Chaves (2007) 3edicin) que W = R U& U 1 U 1 U& R T + R& R T se


cumple. Si consideramos un movimiento de slido rgido tenemos que D = 0 , U& = 0 , y
C
r r
r
W = = R& R T , con lo cual obtenemos que a = a& + T a .

NOTA 4: Vamos exponer un simple ejemplo para la obtencin de T . Vamos asumir que
el sistema e i es girando alrededor del sistema fijo e i , ver Figura 4.19, luego para obtener
T procedimos como sigue. Obtenemos la matriz de transformacin del sistema e i al e i :
cos
A = sin
0

sin
cos
0

0
0
1

(4.57)

cuya tasa de cambio viene dada por:


d (cos )

dt
d (sin )
d (A )
&
A =
dt
dt

d (sin )
dt
d (cos )
dt
0

Universidad de Castilla- La Mancha


Ciudad Real - Espaa

0
&
sin
0 = & cos

0 0

Draft

& cos 0 sin

& sin 0 = & cos


0

cos
sin
0

0
0
0

Por: Eduardo W. V. Chaves (2014)

MECNICA DEL MEDIO CONTINUO: PROBLEMAS RESUELTOS

352

sin
T
&
&
= A A = cos
0

cos
sin
0

0 cos
0 sin
0 0

sin
cos
0

0
0 1 0 0 & 0

0 = & 1 0 0 = & 0 0
0 0 0 0 0 0
1

Con lo cual:
0 &

T = & 0
0 0

0 0

0 = 3
0 2

3
0

2
1

0
i = 0
&

r
donde es el vector axil asociado con el tensor antisimtrico T .

3 = &

e 3
e 1

e 2

e 3

e 2
e 1

Figura 4.19

Universidad de Castilla- La Mancha


Ciudad Real - Espaa

Draft

Por: Eduardo W. V. Chaves (2014)

4 LEYES FUNDAMENTALES DE LA MECNICA DEL MEDIO CONTINUO

353

NOTA 5: Fuerzas inerciales


Vamos considerar un sistema OX 1 X 2 X 3 (ver Figura 4.20) fijo en el espacio (ni traslacin ni
rotacin) este sistema se denomina sistema de referencia inercial. Para este sistema la ley de
Newton es vlida, y si tenemos un slido en cada libre se cumple que:
r
r
F = mA

Vamos considerar tambin que tenemos un observador (atado al sistema ox1 x 2 x3 ) que se
est moviendo (por simplicidad solo vamos considerar traslacin). El sistema ox1 x 2 x3
denominamos de sistema de referencia non-inercial. A travs de suma de vectores podemos decir
que:
r r r
X =c+ x

La derivada material de la ecuacin anterior queda:


r& r r
X = c& + x&

&r& &r& &r&


+x
X =c

Dt

r &r& r
+ &x&
A=c

y si multiplicamos por la masa obtenemos que:


r
&r& + m&xr&
mA = mc

r
r
&r& = Fr mc
&r&
m&x& = mA mc

r r
&r&
ma = F mc
r
Notar que para el observador ha surgido una fuerza adicional (m&c&) a la ley de

Newton, esta fuerza adicional es una fuerza ficticia o una pseudo fuerza que se denomina
de fuerza inercial.

r
mA

X2

r
x

r
X

x2

r
c

x1

X1
X3

Figura 4.20

Universidad de Castilla- La Mancha


Ciudad Real - Espaa

Draft

Por: Eduardo W. V. Chaves (2014)

MECNICA DEL MEDIO CONTINUO: PROBLEMAS RESUELTOS

354

Ejemplo 4.28

Demostrar que la aceleracin en un sistema fijo a f puede ser expresada por:


r
r
r r
r
r r
a f = ar + 2( v r ) + ( x )
r

(4.58)

donde a r y v r son, respectivamente, la aceleracin y la velocidad de una partcula con


r
respecto a un observador que est girando con el sistema x * , (ver Figura 4.18). Considerar
r r
r
tambin que = es la velocidad angular del sistema x * , y dicha velocidad es constante
en el tiempo.
Solucin:
Utilizaremos directamente la ecuacin (4.56) para obtener la velocidad:
r
r
r r
Dx
Dx

=
+ x
Dt f Dt r

r
r r r
v f = vr + x

Aplicaremos la misma definicin con el objeto de obtener la aceleracin, i.e.:


r
r r r
r r r
Dv f
r
r r r
D[v r + x ]
D[v r + x ]
=
=
+ [v r + x ]

Dt
Dt
f
r
Dt f
r
r r
r
r r r
r r
Dv D[ x ]
af = r +
+ v r + ( x )

Dt r Dt r
r
r
r
r
r r Dx
r r r
r r
Dv r D
af =
+
x+
+ v r + ( x )

Dt r Dt r
Dt r
r
r
r& r r r r r r
r r
a = a + x + v + v + ( x )
f

r
r
r& r
r r
r
r r
a f = ar +
x + 2( vr ) + ( x )
r

Como estamos asumiendo una velocidad angular constante, tenemos que & = 0 , i.e. la
aceleracin angular es cero, con eso obtenemos la ecuacin (4.58). Luego, podemos
concluir que:
r
r
r& r
r r
r
r r
a f = ar +
x + 2( v r ) + ( x )

(4.59)

Notar que para obtener la ecuacin anterior no hemos tenido que recurrir a ningn
principio de conservacin. La ecuacin anterior es nada ms y nada menos que relacionar la
aceleracin en un sistema fijo con parmetros definidos en el sistema mvil.
r

r r r

r r r

NOTA 1: Utilizando la identidad a (b c ) = (a c )b (a b )c , (ver Ejemplo 1.17),


r

r r r

r r r

r r r

podemos concluir que ( x ) = ( x ) ( ) x = ( x ) x . Notar que, si


r
= 3e 3 , (ver Figura 4.21), y si tambin adoptamos el sistema ( e r , e , e 3 ) y teniendo en
r

r r

r r r

cuenta que r = 0 , podemos obtener que ( r ) = ( r ) r = r , el cual


representa la aceleracin centrpeta, (ver Ejemplo 2.58). La Tierra gira con una tasa de
2

r
r r
2 rad
rad
rad
. Notar que el trmino ( x ) es muy
=
0.727 10 4
day 86400 s
s
r r
pequeo cuando comparado con 2( vr ) .
r r
NOTA 2: El trmino 2( v r ) , el cual fue establecido por Gustave-Gaspard Coriolis en

3 = 2

1835, est asociado con la fuerza ficticia conocida como fuerza de Coriolis. A continuacin,
r r
representaremos 2( v r ) en el sistema e i de la Figura 4.22.
Universidad de Castilla- La Mancha
Ciudad Real - Espaa

Draft

Por: Eduardo W. V. Chaves (2014)

4 LEYES FUNDAMENTALES DE LA MECNICA DEL MEDIO CONTINUO

x3 , z

r
= 3e 3

r
r

x3

355

e 3
e

r
x

e r

x2 , y

x1 , x

Figura 4.21
3

r
e 3 // x

N
e 2

Latitude
r
x

e 3
e 1
e 3 , e 2

e 2 , e 1

Equator

e 1 , e 3

S
Figura 4.22
La ley de transformacin del sistema e i al e i viene dada por:
e 1 0
1
0 e 1


e2 = sin 0 cos e 2
e cos 0 sin e
3
3

Universidad de Castilla- La Mancha


Ciudad Real - Espaa

Draft

1
0
0

B = sin 0 cos
cos 0 sin

(4.60)

Por: Eduardo W. V. Chaves (2014)

MECNICA DEL MEDIO CONTINUO: PROBLEMAS RESUELTOS

356

3e 3

r
= 3 cos( )e 2 + 3 sin( )e 3

N
e 2

e 3

S
Figura 4.23

r r
El trmino 2( v r ) puede ser obtenido como sigue:
e 1
r r
2( v r ) = 0
v r1

e 2
e 3
3 cos( ) 3 sin( )
vr 2
vr3

= 2e 1 [ 3 cos( )v r 3 3 sin( )v r 2 ] 2e 2 [ 3 sin( )v r1 ] + 2e 3 [ 3 cos( )v r1


= 2[ 3 cos( )v r 3 3 sin( )v r 2 ]e 1 + 2[ 3 sin( )v r1 ]e 2 2[ 3 cos( )v r1 ]e 3

(4.61
)

El parmetro f = 23 sin( ) es conocido como parmetro de Coriolis. Para pequeos valores


de vr 3 la ecuacin anterior se reduce a:
r
r r
Dv r
= 2( v r ) = [23 sin( )vr 2 ]e 1 + [ 23 sin( )vr1 ]e 2 = [ f vr 2 ]e 1 + [ f vr1 ]e 2

Dt r
Dv r 1
Dt = f vr 2

Dv r 2 = f v
r1
Dt

Figura 4.24: Efecto de Coriolis (Ref.: Wikipedia Coriolis effect).

Universidad de Castilla- La Mancha


Ciudad Real - Espaa

Draft

Por: Eduardo W. V. Chaves (2014)

4 LEYES FUNDAMENTALES DE LA MECNICA DEL MEDIO CONTINUO

357

NOTA 3: Deflexin de un cuerpo en cada vertical


Una aplicacin muy sencilla del efecto de Coriolis se presenta a continuacin. Vamos
considerar un observador en la superficie de la Tierra. Vamos considerar tambin que un
slido de masa m est en cada libre con las siguientes condiciones de contorno e inicial: en
d
x3 = v3 = 0) , (v1 = 0) , (v2 = 0) . Como el slido est cayendo,
dt
calcularemos la deflexin del slido, i.e. vamos obtener x1 relacionada con el observador el
t = 0 . ( x3 = h) , ( x1 = 0) , (

cual est atado al sistema que est girando con la Tierra. Adoptaremos el mismo sistema
descrito en la Figura 4.22.
r

Teniendo en cuenta la segundo Ley de Newton ( F = ma f ) (aplicada para un sistema de


referencia inercial), podemos decir que
r
r
r r
r
r r
F = m[ar + 2( v r ) + ( x )]

r
r r
ar = ge 2( vr )

r
r
r r
r r
ma r = F 2m( v r ) = mge 3 2m( v r )

2[3 cos( )vr 3 3 sin( )vr 2 ] 23 cos( )vr 3


r

2[3 sin( )vr1 ]


(a r ) i =
0
=

g
2[3 cos( )vr1 ] g

donde la aceleracin a f viene dada por (4.59), y estamos considerando que el trmino
r
r r
r r
( x ) es muy pequeo cuando comparado con el trmino 2( vr ) cuyas
componentes vienen dadas por (4.61). Luego
d 2 x1
2
ar1 dt2 23 cos( )vr 3
r

d x
(ar )i = ar1 = 22 =
0

a dt2
g

r1 d x3
2
dt

(4.62)

Notar que
d 2 x3
dx
= g integrando
3 = gt + C1

vr 3 = gt
2
dt
dt
dx
t2
t2
3 = gt integrando
x3 = g + C 2

x3 = g + h
dt
2
2

donde

hemos

tenido

t = 0 (vr 3 = 0)
x3 = g

en

C1 = 0 ,

cuenta
y

las

condiciones

t = 0 ( x3 = h)

iniciales
i.e.
Notar

C2 = h .

en
que

t2
gt 2
. Considerando la aceleracin vr 3 = gt dentro de la primera
+h=0h=
2
2

componente de (4.62) podemos obtener que:

d 2 x'1
dx '1
t2
integrando
=

v
=

gt

+ C1 = vr1
2
cos(
)
2
cos(
)
2
cos(
)
3
r
3
3
3
2
dt
dt 2

donde la constante de integracin se obtiene a travs de la condicin inicial:


at (t = 0) {vr1 = 0

C1 = 0

t3
dx '1
integrando
= vr1 = 3 g cos( )t 2
x'1 = 3 g cos( ) + C2
dt
3

Universidad de Castilla- La Mancha


Ciudad Real - Espaa

Draft

Por: Eduardo W. V. Chaves (2014)

MECNICA DEL MEDIO CONTINUO: PROBLEMAS RESUELTOS

358

1
3

Notar tambin que C2 = 0 , con lo cual obtenemos que x'1 = 3 g cos( )t 3 . Como el slido
empez su cada a partir de una altura h , podemos decir que la siguiente relacin se cumple
h=

1 2
gt t =
2

2h
, con lo cual, la ecuacin anterior queda:
g
3

1
g 2h 2
x'1 = 3 g cos( )t 3 = 3 cos( )
3
3 g

NOTA 4: Aceleracin debido a la esfericidad


Sistema local e 1 (este)- e 2 (norte)- e 3 (radialmente haca fuera)

e 2

r
r

Latitud

O
e 3
e 1

r
x

e 3

e 1

e 3
e 2

Ecuador

N Norte
S Sur
E Este
O Oeste

r
r
r = x cos

e 1

Polo Sur
e 3

vr 2

r d
r
d
= x cos
( e1 )
v r1 = r
dt
dt

PN

e 2

r
r
r
x

e 3

e 2

PN

v r1

r
r

PS

r d
( e 2 )
vr 2 = x
dt

e 1

E
e 1

Figura 4.25

Universidad de Castilla- La Mancha


Ciudad Real - Espaa

Draft

Por: Eduardo W. V. Chaves (2014)

4 LEYES FUNDAMENTALES DE LA MECNICA DEL MEDIO CONTINUO

359

Previamente hemos obtenido que la ley de transformacin del sistema e i al e i , (ver


ecuacin (4.57)), viene dada por:
cos
A = sin
0

sin
cos
0

0
0
1

(4.63)

y la ley de transformacin del sistema e i al e i (ver ecuacin (4.60)) viene dada por:
1
0
0

B = sin 0 cos
cos 0 sin

(4.64)

Luego, la ley de transformacin del sistema e i al e i viene dada por:


1
0 cos
0

C = BA = sin 0 cos sin


cos 0 sin 0

sin
cos
0

0 sin
0 = sin cos
1 cos cos

cos
sin sin
cos sin

0
cos
sin

La tasa de cambio de C viene dada por:

& cos
& sin
0

d (C ) & &
C = ( cos cos + & sin sin ) ( & cos sin & sin cos ) & sin
dt
( & sin cos & cos sin ) ( & sin sin + & cos cos ) & cos

Efectuando la operacin algebraica = C&C T podemos obtener que:

0
& sin & cos


v
& = r r1 sin
0
= C&C T = & sin
& cos
x cos
0
&


v r1
xr cos cos

0
vr1 tan vr1

1
0
= r vr1 tan
vr 2
x
vr1
0
vr 2

v r1
sin
r
x cos
0
vr 2
r
x

v
r r1 cos
x cos

vr 2
r

Como era de esperar, resulta ser una matriz antisimtrica. Notar que, segn la Figura 4.25
v
d
d v r 2
= r r1
= r .
las siguientes relaciones se cumplen: &
y &
dt

x cos

dt

Aplicamos la definicin (ver ecuacin (4.55))


r
r
Dv
r r
Dv

=
+ vr
Dt

f Dt r
r r
r
Notar tambin que T v r = v r se cumple, luego:

Universidad de Castilla- La Mancha


Ciudad Real - Espaa

Draft

Por: Eduardo W. V. Chaves (2014)

MECNICA DEL MEDIO CONTINUO: PROBLEMAS RESUELTOS

360

0
r
1
v r = r vr1 tan
x
vr1

vr1vr 2 tan + vr1vr 3


v r1 v r1

1 2

vr 2 vr 2 = r vr1 tan + vr 2 vr 3
x

vr21 vr22
0 vr 3

vr1 tan

0
vr 2

vr1vr 2 tan + vr1vr 3


r
r

1 2
a f = a r + r vr1 tan + vr 2 vr 3
x

vr21 vr22

vr1vr 2 tan + vr1vr 3


r
r

1 2
a r = a f r vr1 tan + vr 2 vr 3
x

vr21 vr22

(4.65)

NOTA 5: Aceleracin de Coriolis + aceleracin de Curvatura


Teniendo en cuenta el trmino de Coriolis (ver Ec. (4.61) y (4.59)) y el trmino debido a la
curvatura, la aceleracin viene dada por:
r
r
r r
r
r
r r
a f = a r + 2( v r ) + T v r + ( x )

(4.66)

vr1vr 2 tan( ) + vr1vr 3


2[3 vr 3 cos( ) 3 vr 2 sin( )]
r r

1 2
T r
2( v r ) + v r =
2[3 vr1 sin( )]
+ r vr1 tan( ) + vr 2 vr 3

2[3 vr1 cos( ) ]


vr21 vr22

(4.67)

donde

Ejemplo 4.29
Consideremos un movimiento de slido rgido en el cual est libre de fuerzas y
consideremos que est libre de torque. a) Demostrar las ecuaciones de Euler del
movimiento:
& 1 = 2 3 (I 2 I3 )
I1

& 2 = 13 (I3 I1 )
I 2
I
3 & 3 = 12 (I1 I 2 )

Ecuaciones de Euler del movimiento

(4.68)

donde Ii son los momentos de inercias principales relacionados con el sistema G xyz con
origen en el centro de masa G , i son las componentes de la velocidad angular del slido
r

( ), y & i

Di
indica la derivada material de la velocidad angular.
Dt

b) Demostrar que la energa cintica es constante.


Solucin:
Las ecuaciones de gobierno para movimiento de slido rgido (ver Ejemplo 4.22) son:
r

F =ma

r&
= HG

Como el slido est libre de fuerzas y de torque, tenemos que:


r

F =0

r r&
= 0 = HG

r
A continuacin, evaluaremos el trmino H& G .

Universidad de Castilla- La Mancha


Ciudad Real - Espaa

Draft

Por: Eduardo W. V. Chaves (2014)

4 LEYES FUNDAMENTALES DE LA MECNICA DEL MEDIO CONTINUO

361

Consideraremos un sistema mvil G xyz atado al slido, (ver Figura 4.26), luego, en esta
r r
situacin tenemos que = .
r
- velocidad angular del slido

r
HG

- velocidad angular del sistema mvil x

=
x3

x1 , x2 , x3 - ejes principales de inercia

x2

x3

x1
G

x1

x 2

G - centro de masa

Figura 4.26
r
En el Ejemplo 4.27 hemos obtenido una forma eficiente de a la hora de calcular H& G , (ver
r r
ecuacin (4.55)), y considerando = obtenemos que:

r
DH G

Dt

r
r
r

DH G
DH G
r
r r
=
+ HG =
+
H Gxyz

Dt
Dt
f
r

Para este problema tenemos que:


I 1
(I Gxyz ) ij = 0
0

0
I2
0

0
0
I 3

1
r

() i = 2

3

Momento angular:
r
r
H Gxyz = I Gxyz
r
( H Gxyz )1 I1
r

( Hr Gxyz ) 2 = 0
( H

Gxyz ) 3 0

componente

s
0
I2
0

r
r
( H Gxyz ) i = (I Gxyz ) ij () j

0 1 I 1 1


0 2 = I 2 2
I 3 3 I 3 3

La tasa del momento angular:


Notar que, ya que el sistema G xyz est atado al slido, la distribucin de masa para este
sistema no cambia con el tiempo, con lo cual I Gxyz tan poco cambiar en el tiempo, i.e.
I& Gxyz = 0 . Con eso podemos obtener que:

r
DH G

Dt

r
DH Gxyz
=
Dt

r&

H Gxyz

Gxyz

Universidad de Castilla- La Mancha


Ciudad Real - Espaa

componente

Draft

r
( H&
) &
& 1 I1
&1
r Gxyz 1 I11 + I1

&

&
& 2 = I 2
& 2
( H Gxyz ) 2 = I 2 2 + I 2
I& + I
r&
& I
&
( H Gxyz ) 3 3 3 3 3 3 3
Por: Eduardo W. V. Chaves (2014)

MECNICA DEL MEDIO CONTINUO: PROBLEMAS RESUELTOS

362

e 1
r r
H Gxyz = 1
I 1 1

e 2
2

e 3
3

I 22

I 3 3

= ( 2 I 3 3 3 I 2 2 )e 1 + ( 3 I1 1 1 I 3 3 )e 2 + (1 I 2 2 2 I1 1 )e 3
= 2 3 (I 3 I 2 )e 1 + 1 3 (I1 I 3 )e 2 + 1 2 (I 2 I 1 )e 3

Componentes:

r r
H Gxyz

2 3 ( I 3 I 2 )

= 1 3 (I1 I 3 )
(I I )
1 2 2 1

Teniendo en cuenta los trminos anteriores podemos calcular


r
DH G

Dt

r
r

r r
= H& Gxyz +
H Gxyz = 0

cuyas componentes son:

{ }

r&
r r
H Gxyz i + H Gxyz

} {}
i

r
= 0

& 1 2 3 (I 3 I 2 ) 0
I1



& 2 + 1 3 (I1 I 3 ) = 0
I 2
I


3 & 3 1 2 (I 2 I1 ) 0

& 1 = 2 3 (I 2 I 3 )
I 1

& 2 = 1 3 (I 3 I 1 )
I 2
I
3 & 3 = 1 2 (I1 I 2 )

b) La energa cintica para un movimiento de slido rgido (ver Ejemplo 4.24 ecuacin
(4.45)) viene dada por:
1
2

1r
2

K(t ) = mv 2 + I
Ya que el sistema adoptado est en el centro de masa v = 0 se cumple, con lo cual:
1
1
K(t ) = k I kj j = [1
2
2

I1
3 ] 0
0

0
I2
0

0 1
1
0 2 = I1 12 + I 2 22 + I 3 32
2
I 3 3

La tasa de la energa cintica queda:

] [

1 D
1
D
& 1 + 2 2 I 2
& 2 + 23I 3
&3
K(t ) = K& (t ) =
I112 + I 2 22 + I 332 = 21 I1
2 Dt
2
Dt
& 1 + 2I 2
& 2 + 3I 3
&3
= 1 I1

Si consideramos las ecuaciones de Euler del movimiento (4.68) las ecuaciones anteriores
pueden ser reescritas como:
K& (t ) = 1 I1& 1 + 2I 2 & 2 + 3I3& 3 = 1 23 (I 2 I3 ) + 213 (I3 I1 ) + 312 (I1 I 2 )
= 1 2 3 (I 2 I3 + I3 I1 + I1 I 2 )
=0

Demostrando as que la energa cintica es constante.

Universidad de Castilla- La Mancha


Ciudad Real - Espaa

Draft

Por: Eduardo W. V. Chaves (2014)

4 LEYES FUNDAMENTALES DE LA MECNICA DEL MEDIO CONTINUO

363

Ejemplo 4.30
Obtener una forma simplificada de las ecuaciones de gobierno de slido rgido para el caso
particular:
a) Slido rgido rotando alrededor de un eje fijo y sin fuerzas.
Solucin:
Consideraremos el sistema fijo OX 1 X 2 X 3 y adoptaremos eje de rotacin el eje x3 , (ver
Figura 4.27) y consideraremos tambin el sistema mvil Ox1 x2 x3 atado al slido.

x1

X 3 , x3

X1

= = 3 e 3 = 3 E 3

3
r
- velocidad angular del slido

x2

- velocidad angular del sistema Ox

X2

sistema OX 1 X 2 X 3 => base ortonormal (E 1 , E 2 , E 3 )


sistema Ox1 x2 x3 => base ortonormal (e 1 , e 2 , e 3 )
Figura 4.27
Si el slido est libre de fuerzas, las ecuaciones de gobierno se reduce a:
r

F =0

r&
= HO

r
donde H& O puede ser calculado a travs de

r
r&
DH O
HO
Dt

DH Oxr
=
r Dt
OX

r
r

DH Oxr
r
+ H Oxr =
r
Dt
Ox

r r
+
H Oxr
r
Ox

Momento angular:

r
r
r
r
componente s
( H Oxr ) i = (I Oxr ) ij ( ) j
H Oxr = I Oxr

r
r
( H& r ) I
( H Oxr )1 I O 11
&
I O 12 I O 13 0 I O 13 3
Ox 1
O 13 3
r r

tasa r& r

& 3
I O 22
( H Ox ) 2 = I O 23
I O 23 0 = I O 23 3
( Hr Ox ) 2 = I O 12
( Hr& r ) I
( H r ) I
&
I O 33 3 I O 33 3
I O 23
O 13
Ox 3
Ox 3 O 33 3
r r
Necesitamos calcular el vector H Oxr :

r r
H Oxr =

e 1
0

e 2
0

I O 13 3

I O 23 3

e 3
3 = I O 23 32 e 1 I O 13 32 e 2
I O 33 3

luego

Universidad de Castilla- La Mancha


Ciudad Real - Espaa

Draft

Por: Eduardo W. V. Chaves (2014)

MECNICA DEL MEDIO CONTINUO: PROBLEMAS RESUELTOS

364

Aplicando

&3
& 3 I O 2332 I O 2332 IO13
I O13
r&


2
2
& 3
& 3 + I O133 = I O133 I O 23
( H O ) i = I O 23
I

&3
0
IO 33
O 33 & 3

r
r&
M O = H O podemos obtener el siguiente sistema de ecuaciones:

M
M
M

O1
O2
O3

M
M
M

&3
= I O 2332 IO13
2
&3
= IO133 I O 23

X
Y

&3
= I O 33

donde & = es la aceleracin angular.


NOTA: Si el slido es prismtico y si adoptamos el eje prismtico el mismo que el eje de
rotacin las ecuaciones anteriores se reducen a:

M
M
M

O1
O2
O3

M
M
M

=0

=0

&3
= I O 3

ya que en esta situacin el sistema Ox1 x2 x3 es el sistema principal de inercia, e.g.:


z
x

I O 1

I Gxyz = 0
0
IO1 = I O 2

0
IO 2
0

0
I O 3

Ejemplo 4.31
Un cuerpo rgido consiste de dos masas m en cada extremidad de una barra sin masa de
longitud 2l . La barra est inclinada con un ngulo respecto de la vertical y rota con
velocidad angular como se indica en la Figura 4.28.
a) Encontrar la cantidadr de momento angular del cuerpo;
b) Encontrar el Par ( M ) para mantener esta rotacin.
Solucin: Aplicaremos las ecuaciones de gobierno de movimiento de slido rgido, ver
Ejemplo 4.22. Adoptaremos un sistema fijo en el espacio OXYZ y un sistema mvil Oxyz
unido al slido, ver Figura 4.29.
Tensor de Inercia I (sistema Oxyz ):
IOxyz

0
0

= 0 2ml 2
0
0

0
0
2ml 2

Velocidad angular (sistema Oxyz ):

Universidad de Castilla- La Mancha


Ciudad Real - Espaa

Draft

Por: Eduardo W. V. Chaves (2014)

4 LEYES FUNDAMENTALES DE LA MECNICA DEL MEDIO CONTINUO

365

r
= cos()i + sin()j + 0k
r

donde es el mdulo de .

l
m

Figura 4.28

Y
r
= J

Zz

l
m

r
Cantidad de movimiento angular H O :
r
r
HO = I

Figura 4.29

0
0 cos()
0
H Ox 0

2
2
H Oy = 0 2ml
0 sin() = 2ml sin()
H Oz 0

0
2ml 2
0
0
r
H = 0i + 2ml 2 sin() j + 0k
O

Universidad de Castilla- La Mancha


Ciudad Real - Espaa

Draft

Por: Eduardo W. V. Chaves (2014)

MECNICA DEL MEDIO CONTINUO: PROBLEMAS RESUELTOS

366

El Par (torque) M viene dado por:


r r&
r&
r r
M = H O = ( H O ) Oxyz + H O
r
r&
r r
Podemos observar que (H O ) Oxyz = 0 y = (sistema mvil unido al slido), luego:

i
j

k
r r&

r r
M = H O = H O = cos()
sin()
0

0
2ml 2 sin() 0

r r&
2
2
2
M = H O = cos()2ml sin()k = ml sin(2)k
Y

r
= cos()i + sin()j + 0k
r
H O = 0i + 2ml 2 sin() j + 0k

r
HO

X
Zz

l
m
x

Figura 4.30
Solucin utilizando el sistema OXYZ
Matriz de transformacin del sistema OXYZ al sistema Oxyz :
T

cos 2 sin 2 0

sin () cos() 0

A = sin cos 0 = cos() sin () 0

2

2

0
1
0
0
1

Tensor de Inercia en el sistema OXYZ :


I OXYZ = A T I Oxyz A
I OXYZ

0
sin () cos() 0 0

= cos() sin () 0 0 2ml 2


0
0
1 0
0

0 sin () cos() 0
0 cos() sin () 0
2ml 2 0
0
1

resultando

Universidad de Castilla- La Mancha


Ciudad Real - Espaa

Draft

Por: Eduardo W. V. Chaves (2014)

4 LEYES FUNDAMENTALES DE LA MECNICA DEL MEDIO CONTINUO

I OXYZ

367

2ml 2 cos 2 ()
2ml 2 sin () cos()
0

2
2
2
= 2ml sin ( ) cos()
2ml sin ()
0

0
0
2ml 2

Cantidad de movimiento angular:


r
r
H OXYZ = A T H Oxyz

0
H Ox

H = 2ml 2 sin()
Oy

H Oz

donde

2
0
H OX sin ( ) cos() 0
2ml cos() sin()

H OY = cos() sin () 0 2ml 2 sin() = 2ml 2 sin 2 ()

H OZ 0

0
1
0
0

El Par:

M = 2 ml 2 sin(2)K

Ejemplo 4.32
Considrese un giroscopio, (ver Figura 4.31) el cual consiste de un aro externo y uno
interno y de un rotor de forma cilndrica y masa m . El aro externo puede girar alrededor
del eje Z definiendo el ngulo (ngulo de precesin), el aro interno puede girar
alrededor el eje y definiendo el ngulo (ngulo de nutacin), el rotor puede rotar
alrededor del eje z definiendo el ngulo (ngulo de rotacin). Los ngulo ( , , ) son
conocidos como ngulos de Euler. Obtener las ecuaciones de gobierno para el giroscopio.
Considerar que el tensor de inercia del rotor con respecto al sistema Oxyz viene dado por:
I 0 0
(I Oxr )ij = 0 I 0
0 0 I

Solucin:
Adoptaremos la base ortonormal del sistema fijo OXYZ por ( I , J , K ), para el sistema
mvil Oxyz adoptaremos la base ortonormal ( i , j , k ).
Velocidad angular del rotor (ver Figura 4.32):
r
+ & j + & k
= & K
= [& sin( ) i + +& cos( )] k ] + & j + & k
= & sin( ) i + & j + [& + & cos( )] k

Las ecuaciones de gobierno para un movimiento de slido rgido son:


r

F = m a

r&
= HO

r
donde H& O puede ser calculado a travs de

r
r
r&
DH O
DH Oxr
r r
=
+ H Oxr
H O

Dt OXr Dt Oxr

Universidad de Castilla- La Mancha


Ciudad Real - Espaa

Draft

Por: Eduardo W. V. Chaves (2014)

MECNICA DEL MEDIO CONTINUO: PROBLEMAS RESUELTOS

368

- ngulo de precesin
Z

Eje de precesin

- ngulo de nutacin
- ngulo de rotacin

&
z

aro externo

aro interno
y

&
&

x
X

Figura 4.31

& K

& cos( )

& sin( )i

Figura 4.32

Universidad de Castilla- La Mancha


Ciudad Real - Espaa

Draft

Por: Eduardo W. V. Chaves (2014)

4 LEYES FUNDAMENTALES DE LA MECNICA DEL MEDIO CONTINUO

369

Momento angular:

r
r
r
r
componente

s
H Oxr = I Oxr
( H Oxr ) i = (I Oxr ) ij () j
r
( H Oxr )1 I 0 0 & sin( ) I & sin( )

r r

&
I &
( Hr Ox ) 2 = 0 I 0
=

( H r ) 0 0 I [& + & cos( )] I [& + & cos( )]

Ox 3

y su tasa viene dada por:


r
( H& r )
&& sin( ) + & cos( )& ]
I& sin( ) I[

r& Ox 1 D

I&
I&&
( H Oxr ) 2 =
=

( Hr& r ) Dt I [& + & cos( )] I D [& + & cos( )]


Ox 3

Dt

Notar que debido a la simetra del rotor en tensor de inercia para el sistema Oxyz no
cambia con el tiempo.
r r
r
Necesitamos calcular el vector H Oxr , donde es la velocidad angular del sistema
mvil. Notar que el sistema mvil puede girar alrededor de K y alrededor de j , y que no
puede girar alrededor del eje k , (ver Figura 4.32), luego, la velocidad angular del sistema
mvil viene dada por:
r
= & K + & j = [& sin( ) i + & cos( )] k ] + & j = & sin( ) i + & j + & cos( ) k
i
j
r
H Oxr = & sin( ) &
I& sin( ) I&

k
& cos( )
I [& + & cos( )]

= {& I [& + & cos( )] & I& cos( )}i {I& sin( ) [& + & cos( )] + & I& sin( ) cos( )}j
+ {I& & sin( ) + I& & sin( )}k

& I [& + & cos( )] & I& cos( )


r

&
( H Oxr ) i = I sin( ) [& + & cos( )] & 2 I sin( ) cos( )

luego
r
DH r
r&
r r
Ox

+
H Oxr ) i
( H O ) i =
(

Dt Oxr i
&&

& &

& I [& + & cos( )] & I& cos( )


I [ sin( ) + cos( )]

2
I&&
=
+ I& sin( ) [& + & cos( )] & I sin( ) cos( )
I D [& + & cos( )]


Dt

&&
& &
& & &
I[ sin( ) + 2 cos( )] + I [ + cos( )]

= I& sin( ) [& + & cos( )] & 2 I sin( ) cos( ) + I&&

D & &
I
[ + cos( )]

Dt

r
r&
Aplicando
M O = H O podemos obtener el siguiente sistema de ecuaciones:

Universidad de Castilla- La Mancha


Ciudad Real - Espaa

Draft

Por: Eduardo W. V. Chaves (2014)

MECNICA DEL MEDIO CONTINUO: PROBLEMAS RESUELTOS

370

M
M
M

M
M
M

O1
O2
O3

x
y
z

&& sin( ) + 2& & cos( )] + I & [& + & cos( )]


= I[
= I [&& & 2 sin( ) cos( )] + I& sin( ) [& + & cos( )]
D & &
=I
[ + cos( )]
Dt

(4.69)

NOTA: Caso Particular: Precesin estacionaria.


En este caso tenemos que las variables , & y & son constantes. Con eso tenemos que
& = 0 , && = 0 y && = 0 , y
r
r
( H Oxr ) 1 I & sin( )
( )1 & sin( )
r & componente s r

r r
0
( ) 2 =
0
; = K
( Hr Ox ) 2 =
;
( H r ) I [& + & cos( )]
(r ) & cos( )
3
Ox 3

r
() 1 x & sin( )

r
0

( ) 2 = y =
r & &

+
)
[
cos(
)]
3

Las ecuaciones (4.69) quedan:

M
M
M

O1
O2

O3

M
M

x
y

=0
= I & 2 sin( ) cos( ) + I& sin( ) [& + & cos( )] = [I z I & cos( )] & sin( )
144244
3
= z

=0

& = constante

&
r

= & K

z k

& k

x = & sin( )

y
r

Figura 4.33: Precesin estacionaria.


NOTA: Para mayores detalles de Movimiento de slido rgido ver Beer & Johnston (1987)
y Beer et al. (2004).

Universidad de Castilla- La Mancha


Ciudad Real - Espaa

Draft

Por: Eduardo W. V. Chaves (2014)

5 Introduccin a:
Ecuaciones Constitutivas,
PVCI, y Estrategias de
Solucin del PVCI
Ejemplo 5.1
Para un material termoelstico simple cules son las ecuaciones constitutivas y variables
libres si tenemos en cuenta la energa libre de Helmholtz ?
Solucin:
Las ecuaciones constitutivas para un material simples estn en funcin de las siguientes
variables libres:

= (F ,T )
( F , T )
F
( F , T )
(F ,T ) =
T
r
r
q0 = q0 ( F , T , Xr T )
P(F , T ) = 0

(ver Chaves (2007)). Tambin se pueden presentar en funcin de las siguientes variables

= (F ,T )
( F , T ) T
=
F

= ( E , T )
( E , T )
S = 0

E
;
( E , T )
( E ,T ) =
T
r
r
q 0 = q0 ( E , T , Xr T )

Universidad de Castilla- La Mancha


Ciudad Real - Espaa

Draft

F
( F , T )
(F ,T ) =
T
r
1 r
q = J q0 ( F , T , Xr T ) F T
r
= J 1 F q0 ( F , T , Xr T )

Por: Eduardo W. V. Chaves (2014)

MECNICA DEL MEDIO CONTINUO: PROBLEMAS RESUELTOS

372

Ejemplo 5.2
Para un determinado material elstico se conoce la expresin de la densidad de energa (por
unidad de volumen), y viene dada por:
1
2

( I E , II E ) = ( + 2 )I E2 2 II E
donde , son constantes del material. I E = I E (E ) , II E = II E (E ) son los invariantes
principales, el primer y segundo invariante principal del tensor de deformacin de GreenLagrange respectivamente. Cules son las ecuaciones constitutivas para este problema?,
justificar. Obtener tambin las expresiones explcitas de las ecuaciones constitutivas en
funcin de , , I E , II E .
Formulario
I E = I E ( E ) = Tr ( E )

II E = II E ( E ) =

1
( TrE ) 2 Tr ( E 2 )
2

I E
=1
E
II E
= Tr ( E )1 E T
E

Solucin:
La expresin de la energa est SOLO en funcin del tensor de deformacin de GreenLagrange (grandes deformaciones). Sabemos que las ecuaciones constitutivas son:

= ( E , T )
( E , T )
S = 0

E
( E , T )
( E ,T ) =
T
r
r
q 0 = q0 ( E , T , Xr T )

Teniendo en cuenta la expresin de la energa dada, concluimos que el problema es


independiente de la temperatura, ya que en la expresin de la energa dada no est en
funcin de la temperatura. Luego, slo me quedo con la ecuacin constitutiva de la tensin
y que podemos obtener como:
S = 0

( E ) ( I E , II E ) ( I E , II E ) I E ( I E , II E ) II E
=
=
+
E
E
E
E
I E
II E

= ( + 2 )I E (1) + ( 2 ) Tr ( E )1 E T
2

Simplificando la expresin anterior, y teniendo en cuenta que E T = E , I E = Tr (E ) ,


obtenemos:
S = I E 1 + 2 E

Universidad de Castilla- La Mancha


Ciudad Real - Espaa

Draft

Por: Eduardo W. V. Chaves (2014)

5 INTRODUCCIN A: ECUACIONES CONSTITUTIVAS, PVCI, Y ESTRATEGIAS DE SOLUCIN

373

Ejemplo 5.3
Considerando la energa libre de Gibbs especfica G(S, T ) = ( E , T )

S : E como

ecuacin constitutiva de energa, obtener las dems ecuaciones constitutivas para un


material termoelstico simple.
OBS: Partir del principio de que G(S, T ) no depende del gradiente de la temperatura.
Solucin:
Partimos de la forma alternativa de la desigualdad de Clausius-Duhem en la configuracin
de referencia:

1r
S : E& 0 & + T& q0 Xr T 0
T

(5.1)

Teniendo en cuenta la energa libre de Gibbs especfica podemos decir que:


& (S, T ) = & ( E , T ) 1 S& : E 1 S : E&
G

& (S, T ) + 1 S& : E + 1 S : E&


& ( E , T ) = G

y reemplazando en la desigualdad (5.1) obtenemos que:


&
1r
1 &
1
S : E& 0 G
S:E+
S : E& + T& q0 Xr T 0
(S, T ) +
0
0

T
r
& (S, T ) S& : E T& 1 q
r
0G
0
0 XT 0
T

(5.2)

Notar que S& : E = E : S& se cumple. La desigualdad anterior nos sugiere que para una
variacin de la energa libre de Gibbs tenemos que tener las siguientes relaciones:
Deformacin para una variacin de tensin; Entropa para una variacin de temperatura;
Flujo de calor para un gradiente de temperatura.
& (S , T ) puede ser expresado como:
El trmino G

DG(S, T ) &
G(S, T ) & G(S, T ) &
T
G(S, T ) =
:S +
T
Dt
S

Y reemplazando en la ecuacin (5.2) obtenemos que:


r
& (S , T ) E : S& T& 1 q
r
0G
0
0 XT 0
T
G(S , T ) &
G(S, T ) &
1r
:S 0
T E : S& 0T& q0 Xr T 0
0
S
T
T
r
1
G(S , T )

G(S, T )

0
+ E : S& 0
+ T& q 0 Xr T 0
S

T
T

(5.3)

La desigualdad anterior debe cumplir para todo proceso termodinmicamente admisible. Si


r
r
tenemos un proceso isotrmico (T& = 0) y adiabtico (q 0 = 0) , la desigualdad de entropa
queda:
G(S , T )

0
+ E : S& 0
S

Universidad de Castilla- La Mancha


Ciudad Real - Espaa

Draft

(5.4)

Por: Eduardo W. V. Chaves (2014)

MECNICA DEL MEDIO CONTINUO: PROBLEMAS RESUELTOS

374

cuya desigualdad tambin debe cumplir para todo proceso. Luego si para una dada
variacin S& se cumple la desigualdad anterior, podemos aplicar un proceso tal que S& = S& ,
violando as el principio de la desigualdad de entropa. Luego, la desigualdad anterior nunca
ser violada si y solo si

G(S, T )
+ E =0
S

E = 0

G(S, T )
S

Teniendo en cuenta la ecuacin anterior (ecuacin constitutiva de deformacin) en la


desigualdad (5.3), obtenemos que:
1r
G(S, T )

G(S, T )

0
+ E : S& 0
+ T& q0 Xr T 0
S
T

(5.5)
G(S, T )
& 1 r
r
0
+ T q0 X T 0
T
T

r
Si ahora tenemos un proceso donde Xr T = 0 (campo de temperatura homogneo), la

desigualdad queda:
G(S, T )

0
+ T& 0
T

La desigualdad anterior nunca ser violada si y solo si


G(S, T )
+=0
T

G(S, T )
T

Que es la ecuacin constitutiva de entropa. Luego, las ecuaciones constitutivas son:


Ecuacin constitutiva de energa G = G(S, T )
G(S, T ) g(S, T )
=
S
S
G(S , T )
Ecuacin constitutiva de entropa =
T
r
r
Ecuacin constitutiva de flujo de calor q 0 = q 0 ( Xr T )

Ecuacin constitutiva de deformacin E = 0

(5.6)

donde g = 0 G . Notar que nuestras variables libres son (S , T ) .


Ejemplo 5.4
Demostrar que para un proceso adiabtico e isotrmico, y sin tasa de cambio de tensin, la
energa libre de Gibbs especfica no puede crecer.
Solucin:
Empezaremos directamente a partir de la desigualdad (5.3):
r
& (S , T ) E : S& T& 1 q
r
0G
0
0 XT 0
T

(5.7)

Teniendo en cuenta un proceso isotrmico y adiabtico las siguientes relaciones se cumplen


r
r
T& = 0 , q0 = 0 , y si consideramos que no hay tasa de cambio de tensin S& = 0 se cumple.
Con lo cual la desigualdad en (5.7) reduce a :
& (S, T ) 0
0G

Universidad de Castilla- La Mancha


Ciudad Real - Espaa

Draft

(5.8)

Por: Eduardo W. V. Chaves (2014)

5 INTRODUCCIN A: ECUACIONES CONSTITUTIVAS, PVCI, Y ESTRATEGIAS DE SOLUCIN

375

& (S, T ) 0 para que la desigualdad de


ya que 0 > 0 , concluimos que se debe cumplir que G
entropa se cumpla.

Ejemplo 5.5
a) Hacer el planteamiento de las ecuaciones de gobierno para un problema de slidos con
las siguientes caractersticas: Proceso isotrmico y adiabtico, rgimen de pequeas
deformaciones, y relacin lineal entre tensin y deformacin.
b) Una vez establecida la relacin lineal entre tensin-deformacin, obtener dicha relacin
para que cumpla que ( ) sea una funcin-de-tensores istropa de valor tensor de
segundo orden.
Solucin:
Para un proceso isotrmico y adiabtico la temperatura y la entropa no juega ningn papel.
Para un rgimen de pequea deformaciones tenemos que:
r

Tensor de deformaciones: E e = sym u


Tensor de Tensiones: P S
F 1

Xr xr , con esta aproximacin la densidad de masa deja

de ser incgnita.
Teniendo en cuenta las ecuaciones fundamentales:
Ecuaciones Fundamentales de la Mecnica del Medio Continuo
(Configuracin Actual)
Ecuacin de continuidad de masa
(Principio de la conservacin de la masa)
Ecuaciones del Movimiento
(Principio de la conservacin del momento
lineal)
Simetra del tensor de Tensiones de Cauchy
(Principio de la conservacin del momento
angular)
Ecuacin de Energa
(Principio de la conservacin de la
Energa)
Desigualdad de Entropa
(Principio de la Irreversibilidad)

r
D
+ ( xr v ) = 0
Dt
r
r
xr + b = v&

(1 ecuacin)
(3 ecuaciones)

= T

(5.10)
(5.11)

u& = : D xr q + r (1 ecuacin)
r

(5.9)

1
T

& ( x, t ) + : D

1
1 r
u& 2 q xr T 0
T
T

(5.12)
(5.13)

Solo quedamos con las siguientes ecuaciones:


1) Ecuaciones de Movimiento

r
r
+ b = v&

2) Ecuacin de Energa

0 u&( X , t ) = S : E& Xr q 0 + 0 r ( X , t )
donde u es la energa interna especfica, y se cumple que

u& = : &

Du
D
[ + T ] = & , donde
=
Dt Dt

es la energa libre de Helmholtz especfica. Tambin se cumple que:

Universidad de Castilla- La Mancha


Ciudad Real - Espaa

Draft

Por: Eduardo W. V. Chaves (2014)

MECNICA DEL MEDIO CONTINUO: PROBLEMAS RESUELTOS

376

& = & e = : &


donde e es la densidad de energa de deformacin, en el cual & e = & + & = & .
Verificamos a travs de la desigualdad de entropa que es un proceso sin disipacin de
energa, es decir, toda energa que se almacena debido al incremento de se recupera con
la disminucin de .
3) De las Ecuaciones Constitutivas (ver Ejemplo 5.1) solo quedamos con:

= ( )
S=

( ) e ( )
=
= ( )

es decir, la energa ( ) y la tensin son funciones solamente de la deformacin. Si


( ) &
calculamos la tasa de la energa libre de Helmholtz & () =
: , y reemplazamos en la

expresin de la energa & = & e = : & , concluimos que:

( ) & & e ( ) &


: =
: = : &

e ( )

Luego, la ecuacin de energa es una ecuacin redundante, es decir, si conozco la tensin


puedo conocer la energa y vise-versa. Resumimos as las ecuaciones de gobierno para el
problema propuesto:
Ecuaciones de Movimiento:

r
r
r
&r& (3 ecuaciones)
+ b = a = v& = u

Ecuacin Constitutiva en Tensin:


( ) =

Ecuaciones Cinemticas:

(5.14)

e ( )
(6 ecuaciones)

r
= sym u (6 ecuaciones)
r
Como incgnitas tenemos: (6), u (3), (6), un total de 15 incgnitas y 15 ecuaciones

luego, el problemas est bien planteado. Para que el conjunto de ecuaciones en derivada
parciales anteriores tenga solucin nica es necesario introducir las condiciones de
contorno e inicial, constituyendo as en un Problema de Valor de Contorno Inicial (PVCI) del
Problema Elstico. El problema que acabamos de plantear es el Problema Elstico Lineal que es
el tema del prximo captulo. Las condiciones de contorno e inicial para este problema son:
Condiciones de contorno en desplazamiento, en S u :
r r
r r
u( x , t ) =u* ( x , t )

r
r
ui ( x, t ) = u i * ( x, t )

(5.15)

r
jk n k = t j * ( x , t )

(5.16)

r
r
u i ( x , t = 0) = u 0 i ( x )
r
u& 0 i ( x ) = v 0 i

(5.17)

Condiciones de contorno en tensiones, en S :


r r
r
( x , t ) n = t * ( x , n , t )

Condiciones iniciales ( t = 0 ):
r r
r
u( x , t = 0) = u 0
r r
r r
u 0 ( x , t )
r r
= u& 0 ( x, t ) = v 0 ( x )
t
t =0

En el caso de un problema esttico o casi-esttico, las ecuaciones de movimiento recaen en


las ecuaciones de equilibrio y las condiciones iniciales son redundantes.
Universidad de Castilla- La Mancha
Ciudad Real - Espaa

Draft

Por: Eduardo W. V. Chaves (2014)

5 INTRODUCCIN A: ECUACIONES CONSTITUTIVAS, PVCI, Y ESTRATEGIAS DE SOLUCIN

Su
dV

377

r r
t * ( x)

r r

b( x )
n

Figura 5.1: Slido sobre acciones externas.


En el apartado Serie de Tensores, captulo 1, hemos visto que podemos aproximar un
tensor a travs de la serie:
2 ( 0 )
1
1 ( 0 )
1
( ) ( 0 ) +
: ( 0 ) + ( 0 ) :
: ( 0 ) + L
0!

1!
2!
( 0 )
2 ( 0 )
1
: ( 0 ) + ( 0 ) :
: ( 0 ) + L
0 +

Considerando el punto de aplicacin 0 = 0 , y ( 0 ) = 0 = 0 , y adems teniendo en


cuenta que la relacin - es lineal, podemos despreciar los trminos de orden superior,
obteniendo entonces que:
( ) =

( 0 )
2 e ( 0 )
: =
: = Ce :

donde C e =

ij =

ij
kl

kl =

2 e ( 0 )
kl = C eijkl kl
ij kl

2 e ( 0 )
es un tensor de cuarto orden simtrico y es conocido como tensor

constitutivo elstico, que contiene las propiedades mecnicas del material.

Observemos que la energa tiene que ser de orden cuadrtica para que la relacin - sea
lineal, ver ecuacin (5.14). Utilizamos la expansin en serie para representar la densidad de
energa de deformacin, obtenemos que:
e
2 e ( 0 )
1 ( 0 )
1
: ( 0 ) + ( 0 ) :
: ( 0 ) + L


1!
2!
2 e ( 0 )
1
e
= 0 + 0 : ( 0 ) + ( 0 ) :
: ( 0 ) + L
2

2 e ( 0 )
1
= :
:
2

1
= : Ce :
2

1
0!

e ( ) = e ( 0 ) +

donde tambin hemos considerado que 0 = 0 e0 = 0, 0 = 0 .


NOTA 1: Aunque la ecuacin de energa es redundante, a la hora de establecer un mtodo
sea analtico o numrico para resolver el problema el punto de partida es a travs de
principios energticos, de ah la importancia de estudiar la energa de un sistema.
NOTA 2: La simetra de C e se comprueba fcilmente. Presenta simetra menor debido a
la simetra de y :
ij = ji

C eijkl = C ejikl

kl = lk

e
e
C ijkl
= C ijlk

y la simetra mayor es debido a que:


Universidad de Castilla- La Mancha
Ciudad Real - Espaa

Draft

Por: Eduardo W. V. Chaves (2014)

MECNICA DEL MEDIO CONTINUO: PROBLEMAS RESUELTOS

378

Ceijkl =

2 e ( ) 2 e ( )
=
= C eklij (simetra mayor)
ij kl
kl ij

NOTA 3: Para una mejor ilustracin del problema planteado, consideremos un caso
particular (caso unidimensional) donde las componentes del tensor de tensiones y de
deformaciones vienen dadas por:
0 0
ij = 0 0 0
0 0 0

0 0
e
ij = 0 0 0 11 = C1111
11 = E
0 0 0

En este caso la relacin lineal tensin-deformacin viene dada por = E (ley de Hooke) y
1
2

1
2

la densidad de energa de deformacin e = = E , y

e ( )

estado actual

()

1
2

e = E

2 e
=
=E.

energa almacenada
1
e =
2

E
0 = 0

e0 = 0

0 = 0

Figura 5.2: Relacin tensin-deformacin (caso unidimensional).


NOTA 4: Debemos enfatizar que en el caso de un proceso elstico la ecuacin constitutiva
( ) es nicamente dependiente del valor actual de , i.e. es independiente de la historia
de deformacin.
b) La funcin-de-tensor ( ) ser istropa si se cumple que:
ij ( kl ) = ij ( kl )

Teniendo en cuenta que la relacin entre - viene dada, en notacin indicial, por
e
ij ( ) = C ijkl
kl , concluimos que:
ij ( kl ) = ij ( kl )

e
e
C ijkl
kl = C ijkl
kl

e
e
C ijkl
= C ijkl

Es decir, el tensor de cuarto orden C e es un tensor istropo. Un tensor de cuarto orden


e
istropo simtrico tiene el formato C ijkl
= ij kl + ( ik jl + il jk ) . Que en notacin
tensorial viene dado por C e = 1 1 + 2 I , donde I I sym es el tensor identidad
simtrico de cuarto orden, y los parmetros y son conocidos como las constantes de
Lam. Como hemos visto en el Captulo 1, un tensor de cuarto orden simtrico viene dado
en funcin de dos constantes ( , ). Ms adelante, veremos que es posible expresar C e en
funcin de otros parmetros, e.g. ( E , ), ( , G ), donde E es el mdulo de Young, es el
coeficiente de Poisson, es el mdulo volumtrico, y G = es el mdulo de elasticidad transversal.
Universidad de Castilla- La Mancha
Ciudad Real - Espaa

Draft

Por: Eduardo W. V. Chaves (2014)

5 INTRODUCCIN A: ECUACIONES CONSTITUTIVAS, PVCI, Y ESTRATEGIAS DE SOLUCIN

379

NOTA 5: En la Figura 5.3 se muestra la relacin tensin-deformacin para un material


istropo. Es interesante observar que debido a que C e es independiente de la direccin los
tensores y comparten las mismas direcciones principales. Para un material istropo
tenemos que ( ) = (1 1 + 2 I) : = Tr ( )1 + 2 :
e ( )
( ) =
4
3
14424

lineal

istropo

( ) = C e :

( ) = Tr ( )1 + 2

Elstico

Es interesante comparar con la ecuacin constitutiva en tensin del Ejemplo 5.2 donde se
considera grandes deformaciones, pero mantiene una relacin lineal entre la tensin y
deformacin.
22

12

e
ij = C ijkl
kl

11

x1

P
ij = a ip a jq pq

22

12

ij = a ip a jq pq

11

22

22

e
ij = C ijkl
kl

12

12

11

x1

22

Material istropo
e
C ijkl

e
C ijkl

11

e kl
ij = C ijkl

11

22

Espacio
principal

e
C ijkl

11

e (kl ) = e ( kl )
x1

Figura 5.3: Relacin tensin-deformacin material istropo.


NOTA 6: Denotamos la densidad de energa de deformacin complementaria por e ( ) el cual es
una funcin de , (ver Figura 5.4), y viene dada por:

Universidad de Castilla- La Mancha


Ciudad Real - Espaa

Draft

Por: Eduardo W. V. Chaves (2014)

MECNICA DEL MEDIO CONTINUO: PROBLEMAS RESUELTOS

380

2 e ( 0 )
1 e ( 0 )
1
: ( 0 ) + ( 0 ) :
: ( 0 ) + L


1!
2!
2 e ( 0 )
1
= 0e + 0 : ( 0 ) + ( 0 ) :
: ( 0 ) + L

2
2 e ( 0 )
1
1
1
1
= :
: = :De : = : Ce :

2
2
2
1
0!

e ( ) = e ( 0 ) +

Note que, si estamos tratando con un material elstico lineal se cumple que e ( ) = e ( )
y =

e ( )
.

Densidad de energa de deformacin complementaria

a) Material elstico lineal.

1
2

()

e () = E 1

( ) = ( )
e

Densidad de energa de deformacin

1
2

e () = E

E
0 = 0

0 = 0

b) Material elstico no-lineal.


Densidad de energa de deformacin complementaria e ()
( )

Densidad de energa de deformacin - e ()

e ( ) e ( )

0 = 0

0 = 0

Figura 5.4: Densidad de energa de deformacin complementaria (caso unidimensional).

Universidad de Castilla- La Mancha


Ciudad Real - Espaa

Draft

Por: Eduardo W. V. Chaves (2014)

5 INTRODUCCIN A: ECUACIONES CONSTITUTIVAS, PVCI, Y ESTRATEGIAS DE SOLUCIN

381

NOTA 7: Notar que

e () = e () tensorial

e ( ) = : e ( ) = 0 G( ) = g( )
donde g( ) = 0 G( ) es la densidad de energa libre de Gibbs (por unidad de volumen)
con signo contrario, (ver ecuaciones en (5.6) del Ejemplo 5.2).
NOTA 8: Teniendo en cuenta la ecuacin constitutiva de tensin para un material elstico,
lineal e istropo ( ) = Tr ( )1 + 2 y considerando la descomposicin aditiva del tensor
Tr ( )
1 + dev , podemos obtener
3

en una parte esfrica y otra desviadora = esf + dev =


que:

Tr ( )

( ) = Tr ( )1 + 2 = Tr ( )1 + 2 ( esf + dev ) = Tr ( )1 + 2
1 + dev
3

dev
= +
Tr ( )1 + 2
3

= Tr ( )1 + 2 dev
= esf + dev

33
23

13
13
11

23
12

12

13

+
m

22
12

11

dev
22
12

ijdev = 2 ijdev

dev
33

23
12

m
23

12

dev
11

Tr ( ) ij = 3 Tr ( ) ij

33

23

13

23

13

22

ij = Tr ( ) ij + 2 ij

13

dev
33

23

13
13

23
12

dev
22
12

dev
11

Figura 5.5: Descomposicin de la ecuacin constitutiva.

Universidad de Castilla- La Mancha


Ciudad Real - Espaa

Draft

Por: Eduardo W. V. Chaves (2014)

MECNICA DEL MEDIO CONTINUO: PROBLEMAS RESUELTOS

382

Recordar que, si estamos en rgimen de pequeas deformaciones la deformacin


volumtrica lineal viene dada por:
v =

V
= 11 + 22 + 33 = Tr ( ) = I
dV0

Si adems sacamos la traza de ( ) = +

2
dev
Tr ( )1 + 2 , obtenemos que:
3

dev
: 1 = +
Tr ( )1 : 1 + 2 : 1
3

Tr ( ) = 3 +
Tr ( )
3

Tr ( )
2

= m = +
v
3
3

donde hemos tenido en cuenta que 1 : 1 = 3 y Tr ( dev ) = 0 . Si consideramos un estado de


tensin de compresin pura ( p > 0 ) tenemos que:
0
p 0

ij = 0 p 0
0
0 p

3 m = Tr ( ) = 3 p < 0

p = +
v = v
3

Por esta razn el coeficiente se conoce como mdulo de deformacin volumtrica, ver Figura
5.6, y viene dado por:
2
3

= +

(5.18)

As como la parte esfrica del tensor est asociada con el cambio de volumen, la parte
desviadora ( dev = 2 dev ) est asociada con el cambio de forma, y el parmetro = G
define la rigidez a dicho cambio, y G se denomina de mdulo de elasticidad transversal.

p
p

1
p

Figura 5.6: Mdulo de deformacin volumtrica.

Universidad de Castilla- La Mancha


Ciudad Real - Espaa

Draft

Por: Eduardo W. V. Chaves (2014)

5 INTRODUCCIN A: ECUACIONES CONSTITUTIVAS, PVCI, Y ESTRATEGIAS DE SOLUCIN

383

NOTA 9: En el laboratorio los parmetros (, ) no son los ms convenientes para ser


obtenidos experimentalmente. Recordar que la forma inversa de la ecuacin constitutiva
( ) = Tr ( )1 + 2 fue obtenida en el Ejemplo 1.92 y viene dada por:
=

1
1

Tr ( )1 indicial
ij =
ij
(11 + 22 + 33 ) ij
2
2 ( 2 + 3 )
2
2 ( 2 + 3 )

11

21
13

12
22
23

13
11
1

21
23 =
2
13
33

12
22

13
1 0 0
(11 + 22 + 33 )

23
0 1 0
2 (2 + 3 )
0 0 1
33

23

Observar tambin que componentes normales de tensin 11 , 22 , o 33 solo producirn


componentes normales de deformacin. Vamos considerar un caso donde solamente
tenemos tensin normal 11 , 22 = 0 , 33 = 0 :
11

21
13

12
22
23

13
11
1

23 =
0
2
0
33

0 0
1 0 0
(11 )

0 0
0 1 0

2 (2 + 3 )
0 0 1
0 0

con lo cual las deformaciones normales quedan:


11 =

( + )
1

11 =
11
11
(11 ) 11 =

2
2 (2 + 3 )
2 2 (2 + 3 )
(2 + 3 )

22 =

(11 ) 22 =
11
2 ( 2 + 3 )
2 (2 + 3 )

33 =

(11 ) 33 =
11
2 (2 + 3 )
2 (2 + 3 )

De la relacin de 11 podemos obtener que:


( + )
11
11 =
(2 + 3 )

donde hemos denominado E =

11 =

(3 + 2 )
11
( + )

11 = E11

(3 + 2 )
, que es conocido como mdulo de Young, o
( + )

mdulo de elasticidad longitudinal.


Como era de esperar, debido a la isotropa del material, la influencia que 11 ejerce sobre
22 y 33 es la misma, y teniendo en cuenta el valor de 11 podemos obtener que:
22 =

(3 + 2 )

11 = 11
11 =
11 =

2 (2 + 3 )
2 (2 + 3 ) ( + )
2( + )

33 =

(3 + 2 )

11 = 11
11 =
11 =

2 (2 + 3 )
2 (2 + 3 ) ( + )
2( + )

donde hemos denominado por =


Notar que =

2( + )

2( + )

y se conoce como coeficiente de Poisson.

2
y si reemplazamos este valor en la expresin del
(1 2 )

mdulo de Young podemos obtener que:

Universidad de Castilla- La Mancha


Ciudad Real - Espaa

Draft

Por: Eduardo W. V. Chaves (2014)

MECNICA DEL MEDIO CONTINUO: PROBLEMAS RESUELTOS

384

+ 2
3
(3 + 2 )
(1 2 )

=
=
E=
( + )
2

(1 2 )

2
+ 2
3
(1 2 )
=
2
+ 1

(1 2 )

+ 2

(1 2 )

+ 1

(1 2 )

6 + 2(1 2 )

(1 2 )
=
= 2(1 + )
2 + (1 2 )

(1 2 )

con lo cual:
G==

E
2(1 + )

2
E
=
(1 2 ) (1 2 )(1 + )

Con lo cual, podemos expresar la ecuacin constitutiva en funcin de ( E , ) :


( ) = Tr ( )1 + 2 =

E
(1 + )(1 2 )

Tr ( )1 +

(1 + )

Podemos tambin expresar mdulo de elasticidad volumtrico en funcin de ( E , ) :


=+

E
E + 2 E (1 2 )
2
2
E
E (1 + )
E
=
+
=
=
=
3
(1 + )(1 2 ) 3 [2(1 + )] 3(1 + )(1 2 ) 3(1 + )(1 2 ) 3(1 2 )

Luego, podemos obtener las relaciones entre los parmetros mecnicos:


G==

f (G; E )
f (G; )
f (G;)
f (G; )
f (E; )
f (E; )
f ( ; )

f ( ; )

E=

GE
9G 3E

G (E 2G )
3G E

9G
3 + G

E 2G
2G
3 2G
2(3 + G )

G (3 + 2G )
+G

+ G

2( + G )

2G (1 + )

2G (1 + )
3(1 2 )

2G
1 2

E
3(1 2 )

9 ( )
3

(9 3E )
9 E
E
(1 + )(1 2 )

3(1 2 )

3
1+

3E
9 E
E
2(1 + )

3( )
2
3(1 2 )
2(1 + )

2
3

2G
3

3 E
6

En el Sistema Internacional de Unidades (SI) tenemos que [G ] = [ ] = [] = [ ] = [ E ] = Pa .

Universidad de Castilla- La Mancha


Ciudad Real - Espaa

Draft

Por: Eduardo W. V. Chaves (2014)

5 INTRODUCCIN A: ECUACIONES CONSTITUTIVAS, PVCI, Y ESTRATEGIAS DE SOLUCIN

385

Dejamos al lector demostrar que


Notacin Tensorial

Notacin indicial
ij = kk ij + 2 ij

= Tr ( )1 + 2

E
E
Tr ( )1 +

(1 + )(1 2 )
(1 + )

ij =

E
E
kk ij +
ij
(1 + )(1 2 )
(1 + )

ij =
kk ij + 2 ij
3

=
Tr ( )1 + 2
3

(5.19)
(5.20)
(5.21)

y
Notacin tensorial

Notacin indicial

1
Tr ( )1 +

2 (3 + 2 )
2

ij =

kk ij +
ij
2 (3 + 2 )
2

(5.22)

1+

Tr ( )1 +
E
E

ij =

1+
kk ij +
ij
E
E

(5.23)

2 3
1
kk ij
ij =
ij
2
18

(5.24)

2 3
1
Tr ( )1
=

2
18

y que el tensor constitutivo elstico para material istropo puede ser escrito como:
C e = 1 1 + 2 I
Ce =

E
E
1 1 +
I
(1 + )(1 2 )
(1 + )
Tensor constitutivo elstico

(5.25)

C e = 1 1 + 2 I 1 1
3

y respectivamente los tensores inversos:


Ce

Ce

Ce

1
1 1 +
I
2 (3 + 2 )
2

(1 + )
De =
1 1 +
I
E
E
1
1 1

De = 1 1 +
I 1 1

9
2 3

De =

Universidad de Castilla- La Mancha


Ciudad Real - Espaa

Draft

Tensor constitutivo elstico


complementario

(5.26)

Por: Eduardo W. V. Chaves (2014)

MECNICA DEL MEDIO CONTINUO: PROBLEMAS RESUELTOS

386

Ejemplo 5.6
En un ensayo de traccin simple los siguientes valores fueron obtenidos para la tensindeformacin:
Punto
1
2
3
4
5

( Pa ) (10 3 )
6,67
13,3
20
24
22

0,667
1,33
2
3
3,6

Determinar el mdulo de Young E y los puntos lmites.


Solucin:
Podemos verificar que los tres primeros puntos mantienen la misma proporcionalidad:
E=

(1) ( 2 ) ( 3)
20
= ( 2 ) = ( 3) =
= 10 000 Pa = 10 kPa
(1)
2 10 3

La grfica tensin-deformacin con los puntos dados se puede apreciar en la Figura 5.7. En
esta figura se sealan los punto: e - lmite elstico; Y - punto de fluencia; u - punto de
tensin ltima. r - punto de ruptura.

(Pa ) 30
u

25

20
15

3; 24

3,6; 22

2; 20

1,33; 13,3

10
0,667; 6,67

5
0

0; 0
0

0, 2%

0,5

1,5

2,5

3,5

(10 3 )

Figura 5.7: Curva tensin-deformacin.

Universidad de Castilla- La Mancha


Ciudad Real - Espaa

Draft

Por: Eduardo W. V. Chaves (2014)

5 INTRODUCCIN A: ECUACIONES CONSTITUTIVAS, PVCI, Y ESTRATEGIAS DE SOLUCIN

387

Ejemplo 5.7
Demostrar que la densidad de energa de deformacin, para un material elstico linear e
istropo, puede ser escrita como:
1
2

e ( ) = ( + 2 ) I 2 2 II

a)

(5.27)

e ( ) =

b)

( + )
1
I 2
II
2 (3 + 2 )
2

(5.28)

e ( ) =

c)

[Tr()]2 + 1
42
dev : dev
4 43
4
2 4243
1

energa puramente
volumtrica

(5.29)

energa puramente
desviadora

e ( ) =
d)

1
I 2 +
6(3 + 2 )
144244
3
Energa asociada
al cambio de volumen

1
J2
2
123

(5.30)

Energa asociada
al cambio de forma

donde es el tensor de deformacin infinitesimal, I = Tr ( ) es el primer invariante


principal del tensor de tensiones de Cauchy , y II dev = J 2 es el segundo invariante
principal de la parte desviadora del tensiones de tensiones de Cauchy.
Solucin:
1
2

a) Teniendo en cuenta la expresin de la energa e = : y ( ) = Tr ( )1 + 2 , (ver


ecuacin (5.19)), podemos obtener que:
1
1
2
2
1
1
2
= Tr ( ) 1
2
:3
1 + : = [Tr ( )] + :
2
2
Tr ( )

e = : = : [Tr ( )1 + 2 ]

(5.31)

1 2
I + :
2

Teniendo en cuenta la definicin del segundo invariante y la simetra de podemos


obtener que:

] [

] [

] [

1
1
1
1
[ Tr ( )] 2 Tr ( 2 ) = I 2 Tr ( ) = I 2 Tr ( T ) = I 2 :
2
2
2
2
2
: = I 2 II
II =

(5.32)

Luego, la ecuacin (5.31) puede ser reescrita como:


1
2

1
2

1
2

e = I 2 + : = I 2 + ( I 2 2 II ) = ( + 2 ) I 2 2 II

Universidad de Castilla- La Mancha


Ciudad Real - Espaa

Draft

Por: Eduardo W. V. Chaves (2014)

MECNICA DEL MEDIO CONTINUO: PROBLEMAS RESUELTOS

388

1
2

b) Teniendo en cuenta que e = : y =

1
Tr ( )1 +
, (ver ecuacin
2 (3 + 2 )
2

(5.22)), podemos obtener que:


1
1
1

:
Tr ( )1 +
e = : =
2
2 2 (3 + 2 )
2
1

[Tr ( )]2 + 1 :
=
2
:3
1+
: =
Tr ( ) 1
4 (3 + 2 )
4
4 (3 + 2 )
4
Tr ( )
=

(5.33)

I 2 +
:
4 (3 + 2 )
4

Si tenemos en cuenta la ecuacin (5.32), tambin se cumple que : = I 2 2 II , con lo


cual la ecuacin anterior queda:

e=
=

1
1

( I 2 2 II )
I 2 +
: =
I 2 +
4 (3 + 2 )
4
4 (3 + 2 )
4
( + )
1
I 2
II
2 (3 + 2 )
2

1
2

c) Teniendo en cuenta que e = : y =

2
Tr ( )1 + 2 , ver ecuacin (5.21),
3

podemos obtener que:


1
2

2
1
= :
Tr ( )1 + 2
2
3

e = :

(5.34)

2
1
2
:3
1 + :
Tr ( ) 1

2
3
Tr ( )


2
= [Tr ( )] + :
2 3

Si consideramos que un tensor de segundo orden puede ser descompuesto de forma aditiva
en una parte esfrica y otra desviadora, i.e. = esf + dev =
expresar el trmino : como:

Tr ( )
1 + dev , podemos
3

Tr ( )
Tr ( )

: =
1 + dev :
1 + dev
3
3

Tr ( )
Tr ( ) dev
Tr ( )
1 : dev +
: 1 + dev : dev
=
1 :1 +
3
3
3
=

[Tr()]2
3

(5.35)

+ dev : dev

donde hemos aplicado que 1 : 1 = 3 , 1 : dev = dev : 1 = Tr ( dev ) = 0 (traza de cualquier


tensor desviador es igual a cero). Con lo cual la ecuacin (5.34) puede ser reescrita como:

Universidad de Castilla- La Mancha


Ciudad Real - Espaa

Draft

Por: Eduardo W. V. Chaves (2014)

5 INTRODUCCIN A: ECUACIONES CONSTITUTIVAS, PVCI, Y ESTRATEGIAS DE SOLUCIN

e =

389

[Tr ( )] + :
3
2

[Tr ( )]2


2
= [Tr ( )] +
+ dev : dev

3
2 3

[Tr( )]2 + dev : dev


2

Para demostrar la ecuacin (5.30) vamos utilizar la definicin del tensor de deformacin
dada por la ecuacin (5.22), =

1
. Con lo cual la energa de
Tr ( )1 +
2 (3 + 2 )
2

deformacin queda:
1
1
1

:
e = : =
Tr ( )1 +
2
2 2 (3 + 2 )
2
1

=
12
:3
+
:
Tr ( ) 1
4 (3 + 2 )
4
Tr ( )
=

(5.36)

[Tr( )]2 + 1 :
4 (3 + 2 )
4

Notar que se cumple que : =

[Tr( )]2
3

+ dev : dev (ver ecuacin (5.35)). Teniendo en

cuenta la ecuacin del segundo invariante de un tensor de segundo orden podemos decir
que:
2
1
[ Tr ( dev )] 2 Tr ( dev )

2
2
1
Tr ( dev )
=
2
T
1
1
1 dev
=
Tr ( dev dev ) =
Tr ( dev dev ) =
: dev
2
2
2

II dev =

donde hemos tenido en cuenta que: la traza del tensor desviador es cero; la simetra del
T
tensor desviador ( dev = dev ), y la propiedad de traza Tr ( A B T ) = A : B . Con eso
podemos decir que:
: =

[Tr( )]2
3

+ dev : dev =

[Tr( )]2
3

2 II dev =

[Tr( )]2
3

+ 2J 2

Reemplazando la ecuacin anterior en la ecuacin (5.36), podemos obtener que:

e=

[Tr( )]2 + 1 :
4
4 (3 + 2 )

1 [Tr ( )]
2
[Tr( )] +
=
+ 2J 2

4 (3 + 2 )
4
3

1
1
[Tr ( )]2 +
=
+
J2
2
4 (3 + 2 ) 12
1
=
[Tr( )]2 + 1 J 2
2
6(3 + 2 )

Universidad de Castilla- La Mancha


Ciudad Real - Espaa

Draft

Por: Eduardo W. V. Chaves (2014)

MECNICA DEL MEDIO CONTINUO: PROBLEMAS RESUELTOS

390

Ejemplo 5.8
a.1) Escribir la densidad de energa de deformacin a.2) y las ecuaciones constitutivas de
tensin de un material elstico linear e istropo en notacin de Voigt, a.2.1) en funcin de
( , ), a.2.2) y en funcin de ( E , ) donde =

E
E
y =
. b) Si
(1 + )(1 2 )
2(1 + )

expresamos el tensor de deformacin en notacin de Voigt tal que { } = [ L(1) ]{u } ,


obtener la matriz [ L(1) ] .
c) Escribir las ecuaciones de movimiento en notacin de Voigt.
Solucin:
a.1) La densidad de energa de deformacin ( e () escalar) viene dada por:
1
2

1
2

1
2

1
2

e () = : C e : = : = : = ij ij
donde hemos utilizado que = C e : . Notar que
ij ij = 1 j 1 j + 2 j 2 j + 3 j 3 j
123
123
123
3131
21 21
1111
+
+
+
3232
22 22
1212
+
+
+
3333
23 23
1313

luego
1
2

1
2

e () = ij ij = (1111 + 22 22 + 33 33 + 212 12 + 2 23 23 + 21313 )


y

1
2

e ( ) = ij ij =

1
[11 22
2

33

12

23

11

22
33 1
T
13 ]
= { } { }
212 2
2 23

213

Luego, los tensores y en notacin de Voigt estarn almacenados como sigue:


11

22

{ } = 33 ;
12
23

13

11

22

{ } = 33
212
2 23

213

a.2.1) La ecuacin constitutiva de tensin en notacin de Voigt queda:

Universidad de Castilla- La Mancha


Ciudad Real - Espaa

Draft

Por: Eduardo W. V. Chaves (2014)

5 INTRODUCCIN A: ECUACIONES CONSTITUTIVAS, PVCI, Y ESTRATEGIAS DE SOLUCIN

11 + 2

+ 2

22
33

+ 2


= C e : Voigt
=
0
0
12 0
23 0
0
0


0
0
13 0

0
0

0
0

0 11
0 22
0 33
{ } = [C ] { }

0 2 12
0 2 23

213

0
0
0

391

(5.37)
Para mayores detalles de la ecuacin anterior ver Ejemplo 1.98 en el captulo 1, donde
tambin hemos obtenido que:
=

Tr ( )1
2
2 ( 2 + 3 )

( 2 + 3 )

11
+ 3 )

2
(
2

22
33 2 ( 2 + 3 )

=
212
0
2 23

213
0

a.2.2) Notar que

2 ( 2 + 3 )
+
( 2 + 3 )

2 ( 2 + 3 )

2 ( 2 + 3 )

2 ( 2 + 3 )
+
( 2 + 3 )

0 11

22
0
33

0 12
23

0 13

0
1

{ } = [C ]1 { }

(5.38)

E
E
E
(1 )
+2
=
(1 + )(1 2 )
2(1 + ) (1 + )(1 2 )
E
=

(1 + )(1 2 )
(1 2 )
E
E
=
=
2(1 + ) (1 + )(1 2 )
2

+ 2 =

Luego, la ecuacin (5.37) puede ser reescrita como:

(1 )

11

(1 )

(1 )
22

33
E
0
0
0
=
12 (1 + )(1 2 )
0
23
0
0

13
0
0
0

0
0

0
0

0
(1 2 )
2

0
0

(1 2 )
2

11

0 22

0 33
212
0 2 23

(1 2 ) 213

2
0
0

(5.39)

Notar que

Universidad de Castilla- La Mancha


Ciudad Real - Espaa

Draft

Por: Eduardo W. V. Chaves (2014)

MECNICA DEL MEDIO CONTINUO: PROBLEMAS RESUELTOS

392

+ =

E
E
E
+
=
(1 + )(1 2 ) 2(1 + ) 2(1 + )(1 2 )

(2 + 3) =

E
E
E
E2
+
=
2
3

2(1 + ) 2(1 + )
(1 + )(1 2 ) 2(1 + )(1 2 )

+
2(1 + )(1 2 ) 1
E
=
=
E
(2 + 3) 2(1 + )(1 2 )
E2
(1 + )(1 2 )
E

=
=
E
2 (2 + 3) (1 + )(1 2 )
E2
1 2(1 + ) 1
=
= 2(1 + )
E
E

Luego, la ecuacin (5.38) puede ser reescrita como:


11
1


22

33 1
=

212 E 0
0
2 23

213
0

0
0

1
0
0

11

0
0
0 22
0
0
0 33

2(1 + )
0
0 12
0
2(1 + )
0 23

0
0
2(1 + ) 13
0

(5.40)

b) Segn la definicin ij = 12 (u i , j + u j ,i ) podemos obtener que:

u1

x1

1 u
u
ij = 1 + 2
2 x 2 x1
1 u
u
1 + 3
2 x3 x1

1 u1 u 2

+
2 x 2 x1
u 2
x 2

1 u 2 u 3

+
2 x3 x 2

u1

x
1
x1

2
0

11

x 2


22 u 3
0
33
x 3
{ } = = u u =
212 1 + 2
2 23 x 2 x1 x 2

u
u
2 13 2 + 3 0

x3 x 2
u1 u 3
x + x x
3
1
3

x 2
0

x1

x3
0

1 u1 u 3

+
2 x3 x1
1 u 2 u 3

+
2 x3 x 2

u 3

x 3


u1
x3
u 2
0 u
3


x 2


x1

{ } = [L(1) ]{u }

NOTA: Si adoptamos la notacin ingenieril, i.e. x1 = x , x 2 = y , x3 = z , u1 = u , u 2 = v ,


u 3 = w , 11 = x , 22 = y , 33 = z , 2 12 = xy , 2 23 = yz , 2 13 = xz , la ecuacin anterior
queda:

Universidad de Castilla- La Mancha


Ciudad Real - Espaa

Draft

Por: Eduardo W. V. Chaves (2014)

5 INTRODUCCIN A: ECUACIONES CONSTITUTIVAS, PVCI, Y ESTRATEGIAS DE SOLUCIN

u
x x
v
0
11 x

y
22 y w
33 z z 0
{ } = = = u v =
2 12 xy
+
2 23 yz y x y

v w
2 13 xz +
0
z y
u + w
z x z

y
0

z
0

z v

0 w

{ } = [L(1) ]{u }

393

(5.41)

&& (ver ecuacin


c) Teniendo en cuenta las ecuaciones de movimiento, + b = v& = u
&& i y su forma explcita:
(5.14), en notacin indicial ij , j + b i = u

&& i
ij , j + b i = i1,1 + i 2, 2 + i 3,3 + b i = u
&&1
11,1 + 12, 2 + 13,3 + b1 = u

&& 2
21,1 + 22, 2 + 23,3 + b 2 = u

&&
31,1 + 32, 2 + 33,3 + b 3 = u 3

11 12 13
&&1
+
+
+ b1 = u

x
x
x
1
2
3

21 22 23
&& 2
+
+
+ b 2 = u

x
x
x
2
3
1
31 32 33
&& 3
+
+
+ b 3 = u

x1
x 2
x 3

Adems, si consideramos las componentes del tensor de tensiones en notacin de Voigt, la


ecuacin anterior queda:

x1
0

x2

x2

x1

x3

x3

x2


11

x3 22 b u
&&
33 1 1

&& 2
0 + b 2 = u
12

&&
b 3 u3
23
x1
13

(5.42)

[ ] { }+ {b } = {u&&}

L(1)

Universidad de Castilla- La Mancha


Ciudad Real - Espaa

Draft

Por: Eduardo W. V. Chaves (2014)

MECNICA DEL MEDIO CONTINUO: PROBLEMAS RESUELTOS

394

Ejemplo 5.9
Considerando un material elstico lineal homogneo e istropo descrito en el Ejemplo 5.5,
obtener las ecuaciones de gobierno de tal forma que resulte en un sistema de tres
ecuaciones y tres incgnitas, a saber: u1 , u 2 , u 3 , (formulacin en desplazamientos).
Solucin:
Como visto en el Ejemplo 5.5 las ecuaciones de gobierno para un material elstico linear e
istropo en rgimen de pequeas deformaciones son:
Notacin tensorial
Ecuaciones de Movimiento:
r
r
&r& (3 ecuaciones)
+ b = v& = u
Ecuacin Constitutiva en Tensin:

Notacin indicial
Ecuaciones de Movimiento:
&& i (3 ecuaciones)
ij , j + b i = u

= Tr ( )1 + 2 (6 ecuaciones)

ij = kk ij + 2 ij (6 ecuaciones)

Ecuacin Constitutiva en Tensin:


(5.43)

Ecuaciones Cinemticas:

Ecuaciones Cinemticas:
r
= sym u (6 ecuaciones)

ij =

1 u i u j
+
2 x j x i

(6 ecuaciones)

Resultando en un sistema de 15 ecuaciones y 15 incgnitas (u i , ij , ij ) .


La divergencia del tensor de tensiones de Cauchy ( ) se puede obtener a travs de las
ecuaciones constitutivas en tensin, i.e.:
ij = kk ij + 2 ij
ij , j = ( kk ij + 2 ij ) , j
ij , j = , j kk ij + kk , j ij + kk ij , j + 2 , j ij + 2 ij , j
{
{
{
=0 j

= 0i

(5.44)

=0 j

ij , j = kk , j ij + 2 ij , j
ij , j = kk ,i + 2 ij , j
r

Si las propiedades mecnicas y son constantes en el material, i.e. no varan con x


(material homogneo), luego, , j

= 0 j y ,j
= 0 j . Podemos tambin expresar los
x j
x j

trminos kk ,i y ij, j en funcin de los desplazamientos. Para ello utilizamos las ecuaciones
cinemticas:
ij =

1 u i u j
+
2 x j xi

1
1
u i , j + u j ,i divergenci
a ij , j = u i , jj + u j ,ij
2
2

Notar que
2ui

=
x j x j x j
u j ,ij

2u j
x j xi

u i

x j

r
r
r
u i , jj [ (u)]i 2 u (Laplaciano del vector u )
i

2u j
x i x j

[ ]

x i

u j

x j

Universidad de Castilla- La Mancha


Ciudad Real - Espaa

r
u j , ji [ ( u)]i

Draft

Por: Eduardo W. V. Chaves (2014)

5 INTRODUCCIN A: ECUACIONES CONSTITUTIVAS, PVCI, Y ESTRATEGIAS DE SOLUCIN

kk =

1 u k u k

+
2 x k
x k

395

u k
gradiente
=
u k ,k
kk ,i = u k , ki = u j , ji

x
k

Con eso la ecuacin (5.44) puede ser reescrita como:


ij , j = kk ,i + 2 ij , j

ij , j = u j , ji + 2

ij , j = ( + )u j , ji + u i , jj

1
u i , jj + u j , ji
2

&& i (ecuaciones de movimiento),


Reemplazando las ecuaciones anteriores en ij , j + b i = u
obtenemos que:
ij , j + b i = &u& i

&& i
( + )u j , ji + u i , jj + b i = u

Resultando as en un sistema con 3 ecuaciones y 3 incgnitas ( u1 , u 2 , u 3 ):


&& i
( + )u j , ji + u i , jj + b i = u
r
r
r
&r&
( + )[ ( u)] + [ (u)] + b = u

Ecuaciones de Navier

(5.45)

Notar que () 2 es el Laplaciano de .


NOTA 1: Las ecuaciones anteriores son conocidas como Ecuaciones de Navier o tambin
como Ecuaciones de Navier-Lam. Las formas explcitas de las ecuaciones en (5.45) se
presentan a continuacin:
&& i
( + )u j , ji + u i , jj + b i = ( + )(u1,1i + u 2, 2i + u 3,3i ) + (u i ,11 + u i , 22 + u i ,33 ) + b i = u

&&1
( + )(u1,11 + u 2, 21 + u 3,31 ) + (u1,11 + u1, 22 + u1,33 ) + b1 = u

&& 2
( + )(u1,12 + u 2, 22 + u 3,32 ) + (u 2,11 + u 2, 22 + u 2,33 ) + b 2 = u

&&
( + )(u1,13 + u 2, 23 + u 3,33 ) + (u 3,11 + u 3, 22 + u 3,33 ) + b 3 = u 3

o an:

2u
2 u1 2 u1
u1 u 2 u 3
&&1
+ b1 = u

+ 21 +
+
+
+
( + )
2
2
x
x1 x1 x 2 x3
x
x

2
3
1

2
2
2
u2 u2 u2
u1 u 2 u 3

&&

+
+
+
( + )
x 2 + x 2 + x 2 + b 2 = u 2
x 2 x1 x 2 x3
2
3

2
2
2
( + ) u1 + u 2 + u 3 + u 3 + u 3 + u 3 + b = u
&& 3
3
2
2
x

x 2

x
x
x

x
x

3
1
2
3

1
2
3

NOTA 2: Hemos demostrado en el Ejemplo 1.105 que se cumple la siguiente relacin:


r
r r
r
r
( a) = ( a) 2 a
indicial

ilq qjk a k , jl = a j , ji a i , jj
Luego, tambin se cumple que

r
r r
r
r
r
(u) 2 u = ( u) ( u)

indicial

u i , jj = u j , ji ilq qjk u k , jl

Con lo cual la ecuacin (5.45) tambin se puede escribir como:


&& i
( + )u j , ji + u i , jj + b i = u

&& i
( + )u j , ji + (u j , ji ilq qjk u k , jl ) + b i = u
&& i
( + 2 )u j , ji ilq qjk u k , jl + b i = u

Lo equivalente en notacin tensorial:


Universidad de Castilla- La Mancha
Ciudad Real - Espaa

Draft

Por: Eduardo W. V. Chaves (2014)

MECNICA DEL MEDIO CONTINUO: PROBLEMAS RESUELTOS

396

r
r
r
&r&
( + )[ ( u)] + [ (u)] + b = u
r
r
r r
r
r
&r&
( + )[ ( u)] + ( u) ( u) + b = u
r
r
r r
r
&r&
( + 2 )[ ( u)] ( u) + b = u

Resumen:

r
r
r r
r
&r&
( + 2 )[ ( u)] ( u) + b = u

(5.46)

&&i
( + 2 )u j , ji ilq qjk u k , jl + b i = u

En el sistema Cartesiano tenemos que:


r
u = u i e i = u1 e 1 + u 2 e 2 + u 3 e 3

r r
r
r
u
u
u
u
u
u
( u) rot (u) = (rot (u) )i e i = 3 2 e 1 + 1 3 e 2 + 2 1 e 3
x 2 x3
x3 x1
x1 x 2
14
14
14
42r44
3
42r44
3
42r44
3
r
r
r
r r (rot (u) )3 (rot (u) )2

( u) =
x 2
x 3

= (rot (u) )1

= (rot (u) )2
r
r

(rot (u) )1 (rot (u) )3


e 1 +

x
x1
3

= (rot (u) )3

r
r

(rot (u) )2 (rot (u) )1


e 2 +
e 3

x1
x 2

r
r
(rot (u) )3 (rot (u) )2 u 2 u1 u1 u 3

x x
x3
2 1 x 2 x3 x3 x1
x 2r

(rot (u) )
r
r r
(rot (u) )3 u 3 u 2 u 2 u1
1

( u) i =
=

x1
x 3
x 3 x 2 x 3 x1 x1 x 2
r
r
(rot (u

) )2 (rot (u) )1 u1 u 3
u 3 u 2

x1 x3 x1 x 2 x 2 x3
x1
x 2

NOTA 3: Si estamos tratando con un material heterogneo, las ecuaciones en (5.44)


quedan:
ij = kk ij + 2 ij

ij , j = ( kk ij + 2 ij ) , j
ij , j = ( kk ) , j ij + (2 ij ) , j = ( kk ) ,i + ( 2 ij ) , j

Teniendo en cuenta que 2 ij = u i , j + u j ,i , kk = u k ,k , la ecuacin anterior queda:


ij , j = ( kk ) ,i + (2 ij ) , j

ij , j = (u k , k ) ,i + (u i , j + u j ,i ) , j

Con lo cual
ij , j + b i = &u& i

(u k ,k ) ,i + (u i , j + u j ,i ) , j + b i = &u& i

(5.47)

Notar que

r
r
u k , k = Tr (u) = ( u)
y sus componentes
u&
u&
u&
u&
Du& i u& i u& i
&& i =
=
+
u
v j = i + i v1 + i v 2 + i v3
t
x j
t
x1
x 2
x3
Dt

Universidad de Castilla- La Mancha


Ciudad Real - Espaa

Draft

Por: Eduardo W. V. Chaves (2014)

5 INTRODUCCIN A: ECUACIONES CONSTITUTIVAS, PVCI, Y ESTRATEGIAS DE SOLUCIN

397

u& 1 u& 1

u&
u&
+
v1 + 1 v 2 + 1 v3

x1
x 2
x3
t

u& 2 u& 2

&
&
u
u
&& i =
+
v1 + 2 v 2 + 2 v3
u
x1
x 2
x 3
t
u&

&
&
u
u
u&
3 + 3 v1 + 3 v 2 + 3 v3
t

x1
x 2
x3

(u i , j + u j ,i ) , j =
(u i , j + u j ,i )
x j

(u i ,1 + u1,i ) +
(u i , 2 + u 2,i ) +
(u i ,3 + u 3,i )
x1
x 2
x3

2 (u1,1 ) +
(u1, 2 + u 2,1 ) +
(u1,3 + u 3,1 )

x 2
x3
x1

2 (u 2, 2 ) +
(u i , j + u j ,i ) , j =
(u 2,1 + u1, 2 ) +
(u 2,3 + u 3, 2 )
x 2
x3
x1

2 (u 3,3 )
(u 3,1 + u1,3 ) +
(u 3, 2 + u 2,3 ) +

x1

x 2
x3
Explcitamente las tres ecuaciones dadas por (5.47) ( i = 1,2,3 ) quedan:

2 (u1,1 ) +
(u1, 2 + u2,1 ) +
(u1,3 + u3,1 ) + b1 = u&&1
[( u)] +

x
x
x
x
1
1
2
3


[( ur )]+ (u2,1 + u1,2 ) + 2 (u2,2 ) + (u2,3 + u3,2 ) + b 2 = u&&2

x1
x2
x3
x2

[( ur )] + (u3,1 + u1,3 ) + (u3,2 + u2,3 ) + 2 (u3,3 ) + b3 = u&&3

x1
x2
x3
x3

( u) + 2 (u1,1 ) +
(u1, 2 + u2,1 ) +
(u1,3 + u3,1 ) + b1 = u&&1

x
x
x
2
3
1

r

( u) + 2 (u2, 2 ) +
(u2,1 + u1, 2 ) +
(u2,3 + u3, 2 ) + b 2 = u&&2

x
x
x
1
3
2

r

( u) + 2 (u3,3 ) +
(u3,1 + u1,3 ) +
(u3, 2 + u2,3 ) + b3 = &u&3

x1
x2
x3

Universidad de Castilla- La Mancha


Ciudad Real - Espaa

Draft

Por: Eduardo W. V. Chaves (2014)

MECNICA DEL MEDIO CONTINUO: PROBLEMAS RESUELTOS

398

NOTA 4: Ecuaciones de Onda


Si aplicamos la divergencia a la ecuacin (5.46) obtenemos que:

r
r
r r
r
&r&
( + 2 ) [ ( u)] ( u) + b = u
1442443
=0
r
r
&r&
( + 2 ) [ ( u)] + b = u
r
r
&r&
( + 2 ) 2 ( u) + b = u

r
r
&r& = ( + 2 ) 2 ( u
) + b
u

r
(5.48)
r
r
( + 2 ) 2
D

(
)
(
)

u
u
b
2
Dt

r
D 2 ( + 2 ) 2

b
Dt 2

2
r
D

= 2 2 + b
2
Dt
r
&
& = 2 2 + b
r r
r
donde hemos considerado que = u y ( v ) = 0 (ver Ejemplo 1.107). En notacin

indicial queda:

&& i ,i
( + 2 )u j , jii ilq qjk u k , jli + b i ,i = u
&& i ,i
( + 2 )u j , jii + b i ,i = u
&& i ,i =
u

( + 2 )

D 2 u i

Dt 2 x i

D 2
Dt 2

= 2

u j , jii + b i ,i

( + 2 ) 2
=

x i x i

u j

x j

(5.49)

b i
+
x i

b
2
+ i
x i x i x i

donde
=

( + 2 )

velocidad de onda-P

(5.50)
r

Si las fuerzas msicas no cambian en el espacio, se cumple que b = 0 , con eso la


ecuacin (5.48) se reduce a:
D 2
= 2 2
Dt 2

Ecuacin de onda P (o Primaria)

(5.51)

La onda-P no tiene rotacin.


r
Si ahora aplicamos el rotacional ( ) a la ecuacin (5.46), obtenemos que:

Universidad de Castilla- La Mancha


Ciudad Real - Espaa

Draft

Por: Eduardo W. V. Chaves (2014)

5 INTRODUCCIN A: ECUACIONES CONSTITUTIVAS, PVCI, Y ESTRATEGIAS DE SOLUCIN

399

r
r
r
r r
r r
r &r&
r
( + 2 ) [ ( u)] ( u) + b = u
r
r
r r
r r
r &r&
( u) + b = u
r
r
r r
r r
D2 r r
( u) + b = 2 ( u)
Dt
2r
r
r r
r r
D
( ) + b =
Dt 2
r
r
r r
D 2
(5.52)
( ) =
Dt 2
r
r
r r
D 2

)
Dt 2
r
r r r
D 2

( )

Dt 2
r
r
r r
D 2

= 2 ( )
2
Dt
r
r r r
donde hemos considerado que = u , y que el campo vectorial b es un campo
r
r r r
r
r
r
conservativo, luego b = 0 se cumple. Notar que [ ( u)] = [ ] = 0 (ver

[
[

]
]

Ejemplo 1.107), y
=

Velocidad de onda-S

(5.53)

r
r r
r
r r
r r
r
r
r
r r
r
( ) = ( ( u)) = 0 (ver Ejemplo 1.107). Con lo cual la ecuacin (5.52) queda:
r

Notar que 2 = ( ) ( ) 2 = ( ) , ya que ( u) = 0

r
r
D 2
= 2 2
2
Dt

Ecuacin de onda-S (o Secundaria, o


onda de corte)

(5.54)

Las ondas de corte no tienen cambio de volumen, solamente presentan cambio de forma.
En el caso que = 0 , la ecuacin (5.51) se reduce a la ecuacin de onda acstica:
D 2
= c 2 2
2
Dt

Ecuacin de onda acstica

(5.55)

Velocidad de propagacin

(5.56)

con
c=

Notar que el campo de desplazamientos puede ser representado por: u = + con


r
r
r r
r
r
= 0 . Podemos probar eso a travs de la identidad ( a) = ( a) 2 a . Si
r
r r
r
r
r
consideramos los vectores u = 2a y = a , y el escalar = a . Con lo cual
r r
r
obtenemos que u = + , y se cumple que:
r r
r r r
r
r r
r
r r
r
u = + ( ) = y u = + ( ) = ( )
Si consideramos que =

E
E
y =
, podemos obtener que:
(1 + )(1 2 )
2(1 + )

Universidad de Castilla- La Mancha


Ciudad Real - Espaa

Draft

Por: Eduardo W. V. Chaves (2014)

MECNICA DEL MEDIO CONTINUO: PROBLEMAS RESUELTOS

400

( + 2 )

E
E
+2
( + 2 )
(1 + )(1 2 )
2(1 + )
(2 2 )
=
=
=
E
(1 2 )

2(1 + )

Con lo cual concluimos que la relacin entre las velocidades de onda-P y onda-S depende
nicamente del coeficiente de Poisson.

a)

b)

Figura 5.8: Desplazamientos debido a una onda-P harmnica plana (a) y una onda-S (b). La
onda-P no tiene rotacin y la onda-S no tiene cambio de volumen.
Ejemplo 5.10
Consideremos el campo de deformacin infinitesimal . a) Demostrar que:
r
r
( )T = 0

qjk til ij ,kl = 0 qt

(5.57)

donde ijk es el smbolo de permutacin.


b) Demostrar que:
ij , kl + kl ,ij il , jk jk ,il = O ijkl

(5.58)

c) Expresar las ecuaciones en (5.57) de forma explcita.


Solucin:
u

u
1
1
j
El tensor de deformacin infinitesimal viene dado por ij =
+ i = (u j ,i + u i , j ) .

2 xi x j 2
r

Si derivamos con respecto a ( x ) obtenemos:


ij
x k

= ij ,k =

1
(u j ,ik + u i , jk )
2

Notar que u i , jk = u i ,kj es simtrico en jk si multiplicamos por un tensor que es


antisimtrico en jk , i.e. qjk = qkj , ste se anula: u i , jk qjk = 0 iq , luego:
1
2

1
2

qjk ij ,k = (u j ,ik + u i , jk ) qjk = u j ,ik qjk


r

Derivamos una vez ms con respecto a ( x ) y obtenemos que:


Universidad de Castilla- La Mancha
Ciudad Real - Espaa

Draft

Por: Eduardo W. V. Chaves (2014)

5 INTRODUCCIN A: ECUACIONES CONSTITUTIVAS, PVCI, Y ESTRATEGIAS DE SOLUCIN

401

1
( qjk ij ,k ) = qjk ij , kl = u j ,ikl qjk
x l
2

Notar que u j ,ikl = u j ,kil = u j ,kli es simtrico en il y til = tli es antisimtrico en il y si


multiplicamos ambos lados de la igualdad por til obtenemos que:
1
2

til qjk ij ,kl = u j ,ikl til qjk = 0 jkt qjk = 0 qt


b) Si en la ecuacin anterior multiplicamos por ambos lados de la igualdad por tab qmn ,
obtenemos que:
tab qmn til qjk ij ,kl = 0 qt tab qmn = O abmn
Recordar que se cumple que tab til = ai bl al bi y qmn qjk = mj nk mk nj , con lo
cual:
tab qmn til qjk ij ,kl = O abmn
( ai bl al bi )( mj nk mk nj ) ij , kl = O abmn
( ai bl mj nk ai bl mk nj al bi mj nk + al bi mk nj ) ij ,kl = O abmn
am,nb an,mb bm,na + bn,ma = O abmn

que es lo mismo que:


am,bn + bn,am an,mb mb,an = O ambn

Q.E.D.

Notar que, si multiplicamos la ecuacin anterior por bn obtenemos que:


am, bn bn + bn, am bn an, mb bn mb, an bn = O ambn bn
am, bb + bb, am ab, bm mb, ba = 0 ambb

[ xr ( xr )]am + [ xr [ xr [Tr ( )]]]am [ xr ( xr )]am [ xr ( xr )]ma = 0 ambb

[ xr ( xr )]am + [ xr [ xr [Tr ( )]]]am = [ xr ( xr )]am + [ xr ( xr )]ma

[ ]

2xr am + [ xr [ xr [Tr ( )]]]am = [ xr ( xr )]am + [ xr ( xr )]ma

or en notacin tensorial:
xr ( xr ) + xr [ xr [Tr ( )]] = xr ( xr ) + [ xr ( xr )]

c) Notar que en (5.57) tenemos 6 ecuaciones independientes ya que 0 qt es simtrico.


Para el caso q = 1, t = 1 tenemos que 1 jk 1il ij ,kl y expandiendo el subndice l obtenemos:
1 jk 1il ij ,kl = 1 jk 1i1 ij , k1 + 1 jk 1i 2 ij ,k 2 + 1 jk 1i 3 ij ,k 3 = 1 jk 1i 2 ij ,k 2 + 1 jk 1i 3 ij ,k 3
Expandiendo el subndice i
1 jk 1il ij ,kl = 1 jk 1i 2 ij ,k 2 + 1 jk 1i 3 ij ,k 3 = 1 jk 132 3 j , k 2 + 1 jk 123 2 j ,k 3 = 1 jk 3 j , k 2 + 1 jk 2 j ,k 3
Expandiendo los dems subndices obtenemos:

1 jk 1il ij ,kl = 1 jk 3 j ,k 2 + 1 jk 2 j , k 3 = 123 32,32 132 33, 22 + 123 22,33 + 132 23, 23
= 32,32 + 33, 22 + 22,33 23, 23 = 33, 22 + 22,33 2 23, 23 = 0
=

2 33
x 22

2 22
x32

Universidad de Castilla- La Mancha


Ciudad Real - Espaa

2 23
=0
x 2 x3
Draft

Por: Eduardo W. V. Chaves (2014)

MECNICA DEL MEDIO CONTINUO: PROBLEMAS RESUELTOS

402

notar que 23, 23 = 32,32 . Dejamos para el lector las siguientes demostraciones:
Para el caso q = 2, t = 2

2 jk 2il ij ,kl = 31,31 + 33,11 + 11,33 13,13 = 33,11 + 11,33 213,13 = 0


=

2 33
x12

2 11
x32

2 13
=0
x1x3

Para el caso q = 3, t = 3

3 jk 3il ij ,kl = 11, 22 12,12 21, 21 + 22,11 = 11, 22 + 22,11 212,12 = 0


=

2 11
x 22

2 22
x12

2 12
=0
x1 x 2

Para el caso q = 1, t = 2
1 jk 2il ij ,kl = 12,33 + 13, 23 + 32,31 33, 21 = 13, 23 + 23,13 33,12 12,33 = 0
=

2 33
2 23
2 13
2 12

+
x 2 x3 x1 x3 x1 x 2 x3 x3 x3

23 13 12

+
x3
x 2
x1

2 33

=0
x1x 2

Para el caso q = 2, t = 3
2 jk 3il ij ,kl = 11,32 + 13,12 + 21,31 23,11 = 13,12 + 12,13 23,11 11, 23 = 0
2 23
2 13
2 11
2 12

+
=
x1 x 2 x1 x3 x1x1 x 2 x 3 x1

13 12 23

+
x1
x3
x 2

2 11

=0
x 2 x 3

Para el caso q = 1, t = 3
1 jk 3il ij ,kl = 12,32 13, 22 22,31 + 23, 21 = 12, 23 13, 22 22,13 + 23,12 = 0
=

2 23
2 13
2 22
2 12

=
+

x 2 x3 x 2 x 2 x1x 3 x1x 2 x 2

12 13 23

x1
x 2
x3

2 22

=0
x1x3

Reagrupando las 6 ecuaciones:

2 33 2 22
2 23
+

2
=0
S11 =
x 2 x3
x 22
x32

2
2
2
S 22 = 33 + 11 2 13 = 0

x1 x3
x12
x32

2
2
11 22
2 12

S
=
+

=0
2
33
x1 x 2
x 22
x12

2
S = 23 + 13 12 33 = 0
12 x x
x 2
x3 x1 x 2
3
1

23 13 12 2 11

=0
S
=
+
+
23
x1 x1
x 2
x 3 x 2 x3

2
S = 23 13 + 12 22 = 0
13 x 2 x1
x 2
x 3 x1 x3

Ecuaciones de
Compatibilidad (3D)

(5.59)

Las ecuaciones anteriores en notacin de Voigt quedan:

Universidad de Castilla- La Mancha


Ciudad Real - Espaa

Draft

Por: Eduardo W. V. Chaves (2014)

5 INTRODUCCIN A: ECUACIONES CONSTITUTIVAS, PVCI, Y ESTRATEGIAS DE SOLUCIN

2
S11 x 2
3
S
2

22

S 33 x 22
=
S12 0
S 23

2
S13

x 2 x 3

2
x32

2
x 22
2
x12

0
2
x12

2
x1x 2

2
x1x3

2
x 2 x3

2
x1x 2
2
12 2
x3
2
1
2
x1x3
2
1
2
x 2 x3

x1x3 11 0


22 0
0
0
33 =
2
212 0

2
x 2 x 3 2 23 0


2 2 13 0
1

2
x1 x 2
2
12 2
x 2

403

0
2
x1x3
2
12 2
x1
2
1
2
x1x 2
1
2

{S } = [L( 2) ] { } = {0}

(5.60)

NOTA 1: Las ecuaciones (5.59) son conocidas como Ecuaciones de Compatibilidad. Las
ecuaciones de compatibilidad nos garantizan que el campo de desplazamiento es nico y
continuo, ver Figura 5.9. En otras palabras, las 6 componentes del tensor de deformacin
no son independientes y no pueden ser arbitrarias.
(Configuracin actual?)
1

1
4

6
9

(Configuracin inicial)

No cumple con las ecuaciones de


compatibilidad

Cumple con las ecuaciones de


compatibilidad

2
5
8

4
7

3
6
9

(Configuracin actual)

Figura 5.9
NOTA 2: Cuando utilizamos un mtodo numrico para la obtencin de la solucin, e.g.
mtodo de los elementos finitos, la forma de garantizar el cumplimiento de las ecuaciones
de compatibilidad es a travs de la continuidad del campo de desplazamientos. Con lo que
respecta la tcnica de los elementos finitos, al hacer el ensamblaje de los elementos finitos
(atamos los nodos) estamos de cierta forma garantizando que las ecuaciones de
compatibilidad se cumplan.
NOTA 3: Cuando el campo de desplazamiento no depende de una direccin, i.e.
r r
u = u( x1 , x 2 ) , las ecuaciones de compatibilidad reducen a:
S 33 =

2 11
x 22

2 22
x12

2 12
=0
x1x 2

Ecuacin de
Compatibilidad (2D)

(5.61)

ya que i 3 = 3i = 0 . La ecuacin anterior en notacin ingenieril queda:

Universidad de Castilla- La Mancha


Ciudad Real - Espaa

Draft

Por: Eduardo W. V. Chaves (2014)

MECNICA DEL MEDIO CONTINUO: PROBLEMAS RESUELTOS

404

Sz =

2 x
y 2

2 y
x 2

2 xy
xy

Ecuacin de
Compatibilidad 2D
(notacin ingenieril)

=0

(5.62)

NOTA 4: Para mejor ilustracin de la condicin de compatibilidad vamos considerar un


ejemplo en dos dimensiones (2D), donde tenemos un campo escalar = ( x1 , x2 ) y que
conocemos las derivadas:

= x1 + 3x2 y
= x12 , podemos ver claramente que este
x1
x2

campo escalar es incompatible ya que

= x1 + 3 x2 = F1
x1

= x12 = F2
x2

x2


2
( x1 + 3 x2 )

=
=
=3
x2
x1 x2 x1

2
( x12 )

=
=
= 2 x1
x2
x1 x2 x1x2

El campo escalar = ( x1 , x2 ) ser compatible si y solo si:

= F1 ( x1 , x 2 )
x1
compatible sii F1 F2
=

x2 x1

= F2 ( x1 , x2 )

x 2

(5.63)

Si consideramos el teorema de Green (ver Captulo 1 en Chaves (2007)) que establece:


r r
r
r
F
F
componentes

F d = (

F) e 3 dS
F1 dx1 + F2 dx2 = 2 1 dS 3

x1 x 2
r r
y considerando tambin la ecuacin (5.63), podemos concluir que: si F = xr , es
r r
r
r r
r
r
compatible si y solo si F d = ( xr F) e 3dS = 0 xr F = 0 .

r
x

r
dS = dSe 3

x2

x3

e 3

x1

Figura 5.10: Teorema de Green.


r

NOTA 5: Vamos considerar que F = ( xr ) a = a ( xr )T , donde es un campo


r
r
tensorial de segundo orden y a es un vector arbitrario independiente de x (constante).
Notar tambin que las siguientes relaciones son vlidas:
r
r r
r
r r
r
r
r
r r
(a) F d = ( xr ) a d = a ( xr )T d = a ( xr )T d

Universidad de Castilla- La Mancha


Ciudad Real - Espaa

Draft

Por: Eduardo W. V. Chaves (2014)

5 INTRODUCCIN A: ECUACIONES CONSTITUTIVAS, PVCI, Y ESTRATEGIAS DE SOLUCIN

(b)

r
r
r
( xr F) dS =

r
r
[
{

r
x

r
x

]}

r
r r
[
(
{

r
r
) a dS =

r
r r
= a xr ( xr )T

r
x

r r
dS = a

r
{

r
x

r
x

)T

405

]} dSr

r
( xr )T

} dS
T

En notacin indicial
r
r
r
r
r
r
r
(a) Fi (d )i = ( xr )ij a j (d )i = a j ( xr )ij (d )i = a j ( xr )ij (d )i

(b)

[ [

r
r
r
r
r
( xr F)i (dS )i = ijk Fk , j (dS )i = ijk a p ( xr )T

] ]

kp , j

r
( dS ) i

r
r
r
= ijk a p , j ( xr )T kp + a p ( xr )T kp , j (dS )i
{

=0

r
r
r
r
= ijk a p ( xr )T kp , j (dS )i = a p ijk ( xr )T kp , j (dS )i

r
= a

= a p ijk psq qk , s
= ap

,j

r
r
(dS )i = a p ijk psq qk , sj (dS )i

r
x

r
( xr )T

r
x

r
( xr )T

} dS

r
( dS ) i

ip

Valdra la pena revisar el Ejemplo 1.109, donde hemos demostrado que la relacin
r
r
( xr ) = ksq qp , s e k e p se cumple, luego ( xr )T = psq qk , s e k e p tambin se
r

cumple. En el Ejemplo 1.109 hemos demostrado que xr ( xr )T = ipq tsj qj , ps e t e i ,


r

el cual es equivalente a xr ( xr )T = psq ijk qk ,sj e i e p .


Considerando el Teorema de Stokes (ver Captulo 1 en Chaves (2007)) concluimos que:
r r
r
r
r

F d = (

r
x

F ) dS

244443
14444

r
r
r
r
r
r
a ( xr )T d = a xr ( xr )T

} dS

r
r
( xr )T d =

r
r
xr ( xr )T

} dS
T

Luego, para un campo compatible se debe cumplir que:

{
r

r
x

r
( xr )T

=0

r
r
xr ( xr )T = 0

r
r
r
x
Consideremos ahora que A = F a donde F es el gradiente de deformacin, F = r , y
X

aplicando el teorema de Stokes obtenemos que:

Universidad de Castilla- La Mancha


Ciudad Real - Espaa

Draft

Por: Eduardo W. V. Chaves (2014)

MECNICA DEL MEDIO CONTINUO: PROBLEMAS RESUELTOS

406

r r
r
r
r
A d = ( xr A ) dS

r
r
r
r
r
( F a) d = ( xr ( F a)) dS

1444444444444442444444444444443

r r
r
a F T d = a

r
xr F

r
dS

{
r

r
x

} dS
T

Luego, el campo x ser compatible si y solo si:

{
r

r
x

=0

r
xr F = 0

Para mayores detalles acerca de las manipulaciones algebraicas ver Ejemplo 1.109.
Ejemplo 5.11
Dado el campo del tensor de deformacin infinitesimal , y el campo de desplazamientos
r
u , (a) demostrar que:
11
r
u i
( J ) ij ( xr u) ij =
= 12 + 3
x j
13 2

12 3

13 + 2
23 1
33

22
23 + 1

donde i son las componentes del vector rotacin.


b) Demostrar tambin que:
k
x p

ip jp
1
1

kij ij , p =
kij

x i
x
2
2
j

23

x1

= 31
x
1

12
x1

23
x 2
31
x 2
12
x 2

23 13 12 23 22

x 3 x 2
x 3
x 3 x 2
31 11 13 12 23

x 3 x3
x1 x3
x1

12


12 11 22 12
x 3 x
x 2 x1
x 2
1

33 23

x x
3
2
13 33

x1
x 3
23 13

x 2
x1

r
k
donde ( xr ) kp =
, y es el tensor spin infinitesimal.
x p

Solucin:
r

a) El gradiente de los desplazamientos J xr u puede ser descompuesto de forma aditiva


en una parte simtrica y otra antisimtrica:
r

J xr u =

] [

r
r
r
r
r
r
1
1
( xr u) + ( xr u) T + ( xr u) ( xr u) T = ( xr u) sym + ( xr u) anti = +
1
4
2
4
3
1
4
2
4
3
2 44424443 1
2 44424443
1
r
= ( xr u) sym

r
= ( xr u) anti

donde la parte simtrica = ( xr u) sym representa el tensor de deformacin infinitesimal y la


r
parte antisimtrica = ( xr u) anti representa el tensor spin infinitesimal (tensor de
r
rotacin). Si consideramos que es el vector axil asociado con el tensor antisimtrico
podemos decir que:

Universidad de Castilla- La Mancha


Ciudad Real - Espaa

Draft

Por: Eduardo W. V. Chaves (2014)

5 INTRODUCCIN A: ECUACIONES CONSTITUTIVAS, PVCI, Y ESTRATEGIAS DE SOLUCIN

12
0

0
ij = 21
31

32

13 0
23 = 12
0 13

12
0

13 0
23 = 3
0 2

23

3
0

407

2
1
0

Con lo cual
(

r
x u) ij

11
u i
=
= 12
x j
13

12
22
23

3
0

13 0
23 + 3
33 2

2 11
12 3 13 + 2

22
23 1
1 = 12 + 3
0 13 2 23 + 1
33

r
b) Recordar del captulo de Tensores que un tensor antisimtrico ( ) y su vector axil ( )
1

estn relacionados entre ellos, en notacin indicial, por ij = k kij k = kij ij . Si


2
r
utilizamos esta ltima ecuacin para obtener el gradiente de podemos obtener que:

k , p = kij ij = kij ij , p
1
2

1
2

, p

Expandiendo los ndices mudos i, j , y solo considerando los trminos distintos de ceros
podemos decir que:
1
2

1
2

k , p = kij ij , p = ( k1212, p + k1313, p + k 21 21, p + k 23 23, p + k 3131, p + k 3232, p )


Notar que para las filas de k , p ( k = 1,2,3 ) tenemos que:

k, p =

1
kij ij , p
2

1
1

(k = 1) 1, p = 2 (123 23, p + 13232, p ) = 2 ( 23, p 32, p ) = 23, p

1
1

( 21313, p + 23131, p ) =
(13, p + 31, p ) = 31, p
(k = 2) 2, p =
2
2

1
1

(k = 3) 3, p = 2 (312 12, p + 321 21, p ) = 2 (12, p 21, p ) = 12, p

donde hemos utilizado la propiedad del tensor antisimtrico ij = ji . Teniendo en


cuenta lo anterior podemos decir que:

k, p

23,1 23, 2
1

kij ij , p = 31,1 31, 2


=
2
12,1 12, 2

23

23,3
x1

31,3 = 31
x1

12,3
12
x1

23
x2
31
x2
12
x2

1 u

23

x3
31
x3
12
x3

(5.64)

Teniendo en cuenta la definicin ij = ( xr u) anti ij = i j = (ui , j u j ,i ) y


2 x j xi 2
derivando con respecto a x p podemos obtener que:
1
1
ij , p = (ui , j u j ,i ), p = (ui , jp u j ,ip )
2
2

Universidad de Castilla- La Mancha


Ciudad Real - Espaa

Draft

Por: Eduardo W. V. Chaves (2014)

MECNICA DEL MEDIO CONTINUO: PROBLEMAS RESUELTOS

408

El valor de la expresin anterior no se ve alterado si sumamos y restamos el trmino


1
u p,ij , con lo cual podemos obtener que:
2
1
1
1
(ui , jp u j ,ip + u p ,ij u p , ij ) = (ui , jp + u p , ij ) (u j , ip + u p ,ij )
2
2
2
1
1
1

= (ui , pj + u p ,ij ) (u j , pi + u p , ji ) = (ui , p + u p ,i ) (u j , p + u p , j )


2
2
2
2
, i
, j

ij , p =

= ip , j jp ,i =

ip
x j

jp
xi

Reemplazando la ecuacin anterior en (5.64) y expandiendo los ndices mudos i, j ,


obtenemos que:
1
1
kij ( ip , j jp ,i ) =
( kij ip , j kij jp ,i )
2
2
1
=
( k12 1 p ,1 + k13 1 p ,3 + k 21 2 p ,1 + k 23 2 p ,3 + k 31 3 p ,1 + k 32 3 p , 2
2
k 12 2 p ,1 k13 3 p ,1 k 21 1 p , 2 k 23 3 p , 2 k 31 1 p ,3 k 32 2 p ,3 )

k, p =

Notar que las filas de k , p ( k = 1,2,3 ) tambin pueden ser representadas por:

k, p

(k = 1) 1, p = 2 ( 123 2 p ,3 + 132 3 p , 2 123 3 p , 2 132 2 p ,3 ) = 3 p , 2 2 p ,3

( k13 1 p ,3 + k 31 3 p ,1 + k13 3 p ,1 k 31 1 p ,3 ) = 1 p ,3 3 p ,1
= (k = 2) 2, p =
2

(k = 3) 3, p = 2 ( 312 1 p ,1 + 321 2 p ,1 312 2 p ,1 321 1 p , 2 ) = 2 p ,1 1 p , 2

Luego:

k, p

( 31, 2 21,3 ) ( 32, 2 22,3 ) ( 33, 2 23,3 )

= (11,3 31,1 ) (12,3 32,1 ) (13,3 33,1 )


( 21,1 11, 2 ) ( 22,1 12, 2 ) ( 23,1 13, 2 )

Con lo cual concluimos que:

k, p =


1
1
kij ij , p = kij ip jp
2
2
xi
x j

23

x1

= 31
x
1
12
x1

23
x2
31
x2
12
x2

23 13 12 23 22 33 23

x3 x2
x3
x3 x2 x3 x2
31 11 13 12 23 13 33

x1 x3
x1
x3 x3 x1 x3
12
11 22 12 23 13


12

x3 x x x x x x

2
2
2
1
1
1

donde hemos tenido en cuenta la simetra de ij = ji .

Universidad de Castilla- La Mancha


Ciudad Real - Espaa

Draft

Por: Eduardo W. V. Chaves (2014)

5 INTRODUCCIN A: ECUACIONES CONSTITUTIVAS, PVCI, Y ESTRATEGIAS DE SOLUCIN

409

NOTA: Vamos suponer que conozcamos el campo de deformacin infinitesimal:

8 x1
x
2
ij =
2

3 2
2 x1 x 3

x2
2
x1
0

3 2
x1 x 3
2

3
x1

con las siguientes condiciones de contorno:


3t
r r
u i ( x = 0, t ) = 0
0

r r
( x = 0, t ) = 0

r r

( x = 0, t ) = 0

Para este ejemplo se cumple que:

k, p

13 12 23 22 33 23


x 3 0
x 3 x 2
x3 x 2
x 2




= 11 13 12 23 13 33 = x1 x 3
x1
x1 x 3
x1 x 3
x 3

0


12 11 22 12 23 13
x 2
x 2 x1
x 2 x1
x1

0
0
3
2

3 2
x1
2

Notar tambin que se cumple que:

k, p

x1

= 2
x
1

3
x1

1
x 2
2
x 2
3
x 2

1

x 3 0
2
= x1 x 3
x 3

3 0

x 3

0
0
3
2

3 2
x1
2

Y a travs de integracin podemos obtener que

1
= 0
x1

= 0 1 = C1 (t )
x 2

1
= 0

x3

2
= x1 x3
x1

2
3 2

x1 x 3 + C 2 (t )
=0
2 =
2
x 2

2 3 2
x1
=

2
x 3

3
=0
x1

3 3
3
= 3 = x 2 + C 3 (t )
2
2
x 2

3
=0
x 3

r
r r r
Aplicando la condicin de contorno ( x = 0, t ) = 0 , concluimos que C i (t ) = 0 . Luego:

Universidad de Castilla- La Mancha


Ciudad Real - Espaa

Draft

Por: Eduardo W. V. Chaves (2014)

MECNICA DEL MEDIO CONTINUO: PROBLEMAS RESUELTOS

410

i = 2 = x12 x 3
2

3 3

x2

Con lo cual podemos definir el tensor spin infinitesimal:


0
ij = 21
31

12
0
32

3
0

13 0
23 = 3
0 2

0
2
3
1 = x 2
2
0 3 2
2 x1 x 3

3
x2
2
0
0

3 2
x1 x 3
2

El campo de desplazamientos puede se obtenido si consideramos que:


11 12
ui
= 12 22
x j
13 23

8 x1
x
2
=
2

3 x2 x
2 1 3

13 0 12
23 + 21 0
33 31 32
x2
2
x1
0

13
23
0

3 2
x1 x3 0
2
3
0 + x2
2
3
3 x2 x
x1
2 1 3

3
x2
2

3 2
x1 x3
2
8x
1
0 = x2
2
3x1 x3
0

0
0

2 x2
x1
0

0
0
x13

u1

x1
u i u 2
=
x j x1

u 3
x1

u1
x 2
u 2
x 2
u 3
x 2

u1

x 3 8 x
1
u 2
x
=
2
x 3 2
3x1 x 3
u 3
x 3

2x2
x1
0

0
0
x13

Y a travs de integracin directa podemos obtener que

u1
= 8 x1
x1

u1
= 2 x 2 u1 = 4 x12 x 22 + K 1 (t )
x 2

u1
=0

x 3

u 2
= x2
x1

u 2
= x1 u 2 = x1 x 2 + K 2 (t )
x 2

u 2
=0

x 3

u 3
= 3 x12 x 3
x1

u 3

3
=0
u 3 = x1 x3 + K 3 (t )
x 2

u1
= x13

x 3

Universidad de Castilla- La Mancha


Ciudad Real - Espaa

Draft

Por: Eduardo W. V. Chaves (2014)

5 INTRODUCCIN A: ECUACIONES CONSTITUTIVAS, PVCI, Y ESTRATEGIAS DE SOLUCIN

411

Las constantes de integracin se obtienen a travs de la condicin de contorno:


4 x12 x 22 + K 1 (t )
r

u i ( x , t ) = x1 x 2 + K 2 (t )
x 3 x + K (t )
3
1 3

K 1 (t ) 3t
r r
u i ( x = 0, t ) = K 2 (t ) = 0
K 3 (t ) 0

r r
x =0

Definiendo as el campo de desplazamientos:


4 x12 x 22 + 3t
r

u i ( x, t ) =
x1 x 2

x
x
1 3

Es interesante verificar que el campo de desplazamientos es compatible, ya que el campo


de deformaciones cumple con las ecuaciones de compatibilidad. Dejamos al lector verificar
si se cumple las ecuaciones de compatibilidad, (ver ecuacin (5.59)).
Ejemplo 5.12
a) Demostrar que las ecuaciones de gobierno para un material elstico lineal e istropo
dadas por (5.43) pueden ser reemplazadas por seis ecuaciones y seis incgnitas ( ij ),
(formulacin en tensin), cuyas ecuaciones son:
Notacin indicial
ij , kk +

2( + )

&&i ), j
kk ,ij
ll , kk ij = 2 ( u
(2 + 3 )
(2 + 3 )

sym

2 ( bi ), j

sym

(5.65)

Notacin tensorial
2xr +

2( + ) r r
&r&) sym 2 r ( b) sym
x [ x [Tr ( )]]
2xr [Tr ( )]1 = 2 xr ( u
x
(2 + 3)
(2 + 3)

donde 2xr xr ( xr ) y 2xr [Tr ( )] xr [ xr [Tr ( )]] .


b) o por
Notacin indicial
ij , kk +

2( + )

&&i ), j
kk ,ij =
( b k ), k ( &u&k ), k ij + 2 ( u
(2 + 3 )
(2 + )

Notacin tensorial
2xr +

sym

2 ( bi ) , j

] [

sym

r
r
2( + ) r r

&r&) 1 + 2 r ( u
&r&) sym 2 r ( b) sym
x [ x [Tr ( )]] =
xr ( b) ( u
x
x
( 2 + 3)
(2 + )

(5.66)
c) Considerando que =

E
E
, =
, expresar las ecuaciones (5.65) y
(1 + )(1 2 )
2(1 + )

(5.66) en funcin ( E , ) .
r

OBS.: Las ecuaciones cinemticas = sym u pueden ser reemplazadas por


ij , kl + kl ,ij il , jk jk ,il = O ijkl

(5.67)

Ver Ejemplo 5.10.

Universidad de Castilla- La Mancha


Ciudad Real - Espaa

Draft

Por: Eduardo W. V. Chaves (2014)

MECNICA DEL MEDIO CONTINUO: PROBLEMAS RESUELTOS

412

Solucin: a) Obtenemos la inversa de la ecuacin constitutiva en tensin ( = C e : ) para obtener:


Ce

: = Ce

: C e : = I sym : = sym =

= Ce

Para un material istropo, ver ecuacin (5.22), el tensor de deformacin viene dado por:
=

1
1

Tr ( )1 indicial

ij =
ij
ss ij .
2
2 (2 + 3 )
2
2 (2 + 3 )
r

Si consideramos que las propiedades mecnicas no varan con x , i.e. ,i

,i

= 0 i obtenemos que:
xi

2 ij

ij , kl =
ij
ss ij =
ij ,kl
ss , kl ij
x k xl
2 (2 + 3 )
2 (2 + 3 )
2
,kl 2

= 0i y
x i

(5.68)

Adems, si multiplicamos la ecuacin (5.67) (ecuaciones cinemticas) por jk obtenemos


que:
ij , kl jk + kl ,ij jk il , jk jk jk ,il jk = O ijkl jk
ik ,kl + kl ,ik il , kk kk ,il = 0 il

(5.69)

Observar que, segn la ecuacin (5.68) se cumplen que:


ik ,kl =

1
1
ik ,kl
ss ,kl ik =
ik ,kl
ss ,il
2
2 (2 + 3 )
2
2 (2 + 3 )

kl ,ik =

1
1
kl ,ik
ss ,ik kl =
lk ,ki
ss ,il
2
2 (2 + 3 )
2
2 (2 + 3 )

il ,kk =

1
il ,kk
ss ,kk il
2
2 (2 + 3 )

kk ,il =

1
1
3

kk ,il
ss ,il kk =
kk ,il
ss ,il
{ 2
+
2
2 (2 + 3 )
2
(
2
3
)

=3

2
1
3
3
ss ,il =
ss ,il
ss ,il =

ss ,il
2
2 (2 + 3 )
2 (2 + 3 )
2 2 (2 + 3 )

2 ij
x k x l

ij ,kl =

1
ij ,kl
ss ,kl ij
2
2 (2 + 3 )

Con lo cual la ecuacin (5.69) queda:


ik ,kl + kl ,ik il ,kk kk ,il = 0 il
1
2

2
2

ik ,kl
ss ,il + lk ,ki il , kk +
ss ,kk il
ss ,il = 0 il
(2 + 3 )
(2 + 3 )
(2 + 3 )

2
2

ss ,il + lk , ki il ,kk +
ss , kk il = 0 il
ik ,kl
+
(2 + 3 )
(2 + 3 ) (2 + 3 )
2( + )

ik ,kl
ss ,il + lk , ki il ,kk +
ss , kk il = 0 il
(2 + 3 )
(2 + 3 )

Universidad de Castilla- La Mancha


Ciudad Real - Espaa

Draft

Por: Eduardo W. V. Chaves (2014)

5 INTRODUCCIN A: ECUACIONES CONSTITUTIVAS, PVCI, Y ESTRATEGIAS DE SOLUCIN

2( + )

ss ,il il , kk +
ss , kk il = ik , kl lk , ki
(2 + 3 )
(2 + 3 )

413

(5.70)

&& i podemos obtener que:


De las ecuaciones de movimiento ij , j + b i = u
&& i ) ,k
ij , jk + ( b i ) , k = ( u

Con lo cual se cumplen que:


&& i ) ,l
ik ,kl + ( b i ) ,l = ( u

&& i ) ,l
ik ,kl = ( b i ) ,l ( u

&& l ) ,i
lk , ki + ( b l ) ,i = ( u

&& l ) ,i .
lk ,ki = ( b l ) ,i ( u

Y notar que ik ,kl lk , ki = ( b i ) ,l ( u&& i ) ,l + ( b l ) ,i ( u&& l ) ,i = 2[( b i ) ,l ]sym 2[( u&& i ) ,l ]sym
Reemplazando la ecuacin anterior (5.70) obtenemos que:
2( + )

sym
&& i ) ,l ]sym
ss ,il il , kk +
ss , kk il = 2[( b i ) ,l ] 2[( u
(2 + 3 )
(2 + 3 )

Reestructurando la ecuacin anterior y por hacer ( l = j ) obtenemos que:


ij ,kk +

2( + )

&& i ) , j
kk ,ij
ll ,kk ij = 2 ( u
(2 + 3 )
(2 + 3 )

sym

2 ( b i ) , j

sym

Cuya ecuacin es la misma que (5.65).


b) Partiendo de la ecuacin anterior obtenemos que:
ij ,kk +

2( + )

&& i ) , j
kk ,ij =
ll ,kk ij + 2 ( u
(2 + 3 )
(2 + 3 )

sym

2 ( b i ) , j

sym

(5.71)

Nuestro objetivo ahora es obtener una expresin para ll, kk . Si multiplicamos la ecuacin
(5.67) por jk li obtenemos que:
ij , kl jk li + kl ,ij jk li il , jk jk li jk ,il jk li = O ijkl jk li
ij , ji + ji ,ij ii , jj jj ,ii = 2 ij ,ij 2 ii , jj = 0

(5.72)

ij ,ij ii , jj = 0

Si recurrimos a la ecuacin constitutiva inversa (ver ecuacin (5.68)), podemos decir que:
ij ,ij =
ii , kk

1
1

ij ,ij
ss ,ij ij =
ij ,ij
ss ,ii
2
2 (2 + 3 )
2
2 (2 + 3 )

2
1

ii , kk
=
ii ,kk
ss ,kk ii =
2
2 (2 + 3 )
2 (2 + 3 )

(5.73)

Con lo cual la ecuacin (5.72) queda:


ij ,ij ii , jj = 0

1
ii ,kk = 0
ij ,ij
ss ,ii
2
2 (2 + 3 )
2 (2 + 3 )

ii ,kk = 0
ij ,ij
+
(2 + 3 ) (2 + 3 )

(5.74)

2 +
ii , kk
ij ,ij =
(2 + 3 )

Universidad de Castilla- La Mancha


Ciudad Real - Espaa

Draft

Por: Eduardo W. V. Chaves (2014)

MECNICA DEL MEDIO CONTINUO: PROBLEMAS RESUELTOS

414

&& i podemos obtener


Si ahora recurrimos a las ecuaciones del movimiento ij , j + b i = u
que:
&& i ) ,i
ij , ji + ( b i ) ,i = ( u

&& i ) ,i ( b i ) ,i
ij , ji = ( u

Con lo cual la ecuacin en (5.74) queda:


2 +
ii ,kk
ij ,ij =
(2 + 3 )
2 +
&& i ) ,i ( b i ) ,i =
( u
(2 + 3 ) ii ,kk

ii ,kk = ll ,kk =

(5.75)

(2 + 3 )
(2 + 3 )
&& k ) ,k ( b k ) ,k =
&& k ) , k
( u
( b k ) , k ( u
2 +
2 +

Reemplazando la ecuacin (5.75) en la ecuacin (5.71), obtenemos que:


ij , kk +

2( + )
&&i ), j
kk ,ij =
ll , kk ij + 2 ( u
(2 + 3)
(2 + 3)

ij , kk +

2( + )
(2 + 3)

&&k ), k ij + 2 ( u
&&i ), j
( b k ) , k ( u
kk ,ij =
(2 + 3)
(2 + 3) 2 +

sym

2 ( b i ) , j

ij , kk +

sym

2 ( b i ) , j

2( + )

&&k ), k ij + 2 ( u
&&i ), j
( b k ), k ( u
kk ,ij =
(2 + 3)
( 2 + )

sym

sym

2 ( bi ), j

sym

sym

(5.76)
Obteniendo as la ecuacin en (5.66)
c) Tras algunas manipulaciones algebraicas podemos obtener que:
1
(1 2 )
(1 2 )
E

;
=
=
=
(2 + 3)
E
(2 + 3)
E
(1 + )(1 2 ) (1 + )

(2 + 3)

(1 2 )
E
(1 2 )
=
E
2(1 + ) 2(1 + )

2( + )
(1 2 )
1

=2
+2
=
(2 + 3)
(1 + )
2(1 + ) (1 + )

( 2 + ) = 2

E
E
E (1 )
+
=
2(1 + ) (1 + )(1 2 ) (1 + )(1 2 )

E
(1 + )(1 2 )
=
=
(2 + ) (1 + )(1 2 )
E (1 )
(1 )
con lo cual la ecuacin (5.65) queda:
ij ,kk +

1
kk ,ij
ll ,kk ij = 2 ( &u&i ) , j
(1 + )
(1 + )

Notacin tensorial
2xr +

sym

2 ( b i ) , j

sym

1
&r&) sym 2 r ( b) sym
xr [ xr [Tr ( )]]
2xr [Tr ( )]1 = 2 xr ( u
x
(1 + )
(1 + )

(5.77)

Universidad de Castilla- La Mancha


Ciudad Real - Espaa

Draft

Por: Eduardo W. V. Chaves (2014)

5 INTRODUCCIN A: ECUACIONES CONSTITUTIVAS, PVCI, Y ESTRATEGIAS DE SOLUCIN

415

y (5.66) queda:
ij ,kk +

&& k ) ,k ij + 2 ( u
&&i ) , j
( b k ) , k ( u
kk ,ij =
(1 + )
(1 )

Tensorial notation
2xr +

] [

sym

2 ( b i ) , j

sym

r
r
1

&r&) 1 + 2 r ( u
&r&) sym 2 r ( b) sym
xr [ xr [Tr ( )]] =
xr ( b) ( u
x
x
(1 + )
(1 )

(5.78)
NOTA: Para un problema esttico la ecuacin anterior quedan:
ij ,kk +
2xr

kk ,ij =
( b k ) ,k ij 2 ( b i ) , j
(1 + )
(1 )

] [

sym

r
r sym
1

+
xr [ xr [Tr ( )]] =
xr ( b) 1 2 xr ( b)
(1 + )
(1 )

Ecuaciones de
Michell

(5.79)

que son conocidas como ecuaciones de Michell.


r

Si las fuerzas msicas no varan con x las ecuaciones de Michell se reducen a:


ij ,kk +
2xr

1
kk ,ij = 0 ij
(1 + )

Ecuaciones de Beltrami

1
+
xr [ xr [Tr ( )]] = 0
(1 + )

(5.80)

que son las conocidas ecuaciones de Beltrami.


&& k = 0 k ) , la ecuacin (5.75) queda:
NOTA 2: Para un problema esttico (u
ll ,kk =

r
(1 + ) r
x ( b )
(1 )
r
(1 + ) r
2xr [Tr ( )] =
x ( b)
(1 )

(2 + 3 )
(1 + )
( b k ) , k =
( b k ) , k
2 +
(1 )

xr { xr [Tr ( )]} =

(5.81)

La ecuacin anterior tambin puede ser obtenida a partir de la ecuacin (5.79) con ( i = j ),
i.e.:
ii ,kk +

1
kk ,ii =
( b k ) ,k ii 2 ( b i ) ,i
{
(1 + )
(1 )
=3

1
ii ,kk =
2 ( b k ) ,k
1 +
(1 + )
(1 )

(2 + )
(2 + )
( b k ) ,k
ii ,kk =

+
(
1
)
(1 )

(1 + )
ii ,kk =
( b k ) , k
(1 )

]
(5.82)

Notar que ii ,kk = kk ,ii y ( b k ) ,k = ( b i ) ,i . La ecuacin anterior en notacin tensorial


queda:
( 2xr ) : 1 +

r
r sym
1
{ xr [ xr [Tr( )]]} : 1 = xr ( b) 1 : 1 2 xr ( b) : 1
(1 + )
(1 )

Universidad de Castilla- La Mancha


Ciudad Real - Espaa

Draft

(5.83)

Por: Eduardo W. V. Chaves (2014)

MECNICA DEL MEDIO CONTINUO: PROBLEMAS RESUELTOS

416

Notar que:
( 2xr ) : 1 { xr ( xr )} : 1 = xr [ xr [Tr ( )]] 2 [Tr ( )]

{ xr [ xr [Tr( )]]} : 1 = xr [ xr [Tr( )]] 2 [Tr( )]


1 :1 = 3
r sym
r
xr ( b)
: 1 = xr ( b)

con lo cual la ecuacin en (5.83) puede ser reescrita como:

] [

r
r
3
1
2 [Tr ( )] =
xr ( b) 2 xr ( b)
(1 + )
(1 )
r
(1 + ) r
2 [Tr ( )] =
x ( b )
(1 )
2 [Tr ( )] +

NOTA 3: Para la elasticidad bidimensional la formulacin en tensin se puede encontrar


en el Ejemplo 7.32.
Ejemplo 5.13
a) Dado un campo escalar , tal que se cumpla:
11 =

2
x 22

22 =

12 = 21 =

x12

2
x1 x 2

(5.84)

Demostrar que
Notacin indicial
, iijj = 0

Notacin tensorial
{ [ ( )]} = 0

(i, j = 1,2)

2 2 = 0

4
4 4
4 +2 2 2 + 4 =0
x1
x1 x2 x2

(5.85)

4 = 0

Considerar un material elstico lineal, un problema esttico y sin fuerzas msicas.


Considerar tambin que el tensor de tensiones de Cauchy es dependiente nicamente de x1
y x 2 , i.e. = ( x1 , x 2 ) .
b) Demostrar si las ecuaciones de equilibrio se cumplen.
Solucin:
a) En el Ejemplo 5.12 (ver ecuacin (5.82)) hemos demostrado que:
ii , kk =

(1 + )
( b k ) , k = 0
(1 )

donde hemos considerado que ( b k ) ,k = 0 . Para el problema propuesto tenemos que


i, k = 1,2 , con lo cual:
ii ,kk = 0
ii ,11 + ii , 22 = 0
11,11 + 22,11 + 11, 22 + 22, 22 = 0

2 11
x12

2 22
x12

2 11
x 22

2 22
x 22

=0

Utilizando la definicin (5.84), concluimos que:


Universidad de Castilla- La Mancha
Ciudad Real - Espaa

Draft

Por: Eduardo W. V. Chaves (2014)

5 INTRODUCCIN A: ECUACIONES CONSTITUTIVAS, PVCI, Y ESTRATEGIAS DE SOLUCIN

2 11
x12

2 22
x12

2 11
x 22

2 22
x 22

417

=0

2 2 2 2 2 2 2 2
+
+
+
=0
x12 x 22 x12 x12 x 22 x 22 x 22 x12

4
4
4
+
2
+
=0
x14
x12 x 22 x 24

Q.E.D.
b) Para el caso bidimensional las ecuaciones de equilibrio (sin fuerzas msicas) se reducen a:

ij , j = 0 i

i1,1 + i 2, 2 = 0 i

11 12
x + x = 0
1
2

21 + 22 = 0
x1
x 2

Utilizando la definicin (5.84), obtenemos que:


11 12
x + x = 0
1
2

21 + 22 = 0
x1
x 2

2
2

=0

2
x 2 x1 x 2
x1 x 2

2
2
+ = 0
x x x
x 2 x12
1
1
2

Con lo cual demostramos que las expresiones de las tensiones dadas por (5.84) cumplen
con las ecuaciones de equilibrio.
NOTA: En la literatura es conocida como funcin de tensin de Airy. (ver Ejemplo 7.32)

Universidad de Castilla- La Mancha


Ciudad Real - Espaa

Draft

Por: Eduardo W. V. Chaves (2014)

MECNICA DEL MEDIO CONTINUO: PROBLEMAS RESUELTOS

418

Ejemplo 5.14
Considerando las ecuaciones de gobierno para un material elstico y lineal, obtener una
r
formulacin equivalente solamente en funcin de desplazamientos- u y tensiones-
(Formulacin Mixta). Utilizar la notacin de Voigt.
Solucin:
Teniendo en cuenta las ecuaciones de gobierno para un problema elstico y lineal:
Notacin tensorial
Ecuaciones de Movimiento:
r
r
&r& (3 ecuaciones)
+ b = v& = u
Ecuacin Constitutiva en Tensin:

Notacin de Voigt
Ecuaciones de Movimiento:

[L ] { } + { b } = { u&&} (3 ecuaciones)

( ) = C e : (6 ecuaciones)

{ } = [C ]{ } (6 ecuaciones)

(1) T

Ecuacin Constitutiva en Tensin:

Ecuaciones Cinemticas:

(5.86)

Ecuaciones Cinemticas:

{ } = [L(1) ] {u } (6 ecuaciones)

r
= sym u (6 ecuaciones)

donde las ecuaciones de notacin de Voigt fueron obtenidas en el Ejemplo 5.8, con

[L ]

(1) T

x1
= 0

x 2

x 2

x1

x3

x 3
0


x1

x3

x 2

Para eliminar la deformacin reemplazamos las ecuaciones cinemticas en la ecuacin


constitutiva, resultando:

{ } = [C ]{ }
{ } = [C ] [L(1) ]{u }
1
1
C
[3
C ] [L(1) ]{u }
[C ] { } = [1
4]24
=[1 ]

[ ]{u } = {0 }

[C ] { } L
1

(1)

Con lo cual quedamos con el siguiente sistema de ecuaciones:

[ ]

L(1) T { } + {b } = {u&&}

1
(1)
[C ] { } L {u } = {0 }

[ ]

(3 ecuaciones)

(5.87)

(6 ecuaciones)

Tambin podemos expresar como:


[0 ]

(1)
L

[ ]

[L ] {u } = { b } + { u&&}
(1) T

[C ]1 { }

{0 }

NOTA 1: La formulacin anterior se conoce como Formulacin Mixta. Y es interesante


observar que en la formulacin en desplazamiento y en tensin obtenidas en los Ejemplo
5.9 y Ejemplo 5.12 respectivamente tenemos derivada segunda de las incgnitas. En la
formulacin mixta tenemos solamente derivada primera de los desplazamientos y tensiones
(incgnitas) y notar tambin que no involucra derivadas de los parmetros mecnicos del
material.
Universidad de Castilla- La Mancha
Ciudad Real - Espaa

Draft

Por: Eduardo W. V. Chaves (2014)

5 INTRODUCCIN A: ECUACIONES CONSTITUTIVAS, PVCI, Y ESTRATEGIAS DE SOLUCIN

419

NOTA 2: Como visto en el Ejemplo 5.5 el problema elstico linear viene gobernado por
el conjunto de ecuaciones en derivadas parciales:
Notacin tensorial
Ecuaciones de Movimiento:
r
r
&r& (3 ecuaciones)
+ b = v& = u
Ecuacin Constitutiva en Tensin:

Notacin de Voigt
Ecuaciones de Movimiento:

[L ] { } + { b } = { u&&} (3 ecuaciones)

( ) = C e : (6 ecuaciones)

{ } = [C ]{ } (6 ecuaciones)

(1) T

Ecuacin Constitutiva en Tensin:

Ecuaciones Cinemticas:

(5.88)

Ecuaciones Cinemticas:

{ } = [L(1) ] {u } (6 ecuaciones)

r
= sym u (6 ecuaciones)

resultando en un sistema con 15 ecuaciones y 15 incgnitas (u i , ij , ij ) .


Adems, hemos podido comprobar a travs del Ejemplo 5.9, del Ejemplo 5.12, y del
Ejemplo 5.14, que el problema elstico lineal considerando un material homogneo e
istropo tambin puede ser representado por:
1) Formulacin en desplazamientos (ver Ejemplo 5.9):
&& i
( + )u j , ji + u i , jj + b i = u
r
r
r
&r&
( + )[ ( u)] + [ (u)] + b = u

Ecuaciones de Navier

(5.89)

resultando en 3 ecuaciones y 3 incgnitas (u i ) .


2) Formulacin en tensin (ver Ejemplo 5.12):
Notacin indicial
ij , kk +

2( + )

&&i ), j
kk ,ij
ll , kk ij = 2 ( u
(2 + 3 )
(2 + 3 )

Notacin tensorial
2xr +

sym

2 ( bi ), j

sym

(5.90)

2( + ) r r
&r&) sym 2 r ( b) sym
x [ x [Tr ( )]]
2xr [Tr ( )]1 = 2 xr ( u
x
(2 + 3)
(2 + 3)

resultando en 6 ecuaciones y 6 incgnitas ( ij ) .


3) Formulacin mixta (ver Ejemplo 5.12):

[ ]

L(1) T { } + {b } = {u&&}

1
(1)
[C ] { } L {u } = {0 }

[ ]

(3 ecuaciones)
(6 ecuaciones)

(5.91)

resultando en 9 ecuaciones y 9 incgnitas ( u i , ij ).

Universidad de Castilla- La Mancha


Ciudad Real - Espaa

Draft

Por: Eduardo W. V. Chaves (2014)

MECNICA DEL MEDIO CONTINUO: PROBLEMAS RESUELTOS

420

Ejemplo 5.15
Dados dos sistemas constituidos por el mismo material elstico lineal y con condiciones de
cargas distintas:
r

Sistema I

Fuerzas de superficie - t *

Fuerzas msicas especfica- b


S

Su
dV

r
u*

r r
t * ( x)

r r
b( x )

Campo de tensin -
Campo de deformacin -

Campo de desplazamiento - u

Fuerzas de superficie - t *

Sistema II

Fuerzas msicas especfica- b


Su
r
u*

B
dV

r r
t * ( x)

r
b

Campo de tensin -
Campo de deformacin -

Campo de desplazamiento - u

Figura 5.11: Dos sistemas bajo cargas externas.


Demostrar el Teorema de Betti:

: dV = : dV

Teorema de Betti

(5.92)

Solucin:
Teniendo en cuenta la ecuacin constitutiva de tensin, = C e : , en notacin indicial:
e
ij = C ijkl
kl

Si a ambos lados de la ecuacin multiplicamos por el campo obtenemos que:


Mayor de C
Simetra

ij ij = ij C eijkl kl

e
ij ij = ij C ijkl
kl = kl C eklij ij

e
donde hemos aplicado la simetra mayor del tensor constitutivo elstico ( C ijkl
= C eklij ).

Como los dos sistemas estn constituidos por el mismo material se cumple que = C e : .
Con lo cual la relacin anterior queda:
ij ij = ij C eijkl kl = kl C eklij ij = kl kl

Notacin tensorial

: = :

Si ahora integramos sobre todo el volumen, obtenemos el teorema de Betti:

: dV = : dV

Universidad de Castilla- La Mancha


Ciudad Real - Espaa

Draft

(5.93)

Por: Eduardo W. V. Chaves (2014)

5 INTRODUCCIN A: ECUACIONES CONSTITUTIVAS, PVCI, Y ESTRATEGIAS DE SOLUCIN

421

e
NOTA 1: La ecuacin anterior solo se cumple si C ijkl
= C eklij , es decir, si C e presenta
e
simetra mayor. En otras palabras, la condicin C ijkl
= C eklij impone la existencia de una
funcin de energa almacenada, tal que:

e
C ijkl
=

2 e
2 e
=
= C eklij
ij kl kl ij

NOTA 2: El teorema de Betti (Teorema de la Reciprocidad) es el punto de partida para la


obtencin de la formulacin del Mtodo de los Elementos de Contorno.
NOTA 3: El teorema de Betti tambin puede ser expresada de otra forma que
demostramos a continuacin.

2 x j

1 u
Recordar que ij = i +

u j 1
= (u i , j + u j ,i ) , con eso para el sistema II tambin se
xi 2

1
2

cumple que ij = ( ui , j + u j ,i ) . Con lo cual:

ij ij dV

= ij ij dV
V

1
1
ij (u i , j + u j ,i )dV =
ij ( ui , j + u j ,i ) dV
2V
2V

u
ij

i , j dV

(5.94)

= ij ui , j dV
V

donde ij u i , j = ij u j ,i y ij ui , j = ij u j ,i se cumplen debido la simetra de y de ,


respectivamente. Y adems notar que:
( ij u i ), j = ij , j u i + ij u i , j

ij u i , j = ( ij u i ), j ij , j u i

( ij ui ), j = ij , j ui + ij ui , j

ij ui , j = ( ij ui ), j ij , j ui

Con lo cual la ecuacin (5.94) queda:

u
ij

= ij ui , j dV

i , j dV

( u ),

( u ),

ij

ij , j u i dV = ( ij ui ), j ij , j ui dV

ij

(5.95)

dV ij , j u i dV = ( ij ui ), j dV ij , j ui dV

Aplicando el teorema de la divergencia a las primeras integrales de cada lada de la ecuacin,


obtenemos que:

u n dS
ij

ij , j u i dV

= ij ui n j dS ij , j ui dV
S

t i u i dS ij , j u i dV = t i ui dS ij , j ui dV
S

(5.96)

donde hemos aplicado la definicin n = t y n = t . La ecuacin anterior en notacin


tensorial queda:

r r
r
r r
r
t udS ( ) udV = t u dS ( ) u dV

Universidad de Castilla- La Mancha


Ciudad Real - Espaa

Draft

(5.97)

Por: Eduardo W. V. Chaves (2014)

MECNICA DEL MEDIO CONTINUO: PROBLEMAS RESUELTOS

422

Si recurrimos a las ecuaciones de movimiento se cumple que:


r
&r&
+ b = u

r r
r
&&
&r&
= (b u ) y + b = u

r r
&&)
= (b u

Con lo cual la ecuacin (5.97) queda:

r r r
r r
r r r
r r
&&
&&) u dV
t udS + (b u ) udV = t u dS + (b u

Teorema de Betti

(5.98)

Notar que, si consideramos S = S u + S podemos decir que:

r r
t udS =

r r
t u dS =

r r
r r
t * udS + t u* dS

r r
r r
t * u dS + t u * dS

Su

(5.99)

Su

Para el caso particular cuando el sistema est en equilibrio y en la ausencia de fuerzas


msicas, la ecuacin (5.98) queda:

r r
r r
t udS = t u dS

(5.100)

Si adems las fuerzas de superficies son fuerzas concentradas, podemos decir que:
r
r
r
r
F loc u loc = F loc u loc

Fi loc u loc
= Filoc uiloc
i

(5.101)

Ejemplo 5.16
Considerando el problema planteado en la Figura 5.11, demostrar el Principio del Trabajo
Virtual que establece que:

r r r
r r
&&) u dV = : dV
t * u dS + (b u

1444442444443
Trabajo externo virtual total

Principio del Trabajo Virtual

V
14243

(5.102)

Trabajo interno
virtual total

con u = u* en S u .
Solucin:
Partimos directamente de la relacin:

ij ij dV

1
ij ( ui , j + u j ,i )dV = ij ui , j dV
2V
V

Notar que ( ij ui ), j = ij , j ui + ij ui , j

ij ij dV

(5.103)

ij ui , j = ( ij ui ), j ij , j ui , luego:

= ij ui , j dV = ( ij ui ), j ij , j ui dV

ij ij dV = ( ij ui ), j dV ij , j ui dV
V

(5.104)

Aplicando el teorema de la divergencia a la primera integral del lado derecho, obtenemos


que:

Universidad de Castilla- La Mancha


Ciudad Real - Espaa

Draft

Por: Eduardo W. V. Chaves (2014)

5 INTRODUCCIN A: ECUACIONES CONSTITUTIVAS, PVCI, Y ESTRATEGIAS DE SOLUCIN

ij ij dV

423

= ( ij ui ), j dV ij , j ui dV = ij ui n j dS ij , j ui dV

(5.105)

= t *i ui dS ij , j ui dV
S

donde hemos aplicado la definicin n = t * . La ecuacin anterior en notacin tensorial


viene dada por:

r
r r
: dV = t * u dS ( ) u dV

r r
&&) (ecuaciones de movimiento), con lo
= (b u

&&
Notar que + b = u

cual, la ecuacin (5.106) queda:

(5.106)

r r r
r r
&&) u dV = : dV
t * u dS + (b u

V
14243

1444442444443

Trabajo interno
virtual total

Trabajo externo virtual total

que es conocido como el Principio del Trabajo Virtual. Observar que no hemos tenido que
recurrir la simetra mayor de C e para su demostracin.
Para el caso particular cuando el sistema est en equilibrio y en la ausencia de fuerzas
msicas, la ecuacin anterior queda:

r r
t * u dS = : dV

(5.107)

Si adems las fuerzas de superficies son fuerzas concentradas, podemos decir que:
Notacin Tensorial

Notacin de Voigt

r
r
F loc u loc = : dV

{F } {u }= { } { } dV
loc T

loc

(5.108)

donde {F loc }= {F1 , F2 ,..., Fn }T , {u loc }= {U1 , U2 ,..., Un }T , y la direccin de la componente uiloc
es igual a direccin de la componente Filoc .
F2

F1

F3
u

u
,

REAL

VIRTUAL
Figura 5.12

NOTA 1: En otras palabras, el Principio del trabajo Virtual afirma que: Una estructura
est en equilibrio, bajo la accin de un sistema de fuerzas exteriores, si y solo si el trabajo

Universidad de Castilla- La Mancha


Ciudad Real - Espaa

Draft

Por: Eduardo W. V. Chaves (2014)

MECNICA DEL MEDIO CONTINUO: PROBLEMAS RESUELTOS

424

externo virtual total es igual al trabajo interno virtual total, cualquier que sea el campo de
r
desplazamiento virtual ( u ).
NOTA 2: El Principio del Trabajo Virtual se utiliza en las tcnicas de discretizacin del
problema como por ejemplo la Tcnica de los Elementos Finitos, en el cual la incgnita
fundamental es el campo de desplazamiento.
NOTA 3: Es de fcil demostracin que la ecuacin (5.102) tambin es vlida para las tasas
r

&
de los campos virtuales u , & , i.e.:

r r r&
r r&
&&) u dV = : & dV
t * u dS + (b u

S
1
44444V2444443

Principio del trabajo virtual

V
14243

(5.109)

Trabajo interno
virtual total

Trabajo externo virtual total

Tambin es valida para una variacin de los campos u , , i.e.:

r r
r
r
r
&&) u dV = : dV
t * u dS + (b u

1444444V2444444
3
Trabajo externo virtual total

Principio del trabajo virtual

V
14243

(5.110)

Trabajo interno
virtual total

NOTA 4: Podemos definir tambin el Principio del trabajo virtual complementario donde las
incgnitas fundamentales son tensiones (fuerzas):

r r
&r& r
t u* dS ur + (b u ) udV =

S ur

: dV

Principio del trabajo virtual


complementario

V
14243

1444442444443

Trabajo interno
virtual complementario total

Trabajo externo virtual complementario total

(5.111)

con n = t * en S . Considerando un caso esttico sin fuerzas msicas y considerando


que las nicas acciones externas son fuerzas concentradas, el principio del trabajo virtual
complementario viene dado por:
r
loc r loc
F
u43
142

Trabajo externo virtual


complementario total
(Fuerzas concentradas)

Principio del trabajo virtual


complementario (Caso esttico,
sin fuerzas msicas y fuerzas
concentradas)

: dV

V
14243

Trabajo interno
virtual complementario total

(5.112)

Ejemplo 5.17
Considrese un sub-dominio ( ), en equilibrio, y constituido por un material homogneo,
elstico linear e istropo. Considrese tambin que en unos puntos del contorno del subdominio hay unas fuerzas concentradas aplicadas {F ( e ) } {F loc } y que el campo de
r
r
desplazamiento es aproximado a travs de {u( x )} = [N ( x )]{u (e ) } donde {u ( e ) } {u loc } son
los desplazamientos en los puntos donde se aplican las fuerzas concentradas. Demostrar
que las ecuaciones de gobierno para un problema elstico linear e istropo pueden ser
reemplazadas por:

{F }= [K ]{u }
(e)

(e )

(e)

donde

[K ] = [B] [C ][B] dV
T

(e)

Universidad de Castilla- La Mancha


Ciudad Real - Espaa

Draft

(5.113)

Por: Eduardo W. V. Chaves (2014)

5 INTRODUCCIN A: ECUACIONES CONSTITUTIVAS, PVCI, Y ESTRATEGIAS DE SOLUCIN

425

donde [C ] es la matriz constitutiva elstica en notacin de Voigt, y obtener la expresin de


[B( xr )] .
Nota: Usar el Principio del Trabajo Virtual.
Solucin:
Podemos partir directamente de la ecuacin (5.108), el cual es equivalente a:
r
r
r
F loc u loc = : dV = : ( sym u ) dV

(5.114)

r
r
r
u loc F loc = ( sym u ) : dV

La ecuacin anterior en notacin de Voigt queda:


r
r
r
u loc F loc = ( sym u ) : dV Voigt

u ( e )

{ } {F }= { } { } dV

(e)

(5.115)

Notar que en la ecuacin anterior ya est teniendo en cuenta las ecuaciones de equilibrio (ver
ecuaciones (5.106)-(5.108)). La ecuacin constitutiva de tensin, en notacin de Voigt, viene dada
r
r
por { ( x )} = [C ]{ ( x )} , donde el campo del tensor de deformacin viene dado por
r
r
( x ) = sym u (Ecuaciones cinemticas). Tenemos que hacer la representacin de la parte

simtrica del gradiente del campo de desplazamiento, ij = 12 (u i , j + u j ,i ) , en notacin de


Voigt. En el Ejemplo 5.8 hemos demostrado que:

u1

x1
x1

2
0

11

x 2


u

3
22
0
r
x3
33
{ ( x )} = = u u =
2 12 1 + 2
2 23 x 2 x1 x 2

u
u
2 13 2 + 3 0

x3 x 2
u1 u 3
x + x x
3
1
3

x 2
0

x1

x 3
0


u1
x3
u 2
0 u
3


x 2


x1

{ ( xr )} = [L(1) ]{u( xr )}

Luego

{ ( xr )} = [L(1) ]{u( xr )} = [L(1) ][N ( xr )]{u (e) }= [B( xr )] {u (e) }


donde definimos que:

[B( xr )] = [L(1) ][N ( xr )]

(5.116)

El campo de tensin se puede obtener por:

{ ( xr )} = [C ]{ ( xr )} = [C ][B( xr )]{u (e) }

Podemos utilizar la misma aproximacin del campo de desplazamiento para aproximar el


campo de desplazamiento virtual, con lo cual tambin se cumple que:

{u ( xr )} = [N ( xr )]{u (e) }

{ ( xr )} = [B( xr )] {u (e) }

Luego, la ecuacin (5.115) queda:


Universidad de Castilla- La Mancha
Ciudad Real - Espaa

Draft

Por: Eduardo W. V. Chaves (2014)

MECNICA DEL MEDIO CONTINUO: PROBLEMAS RESUELTOS

426

{u } {F }= { } { } dV = {[B( xr )]{u }} [C ][B( xr )]{u } dV


(e) T

(e)

(e)

(e)

o an:

{u } {F }= {u } [B( xr )] [C ][B( xr )]{u }dV


(e) T

(e) T

(e)

(e)

(5.117)

Observar que ni {u (e ) } ni {u (e ) } son dependientes de x , con lo cual:

{u } {F }= {u } [B( xr )] [C ][B( xr )] dV {u }

(e) T

(e) T

(e )

(e)

r T
r
F ( e ) = [B( x )] [C ][B( x )] dV u (e )

{ }
{F } = [K ]{u }
(e)

(e)

(5.118)

{ }

(e )

NOTA: [ K (e ) ] es conocida como matriz de rigidez del sub-dominio (elemento finito), y la


r

matriz [N ( x )] de la relacin {u( x )} = [N ( x )]{u (e ) } se conoce como la matriz que contiene


las funciones de forma. Las funciones de forma son las funciones definidas en un dominio que
r
nos permite obtener el valor de una funcin incgnita {u( x )} en cualquier punto del

dominio a travs de los valores nodales conocidos {u (e ) } . Por ejemplo, supongamos que el
sub-dominio ( ) viene caracterizado por un cuadriltero, luego podemos obtener el valor
r
del campo escalar T ( x ) a travs de sus valores nodales {T (e ) } como sigue:
T (e ) _ 2

{ }

r
r
T ( x ) = [N ( x ) ] T ( e )

T (e) _ 1
(e) _ 2
r T

= [N ( x )] ( e ) _ 3

T
(e) _ 4

T (e ) _ 3

r
T ( x)

T (e ) _ 1

T (e ) _ 4

Ejemplo 5.18
a) Considrese un subdominio ( ) constituido por un material elstico lineal e istropo.
Considrese tambin que en unos puntos del contorno del sub-dominio hay unas fuerzas
concentradas aplicadas {F ( e ) } {F loc } (fuerzas nodales) y que el campo de desplazamiento
r
r
es aproximado a travs de {u( x )} = [N ( x )]{u (e ) } donde {u ( e ) } {u loc } (desplazamientos
nodales) son los desplazamientos en los puntos donde se aplican las fuerzas concentradas.
Demostrar que las ecuaciones de gobierno para un problema elstico linear e istropo
pueden ser reemplazadas por:

[K ]{u }+ [M ]{u&& }= {F }
(e)

(e)

(e)

( e)

(e)

(5.119)

donde [ K ( e ) ] = [B]T [C ] [B] dV (matriz de rigidez), [ M ( e ) ] = [ N ]T [ N ] dV (matriz de


V

masa), [C ] es el tensor constitutivo elstico en notacin de Voigt.

b) Demostrar que la ecuacin en (5.119) representa un sistema conservativo.


Universidad de Castilla- La Mancha
Ciudad Real - Espaa

Draft

Por: Eduardo W. V. Chaves (2014)

5 INTRODUCCIN A: ECUACIONES CONSTITUTIVAS, PVCI, Y ESTRATEGIAS DE SOLUCIN

427

OBS.: Usar el Principio del Trabajo Virtual y considerar las siguientes aproximaciones para
r
los campos {( x )} en funcin de sus respectivos valores nodales { (e ) } :
r r
r
{u( x )} = [N ( x )]{u ( e ) } (campo de desplazamientos)

r& r
r
{u
( x )} = [N ( x )]{u& ( e ) } (campo de velocidad)

&r& r
r
&& ( e ) } (campo de aceleracin)
{u( x )} = [N ( x )]{u
r
{ ( x )} = L(1) [N ( xr )]{u ( e ) } = [B( xr )]{u ( e ) } (campo de deformacin)

r
r
(e)
{ ( x )} = [C ]{ ( x )} = [C ][B]{u } (campo de tensin)
r
r
{t * ( xr )} = [N t ( xr )]{ f r( e ) } (campo del vector traccin)
t

r
r
r
(e )
b r
{b( x )} = [N ( x )]{ f br } (campo de fuerzas msicas)

[ ]

(5.120)

Usar las mismas aproximaciones para los respectivos campos virtuales.


Solucin:
El Principio del Trabajo Virtual postula que:

: dV =

V
14243
Trabajo interno
virtual total

r r r
r r
&&) u dV
t * u dS + (b u

1444442444443
Trabajo externo virtual total

Reescribimos la ecuacin anterior en notacin de Voigt:


r

&r&

{} {} dV = {t} {u} dS + {b} {u} dV {u} {u} dV


T

S &

r
r r
r
r
&r&} dV
= {u}T {t} dS + {u}T {b} dV {u}T {u

S &

(5.121)

Utilizando las aproximaciones para los campos, podemos obtener los siguientes trminos:

{} {} dV = {} {} dV = {[B]{u }} {[C ][B]{u }}dV =


T

(e)

(e)

= {u ( e ) }T [B] [C ][B]{u (e ) } dV

T
= {u ( e ) }T [B ] [C ][B] dV {u ( e ) }

= {u ( e ) }T K ( e ) {u ( e ) }

r r
{u}T {t} dS =

S &

{[N ]{u }} [N ]{ f
(e)

r
t

r( e ) } dS
t

= {u (e ) }T [N ] [Nt ]{ f tr(e ) } dS

S &

S &

r
r
T
= {u (e ) }T [N ] [Nt ] dS { f tr( e ) } = {u (e ) }T [Gt ]{ f tr( e ) }

S &

= {u (e ) }T {Frt( e ) }

Universidad de Castilla- La Mancha


Ciudad Real - Espaa

Draft

Por: Eduardo W. V. Chaves (2014)

MECNICA DEL MEDIO CONTINUO: PROBLEMAS RESUELTOS

428

r
r
{u}T {b} dV =

{[N ]{u }} [N
(e )

r
b

T
]{ f bre } dV = {u ( e ) }T [N ] [Nb ] dV { f bre }

= {u (e ) }T [Gb ]{ f br(e ) }
r
= {u (e ) }T {Fbr( e ) }

r
&r&} dV =
{u}T {u

{[N ]{u }} [N ]{u&&


(e)

(e )

&& ( e ) } dV
} dV = {u (e ) }T [N ] [N ]{u

(e)

T
&& }
= {u (e ) }T [N ] [N ] dV {u

V
(e) T
(
e
)
&& }
= {u } [M ]{u

Teniendo en cuenta los trminos anteriores en la ecuacin (5.121) obtenemos que:


r
r
&& ( e ) }
{u ( e ) }T [ K ( e ) ]{u( e ) } = {u (e ) }T {Ftr( e ) } + {u ( e ) }T {Fbr( e ) } {u (e ) }T [M ]{u
r
r
&& ( e ) }
{u ( e ) }T [ K ( e ) ]{u(e ) } = {u ( e ) }T {Ftr(e ) } + {Fbr(e ) } [M ]{u

Ya que el desplazamiento virtual es arbitrario, concluimos que:

r
r
&& ( e ) }
[ K (e ) ]{u(e ) } = {Ftr( e ) } + {Fbr( e ) } [M ]{u
r
r
&& (e ) } = {Fr( e ) } + {Fr(e ) }
[ K (e ) ]{u(e ) } + [M ]{u
t
b
r (e)
(e)
(e )
(
e
)
&& } = {F }
[ K ]{u } + [M ]{u

b) Para demostrar que el sistema anterior es conservativo, i.e. sin disipacin de energa,
vamos considerar la discretizacin del tiempo donde el tiempo actual viene representado
por t y el tiempo sucesivo por t + t , donde t es el incremento de tiempo. Notar que
para cualquier tiempo la ecuacin anterior se cumple, luego:
r
&& ( e ) }t = {F (e ) }t
[ K (e ) ]t {u(e ) }t + [M ]t {u
r
r
(e)
(e)
&& ( e ) }t + t = {F ( e ) }t + t = {F ( e ) }t
[ K ]t + t {u }t + t + [M ]t + t {u
&& ( e ) }t + t = [ K ( e ) ]t {u( e ) }t + [M ]t {u
&& ( e ) }t
[ K (e ) ]t + t {u( e ) }t + t + [M ]t + t {u
r
donde la fuerza {F (e ) } es constante con el tiempo, con eso demostramos que el sistema es
r
conservativo. Notar tambin que, si el vector {F (e ) } es constante en el tiempo, luego:

(
(

)
)

r
r
D
&& (e ) } = D {F ( e ) } = {0 }
[ K ( e ) ]{u ( e ) } + [M ]{u
Dt
Dt
(e)
(e)
(e )
&
&
[ K ]{u } + [M ]{u } = constante en el tiempo

NOTA 1: La ecuacin en (5.119) representa un movimiento harmnico simple. Vamos


considerar un caso unidimensional donde [ K (e ) ] representa la constante de muelle k , [M ]
representa la masa m , y el desplazamiento y aceleracin vienen representados
respectivamente por u y u&& , (ver Figura 5.13). Con eso, la ecuacin (5.119) queda:
ku + mu&& = 0

Universidad de Castilla- La Mancha


Ciudad Real - Espaa

Draft

ku = mu&&

Por: Eduardo W. V. Chaves (2014)

5 INTRODUCCIN A: ECUACIONES CONSTITUTIVAS, PVCI, Y ESTRATEGIAS DE SOLUCIN

429

u
t

Figura 5.13: Modelo mecnico para un movimiento harmnico simple.


Notar que la energa se conserva. Considerando la energa interna debido al muelle ( 12 uku )
y la energa cintica ( 12 u&mu& = 12 mv 2 ) debido a la masa m , y aplicando la ecuacin de energa
podemos obtener que:
DK DU DW DQ
+
=
+
=0
Dt
Dt 1Dt
Dt
4243
=0

D 1

D 1
uku = 0
u&mu& +
Dt 2

Dt 2
mu&&u& + kuu& = 0
(mu&& + ku )u& = 0

s =W

(5.122)

mu&& + ku = 0

donde K es la energa cintica y U es la energa interna.


NOTA 2: Si hacemos un experimento con el modelo mecnico descrito en la Figura 5.13
vamos verificar que en realidad el movimiento no es conservativo, i.e. hay una pierda de
energa a lo largo del tiempo. En otras palabras hay un amortiguamiento del sistema hasta
alcanzar el reposo, (ver Figura 5.14). Este fenmeno se debe al mecanismo interno de la
estructura. En la prctica, este amortiguamiento intrnseco a las estructuras se puede tratar
a travs de un parmetro c (amortiguamiento) multiplicado por la velocidad. Con eso la
ecuacin (5.122) queda:
mu&& + cu& + ku = 0

Si tenemos en cuenta el fenmeno de amortiguamiento en la ecuacin (5.119), sta puede


ser reescrita como:

[K ]{u }+ [C ]{u& }+ [M ]{u&& }= {F }


(e)

(e)

(e)

(e)

(e)

(e)

(e)

(5.123)

donde [C (e ) ] es la matriz de amortiguamiento. Notar que para la resolucin del sistema


(5.123) necesitaremos hacer tambin una integracin en el tiempo. Para mayores
informaciones sobre Dinmica Estructural ver Tedesco et al. (1998).

Universidad de Castilla- La Mancha


Ciudad Real - Espaa

Draft

Por: Eduardo W. V. Chaves (2014)

MECNICA DEL MEDIO CONTINUO: PROBLEMAS RESUELTOS

430

Figura 5.14: Modelo mecnico para un movimiento con amortiguamiento.

Ejemplo 5.19
Si consideramos la energa potencial elstica total dada por:
r r
r r
r
(u) = e ( ) dV t * udS ( b) udV

Energa Potencial Total

(5.124)

2 : dV

(5.125)

r r
r r
U ext = t * udS + ( b) udV

(5.126)

donde
U int = e ( ) dV =
V

Si consideramos tambin que la primera variacin de , denotada por , es igual a cero


para un valor estacionario de , demostrar que: si = 0 es equivalente al valor
r
estacionario de , entonces (u) corresponde realmente a un valor mnimo.
r r

OBS.: Considerar que durante el proceso de deformacin las acciones externas ( t * , b ) no


varan con la deformada. Considerar tambin un material elstico lineal.
Solucin:
La primera variacin ( ) se puede obtener como sigue:

Universidad de Castilla- La Mancha


Ciudad Real - Espaa

Draft

Por: Eduardo W. V. Chaves (2014)

5 INTRODUCCIN A: ECUACIONES CONSTITUTIVAS, PVCI, Y ESTRATEGIAS DE SOLUCIN

1
r r
r r
=
: dV t * udS ( b) udV
2

S
V
V

r
r
r
r
1
= : dV t * udS ( b) udV
2
V
S
V

431

(5.127)

r
r
r
r
1
( : ) dV t * udS ( b) udV
2
S
V

Notar que:

1
1
1
( : ) = ( : + : ) = (C e : ) : + :
2
2
2
1
1
1
= (C e : ) : + : = : C e : + : = [ : + : ]
2
2
2
= :

e ( ) =

] [

(5.128)

e
:

donde hemos considerado que =

e
(ver Ejemplo 5.5). Para rgimen de pequeas

deformaciones, la ecuacin anterior tambin puede ser escrita como:


e ( ) =

r
r
r
e
: = : = : ( symu) = : ( sym u) = : (u)

(5.129)

donde hemos utilizado la propiedad A sym : B = A sym : (B sym + B anti ) = A sym : B sym . Con lo
cual la ecuacin (5.127) queda:
r
r
r
r
1
( : ) dV t * udS ( b) udV
2
V
S
V
r
r* r
r
= : dV t udS ( b) udV

r
r
r
r
= e dV t * udS ( b) udV

Teniendo en cuenta la definicin de dada por la ecuacin (5.124), la expresin


r
r
(u + u) puede ser obtenida como
r r
r r
r
r
r
r
(u + u) = e ( + ) dV t * (u + u)dS ( b) (u + u)dV

(5.130)

Utilizamos serie de Taylor para aproximar el trmino e ( + ) , resultando:

e ( + ) = e () +
Notar que

1
e ( )
2 e ( )
: + :
: + ...
2

(5.131)

2 e ( )
e ( )
: = e (ver ecuacin (5.128)) y C e =
(ver Ejemplo 5.5).

Adems recordar que para un problema elstico lineal la energa de deformacin requiere
una funcin cuadrtica. Con lo cual la ecuacin (5.131) queda:

Universidad de Castilla- La Mancha


Ciudad Real - Espaa

Draft

Por: Eduardo W. V. Chaves (2014)

MECNICA DEL MEDIO CONTINUO: PROBLEMAS RESUELTOS

432

1
2 e ( )
e ( )
: + :
: + ...
2

1
e ( ) + e + : C e :
2

e ( + ) = e () +

Con lo cual, la ecuacin (5.130) puede ser reescrita como:


r
r
(u + u) = e ( + ) dV

r
r
r
r
r
r
t * (u + u)dS ( b) (u + u)dV

= e ( ) dV + e dV +
V

r*

= ( ) dV
e

2 : C

: dV
r

(u + u)dS ( b) (u + u)dV
V

r r
r r
t * udS ( b) udV +

+ e dV
V

r
r
r
r
1
t * udS ( b) udV +
: C e : dV
2
V
V

(5.132)
Notar que:

r r
r r
r
(u) = e ( ) dV t * udS ( b) udV

r
r
r
r
= e dV t * udS ( b) udV = 0

Teniendo en cuenta lo anterior en la ecuacin (5.132) podemos obtener que:


r
r
r
1
(u + u) = (u) + +
: C e : dV
2
V

r
r
r
1
(u + u) (u) = +
: C e : dV
2
V

r
r
r
1
(u + u) (u) =
: C e : dV
2
V

donde hemos considerado que = 0 . Notar que el trmino : C e : > 0 siempre ser
positivo para cualquier valor de ya que C e es un tensor definido positivo (ver Captulo
1). Con lo cual garantizamos que:
r
r
r
1
= (u + u) (u) =
: C e : dV > 0
2
V
r
= 0
(u) es un mnimo

r
r
r
(u + u) > (u)

NOTA 1: Para un problema elstico linear el punto de equilibrio corresponde al valor mnimo
de , ver Figura 5.15. Este es el conocido Principio de la Energa Potencial Mnima.

Universidad de Castilla- La Mancha


Ciudad Real - Espaa

Draft

Por: Eduardo W. V. Chaves (2014)

5 INTRODUCCIN A: ECUACIONES CONSTITUTIVAS, PVCI, Y ESTRATEGIAS DE SOLUCIN

433

NOTA 2: Para el caso particular donde las acciones externas estn constituidas por fuerzas
concentradas y en ausencia de fuerzas msicas, tenemos que:

{ } {u }

r
1
(u) = U int + U ext =
: dV F loc
2
V

loc

Energa Potencial Total

(5.133)

(u )
F

(u ( 2 ) )
(u )

=0
u

= 0

(u ( 3 ) )

Deformada correspondiente al
equilibrio

u (2)

u ( 3)

Figura 5.15
NOTA 3: A travs de la ecuacin (5.133) podemos demostrar el Teorema de

Castigliano- Parte I:

r
(u)
loc

{u }

{F loc } =

U int
loc

{u }

U ext
loc

{u }

U int
loc

{u }

]=0

{F loc }T {u loc }
loc

{u }

U int
{u loc }

donde {F loc } = {F1 , F2 ,..., Fn }T , {u loc } = {U1, U2 ,..., Un }T . Notar que en la expresin anterior el
trmino U int tiene que estar en funcin de {uloc } .
NOTA 4: Para una mejor ilustracin del problema planteado, vamos considerar una barra
de longitud L y con rea de la seccin transversal A . Consideremos tambin que el campo
de tensin y de deformacin son campos homogneos y dados por:
0 0
ij = 0 0 0
0 0 0

0 0
e
ij = 0 0 0 11 = C1111
11 = E
0 0 0

Consideremos tambin que el campo de desplazamiento viene representado por una


aproximacin lineal ( u ( x) = a1 + a2 x ) y en las extremidades de la barra tenemos que:

Universidad de Castilla- La Mancha


Ciudad Real - Espaa

Draft

Por: Eduardo W. V. Chaves (2014)

MECNICA DEL MEDIO CONTINUO: PROBLEMAS RESUELTOS

434

(3D )

V = AL (volumen)

F = dA
A

(1)

F ,U

(1D )

(1)

( 2)

( 2)

,U

Figura 5.16
El objetivo ahora es escribir la energa potencial total en funcin de U(1) ,U( 2) . Verificar que,
debido a las cargas concentradas tenemos que:
U ext = {F loc }T {uloc } = {F (1)

U(1)
F ( 2 ) } ( 2 ) = F (1)U(1) + F ( 2 )U( 2 )
U

(5.134)

En este ejemplo la relacin lineal tensin-deformacin viene dada por = E (ver


1
2

1
2

Ejemplo 5.5 NOTA 3) y la densidad de energa de deformacin e = = E .


Luego, la energa interna total viene dada por:
U int =

2 : dV

1D

U int =

notar que 11 =

1
1
1
dV =
E dV =
E 2 dV
2V
2V
2V

u1 u ( x)
=
= , con lo cual:
x1
x

U int =

1
1
u ( x)
E 2 dV =
E
dV
2V
2 V x

(5.135)

Nuestro objetivo ahora es expresar el campo de desplazamiento en funcin de sus valores


nodales U(1) ,U( 2) . Para ello vamos utilizar la aproximacin adoptada u ( x) = a1 + a2 x , donde
se cumple que:

= a1 + a2 L

u ( x = 0) = U(1) = a1
u ( x = L ) = U( 2 )

U(1) 1 0 a1
( 2) =

U 1 L a2

Ahora nuestro objetivo es definir los coeficientes a1 , a 2 . Para ello, obtenemos la inversa
de la relacin anterior:
U (1) 1 0 a1
(2) =

U 1 L a 2

inversa

a1 1 L 0 U (1)
=
( 2)
a 2 L 1 1 U

a1 = U (1)

1 ( 2)
(1)
a 2 = U U
L

con lo cual:

Universidad de Castilla- La Mancha


Ciudad Real - Espaa

Draft

Por: Eduardo W. V. Chaves (2014)

5 INTRODUCCIN A: ECUACIONES CONSTITUTIVAS, PVCI, Y ESTRATEGIAS DE SOLUCIN

u ( x) = a1 + a2 x = U(1) +

435

1 (2)
x
x

U U(1) x = 1 U(1) + U( 2 )
L
L
L

(5.136)

(1)
r

x x U
u ( x) = 1 ( 2 ) = [N ( x )] u(e )
L L U

{ }

Con lo cual la ecuacin (5.135) puede ser reescrita como:


U int =

2
1
E 1 (2)
E
u ( x)
( 2) 2
(1)
2U(1)U( 2 ) + U(1) dV
E
U U dV = 2 U
dV =

2 V x
2VL
2L V

Notar que U(1) y U( 2) no dependen de x , con lo cual:


U int =

2
2
2
E (2) 2
E
2U(1)U( 2 ) + U(1) dV = 2 U( 2 ) 2U(1)U( 2 ) + U(1) V
U
2

2L
2L
V

(5.137)

2
2
EAL ( 2) 2
EA ( 2 ) 2
=
2U(1)U( 2 ) + U(1) =
2U(1)U( 2 ) + U(1)
U
U
2

2L
2L

Luego, la energa potencial total (ver ecuacin (5.133)) viene dada por las ecuaciones
(5.134) y (5.137), i.e.:

r
2
EA ( 2 ) 2
(u) = U int U ext =
2U(1)U( 2 ) + U(1) F (1)U(1) + F ( 2 )U( 2 ) = (U(1) , U( 2 ) )
U

2L

Como buscamos el estado estacionario, hay que cumplir que:


(U(1) , U( 2) )

U(1)

(1)
( 2)
(U , U )

U( 2 )

=
=
=
=
=
=

2
EA ( 2 ) 2

U 2U(1)U( 2) + U(1) F (1)U(1) + F ( 2 )U( 2 ) = 0


(1)

U 2 L

EA
2U( 2) + 2U(1) F (1) = 0
2L
EA (1)
U U( 2 ) F (1) = 0
L
2
EA ( 2 ) 2

U 2U(1)U( 2) + U(1) F (1)U(1) + F ( 2)U( 2 ) = 0


(2)

2
L
U

EA
2U( 2 ) 2U(1) F ( 2 ) = 0
2L
EA ( 2)
U U(1) F ( 2 ) = 0
L

(
(

(
(

Reestructurando las ecuaciones anteriores en forma matricial obtenemos que:


(1)
(1)
EA 1 1 U F
=

L 1 1 U ( 2 ) F ( 2 )

[K ]{u }= {F }
(e)

(e)

(e)

(5.138)

Notar que [ K (e ) ] no tiene inversa ya que det[ K ( e ) ] = 0 . Para que el problema tenga
solucin nica, tenemos que introducir las condiciones de contorno.
Notar que la matriz [ K (e ) ] de la expresin anterior podra haber sido obtenida a travs de
la ecuacin (5.118) (ver Ejemplo 5.17), donde para este caso particular tenemos que
[C ] = E , y a travs de la ecuacin (5.136) podemos obtener que:

[B( xr )] = [L(1) ][N ( xr )] =

Universidad de Castilla- La Mancha


Ciudad Real - Espaa

x x 1 1
1 L L = L L

Draft

Por: Eduardo W. V. Chaves (2014)

MECNICA DEL MEDIO CONTINUO: PROBLEMAS RESUELTOS

436

Luego:
1

r T
r
1 1 1
L 1 1
= [B( x )] [C ][B( x )] dV =
E dV = E 2
dV
1 L L
L 1 1
V
V
V

L
E 1 1
E 1 1
EA 1 1
= 2
dV = 2
V=

L 1 1
L 1 1 V
L 1 1

[K ]
(e)

NOTA 5: Es interesante notar que, inicialmente tenamos un problema continuo (infinitos


puntos materiales) representado por las ecuaciones de gobierno y hemos transformado este
problema continuo en un problema discreto representado por un conjunto de ecuaciones
discretas (5.138). En otras palabras, hemos aplicado una tcnica numrica para resolver el
problema. El principal objetivo de cualquier tcnica numrica es transformar un problema
continuo en un conjunto de ecuaciones discretas, entre estas tcnicas numricas podemos
citar: Diferencias Finitas, Elementos Finitos, Elementos de Contorno, Volumen Finito, etc.

NOTA 6: Analizando [N ( x )]
T

r
r
r

x x
Note que las funciones de forma son [N ( x )] = [N1 ( x ) N 2 ( x )]T = 1 . Si
L L

dibujamos estas funciones dentro del dominio obtenemos que:


N1

N1 ( x = 0) = 1
x

L
N1 ( x = L ) = 0
N 2 ( x = 0) = 0
x
N 2 ( x) =
L
N 2 ( x = L) = 1

N2

N1 ( x ) = 1

N1 ( x ) + N 2 ( x ) = 1

Figura 5.17
r

La aproximacin adoptada para [ N ( x )] depender del problema. Para el problema


analizado en este ejemplo tenemos que la deformacin es constante dentro del dominio,
luego, es suficiente adoptar una aproximacin linear para el campo de desplazamientos ya
que por definicin =
r

u ( x)
. Como consecuencia necesitamos apenas dos nodos para
x

definir [ N ( x )] . Si el problema requiere una funcin cuadrtica para el campo de


r
desplazamientos, necesitaremos tres puntos para definir [ N ( x )] , y as sucesivamente.

Universidad de Castilla- La Mancha


Ciudad Real - Espaa

Draft

Por: Eduardo W. V. Chaves (2014)

5 INTRODUCCIN A: ECUACIONES CONSTITUTIVAS, PVCI, Y ESTRATEGIAS DE SOLUCIN

437

NOTA 7:

Principio del Potencial de Energa Estacionario


En este ejemplo hemos establecido el Principio de la Energa Potencial Estacionaria, (ver
ecuacin (5.124)):
r r
r r
r
(u) = e ( ) dV t * udS ( b) udV

(5.139)

1
2

donde hemos tenido en cuenta que e () = : . El funcional es estacionario si y solo si


r
ur (u) = 0 .

Principio Variacional de Hellinger-Reissner


En el Ejemplo 5.5 en la NOTA 7 hemos establecido que
tensorial

e ( ) = : e ( ) = 0 G( ) = g( )
e () = e ()
e ( ) = : g( )

(5.140)

donde g( ) es la densidad de energa libre de Gibbs con el signo invertido.


g() - Energa almacenada complementaria

e () - Energa almacenada

Figura 5.18: Energas almacenadas.


Reemplazando el valor e ( ) = : g( ) en el funcional (5.139) obtenemos que:
r
(u) = e ( ) dV

r r
r r
t * udS ( b) udV

r
HR (u, ) = : g( ) dV

r r
r r
t * udS ( b) udV

(5.141)

Notar que : = : ( sym u) = : (u) . Con lo cual podemos obtener que:


r r
r r
r
r
HR (u, ) = : (u) g( ) dV t * udS ( b) udV

Universidad de Castilla- La Mancha


Ciudad Real - Espaa

Draft

(5.142)

Por: Eduardo W. V. Chaves (2014)

MECNICA DEL MEDIO CONTINUO: PROBLEMAS RESUELTOS

438

El Funcional (5.141) es estacionario para una variacin arbitraria de u y desapareciendo en


S ur si y solo si satisface las ecuaciones de equilibrio. Y es estacionario para una variacin
de si y solo si satisface la ecuacin constitutiva (tensin-deformacin). Este principio es
conocido como Principio Variacional de Hellinger-Reissner.
r
r
ur HR (u, ) = : (u) dV

r
r
r
r
t * udS ( b) udV = 0

= ij (u) i , j dV
V

V
*
i (u) i

dS ( b) i (u) i dV = 0

= ij (u) i n j dS ij , j (u) i dV
S

= ij , j + ( b) i (u) i dV +
V

t *i (u) i dS ( b) i (u) i dV = 0

(5.143)

t *i (u) i dS = 0

ij n j

En el volumen obtenemos las ecuaciones de equilibrio: ij , j + ( b) i = 0 i .


En la superficie S la condicin de contorno: ij n j t *i = 0 i
r
r
HR (u, ) = : ( sym u) g( ) dV = 0

r
g( )
: dV = 0
= ( sym u) :

(5.144)

r
g( )

: dV = 0
= ( sym u)

V

En el volumen obtenemos las ecuaciones constitutivas: ( sym u)

g( )
=0.

Principio Variacional de Hu-Washizu


El Principio de Hu-Washizu es una generalizacin del Principio de Hellinger-Reissner,
r
donde el funcional adems de depender de (u, ) tambin depende del campo de
deformacin , y viene dado por:
r
HW (u, , ) =

[ () : (
e

r r
r r
r
r
u) ( b) u dV ( n ) (u* u)dS t * udS

sym r

S ur

(5.145)
y estacionario para las siguientes situaciones:
r
ur HW (u, , ) = 0
r r

En S ur
u = 0

Ecuaciones de equilibrio

r
HW (u, , ) = 0

Ecuaciones cinemticas

Condicin de contorno en S ur

r
HW (u, , ) = 0

Ecuaciones constitutivas tensin - deformacin

Universidad de Castilla- La Mancha


Ciudad Real - Espaa

Draft

Por: Eduardo W. V. Chaves (2014)

5 INTRODUCCIN A: ECUACIONES CONSTITUTIVAS, PVCI, Y ESTRATEGIAS DE SOLUCIN

439

Es decir:
r
ur HW (u, , ) = 0

[ : ( ur ) ( br ) ur ] dV tr ur dS
r
r
r
r
r
r
(u, , ) = [( ) (u) ( b) u] dV t udS
r
r
r
r
r
(u, , ) = [( ) ( b)] (u) dV t udS

r
ur HW (u, , ) =

sym

ur HW

ur HW

r
HW (u, , ) = 0
r
r
r
r
HW (u, , ) = : ( sym u) dV ( n ) (u* u)dS = 0

S ur

r
r
HW (u, , ) = ( sym u) : dV
V

[n (u

r*

r
u) : dS = 0

S ur

r
HW (u, , ) = 0
r
HW (u, , ) = e ( ) : ( ) dV = 0

e ( )

e ( )

r
HW (u, , ) =
: : ( ) dV =
: dV = 0

V
V

NOTA 8: Discretizacin de los Campos


El principio variacional de Hu-Washizu tambin puede ser escrito como:

r r
r r
r
HW = e ( ) : ( symu) ( b) u dV t * udS = 0
V

r
r
r
r
r
= e ( ) : dV : ( sym u) dV ( b) u dV t * udS = 0

r
r
r
r r
= : e ( ) dV + : ( sym u ) dV u ( b)dV u t * dS = 0

r
r
r
r r
= : dV + : ( symu ) dV u ( b) dV u t * dS = 0

En la implementacin del mtodo de los elementos finitos, en general, utilizamos la


notacin de Voigt cuando estamos tratando con tensores simtricos. Utilizando la notacin
de Voigt, la ecuacin anterior queda:
HW =

T
T
T
T
sym
*
{} {} dV + {} { u }dV {u} {b}dV {u} {t }dS = 0

{} {} dV + {} {
T

{ }

r T r
u dV = {u} b dV +

sym r

{}

r r
{u}T t * dS

(5.146)

Vamos considerar las aproximaciones para los campos de desplazamientos, deformaciones,


y tensiones, ver Jirsek (1998), como sigue:
Universidad de Castilla- La Mancha
Ciudad Real - Espaa

Draft

Por: Eduardo W. V. Chaves (2014)

MECNICA DEL MEDIO CONTINUO: PROBLEMAS RESUELTOS

440

r
{u} [N ]{d} + [N c ]{d c }

{} [B]{d} + [G]{e}

{} [S]{s}

r
{u} [N ]{d} + [N c ]{d c }

{} [B]{d} + [G]{e}

{} [S]{s}

(5.147)

donde las matrices [N ] contiene las funciones de interpolacin para el campo de


desplazamiento (funciones de forma), y [B] contiene las derivadas de las funciones de
forma (matriz de interpolacin para el campo de deformacin). [N c ] y [G] son matrices
que contienen trminos de enriquecimientos para desplazamiento y deformacin
respectivamente. [S] es la matriz de interpolacin para el campo de tensin. {d} , {d c } ,
{e} y {s} contienen, respectivamente, los correspondientes grados de libertad de
desplazamientos nodales, modos de enriquecimiento de desplazamientos, modos de
enriquecimiento de deformacin, y parmetros de tensin. Teniendo en cuenta la variacin
del principio de Hu-Washizu:
T
T *
T
T
sym
{} {} dV + {} { u }dV = {u} {b}dV + {u} {t }dS

V
144244
3

V 44424443
1

(5.148)

144444
42S4
44444
3

podemos obtener que:


(1)

{} {} dV = {[B]{d} + [G]{e}} {} dV = {[B]{d}} {} dV + {[G]{e}} {} dV


T

= {d}

[B] {} dV + {e} [G] {} dV


T

r
T
(2) {} symu dV =

{} {
T

sym r

V
144424443

2.1

Universidad de Castilla- La Mancha


Ciudad Real - Espaa

u dV +

Draft

{} {
T

sym

r
u dV

V
1444
424444
3

2.2

Por: Eduardo W. V. Chaves (2014)

5 INTRODUCCIN A: ECUACIONES CONSTITUTIVAS, PVCI, Y ESTRATEGIAS DE SOLUCIN

{} {
T

(2.1)

sym r

u dV =

{[S]{s}} { ([N ]{d} + [N ]{d }) ([B]{d} + [G]{e})}dV


T

sym

441

{[S]{s}} { ([N ]{d} + [N ]{d })}dV {[S]{s}} {([B]{d} + [G]{e})}dV


T

sym

[S] { ([N ]{d} + [N ]{d })}dV {s} [S] {([B]{d} + [G]{e})}dV

= {s}

sym

[S] { ([N ]{d})}+ [S] { ([N ]{d })}dV {s} [S] {([B]{d} + [G]{e})}dV

= {s}

sym

sym

[S] [B]{d} + [S] [B ]{d }dV {s} [S] [B]{d} + [S] [G]{e}dV

= {s}

[S] [B ]{d }dV {s} [S] [G]{e}dV

= {s}

= {s}

[S] { [B ]{d } [G]{e} }dV


T

donde hemos considerado que { sym ([N ]{d})} = [B]{d} y { sym ([N c ]{d c })} = [B c ]{d c } .

( 2 .2 )

T
sym
sym
{} { u }dV = { u } {}dV

{ ([N ]{d} + [N ]{d }) ([B]{d} + [G]{e}) } {[S]{s}}dV


T

sym

{ ([N ]{d}) + ([N ]{d }) ([B]{d} + [G]{e}) } {[S]{s}}dV


sym

sym

{ [B]{d} + [B ]{d } ([B]{d} + [G]{e}) } {[S]{s}}dV


T

{ [B ]{d } [G]{e} } {[S]{s}}dV


T

{ {d } [B ] {e} [G] }{[S]{s}}dV


T

= {d c }

[B ] {[S]{s}}dV {e} [G] {[S]{s}}dV


T

(3)

T
T *
{u} {b}dV + {u} {t }dS

= {d} { f ext } + {d c } { f c }
T

Teniendo en cuenta los trminos anteriores, la ecuacin (5.148) queda:

Universidad de Castilla- La Mancha


Ciudad Real - Espaa

Draft

Por: Eduardo W. V. Chaves (2014)

MECNICA DEL MEDIO CONTINUO: PROBLEMAS RESUELTOS

442

{d}T [B]T {} dV + {e}T [G]T {{} {[S]{s}}} dV + {s}T [S]T { [B c ]{d c } [G]{e} }dV +
V

+ {d c }

[B ] {[S]{s}}dV = {d} { f } + {d } { f }
T

ext

(5.149)
r

Ya que {u} , {} y {} son variables de campos independientes, podemos concluir que:

T
T
T
{d} [B ] {} dV = {d} { f ext }

T
T
{e} [G] {{} {[S]{s}}} dV = {0}

T
T
{s} [S] [B c ]{d c } [G]{e} dV = {0}
V

{d }T [B ]T {[S]{s}}dV = {d }T { f } = {0}
c
c
c
c
V

(5.150)

Si consideramos que { f c } = {0}, la ecuacin anterior puede ser reescrita como:

T
[B ] {} dV = { f ext }
V

T
[G] {{} {[S]{s}}} dV = {0}
V

T
[S] [B c ]{d c } [G]{e} dV = {0}
V

[B ]T {[S]{s}}dV = {0}
c

(5.151)

Teniendo en cuenta la relacin tensin-deformacin:

{} = [C]{} = [C]{[B]{d} + [G]{e}}

(5.152)

y sustituyendo en la ecuacin (5.151) podemos obtener que:

T
T
[B ] [C][B] dV {d} + [B] [C][G] dV {e} = { f ext }
V
V

T
T
T
[G] [C][B] dV {d} + [G] [C][G] dV {e} [G] [S] dV {s} = {0}
V
V
V

T
T
[S] [B c ] dV {d}c [S] [G] dV {e} = {0}
V
V

B T [S] dV {s} = {0}


c

[ ]

Universidad de Castilla- La Mancha


Ciudad Real - Espaa

Draft

(a)
(b)
(5.153)
(c)
(d)

Por: Eduardo W. V. Chaves (2014)

5 INTRODUCCIN A: ECUACIONES CONSTITUTIVAS, PVCI, Y ESTRATEGIAS DE SOLUCIN

443

Reescribiendo las ecuaciones anteriores en forma matricial podemos obtener que:


[B]T [C][B]
T
[G] [C][B]

[B]T [C][G]
[G]T [C][G]
T
[S] [G]

0
T
[G] [S]
0

[B c ] [S]
T

dV
T
[S] [B c ]
0
0
0

{d} { f ext }
{e} {0}

{
}
s
{
}
0

{d c } {0}

(5.154)

Vamos suponer que no introducimos ningn enriquecimiento de desplazamientos, luego se


cumple que {d c } = {0} [B c ] = [0] , con eso la ecuacin en (5.153)(c) queda:

[S] [G] dV {e} = {0}


T

(5.155)

Con lo cual [S] = [1] (matriz identidad) es el requerimiento para que la funcin definida a
trozo de tensin {} sea constante. La condicin de compatibilidad (5.155) queda:

[G] dV = [0]

(5.156)

Discontinuidades en los campos de desplazamientos y deformaciones Aplicando


el Principio del Trabajo Virtual
Como hemos visto en ejemplo anterior, el trabajo virtual es el trabajo realizado por las
fuerzas reales sobre los desplazamientos virtuales. Un desplazamiento virtual es cualquier
desplazamiento consistente con las restricciones de la estructura, i.e. es aquel que satisface
las condiciones de contorno.
El principio establece que el trabajo realizado por las fuerzas internas debe ser igual al
trabajo realizado por las fuerzas externas:

: & dV =

V
14243
Trabajo interno
virtual total

r r r&
r r&
&&) u dV
t * u dS + (b u

144444V2444443
Trabajo externo virtual total

(5.157)

Wext

Wint

para todo desplazamiento admisible u .


Vamos considerar que el sistema est discretizado en elementos finitos (CST-Constant
Strain Triangle) y que todas las fuerzas externas estn aplicadas en los nodos 1-2-3-4 (CSTConstant Strain Triangle) (ver Figura 5.19).

Universidad de Castilla- La Mancha


Ciudad Real - Espaa

Draft

Por: Eduardo W. V. Chaves (2014)

MECNICA DEL MEDIO CONTINUO: PROBLEMAS RESUELTOS

444

Fy3

Fx3

u 2y

4y
Fx2

(a) fuerzas nodales

4x

u 1y

Fx1

u 3x

Fy2

Fy1

u 3y

u 1x

u x2

(b) desplazamientos nodales

Figura 5.19: Elemento finito CST discontinuo.


Las fuerzas y desplazamientos nodales pueden ser expresados como sigue:

{F}

{F}

Fx1

Fy1
Fx2

Fy2

Fx3
Fy3

0
0

{a e }
{ e }

u 1x
1
u y
u 2
x
u y2
= 3
ux
u 3
y4
x
4
x

(5.158)

Luego, el trabajo virtual exterior queda:

{F}
{ *e }T
{F}

Wext = {a*e }T {F} + { *e }T {F} = {a *e }T

(5.159)

Consideramos que el campo de deformacin y el campo virtual de deformacin vienen


dados por la composicin de dos partes:
~
{} = {} + {}

~
{ * } = { * } + { * }

(5.160)

Luego, el trabajo virtual interior queda:

~
~ T
Wint = {}T {} d V = {}T { * } + { * } d V = { *} + { *} {} d V

(5.161)

Formulacin simtrica

La discretizacin para esta primera aproximacin es:


{} = [B ]{a} + [G e ]{ e } = {[B ]
123 1
424
3
{ }

[G e ]}

{~ }

{a e }
*
B ]{a *e } + [G e ]{ *e } = {[B ]
; { } = [1
23 1
424
3
{

}
e
~*
*

{ }

{ }

[G e ]}{a e* }
{ e }

(5.162)

Notar que estamos utilizando las mismas funciones aproximantes [B] , [Ge ] para los
campos reales y virtuales de deformacin. El campo de tensin queda:
{a e }

{ e }

{} = [C ]{} = [C ]{[B] [Ge ]}


Universidad de Castilla- La Mancha
Ciudad Real - Espaa

Draft

(5.163)

Por: Eduardo W. V. Chaves (2014)

5 INTRODUCCIN A: ECUACIONES CONSTITUTIVAS, PVCI, Y ESTRATEGIAS DE SOLUCIN

445

Reemplazando las aproximaciones (5.162) y (5.163) dentro de la ecuacin (5.161), el trabajo


virtual interior quedas:
Wint =

~ T
{ } + { * } {} d V = {[B ]
V

{
V

{a * }
= e*
{ e }
V

= {a*e }T

*
[Ge ]}{a e*} [C ]{[B]
{ e }

[Ge ]}

{a e }
dV
{ e }

{a e }
dV
{ e }

{[B] [Ge ]}T [C ]{[B] [Ge ]}

{ *e }T

] [G[B]] [C ]{[B]
T

(5.164)

{a e }

{ e }

[Ge ]} d V

Aplicando Wext = Wint (ver Ec. (5.159) y (5.164)) podemos obtener que
[B]T [C ][B ]

T
[Ge ] [C ][B]
e

{F}
=
{F} V

[B]T [C ][Ge ] dV {a e }

[Ge ]T [C ][Ge ] { e }

(5.165)

y considerando la continuidad del vector de traccin, i.e. {F} = {0} , concluimos que:
[B]T [C ][B] [B]T [C ][Ge ]
{F}
dV
=
T
T
{0} Ve [Ge ] [C ][B ] [Ge ] [C ][Ge ]
14444442444444
3

{a e }

{ e }

(5.166)

[K e ]

Formulacin antisimtrica

Para esta segunda formulacin consideraremos que los campos reales y virtuales de
deformacin vienen dados por:
{a e }
{} = [B] {a e } + [Ge ]{ e } = [[B] [Ge ]]

1
424
3 1
424
3
{ e }
{ }

{~}

(5.167)

{a* }
{ * } = [B]{a*e } + [G*e ]{ *e } = [B] [G*e ] e*
{ e }

donde hemos considerado diferentes aproximaciones para los campos reales y virtuales i.e.
[Ge ] [G*e ] .
Utilizando la ecuacin (5.161), y la discretizacin (5.167) podemos obtener que:

Wint = {a*e }T

{ *e }T

} [B] [C ]{[B] [G ]} dV
[G ]
T

Ve

* T
e

{a e }

{ e }

(5.168)

Considerando Wext = Wint y la continuidad del vector de traccin, podemos obtener que:
[B]T [C ][B] [B]T [C ][Ge ]
{a e }
{F}
dV

= * T
T
*
{0} Be Ge [C ][B] Ge [C ][Ge ]
{ e }
14444442444444
3

[ ]

[ ]

(5.169)

[K e ]

Universidad de Castilla- La Mancha


Ciudad Real - Espaa

Draft

Por: Eduardo W. V. Chaves (2014)

MECNICA DEL MEDIO CONTINUO: PROBLEMAS RESUELTOS

446

Ejemplo 5.20
Consideremos una barra de longitud L y rea transversal A que est sometida a una
deformacin debido a su peso propio, (ver Figura 5.20 (a)). La barra est fija en el topo y
est en equilibrio esttico. Use la energa potencial total para obtener una expresin anloga
a la obtenida en el Ejemplo 5.19 en la NOTA 4, i.e. obtener una expresin equivalente a
[ K (e ) ]{u(e ) } = {F (e ) } asociada con este problema. Obtener tambin el campo de
desplazamientos.
Hiptesis: material elstico linear homogneo e istropo, rgimen de pequeas
deformaciones.

u=0
y
z

U(1) = 0

L
2

dV
r
b

U( 2)

L
2

g

bi = 0
0

a) 3D

U ( 3)

b) 1D
Figura 5.20

Solucin:
Para encontrar que aproximacin debemos adoptar para el campo de desplazamientos,
r r
analizaremos las ecuaciones de equilibrio ( + b = 0 ):
ij , j + b i = i1,1 + i 2, 2 + i 3,3 + b i = &u&i = 0 i
11,1 + 12, 2 + 13,3 + b1 = 0

21,1 + 22, 2 + 23,3 + b 2 = 0

31,1 + 32, 2 + 33,3 + b 3 = 0

11 12 13
+
+
= b1

x2
x3
x1
21 22 23
+
+
= b 2

x 2
x3
x1
31 32 33
+
+
= b 3

x1
x2
x3

y teniendo en cuenta que para este problema se cumple que:

Universidad de Castilla- La Mancha


Ciudad Real - Espaa

Draft

Por: Eduardo W. V. Chaves (2014)

5 INTRODUCCIN A: ECUACIONES CONSTITUTIVAS, PVCI, Y ESTRATEGIAS DE SOLUCIN

0 0
ij = 0 0 0
0 0 0

447

0 0
e
ij = 0 0 0 11 = C1111
11 = E
0 0 0

quedamos nicamente con la primera ecuacin de las ecuaciones de equilibrio:


11
= g
x1

= g
x

Notacin Ingenieril

(5.170)

Notar que el trmino g es uniforme en la barra, y segn la primera ecuacin de equilibrio


la tensin deber ser una funcin lineal en x . Y si consideramos que = E , tambin
requiere una funcin lineal en x , y como consecuencia el campo de desplazamientos u
u
.
x

debe ser una funcin cuadrtica en x ya que =

Luego, adoptaremos una funcin cuadrtica para la aproximacin del campo de


desplazamientos, i.e. u ( x) = a1 + a 2 x + a 3 x 2 , con lo cual necesitaremos de tres puntos para
poder definir dicha funcin. Adoptaremos los puntos x = 0 , x =

L
y x = L . Con eso,
2

podemos decir que:

L
L
L2
(2)
u ( x = ) = U = a1 + a2 + a3
2
2
4
( 3)
2
u ( x = L) = U = a1 + a2 L + a3 L

u ( x = 0) = U(1) = a1

U(1) 1 0
(2) L
U = 1
U(3) 2
1 L

0 a1
L2
a 2
4
L2 a3

Calculando la forma inversa de la ecuacin anterior obtenemos que:


L2
0
0 U (1)
a1
(2)
1
a 2 = 2 3L 4 L L U
a L 2
4 2 U (3)
3

a1 = U (1)

3 (1) 4 ( 2 ) 1 (3)

U + U U
a 2 =
4
L
L

2 (1) 4 ( 2 ) 2 (3)

a 3 = 2 U 2 U + 2 U
L
L
L

Y a su vez el campo de desplazamientos en funcin de U (1) , U( 2) , y U (3) resulta:


4
2
3 (1) 4 ( 2) 1 (3)
2

u = a1 + a 2 x + a 3 x 2 = U (1) +
U + U U a 2 x + 2 U (1) 2 U ( 2) + 2 U (3) a 3 x 2
L
L
L
L
4

y simplificando la ecuacin anterior obtenemos que:


3x 2 x 2
+ 2
u ( x) = 1
L
L

(1) 4 x 4 x 2
U +

L L2

( 2) x 2 x 2
U +

L + L2

( 3)
U

= N 1U (1) + N 2 U ( 2 ) + N 3 U (3)
= [N 1

N2

(5.171)

U (1)

N 3 ]U ( 2 ) = [N ] u ( e )
U (3)

{ }

donde N 1 ( x) , N 2 ( x) y N 3 ( x) son las funciones de forma, (ver NOTA 1).

Universidad de Castilla- La Mancha


Ciudad Real - Espaa

Draft

Por: Eduardo W. V. Chaves (2014)

MECNICA DEL MEDIO CONTINUO: PROBLEMAS RESUELTOS

448

El objetivo ahora es expresar la energa potencial total en funcin de los valores nodales
(grados de libertad) U (1) , U ( 2) y U (3) .
El trmino U ext (ver ecuacin (5.126)) puede ser reescrito como:
U ext =

r r
r r
r r
t * udS + ( b) udV = ( b) udV = gu ( x) Adx = gA u ( x)dx

3x 2 x 2
= gA 1 + 2
L
L

0
L

(1) 4 x 4 x 2
U +

L
L2

( 2) x 2 x 2
U +

L + L2

( 3)
U dx

Tras la integracin obtenemos que:


2 L ( 2 ) L ( 3)
L
U ext (U (1) , U ( 2 ) , U (3) ) = gA U (1) +
U + U
6
3
6

(5.172)

El trmino U int para este problema es anlogo al obtenido en la ecuacin (5.135), i.e.
U

int

1
1
1
u ( x)
u ( x)
=
E 2 dV =
E
AE
dV =
dx
2V
2 V x
20
x

(5.173)

donde
u 3 x 2 x 2 (1) 4 x 4 x 2 ( 2) x 2 x 2
=
+ 2
1 + 2 U + 2 U +
L
x x
L
L
L
L
L
3 4 x (1) 4 8 x ( 2) 1 4 x (3)
=
+ 2 U + 2 U + + 2 U
L L
L L
L L

( 3)
U

luego
2

int

EA 3 4 x (1) 4 8 x ( 2 ) 1 4 x (3)
=
+ 2 U + 2 U +
+
U dx

2 0 L
L
L L2
L L

Resolviendo analticamente la integral obtenemos que:


U int (U (1) , U ( 2 ) , U (3) ) =

2
2
EA (1) 2
+ 16U ( 2 ) + 7U (3) + 2U (1) U (3) 16U (1) U ( 2 ) 16U ( 2 ) U (3)
7U

6L

(5.174)
Con lo cual la energa potencial total queda:
(U (1) , U ( 2 ) , U (3) ) = U int U ext
=

2
2
EA (1) 2
7U
+ 16U ( 2 ) + 7U (3) + 2U (1) U (3) 16U (1) U ( 2 ) 16U ( 2 ) U (3)

6L
2 L ( 2 ) L ( 3)
L
U + U
gA U (1) +
3
6
6

Como estamos buscando el estado estacionario, lo siguiente debe cumplirse:


(U (1) , U ( 2 ) , U (3) )
=0

U (1)

(1)
(2)
( 3)
(U , U , U )
=0

U ( 2 )

(U (1) , U ( 2 ) , U (3) )
=0

U (3)
Universidad de Castilla- La Mancha
Ciudad Real - Espaa

gAL
EA
7U (1) 8U ( 2 ) + U (3)
=0
3L
6
2 gAL
EA
8U (1) + 16U ( 2 ) 8U (3)
=0
3L
3

gAL
EA (1)
U 8U ( 2) + 7U (3)
=0
3L
6

Draft

Por: Eduardo W. V. Chaves (2014)

5 INTRODUCCIN A: ECUACIONES CONSTITUTIVAS, PVCI, Y ESTRATEGIAS DE SOLUCIN

449

Reestructurando las ecuaciones anteriores en forma de matriz concluimos que:


(1)
1
7 8 1 U
( 2 ) gAL
EA

8 16 8 U =
4
3L
6
( 3)

1 8 7 U
1

[K ]{u }= {F }
(e )

(e)

(e)

(5.175)

Notar que [ K (e ) ] no tiene inversa, ya que det[ K ( e) ] = 0 . Para resolver el problema


tenemos que introducir las condiciones de contorno. Segn el planteamiento del problema
el desplazamiento en x = 0 es igual a cero, i.e. U (1) = 0 . Una forma de aplicar esta condicin
de contorno en el sistema discreto de ecuaciones (5.175) es eliminar fila y columna
asociadas al grado de libertad U (1) , i.e.:
(1)
1
7 8 1 U

EA
gAL

(2)
8 16 8 U =
4
3L
6
( 3)

1 8 7 U
1

0 U(1)
0
1 0
( 2 ) gAL
EA

0 16 8 U =
4
3L
6
( 3)

0 8 7 U
1

EA 16 8 U ( 2 ) gAL 4
=

3L 8 7 U (3)
6 1
1

EA 16 8 EA 16 8 U ( 2 ) gAL EA 16 8 4


8 7
8 7 (3) =
3L 8 7 1
3
3
6
L
L
U

U ( 2 ) gAL EA 16 8 4 gAL L 7 8 4

( 3) =


6 3L 8 7 1
6 16 EA 8 1 1
U
U ( 2 ) gL2 3
( 3) =

8 E 4
U

Al sustituir los valores nodales U (1) , U( 2) , y U (3) en el campo de desplazamiento (ver


ecuacin (5.171)) podemos obtener que:
3x 2 x 2
u = 1 + 2
L
L

(1) 4 x 4 x 2 ( 2 ) x 2 x 2 (3)
U +
U +

L
L + L2 U
L2

3 x 2 x 2 4 x 4 x 2 3 gL2 x 2 x 2 4 gL2
+

= 1 + 2 0 + 2
L + L2 8 E
L
L
8
E
L
L

Al simplificar la ecuacin anterior concluimos que:


u (Q ) =

g
2E

(2 Lx x 2 )

(5.176)

el cual coincide con la solucin exacta del problema planteado. Los campos de
deformacin y de tensin pueden ser obtenidos como sigue:
(Q ) =

u ( x) g
g
(2 Lx x 2 ) =
( L x)
=

x 2 E
x
E

(5.177)

y
( Q ) = ( Q ) E = g ( L x )

Si reemplazamos los valores nodales


potencial total obtenemos que:

Universidad de Castilla- La Mancha


Ciudad Real - Espaa

(5.178)

U (1) , U( 2 ) , y U (3) en la expresin de la energa

Draft

Por: Eduardo W. V. Chaves (2014)

MECNICA DEL MEDIO CONTINUO: PROBLEMAS RESUELTOS

450

(Q ) =

1 ( g ) 2 AL3
6
E

NOTA 1: La Funciones de Forma [N ( x )]


Notar que las funciones de forma son:
3x 2 x 2
N 1 ( x) = 1 + 2
L
L

N 3 ( x) =

x 2x 2
+ 2
L
L

N 1 ( x = 0) = 1

N1 ( x = 2 ) = 0
N ( x = L) = 0
1
N 3 ( x = 0) = 0

N 3 ( x = 2 ) = 0
N ( x = L) = 1
3

4x 4x 2
N 2 ( x) =
2
L
L

N 2 ( x = 0) = 0

L
N 2 ( x = 2 ) = 1
N ( x = L) = 0
2

Notar tambin que se cumple que N 1 ( x) + N 2 ( x) + N 3 ( x) = 1 . Si dibujamos estas funciones


dentro del dominio obtenemos que:
N2
N1

3x 2 x 2
N
1
=

+ 2
1
L
L

4x 4x2
2
N 2 (=
L
L

x 2x2
+ 2
N3 =
L
L

N3

1
x=0

x=

x=L

L
2

N 1 ( x) + N 2 ( x) + N 3 ( x) = 1

Figura 5.21: Funciones de forma (1D) funcin cuadrtica.


NOTA 2: Solucin analtica (exacta)
Empezamos por la ecuacin de equilibrio

= g , (ver ecuacin (5.170)), y al integrar


x

esta expresin podemos obtener que:

integrando
= g
= gx
x

= gx + C1

La constante de integracin puede ser obtenida a travs de la condicin en x = 0 . En esta


posicin ( x = 0 ) la fuerza total viene dada por F = gV = gAL , y la tensin puede ser
F gAL
=
= gL . Luego, la constante de integracin queda
A
A
definida por ( x = 0) = C1 = gL , y a su vez el campo de tensin:

calculada por ( x = 0) =

= gx + gL = g ( L x)

El campo de deformacin puede obtenido como sigue:


= E

Universidad de Castilla- La Mancha


Ciudad Real - Espaa

Draft

g ( L x)
=
E
E

Por: Eduardo W. V. Chaves (2014)

5 INTRODUCCIN A: ECUACIONES CONSTITUTIVAS, PVCI, Y ESTRATEGIAS DE SOLUCIN

Teniendo en cuenta la ecuacin cinemtica =


=

u
x

integrando

u
, y al integrar podemos obtener que:
x

g ( L x )
u = x =
x
E

451

u ( x) =

x2
+ C2
Lx

E
2

En x = 0 no hay desplazamiento, luego u ( x = 0) = 0 C 2 = 0 , con lo cual el campo de


desplazamiento, ver Figura 5.22, queda:
u ( x) =

x 2 g
=
Lx
(2 Lx x 2 )

E
2 2 E

Notar que, para problemas sencillos la solucin analtica se puede obtener fcilmente, y la
solucin exacta (analtica) sirve como indicador para saber si la aproximacin adoptada es la
adecuada. Notar que, la solucin analtica (ecuacin anterior) es la misma que la solucin
numrica (5.176) en la cual hemos utilizado la funcin cuadrtica para aproximar el campo
de desplazamientos.
( x = 0) =

u=

L + L

g
2E

( x = 0) = gL

gL
E

( 2 Lx x 2 )
=

g
E

( L x)

= g ( L x )

a) Desplazamiento vertical
(segn direccin x)

b) Deformacin

c) Tensin

Figura 5.22
NOTA 3: Notar que, si utilizamos una funcin cbica para aproximar el campo de
desplazamientos, la solucin debe ser la misma que la solucin exacta. A continuacin
verificamos este hecho:
Campo de desplazamientos (funcin cbica):
u ( x) = a1 + a2 x + a3 x 2 + a4 x 3 = a2 x + a3 x 2 + a4 x 3 .

Notar que, en x = 0 no hay desplazamientos, luego u ( x = 0) = a1 = 0 .


Luego, u ( x) = a2 x + a3 x 2 + a4 x 3

u
= a2 + 2a3 x + 3a4 x 2
x

Potencia interna:

Universidad de Castilla- La Mancha


Ciudad Real - Espaa

Draft

Por: Eduardo W. V. Chaves (2014)

MECNICA DEL MEDIO CONTINUO: PROBLEMAS RESUELTOS

452
L

int

AE u ( x)
AE
(a2 + 2a3 x + 3a4 x 2 ) 2 dx
=

dx =
2 0 x
2 0

AEL
(27 L4 a42 + 45 L3 a4 a3 + 30 L2 a2 a4 + 20 L2 a32 + 30 La2 a3 + 15a22 )
30

Potencia exterior:
L

1
1
1

U ext = gA u ( x)dx = gA (a 2 x + a 3 x 2 + a 4 x 3 )dx = gA L2 a 2 + L3 a 3 + L4 a 4


2
3
4

x
0

Energa potencial total:


(a2 , a3 , a4 ) = U int U ext
=

AEL
(27 L4 a42 + 45 L3 a4 a3 + 30 L2 a2 a4 + 20 L2 a32 + 30 La2 a3 + 15a22 )
30
1
1
1

gA L2 a2 + L3 a3 + L4 a4
3
4
2

Cmo estamos buscando el estado estacionario, se debe cumplir que:


(a2 , a3 , a4 )
=0

a2

(a , a , a )
2
3
4
=0

(a , a , a )
2
3
4

=0

a4

gAL2
EAL
=0
(30 L2 a4 + 30 La3 + 30a2 )
30
2

EAL
gAL3
=0
(45 L3 a4 + 40 L2 a3 + 30 La2 )
30
3

EAL
gAL4
=0
(54 L4 a4 + 45 L3 a3 + 30 L2 a2 )
30
4

Simplificando y reestructurando el conjunto de ecuaciones anteriores en forma matricial


obtenemos que:
30
E
30 L
30
2
30 L

30 L
40 L2
45 L3

30 L2 a 2
6L
a 2
2 gL
g 2 Solucin 1

3
45L a 3 =
a 3 =
g
4 L

2E
2
3L3
a 4
0
54 L4 a 4

Luego, el campo de desplazamientos ( u ( x) = a2 x + a3 x 2 + a4 x 3 ) queda definido por


u ( x) = a2 x + a3 x 2 + a4 x 3 =

1
g
(2 gLx gx 2 ) =
(2 Lx x 2 )
2E
2E

cuya solucin es la misma al emplear una aproximacin cuadrtica para el campo de


desplazamientos.
A continuacin, vamos adoptar una funcin lineal para aproximar el campo de
desplazamientos u ( x) = a2 x , luego
Potencia interna:
L

int

AE u ( x)
AE
AEL 2
(a2 ) 2 dx =
a2
=

dx =
2 0 x
2 0
2

Potencia externa:

U ext = gA u ( x)dx = gA (a 2 x)dx =


x

Universidad de Castilla- La Mancha


Ciudad Real - Espaa

1
gAL2 a 2
2

Draft

Por: Eduardo W. V. Chaves (2014)

5 INTRODUCCIN A: ECUACIONES CONSTITUTIVAS, PVCI, Y ESTRATEGIAS DE SOLUCIN

453

La energa potencial total:


(a 2 ) = U int U ext =

AEL 2 1
a 2 gAL2 a 2
2
2

Punto de equilibrio:
(a 2 )
=0
a 2

gAL2

EALa 2

=0

a2 =

( L) =

u ( L ) gL
=
2E
x

gL
2E

Luego
u ( L ) = a2 x =

gL
2E

( L ) = E =

1
gL
2

En este caso, no es una buena aproximacin al utilizar el caso lineal.


En este caso la energa potencial total queda ( L ) =

AEL 2 1
1 ( g ) 2 AL3
a2 gAL2 a2 =
.
2
2
8
E

NOTA 4:
A continuacin, vamos establecer la matriz de rigidez [ K (e ) ] del elemento barra
considerando una aproximacin lineal para el campo de desplazamiento. Adoptando la
funcin u ( x) = a1 + a 2 x se cumple que:

= a1 + a 2 L

u ( x = 0) = U (1) = a1
u ( x = L) = U ( 2 )

U (1) 1 0 a1 Inversa a1 1 L 0 U (1)


( 2) =
a = 1 1 ( 2)
U
2 L
U 1 L a 2

Con lo cual el campo de desplazamiento queda:


x
x
1
1

u ( x) = a1 + a 2 x = U (1) + U (1) + U ( 2 ) x = 1 U (1) + U ( 2 )


L
L
L

x x U (1)
= 1 ( 2)
L L U

{ }

= [N ] u ( e )

cuya expresin ya fue obtenida en el Ejemplo 5.19 (NOTA 4). Con lo cual la matriz de
rigidez ser la misma, diferenciando nicamente por el vector de fuerzas nodales.
Potencia interna:
L

U int =

2
AE u ( x)
AE 1 (1) 1 ( 2 )
AE (1) 2
U 2U (1)U ( 2 ) + U ( 2)

dx =
U + U dx =

2 0 x
2 0 L
L
2L

Potencia externa:
L

x
1
U ext = gA u ( x)dx = gA 1 U (1) + U ( 2 ) dx = gAL(U (1) + U ( 2 ) )
L
L
2

x
0

La energa potencial total:


(U(1) , U( 2 ) ) = U int U ext =

2
AE (1) 2
1
U 2U(1)U( 2 ) + U( 2 ) gAL(U(1) + U ( 2 ) )

2L
2

Punto de equilibrio:

Universidad de Castilla- La Mancha


Ciudad Real - Espaa

Draft

Por: Eduardo W. V. Chaves (2014)

MECNICA DEL MEDIO CONTINUO: PROBLEMAS RESUELTOS

454

(U (1) , U ( 2 ) )
(1)

U
(U (1) , U ( 2 ) )
U

(2)

=0

1
1
AE
AE (1)
( 2U (1) 2U ( 2 ) ) gAL = 0
(U U ( 2 ) ) = gAL
2L
2
2
L

=0

1
1
AE
AE ( 2 )
( 2U ( 2 ) 2U (1) ) gAL = 0
(U U (1) ) = gAL
2L
2
2
L

Reestructurando en forma matricial obtenemos que:


1
EA 1 1 U (1) 1
( 2 ) = gAL

L 1 1 U 2
1

[K ]{u }= {F }
(e)

(e)

(e )

Como hemos visto en la Nota anterior, al utilizar la aproximacin lineal para el campo de
desplazamiento hay un error. Qu pasara si dividiramos el dominio en subdominios y
aplicar la aproximacin lineal para cada subdominio (elemento finito)?. Para establecer el
campo de desplazamiento vamos adoptar un elemento genrico, donde el inicio del
elemento es x (i ) y el final es x ( f ) , ver Figura 5.23 (a).
U(1) = 0
x (i )

(EA)

U(i )
e

(1)

x( f )

L(1) =

L
2

L( 2 ) =

L
2

U( 2)

U( f )

(EA) ( 2)

2
U ( 3)

a) Elemento genrico

b) Discretizacin en 2 elementos
Figura 5.23

Teniendo en cuenta la aproximacin lineal u ( x) = a1 + a 2 x se cumple que:


U (i ) 1 x (i ) a1
u ( x = x (i ) ) = U (i ) = a1 + a 2 x (i )

(f) =

( f )
u ( x = x ( f ) ) = U ( f ) = a1 + a 2 x ( f )
U 1 x a 2
x ( f ) x (i ) U (i )
a
1
Inversa

1 = ( f )

(i )
1 U ( f )
a 2 ( x x ) 1

Luego el campo de desplazamientos queda:


u ( x) = a1 + a 2 x

u ( x) =

1
(x

(f)

(i )

x )

( x ( f ) x)U (i ) +

1
(x

(f)

(i )

x )

( x x (i ) )U (i )

y su derivada:
u ( x)
1
1
= (f)
U (i ) + ( f )
U (i )
(i )
x
(x x )
( x x (i ) )

Universidad de Castilla- La Mancha


Ciudad Real - Espaa

Draft

Por: Eduardo W. V. Chaves (2014)

5 INTRODUCCIN A: ECUACIONES CONSTITUTIVAS, PVCI, Y ESTRATEGIAS DE SOLUCIN

455

Vamos dividir el dominio en 2 sub-dominios (2 elementos finitos), ver Figura 5.23 (b),
donde se cumple que:
Elemento e = 1 :

L
x( f ) =

2
U (i ) = U (1)

U ( f ) = U ( 2)
x (i ) = 0

(1)
2 x (1)
2 x ( 2)
u = 1 L U + 1 L U

(1)
u = 2 U (1) + 2 U ( 2)
x
L
L

Elemento e = 2 :

x( f )

(i )
( 2)
U =U

U ( f ) = U (3)
L
2
=L

x (i ) =

( 2)
2 x ( 2) 2 x
( 3)
u = 2 L U + L 1U

( 2)
u = 2 U ( 2 ) + 2 U (3)
x
L
L

Potencia interna:
U

int

L
2

u (1)
1
1
u ( x)
=
( AE ) (1)
AE
dx =
20
20
x
x

L
(2)

1
( 2 ) u
dx +

(
)
AE

x
2L

dx

( AE )
L

(1)

U (1) 2 2U (1) U ( 2 ) + U ( 2 ) 2 + ( AE )

(2)

U ( 2 ) 2 2U ( 2 ) U (3) + U (3) 2

Potencia externa
L
2

U ext = g Au ( x)dx = g A(1)u (1) dx + g A( 2)u ( 2 ) dx


x

L
2

1
1
1
gA(1) LU(1) + gL( A(1) + A( 2) )U( 2) + gA( 2) LU(3)
4
4
4

La energa potencial total


(U (1) , U ( 2 ) , U (3) ) = U int U ext
=

2
2
( AE ) (1) (1) 2
( AE ) ( 2 ) ( 2) 2
2U (1) U ( 2 ) + U ( 2 ) +
2U ( 2 ) U (3) + U (3)
U
U

L
L
1
1
1

gA (1) LU (1) + gL( A (1) + A ( 2 ) )U ( 2 ) + gA ( 2) LU (3)


4
4
4

Punto de equilibrio
(U ( a ) )
U (1)
(U ( a ) )
U ( 2 )
(U ( a ) )
U (3)

=0

2( AE ) (1) (1)
1
(U U ( 2) ) gLA (1) = 0
L
4

=0

2( AE ) ( 2 ) ( 2 )
2( AE ) ( 2) ( 2 )
1
(U U (1) ) +
(U U (3) ) gL( A (1) + A ( 2 ) ) = 0
L
L
4

=0

2( AE ) ( 2 ) (3)
1
(U U ( 2 ) ) gLA ( 2 ) = 0
4
L

Universidad de Castilla- La Mancha


Ciudad Real - Espaa

Draft

Por: Eduardo W. V. Chaves (2014)

MECNICA DEL MEDIO CONTINUO: PROBLEMAS RESUELTOS

456

Reestructurando en forma matricial obtenemos que:

2
L

(1)

[ A (1) ]
U 1

(1)
(2)
(2)
(
2
)
(
1
)
(
2
)
+ ( AE ) ] [( AE ) ] U = gL [ A + A ]
[( AE )
4

[ A ( 2 ) ]
[( AE ) ( 2 ) ]
[( AE ) ( 2 ) ] U (3)

[( AE ) (1) ]

[( AE ) (1) ]

[( AE ) (1) ]

[K ]{u} = {F }
Notar que la matriz anterior podra haber sido obtenida directamente, si tenemos en cuenta
la matriz de rigidez del elemento y el vector de fuerzas nodales del elemento e :

[k ] = ( EAL )
(e)

(e)

(e)

Elemento e = 1 , ( L(1) =

[k ] = 2( EAL )
(1)

(1)

[k ] = 2(EAL )
( 2)

(2)

{f } = 12 g (LA)
(e)

(e)

L
):
2

1 1
1 1

Elemento e = 2 , ( L( 2) =

1 1
1 1

{f } = 14 gLA

{f }= 14 gLA

(1)

(1) 1

{u } = U

{u }= U

(1)

(1)

( 2)

L
):
2

1 1
1 1

( 2)

( 2 ) 1

(2)

( 3)
U
( 2)

A continuacin aadimos la contribucin de cada elemento en la matriz de rigidez global


de la estructura [K ] y en el vector de fuerzas globales. Este procedimiento se denomina de
ensamblaje.
Considerando que ( EA) (1) = ( EA) ( 2) = EA (los elementos finitos tienen las mismas
propiedades) el sistema discreto queda:
(1)
1
1 1 0 U

2 AE
1

(2)
1 2 1 U = gLA2

4
L
1
0 1 1 U (3)

Aplicando las condiciones de contorno y resolviendo el sistema obtenemos los


desplazamientos nodales:
(1)
0
1 0 0 U

2 AE
1

( 2)
0 2 1 U = gLA2

4
L
1
0 1 1 U(3)

U(1)
0
( 2) gL2
U =
3
U(3) 8E 4

Solucin

Que coincide con lo solucin exacta.


El procedimiento que acabamos de desarrollar es la base de los Mtodos de los Elementos
Finitos que bsicamente consiste en:
Adoptar una aproximacin para el campo de la incgnita;
Dividir (discretizar) el dominio en sub-dominios (elementos finitos);
Establecer la matriz de rigidez de cada sub-dominio y del vector de fuerzas nodales;

Universidad de Castilla- La Mancha


Ciudad Real - Espaa

Draft

Por: Eduardo W. V. Chaves (2014)

5 INTRODUCCIN A: ECUACIONES CONSTITUTIVAS, PVCI, Y ESTRATEGIAS DE SOLUCIN

457

Ensamblar en la matriz de rigidez global de la estructura;


Aplicar las condiciones de contorno;
Resolver el sistema.
Para mayores detalles sobre el Mtodo de los Elementos Finitos ver Zienkiewicz & Taylor
(1994).
Ejemplo 5.21
Demostrar que:

r r
r&
r
r
r r
&r&( Xr , t ) u& dV = P : r u& dV
t * ( X , t ) u dS 0 + 0 b( X , t ) u
0
0
X

V0

(5.179)

V0

donde u es un campo virtual de desplazamientos, y P es el primer tensor de tensiones de


Piola-Kirchhoff.
configuracin de
referencia

configuracin
actual

S 0

V0

B0

r r
r
t * ( X , t ) = t *0

dV0

r r
u( X , t )

B
r r
u( x , t )

r r
r
0 b( X , t ) = 0 b 0

r r
t * ( x, t )

dV

r r

b( x , t )
Figura 5.24

Solucin:
r

r r

Aunque las variables t * ( X , t ) y b( X , t ) no sean variables intrnsecas de la configuracin de


referencia como las variables 0 , S 0 , V0 entre otras, por simplicidad vamos denotar
r r
r
r r
r
t * ( X , t ) = t *0 y b( X , t ) = b 0 y para el campo de desplazamiento Lagrangeano.

Recordemos tambin que (ver Captulo 2 en Chaves (2007)):

r
r
r
r
xi ( X , t )
xi ( X , t ) u& i ( X , t ) &
D
&
Fij Fij =
=
=
= ui, J ( X , t )
Dt
t X j
X j 142
t 4
X j
3
x&i

r r
F& = l F = Xr u& ( X , t )

r r
= F& F 1 = Xr u& ( X , t ) F 1
r r
r r
F& 1 = F 1 l = F 1 Xr u& ( X , t ) F 1 = F 1 xr u& ( x , t )
l

Teniendo en cuenta las relaciones anteriores, tambin se cumple para un campo virtual de
desplazamiento:
Universidad de Castilla- La Mancha
Ciudad Real - Espaa

Draft

Por: Eduardo W. V. Chaves (2014)

MECNICA DEL MEDIO CONTINUO: PROBLEMAS RESUELTOS

458

r& r
r& r
r& r
F& = Xr u ( X , t ) y F& 1 = F 1 Xr u ( X , t ) F 1 = F 1 xr u ( x , t )

Con lo cual podemos decir que:

r
P : F& dV0 = PiJ F&iJ dV0 = PiJ u& i , J ( X , t ) dV0

V0

V0

(PiJ u& i ) , J = PiJ , J u& i + PiJ u& i , J

V0

PiJ u& i , J = (PiJ u& i ) , J PiJ , J u& i

con lo cual:

r
r
r
P : F& dV0 = PiJ u& i , J ( X , t ) dV0 = (PiJ u& i ( X , t )) , J PiJ , J u& i ( X , t ) dV0

r
r
P : F& dV0 = (PiJ u& i ( X , t )) , J dV0 PiJ , J u& i ( X , t ) dV0

r
r
P : F& dV0 = PiJ u& i ( X , t )n J dS 0 PiJ , J u& i ( X , t ) dV0

V0

V0

V0

V0

V0

V0

V0

S0

V0

donde hemos aplicado el teorema de la divergencia. En notacin tensorial queda:

r
r
P : F& dV0 = (P n ) u& i ( X , t ) dS 0 ( Xr P) u& i ( X , t ) dV0

V0

S0

V0

Recordar que la ecuacin de movimiento en la configuracin de referencia viene dada por:


r
&r&( Xr , t )
Xr P + 0 b 0 = 0 u

r
&r&( Xr , t )
Xr P = 0 b 0 u

r& r
F& = Xr u ( X , t )

y teniendo en cuenta que donde por definicin se cumplen que


r
t *0 = P n obtenemos:

r
r
P : F& dV0 = (P n ) u& i ( X , t ) dS 0 ( Xr P ) u& i ( X , t ) dV0

r
r& r
r &r& r
&r&( Xr , t ) u& ( Xr , t ) dV
P : Xr u ( X , t ) dV0 = t *0 u
( X , t ) dS 0 + 0 b 0 u
i
0

V0

S0

V0

V0

S0

V0

Recordatorio: Recordemos del captulo 4 en Chaves (2007) que la potencial tensional


puede ser expresada de distintas formas, a saber:

w int (t ) = P : F& dV0 = S : E& dV0 = P : F& dV0 =


V0

y D=l

sym

V0

V0

1
1
S : C& dV0 =
P : F& dV
2V
J
V

P : F& dV = : D dV = {
J : D dV0 = : D dV0
V

V0

V0

r r sym
= xr u& ( x , t)
(ver Ejemplo 2.37)

NOTA 1: Recordar que ni P ni F& estn en ninguna configuracin, pero el escalar P : F&
s que est en la configuracin de referencia.
NOTA 2: Teniendo en cuenta lo anterior. El trabajo interno virtual total tambin se puede
expresar como:

Universidad de Castilla- La Mancha


Ciudad Real - Espaa

Draft

Por: Eduardo W. V. Chaves (2014)

5 INTRODUCCIN A: ECUACIONES CONSTITUTIVAS, PVCI, Y ESTRATEGIAS DE SOLUCIN

459

sym
r& r
r& r
r& r
: D dV = : xr u ( x , t ) dV = : xr u ( x , t ) dV = P : F& dV0 = P : Xr u ( X , t ) dV0

V
V
V
V
V

r& r

Note que, debido a la simetra de se cumple que : xr u ( x , t )

sym

r& r
= : xr u ( x , t ) .

NOTA 3: La ecuacin (5.179) tambin es vlida para una variacin del campo virtual:

r r
r
r
r
r* r
&r&( Xr , t ) u dV = P : r u dV
( X , t ) u dS 0 + 0 b( X , t ) u
0
0
X

V0

V0

(5.180)

Ejemplo 5.22
a) Dado un tensor de segundo orden simtrico A = A sym demostrar que podemos hacer la
A = AP + AS
donde
AP = PP : A ,
AS = PS : A ,
descomposicin
aditiva
P P = (b b ) (b b ) , y P S = I sym (b b ) (b b ) , donde b es un versor segn una
dada direccin, y I sym es el tensor identidad de cuarto orden simtrico. b) Demostrar que la
ecuacin constitutiva de tensin = C e : puede ser escrita como:
P C PP
S = SP
C

C PS
:
C SS

PP
ijP Cijkl
S = SP
ij Cijkl

PS
kl
Cijkl

SS
Cijkl kl

(5.181)

donde
C PP = P P : C e : P P

PP
P
P
Cijkl
Cepqst Pstkl
= Pijpq

C PS = P P : C e : P S

PS
P
S
Cijkl
Cepqst Pstkl
= Pijpq

C SP = P S : C e : P P

SP
S
P
Cijkl
Cepqst Pstkl
= Pijpq

C SS = P S : C e : P S

(5.182)

SS
S
S
Cijkl
Cepqst Pstkl
= Pijpq

Solucin:
a) Considerando la base Cartesiana podemos decir que:
A = A ij (e i e j ) = A i1 (e i e 1 ) + A i 2 (e i e 2 ) + A i 3 (e i e 3 )

= A 11 (e 1 e 1 ) + A 21 (e 2 e 1 ) + A 31 (e 3 e 1 ) + A 12 (e 1 e 2 ) + A 22 (e 2 e 2 ) + A 32 (e 3 e 2 )
+ A 13 (e 1 e 3 ) + A 23 (e 2 e 3 ) + A 33 (e 3 e 3 )

Componentes:
A 11
A ij = A 21
A 31

A12
A 22
A 32

A13 0
A 23 = A 21
A 33 A 31

A 12
A 22
A 32

A13 A11
A 23 + 0
A 33 0

0 0
0 0 = A ij + A ij
0 0

Notar que la componente normal A 11 = A (Ne1 ) (segn direccin e 1 ) puede ser escrita como
A 11 = A : (e 1 e 1 ) = (e 1 e 1 ) : A , con eso el tensor A = A 11 (e 1 e 1 ) puede ser reescrito

como:
A = (e 1 e 1 ) A 11 (e 1 e 1 ) A 11 = (e 1 e 1 ) (e 1 e 1 ) : A

Universidad de Castilla- La Mancha


Ciudad Real - Espaa

Draft

Por: Eduardo W. V. Chaves (2014)

MECNICA DEL MEDIO CONTINUO: PROBLEMAS RESUELTOS

460

luego
A = A A = A (e 1 e 1 ) (e 1 e 1 ) : A = I sym : A (e 1 e 1 ) (e 1 e 1 ) : A

= I sym (e 1 e 1 ) (e 1 e 1 ) : A

Aunque la demostracin ha sido realizada utilizando el versor e 1 , lo anterior es vlido para


cualquier vector unitario ( b ), i.e.:
A P = (b b ) (b b ) : A = P P : A
A S = I sym (b b ) (b b ) : A = P S : A

Notar que A (Nb ) = (b b ) : A = b A b es la componente normal segn la direccin b , i.e.

paralelo a b . Es interesante revisar el Ejemplo 1.116.


b) Aplicamos la definicin anterior para obtener:
= P + S = PP : + PS :
= P + S = PP : + PS :

con lo cual, y considerando = C e : , podemos obtener que:


P = P P : = P P : C e : = P P : C e : ( P + S ) = P P : C e : P + P P : C e : S
= P P : Ce : P P : + P P : Ce : P S :
S = P S : = P S : C e : = P S : C e : ( P + S ) = P S : C e : P + P S : C e : S
= P S : Ce : P P : + P S : Ce : P S :

luego
P P P : C e
S= S
e
P : C

P P : C e P
:
P S : C e S

o
P P P : C e : P P
S= S
e
P
P : C : P

Universidad de Castilla- La Mancha


Ciudad Real - Espaa

Draft

P P : C e : P S
:
P S : C e : P S

Por: Eduardo W. V. Chaves (2014)

6 Elasticidad Lineal
6.1 Elasticidad Tridimensional
Ejemplo 6.1
El cilindro indefinido de la Figura 6.1 constituido por un material elstico lineal e istropo,
est sometido al siguiente estado de deformacin (en coordenadas cilndricas):
err = e = a sin
a cos
2
ezz = ez = erz = 0
er =

(6.1)

donde eij son las componentes del Tensor de deformacin de Almansi.


r

Calcular el vector traccin t en el contorno, en coordenadas cilndricas.


Hiptesis: Rgimen de pequeas deformaciones. Considerar las constantes de Lam , .
x3

e z

e
e r

x2

r
x1

Figura 6.1.

Universidad de Castilla- La Mancha


Ciudad Real - Espaa

Draft

Por: Eduardo W. V. Chaves (2014)

MECNICA DEL MEDIO CONTINUO: PROBLEMAS RESUELTOS

462

Solucin:
Rgimen de pequeas deformaciones: e E
rr
( r , , z ) = r
rz

r

z

= Tr ( )1 + 2

a sin
rz
a cos
z =
2
zz 0

a cos
2
a sin
0

Tr ( ) = 2a sin

donde

(6.2)

(6.3)

luego,

a sin
1 0 0
a cos
= 2a sin 0 1 0 + 2
2
0 0 1
0

a cos
2
a sin
0

0
a cos
2a sin + 2 a sin

0
a cos
2a sin + 2 a sin
(r,r, =

0
0
2a sin
r

(6.4)

(6.5)

Teniendo en cuenta el vector traccin t (n) = n , donde n = (1,0,0) , podemos obtener


que:
t 1(n ) 2a sin + 2 a sin
(n )

a cos
t 2 =

t (n )

0
3

(6.6)

Ejemplo 6.2
El paraleleppedo de la Figura 6.2 se deforma de la manera indicada por las lneas de trazo.
Los desplazamientos vienen dados por las siguientes relaciones:
u = C1 xyz

v = C2 xyz

w = C3 xyz

(6.7)

a) Determinar el estado de deformacin en el punto E , cuando las coordenadas del punto


E en el cuerpo deformado son E (1,503; 1,001; 1,997) ;
b) Determinar la deformacin normal en E en la direccin de la lnea EA ;
c) Calcular la distorsin angular en E del ngulo recto formado por las lneas EA y EF .
d) Determinar el incremento de volumen y la deformacin volumtrica media.
Solucin:
a) El estado de deformacin en funcin de los desplazamientos es:
ij =

Universidad de Castilla- La Mancha


Ciudad Real - Espaa

1 u i u j
+
2 x j x i

Draft

(6.8)

Por: Eduardo W. V. Chaves (2014)

6 ELASTICIDAD LINEAL

463

Explcitamente en notacin ingenieril:

x
1
ij = xy
2
1
2 xz

1
xy
2
y
1
yz
2

u
1
xz
x

1 v u
1
yz = +
2 2 x y
z 1 w u
+

2 x z

1 v u
+
2 x y
v
y
1 w v

+
2 y z

1 w u
+

2 x z
1 v

+
2 y z

(6.9)

z
F F

G G

D D

2m
E
C C

O O

y
1,5m

A A

B B

1m

Figura 6.2
Para determinar el estado de deformacin en cualquier punto necesitamos determinar el
campo de desplazamientos.
Determinacin de las constantes:
Reemplazando los valores dados para el punto E (1,5; 1,0; 2,0) , resulta:
u ( E ) = X 1( E ) x1( E ) = 1,503 1,5 = C1 (1,5)(1,0)(2,0) C1 = 0,001
0,001
3
= 1,997 2,0 = C 3 (1,5)(1,0)(2,0) C 3 = 0,001

v ( E ) = X 2( E ) x 2( E ) = 1,001 1,0 = C 2 (1,5)(1,0)(2,0) C 2 =


w ( E ) = X 3( E ) x 3( E )

(6.10)

Con lo cual el Campo de Desplazamiento queda:


Notacin Ingenieril

Notacin Cientfica

u = 0,001 xyz
0,001
v=
xyz
3
w = 0,001 xyz

Universidad de Castilla- La Mancha


Ciudad Real - Espaa

u1 = 0,001 X 1 X 2 X 3
0,001
X1 X 2 X 3
3
u 3 = 0,001 X 1 X 2 X 3

u2 =

Draft

(6.11)

Por: Eduardo W. V. Chaves (2014)

MECNICA DEL MEDIO CONTINUO: PROBLEMAS RESUELTOS

464

u
= 0,001 yz = 0,002 = 11
x
v 0,001
y =
=
xz = 0,001 = 22
3
y
w
= 0,001xy = 0,0015 = 33
z =
z
v u 0,001
0,011
=
= 2 12
xy = +
yz + 0,001xz =
3
3
x y
x =

w u
xz =
+
= 0,001yz + 0,001xy = 0,0005 = 213
x z
w v
0,001
yz =
+ = 0,001xz +
xy = 0,0025 = 2 23
3
y z

Campo de deformacin:

yz

1 yz

ij = 0,001 + xz
2 3

1
(xy yz )
2

1 yz

+ xz
2 3

xz
3
1 xy

xz
2 3

1
(xy yz )
2

1 xy

xz
2 3

xy

Estado de deformacin en el punto E ( x = 1,5; y = 1,0; z = 2,0) :

ij

x
1
= xy
2
1
2 xz

1
xy
2
y
1
yz
2

0,011
1

0,00025

xz 0,002
6
2

1
0,011

yz =
0
,
001
0
,
00125

z 0,00025 0,00125 0,0015


(6.12)

b) Componente normal segn direccin M :

M = M M

indicial

M = ij M i M j

(6.13)

Expandiendo la expresin anterior y considerando la simetra del tensor de deformacin:


M = 11 M 12 + 22 M 22 + 33 M 32 + 2 12 M 1 M 2 + 213 M 1 M 3 + 2 23 M 2 M 3

(6.14)

en notacin Ingenieril:
M = x M 12 + y M 22 + z M 32 + xy M 1 M 2 + xz M 1 M 3 + yz M 2 M 3

(6.15)

El versor M i viene definido por los cosenos directores de la direccin de la lnea EA :


M 1 = 0 ;

1
M 2 =
;
5

2
M 3 =
5

(6.16)

Reemplazando los correspondientes valores en la ecuacin (6.15), obtenemos que:

Universidad de Castilla- La Mancha


Ciudad Real - Espaa

Draft

Por: Eduardo W. V. Chaves (2014)

6 ELASTICIDAD LINEAL

465

M = y M 22 + z M 32 + yz M 2 M 3
2
4
1
M = 0,001 + (0,0015) + (0,0025)
5
5
5
3
M = 2 10

(6.17)

c) Para el caso de pequeas deformaciones, la distorsin del ngulo recto formado por las
lneas EA y EF , ( = 90 ) ser:
M N =

N
1 2M
1
M N =
2
2
sin

= M N

componente

M N = ij M i N j (6.18)

Para mayores detalles de la expresin anterior ver Captulo 2- Cinemtica del Continuo
(pequeas deformaciones). Expandiendo la expresin anterior y considerando la simetra
del tensor de deformacin:

M N = 11 M 1 N 1 + 22 M 2 N 2 + 33 M 3 N 3 + 12 M 1 N 2 + M 2 N 1 +

+ 13 M 1 N 3 + M 3 N 1 + 23 M 2 N 3 + M 3 N 2

(6.19)

o en notacin ingenieril:
M N
2

= x M 1 N 1 + y M 2 N 2 + z M 3 N 3 +

xy
2

(M

N 2 + M 2 N 1 +

yz

M 2 N 3 + M 3 N 2
+ xz M 1 N 3 + M 3 N 1 +
2
2

(6.20)

y considerando los versores segn las direcciones EA y EF respectivamente:

M i = 0

1
5

N i = [ 1 0 0]

(6.21)

podemos obtener que:


M N
2
M N

1
2
0,011
+ (0,00025)( 1)

= 12 M 2 N 1 + 13 M 3 N 1 =
(1)
6
5
5

(6.22)

= 5,96284793998 10 4

2
M N = 1,1925696 10 3

Solucin Alternativa
Podemos crear una base ortonormal asociada a los versores M y N a travs del producto
vectorial P = M N . Las componentes del versor P vienen das por:
e 1
N = 0
P = M
1

e 2
1

e 3
2

5
0

5
0

2
1
e2
e3
5
5

Pi = 0

2
5

(6.23)

Luego, la matriz de transformacin del sistema X 1 X 2 X 3 para la base M , N , P viene


dada por:

Universidad de Castilla- La Mancha


Ciudad Real - Espaa

Draft

Por: Eduardo W. V. Chaves (2014)

MECNICA DEL MEDIO CONTINUO: PROBLEMAS RESUELTOS

466

M 1

A = a ij = N 1
P
1

M 2
N 2
P
2

M 3 0

N 3 = 1
P3 0

1
5
0
2
5

5
0
1
5

(6.24)

Aplicando la ley de transformacin de las componentes de un tensor de segundo orden, es


decir, ij = a ik a jl kl o en forma matricial = A A T :
1

= 1
0

5
0
2
5

0,011
0,00025

2 0,002
0
6

5 0,011

0
0,001
0,00125 1

1
0,00025 0,00125 0,0015 0

5

1
5
0
2
5

5
0
1
5

(6.25)

Resultando:
M N =

2 10 3

ij = 5,96284794 10 4

4
2,5 10

M N
2

5,96284794 10 4
2 10 3
1,75158658 10 3

3
1,75158658 10

1,5 10 3

2,5 10 4

(6.26)

NOTA: Observar que no se trata de un caso de deformacin homognea, es decir, las


aristas que en la configuracin inicial son rectas, en la configuracin deformada no ms
sern rectas. Para obtener los versores deformados tenemos que aplicar la transformacin
lineal m = F M y n = F N , donde F es el gradiente de deformacin.
d) La deformacin volumtrica viene definida por

V =

( dV )
, donde dV
dV

es el

diferencial de volumen.
Caso de pequeas deformaciones:
V =

(dV )
= x + y + z
dV

(dV ) = ( x + y + z )dV

(6.27)

Integrando en el volumen podemos obtener el incremento de volumen:


V =

2, 0

+ y + z dV = 0,001

1, 5

yz +

z =0 y =0 x =0

xz

xy dxdydz = 1,125 10 3 m 3
3

(6.28)

Luego:
V =

( dV ) 1,125 10 3
= 0,375 10 3
=
dV
1,5 1,0 2,0

Universidad de Castilla- La Mancha


Ciudad Real - Espaa

Draft

(6.29)

Por: Eduardo W. V. Chaves (2014)

6 ELASTICIDAD LINEAL

467

Ejemplo 6.3
El estado de tensiones en un punto de una estructura que est constituida por un material
elstico, lineal e istropo, viene dado por:
6 2 0
ij = 2 3 0 MPa
0 0 0

a) Determinar las componentes del tensor de deformacin ingenieril. Considrese que el


mdulo de elasticidad longitudinal ( E = 207GPa ) y el mdulo de elasticidad transversal
( G = 80GPa );
b) Si un cubo de 5cm de lado est sometido a este estado tensional, cul ser el cambio de
volumen que experimenta el cubo?
Solucin:
Las deformaciones pueden ser obtenidas partiendo de las siguientes relaciones:

)]

1
x y + z = 3,333 10 5
E
1
y = y ( x + z ) = 2,318 10 5
E
1
z = z x + y = 4,348 10 6
E
1
xy = xy = 2,5 10 5
G
1
xy = xy = 0
G
1
xy = xy = 0
G
x =

)]

(6.30)

donde el coeficiente de Poisson puede ser obtenido partiendo de la relacin:


G=

E
2(1 + )

207
E
1 =
1 0,29375
2G
160

Luego:
0
33,24 12,5

0 10 6
ij = 12,5 23,01
0
0
4,257

Solucin Alternativa
1

En el captulo 1 hemos demostrado que C e =

(1 + )

I 1 1 , con eso obtenemos


E
E

que:
1
(1 + )
(1 + )

(1 + )

= Ce : =
I 1 1 : =
I : 1 1 : =
Tr ( )1
E
E
E
E
E
E

En componentes:

Universidad de Castilla- La Mancha


Ciudad Real - Espaa

Draft

Por: Eduardo W. V. Chaves (2014)

MECNICA DEL MEDIO CONTINUO: PROBLEMAS RESUELTOS

468

11
(1 + )
12
ij =
E
13

13
1 0 0

23 Tr ( ) 0 1 0
E
0 0 1
23 33
0
6 2 0
1 0 0 33,24 12,5

6
6
= 6,251 10 2 3 0 4,2609 10 0 1 0 = 12,5 23,01
0 10 6
0 0 0
0 0 1 0
0
4,257

donde tenemos que

12
22

(1 + )
1
6
= 6,25 10 6
, Tr ( ) = 4,25725 10 .
E
MPa E

En el rgimen de pequeas deformaciones la deformacin volumtrica (lineal) es igual a la


traza del tensor de deformacin:
DVL V = I = (33,24 23,01 4,257 ) 10 6 = 5,973 10 6

Luego, la variacin de volumen queda:


V = V V0 = 5,973 10 6 (5 5 5) = 7,466 10 4 cm 3

Ejemplo 6.4
Un paraleleppedo de dimensiones a = 3cm , b = 3cm , c = 4cm , constituido por un material
homogneo elstico lineal e istropo se aloja en una cavidad de la misma forma y
dimensiones, ver Figura 6.3, cuyas paredes son de un material lo suficientemente rgido
para poderlo suponer indeformable. Sobre la abertura de la cavidad de dimensiones a b y
a travs de una placa rgida de peso y rozamiento despreciables se aplica,
perpendicularmente a ella, una fuerza F = 200 N que comprime al bloque elstico, (ver
Ortiz Berrocal (1985)). Si el coeficiente de Poisson es = 0,3 y el mdulo de elasticidad
E = 2 10 4 N / cm 2 , calcular:
a) Las fuerzas laterales ejercidas por las paredes de la cavidad sobre el paraleleppedo;
b) La variacin de altura experimentada por el mismo.
F
z
a
y
c
x
b

Figura 6.3

Universidad de Castilla- La Mancha


Ciudad Real - Espaa

Draft

Por: Eduardo W. V. Chaves (2014)

6 ELASTICIDAD LINEAL

469

Solucin:
En cualquier punto del cuerpo elstico habrn solo tensiones normales, x , y y z . La
tensin z viene dada por:
z =

200 200 200 N


=
=
ab
3 3
9 cm 2

(6.31)

Observemos que debido a la simetra, las tensiones x y y sern iguales, luego:

)]

1
x y + z = 0
E
x ( x + z ) = 0
x = y =

x =

1
[ x ( x + z )] = 0
E

(6.32)

z
(1 )

obteniendo as:
x =

z
0,3 200
200 N
=

=
(1 ) (1 0,3) 9
21 cm 2

(6.33)

La fuerza que ejerce la pared sobre el cuerpo elstico viene dado por:
200
3 4 = 114,28 N
21
200
Fx = x b c =
3 4 = 114,28 N
21
Fy = y a c =

La deformacin z viene dada por:


z =

)]

1
1
1
z x + y = [ z 2 x ] =
E
E
2 104

200
200
4
9 + 2 0,3 21 = 8,25 10

Luego, la variacin de altura viene dada por:


c = z c = 8,25 10 4 4 = 0,0033cm

(6.34)

Ejemplo 6.5
Bajo la restriccin de la teora de pequeas deformaciones, para un campo de
desplazamientos dado por:
r
u = ( x1 x3 ) 2 10 3 e 1 + ( x 2 + x3 ) 2 10 3 e 2 x1 x 2 10 3 e 3

a) Determinar el tensor de deformacin infinitesimal, el tensor de rotacin infinitesimal en


el punto P (0,2 1) ;
Solucin: Gradiente de los desplazamientos
u1

x1
u i u 2
=
x j x1

u 3
x1

u1
x 2
u 2
x 2
u 3
x 2

u1

x3 2( x x )
0
2( x1 x3 )
1
3
u 2
0
2( x 2 + x 3 ) 2( x 2 + x3 ) 10 3
=

x3

0
x1
u 3 x 2
x3

Universidad de Castilla- La Mancha


Ciudad Real - Espaa

Draft

Por: Eduardo W. V. Chaves (2014)

MECNICA DEL MEDIO CONTINUO: PROBLEMAS RESUELTOS

470

Aplicado al punto P(0,2 1)


u i
x j

2( x1 x 3 ) 2 0 2
0
2( x1 x 3 )

=
0
2( x 2 + x3 ) 2( x 2 + x 3 ) = 0 2 2 10 3
x 2
2 0 0
x1
0

Adems teniendo en cuenta que:


u i
= ij + ij
x j

donde:
Tensor de deformacin infinitesimal
u j
1 u
ij = i +
2 x j xi

Tensor spin infinitesimal

2 0 2

u
= 0 2 1 10 3 ij = 1 u i j


2 x j x i
2 1 0

0 0 0

(6.35)
= 0 0 1 10 3


0 1 0

Ejemplo 6.6
Bajo la restriccin de la teora de pequeas deformaciones, para un campo de
desplazamientos dado por:
r
u = a ( x12 5 x 22 ) e 1 + (2 a x1 x 2 )e 2 (0) e 3

a) Determinar el tensor de deformacin lineal, y el tensor de rotacin lineal;


b) Obtener las deformaciones principales y las tensiones principales;
c) Dado el mdulo de elasticidad transversal G , qu valor toma el mdulo de Young E
para que haya equilibrio en cualquier punto?
OBS.: Las fuerzas msicas son despreciables.
Solucin:
a) Considerando que u1 = a ( x12 5 x 22 ) , u 2 = 2 a x1 x 2 , u 3 = 0 , las componentes del
gradiente de los desplazamientos son:
2 x1a
r

u
i
( xr u) ij =
= 2ax2
x j
0

10ax2
2ax1
0

0
0
0

Descomponiendo de forma aditiva el gradiente de los desplazamientos en una parte


simtrica (tensor de deformacin lineal - ij ) y en una parte antisimtrica (tensor spin
infinitesimal- ij ):
u i
= ij + ij
x j

donde
1 u u j
ij = i +
2 x j x i

2x a
1 1
= 2ax 2
2

10ax 2
2ax1
0

0 2 x1 a
0 + 10ax 2
0 0

2ax 2
2ax1
0

0 2 x1 a

0 = 4ax 2
0 0

4ax 2
2ax1
0

0
0
0

y
Universidad de Castilla- La Mancha
Ciudad Real - Espaa

Draft

Por: Eduardo W. V. Chaves (2014)

6 ELASTICIDAD LINEAL

1 u u j
ij = i
2 x j x i

2x a
1 1
= 2ax 2
2

10ax 2
2ax1
0

471

0 2 x1 a
0 10ax 2
0 0

2ax 2
2ax1
0

0 0

0 = 6ax 2
0 0

6ax 2

0
0
0

0
0

b) Deformaciones principales.
2 x1 a

4ax 2

4ax 2

2ax1

=0

(2 x a ) (4ax )
2

=0

(2 x a ) = (4ax )
2

1 = 2 x1 a + 4ax 2

2 = 2 x1 a 4ax 2
0
0
2 x1a + 4ax2

0
2 x1a 4ax2 0 (deformaciones principales)
ij =

0
0
0

2 x1 a = 4ax 2

Ya que las deformaciones y tensiones comparten el mismo espacio principal podemos


utilizar la relacin ij = 4 x1a ij + 2 ij en el espacio principal:
0
1 0 0
2 x1a + 4ax2

ij = 4 x1a ij + 2 ij = 4 x1a 0 1 0 + 2
0
2 x1a 4ax2
0 0 1

0
0
0
0
4 x1a + 2 (2 x1a + 4ax2 )

0
0
=
4 x1a + 2 ( 2 x1a 4ax2 )

0
0
4 x1a

0
0
0

c) Partiendo de la ecuacin de equilibrio:

r r
+ {
b = 0 Indicial

ij , j = 0 i
r
=0

Expandiendo:

ij , j = 0 i

11,1 + 12, 2 + 13,3 = 0

21,1 + 22, 2 + 23,3 = 0

31,1 + 32, 2 + 33,3 = 0

11 12 13
+
+
=0

x 2
x 3
x1
21 22 23
+
+
=0

x
x
x
1
2
3

31 32 33
+
+
=0

x1
x 2
x 3

donde las componentes del tensor de tensiones son ij = kk ij + 2 ij , siendo kk = 4 x1 a


luego ij = 4 x1 a ij + 2 ij
11 = 4 x1 a 11 + 2 11 = 4 x1 a + 2 (2 x1 a ) = 4 x1 a ( + )
12 = 4 x1 a 12 + 2 12 = 2 (4ax 2 ) = 8 ax 2
13 = 0

ij = 4 x1 a ij

11
1 0 0

+ 2 ij = 4 x1 a 0 1 0 + 2 12
13
0 0 1

Universidad de Castilla- La Mancha


Ciudad Real - Espaa

Draft

12
22
23

13
23
33

Por: Eduardo W. V. Chaves (2014)

MECNICA DEL MEDIO CONTINUO: PROBLEMAS RESUELTOS

472

1 0 0
2 x1 a

ij = 4 x1 a 0 1 0 + 2 4ax 2
0 0 1
0

8 ax 2
0 4 x1 a ( + )
0

0 = 8 ax 2
4 x1 a ( + )
0
0
0
0
4 x1 a

4ax 2
2ax1
0

Luego, la primera ecuacin de equilibrio queda:


11 12 13
=0
+
+
x 3
x1
x 2

4a( + ) 8 a = 0

+ = 2

= =G

G (3 + 2G )
, que se puede obtener a travs de
+G
E
E
las relaciones =
, =G=
. Luego, concluimos que:
(1 + )(1 2 )
2(1 + )

Adems teniendo en cuenta la relacin E =

E=

G (3 + 2G ) G (3G + 2G )
=
= 2,5G
+G
G+G

Ejemplo 6.7
El estado tensional en un punto del medio continuo viene dado a travs de las
componentes del tensor de tensiones de Cauchy por:
26 6 0
ij = 6
9 0 kPa
0
0 29

Considerando un material elstico, lineal, homogneo e istropo. Se pide:


a) Obtener los invariantes principales de ;
b) Obtener la parte esfrica y desviadora de ;
c) Obtener los autovalores y autovectores de ;
d) Dibujar el crculo de Mohr en tensiones. Obtener tambin la tensin normal y la tensin
tangencial mxima.
e) Considerando un rgimen de pequeas deformaciones y teniendo en cuenta que el
material que constituye el medio continuo tiene como propiedades mecnicas
= 20000kPa y = 20000kPa ( , son las constantes de Lam). Obtener las
componentes del tensor de deformacin infinitesimal;
f) Obtener los autovalores y autovectores de .
Solucin:
I = 12 10 3

II =

0 29

10 6 +

26

29

10 6 +

26 6
6

10 6 = 763 10 6

III = det ( ) = 7830 10 9

Parte esfrica y desviadora ij = ijdev + ijesf :


m =

( 29 26 + 9)
1
ii =
= 4 10 3 Pa
3
3

Universidad de Castilla- La Mancha


Ciudad Real - Espaa

Draft

Por: Eduardo W. V. Chaves (2014)

6 ELASTICIDAD LINEAL

ijhid

ijdev

ijesf

473

4 0 0
= 0 4 0 kPa
0 0 4

6
0 30 6 0
26 4

= 6
94
0 = 6
5 0 kPa
0
0 25
0
29 4 0

Resolviendo la ecuacin caracterstica, los autovalores son:


I = 29kPa , II = 10kPa , III = 27 kPa :

Los autovectores:
I = 29kPa

direccin
principal

II = 10kPa

direccin
principal

III = 27 kPa

n i(1) = [0 0 1]

n i( 2 ) = [0,1644 0,98639 0]

n i(3) = [0,98639 0,1644 0]

direccin
principal

S (kPa)

S max =

29 (27)
= 28
2

N (kPa)

III = 27

I = N max = 29

II = 10

ij = Tr ( ) ij + 2 ij

donde

inversa

ij =

1
Tr ( ) ij +
ij
2 (3 + 2 )
2

= 5 10 9 ( Pa ) 1 , Tr ( ) = 1,2 10 4 ( Pa)
2 (3 + 2 )

1 0 0
26 6 0

8
ij = (5 10 )(1,2 10 ) 0 1 0 + 2,5 10 6
9 0 103
0 0 1
0
0 29
0
1 0 0
26 6 0 7,1 1,5

5
5
9 0 = 1,5 1,65
= 6 10 0 1 0 + 2,5 10 6
0 10 4
0 0 1
0
0 29 0
0 6,65
9

Como el material es istropo, el tensor de tensiones y el tensor de deformaciones


comparten el mismo espacio principal (mismas direcciones principales), luego:

Universidad de Castilla- La Mancha


Ciudad Real - Espaa

Draft

Por: Eduardo W. V. Chaves (2014)

MECNICA DEL MEDIO CONTINUO: PROBLEMAS RESUELTOS

474

ij =

ij
Tr ( ) ij +
2 (3 + 2 )
2

0 66,5 0
0
1 0 0
29 0

5
= 6 10 0 1 0 + 2,5 10 0 10
0 = 0 19
0 10 5
0 0 1
0 0 27 0
0 73,5
5

Las direcciones principales del tensor de deformaciones son las mismas que las del tensor
de tensiones para un material istropo.
Ejemplo 6.8
Demostrar que las ecuaciones constitutivas de tensin, para un material elstico lineal
homogneo e istropo, se puede representar por el conjunto de ecuaciones:
dev = 2 dev

Tr ( ) = 3Tr ( )

donde = G es la constante de Lam, es el mdulo de deformacin volumtrica.


Solucin:
= C e : = [1 1 + 2 I] : = Tr ( )1 + 2
= dev + esf = Tr ( )1 + 2 ( dev + esf )
Tr ( )
1 = Tr ( )1 + 2 ( dev + esf )
3
Tr ( ) Tr ( )
1
= Tr ( )1 + 2 dev + 2

3
3
2
Tr ( )

1
= +
Tr ( )1 + 2 dev
3
3

dev +
dev
dev

La traza del tensor de tensiones:


Tr ( ) = : 1 = [Tr ( )1 + 2 ] : 1 = Tr ( )3 + 2 Tr ( ) = (3 + 2 )Tr ( )

Con lo cual:
2
Tr ( )

dev
1
dev = +
Tr ( )1 + 2
3
3

(3 + 2 )Tr() 1
2

dev
dev = +
Tr ( )1 + 2
3
3

(3 + 2 )Tr() 1 + 2 dev
2

dev = +
Tr ( )1
3
14434444
424444444
3
=0

A las ecuaciones dev = 2 dev tenemos que aadir la restriccin:

(3 + 2 )Tr( )
2
2
Tr ( )

1=0
1=0

Tr ( )1
+
Tr ( )1
+
3
3
3
3

Tr ( )1 = 3 +
Tr ( )1 = 3 Tr ( )1
Tr ( )1
3

Universidad de Castilla- La Mancha


Ciudad Real - Espaa

Draft

Por: Eduardo W. V. Chaves (2014)

6 ELASTICIDAD LINEAL

475

o Tr ( ) = 3 Tr ( ) .

23

13
13
11

23
12

12

11

+
m

22
12

12

dev
22
12

ijdev = 2 ijdev

dev
33

23
12

m
23

23

dev
11

Tr ( ) ij = 3 Tr ( ) ij

33

13

13

23

13

22

ij = Tr ( ) ij + 2 ij

13

dev
33

33

23

13
13

23
12

dev
22
12

dev
11

Solucin Alternativa:
Partiendo de la ecuacin constitutiva en tensin para un material elstico, lineal,
homogneo e istropo: = ( ) = Tr ( )1 + 2 . Adems como estamos en rgimen
linear se cumple que = () = ( esf + dev ) = ( esf ) + ( dev ) , donde:
( esf ) = Tr ( esf )1 + 2 esf
2
Tr ( )
Tr ( )
Tr ( )
esf = Tr
11 + 2
1 = Tr ( )1 + 2
1 = ( +
) Tr ( )1 = Tr ( )1
3
3
3
3

Tr ( )
1 = Tr ( )1
3
Tr ( )1 = 3 Tr ( )1
( dev ) = Tr ( dev )1 + 2 dev = 2 dev
1
424
3
=0

[ ] = Tr[ ] .

Observar que se cumple que Tr ( esf ) = Tr esf

Universidad de Castilla- La Mancha


Ciudad Real - Espaa

Draft

Por: Eduardo W. V. Chaves (2014)

MECNICA DEL MEDIO CONTINUO: PROBLEMAS RESUELTOS

476

NOTA: Es interesante observar que: si en un material istropo tenemos un estado


puramente esfrico de comprensin:
p

p>0

0
p 0

ij = 0 p 0
0
0 p

p
p

Tr ( ) = 3 p < 0

E
, podemos
3(1 2 )
concluir que: si > 0,5 eso implica que < 0 y como consecuencia Tr ( ) > 0 , es decir, una

tenemos que Tr ( ) = 3 Tr ( ) < 0 , y teniendo en cuenta que =

expansin, lo que no tiene sentido fsico que en un material istropo sometido a un estado
de compresin haya una expansin.
Ejemplo 6.9
Un paraleleppedo de cierto material elstico lineal e istropo que a cierta temperatura tiene
dimensiones a = 0,10m , b = 0,20m , c = 0,30m , ver Figura 6.4, se introduce en una cavidad
de anchura b de paredes rgidas, planas y perfectamente lisas, de tal forma que dos caras
opuestas del paraleleppedo estn en contacto con las paredes de la cavidad, (ver Ortiz
Berrocal (1985)). Una vez en esta posicin se eleva la temperatura del prisma en
T = 30 C .
a) Calcular los valores de las tensiones principales en los puntos del paraleleppedo.
b) Hallar las componentes de las deformaciones.
Datos: mdulo de elasticidad E = 2 10 6 N / m 2 , coeficiente de Poisson = 0,3 , coeficiente
de expansin trmico del material es igual a 1,25 10 5 C 1 .
Solucin:
Como el slido puede deformarse libremente segn las direcciones x , z , luego est libre
de tensiones normales x = z = 0 . El slido est restringido al movimiento segn la
direccin y luego y = 0 :
y =

1
1
y ( x + z ) + T = y + T = 0
E
E

y = E T

resultando que:
y = E T = 2 106 1,25 10 5 (30) = 750

N
m2

Componentes del tensor de tensiones:

b)

0
0
0

ij = 0 750 0 Pa
0
0
0
y
x = z =
+ T = 4,875 10 4
E

El tensor de deformacin queda:

Universidad de Castilla- La Mancha


Ciudad Real - Espaa

Draft

Por: Eduardo W. V. Chaves (2014)

6 ELASTICIDAD LINEAL

477

0
4,875 0

ij = 0
0
0 10 4
0
0 4,875
z

Datos:

a = 0,10m
b = 0,20m
c = 0,30m
E = 2 106 N / m 2

= 0,3
T = 30 C

= 1,25 10 5 C 1

c
y

b
x

Figura 6.4.
Ejemplo 6.10
En el fondo de un recipiente, cuyo hueco interior es prisma recto, de base cuadrada, de
0,10 0,10m , se coloca un bloque de caucho sinttico de 0,10 0,10 0,5m , tal como se
indica en la Figura 6.5. El bloque ajusta perfectamente en el recipiente de paredes rgidas
lisas.
Las caractersticas elsticas del caucho sinttico son E = 2,94 10 6 N / m 2 y = 0,1 .
Sobre el caucho se vierten 0,004m 3 de mercurio (material incompresible), cuya densidad de
masa es 13580kg / m 3 . Se pide:
a) La altura H que alcanza el mercurio sobre el fondo del recipiente;
b) El estado tensional en un punto genrico del bloque de caucho
OBS.: Desprciese el peso del caucho. Considrese la aceleracin de la gravedad
g = 10m / s 2 .
Solucin:
Primero calculamos la fuerza total del mercurio ejerce sobre el caucho:
kg
kgm

m
F = V g = 0,004(m 3 ) 13580 3 10 2 = 543,20 2 N
s
m
s

La tensin normal segn la direccin z viene dada por:


z =

Universidad de Castilla- La Mancha


Ciudad Real - Espaa

F 543,20
N
=
= 54,320 10 3 2
A
(0,1 0,1)
m

Draft

Por: Eduardo W. V. Chaves (2014)

MECNICA DEL MEDIO CONTINUO: PROBLEMAS RESUELTOS

478

Segn las direcciones x , y el caucho no se deforma: x = y = 0

x = ( y + z )

y = ( x + z )

1
x ( y + z ) = 0
E
1
y = y ( x + z ) = 0
E
x =

{[

y = ( x + z ) = ( y + z ) + z =

Paredes rgidas

( 2 + )

z =
z = 6035,55Pa = x
2
(1 )
(1 )

Paredes rgidas

Mercurio

Caucho
L0 = 0,5m
y

Figura 6.5.
La deformacin normal segn la direccin z :
z =

1
1
{ 54320 0,1[2(6035,55)]} = 0,0180656
z ( x + y ) =
E
2,94 10 6

b) Variacin de la longitud del caucho:


L = L0 z = 0,5 ( 0,018656 ) = 0,00903m

La altura H que alcanza el mercurio sobre el fondo del recipiente:


H = hmer + ( L0 L)

donde hmer viene dado por:

Universidad de Castilla- La Mancha


Ciudad Real - Espaa

Draft

Por: Eduardo W. V. Chaves (2014)

6 ELASTICIDAD LINEAL

Vmer = b 2 hmer = 0,004

479

hmer =

0,004
= 0,4m
0,1 0,1

Resultando as que:
H = hmer + ( L0 L) = 0,4 + (0,5 0,00903) = 0,891m

Ejemplo 6.11
En un ensayo de laboratorio de un material elstico lineal se han obtenido las siguientes
relaciones:
31
1
21
y +
z
x = x +
E1
E2
E3
32
12
1
z
x + y +
y =
E1
E2
E3

(6.36)

13
23
1
x +
y + z
z =
E1
E2
E3

donde 12 = 0,2 , 13 = 0,3 , 23 = 0,25 , E1 = 1000 MPa , E 2 = 2000MPa , E3 = 1500MPa .


Sabiendo que el material analizado es un MATERIAL ORTTROPO. Obtener los valores
de 21 , 31 , 32 .
Solucin:
La matriz constitutiva para un material orttropo presenta el siguiente formato:
C11
C
12
C
[C ] = 13
0
0

C12

C22
C23
0
0
0

C13

C23
C33
0
0
0

0
0

0
0

C44
0
0

0
C55
0

0
0
0

0
0

C66

Simetra Orttropa
9 constantes independientes

(6.37)

Reestructurando (6.36) obtenemos que:


1

E

xx 1
12
yy E
zz 1
= 13

xy

2 yz E1
0

2 xz
0
0

21

E2
1

E2
23

E2
0
0
0

31
0

E3
32
0

E3
1

0
E3
0
C44
0
0

0
0

0
0
0
0

C55
0

0
xx

0 yy

zz
0 xy

0 yz

0 xz
C66

(6.38)

Luego para un material orttropo debe cumplir que:

Universidad de Castilla- La Mancha


Ciudad Real - Espaa

Draft

Por: Eduardo W. V. Chaves (2014)

MECNICA DEL MEDIO CONTINUO: PROBLEMAS RESUELTOS

480

21 12

E2 E1

31 13

E3 E1

32 23

E3 E2

obteniendo as que

21
E2

31
E3

32
E3

12

13

23

E1
E1
E2

21 =

E2 12 2000 0,2
=
= 0,4
1000
E1

31 =

E3 13 1500 0,3
=
= 0,45
1000
E1

32 =

E3 23 1500 0,25
=
= 0,1875
2000
E2

Ejemplo 6.12
Dado un material elstico lineal, homogneo e istropo con las siguientes propiedades
elsticas: E = 71 GPa y G = 26,6 GPa . Determinar las componentes del tensor de
deformacin y la densidad de energa de deformacin en un punto del cuerpo si las
componentes del tensor de tensiones en este punto son:
20 4 5
ij = 4 0 10 MPa
5 10 15

Solucin:
Obtenemos el coeficiente de Poisson partiendo de la relacin:
=

G=

E
2(1 + )

E
1 = 0,335
2G
1
[11 ( 22 + 33 )] = 1 9 [20 0,335 (0 + 15 )]10 6 = 211 10 6
E
71 10
1
1
[0 0,335 (20 + 15 )]10 6 = 165 10 6
= [ 22 (11 + 33 )] =
E
71 10 9
1
1
= [ 33 (11 + 22 )] =
[15 0,335 (20 + 0 )]10 6 = 117 10 6
E
71 10 9

11 =
22
33

1+
1 + 0,335
12 =
( 4 10 6 ) = 75 10 6
9
E
71 10
1+
1 + 0,335
13 =
13 =
(5 10 6 ) = 94 10 6
E
71 10 9
1+
1 + 0,335
(10 10 6 ) = 188 10 6
23 =
23 =
E
71 10 9
12 =

En forma matricial:
211 75 94
ij = 75 165 188 10 6
94
188 117

La densidad de energa de deformacin para un material elstico lineal viene dada por:

Universidad de Castilla- La Mancha


Ciudad Real - Espaa

Draft

Por: Eduardo W. V. Chaves (2014)

6 ELASTICIDAD LINEAL

1
2

1
2

481

1
2

e = : C e : = : indicial
e = ij ij
Considerando la simetra de los tensores de tensin y de deformacin la densidad de
energa de deformacin resulta:
1
[1111 + 22 22 + 33 33 + 212 12 + 2 23 23 + 21313 ]
2
1
= [( 211)( 20) + ( 165)(0) + (117 )(15) + 2( 75)( 4) + 2(188)(10) + 2(94)(5) ]
2
= 5637 ,5 J / m 3

e =

Podemos tambin obtener la densidad energa de deformacin utilizando la ecuacin:

e=

1
1
1
1
I 2 II dev =
I 2 + J 2
6(3 + 2 )
2
6(3 + 2 )
2

y considerando que: I = 3,5 10 7 ; II = 2,4933 1014 ; 5,3804 10 10 Pa ; = G ,


obtenemos:

e 5638 ,03 J / m 3
La diferencia entre los resultados obtenidos es debida a la aproximacin numrica.
Ejemplo 6.13
Expresar la densidad energa de deformacin en funcin de los invariantes principales de
.
Solucin:
1
2

1
2

e = : = : [Tr ( )1 + 2 ] =
=

[Tr ( )]

+ Tr ( T ) =

2
[Tr ( )]2
=
+ Tr ( 2 )
2

Tr ( )
2

[Tr ( )]

2
1 + : =
:3
1

[Tr ( )]2

Tr ( )

+ :

+ Tr ( )

Podemos sumar y restar el trmino [Tr ( )]2 sin alterar la expresin:

e=
=

[Tr ( )]2
2

+ [Tr ( ) ]2 + Tr ( 2 ) [Tr ( ) ]2

1
( + 2 )[Tr ( )]2 [Tr ( )]2 Tr ( 2 )
2

Considerando que los invariantes principales de son I = Tr ( ) , II =


obtenemos que:

1 2
I Tr ( 2 ) ,
2

1
2

e = ( + 2 )I 2 2 II = e ( I , II )

Universidad de Castilla- La Mancha


Ciudad Real - Espaa

Draft

Por: Eduardo W. V. Chaves (2014)

MECNICA DEL MEDIO CONTINUO: PROBLEMAS RESUELTOS

482

Ejemplo 6.14
Se conocen las respuestas de un slido termoelstico lineal en equilibrio a un sistema de
r

r (I )
en S ; u* ; en S ur ; T ( I ) ) y a otro sistema de acciones
r ( II )
en S ; u* ; en S ur ; T ( II ) ) . Justificar (demostrar) cul sera la respuesta

acciones I (b ( I ) , t *

(I )

r
r ( II )
II (b ( II ) , t *
al sistema I + II , (ver Oliver (2000)).

Solucin:
Como estamos en el rgimen lineal se cumplen que:
r r
r
b = b ( I ) + b ( II )

T = T ( I ) + T ( II )

r
r ( I ) r ( II )
t* = t* + t*

r
r ( I ) r ( II )
u * = u* + u *

Lo mismo para los campos:


r r
r
u = u ( I ) + u ( II )

= ( I ) + ( II )

= ( I ) + ( II )

Partiendo de las ecuaciones de gobierno del problema termoelstico lineal en equilibrio:

Ecuaciones de Equilibrio:

Ecuaciones Cinemticas:

r
r
r
r
r
r
xr + b = xr ( ( I ) + ( II ) ) + (b ( I ) + b ( II ) ) = [ xr ( I ) + b ( I ) ] + [ xr ( II ) + b ( II ) ] = 0

{[
{ [

] [

r
r
r
1 r r (I )
1
x u + ( xr u ( I ) ) T + xr u ( II ) + ( xr u ( II ) ) T
2
2
r ( II )
r (I )
r ( II ) T
+ xr u
+ xr u + xr u

= ( I ) + ( II ) =

]}

] [

r
1
xr u ( I )
2
r
r
r
r
r
rT
T
1
1
= xr u ( I ) + u ( II ) + xr (u ( I ) + u ( II ) ) = xr u + [ xr u] =
2
2
=

] [

]} {

Ecuacin Constitutiva:

= C e : + MT

donde M es el tensor de tensiones trmicas


= C e : + MT = C e : ( ( I ) + ( II ) ) + M (T ( I ) + T ( II ) )
= (C e : ( I ) + MT ( I ) ) + (C e : ( II ) + MT ( II ) )
= ( I ) + ( II )

Se comprobando as que se cumplen todas las condiciones. Luego, tambin se puede


aplicar el principio de la superposicin al problema termoelstico lineal, como era de
esperar ya que estamos en el rgimen lineal.
Ejemplo 6.15
Considrese una barra de 7,5m de longitud y 0,1m de dimetro que est constituida por un
material cuyas propiedades son: E = 2,0 10 11 Pa y = 20 10 6
est a 15 C y la temperatura aumenta a 50 C .

1
. Inicialmente la barra
C

a) Determinar el alargamiento de la barra considerando que la barra pueda expandirse


libremente;
b) Suponga que la barra ya no puede alongarse libremente porque en sus extremos se han
colocado bloques de hormign, ver Figura 6.6(b). Obtener la tensin en la barra.
Universidad de Castilla- La Mancha
Ciudad Real - Espaa

Draft

Por: Eduardo W. V. Chaves (2014)

6 ELASTICIDAD LINEAL

483

OBS.: Considerar el problema en una dimensin.


x
L = L(1) + L( 2 )
L(1)
T

L
L
L( 2)

b)

a)

Figura 6.6: Barra bajo variacin de temperatura.


Solucin:
a) Para obtener el alargamiento debemos calcular previamente la deformacin segn la
direccin del eje de la barra:
ij = T ij

Como se trata de un caso unidimensional solo consideraremos la componente de la


deformacin segn el eje x , 11 = x , luego:
11 = x = 20 10 6 (50 15) = 7 10 4

El alargamiento se obtiene segn la integral:


L

L = x dx = x L = 7 10 4 7,5 = 5,25 10 3 m
0

Observar que como la barra puede expandirse libremente, sta est libre de tensin.
b) Si las extremidades no pueden moverse, surgirn tensiones uniformes que vienen dadas
por:
x = E T = E " x " = 2,0 1011 7 10 4 = 1.4 10 8 Pa

Observemos que en el caso 2) no hay deformacin, ya que L = 0 . No obstante, es


equivalente a una barra de longitud L + L en la cual aplicamos una tensin de compresin
hasta obtener la longitud final igual a L .
Ejemplo 6.16
Dado un material elstico lineal, homogneo e istropo con las siguientes propiedades
mecnicas: E = 10 6 Pa (mdulo de Young), = 0,25 (coeficiente de Poisson),
= 20 10 6 C 1 (Coeficiente de dilatancia trmica).
Considrese que en un determinado punto del slido se conoce el estado tensional y que
viene dado por:
12 0 4
ij = 0 0 0 Pa
4 0 6

Se pide:

Universidad de Castilla- La Mancha


Ciudad Real - Espaa

Draft

Por: Eduardo W. V. Chaves (2014)

MECNICA DEL MEDIO CONTINUO: PROBLEMAS RESUELTOS

484

a) Determinar las tensiones y direcciones principales; Obtener la tensin tangencial


mxima.
b) Las deformaciones en dicho punto. Determinar tambin las deformaciones y direcciones
principales;
c) Determinar la densidad de energa de deformacin.
d) Si ahora, a ste slido sufre una variacin de temperatura T = 50 C , Cual es el estado
de deformacin final en este punto?
e) Se puede decir que estamos ante un caso de Tensin Plana?
Solucin:
a) Obtenemos los autovalores al resolver el determinante caracterstico. Adems
observemos que ya se conoce un autovalor 2 = 0 que est asociado a la direccin
n i( 2 ) = [0 1 0] . Luego, es suficiente resolver el determinante:
12

=0

2 18 + 56 = 0

Resolviendo la ecuacin cuadrtica obtenemos que:


(1,3) =

18 324 224
2

1 = 14

3 = 4

14 0 0
'ij = 0 0 0 Pa
0 0 4

A continuacin obtenemos las direcciones principales (autovectores que deben ser


versores, vectores unitarios), resultando:
1 = 14

autovector

2 = 0

autovector

3 = 4

autovector

2
0
n i(1) =
5
n i( 2 ) = [0 1 0]
1
n i(3) =
5

1
= [0 ,8944 0 0 ,4472]
5
2
= [0,4472 0 0 ,8944]
5

Haciendo el cambio de nomenclatura tal que I > II > III , tenemos que I = 14 ,
II = 4 , III = 0 .
Podemos obtener la tensin tangencial mxima como:
S max =

Universidad de Castilla- La Mancha


Ciudad Real - Espaa

I III
(14) (0)
=
= 7 Pa
2
2

Draft

Por: Eduardo W. V. Chaves (2014)

6 ELASTICIDAD LINEAL

485

S (Pa )

S max = 7

N (Pa)
II = 4

III = 0

I = 14

b) Las componentes del tensor de tensiones de Cauchy vienen dadas por:


ij = Tr ( ) ij + 2 ij

inversa

ij =

Tr ( ) ij +
ij
2
2 (3 + 2 )

Recordar que = C e : , la inversa = C e : .


donde

Tr ( ) = 18 ,

E
(1 + )(1 2 )

= 4 105 Pa ,

=G=

E
= 4 105 Pa ,
2(1 + )

1
= 1,25 10 6 ,
2

Tr ( )
= 2,5 10 7 Pa ,
= 4,5 10 6 Pa
2 (3 + 2 )
2 (3 + 2 )

0
5
1 0 0
12 0 4 10,5

6
6
ij = 4,5 10 0 1 0 + 1,25 10 0 0 0 = 0 4,5 0 10 6

0 0 1
4 0 6 5
0
3

Para un material istropo lineal las direcciones principales de las tensiones y deformaciones
coinciden. Las deformaciones principales podemos obtener trabajando en el espacio
principal: ij =

Tr ( ) ij +
ij :
2
2 (3 + 2 )

0
0
1 0 0
14 0 0 13

6
6
ij = 4,5 10 0 1 0 + 1,25 10 0 0 0 = 0 4,5 0 10 6

0 0 1
0 0 4 0
0
0,5

1
2

1
2

c) La densidad de energa de deformacin viene dada por e = : = ij ij . Podemos


utilizar el espacio principal donde se cumple que:
0
0
13

ij = 0 4,5 0 10 6
0
0
0,5

14 0 0
ij = 0 0 0
0 0 4

Luego, podemos utilizar el espacio principal para obtener la densidad de energa de


deformacin:

Universidad de Castilla- La Mancha


Ciudad Real - Espaa

Draft

Por: Eduardo W. V. Chaves (2014)

MECNICA DEL MEDIO CONTINUO: PROBLEMAS RESUELTOS

486

1
2

1
2

e = ij ij = ij ij =

1
[11 11 + 3333 ] = 92 10 6 J3
2
m

d) Utilizando el principio de la superposicin, podemos decir que:


ij = ij ( ) + ij (T ) = ij ( ) + T ij

Luego,
0
5
0
5
10,5
1 0 0 1010,5

6
6
ij = 0 4,5 0 10 + 20 10 500 1 0 = 0
995,5
0 10 6
5
0 0 1 5
0
3
0
1003

Las direcciones principales del tensor de deformacin infinitesimal son las mismas del
tensor de tensiones.
e) No podemos decir que se trata de un estado de tensin plana ya que no tenemos
informacin del estado tensional de todo el medio continuo. Solo estaremos en el caso de
tensin plana cuando el CAMPO de tensin es independiente de una direccin.
Ejemplo 6.17
Considrese una barra donde en una de las extremidades se aplica una fuerza igual a
6000 N como se indica en la Figura 6.7. Determinar x , y , z y el cambio de longitud en
las dimensiones de la barra. Considrese tambin que la barra est constituida por un
material cuyas propiedades elsticas son: Mdulo de Young: E = 10 7 Pa ; Coeficiente de
Poisson: = 0,3 .
Hiptesis: Rgimen de pequeas deformaciones, y material elstico lineal, homogneo e
istropo.

1m
100 m

1m

y, v

y =

x, u

6000
11

F = 6000 N

z, w

Figura 6.7

Universidad de Castilla- La Mancha


Ciudad Real - Espaa

Draft

Por: Eduardo W. V. Chaves (2014)

6 ELASTICIDAD LINEAL

487

Solucin:
Teniendo en cuenta que para un material elstico linear e istropo, las tensiones normales
solo producen deformaciones normales, donde:

1
(0,3)(6000 )
x y + z = y =
= 0,00018
E
E
10 7
y 6000
1
y = y ( x + z ) =
=
= 0,0006
E
E
10 7
1

z = z x + y = y = 0,00018
E
E
x =

)]

)]

Los cambios de longitud en las dimensiones de la seccin son:


u = w = 0,00018 1 = 1,8 10 4 m

y de la longitud:
v = 0,0006 100 = 6,0 10 2 m

Ejemplo 6.18
Consideremos un prisma cuadrangular regular cuyo material elstico linear e istropo tiene
de mdulo de elasticidad EP = 27,44 105 N / cm2 y coeficiente de Poisson = 0,1 . La
longitud del lado de la seccin recta es a = 20cm . En ambas bases del prisma se colocan
dos placas perfectamente lisas y rgidas, de peso despreciable, unidas entre s mediante
cuatro cables de seccin AC = 1cm2 y mdulo de elasticidad EC = 19,6 106 N / cm2 de
longitudes iguales a la altura del prisma l = 1m , simtricamente dispuesto, como indica en la
Figura 6.8, (ver Ortiz Berrocal (1985)).
Sobre dos caras laterales opuestas del prisma se aplica una fuerza de compresin uniforme
p = 7350 N / cm 2 . Se pide calcular:
a) Tensin en los cables;
b) Tensiones principales en el prisma;
c) Variacin de volumen experimentada por el prisma.
Solucin:
Verifiquemos que el cable y el prisma tienen que deformarse, segn direccin z , de igual
manera:
Pz = Cz

En el cable se cumple que:


Cz = EC Cz

Cz =

Cz
EC

El campo de tensiones en el prisma viene dado por:

0
0 0

P
0
ij = 0 p

4Cz AC

0 0 a 2
Universidad de Castilla- La Mancha
Ciudad Real - Espaa

Draft

Por: Eduardo W. V. Chaves (2014)

MECNICA DEL MEDIO CONTINUO: PROBLEMAS RESUELTOS

488

a) Configuracin de referencia

b) Configuracin deformada

z
z

l = 1m

a
x

Figura 6.8
Deformacin del prisma segn direccin z :
Pz =

1
1 4Cz AC
z x + y =
+ p

2
EP
EP
a

)]

Aplicando que Pz = Cz :
Pz = Cz

Cz
1 4Cz AC

p
+

=
EP
a2
EC

Tras algunas manipulaciones algebraicas obtenemos la tensin en el cable:


c =

EC pa 2
( EP a 2 + 4 EC AC )

0,1 19,6 106 7350 202


N
= 4900 2
5
2
6
(27,44 10 20 + 4 19,6 10 1)
cm

La tensin normal segn direccin z en el prisma queda:


Pz =

4C AC
4 4900 1
N
=
= 49 2
2
2
20
a
cm

Tensor de tensiones en el prisma:


ijP

0
0
0
N

= 0 7350
0
cm 2
0
49
0

Variacin de volumen en el prisma:


V = V V 0

donde V = I es la deformacin volumtrica lineal (pequeas deformaciones):


Universidad de Castilla- La Mancha
Ciudad Real - Espaa

Draft

Por: Eduardo W. V. Chaves (2014)

6 ELASTICIDAD LINEAL

V = I = x + y + z =

x + y + z
EP

489

(1 2 ) = 2,12857 103

y V0 = 4 10 4 cm 3 es el volumen del prisma, resultando:


V = V V0 = (2,12857 10 3 )(4 10 4 ) = 85,1428cm 3

Ejemplo 6.19
Dos paraleleppedos iguales y formados por el mismo material (elstico lineal e istropo) y
de dimensiones a b c , se colocan a uno y otro lado de una placa lisa rgida adosados a
ella por sus caras a c , de tal forma que sus ejes de simetra perpendiculares a dichas caras
sean coincidentes. Ambos paraleleppedos, junto con la placa, se introducen en una ranura
de anchura igual a dos veces la longitud de la arista b ms el espesor de la placa. Las
paredes de la ranura son planas, rgidas y perfectamente lisas, (ver Ortiz Berrocal (1985)).
Se aplican respectivamente a los bloques en sus caras superiores y perpendiculares a ellas
fuerzas uniformemente repartidas p1 y p 2 por unidad de superficie.
Conociendo el mdulo de elasticidad longitudinal E y el coeficiente de Poisson , se pide
calcular:
a) Las tensiones principales en ambos bloques
b) Las variaciones de longitud de las aristas de los bloques.
z

a
p1

p2

c
y

Figura 6.9.
Solucin:
Prisma 1: (x1) = 0

(y1)

(z1) = p1

Prisma 2: (x2) = 0

(y2)

(z2) = p 2

Universidad de Castilla- La Mancha


Ciudad Real - Espaa

Draft

Por: Eduardo W. V. Chaves (2014)

MECNICA DEL MEDIO CONTINUO: PROBLEMAS RESUELTOS

490

Por compatibilidad de tensin:


(y1) = (y2) = y
(y1) + (y2 ) = 0

y (z1)

)]

)]

1 (1)
1
y (x1) + (z1) + (y2) (x2 ) + (z2 ) = 0
E
E
( 2)
+ y z = 0

y + p1 + y + p 2 = 0

] [

] [

Resultando
y =

( p1 + p2 )

Prisma 1: (x1) = 0

(y1) =

Prisma 2: (x2) = 0

(y2 ) =

( p1 + p2 )
2
;

(z1) = p1

(z2 ) = p2

( p1 + p2 )
2

Las deformaciones en cada prisma viene dadas por:


Prisma 1:

(y1)
(z1)

[
[

)]

)]

)]

[
[

)]

)]

)]

1 (1)
x (y1) + (z1) =
[ ( p1 + p2 ) + 2 p1 ]
2E
E

1
= (y1) (x1) + (z1) =
( p1 p2 )
2E
E
1
1 2
= (z1) (x1) + (y1) =
( p1 + p2 ) 2 p1
E
2E

(x1) =

Prisma 2:

1 ( 2)
x (y2) + (z2 ) =
[ ( p1 + p2 ) + 2 p2 ]
2E
E

1
= (y2 ) (x2) + (z2 ) =
( p2 p1 )
2E
E
1
1 2
= (z2 ) (x2) + (y2 ) =
( p1 + p2 ) 2 p2
E
2E

(x2 ) =
(y2 )
(z2 )

Variacin de las aristas:


Prisma 1
a
[ ( p1 + p 2 ) + 2 p1 ]
2E
b
= (y1) b =
( p1 p 2 )
2E
c
= (z1) c =
2 ( p1 + p 2 ) 2 p1
2E

Prisma 2
a
[ ( p1 + p 2 ) + 2 p 2 ]
2E
b
= (y2 ) b =
( p 2 p1 )
2E
c
= (z2) c =
2 ( p1 + p 2 ) 2 p 2
2E

a (1) = (x1) a =

a ( 2 ) = (x2 ) a =

b (1)

b ( 2 )

c (1)

c ( 2)

(6.39)

Ejemplo 6.20
Un cubo metlico que tiene longitud de arista a = 0,20m se sumerge en el mar a una
profundidad z = 400m .

Universidad de Castilla- La Mancha


Ciudad Real - Espaa

Draft

Por: Eduardo W. V. Chaves (2014)

6 ELASTICIDAD LINEAL

491

Conociendo el mdulo de elasticidad longitudinal del metal E = 21 1010 Pa , el coeficiente


de Poisson = 0,3 , calcular la variacin de volumen que experimenta el cubo sumergido.
Considerar la aceleracin de la gravedad igual a g = 10m / s 2 .
OBS.: Aunque la densidad de masa vara con la temperatura, salinidad, y presin
(profundidad) considerar la densidad de masa del agua del mar igual a = 1027 kg / m 3 .
Solucin:
Debido a la profundidad y a las dimensiones del cubo podemos tomar como una buena
aproximacin que todo el cubo est sometido a una misma presin, ver Figura 6.10.

h = 400m

Figura 6.10
Podemos obtener la presin a partir de la segunda ley de Newton F = ma = V g (peso de la
columna de agua) y dividiendo por el rea:
p=

kg
kg m
F V g Ah g
m
=
=
= gh = 1027 3 10 2 400m = 4,108 10 6 2 2 = 4,108 10 6 Pa
A
A
A
m
s
m s

Luego, las componentes del tensor de tensiones en el cubo vienen dadas por:
0 4,108
0
0
p 0

ij = 0 p 0 = 0
4,108
0 MPa
0
4,108
0 p 0
0

Como solo tenemos componentes normales de tensin y el material es istropo, solo habr
componentes normales de deformacin e iguales:
z = y = x =

)]

1
1
x y + z =
[ 4,108 0,3 ( 4,108 4,108 )] 10 6
E
21 1010

Resultando
z = y = x = 7,82 10 6

En pequeas deformaciones la deformacin volumtrica lineal es igual a la traza del tensor


de deformaciones infinitesimal:

Universidad de Castilla- La Mancha


Ciudad Real - Espaa

Draft

Por: Eduardo W. V. Chaves (2014)

492

MECNICA DEL MEDIO CONTINUO: PROBLEMAS RESUELTOS

V
= DVL V = Tr ( )
V0

V = V0 Tr ( ) = 0,2 3 ( 2,346 10 5 ) = 1,8768 10 7 m 3

donde hemos considerado que Tr ( ) = 2,346 10 5 .


Ejemplo 6.21
Un cilindro macizo, de 0,05m de radio de la base y 0,25m de altura, est constituido por
un material elstico lineal, de mdulo de elasticidad longitudinal E = 3 10 4 MPa y
coeficiente de Poisson = 0,2 . Dicho cilindro se sita entre los pistones de una prensa,
que se pueden considerar infinitamente rgidos, y todo ello se encierra en recipiente
hermtico, como se indica en la Figura 6.11.
Se llena el recipiente con aceite, y mediante el mecanismo adecuado, se eleva la presin en
el fluido hasta 15MPa . Haciendo funcionar la prensa, se aplica una fuerza axil total de
F = 2,35619 10 5 N sobre las bases del cilindro. Esta fuerza axil es el resultado debido a la
accin de la prensa ms el producido por la presin del aceite sobre los pistones, y se puede
considerar uniformemente repartido sobre las bases.
Se pide determinar, en un punto genrico del cuerpo:
a) Las componentes del tensor de tensiones;
b) Las componentes del tensor de deformaciones;
c) Las componentes del campo de desplazamientos ( u , v , w ).
F

aceite
y

0,25m

Corte AA
F
0,1m

Figura 6.11: Ensayo de compresin triaxial.


Solucin:
a) Tensor de tensiones

Universidad de Castilla- La Mancha


Ciudad Real - Espaa

Draft

Por: Eduardo W. V. Chaves (2014)

6 ELASTICIDAD LINEAL

z =

F
2,35619 10 5
=
= 30 MPa
A
(0,05) 2

493

x = y = 15MPa

Las componentes del tensor de tensiones son:


0
15 0

ij = 0
15
0 MPa
0
0
30

b) Para un material elstico, lineal, homogneo e istropo, las tensiones normales solo
producen deformaciones normales, luego:

)]

x = E x y + z

y = y ( x + z )
E

z = E z x + y

)]

Sustituyendo los valores de las variables obtenemos las siguientes componentes para el
Tensor de Deformaciones:
0
2 0

ij = 0 2 0 10 4
0
0 8

c) Campo de desplazamientos
Como estamos en pequeas deformaciones se cumplen las siguientes relaciones:
x =

u
x

y =

v
y

z =

w
z

Integrando y obteniendo los valores de las constantes de integracin obtenemos finalmente


el campo de desplazamientos:
u = 2 10 4 x

v = 2 10 4 y

w = 8 10 4 z

Ejemplo 6.22
Un hexaedro regular, de 0,1m de lado, est constituido por un material cuyas propiedades
mecnica viene representadas por las constantes de Lam: = 8333,33MPa ,
= 12500 MPa .
Mediante una mquina de ensayos adecuada se le impone la deformacin representada en la
Figura 6.12, en la cual todas las caras continan siendo planas, las caras AEFB y DHGC
pasan a ser rombos y las restantes continan siendo cuadradas. En este estado se pide
calcular:
a) El campo de desplazamientos;
b) El campo de deformaciones;
c) El campo de tensiones;
d) Las deformaciones y tensiones principales en el centro del hexaedro;
e) Las acciones ejercida por la mquina de ensayo sobre las caras ABFE y BCGF .

Universidad de Castilla- La Mancha


Ciudad Real - Espaa

Draft

Por: Eduardo W. V. Chaves (2014)

MECNICA DEL MEDIO CONTINUO: PROBLEMAS RESUELTOS

494

z
H

G
tan( ) = 0,001

C = C
D = D

A = A

y
B = B

Figura 6.12: Hexaedro deformado.


Solucin:
a) Segn la Figura 6.12 podemos verificar que solo habr componentes tangenciales de
deformacin. Adems verificamos tambin que no hubo desplazamientos segn las
direcciones x y z , luego u = 0 , w = 0 . A travs de una analoga de tringulos podemos
sacar el desplazamiento v :
tan( ) = 0,001 =

v
z

v( z ) = 0,001z

Campo de desplazamiento:
z

u = 0

v( z ) = 0,001z
w = 0

v(z )

b) Teniendo en cuenta las componentes del tensor de deformaciones:

Universidad de Castilla- La Mancha


Ciudad Real - Espaa

Draft

Por: Eduardo W. V. Chaves (2014)

6 ELASTICIDAD LINEAL

x
1
ij = xy
2
1
2 xz

1
xy
2
y
1
yz
2

u
1
xz
x

1
1 v u
yz = +
2 2 x y

z 1 w u
2 x + z

495

1 v u
+
2 x y
v
y

1 w v

+
2 y z

1 w u
+

2 x z
1 w v

+
2 y z

Concluimos que x = y = z = xy = xz = 0 y la componente yz viene dada por:


yz =

x
1
ij = xy
2
1
2 xz

1
xy
2
y
1
yz
2

v w
+
= 0,001
z y
1

xz
2
0
0
0
1

yz = 0
0
0,0005
2

0 0,0005
0
z

c) Campo de tensiones = Tr ( )1 + 2
Considerando Tr ( ) = 0 , = 8333,33MPa , = 12500MPa , resulta:
0
0
0
0
0 0

ij = 2 (12500) 0
0
0,0005 MPa = 0 0 12,5 MPa
0 0,0005
0 12,5 0
0

d) Deformaciones principales:

0,0005
=0
0,0005

2 = 0,0005 2

2 = +0,0005
= 0,0005
3 = 0,0005

Recordemos que en pequeas deformaciones las direcciones principales de tensiones


coinciden con las direcciones principales de deformaciones, luego podemos aplicar la
expresin = Tr ( )1 + 2 en el espacio principal de deformacin:
0
0 0 0
0
0

0 = 0 12,5
0 MPa
ij = 2 (12500) 0 0,0005
0
0
0,0005 0 0 12,5

e) Para obtener la fuerza total en una cara, multiplicamos la fuerza de superficie por el rea
de la respectiva cara. La fuerza de superficie se obtiene a travs de la expresin del vector

tensin t (n) = n . Para la cara ABFE la normal viene dada por n i = [1,0,0] , luego:
t 1 ( ABFE ) 0 0
0 1 0
( ABFE )

= 0 0 12,5 0 = 0
t 2
t ( ABFE ) 0 12,5 0 0 0


3

Para la cara BCGF la normal viene dada por n i = [0,1,0] , luego

Universidad de Castilla- La Mancha


Ciudad Real - Espaa

Draft

Por: Eduardo W. V. Chaves (2014)

MECNICA DEL MEDIO CONTINUO: PROBLEMAS RESUELTOS

496

t 1 ( BCGF ) 0 0
0 0 0
( BCGF )

= 0 0 12,5 1 = 0 MPa
t 2
t ( BCGF ) 0 12,5 0 0 12,5

Si hacemos el mismo procedimiento para las dems caras verificamos que la representacin
de las fuerzas de superficies viene indicada tal y como se muestra en la figura abajo:
z

C = C

D = D
A = A

B = B

Ejemplo 6.23
Sobre el prisma recto de la Figura 6.13 actan las fuerzas F1 = 10 N y F2 = 2 N sobre las
caras indicadas. Las longitudes de las aristas del prisma son: AB = 4cm , AD =

10
cm ,
3

AA = 2cm . Sabiendo que el material que constituye el prisma tiene como propiedades
N
termo-mecnicas: Mdulo de Young E = 2,5 10 6 2 , coeficiente de Poisson = 0,25 , y
cm
1
coeficiente de expansin trmico = 5 10 8
.
C

a) Obtener las tensiones principales; b) Obtener las componentes del vector tensin en el
plano . Es en el plano donde acta la mxima tensin tangencial? Justificar la
respuesta. c) Obtener el valor de las fuerzas F1 y F2 que se deben aplicar para que no haya
desplazamiento, segn las direcciones x1 y x2 , cuando el prisma est sometido a una
variacin de temperatura de T = 20 C .

Universidad de Castilla- La Mancha


Ciudad Real - Espaa

Draft

Por: Eduardo W. V. Chaves (2014)

6 ELASTICIDAD LINEAL

x2

497

A2
A1

F2

F1

F1

x1

60

F2
x3

Figura 6.13.
a) Campo de tensiones

A1 = 8,0 , A2 = 4

10
3

F1
A
1
ij = 0

0
0
0
1,25
N

0 = 0 0,15 0

cm 2

0
0
0

0
F2
A2
0

Que son las propias tensiones principales.


x2

r
t (n)

D
x1

60

x3

Figura 6.14.

Universidad de Castilla- La Mancha


Ciudad Real - Espaa

Draft

Por: Eduardo W. V. Chaves (2014)

MECNICA DEL MEDIO CONTINUO: PROBLEMAS RESUELTOS

498

1
La normal (vector unitario) tiene componentes: n i = ;
; 0 , ver Figura 6.14.

r
t (n) = n

t i(n) = ijn j

indicial

r
Luego, el vector tensin t (n) viene dado por:

t i(n)

0
0
1,25
= 0 0,15 0

0
0
0

2 1,0825
1
= 0,075
2
0 0

La componente normal:

N = t (n) n = t i(n)n i

N = [1,0825 0,075 0]

2
1
= 0,9
2
0

La componente tangencial:
r
t (n)

= 2N + 2S

S =

r
t (n)

2N

donde
r
t (n )

1,0825
r (n ) r (n )
(n ) (n )
= t t = t i t i = [1,0825 0,075 0] 0,075 = 1,1775
0

Luego:
S =

r
t (n)

2N = 1,1775 0,9 2 = 0,60621778

Si dibujamos el crculo de Mohr de tensiones

I = 1,25

III = 0,15

N ( N / cm 2 )

Verificamos que en cualquier punto del slido la tensin tangencial mxima est en un
2
;
2

plano definido por la normal n i =

Universidad de Castilla- La Mancha


Ciudad Real - Espaa

2
; 0 y tiene como valor mximo
2

Draft

Por: Eduardo W. V. Chaves (2014)

6 ELASTICIDAD LINEAL

max =

499

I III
= 0,7 > S
2

c) Consideremos el campo de deformaciones:


=

1+

Tr ( )1 + T 1
E
E

ij =

1+

ij Tr ( ) ij + T ij
E
E

Para el caso en particular Tr ( ) = 11 + 22 :


0 0 0
0 0 0 = 1 +

E
0 0 33

11 0
0
22

0
0

0
1 0 0

0 + T Tr ( ) 0 1 0
E

0 0 1
0

Luego, montamos el siguiente sistema:

1+

1+

11 + T Tr ( ) =
11 + T (11 + 22 )
11 = 0 =
E
E
E
E

= 0 = 1 + + T Tr ( ) = 1 + + T ( + )
22
22
11
22

22
E
E
E
E

Resolviendo el sistema anterior, obtenemos que:


11 = 22 =

E T
N
= 3,33333 2
(1 )
cm

Luego, las fuerzas vienen dadas por:


F1 = 11 A1 = 26,66666 N

F2 = 22 A2 = 44,44444 N

Universidad de Castilla- La Mancha


Ciudad Real - Espaa

Draft

Por: Eduardo W. V. Chaves (2014)

MECNICA DEL MEDIO CONTINUO: PROBLEMAS RESUELTOS

500

6.2 Elasticidad Bidimensional


Ejemplo 6.24
a) Definir el estado de deformacin plana y el estado de tensin plana. b) Obtener las
relaciones ( ) y ( ) para el caso de deformacin plana y tensin plana. c) Dar ejemplos
prcticos donde se pueden aplicar el estado de tensin plana y de deformacin plana.
Solucin:
a.1) El caso de tensin plana se aplica a estructuras donde una de las dimensiones es mucho
menor que las otras dos y la carga se aplica segn el plano definido por las dos dimensiones
predominantes. En este caso el campo de tensiones de Cauchy tiene la siguiente
caracterstica i 3 = 3i = 0 .
a.2) El caso de deformacin plana se aplica a estructuras prismticas donde una de las
dimensiones (eje prismtico) es mucho mayor que las otras dos y la carga se aplica normal
al eje prismtico. En este caso el campo de deformaciones tiene la siguiente caracterstica
i 3 = 3i = 0 . Adems, para que estas condiciones se cumplan, el material y la carga no
deben cambiar a lo largo del eje prismtico.
b.1 Estado de Tensin Plana) En el caso de tensin plana, las componentes del tensor
de tensin sern:
11
ij = 12
0

12
22
0

0 x
0 = xy
0 0

xy
y
0

0
0
0

(6.40)

Partimos de la ecuacin de deformacin:


=

Tr ( )1 +

2
2 (3 + 2 )

ij =

Tr ( ) ij +
ij
2
2 (3 + 2 )

y su traza viene dada por:


:1 =

1
3
1
1
Tr ( )1 : 1 +
:1 =
Tr ( ) +
Tr ( ) =
Tr ( )
2 (3 + 2 )
2
2 (3 + 2 )
2
(3 + 2 )

Tr ( ) =

1
Tr ( )
(3 + 2 )

Tr ( ) = (3 + 2 ) Tr ( )

La componente 33 deja de ser incgnita ya que:


33 =

1
Tr ( ) 33 +
Tr ( )
33 =
{
{
2 (3 + 2 )
2 = 0 2 (3 + 2 )
=1

33 =

(3 + 2 ) Tr ( ) =
Tr ( ) =
Tr ( )
2 (3 + 2 )
2 (3 + 2 )
2

33 =

( 11 + 22 + 33 )
2

33 =

( 11 + 22 )
( + 2 )

Universidad de Castilla- La Mancha


Ciudad Real - Espaa

Draft

(6.41)

Por: Eduardo W. V. Chaves (2014)

6 ELASTICIDAD LINEAL

501

Las componentes del tensor de tensiones ij = Tr ( ) ij + 2 ij quedan:


0
11 12
1 0 0

ij = (11 + 22 + 33 ) 0 1 0 + 2 12 22 0
0
0 0 1
0 33
11
1 0 0
2

( 11 + 22 )]0 1 0 + 2 12
= [(11 + 22 ) +
( + 2 )
0
0 0 1
0
11 12
1 0 0
2

( 11 + 22 ) 0 1 0 + 2 12 22 0
=
( + 2 )
0
0 0 1
0 33

12
22
0

0
0
33

(6.42)

Las ecuaciones anteriores pueden ser escritas en notacin indicial como:

;
ij = ( + 2 ) Tr ( ) ij + 2 ij

1
=
ij
Tr ( ) ij +
ij 2 (3 + 2 )
2

(i, j = 1,2)

con

Tr ( ) = 11 + 22

(6.43)
(i, j = 1,2,3)

(lo mismo que 3D)

E
E

; (i, j = 1,2)
con
Tr ( ) = 11 + 22
ij = (1 2 ) Tr ( ) ij + (1 + ) ij

= Tr ( ) + (1 + )
(i, j = 1,2,3)
(lo mismo que 3D)
ij
ij
ij
E
E

(6.44)

La ecuacin (6.42) tambin puede ser reescrita como:


4 ( + )

2
11 +
22

( + 2 )

( + 2 )

ij =
2 12

2 12
4 ( + )

22 +
11
( + 2 )
( + 2 )

Teniendo en cuenta las relaciones entre variables mecnicas podemos decir que:
4 ( + )
E
2
E
E
E (1 )
E (1 )
,
, 2 =
, luego:
=
=
=
=
2
2
(1 + ) (1 + )(1 ) (1 2 )
( + 2 )
(1 ) ( + 2 ) (1 )

( + ) (1 )
0
22
12

11
E

ij =
(
1

)
0

12
22
11
(1 2 )

0
0
0

Solucin alternativa: Notacin de Voigt y notacin ingenieril


Considerando las condiciones i 3 = 3i = 0 , la relacin ( ) queda:

Universidad de Castilla- La Mancha


Ciudad Real - Espaa

Draft

Por: Eduardo W. V. Chaves (2014)

MECNICA DEL MEDIO CONTINUO: PROBLEMAS RESUELTOS

502

1
x E

y
E

z
= E
xy 0

yz 0

zx
0

E
1
E

E
1
E

1
G

1
G

y
0

xy
0
yz
0

zx

(6.45)

Eliminando columnas y filas correspondientes a la tensin cero, la relacin de deformacin,


( ) , para el caso de tensin plana viene dada por:
1
x E

y = E
xy
0

E
1
E
0

0

x
1
x G= E
1
2 (1+ )
0 y y =
E

xy
0
1 xy

G

1
0

x

0 y
2(1 + ) xy
0

(6.46)

La inversa de la relacin anterior resultar la ley de Hooke, ( ) , para el caso de tensin


plana:
x
E

y = 1 2
xy

0 x
1

1
0 y

1
0 0
xy
2

(6.47)

Notar que en el estado plano de tensin la componente de la deformacin z no resultar


ser igual a cero, ya que la deformacin normal z no solo depende de la tensin normal a
esta direccin, i.e.:
z =

)]

[ (

1
1
z x + y = x + y
E
E

)]

(6.48)

Las componentes del tensor de deformaciones quedaran representadas por:


x

ij = 12 xy
0

xy

1
2

y
0

0
z

(6.49)

b.2 Estado de Deformacin Plana)


En el caso de deformacin plana se cumple que:
11
ij = 12
0

12
22
0

0 x

0 = 12 xy
0 0

1
2

xy
y
0

0
0

(6.50)

Partimos de la ecuacin de tensin:

Universidad de Castilla- La Mancha


Ciudad Real - Espaa

Draft

Por: Eduardo W. V. Chaves (2014)

6 ELASTICIDAD LINEAL

= Tr ( )1 + 2

503

ij = Tr ( ) ij + 2 ij

y su traza puede ser obtenida como sigue:


: 1 = Tr ( )1 : 1 + 2 : 1
Tr ( ) =

Tr ( ) = 3Tr ( ) + 2 Tr ( ) = [3 + 2 ]Tr ( )

+ 22 + 33
Tr ( )
= 11
3 + 2
3 + 2

La componente 33 deja de ser incgnita ya que:


ij = Tr ( ) ij + 2 ij 33 = Tr ( ) 33 + 2 33 33 = Tr ( )

(6.51)

Luego, la componente 33 queda:

33 = Tr ( ) =
33

3 + 2

3 + 2

33 =

(11 + 22 + 33 )

(11 + 22 )

3 + 2

=
(11 + 22 )
33 1
3 + 2 3 + 2
33 =

(11 + 22 )
2( + )

Las componentes de deformacin ij =

ij resultan:
Tr ( ) ij +
2 (3 + 2 )
2

0
11 12
1 0 0
1

(11 + 22 + 33 ) 0 1 0 +
0
12 22
ij =

2 (3 + 2 )
2
0
0 0 1
0 33
1 0 0
11

11 + 22 +
(11 + 22 ) 0 1 0 + 12
=
2 (3 + 2 )
2( + )
0 0 1 2 0

0
1 0 0
11 12
1

(11 + 22 ) 0 1 0 +
0
=
12 22

4 ( + )
2
0 0 1
0
0 33

12
22
0

0
0 (6.52)
33

En notacin indicial la ecuacin anterior queda:

;
ij = 4 ( + ) Tr ( ) ij + 2 ij

= Tr ( ) + 2
(i, j = 1,2,3)
ij
ij
ij

(i, j = 1,2)

Tr ( ) = 11 + 22

con

(6.53)

(lo mismo que 3D )

(1 + )
(1 + )

ij
;
Tr ( ) ij +
ij =
E
E

E
E
ij =
ij
Tr ( ) ij +

(1 + )(1 2 )
(1 + )

(i, j = 1,2)
(i, j = 1,2,3)

con

Tr ( ) = 11 + 22

(6.54)
(lo mismo que 3D)

La ecuacin en (6.52) puede ser reescrita como:

Universidad de Castilla- La Mancha


Ciudad Real - Espaa

Draft

Por: Eduardo W. V. Chaves (2014)

MECNICA DEL MEDIO CONTINUO: PROBLEMAS RESUELTOS

504

(2 + )

11 +
22

4 ( + )

4 ( + )

1
ij =
12
2

1
12
2
(2 + )

22 +
11
4 ( + )
4 ( + )

Teniendo en cuenta las relaciones entre parmetros mecnicos podemos decir que:
( + 2 )
(1 + )(1 )
(1 + )

(1 + ) 1
,
,
, luego:
=
=
=
4 ( + )
E
4 ( + )
E
2
E

(1 )
11
22
(1 + )
ij =
12

12
(1 ) 22 11
0

Solucin alternativa: Notacin de Voigt y notacin ingenieril


Partiendo de la ley de Hooke generalizada y eliminando columnas y filas correspondientes a
deformaciones cero, i.e.:
x

y
z

xy

yz

zx

E
0
0
0
=

(
1
)(
1
2
)
+

0
0

0
0
0

0
0
1 2
2

0
0
0

1 2
2

x
0 y

0 z

0 xy

0 yz

1 2 zx
2
0

(6.55)

resultando:

1
E

1
y = (1 + )(1 2 )

xy
0
0


x
0 y
1 2
xy
2
0

(6.56)

La tensin segn la direccin z viene dada por:


z =

E
x + y
(1 + )(1 2 )

(6.57)

0 x

0 y
2 xy

(6.58)

La inversa de la relacin (6.56) es:


x
1+
y = E
xy

Universidad de Castilla- La Mancha


Ciudad Real - Espaa

Draft

Por: Eduardo W. V. Chaves (2014)

6 ELASTICIDAD LINEAL

505

c.1 Ejemplos de aplicacin del estado de tensin plana)


2D
q

Figura 6.15: Viga de gran canto.


c.2 Ejemplo de aplicacin del estado de deformacin plana)
y
p - presin

2D
y
x
p

Seccin
por unidad de
longitud

Figura 6.16: Cilindro bajo presin.

Universidad de Castilla- La Mancha


Ciudad Real - Espaa

Draft

Por: Eduardo W. V. Chaves (2014)

506

MECNICA DEL MEDIO CONTINUO: PROBLEMAS RESUELTOS

2D

Figura 6.17: Tnel.

2D
1

Seccin de la presa

Figura 6.18: Presa.

Universidad de Castilla- La Mancha


Ciudad Real - Espaa

Draft

Por: Eduardo W. V. Chaves (2014)

6 ELASTICIDAD LINEAL

507

Ejemplo 6.25
En la Figura 6.19(a) se representa un dispositivo de apoyo en fajan de una mquina. Dicho
aparato de apoyo est constituido por un bloque de neopreno de dimensiones ( 50 20cm ),
representado en la Figura 6.19(b) por el elemento ABCD.
y

a)

b)

1,1

D
A

1,2

A
1

20

1,1

50

Dimensiones en centmetros - cm
Figura 6.19
Bajo accin de cargas vertical y horizontal que transmite la mquina al apoyo el bloque de
neopreno se deforma como se indica en la Figura 6.19(b) (ABCD) y an se puede
considerar que el campo de desplazamiento ( u, v) est dado por unas ecuaciones lineales
del tipo:
u = a1 x + b1 y + c1
v = a2 x + b2 y + c2

donde a1 , b1 , c1 , a 2 , b2 , c 2 son constantes a determinar.


Hiptesis:
1 Material elstico lineal e istropo con el Mdulo de elasticidad longitudinal igual a
1000 N / cm 2 y el mdulo de elasticidad transversal igual a

1
N / cm 2 .
0,0028

2 Se supondr que se trata de un estado de deformacin plana.


a) Calcular las componentes del tensor de deformacin y la deformacin volumtrica en
cualquier punto;
b) Calcular las tensiones en cualquier punto;
c) Mxima tensin normal;
d) Determinar el Alargamiento unitario en la direccin de la diagonal AC .
Solucin:
u = a1 x + b1 y + c1

v = a 2 x + b2 y + c 2

(6.59)

Segn Figura 6.19 sacamos que:

Universidad de Castilla- La Mancha


Ciudad Real - Espaa

Draft

Por: Eduardo W. V. Chaves (2014)

MECNICA DEL MEDIO CONTINUO: PROBLEMAS RESUELTOS

508

u (0;0) = 1 = c1

u (50;0) = 1,1 = 50a1 + 1 a1 = 0,002


u (0;20) = 1,1 = 20b1 + 1 b1 = 0,005

u = 0,002 x + 0,005 y + 1

(6.60)

Para desplazamiento vertical:


v(0;0) = 0 = c 2

u (50;0) = 0 = 50a 2 a 2 = 0

u (0;20) = 1 = 20b2 b2 = 0,05

v = 0,05 y

(6.61)

Luego:
u = 0,002 x + 0,005 y + 1

v = 0,05 y

a)

(6.62)

Deformaciones
x =

u
= 0,002
x

y =

v
= 0,05
y

xy =

u v
+
= 0,005
y x

(6.63)

Deformacin volumtrica lineal (pequeas deformaciones):


DVL = V = x + y + z = I = 0,048

b)
G=

(6.64)

Tensiones
E
E
=
1 = 0,4
2(1 + )
2G

E
(1 ) x + y = 3571,4286 [(0,6) 0,002 0,4 0,05] = 67,1428
(1 + )(1 2 )
E
y =
(1 ) y + x = 3571,4286 [(0,6) (0,05) + 0,4 0,002] = 104,2857
(1 + )(1 2 )
1
xy = G xy =
0,005 = 1,785714
0,0028
x =

Una solucin alternativa es utilizar: ij =


x

ij = 12 xy
1 xz
2

1
2

xy
y

1
2

yz

1
2
1
2

xz 0,002

yz = 12 (0,005)
z
0

1
2

E Tr ( )
E
ij , donde:
ij +
(1 + )(1 2 )
(1 + )
(0,005) 0

0,05 0
0
0

N
E Tr ( )
E
N
= 714,285714 2
= 68,571429 2 ,

(
1
+
)
(1 + )(1 2 )
cm
cm

0,002
1 0 0

ij = 68,5714290 1 0 + 714,285714 12 (0,005)

0 0 1
0

1
2

(0,005) 0

0,05 0
0
0

Resultando

Universidad de Castilla- La Mancha


Ciudad Real - Espaa

Draft

Por: Eduardo W. V. Chaves (2014)

6 ELASTICIDAD LINEAL

509

0
67,1428 1,785714
N

ij 1,785714 104,2857
0
cm 2

68,571
0
0

c)

Tensiones principales
(1,2) =

x + y
2

x y

2

+ 2xy

(6.65)
2

(1,2)

67,1428 104,2857
67,1428 + 104,2857
2
=

+ 5,35714
2
2

= 171,4285 19,328675

(6.66)

1 = 152,099824 N cm 2

2
2 = 190,757175 N cm

d)

(6.67)

Alargamiento unitario

La diagonal ( AC ) mide inicialmente:


(6.68)

L0 = AC = 50 2 + 20 2 = 53,852cm

Diagonal deformada
AC = 50,2 2 + 19 2 = 53,675cm

L = AC AC = 0,177cm

(6.69)

El alargamiento unitario es:


=

L 0,177
=
= 0,0033
L0
53,852

(6.70)

Ejemplo 6.26
En un punto de un suelo que podemos considerar como un slido elstico lineal e
istropo se conoce la deformacin volumtrica V = 2 10 3 , la deformacin tangencial
12 = 3 10 3 y la deformacin horizontal que es nula 11 = 0 . El suelo est sometido a
un estado de deformacin plana en el plano x1 x 2 . Se pide:
a) Componentes cartesianas del tensor de deformacin. Obtener las deformaciones
principales y la orientacin de las mismas, definiendo el ngulo que forman con lo sejes
( x1 , x 2 , x 3 ) .
b) Suponiendo que las constante elsticas son E = 50MPa , =

1
, obtener las
4

componentes del tensor de tensiones y sus valores principales. Obtener asimismo las
direcciones en las que las tensiones normales y tangenciales son mximas o mnimas y sus
valores.
c) Obtener la densidad de energa de deformacin por unidad de volumen.
Solucin:
a) Las componentes del tensor de deformacin infinitesimal son:

Universidad de Castilla- La Mancha


Ciudad Real - Espaa

Draft

Por: Eduardo W. V. Chaves (2014)

MECNICA DEL MEDIO CONTINUO: PROBLEMAS RESUELTOS

510

ij = 3 10 3

0
0

3 10 3
22
0

Deformacin volumtrica DVL V = I = 11 + 22 + 33 = 2 10 3 22 = 2 10 3 . Con


lo cual:
0
3 0
0

3
plana
3
n
ij =
ij = 3 2 0 10 3 deformaci
10
3 2
0
0
0

Deformaciones principales:
0 3
=0
3 2

2 + 2 3 = 0

1 = 1

2 = 3

Las deformaciones principales son:


1 = 1 10 3

2 = 3 10 3

b)

1 10 3
ij =
0

0
3
3 10

y
1
xy

yy

Observemos que el radio es R = (1 (3)) / 2 = 2 . Luego:


tan(2) =

3
1

2 = arctan( 3 )

b) Aplicando ij = Tr ( ) ij + 2 ij , donde =

= 30

E
= 20 MPa ,
(1 + )(1 2 )

E
= 20MPa , Tr ( ) = 2 10 3 . Luego:
2(1 + )
0
3 0
1 0 0

ij = Tr ( ) 0 1 0 + 2 3 2 0 10 3
0
0 0 1
0
0

Universidad de Castilla- La Mancha


Ciudad Real - Espaa

Draft

Por: Eduardo W. V. Chaves (2014)

6 ELASTICIDAD LINEAL

511

Resultando
40
0
40
3 0
40 3
0
0
0

3
ij = 0
40
MPa
= 40 3 120
0 + 40 3 2 0 10
0 kPa
1
4
2
4
3

0
0
=103 Pa
0
40
40
0
0
0
0

Crculo de Mohr
S =

(10 3 )
2

( N = 0; S = 3 )
2

I =1

III = 3

N 10 3

( N = 0; S = 2)
( N = 2; S = 3 )

Como el material es istropo, las direcciones principales de las tensiones coinciden con las
direcciones principales de las deformaciones. Y adems, recordemos que los autovalores de
y estn relacionados, cuya expresin se demuestra a continuacin.
Reemplazando el valor de = Tr ( )1 + 2 en la definicin de autovalor, autovector:
n = n

(Tr()1 + 2 ) n = n
Tr ( )n + 2 n = n
2 n = ( Tr ( ) )n

Tr ( )1 n + 2 n = n

2 n = n Tr ( )n

Tr ( )
n = n
n =
2

Luego:
=

Tr ( )
2

= 2 + Tr ( )

Pudiendo as obtener los autovalores de :


(1) I = 2 (1) + Tr ( ) = (40 10 6 ) (1 10 3 ) + (20 10 6 ) (2 10 3 ) = 0
( 2 ) II = 2 (2 ) + Tr ( ) = (40 10 6 ) (0) + (20 10 6 ) (2 10 3 ) = 40 10 3 Pa
(3) III = 2 (3) + Tr ( ) = (40 10 6 ) (3 10 3 ) + (20 10 6 ) (2 10 3 ) = 160 10 3 Pa

Tambin podemos utilizar la expresin = Tr ( )1 + 2 en el espacio principal:


Universidad de Castilla- La Mancha
Ciudad Real - Espaa

Draft

Por: Eduardo W. V. Chaves (2014)

MECNICA DEL MEDIO CONTINUO: PROBLEMAS RESUELTOS

512

40
0
0
0
0
0
1 0 0

3
MPa
ij = 0
40
0 + 40 0 3 0 10
0 kPa
142
4
3 = 0 160
0
=103 Pa
0
0 0 0
40
40
0
0

Crculo de Mohr en tensiones:

160

40

N (kPa)

0
1
2

c) La densidad de energa de deformacin e = : . Si utilizamos el espacio principal


tenemos que:
1
0

0
0
0
= 0 160
0 10 3 Pa

3 0
40
0

0
2

0
0

1
0

0
2
0

0 1 0 0
0 = 0 3 0 10 3
3 0 0 0

m
m

con lo cual
1
1
1
2
2
2
m
N m
J
1
= (0)(1) + ( 160 10 3 )(3 10 3 ) + (40 10 3 )(0) = 240 Pa = 240 2
= 240 3
m
2
m m
m

e = ij ij = ij ij = (1 1 + 2 2 + 3 3 )

Ejemplo 6.27
Un slido se halla sometido a deformacin plana, siendo las componentes del tensor de
deformacin lineal en un determinado punto:
3 0
2

ij = 3 10 0 10 3
0
0
0

Considrese que el slido tiene un comportamiento elstico lineal e istropo con las
propiedades mecnicas: mdulo elstico de Young E = 10MPa y coeficiente de Poisson
= 0,25 .
a) Obtener la deformacin volumtrica y el tensor de deformacin desviadora;
b) Obtener las deformaciones principales y las direcciones en que se producen;
c) Obtener las componentes del tensor de tensiones de Cauchy;
d) Obtener las mximas y mnimas tensiones normales;

Universidad de Castilla- La Mancha


Ciudad Real - Espaa

Draft

Por: Eduardo W. V. Chaves (2014)

6 ELASTICIDAD LINEAL

513

e) Se sabe que el material rompe cuando en algn plano se alcanza una tensin tangencial
que supere 40 kPa . Verificar si se produce la rotura.
Solucin:
a) Deformacin volumtrica ( V ):
V = I = Tr ( ) = ( 2 10) 10 3 = 12 10 3

Descomposicin aditiva del tensor de deformacin = esf + dev , donde la parte esfrica:
ijesf

0
4 0
Tr ( )

ij = 0 4 0 10 3
=
3
0
0 4

y la parte desviadora queda:


ijdev

= ij

ijesf

2
0
3
0 4 0
2 3 0

3
= 3 10 0 0 4 0 10 = 3 6 0 10 3
0
0 0 4
0 4
0
0 0

b) Las deformaciones principales obtenemos al resolver el determinante caracterstico:


2
3
=0
3
10

2 + 12 + 11 = 0

Solucin de la ecuacin cuadrtica:


(1, 2 ) =

(12) (12) 2 4(1)(11)


2(1)

12 10
2

(1) = 1,0

( 2 ) = 11

Luego, las deformaciones principales son:


1 = 1,0 10 3

Direcciones principales ( ij ij )n (j ) = 0 i

2 = 11,0 10 3

(i, j = 1,2)

Direccin principal asociada al valor principal (1) = 1,0 :


(1)
(1)
(1)
(1)
3
2 (1)
n1(1) 0 n1 + 3n 2 = 0 n1 = 3n 2
=

10 (1) n (21) 0 3n1(1) 9n (21) = 0


3

restriccin n1(1) + n (21) = 1 , con eso obtenemos que (3n (21) ) 2 + n (21) = 1 n (21) =
n1(1) =

1
10

, y que

3
10

Direccin principal asociada al valor principal (1) = 11,0 :


( 2)
( 2)
3
2 (11)
n1( 2) 0 9n1 + 3n 2 = 0

3
10 (11) n (22) 0 3n1( 2 ) + n (22 ) = 0 n (22) = 3n1( 2 )

Con la restriccin n1( 2) + n (22) = 1 , obtenemos que n1( 2) =

1
10

, y que n (22) =

3
10

Resumiendo as que:
Universidad de Castilla- La Mancha
Ciudad Real - Espaa

Draft

Por: Eduardo W. V. Chaves (2014)

MECNICA DEL MEDIO CONTINUO: PROBLEMAS RESUELTOS

514

1 = 1 10 3
2 = 11 10 3

principal
direccin

direccin principal

1 = 0

3
n (i1) =
10

principal
direccin

n i(3)

1
10

1
n i( 2 ) =
10

3
10

= [0 0 1]

c) Las componentes del tensor de tensiones de Cauchy vienen dadas por:


ij = Tr ( ) ij + 2 ij

donde =

E
(1 + )(1 2 )

= 4 MPa , = G =

E
= 4 MPa , Tr ( ) = 12 10 3 :
2(1 + )

3 0
0
1 0 0
2
64 24

3
ij = 4 (12) 0 1 0 + 2 (4) 3 10 0 10 MPa = 24 128
0 kPa

0 0 1
0
0
48
0 0
0

Como el material es istropo las direcciones principales de las tensiones y deformaciones


coinciden. Las tensiones principales obtenemos trabajando en el espacio principal
ij = Tr ( ) ij + 2 ij :

0
0
56
1 0 0
1 0 0

3
ij = 4 (12) 0 1 0 + 2 (4) 0 11 0 10 MPa = 0
136
0 kPa

0
0 0 1
0
48
0 0
0

d) Dibujamos el crculo de Mohr en tensiones con I = 48kPa , II = 56kPa ,


III = 136kPa :
S (kPa)

S max = 44

II = 56
III = 136

I = 48

N (kPa)

Podemos obtener la tensin tangencial mxima a travs de su definicin:


S max =

I III
(48) (136)
=
= 44kPa
2
2

Con lo cual podemos comprobar que el material alcanzar la rotura.

Universidad de Castilla- La Mancha


Ciudad Real - Espaa

Draft

Por: Eduardo W. V. Chaves (2014)

6 ELASTICIDAD LINEAL

515

Ejemplo 6.28
Una roseta a 45 , como se indica en la Figura 6.20, que se utiliza para calcular la
deformacin en una parte de una estructura, proporciona las siguientes lecturas:
x = 0,33 10 3

x = 0,22 10 3

y = 0,05 10 3

Cul es la tensin de corte mxima en el punto en cuestin? Sabiendo que el material


(elstico lineal e istropo) que constituye la estructura tiene las siguientes propiedades
elsticas: E = 29000 Pa (Mdulo de Young); = 0,3 (Coeficiente de Poisson). Se pide:
a) Determinar las deformaciones principales y las direcciones principales de las
deformaciones;
b) Determinar las tensiones principales y las direcciones principales de las tensiones.
c) Que conclusin se puede sacar de las direcciones principales de las tensiones y de las
deformaciones?
Nota: Considerar el caso de deformacin plana y el material elstico, lineal e istropo.
y
x

45
45

Figura 6.20: Roseta.


Solucin:
Primero tenemos que obtener las componentes del tensor de deformacin en el sistema
x, y, z . Para ello, utilizaremos la ley de transformacin de coordenadas para obtener la
componente xy = 212 . Recordando que en el caso bidimensional la componente normal
puede obtenerse como:
=
11

11 + 22 11 22
+
cos( 2) + 12 sin( 2)
2
2

(6.71)

cuya expresin fue obtenido a travs de la transformacin de coordenadas:

11

12
0

12
22
0

0 cos sin 0 11
0 = sin cos 0 12
0
1 0
0 0

12
22
0

0 cos sin 0
0 sin cos 0
0 0
0
1

Es interesante ver tambin el Ejemplo 1.99 (captulo 1)


La expresin (6.71) en notacin ingenieril:
x =

x + y

Universidad de Castilla- La Mancha


Ciudad Real - Espaa

x y
2

Draft

cos( 2) +

xy
2

sin( 2)

Por: Eduardo W. V. Chaves (2014)

516

MECNICA DEL MEDIO CONTINUO: PROBLEMAS RESUELTOS

Despejando xy obtenemos:
xy

x x cos 2 ( 2) y sin 2 ( 2)
= 0,16 10 3
= 2

sin( 2)

Luego:
0,33 0,08 0
ij = 0,08 0,05 0 10 3
0
0
0

Las tensiones:

E
(1 2 ) x + y = 12,0462 Pa
(1 + )(1 2 )
E
(1 2 ) y + x = 3,5692 Pa
y =
(1 + )(1 2 )
E
xy =
xy = 1,7846 Pa
2(1 + )
E
z =
x + y = 4,684 Pa
(1 + )(1 2 )
x =

Tensin de corte mxima:


x + y
max =
2

+ 2xy = 4,5988 Pa

a) La ecuacin caracterstica para el tensor de deformacin es:


2 0,28 2,29 10 2 = 0

( 10 3 )

Los autovalores (las deformaciones principales) vienen dados por:


1 = 0,346155 10 3

2 = 0,06615528 10 3

Los autovectores del tensor de deformacin:


a 1
Autovector
asociado

0,9802 0,1979 0
Autovector asociado a 2
0,1979 0,9802 0
Autovector asociado a
3 0
0
1

b) Dadas las componentes del tensor de tensin:


12,0462 1,7846
ij = 1,7846 3,5692
0
0

0
0 Pa
4,684

A travs del determinante caracterstico podemos obtener los autovalores, tensiones


principales:
1 = 12,40654

2 = 3,208843

3 = 4,684

Los autovectores del tensor de tensiones son:

Universidad de Castilla- La Mancha


Ciudad Real - Espaa

Draft

Por: Eduardo W. V. Chaves (2014)

6 ELASTICIDAD LINEAL

517

a 1
Autovector
asociado

0,9802 0,1979 0
Autovector asociado a 2
0,1979 0,9802 0
Autovector asociado a
3 0
0
1

Comparando los autovectores del tensor de tensiones y de deformaciones concluimos que


son los mismos.
Las direcciones principales de tensin y deformacin son coincidentes solo para el caso de
material istropo.
b) Solucin Alternativa para la Componentes del tensor de tensiones:
Conocidas las componentes del tensor de deformaciones:
0,33 0,08 0
ij = 0,08 0,05 0 10 3
0
0
0

Aplicamos la ecuacin constitutiva: ij = Tr ( ) ij + 2 ij , donde las constantes de Lam


vienen dadas por:

= 16,7307692 103 Pa
(1 + )(1 2 )
E
=
= 11,15384615 103 Pa
2(1 + )

y Tr ( ) = 0,27999972 10 3 0,28 10 3 , con lo cual: ij = Tr ( ) ij + 2 ij viene dado


por:
1 0 0
11 12 13
1 0 0
0,33 0,08 0

ij = Tr ( ) 0 1 0 + 2 12 22 23 = Tr ( ) 0 1 0 + 2 0,08 0,05 0 10 3
0 0 1
13 23 33
0 0 1
0
0
0
0
12,0461 1,784615

0 Pa
= 1,784615 3,5692

0
0
4,6846

Como el material es istropo y comparten las mismas direcciones principales, luego


podemos utilizar la misma expresin ij = Tr ( ) ij + 2 ij en el espacio principal, i.e.:
1 0 0
1 0 0

ij = Tr ( ) 0 1 0 + 2 0 2 0
0 0 1
0 0 3
0
0
0
0
1 0 0
0,346155
12,40752

3
0
0
3,20783
0 Pa
= Tr ( ) 0 1 0 + 2
0,0662 0 10 =
0 0 1

0
0
0
0
0
4,6846

Universidad de Castilla- La Mancha


Ciudad Real - Espaa

Draft

Por: Eduardo W. V. Chaves (2014)

MECNICA DEL MEDIO CONTINUO: PROBLEMAS RESUELTOS

518

Ejemplo 6.29
Una delta de roseta (aparato para obtener la deformacin) tiene la forma de un tringulo
equiltero, y registra deformaciones longitudinales en las direcciones x1 , x1 y x1 como se
muestra en la Figura 21.
x2

x1

x1

30
60
30

60

60

x1

Figura 21
Si las deformaciones calculadas en estas direcciones son:
11 = 4 10 4

= 110 4
11

= 4 10 4
11

+ 22 .
Determinar 22 = y , 212 = xy , 22 y . Mostrar que 11 + 22 = 11

Hiptesis: Considerar caso de deformacin plana.


Solucin:
Utilizando la ley de transformacin de las componentes de un tensor de segundo orden que
es independiente de una direccin, podemos decir que se cumple que:
=
11

11 + 22 11 22
+
cos( 21 ) + 12 sin( 21 )
2
2

(6.72)

=
11

11 + 22 11 22
+
cos( 2 2 ) + 12 sin( 2 2 )
2
2

(6.73)

donde 1 = 60 y 2 = 120 . Luego, combinando las expresiones anteriores, eliminamos


12 , resultando:
22 =


2
+ 11
11 = 4,66667 10 4
11
3
2

Una vez obtenido el valor de 22 = 4,66667 10 4 , podemos reemplazar en la ecuacin


(6.72) y obtenemos que:
xy = 2 12 =

1
3

(4 11 11 3 22 ) = 3,46410 10 4

12 = 1,73205 10 4

Para obtener 22 , primero determinando el ngulo de giro con respecto a x1 que es


3 = 60 +90 = 150 , resultando:

Universidad de Castilla- La Mancha


Ciudad Real - Espaa

Draft

Por: Eduardo W. V. Chaves (2014)

6 ELASTICIDAD LINEAL

22 =

519

11 + 22 11 22
+
cos( 2 3 ) + 12 sin( 2 3 ) = 0,33333 10 4
2
2

+ 22 = 0,66667 10 4 . Recordar que la traza Tr ( ) es


Comprobando as que 11 + 22 = 11
un invariante.

Ejemplo 6.30
Considrese una seccin transversal de una presa que presenta el campo de desplazamiento
dado por:
y, v

u ( x, y ) = 4 x 2 y 2 + 2 xy + 2

v ( x, y ) = 4 y 2 x 2 + 2 xy + 5

x, u

El material elstico lineal e istropo que constituye dicha estructura presenta las siguientes
propiedades mecnicas: E = 100 MPa , G = 35,7 MPa , = 0,4 y est sometido a un nivel de
carga tal que se puede considerar que est en el rgimen de pequeas deformaciones.
a) Obtener el campo de tensin;
b) Demostrar que si se cumplen las ecuaciones de equilibrio para el campo de
desplazamiento dado.
Solucin:
a) Clculo de las componentes del tensor de deformacin:
x =

u
= 8 x + 2 y
x

y =

v
= 8 y + 2 x
y

xy =

u v
+
=0
y x

Luego, las componentes del tensor de deformacin quedan:


8 x + 2 y
ij =
0

0
0
8 y + 2 x 0
0
0

b) Para una presa, como ya hemos visto, podemos analizarla segn la aproximacin del
estado de deformacin plana:

x
1
E

y = (1 + )(1 2 )
xy
0

1
0

0 x
0,6 0,4 0 8 x + 2 y

0 y = 357 ,1428 0,4 0,6 0 8 y + 2 x MPa


1
0

0 0,3
0
xy
2

x
4 x 2 y

y = 357 ,1428 2 x 4 y MPa
xy

Universidad de Castilla- La Mancha


Ciudad Real - Espaa

Draft

Por: Eduardo W. V. Chaves (2014)

MECNICA DEL MEDIO CONTINUO: PROBLEMAS RESUELTOS

520

z =

E
x + y = 357 ,1428 [( 8 x + 2 y ) + ( 8 y + 2 x )]
(1 + )(1 2 )

Las ecuaciones de equilibro quedan:


x xy xz
+
+
+ b x = 0

y
z
x
xy y yz
+
+
+ b y = 0

y
z
x

yz z
xz +
+ b z = 0
+
z
y
x

4 + 0 + 0 + 0 0 No cumple

0 4 + 0 + 0 0 No cumple

z
0 + 0 +
+0=0
z

Lo que indica que el campo de desplazamientos dado no cumple las ecuaciones de


equilibrio.
Ejemplo 6.31
Una presa de gravedad de seccin transversal triangular est construida mediante hormign
de peso especfico

5
( es el peso especfico del agua), siendo su forma y dimensiones
2

transversales las indicadas en la Figura 6.22. La solucin de tensiones (campo de tensin)


para este problema de deformacin plana es conocida y viene dada por:
11 = x 2

22 =

Considerar: Coeficiente de Poisson: =

( x1 3 x 2 )
2

12 = x1

1
; Mdulo de elasticidad longitudinal E .
4

a) Hacer la representacin grfica de las fuerzas de superficies que debe ejercer el terreno
sobre el lado AB , para que la solucin indicada sea la correcta;
b) Obtener las tensiones principales en los puntos A y B . A partir del crculo del Mohr en
tensiones, obtener los valores extremos de las tensiones en los respectivos puntos.
c) Obtener el campo de deformacin en la presa.
x1

O
45

= g

g -aceleracin de la gravedad

- densidad de masa

[ ] =

kg m
N
= 3
3
2
m s
m

x2

Figura 6.22.

Universidad de Castilla- La Mancha


Ciudad Real - Espaa

Draft

Por: Eduardo W. V. Chaves (2014)

6 ELASTICIDAD LINEAL

521

NOTA: Aunque en la literatura se denomina de peso especfico, en realidad es el


r

mdulo de la fuerza msica por unidad de volumen, i.e. = p = b = g , donde b es la

[r ]

N
m
= 2 . Recordar que, en el Sistema Internacional
kg s

fuerza msica por unidad de masa b =

de Unidades el trmino especfico se refiere siempre a por unidad de masa, que no es el caso
de , el trmino correcto sera densidad de peso, por decir algo, ya que en el SI el trmino
densidad siempre se refiere a por unidad de volumen.
Solucin:
a) Campo de tensin y de deformacin en la presa:
x 2

ij = x1

x1
0

( x1 3 x 2 ) 0
2

0
33

11
ij = 12
0

12
22
0

0
0
0

Obtenemos la fuerza de superficie a travs del vector traccin t (n) = n . Para el lado AB
tenemos como normal el vector n i = [0,1,0] :

t 1 ( AB ) x 2
( AB )
t 2
= x1
AB
(
)
t

3
0

x1

0 0 x1

( x1 3 x 2 ) 0 1 = ( x1 3 x 2 )
2

33 0
0
0

Fuerza de superficie en la base de la presa:


h

( AB )

3h
2

t1

t2

(segn direccin x1 )

( AB )

(segn direccin x 2 )
x1

O
45

2
B

t (n)

x2

Universidad de Castilla- La Mancha


Ciudad Real - Espaa

Draft

Por: Eduardo W. V. Chaves (2014)

MECNICA DEL MEDIO CONTINUO: PROBLEMAS RESUELTOS

522

b) Notar que 33 ya es una tensin principal. Partiendo de = Tr ( )1 + 2 podemos


obtener 33 :
ij = Tr ( ) ij + 2 ij 33 = Tr ( ) 33 + 2 33 33 = Tr ( )

A continuacin determinamos Tr ( ) . Para ellos hacemos el doble producto escalar de


= Tr ( )1 + 2 con el tensor identidad de segundo orden, resultando:
: 1 = Tr ( )1 : 1 + 2 : 1
Tr ( ) =

Tr ( ) = 3Tr ( ) + 2 Tr ( ) = [3 + 2 ]Tr ( )

+ 22 + 33
Tr ( )
= 11
3 + 2
3 + 2

Luego la componente 33 queda definida como:


33 = Tr ( ) =
33

3 + 2

3 + 2

33 =

(11 + 22 + 33 )

3 + 2

(11 + 22 )

=
(11 + 22 )
33 1
3 + 2 3 + 2
33 =

(11 + 22 ) = (11 + 22 )
2( + )

Reemplazando los valores de 11 , 22 , obtenemos que:


33 =

(11 + 22 ) = x2 + ( x1 3x2 ) = [x1 5 x2 ] = [x1 5 x2 ]


2( + )
2
2
8

donde hemos considerado que =

.
2( + )

El estado tensional en el punto A( x1 = 0; x 2 = h) viene dado por:

ij( A)

x 2

= x1

x1

( x1 3 x 2 )
2
0


h

0
= 0

[x1 5 x 2 ] 0
8

0

0
3h
2
0


0 1 0

3
0 =0
2

5h
0
0
8

0 h

5
8

Notar que estas componentes ya son las tensiones principales en el punto A .


Crculo de Mohr en tensiones en el punto A :
S ( h)

S max = 0,4375

1,5

Universidad de Castilla- La Mancha


Ciudad Real - Espaa

Draft

0,625

N (h)

Por: Eduardo W. V. Chaves (2014)

6 ELASTICIDAD LINEAL

523

El estado tensional en el punto B( x1 = h; x 2 = h) viene dado por:

ij( B )

x 2

= x1

x1

( x1 3 x 2 )
2
0


h

0
= h

[x1 5 x 2 ] 0
8

(h 3h)
2
0

1 1 0


0
= 1 1 0 h
1

0

[h 5h]
2
8

Las tensiones principales en el punto B( x1 = h; x 2 = h) vienen dadas por:


1
1
=0
1
1

(1 ) 2 1 = 0

(1 ) 2 = 1

( 1 ) = 1

1 = 2

2 = 0
S ( h)

S max = 1

0,5

N (h)

c) Podemos obtener la expresin del campo de deformacin partiendo de la expresin:


= Tr ( )1 + 2 :
= Tr ( )1 + 2

2 = Tr ( )1

Recordemos que anteriormente hemos obtenido que Tr ( ) =


=

Tr ( )1
2
2

Tr ( )
, luego:
3 + 2

1
1

Tr ( )1 =

Tr ( )1
2
2
2
2 (3 + 2 )

Tambin podemos expresar la relacin anterior en funcin de los parmetros E y :

=G=

(3 + 2 )
1
1
E
1
(1 + )
; E=

=
2(1 + )
2
E
+
(3 + 2 ) E ( + )

=
=
2 (3 + 2 ) 2 E ( + ) E

Luego:
=

Tr ( )1
2
2 (3 + 2 )

(1 + )

Tr ( )1
E
E

La traza de viene dada por:

Universidad de Castilla- La Mancha


Ciudad Real - Espaa

Draft

Por: Eduardo W. V. Chaves (2014)

MECNICA DEL MEDIO CONTINUO: PROBLEMAS RESUELTOS

524

Tr ( ) = 11 + 22 + 33


5
= ( x 2 ) + ( x1 3 x 2 ) + ( x1 5 x 2 ) = ( x1 5 x 2 )
2
8
8

Pudiendo as obtener las componentes del tensor de deformaciones con =


ij =

x 2
5
ij =
x1
4E
0

1
:
4

5
5
ij
( x1 5 x 2 ) ij
4E
32 E

x1

( x1 3 x 2 )
2
0

1 0 0

5
( x1 5 x 2 ) 0 1 0
0

32 E
0 0 1

[x1 5 x 2 ]
8

1
x1
0
8 ( x1 + 3 x 2 )

5
1

x1
(3 x1 + 7 x 2 ) 0
=
4E
8

0
0
0

Ejemplo 6.32
En el Ejemplo 5.12 hemos obtenido la Formulacin en Tensin para el caso de elasticidad
tridimensional. Obtener la formulacin equivalente para la elasticidad bidimensional
considerando el estado de deformacin plana y el estado de tensin plana.
Solucin:
Recordar que las ecuaciones de gobierno para el problema elstico son:
Notacin tensorial
Ecuaciones de Movimiento:
r
r
&r& (3 ecuaciones)
+ b = v& = u
Ecuacin Constitutiva en Tensin:

Notacin indicial
Ecuaciones de Movimiento:
&& i (3 ecuaciones)
ij , j + b i = u

( ) = Tr ( )1 + 2 (6 ecuaciones)

ij = kk ij + 2 ij (6 ecuaciones)

Ecuacin Constitutiva en Tensin:


(6.74)

Ecuaciones Cinemticas:

Ecuaciones Cinemticas:
r
= sym u (6 ecuaciones)

ij =

1 u i u j
+
2 x j x i

(6 ecuaciones)

Las ecuaciones cinemticas pueden ser reemplazadas por las ecuaciones de compatibilidad
(ver Ejemplo 5.10):
ij , kl + kl ,ij il , jk jk ,il = O ijkl

En el caso bidimensional las ecuaciones de compatibilidad (ver Ejemplo 5.10 NOTA 3)


se reducen a:
S33 =

2
2
212
2 x y xy
211 2 22
Notacin Ingenieril
2
0
=0

S
=
+
z
xy
x1x2
x 2
x22
x12
y 2

Universidad de Castilla- La Mancha


Ciudad Real - Espaa

Draft

(6.75)

Por: Eduardo W. V. Chaves (2014)

6 ELASTICIDAD LINEAL

525

Y las ecuaciones de movimiento se reducen a:


&&i
ij , j + bi = i1,1 + i 2, 2 + i 3,3 + bi = u
2D
&&i

i1,1 + i 2, 2 + bi = u

&&1
11,1 + 12, 2 + b1 = u

&&2
21,1 + 22, 2 + b 2 = u

(i = 1,2)

11 12
&&
x + x + b1 = u1 = a1
1
2

21 + 22 + b = u
&&2 = a2
2
x1
x2

o en notacin Ingenieril:
x xy
11 12
&&
+
+ b x = a x

x + x + b1 = u1
y
1
x
2
Notacin Ingenieril

21 + 22 + b = u
xy + y + b = a
&
&
2
2
y
y
x1
x
x2
y

Derivando la primera ecuacin con respecto a x y la segunda con respecto a y podemos


obtener que:
x xy
+
+ b x = a x

y
x

xy + y + b = a
y
y
x
y

2 x 2 xy

= ( a x b x )
2 +
xy x
x
2
2

xy y
xy + y 2 = y ( a y b y )

2 xy
2

= 2x + ( ax b x )

x
x
xy
2
2 y
xy
=

+ ( a y b y )
xy
y 2 y

Sumando las dos ecuaciones podemos decir que


2 xy

2 y
2 x
2
+ ( a y b y )
= 2 + ( a x b x )
xy
x
y 2 y
x

(6.76)

a) Estado de Tensin Plana


Para el caso de tensin plana las ecuaciones constitutivas fueron obtenidas en el Ejemplo
6.24 y el campo de deformacin (ver ecuacin (6.46)) viene dado por:
x
1
1
y = E
xy
0

1
0

x

y
2(1 + ) xy
0
0

x = E x E y

x + y
y =
E
E

2(1 + )
xy
xy =
E

Reemplazando las componentes de la deformacin en la ecuacin de compatibilidad


(ecuaciones cinemticas), y considerando el material homogneo, podemos obtener que:

Universidad de Castilla- La Mancha


Ciudad Real - Espaa

Draft

Por: Eduardo W. V. Chaves (2014)

MECNICA DEL MEDIO CONTINUO: PROBLEMAS RESUELTOS

526

2
2
2 x y xy
+

=0
xy
x 2
y 2

2
y 2

2
1
x y + 2
E x
E

2
1

2(1 + )

x + y
xy = 0

E
E
x
y
E

2
2
2
1 2 x y 2 x 1 y 2(1 + ) xy

=0
E y 2
E y 2
E x 2
E x 2
E
xy

(6.77)

2
2 xy
2 y
2 x y
2 x

2
(
1
+

)
=0

xy
x 2
y 2
x 2
y 2

Para considerar simultneamente las dos ecuaciones del movimiento utilizaremos la


ecuacin (6.76), y multiplicamos por 2(1 + ) para obtener:
2(1 + )

2 y

2 x

+ ( a y b y )

= (1 + )
+
(

)
+
(
1
+
)

x
x
2
2
y
xy
x
y

2 xy

Y reemplazando en la ecuacin (6.77) podemos obtener que:


2
2 xy
2 y
2 x y
2 x

2
(
1
)
+

+
=0

xy
x 2
x 2
y 2
y 2
2
2 y
2 x

2 x y
2 x
+ ( a x b x )
+

(
1
)

2
2
2
2
2
x
x
x
y
y
x

2 y
(1 + )
+ ( a y b y ) = 0
2
y

Simplificando la ecuacin anterior obtenemos que:


Formulacin en tensin 2D Estado de Tensin Plana
2
2

2 x 2 x y y

+
+
= (1 + ) ( a x b x ) + ( a y b y )
+
2
2
2
2
y
x
y
y
x
x

(6.78)

Para el caso esttico o casi esttico la ecuacin anterior queda:


Formulacin en tensin 2D Estado de Tensin Plana (caso esttico)
2
2

2 x 2 x y y

+
+
= (1 + ) ( b x ) + ( b y )
+
2
2
2
2
y
y
y
x
x

(6.79)

a) Estado de Deformacin Plana


Para el caso de tensin plana las ecuaciones constitutivas fueron obtenidas en el Ejemplo
6.24 y el campo de deformacin (ver ecuacin (6.58)) viene dado por:
x
1+
y = E
xy

1
0

Universidad de Castilla- La Mancha


Ciudad Real - Espaa

0 x

0 y
2 xy

Draft

(1 + )
(1 + )(1 )

x
y
x =
E
E

(1 + )(1 )
(1 + )

x +
y
y =
E
E

2(1 + )
xy
xy =
E

Por: Eduardo W. V. Chaves (2014)

6 ELASTICIDAD LINEAL

527

Reemplazando las componentes de la deformacin en la ecuacin de compatibilidad


(ecuaciones cinemticas), y considerando el material homogneo, podemos obtener que:
2
2
2 x y xy
+

=0
xy
y 2
x 2

2
y 2

(1 + ) 2 (1 + )
(1 + )(1 )
(1 + )(1 )
x
y + 2
x +
y

E
E
E
E

2 2(1 + )

xy = 0

xy E

(1 )

(6.80)

2 y
2 xy
2 y
2 x
2 x
+

(
1

)
2
=0
xy
x 2
x 2
y 2
y 2

Para considerar simultneamente las dos ecuaciones del movimiento utilizamos la ecuacin
(6.76), i.e.:
2

2 x
2 y

+ ( a y b y )
+
=
(
)
x
x +
2
2
y
x
xy x

2 xy

Y reemplazando en la ecuacin (6.80) podemos obtener que:


(1 )

2 y
2 xy
2 y
2 x
2 x
(
1
)
2

=0

xy
x 2
x 2
y 2
y 2

(1 )

2 y 2 x
2 y

2 x
2 x
+ ( a x b x )
+
(
1

2
2
2
2
2
x
x
x
y
y
x

2 y

+ ( a y b y ) = 0
2
y

Simplificando la ecuacin anterior obtenemos que:


Formulacin en tensin 2D Estado de Deformacin Plana
2
2

2 x 2 x y y

1
+
+
=
+
( a x b x ) + ( a y b y )

2
2
2
2
y
(1 ) x
y
y
x
x

(6.81)

Para el caso esttico o casi esttico la ecuacin anterior queda:


Formulacin en tensin 2D Estado de Deformacin Plana (caso esttico)
2
2

2 x 2 x y y
1

+
+
=
+
( b x ) + ( b y )

2
2
2
2
(1 ) x
y
y
y
x
x

(6.82)

NOTA 1: recordar que las fuerzas msicas puede ser representadas a travs de un potencial
r
r
, i.e. b = xr , ya que b es un campo conservativo. Luego, podemos decir que

y by =
. Recordar tambin que el Ejemplo 5.13 hemos definido la
x
y
Funcin de Tensin de Airy . Si tenemos en cuenta las fuerzas msicas podemos decir

bx =

que:

Universidad de Castilla- La Mancha


Ciudad Real - Espaa

Draft

Por: Eduardo W. V. Chaves (2014)

MECNICA DEL MEDIO CONTINUO: PROBLEMAS RESUELTOS

528

x =

2
y 2

y =

2
x 2

xy = yx =

2
xy

(6.83)

con lo cual
x = +

2
y 2

y = +

2
x 2

(6.84)

Reemplazando en la ecuacin (6.79) y considerando el campo de densidad de masa


homogneo podemos obtener que:
2
2
b x b y

2 x 2 x y y

(
1
)
(
)
(
)
(
1
)
b
b
+
+
+
=

+
+

+
+

x
y

y
y
x 2
y 2
y 2
x 2
x

2
x 2

2 2
+ 2 + 2

y y

2 2
+ 2 + 2

y y

2 2

2 2
2
+ 2 + 2 + 2 = (1 + ) 2 + 2

x x
x
y

2 2
2 2
4
4
4

+
+
+
+
=
+
2
2
(
1
)

2 + 2
x 2 y 2
x 4
x 2 y 2 y 4
y

2 2
4
4
4
+
+
=
+

2
[(
1
)
2
]

2 + 2
x 4
x 2 y 2 y 4
y
x

Resultando:
Formulacin en tensin 2D Estado de Tensin Plana (caso esttico)
2 2
4
4
4
2
(
1
)

+
+
=

2 + 2
y
x 4
x 2 y 2 y 4
x

(6.85)

Si ahora reemplazamos las tensiones dadas por (6.84) en las ecuaciones (6.82) podemos
obtener que:

2 2
2 2
2 2
2
b x b y
+ 2 + 2 + 2 + 2 + 2 + 2 + 2 =
+

y
y y
y y
x x
x (1 ) x

2 2
4
4
4
2 2
4 + 2 2 2 + 4 + 2 2 + 2 =
+

x
x y
y
y (1 ) x 2 y 2
x
2
x 2

2 2
1
4
4
4
2 + 2

2
2

+
+
=

(1 )
y
x 4
x 2 y 2 y 4
x

resultando
Formulacin en tensin 2D Estado de Deformacin Plana (caso esttico)
4
4
4
(1 2 ) 2 2
=

+
+
+
2

(1 ) x 2 y 2
x 4
x 2 y 2 y 4

(6.86)

Prximo a la superficie de la Tierra podemos decir con una aproximacin bastante


aceptable que las fuerzas msicas no varan con lo cual

Universidad de Castilla- La Mancha


Ciudad Real - Espaa

Draft

2 2
0 . Con lo cual las

y
x

Por: Eduardo W. V. Chaves (2014)

6 ELASTICIDAD LINEAL

529

ecuaciones de gobierno para el problema elstico bidimensional pueden ser representadas a


travs de una nica ecuacin y una nica incgnita:
Formulacin en tensin 2D (caso esttico y campo de fuerzas msicas homogneo)
(6.87)

4
4
4
+
2
+
=0
x 4
x 2y 2 y 4

Notar que inicialmente tenamos 3 incgnitas ( x , y , xy ) (formulacin en tensin caso


bidimensional), y hemos reducido el problema a una ecuacin y 1 incgnita .
NOTA 2: Recordar que la solucin analtica (exacta) en la mayora de los casos prcticos es
bastante compleja y hasta imposible. Por eso recurrimos a las tcnicas numricas, que
consiste en dado un problema buscamos la solucin. En la era de G.B. Airy (1862) la nica
posibilidad de solucin era la analtica, ya que las tcnicas numricas eran escasas. El
problema elstico se solan tratar a travs de un procedimiento inverso (Laier&Barreiro
(1983)), i.e. para una dada solucin de la ecuacin (6.87) buscaban el problema que
representaba dicha solucin.
Por ejemplo, vamos suponer que la funcin de tensin de Airy viene dada por el
polinomio:
= K1 x 2 + K 2 xy + K 3 y 2

(6.88)

donde K1 , K 2 , y K3 son constantes. Si no estamos considerando las fuerzas msicas. El


campo de tensin (6.83) queda:
x =

2
= 2K 3
y 2

y =

2
= 2K1
x 2

xy =

2
= K 2
xy

(6.89)

Notar que el campo de tensin es homogneo (uniforme). En el caso particular cuando


K1 = K 2 = 0 obtenemos el problema de una barra sometida a una fuerza F en las
extremidades, (ver Figura 6.23):
Campo de tensin (Barra sometida a esfuerzo axil):
F

x = A = 2 K 3

y = 0

xy = 0

K3 =

F
2A

Campo de deformacin (Barra sometida a esfuerzo axil):


x 2K 3
F

x = E = E = EA

x
2K3
F

=
=
y =
E
E
EA

xy = 0

Universidad de Castilla- La Mancha


Ciudad Real - Espaa

Draft

Por: Eduardo W. V. Chaves (2014)

MECNICA DEL MEDIO CONTINUO: PROBLEMAS RESUELTOS

530

y, v

2 D ( x, y )

A (rea)

x, u

x =

F
A

Figura 6.23
Campo de desplazamientos (Barra sometida a esfuerzo axil):
u
F
F
integrando en x
x + f 1 ( y ) + C1

u ( x, y ) =
= x =
EA
EA
x
v
F integrando en y
F
y + f 2 ( x) + C 2

v( x, y ) =
= y =
EA
EA
y
v u
= xy = 0
+
x y

donde f1 ( y ) es una funcin de y , f 2 ( x) es una funcin de x , C1 y C 2 son constantes de


integracin. A travs de la tercera ecuacin podemos obtener que:
v u
= xy = 0
+
x y
F
F

y + f 2 ( x) + C 2 +
x + f 1 ( y ) + C1 = 0

x EA
y EA

f ( x) f 1 ( y )
2
+
=0
x
y
f ( x)
f ( y )
2
= 1
x
y

La nica solucin posible es cuando:


f 2 ( x)
= C3
x

f 2 ( x)
= C 3
x

donde C3 es una constante. Luego el campo de desplazamiento queda:


F

x + C3 y + C1
u ( x, y ) =

EA

v( x, y ) = F y C x + C
3
2

EA

donde C1 , C2 y C3 se obtienen a travs de las condiciones de contorno. Vamos suponer


que la barra tiene las condiciones de apoyo tal y como se indica en la Figura 6.24.

Universidad de Castilla- La Mancha


Ciudad Real - Espaa

Draft

Por: Eduardo W. V. Chaves (2014)

6 ELASTICIDAD LINEAL

y, v

531

2 D ( x, y )

A (rea)

u ( x = 0, y ) = 0

v( x = 0, y = 0) = 0

x, u

x =

F
A

Figura 6.24
Segn las condiciones de apoyo de la Figura 6.24 podemos decir que:
u ( x = 0, y = 0) = 0 u ( x = 0, y = 0) = C1 = 0

v( x = 0, y = 0) = 0 u ( x = 0, y = 0) = C 2 = 0
u ( x = 0, y ) = 0 u ( x = 0, y = 0) = C = 0
1

Resultando el siguiente campo de desplazamiento:


F

u = EA x

v = F y
EA

Vamos considerar ahora que la funcin de tensin de Airy viene dada por:
= K1 y 3

(6.90)

donde K1 es una constante. Si no estamos considerando las fuerzas msicas, el campo de


tensin (6.83) queda:
2

x =

y 2

= 6K1 y

y =

2
x 2

=0

xy =

2
=0
xy

(6.91)

Notar que el campo de tensin x ( y ) solo vara con y . Este es el caso de flexin pura, ver
Figura 6.25. Para una dada seccin tenemos que:

F = x dA = 0

(Fuerza resultante en una seccin )

M = y x dA = 6 K1 y 2 dA = 6 K1 y 2 dA = 6 K1 I z
A

( Momento Flector )

donde I z = y 2 dA es el segundo momento de rea de la seccin transversal (momento de


A

inercia). Con eso podemos decir que


M = 6 K1 I z

K1 =

M
6I z

Campo de tensin (Flexin pura):


Universidad de Castilla- La Mancha
Ciudad Real - Espaa

Draft

Por: Eduardo W. V. Chaves (2014)

MECNICA DEL MEDIO CONTINUO: PROBLEMAS RESUELTOS

532

x = 6K1 y =

M
y
Iz

y = 0

xy = 0

Campo de deformacin (Flexin pura):


x 6 K1 y
M

x = E = E = EI y
z

6 K1 y M
x
y
=
=
y =
E
E
EI z

xy = 0

y
x

Seccin transversal
Figura 6.25
Campo de desplazamiento (Flexin pura):
u
M
M
integrando en x
yx + f1 ( y ) + C1
u ( x, y ) =
x = x = EI y
EI
z
z

v
M
M 2
y integrando
y + f 2 ( x) + C2
en
y v( x, y ) =
= y =
2
y
EI
EI

z
z

v u
= xy = 0
+
x y

Teniendo en cuenta que:


v u
= xy = 0
+
x y
M

M 2

y + f 2 ( x) + C 2 +
yx + f1 ( y ) + C1 = 0
x 2 EI z
y EI z

f 2 ( x) M
f1 ( y )

+
x+
=0
x
EI z
y

f 2 ( x) M
f ( y )
+
x= 1
x
EI z
y

Universidad de Castilla- La Mancha


Ciudad Real - Espaa

Draft

Por: Eduardo W. V. Chaves (2014)

6 ELASTICIDAD LINEAL

533

De forma anloga al ejemplo anterior concluimos que:


f 2 ( x) M
+
x = C3
x
EI z

f1 ( y )
= C3
y

Integrando las relaciones anteriores podemos obtener que:


f 2 ( x) M
x = C3
+
EI z
x

f1 ( y )
= C3
y

f1 ( y ) = C3 y

f 2 ( x) = C 3 x

M 2
x
2 EI z

Con lo cual el campo de desplazamiento queda:


M
M

u = EI yx + f 1 ( y ) + C1 = EI yx C 3 y + C1

z
z

v = M y 2 + f ( x) + C = M y 2 + C x M x 2 + C = M ( y 2 + x 2 ) + C x + C
2
2
3
2
3
2

2 EI z
2 EI z
2 EI z
2 EI z

donde las constantes C1 , C2 y C3 se obtienen a travs de las condiciones de contorno.


Vamos suponer que es una viga en voladizo tal y como se indica en la Figura 6.26.
y, v

x, u

Deformada
Figura 6.26
Con estas condiciones de contorno podemos concluir que:
u ( x = 0, y = 0) = 0 u ( x = 0, y = 0) = C1 = 0

v( x = 0, y = 0) = 0 u ( x = 0, y = 0) = C 2 = 0
v ( x, y ) =

M
( y 2 + x 2 ) + C 3 x + C 2
2 EI z

v( x, y ) M
x + C3
=
EI z
x

v( x = 0, y = 0)
= 0 = C3
x

Con lo cual el campo de desplazamiento para la viga en voladizo queda:


M

u ( x, y ) = EI xy

v( x, y ) = M ( y 2 + x 2 )

2 EI z

Universidad de Castilla- La Mancha


Ciudad Real - Espaa

Draft

Por: Eduardo W. V. Chaves (2014)

MECNICA DEL MEDIO CONTINUO: PROBLEMAS RESUELTOS

534

6.3 Introduccin a Elementos Estructurales 1D


Ejemplo 6.33
Considrese un elemento de barra que presenta una de las dimensiones mucho mayor que
las otras dos dimensiones. Obtener los esfuerzos posibles que pueden surgir en una seccin
transversal de la barra. Adoptar el sistema de coordenadas indicado en la Figura 6.27 y
utilizar notacin ingenieril.
Diagrama de
deformacin

z
z

x (z )

y
y

Diagrama de
tensin

x (z )

eje neutro

a) viga

b) seccin de la viga
Figura 6.27: Vigas.

Hiptesis:

Rgimen de pequeas deformaciones y pequeas rotaciones;

Materiales elstico, linear, homogneo e istropo;

Para obtener los esfuerzos resultantes debido a la componente x considerar que


cualquier seccin transversal plana de la barra antes de la deformada permanece plana
tras la deformacin.

Solucin:
Los esfuerzos se obtienen al integrar las tensiones sobre el elemento de rea de la seccin
transversal de la barra. Luego, en el caso genrico en una cara de la seccin transversal
(segn el sistema adoptado) pueden surgir las tensiones x , xy y xz .
Si hacemos un corte en una seccin segn la orientacin del plano , el estado tensional
en un punto situado en esta seccin ser el indicado en la Figura 6.28.
La pregunta que tenemos que hacer es: Cmo vara el campo de tensiones en una seccin
transversal?
Como el material es elstico y lineal, la tensin vara linealmente con la deformacin, luego,
la componente x = E x . Como estamos en el rgimen de pequeas deformaciones se
u
. Luego para que la seccin transversal tras la deformacin sigua siendo
x
un plano el campo de desplazamiento u ( y, z ) en la seccin tiene que definir un plano.

cumple que x =

Universidad de Castilla- La Mancha


Ciudad Real - Espaa

Draft

Por: Eduardo W. V. Chaves (2014)

6 ELASTICIDAD LINEAL

535

xz

xy

x
A - rea de la seccin

Figura 6.28: Tensiones en un seccin de viga.


Podemos tener las siguientes posibilidades:
a) La seccin se desplaza segn la direccin x ,
en este caso la deformacin es constante en la
seccin como consecuencia la tensin es
constante en la seccin. Al integrar esta tensin
en el rea de la seccin vamos obtener el
esfuerzo normal (denotado por N ) que puede
ser (traccin si el esfuerzo es positivo caso
contrario el esfuerzo es de compresin).

u (1)
y

Figura 6.29

u (2)
y

u ( 2)

b) Otra posibilidad de desplazamiento para que


mantenga la seccin plana, es cuando en la seccin hay
un giro alrededor del eje y , en este caso el campo de
desplazamiento en la seccin viene dado por la Figura
6.30. En este caso la deformacin vara segn un plano
en la seccin. Notar que al integral el campo de
desplazamiento en el rea de la seccin resulta cero, lo
mismo pasa si integramos el campo de deformacin o
de tensin. Es decir que la fuerza resultante ser igual
a cero, pero hay momento. Debido a este campo de
desplazamiento surgir el momento flector segn la
direccin y , que denotamos por M y . El causante de
dicho desplazamiento es la deflexin de la viga segn
la direccin z (desplazamiento w( x) ).

Figura 6.30

Universidad de Castilla- La Mancha


Ciudad Real - Espaa

Draft

Por: Eduardo W. V. Chaves (2014)

MECNICA DEL MEDIO CONTINUO: PROBLEMAS RESUELTOS

536

c) Otra posibilidad es una rotacin de la


seccin alrededor del eje z , ver Figura 6.31,
en este caso tambin el sumatorio de las
fuerzas en la seccin es igual a cero, pero
surgir un momento flector segn la
direccin z , que denotaremos por M z . El

u ( 3)
y

causante del desplazamiento u (3) es la


deflexin de la viga segn la direccin y
(desplazamiento v( x) ).

u (3)

Figura 6.31
Tambin podemos tener una combinacin de los casos mencionados anteriormente. Para el
caso genrico el campo de tensin x en la seccin transversal viene representado por la
dado por Figura 6.32.
Considerando Figura 6.33, tambin podemos decir que el momento flector M y puede ser
expresado como:

M y = (x2 ) zdA =
A

S z

zdA = S
c
c
A

dA =

S
Iy
c

(6.92)

donde I y = z 2 dA es el momento de inercia de rea con respecto al eje y . Teniendo en


A

cuenta que

S (x2 )
, ver Figura 6.33, obtenemos que:
=
c
z

(x2 ) ( z ) =

My
Iy

(6.93)

Anlogamente, podemos obtener que:


(x3) ( y ) =

Mz
y
Iz

(6.94)

Teniendo en cuenta que x = E x , las expresiones anteriores quedan:


(x2 ) ( z ) =

My
EI y

(x3) ( y ) =

Mz
y
EI z

(6.95)

donde EI es la rigidez a flexin de la barra.


Cuando la barra est solo sometida a las tensiones (x2) y/ (x3) decimos que la barra est
bajo flexin pura. Si actan las tensiones (x1) ms (x2) y/ (x3) decimos que la barra est
bajo accin de flexin compuesta. En este caso tenemos que:
x ( y, z ) = (x1) + (x2 ) ( z ) + (x3) ( y ) =

Universidad de Castilla- La Mancha


Ciudad Real - Espaa

Draft

M
N My
+
z z y
A
Iy
Iz

(6.96)

Por: Eduardo W. V. Chaves (2014)

6 ELASTICIDAD LINEAL

x ( y, z ) =

537

M
N My
+
z z y
A
Iy
Iz

x ( y, z )
x

144444444444444444424444444444444444443

(x1)

(x2 )
y

N = (x1) dA
A

(x3)

M y = z (x2 ) dA
A

M z = y(x3) dA
A

Mz

My

144424443

144424443

( 2)
x dA =

( 3)
x dA

=0

Figura 6.32: Esfuerzo normal y momento flector (Flexin compuesta).


S
c

x (z )
eje neutro

Figura 6.33: Distribucin de tensin en una seccin de viga.

Universidad de Castilla- La Mancha


Ciudad Real - Espaa

Draft

Por: Eduardo W. V. Chaves (2014)

MECNICA DEL MEDIO CONTINUO: PROBLEMAS RESUELTOS

538

Esfuerzo Cortante y Momento Torsor


Debido a las tensiones tangenciales, pueden surgir los esfuerzos cortantes Q z y Q y , (ver
Figura 6.34):

Q z = xz dA

(6.97)

xy z dA

(6.98)

Q y = xy dA

y momento torsor ( M T ) (ver Figura 6.49):


MT =

xz y

xz ( y , z )

xy ( y, z )

xz dA

xy dA

Qz

Qy

Figura 6.34: Tensiones tangenciales Esfuerzo cortante.

Resumimos en la Figura 6.35 los esfuerzos que pueden surgir en un elemento de barra.
z, w

Mz

y, v

My
Qz

Qy

M x MT

x, u

Figura 6.35: Esfuerzos en un elemento tipo barra.

Universidad de Castilla- La Mancha


Ciudad Real - Espaa

Draft

Por: Eduardo W. V. Chaves (2014)

6 ELASTICIDAD LINEAL

539

NOTA 1: Los esfuerzos dependern de la carga que est sometida la barra, ver Figura 6.36.

a)

Carga

z, w

y, v

Esfuerzos
My

N
qz
m

Qz

x, u

b)

Carga
z, w

y, v

Esfuerzos
N
qy
m

Mz
Qy

x, u

c)

Carga
z, w

Esfuerzos

y, v
N
F [N ]

x, u

d)

Carga
z, w

y, v

Esfuerzos
Nm
mT

M x MT

x, u

Figura 6.36: Algunos casos de cargas en un elemento de barra.

Universidad de Castilla- La Mancha


Ciudad Real - Espaa

Draft

Por: Eduardo W. V. Chaves (2014)

540

MECNICA DEL MEDIO CONTINUO: PROBLEMAS RESUELTOS

NOTA 2: En general, las barras reciben nombres particulares dependiendo de cmo


fueron concebidas para trabajar, en otras palabras qu papel van desempear dentro de las
obras civiles. Algunos ejemplo se presentan a continuacin:
Vigas de Edificios: En general el esfuerzo predominante es el momento flector (ver
Figura 6.36 (a). En las vigas de los puentes el esfuerzo cortante y momento torsor ya no
pueden ser despreciados.
Celosas: Son barras que son concebidas para trabajar nicamente a compresin o a
traccin, (ver Figura 6.36(c) y Figura 6.37).

a) Cpula geodsica

b) Torre de alta tensin

Figura 6.37: Ejemplos de celosas.


Columnas: Son barras en las cuales el esfuerzo axil y los momentos flectores son los
esfuerzos predominantes.
Prticos Planos: Son barras que los esfuerzos predominantes son el momento flector y el
esfuerzo cortante, superposicin de los esfuerzos Figura 6.36(a)+ Figura 6.36 (c).
Prticos Espaciales: Son barras donde tendremos que tener en cuenta todos los
esfuerzos, ver Figura 6.35.
NOTA 3: Alabeo de la seccin
En el ejemplo de torsin (ver Ejemplo 6.36) vamos demostrar que solamente en el caso de
seccin transversal circular la seccin permanece plana tras la aplicacin del momento
torsor.
El alabeo de la seccin es un fenmeno que surge debido al aumento de las tensiones
tangenciales en un punto y disminucin en otro, ver Figura 6.39(a). En secciones circulares

Universidad de Castilla- La Mancha


Ciudad Real - Espaa

Draft

Por: Eduardo W. V. Chaves (2014)

6 ELASTICIDAD LINEAL

541

no hay alabeo, ya que la distribucin de la tensin tangencial en la seccin vara como se


muestra en la Figura 6.39(b).

z
xz
xy

Figura 6.38: Tensiones tangenciales Momento torsor.


( y, z )

max

( r )
r

a) Seccin rectangular

b) Seccin circular

Figura 6.39: Distribucin de tensiones tangenciales.


NOTA 4: Deflexin de la Barra
Vamos considerar la deflexin de una viga segn la direccin z (desplazamiento w( x) ), ver
Figura 6.40. A travs de la Figura 6.40 podemos concluir que:
u ( 2 ) = w, x z
(x2 ) ( z )

E (x2 )

(x2 ) ( z ) =

du ( 2 )
d 2w
= 2 z w, xx z
dx
dx

(6.99)

= Ew, xx z

Luego si consideramos la deflexin de la barra segn la direccin y (desplazamiento v( x) )


vamos obtener que:
u ( 3) = v , x y

(x3) ( y )

= Ev, xx y

E (x3)

(x3) ( y ) =

du (3) d 2 v
= 2 y v, xx y
dx
dx

(6.100)

Notar que hemos invertido el signo ya que el momento flector M z (misma direccin que
z ) producir el campo de desplazamiento en la seccin con sentido contrario al presentado
por la Figura 6.40.

Universidad de Castilla- La Mancha


Ciudad Real - Espaa

Draft

Por: Eduardo W. V. Chaves (2014)

MECNICA DEL MEDIO CONTINUO: PROBLEMAS RESUELTOS

542

y = w, x
u

( 2)

Pequeos ngulos

= y z = w, x z

tan

(tras la deformada)
a
w

dw
w, x
dx

eje neutro
z, w
n
a

x, u

(antes de la deformada)

Figura 6.40: Desplazamiento en un seccin transversal de la barra.


Si comparamos las ecuaciones (6.93) y (6.99), podemos concluir que:
(x2 ) ( z ) =
(x2 ) ( z )

My
Iy

= Ew, xx

w, xx

d 2w
dx 2

My
EI y

(6.101)

Anlogamente, si comparamos las ecuaciones (6.94) y (6.100) podemos obtener:

( 3)
x ( y ) = Ev, xx y

(x3) ( y ) =

Mz
Iz

v, xx

d 2v M z
=
EI z
dx 2

(6.102)

Notar que para la obtencin de las ecuaciones (6.101) y (6.102) hemos considerado las
ecuaciones constitutivas y las ecuaciones cinemticas. Para completar las ecuaciones de gobierno
necesitamos aadir las ecuaciones de equilibro en un elemento diferencial de barra. Como en
una seccin ya hemos perdido la informacin de la simetra del tensor de tensiones
tendremos que aplicar el Principio de la conservacin del momento lineal y el principio de
conservacin del momento angular. En otras palabras el sumatorio de fuerzas igual a cero y
el sumatorio de momentos igual a cero en un elemento diferencial de barra dx .

Universidad de Castilla- La Mancha


Ciudad Real - Espaa

Draft

Por: Eduardo W. V. Chaves (2014)

6 ELASTICIDAD LINEAL

543

NOTA 5: Tensin tangencial en la seccin transversal


Supongamos que tengamos varias capas de placas completamente lisas apoyada sobre dos
apoyos como indica la Figura 6.41 (a). Como las placas pueden desplazarse libremente
entre camadas no hay tensin tangencial tras la aplicacin de la fuerza, ver Figura 6.41 (b).
Si todas las placas estn unidas formando parte de una nica pieza monoltica, el
desplazamiento tangencial entre camadas ya estar limitado, surgiendo entonces las
tensiones tangenciales (ver Figura 6.41 (c)). Esta tensin tangencial es el causante de la
aparicin de la tensin tangencial en la seccin transversal (ver Figura 6.41 (c) y (d)).

a)
seccin transversal
P

b)
seccin transversal

c)

seccin transversal

d)

d
c

a
x

d
c

Figura 6.41: Viga sometida a flexin

Universidad de Castilla- La Mancha


Ciudad Real - Espaa

Draft

Por: Eduardo W. V. Chaves (2014)

MECNICA DEL MEDIO CONTINUO: PROBLEMAS RESUELTOS

544

Ejemplo 6.34
Obtener la ecuacin de gobierno para una viga con rigidez a flexin constante ( EI y ) , que
est sometida a una carga uniformemente distribuida por unidad de longitud (q z ) , ver
Figura 6.42.
z, w
y, v

Esfuerzos

N
qz
m

My
Qz

x, u

Carga

dx

Figura 6.42: Viga sometida a una carga uniformemente distribuida.


Solucin:
Para este caso, el elemento diferencial de viga dx estar sometido a los esfuerzos como
indicado en la Figura 6.43.
dx
2
y, v

z, w
My
A

My +

x, u

Qz

My

M y
x

dx

Qz

Qz +

Qz
dx
x

dx

Figura 6.43: Elemento diferencial de viga


Haciendo el equilibrio de fuerzas y de momentos en el elemento diferencial de viga (punto
B ) podemos obtener que:

=0

M yB = 0

Q z

Qz + Qz +
dx + qdx = 0
x

Q z
= q
x

M y

dx
=0
dx + qdx
M y Q z dx + M y +
2
x

(6.103)
M y
x

= Q z (6.104)

donde hemos considerado que dxdx 0 .

Universidad de Castilla- La Mancha


Ciudad Real - Espaa

Draft

Por: Eduardo W. V. Chaves (2014)

6 ELASTICIDAD LINEAL

545

Para completar las ecuaciones de gobierno del problema de viga, tenemos que introducir
las ecuaciones constitutivas y ecuaciones cinemticas, que es lo mismo que considerar la
ecuacin (6.101), luego
w, xx =

M y
x

My

EI y

= Qz =

2M y

x 2

M y = EI y w, xx EI y

2w
x 2

3w
EI y w, xx = EI y w, xxx EI y
x
x 3

(6.105)

Q z
2
4w
= 2 EI y w, xx = EI y
= q
x
x
x 4

con lo cual definimos la ecuacin diferencial de viga:


4w

EI y

x 4

=q

(6.106)

NOTA: Tensiones tangenciales en la seccin transversal


z

a)
b
2

xz (z )

A B
z

x
a

QzA

My +

My

b)

Qz +

dx

x +

x
dx
x

F2 = x ( z )dA
A

F3 =

seccin transversal

Qz
dx
x

dx
x (z )

M y

( ( z, y)dy )dx

F1 = x + x dx dA
x

eje neutro
Figura 6.44:
Universidad de Castilla- La Mancha
Ciudad Real - Espaa

Draft

Por: Eduardo W. V. Chaves (2014)

MECNICA DEL MEDIO CONTINUO: PROBLEMAS RESUELTOS

546

Haciendo el equilibrio de fuerza segn la direccin x (ver Figura 6.44 (b)) podemos
obtener que:

=0

F1 F2 F3 = 0

x
dx dA x dA dx ( z, y )dy = 0
x

(6.107)

( z )
dx ( z , y )dy = x
dx dA

Si invocamos las ecuaciones (6.93) y (6.104) podemos decir que:


(x2 ) ( z ) =

My
Iy

(x2) ( z ) M y M y z
Q z
=
z =
= z
x
x I y
x I y
Iy

donde estamos considerando que I y es constante en la viga. Teniendo en cuenta la


ecuacin anterior dentro de la ecuacin (6.107) podemos obtener que:
( z )
dx ( z , y )dy = x
dx dA
x

Q z
Q
dx ( z , y )dy = z dx dA = z dx zdA
Iy

Iy

A
A

con lo cual podemos concluir que:


Qz

( z, y)dy = I zdA =
y A

Qz z
Iy

donde

z = zdA

(6.108)

Para la seccin transversal a b y si consideramos como una buena aproximacin

ave ( z )a = ( z, y )dy

donde ave (z ) es el promedio de las tensiones tangenciales en (z ) .


( z, y )

A
z

Luego, podemos obtener que:


ave ( z ) =

Universidad de Castilla- La Mancha


Ciudad Real - Espaa

Qz z
Iy

Draft

ave ( z ) =

Qz z
I ya

Por: Eduardo W. V. Chaves (2014)

6 ELASTICIDAD LINEAL

547

Ejemplo 6.35
Demostrar:
a) Primer Teorema de Mohr
El ngulo que hay comprendido entre dos tangentes en dos puntos cualesquiera de la
deflexin de la viga (curva elstica), es igual al rea del diagrama

My

entre estos dos

EI y

punto.
b) Segundo Teorema de Mohr
Para una viga originalmente recta, sometida a momento flector, la lnea vertical
interceptada entre un punto B y la tangente a la curva elstica en otro punto A es igual al
primer momento del diagrama

My
EI y

sobre el punto donde la intercepcin es medida.

Solucin:
a) Primer Teorema de Mohr
Hemos visto en el final del captulo 1 (Notas Complementarias - curvatura) que se cumplen
las siguientes ecuaciones:
=

d d dx
w
=
=
ds
dx ds [1 + ( w) 2 ]

1
1
[1 + ( w) 2 ] 2

w
3
[1 + ( w) 2 ] 2

1
r

(6.109)

ds = d =

A B _ A

(6.110)

donde w =

w
2w
w, x w = 2 w, xx , siendo w la deflexin, es la curvatura, ds es el
x
x

elemento infinitesimal de longitud de arco.


Para pequeas curvaturas se cumple que:
=

1
2w
w 2 w, xx
r
x

ds dx

tan

cos 1

d d

ds
dx

Teniendo en cuenta la ecuacin (6.101) podemos concluir que:


w, xx =

My

EI y

My
EI y

y si aplicamos la ecuacin (6.110) podemos obtener que:


B

ds = d = B _ A
A

donde el trmino

EI
A

My

EI

ds

My

EI
A

dx = d = B _ A

Draft

(6.111)

ds corresponde al rea del diagrama definido por

Universidad de Castilla- La Mancha


Ciudad Real - Espaa

My

My
EI y

Por: Eduardo W. V. Chaves (2014)

MECNICA DEL MEDIO CONTINUO: PROBLEMAS RESUELTOS

548

z
= B A

ds

B
r

wA

My

EI y

wB

A=

My

EI

My

EI y

dx

x
x

Figura 6.45:
b) Segundo Teorema de Mohr
Multiplicamos la ecuacin (6.109) por x obtenemos que:
B

d
integrando
x =
xds = xd
x
ds
A
A

My
EI y

xdx = xd = ( x ) B ( x ) A d
A

B
My

dx x = xd = ( x ) B ( x ) A d
EI y
A
A

donde

My

EI
A

xdx es el primer momento de rea del diagrama

Universidad de Castilla- La Mancha


Ciudad Real - Espaa

Draft

My
EI y

, ver Figura 6.45.

Por: Eduardo W. V. Chaves (2014)

6 ELASTICIDAD LINEAL

549

6.4 Torsin
Ejemplo 6.36
Considrese las hiptesis (aproximaciones) para el problema de torsin de Saint-Venant,
(ver Figura 6.46):

El cuerpo es prismtico (considerar como eje prismtico el eje x1 );

El ngulo de torsin unitario es constante segn eje x1 ;

La proyeccin de las secciones en el plano x2 x3 tiene movimiento de cuerpo


rgido (rotacin alrededor del eje x1 ).

Demostrar que el problema viene gobernado por la ecuacin:


2u1 = 0

con

u1 = u1 ( x2 , x3 )

2u 2u
G 21 + 21 = 0
x3
x2

u1
u1

G
x 3 + G
+ x2
x 2 x 2
x 3 x3

(6.112)
(6.113)

= 0

(6.114)

x1 = x

x, x1

MT

z = x3

P2
x1

S2

P2

y = x2

x1

S1

r
r

P1

Figura 6.46: Torsin aplicado a un cuerpo prismtico.

Universidad de Castilla- La Mancha


Ciudad Real - Espaa

Draft

Por: Eduardo W. V. Chaves (2014)

MECNICA DEL MEDIO CONTINUO: PROBLEMAS RESUELTOS

550

OBS.: Con estas hiptesis las secciones planas no necesariamente permanecen planas y
restringe a problemas de torsin de cuerpos prismticos.
Considerando un cuerpo prismtico y aplicando un momento torsor en la extremidad libre
el cuerpo se desplazar segn se indica en la Figura 6.46.
Solucin:

Consideremos un punto P1 situado en la seccin fija S1 cuyo vector posicin es r como


se indica en la Figura 6.46. Si consideramos otra seccin S 2 (libre de girar y alabear) que
dista de x1 de la seccin S1 y proyectamos el punto P1 en la seccin S 2 obtendremos el
punto P2 . Tras la aplicacin del momento de torsin la seccin S 2 podr girar y alabearse
libremente, luego el punto originalmente representado por P2 pasa a ocupar el punto P2
como indicado en la Figura 6.47.
x3
x2 = r cos

u2

x3 = r sin

P2
r

u3

rx1

x1

r 2 = x 22 + x32

P2

x2

Figura 6.47: desplazamiento en la seccin del cuerpo prismtico.


Geomtricamente (Figura 6.47) podemos obtener que los desplazamientos vienen dados
por:
u2 = r x1 sin = x3 x1

u3 = r x1 cos = x2 x1

(6.115)

con: x2 = r cos , x3 = r sin . Siendo u2 el desplazamiento segn direccin x2 y u3 el


desplazamiento segn direccin x3 . El desplazamiento del punto P2 segn direccin x1
puede ser cualquiera, luego podemos decir que:
u1 = u1 ( x 2 , x 3 )

Los desplazamientos
independientes de x1 .

;
u1

u 3 = x 2 x1

(6.116)

son denominados desplazamientos por alabeo siendo

Universidad de Castilla- La Mancha


Ciudad Real - Espaa

u 2 = x 3 x1

Draft

Por: Eduardo W. V. Chaves (2014)

6 ELASTICIDAD LINEAL

551

(Ecuaciones cinemticas)
Las relaciones desplazamiento-deformacin quedan:
11 =

u1
=0;
x1

12

1 u1 u2

+
2 x2 x1

1 u1

x3 ;
2 x2

22 =

u 2
= 0;
x 2

23

33 =

1 u2 u3

+
2 x3 x2
1
= ( x1 + x1 ) ;
2
=0

13

u 3
= 0;
x3

1 u1 u3

+
2 x3 x1

1 u
= 1 + x2
2 x3

Resultando

11 12
ij = 12 22
13 23

13

1 u
23 = 1 x3
2 x2

33
u

1 + x2
x3

u1

x3

x2
0
0

u1

x + x2
3

(6.117)

(Ecuaciones constitutivas)
Reemplazando las deformaciones en las ecuaciones constitutivas:
ij =

E
E
E
kk ij +
ij =
ij
(1 + )(1 2 )
(1 + )
(1 + )

(6.118)

Notar que kk = 0 , resultando las siguientes componentes de tensin:

E
1 E u1
x3
ij =
ij =

(1 + )
2 (1 + ) x2

1 + x2
x3

u1

x3
x2

13
u

0 = G 1 x3

x2
0
u

1 + x2

x3

u1

x3
x2

0
= 12
13

12
0
0

Universidad de Castilla- La Mancha


Ciudad Real - Espaa

Draft

0
0

0
0

u1

+ x2
x3

u1

+ x2
x3

(6.119)

Por: Eduardo W. V. Chaves (2014)

MECNICA DEL MEDIO CONTINUO: PROBLEMAS RESUELTOS

552

(Ecuaciones de equilibrio)
Utilizando las ecuaciones de equilibrio sin considerar las fuerzas msicas, obtenemos:
11 12 13
+
+
=0

x2
x3
x1
12 22 23
+
+
=0

x2
x3
x1
13 23 33
x + x + x = 0
2
3
1

12 13
+
=0

x3
x2
12
= 0 12 = 12 ( x2 , x3 )

x1
13
= 0 13 = 13 ( x2 , x3 )

x1

(6.120)

y reemplazando las componentes 12 y 13 en la primera ecuacin de las ecuaciones de


equilibrio, podemos obtener:
12 13
=0
+
x 3
x 2

G
x 2

u1

u1

x 3 + G
+ x2

x
x
3 x 3
2

2 u1 2 u1
=0
G 2 +
2

x
3
2

2 u1 = 0

donde

(6.121)

= 0

(6.122)

u1 = u1 ( x 2 , x 3 )

(6.123)

que es la Ecuacin Diferencial de Torsin (Ecuacin de Laplace). Observar que esta ecuacin es la
ecuacin de Helmholtz para el caso particular de un material homogneo.
NOTA 1: Barra prismtica de seccin circular
Notar que cuando la barra tiene seccin circular no hay alabeo, luego, u1 = 0 .
A

MT

x1 =

dx1 = d
=

x1

d
dx1

Figura 6.48: Barra circular sometida a torsin.


Con eso, obtenemos que el campo de deformacin en una seccin viene dado por:
11
ij = 12
13

12
22
23

x3 x2
13
0
0

23 = x3
0
0 = x3
2
2
x2
x2
33
0
0

x3
0
0

x2
0
0

El campo de tensin en una seccin viene dado por:


Universidad de Castilla- La Mancha
Ciudad Real - Espaa

Draft

Por: Eduardo W. V. Chaves (2014)

6 ELASTICIDAD LINEAL

11
ij = 12
13

12
22
23

553

13 0
Gx3 Gx 2
0

23 = Gx3
0
0 = G x 3
x 2
33 Gx 2
0
0

z, x 3

x3
0
0

x2
0
0

13

12
13

12

max

( r )

y, x 2
R

Figura 6.49: Tensiones tangenciales Momento torsor.


El momento torsor viene dado por:
MT =

13 x2

12 x3 )dA =

((Gx ) x
2

(Gx3 ) x3 )dA = G x22 + x32 dA = G r 2 dA

(6.124)

= GJ

donde J = r 2 dA =
A

R 4
es el momento polar de inercia.
2

Segn la Figura 6.49 podemos concluir que:


(12 ) 2 + (13 ) 2 = 2

(G) 2 ( x 22 + x 32 ) = 2

(G) 2 r 2 = 2 (r ) = Gr

MT
, con lo cual la ecuacin anterior queda:
J

Segn ecuacin (6.124) se cumple que G =


(r ) = Gr

(r ) =

MT
r
J

(6.125)

con lo cual el ngulo de torsin unitario tambin se puede escribir como:


=

MT
GJ

(r )
Gr

M T ( r )
=
= G
r
J

(6.126)

El trmino GJ se denomina de mdulo de rigidez a torsin.


El valor mximo de ( r ) se produce en r = R :
(r = R ) = max = GR

Universidad de Castilla- La Mancha


Ciudad Real - Espaa

Draft

(r = R ) = max =

MT
R
J

Por: Eduardo W. V. Chaves (2014)

MECNICA DEL MEDIO CONTINUO: PROBLEMAS RESUELTOS

554

NOTA 2: Funcin de Tensin de Prandtl


Vamos adoptar una funcin de tensin de Prandtl (tambin conocida con funcin de
tensin) tal que:
12 =

x3

13 =

x2

(6.127)

Verificando que se cumple con las ecuaciones de equilibrio, (ver ecuacin (6.120)):
12 13
+
=0
x2
x3

=0
x2 x3 x3 x2

Recordar que las ecuaciones de gobierno del problema elstico lineal en funcin de las 6
componentes del tensor de tensiones vienen dada por la formulacin en tensin, (ver
Ejemplo 5.12). Si adems consideramos un caso esttico y donde las fuerzas msicas no
r
varan con x recaemos en las ecuaciones de Beltrami:
ij , kk +
2xr

1
kk , ij = 0ij
(1 + )

1
+
xr [ xr [Tr ( )]] = 0
(1 + )

Ecuaciones de Beltrami

(6.128)

Teniendo en cuenta las componentes del tensor de tensiones (ver ecuacin (6.119)):

ij = G 1 x3

x2
u

1 + x2

x3

u1

x3
x2

0
0

u1

+ x2
x3
0

0
= 12

13
0

12
0
0

13
0
0

Teniendo en cuenta que la traza es cero, kk = 0 , las ecuaciones de Beltrami se reducen a:


ij , kk = ij ,11 + ij , 22 + ij , 22 = 0 ij
ij ,kk

11,11

= 12,11
13,11

ij ,kk

0
(12,11 + 12, 22 + 12,33 ) (13,11 + 13, 22 + 13,33 )

= (12,11 + 12, 22 + 12,33 )


0
0
= 0 ij
( 13,11 + 13, 22 + 13,33 )

0
0

12,11
22,11
23,11

13,11 11, 22

23,11 + 12, 22
33,11 13, 22

12, 22
22, 22
23, 22

13, 22 11,33

23, 22 + 12,33
33, 22 13,33

12,33
22,33
23,33

13,33

23,33 = 0 ij
33,33

Con lo cual quedamos con dos ecuaciones:


12,11 + 12, 22 + 12,33 = 0

13,11 + 13, 22 + 13,33 = 0

Observar que 12 = 12 ( x2 , x3 ) y 13 = 13 ( x2 , x3 ) (ver ecuacin (6.120)), con lo cual las


ecuaciones anteriores quedan:

Universidad de Castilla- La Mancha


Ciudad Real - Espaa

Draft

Por: Eduardo W. V. Chaves (2014)

6 ELASTICIDAD LINEAL

12, 22 + 12,33 = 0

13, 22 + 13,33 = 0

2 12 2 12
=0
2 +
x32
x2
2
2
13 + 13 = 0
x 2
x32
2

555

2xr 12 = 0
2
xr 13 = 0

(6.129)

Si consideramos las tensiones definidas en (6.127) dentro de las ecuaciones en (6.129)


podemos obtener que:
2xr 12 = 0
2
xr 13 = 0

2 2
2 + 2 =0
=
2xr = 0
xr
x

x
x
x
3
3
3


2 x3
(6.130)

2
2

2r
2r
2r

x x = x x = 0 x x = x x 2 + x 2 = 0
2
2
2
3
2
2

Como 2xr no vara ni con x2 y ni x3 concluimos que:


2xr =

2 2
+
= F = constante
x22 x32

(6.131)

Luego, cualquier funcin que satisfaga la ecuacin anterior cumplir tambin con las
ecuaciones de equilibrio y de compatibilidad.
Partiendo de las componentes de tensin:
u

12 = G 1 x3
x 2

13

u
= G 1 + x 2
x3

u1 12
=
+ x3
x 2
G

u1 13
=
x2
x3
G

(6.132)

Derivando 12 con respecto a x3 y 13 con respecto a x2 obtenemos que


u1

=
x 3 x 2 x 3

12
+ x3

1 12
+
=
G x 3

u1

=
x 2 x 3 x 2

13

x2
G

1 13

=
G x 2

y restando las dos expresiones podemos obtener que


1 13
u1
u1 1 12

=
+
+
x3 x2 x2 x3 G x3
G x2
0=

1 12
1 13
+ 2
G x3
G x2

12 13

= 2G
x3
x2

Reemplazando el valor de la tensin 12 =


obtenemos que

12 13
= 2G

x2
x3

y 13 =
en la expresin anterior
x3
x2

2 2
= 2G .
+
=
+
x3 x3 x2 x2 x22 x32

y si tenemos en cuenta la ecuacin (6.131) concluimos que:


2 2
+
= 2G = constante = F
x22 x32

Universidad de Castilla- La Mancha


Ciudad Real - Espaa

xr ( xr ) 2xr = 2G = constante

Draft

(6.133)

Por: Eduardo W. V. Chaves (2014)

MECNICA DEL MEDIO CONTINUO: PROBLEMAS RESUELTOS

556

donde 2xr es el Laplaciano.


Para completar el problema tenemos que definir las condiciones de contorno. Para el
problema de torsin la condicin de contorno es la ausencia de tensin normal en la
superficie externa del cuerpo prismtico, ver Figura 6.50.
x3

n i = 1ij

13

12

dx2

ds

0
dx

n i = n 2 = 3
n ds
3 dx2

ds

dx3

dx j

ds
x2

kij - smbolo de permutacin


ds - longitud de arco

Figura 6.50: Seccin transversal (barra prismtica).


Teniendo en cuenta el vector tensin en funcin de las componentes del tensor de
tensiones de Cauchy:

t (i n)

= ij n j

t 1(n ) 11
(n )
t 2 = 12
t (n )
3 13

t 1(n ) 0

t (2n) = 12
t (n )
3 13

12
22
23

13 n 1 0

23 n 2 = 0
33 n 3 0

13 0 12 n 2 + 13n 3 0


0
0 n 2 =
= 0

0
0 n 3

12
0
0

Con lo cual solo quedamos con la siguiente condicin de contorno:


12n 2 + 13n 3 = 0

Teniendo en cuenta las tensiones dadas por (6.127) la condicin anterior puede ser reescrita
como
12 n 2 + 13n 3 = 0

dx3 dx 2
=0
+

x3 ds x 2 ds



n2
n3 = 0
x 2
x 3

dx3 dx 2

=0
x3 ds x 2 ds

d
=0
ds

(6.134)

Con lo cual concluimos que es constante en el contorno y puede asumir cualquier valor
en el contorno, y adoptaremos como cero.
Vamos considerar que:
F3 ( x2 , x3 ) = 12 =

Universidad de Castilla- La Mancha


Ciudad Real - Espaa

x3

Draft

F2 ( x2 , x3 ) = 13 =

x2

(6.135)

Por: Eduardo W. V. Chaves (2014)

6 ELASTICIDAD LINEAL

557

La funcin ser compatible si:

= F2 ( x2 , x3 )
x2
compatible sii F2 F3
=

x
x2

3
= F3 ( x2 , x3 )

x3

(6.136)

Si consideramos el teorema de Green (ver Captulo 1 en Chaves (2007) 3 edicin) que


establece:
r r
r
r
F F

s F2 dx2 + F3dx3 = 2 3 dS1
F d = ( xr F) e 1dS componente

x3 x2

r
dS = dSe 1

x3

x1

e 1

x2

Figura 6.51: Teorema de Green.


Con lo cual concluimos que:

F
F


dS1 = 0
+ F3 dx3 = 2 3 dS1

dx 2 +
dx3 =

x
3
2
2
3
3
2
2
3

dx 2 +
dx 3 = xr dx = d = 0
x 3
x 2

F dx
2

Asumiremos valor cero en el contorno que tambin se cumple la condicin anterior.


Teniendo en cuenta el valor del momento torsor:
MT =

13 x2

12 x3 )dA = ( , 2 x2 + ,3 x3 )dA = ( ,i xi )dA

(i = 2,3)

notar que (xi ),i = ,i xi + xi ,i = ,i xi + 2 ,i xi = (xi ),i 2 , donde hemos aplicado la


propiedad de la delta de Kronecker xi ,i = ii = 22 + 33 = 2 . Con lo cual
M T = ( ,i x i )dA = ((x i ) ,i 2 )dA = (x i ) ,i dA +

= 2 A +

(2 )dA = (x )n d + (2 )dA
i

(2 )dA
A

Si consideramos que en el contorno el valor de es cero, i.e. = 0 , podemos obtener


que:

M T = 2 dA
A

Universidad de Castilla- La Mancha


Ciudad Real - Espaa

Draft

(6.137)

Por: Eduardo W. V. Chaves (2014)

MECNICA DEL MEDIO CONTINUO: PROBLEMAS RESUELTOS

558

El mismo desarrollo en notacin tensorial (2D) queda:


r
r
M T = ( ,i xi )dA = ( xr x )dA = x n d +

r
x

x )dA = 2 A

+ 2 dA

= 2 dA
A

donde hemos utilizando la propiedad 2 A = x n d , (ver Ejemplo 1.125).

NOTA 3: Analoga de Membrana de Prandtl


Considrese una membrana homognea atada en las extremidades, (ver Figura 6.52).
Aplicando una presin q , [q ] =

N
, sobre esta membrana surgir un estado de traccin S .
m2

A continuacin definiremos las ecuaciones de gobierno para este problema.


x1

ver Figura 6.54


Sdx3

u1 ( x2 , x3 )

Sdx3
x2

x3
x3
Sdx2
dx3
dx2

Sdx2

x1
x2

Figura 6.52: Membrana bajo presin

Universidad de Castilla- La Mancha


Ciudad Real - Espaa

Draft

Por: Eduardo W. V. Chaves (2014)

6 ELASTICIDAD LINEAL

559

Notar que la inclinacin de la membrana en el punto ( x2 , x3 ) viene dada por la derivada de


u1
u
y 1 , es corresponde a la tangente a la curva en este punto.
x3
x 2
u
u
Si denominamos por tan( 2 ) = 1 y tan( 3 ) = 1 , Si consideramos ngulos muy
x 2
x 3
pequeos se cumple que tan( ) sin( ) = . En un elemento diferencial dx 2 dx3 la

la funcin u1 ( x2 , x3 ) , i.e.

variacin de la tangente vienen dada como indica la Figura 6.53.

3 +

3
dx 3
x 3

3 +

( x 2 + dx 2 , x 3 + dx3 )

( x 2 , x 3 + dx3 )

2 +

3
3
dx 3 +
dx 2
x 3
x 2

2
dx 3
x 3

2 +
r

2
2
dx 2 +
dx 3
x 3
x 2

dx 3

2 +

( 3 )

( x 2 + dx 2 , x3 )

( x 2 , x3 )
x3

2
dx 2
x 2

( 2 )

3 +
dx 2

x2

3
dx 2
x 2

Figura 6.53: Variacin de la tangente en el elemento diferencial.


Si consideramos que no hay distorsin en la membrana, solo hay traccin, tenemos que
3
2
dx 2 = 0 y
dx 3 = 0 , (ver Figura 6.54).
x 2
x 3

Universidad de Castilla- La Mancha


Ciudad Real - Espaa

Draft

Por: Eduardo W. V. Chaves (2014)

MECNICA DEL MEDIO CONTINUO: PROBLEMAS RESUELTOS

560

Sdx 2

3 +

3
dx 3
x3
Sdx3

dx3

2 +

2
dx 2
x 2

dx 2
Sdx3

3
Sdx 2

2
2
Sdx3 sin 2 +
dx2 Sdx3 2 +
dx2
x2
x2

Sdx3

2 +
2
Sdx3

2
dx2
x2

Sdx 3 sin( 2 ) Sdx 3 2

Figura 6.54
Aplicando la condicin de equilibrio de fuerzas segn la direccin x1 obtenemos que:

x1

=0

u 2u

u 2u

u
u
qdx2 dx3 + Sdx3 sin 1 + 21 dx2 Sdx3 sin 1 + Sdx2 sin 1 + 21 dx3 Sdx2 sin 1 = 0

x
x
x
2
x3
2 x2

3 x3

u 2u

u 2u
u
u
qdx2 dx3 + Sdx3 1 + 21 dx2 Sdx3 1 + Sdx2 1 + 21 dx3 Sdx2 1 = 0
x2
x3

x3 x3

x2 x2
qdx2 dx3 + Sdx3
qdx2 dx3 +

u1 2u1
u
u 2u
u
+ 2 Sdx3dx2 Sdx3 1 + Sdx2 1 + 21 Sdx2 dx3 Sdx2 1 = 0
x2 x2
x2
x3 x3
x3

2u1
2u
Sdx3dx2 + 21 Sdx2 dx3 = 0
2
x2
x3

2u 2u
qdx2 dx3 + Sdx2 dx3 21 + 21 = 0
x3
x2

Universidad de Castilla- La Mancha


Ciudad Real - Espaa

Draft

Por: Eduardo W. V. Chaves (2014)

6 ELASTICIDAD LINEAL

561

con lo cual concluimos que la ecuacin de gobierno de la membrana bajo presin viene
dada por:
2u1 2u1 q
+ 2 =
S
x22
x3

xr ( xr u1 ) 2xr u1 =

q
S

(6.138)

Haciendo una analoga entre la ecuacin anterior con la ecuacin de torsin dada por
(6.133), podemos concluir que u1 = y 2G =

q
= constante . Con lo cual podemos decir
S

que el momento torsor (ver ecuacin (6.137)) est relacionado con el volumen formado
por la membrana y viene dado por:

M T = 2 dA = 2 u1 ( x2 , x3 ) dA = 2Vmemb
A

Ejemplo 6.37
a) Utiliza la Funcin de Tensin de Prandtl para demostrar que un seccin elptica
sometida a un momento torsor M T estar sometida a un estado tensional dado por:
12 =

2M T
x3
ab 3

13 =

2M T
x2
a 3b

(6.139)

b) Dibujar la distribucin de tensiones en la seccin transversal;


c) Obtener la funcin u1 ( x2 , x3 ) (alabeo).
x3

13
12
x2

Figura 6.55: Seccin elptica.


Solucin:
a) La ecuacin de la elipse viene dada por:
x22 x32
+
1 = 0
a2 b2

Ya que el valor de la funcin de tensin es constante en el contorno, podemos asumir


que:
x22

= m

Universidad de Castilla- La Mancha


Ciudad Real - Espaa

x32
1
2
b

Draft

(6.140)

Por: Eduardo W. V. Chaves (2014)

MECNICA DEL MEDIO CONTINUO: PROBLEMAS RESUELTOS

562

donde m es una constante a ser determinada. De la ecuacin anterior podemos obtener


que:
2x

= m 22
x 2
a
2x

= m 23
x3
b

2 2m
= 2
x 22
a

2 2m
= 2
x32
b

Reemplazando en la ecuacin (6.133) obtenemos que:


2
x 22

= 2G = constante = F

x 32

2( a 2 + b 2 )
=F
m
2 2

a b

2m 2m
+
=F
a2 b2

2
2
m 2 + 2
b
a

=F

a 2b 2
m=
F
2( a 2 + b 2 )

(6.141)

Reemplazando el valor de m en la ecuacin (6.140) obtenemos que:


x22

= m

x32

b2

a 2 b 2 x 22 x32
F

2(a 2 + b 2 ) a 2 b 2

(6.142)

Prximo paso: determinar F


Reemplazando el valor de en la ecuacin del momento torsor dada por (6.137) podemos
obtener que:
2 2
a 2 b 2

x 22 x 32

x 22 x32

F dA = a b

M T = 2 dA = 2
1
F
+ 2 1 dA
+

2
2 2
2
2
2
2


b
b
(a + b ) A a


A
A
2(a + b ) a
1

a 2b 2
a 2b 2
1
1
1

2
2
x
dA
x
dA
dA
= 2
+

=
F
F 2 I x 3 + 2 I x 2 A

2
3
2
2
2
2
2
b A
b
(a + b ) a A

(a + b ) a
A

donde:

I x 3 = x 22 dA - Momento de inercia de la seccin transversal con respecto al eje x3 ;


A

I x 2 = x32 dA - Momento de inercia de la seccin transversal con respecto al eje x2 ;


A

A=

dA - rea de la seccin transversal.


A

Para seccin elptica se cumple que:


I x3 =

ba 3
4

I x2 =

b 3 a
4

A = ab

Con lo cual la ecuacin del momento torsor queda:


MT =

1 ba 3

1
1 b 3 a
a 2b 2
a 2b 2
1

F
F
I
I
A
=
+
ab
2
2
2
2 x3
2 x2
2
2
2

4
4
(a + b ) a
b
b
(a + b ) a

a 3b 3
F
2( a 2 + b 2 )

Y a su vez el valor de F queda determinado:

Universidad de Castilla- La Mancha


Ciudad Real - Espaa

Draft

Por: Eduardo W. V. Chaves (2014)

6 ELASTICIDAD LINEAL

F=

563

2M T (a 2 + b 2 )
a 3b 3

La funcin de tensin (6.142) queda:

a 2 b 2 x 22 x32

+ 2 1F
2
2 2
b
2( a + b ) a

2
2

x
M x
= T 22 + 32 1
ab a
b

2M T (a 2 + b 2 )
a 2 b 2 x22 x32

a 3b 3
2(a 2 + b 2 ) a 2 b 2

Las tensiones definidas en (6.127) quedan


12 =

=
x3 x3

M T x22 x32

2M T

=
1
x3
+

2
2

b
ab 3
ab a

13 =
=
x2
x 2

(6.143)

M T x22 x32
2M T

=
x2
1
+

2
2
3

b
ab a
a b

b) A travs de las ecuaciones anteriores podemos obtener que:


x3 = 0

x2 = 0

12 = 0

2M T
2M

x2 = a )
(
13 max = 2T
13 = 3 x2
a b
a b

2M T
2M T

x2 = a )
(
12 max =
x
12 =
3 3
ab
ab 2

= 0
13

cuyas componentes podemos apreciar en la Figura 6.56. A travs del teorema de Pitgoras
la tensin tangencial resultante queda:
2

2M T
2M

x + 3T x 2
= ( 12 ) + (13 ) =
3 3
ab
a b
2

12 max =

2M T
ab 2

x 22

2M T
ab

b4

x 32
a4

x3

13 max =

2M T
a 2 b

x2

Figura 6.56: Distribucin de tensiones.

Universidad de Castilla- La Mancha


Ciudad Real - Espaa

Draft

Por: Eduardo W. V. Chaves (2014)

MECNICA DEL MEDIO CONTINUO: PROBLEMAS RESUELTOS

564

c) Podemos obtener el ngulo de torsin unitario a travs de la ecuacin (6.141):

F = 2G

2M T (a 2 + b 2 )


3 3
M (a 2 + b 2 )
a
b

F
= T
=
=
2G
2G
a 3b 3G

Teniendo en cuenta los campos de desplazamientos dados por (6.115), podemos obtener
que:

M T (a 2 + b 2 )
=

u
x
x
x3 x1
2
3
1

a 3b 3G

2
2
u = x x = M T (a + b ) x x
2
1
2 1
3
a 3b 3G

(6.144)

y si adems tenemos en cuenta las ecuaciones en (6.132) podemos decir que:


u1 12
=
+ x3
G
x2
u1 13
=
x2
G
x3

u1 12 M T (a 2 + b 2 )
x3
=
+
G
x2
a 3b 3G

u1 13 M T (a 2 + b 2 )
=

x2
G
x3
a 3b 3G

(6.145)

Integrando las ecuaciones anteriores obtenemos que:

M (a 2 + b 2 )
u1 = 12 + T 3 3
x3
a b G
G

x2

u1 =

M (a 2 + b 2 )
12
x2 + T 3 3
x3 x2 + f ( x3 )
G
a b G

M (a 2 + b 2 )
x2 x3
u1 = 13 T 3 3
a b G
G

u1 =

13
M (a 2 + b 2 )
x3 T 3 3
x3 x2 + f ( x2 )
G
a b G

Reemplazando los valores de 12 y 13 (ver ecuacin (6.143)) en la ecuacin anterior


obtenemos que:
M T (a 2 + b 2 )
M x x
2M T 1
u1 =
+
x
x
x3 x2 + f ( x3 ) = T 33 32 (b 2 a 2 ) + f ( x3 )

2
3 3
3 3
a b G
Ga b
ab
G
M x x
M (a 2 + b 2 )
2M
1
u1 = 3T x2 x3 T 3 3
x3 x2 + f ( x2 ) = T 33 32 (b 2 a 2 ) + f ( x2 )
G
a b G
Ga b
a b

Como las dos ecuaciones tienen que ser iguales en el mismo punto ( x2 , x3 ) , concluimos
que f ( x2 ) = f ( x3 ) = 0 , con lo cual:
u1 ( x2 , x3 ) =

M T (b 2 a 2 )
x2 x3
G a 3b 3

La funcin anterior en la seccin transversal se puede apreciar en la Figura 6.57.


Para el caso particular a = b , recuperamos las relaciones para la seccin circular, (ver
NOTA 1 del Ejemplo 6.36). Notar tambin que en este caso no hay alabeo ya que
u1 ( x2 , x3 ) = 0 .

Universidad de Castilla- La Mancha


Ciudad Real - Espaa

Draft

Por: Eduardo W. V. Chaves (2014)

6 ELASTICIDAD LINEAL

565

x3
u1 negativo (baja)

u1 positivo (sube)

x2

Figura 6.57: Funcin u1 ( x2 , x3 ) .

6.5 Energa de Deformacin para Elementos 1D


Ejemplo 6.38
Obtener la energa total de deformacin para un elemento de viga de longitud L donde
actan los esfuerzos esquematizados en la Figura 6.58.
z, w

Mz

y, v

My

Qz

Qy

M x MT

x, u

Figura 6.58: Esfuerzos internos en un elemento tipo barra.

Solucin:
La energa de deformacin de la barra viene dada por:
U int =

1
1
ij ij dV
: dV =
2V
2V

1
(11 11 + 22 22 + 33 33 + 212 12 + 2 23 23 + 213 13 )dV
2V

1
( x x + y y + z z + xy xy + yz yz + xz xz )dV
2V

En una seccin transversal de la viga la componente normal x viene representada


x = (x1) + (x2) + (x3) . La energa asociada a la tensin normal (x1) = E (x1) (Figura 6.32)
puede ser expresada en funcin del esfuerzo normal:

Universidad de Castilla- La Mancha


Ciudad Real - Espaa

Draft

Por: Eduardo W. V. Chaves (2014)

MECNICA DEL MEDIO CONTINUO: PROBLEMAS RESUELTOS

566

int

(1)
1
1 x
1
=
(x1) (x1) dV =
dV =
2V
2V E
2

N2

EA
2

dAdx =

1
2

N2
dx
EA

(6.146)

Anlogamente, podemos obtener la energa de deformacin asociada a la tensin normal


(x2) = E (x2 ) , (ver ecuacin (6.93)):
U

int

1
1
=
(x2 ) (x2 ) dV =
2V
2

My

Iy

My

1
z
zdAdx =
EI y
2

M y2

1
z dAdx =
2
2
EI y A

M y2

EI

dx

(6.147)

Anlogamente, si consideramos (x3) (ver ecuacin (6.94)), obtenemos:


U int =

1
2

M z2
dx
EI z

(6.148)

La componente xz ( z ) = G xz est asociada con el esfuerzo cortante Q z , (ver ecuacin


(6.97)), donde G es el mdulo de elasticidad transversal. En la ecuacin (6.108) hemos
obtenido que:

( z, y)dy =

Qz
Q
zdA = z z
Iy A
Iy

donde

z = zdA

(6.149)

Luego, la energa de deforma asociada con xz ( z ) = G xz viene dada por:


U

int

2
1
1 xz
1 1 2
1 1
dV =
=
dV =
dAdx =
2V
2V G
2 0 GA
2 0 G

1 1
=
2 0 G

Qz z

Iy

2
L

dz dx = 1 Q z z

2 0 G I y

2 dydz dx

(6.150)

2
L

dz dx = 1 z Q z dx

2 0 GA

donde z es el factor de forma de la seccin y viene dado por:

z = A z
Iy

dz

(6.151)

Anlogamente podemos obtener que:


U

int

2
2
L
1
1 xy
1 yQy
dV =
dx
=
xy xy dV =
2V
2V G
2 0 GA

y
y = A
I
A z

donde

dA

(6.152)

Por ltimo, la energa de deformacin debido al esfuerzo momento torsor. Si consideramos


una seccin transversal circular (ver ecuacin (6.125)) se cumple que:
( r ) =

MT
r
J

Considerando la energa en la seccin circular debido a tensin tangencial (r ) que est


relacionada con la deformacin tangencial por ( r ) = G ( r ) :
U

int

2
2
L
1
1 1 MT
1 M T2
1
1
dV =
r dV =
r 2 dAdx =
=
(r ) (r )dV =

2
2V G
2V G J
2 0 GJ A
2
2V

M T2
dx
GJ

(6.153)

Universidad de Castilla- La Mancha


Ciudad Real - Espaa

Draft

Por: Eduardo W. V. Chaves (2014)

6 ELASTICIDAD LINEAL

567

donde J = r 2 dA es el momento polar de inercia. Para una seccin rectangular podemos


A

obtener una relacin equivalente:


L

U int =

2
1 MT
dx
2 0 GJ Eq

(6.154)

donde J Eq es el momento de inercia a torsin equivalente.


Luego, la energa total de deformacin para un elemento de viga viene dada por:
U int =

2
2
L
M T2
1 N 2 M y M z2 y Q y z Qz2
+
+
+
+
+
GA
GA
EJ Eq
2 0 EA EI y EI z

dx

(6.155)

NOTA 1: Energa potencial externa


A continuacin proporcionamos algunas expresiones para el clculo de la energa potencial
externa.
Carga Concentrada: U ext = Pw p , donde P - Carga concentrada y w p - desplazamiento
segn direccin de P .
y

deflexin de la lnea neutra

wp
x

Figura 6.59: Carga concentrada.

Carga uniformemente distribuida: U

ext

= q( x) w( x)dx , donde q ( x) - Carga distribuida,


0

w p - desplazamiento segn direccin de q ( x) .

deflexin de la lnea neutra

y
z
w( x)

q ( x)

Figura 6.60: Carga uniformemente distribuida.

Universidad de Castilla- La Mancha


Ciudad Real - Espaa

Draft

Por: Eduardo W. V. Chaves (2014)

MECNICA DEL MEDIO CONTINUO: PROBLEMAS RESUELTOS

568

En el caso que q ( x) sea constante dentro del dominio, es decir no es una funcin de x , el
L

potencial queda U ext = q w( x)dx


0

dw
A
= M y wA
dx

Momento concentrado: U ext = M yA


My

wA

Figura 6.61: Momento concentrado en el punto A .


NOTA 2: Ejemplo de aplicacin. Vamos considerar en un elemento de viga con los
esfuerzos y grados de libertad (desplazamientos) indicados en la Figura 6.62.
Vamos considerar que a cada nodo est asociado a dos desplazamientos: desplazamiento
vertical ( w ) y una rotacin ( y =

w1

dw
w ).
dx

y1 = w1

{u }
(e )

y 2 = w2

w2

w1

y1
=
w2
y 2

a) Desplazamientos
y

Fz1

M y1

{f }

M y2

Fz 2

(e)

F z1
M
y1
=

Fz 2
M y 2

b) Esfuerzos

Figura 6.62: elemento finito tipo viga.

Universidad de Castilla- La Mancha


Ciudad Real - Espaa

Draft

Por: Eduardo W. V. Chaves (2014)

6 ELASTICIDAD LINEAL

569

Vamos adoptar como funcin aproximada para la deflexin de la viga un polinomio de


tercer grado:
(6.156)

w = ax 3 + bx 2 + cx + d

La primera derivada de la deflexin viene dada por:


dw
w = 3ax 2 + 2bx + c
dx

(6.157)

Luego, para los extremos de la viga (nodos 1 y 2 ) quedamos con:


x = 0 ( w = w1 )

w1 = d

x = 0 ( w = w1 )

dw
= w1 = c
dx

x = L ( w = w2 )

w2 = aL3 + bL2 + cL + d

x = L ( w = w2 )

w2 = 3aL2 + 2bL + c

(6.158)

(6.159)

donde w1 , w2 indican la deflexin en los nodos 1 y 2 respectivamente. El sistema anterior


en forma matricial quieda:
w1 0
w 0
1 =
w2 L3
2
w2 3L

2L
0 1 a
a

1 0 b inversa b 1 3L2
=

c L4 0
L2 L 1 c
4


2 L 1 0 d
L
d
0
0

Con eso concluimos que a =

L2

2L

2 L3
L4

3L2
0

L2 w1

L3 w1
0 w2

0 w2

(6.160)

2
(w1 w2 ) + 12 (w2 + w1 ) ; b = 32 (w2 w1 ) 1 (w2 + 2w1 ) ;
3
L
L
L
L

c = w1 ; d = w1 .

Sustituyendo los valores de a , b , c y d en la expresin de la deflexin de la viga (6.156),


obtenemos que:
2
2
x 3

x 3
x3 2x 2

x3 x2
x
x
+ x + w2 2
w = w1 2 3 + 1 + w2 2 + 3 + w1 2
L
L
L
L
L

L
L

(6.161)
Recordar que hemos adoptado como grados de libertad en los nodos la rotacin y (de
acuerdo con nuestro sistema de coordenadas adoptado, ver Figura 6.62), y no la primera
derivada de la deflexin w' . Pero, ellas estn relacionadas entre si a travs de la expresin
y = w' . Con eso, la expresin de la deflexin queda:
2
2
x 3

x 3
x 3 2x 2

x3 x2
x
x
w = w1 2 3 + 1 + w2 2 + 3 y1 2
+ x y 2 2
L
L
L
L

L
L
L

(6.162)
La primera derivada viene dada por:
6x 2 6x
6x 2 6x
3x 2 4 x
3x 2 2 x
w = w1 3 2 + w2 3 + 2 y1 2
+ 1 y 2 2
L
L
L
L
L
L
L

(6.163)

La segunda derivada queda:

Universidad de Castilla- La Mancha


Ciudad Real - Espaa

Draft

Por: Eduardo W. V. Chaves (2014)

MECNICA DEL MEDIO CONTINUO: PROBLEMAS RESUELTOS

570

6x
4
12 x 6
12 x 6
6x 2
w = w1 3 2 + w2 3 + 2 y1 2 y 2 2
L
L
L
L
L
L
L
L

(6.164)

Ser de utilidad obtener las siguientes integrales:


L

w( x)dx =

w 2 dx =

0
L

L
L
L2
L2
w1 + w2
y1 +
y2
2
2
12
12

(6.165)

13L 2 13L 2 L3 2
9L
11L2
13L2
L3 2
w1 +
w2 +
y1 +
y2 +
w1 w2
w1 y1 +
w1 y 2 + L
35
35
105
105
35
105
210

13L2
11L2
L3
w 2 y1 +
w2 y 2
y1 y 2
210
105
70

(6.166)
L

w dx = 5L w
2

2
1

6 2 2L 2 2L 2
12
1
1
w2 +
y1 +
y2
w1 w2 w1 y1 w1 y 2 + L
5L
15
15
5L
5
5
L+

1
1
L
w2 y1 + w2 y 2 y1 y 2
5
5
15

(6.167)
L

12

w dx = L
2

w12 +

12 2 4 2
4
24
12
12
w2 + y1 + y22 3 w1 w2 2 w1 y1 2 w1 y 2 + L
3
L
L
L
L
L
L
L+

12
12
4
w2 y1 + 2 w2 y 2 + y1 y 2
2
L
L
L

(6.168)
L

x w( x)dx =
0

3L
7L
L
L
w1 +
w2
y1 +
y2
20
20
30
20

(6.169)

Ejemplo 1: Consideremos un elemento de viga, con la rigidez a flexin EI y constante,


sometido a una carga uniformemente distribuida de intensidad q .
z

Figura 6.63: Viga sometida a una carga uniforme.


La energa potencial total del sistema viene dada por:
=U

int

ext

1
=
2

M y2

EI
0

Universidad de Castilla- La Mancha


Ciudad Real - Espaa

dx qw( x)dx =
0

Draft

EI y
2

w 2 dx qw( x) dx

(6.170)

Por: Eduardo W. V. Chaves (2014)

6 ELASTICIDAD LINEAL

571

donde hemos tenido en cuenta la ecuacin (6.101), i.e. M y = EI y w . Si estamos


considerando que q es independiente de x , la energa potencial de las fuerzas externas
resulta:
L

U ext = qw( x)dx = q w( x)dx


0

(6.171)

Sustituyendo la ecuacin (6.165) en la ecuacin anterior obtenemos la expresin de U ext en


funcin de los parmetros nodales w1 , w2 , y1 y y 2 , i.e.:
L

L
L
L2
L2
U ext = q w( x) dx = q w1 + w2
y1 +
y 2
2
12
12

2
0

(6.172)

Considerando que EI y es constante dentro del elemento, la energa potencial de las fuerzas
internas viene dada por:
U

int

EI y
2

dx

(6.173)

Utilizando la expresin obtenida en (6.168) la ecuacin anterior resulta:


U int =

EI y
2

w
0

dx =

EI y 12 2 12 2 4 2 4 2 24
12
3 w1 + 3 w2 + y1 + y 2 3 w1 w2 2 w1 y1 L
L
L
2 L
L
L
L
L

12
2

w1 y 2 +

12
2

w 2 y1 +

12
2

w2 y 2 +

y1 y 2
L

(6.174)
Luego, la energa potencial total (6.170) queda:
=

EI y 12 2 12 2 4 2
4 2
24
12
12
3 w1 + 3 w2 + y1 + y 2 3 w1 w2 2 w1 y1 2 w1 y 2 + L
L
L
2 L
L
L
L
L
L

L
L2
L2
12
12
4

L + 2 w2 y1 + 2 w2 y 2 + y1 y 2 q w1 + w2
y1 +
y 2
L
2
12
12
L
L

(6.175)
Segn el principio del valor estacionario de la energa potencial total hay que cumplir que:

=0
w1

EI y 24
L
24
12
12

3 w1 3 w2 2 y1 2 y 2 q = 0
2 L
2
L
L
L

(6.176)

=0
y1

EI y 8
12
12
4
L2

=0
y1 2 w1 + 2 w2 + y 2 + q
2 L
L
12
L
L

(6.177)

=0
w2

EI y 24
L
24
12
12

3 w2 3 w1 + 2 y1 + 2 y 2 q = 0
2 L
2
L
L
L

(6.178)

EI y 8
12
12
4
L2

w
+
w
+

q
=0

1
2
y2
y1
2 L
L
12
L2
L2

(6.179)

=0
y 2

Reestructurando las expresiones anteriores en forma de matriz, obtenemos que:

Universidad de Castilla- La Mancha


Ciudad Real - Espaa

Draft

Por: Eduardo W. V. Chaves (2014)

MECNICA DEL MEDIO CONTINUO: PROBLEMAS RESUELTOS

572

12 EI
y

3
L

6 EI y
L2

12 EI y
L3

6 EI y
L2

6 EI y

12 EI y

L
4 EI y

L
6 EI y

L
6 EI y

L2
12 EI y

L2
2 EI y

L3
6 EI y

L2

6 EI y
qL

L
2
w1
2 EI y qL2

L y1 = 12
w qL
6 EI y 2

L2 y 2 2 2

qL
4 EI y
12
L

(6.180)

o aun de forma ms compacta:

[Ke ] {u }= {f }
(1)

(e )

(6.181)

(e)

con:

[Ke ]
(1)

12 EI
y

L3

6 EI y
2

= L
12 EI y
L3

6 EI y
L2

6 EI y

12 EI y

L
4 EI y

L
6 EI y

L
6 EI y

L2
12 EI y

L2
2 EI y

L3
6 EI y

L2

6 EI y

L2

2 EI y
L

6 EI y
L2

4 EI y
L

{f }
(e)

qL

2
qL2

= 12
qL
2
2
qL
12

(6.182)

donde [ Ke (1) ] es la matriz de rigidez del elemento finito propuesto. Es interesante destacar
que la matriz [ Ke (1) ] no tiene inversa ( det[ Ke (1) ] = 0 ), ya que estamos tratando con una
estructura hipoesttica, en este caso la estructura tiene infinitas soluciones ya que no hemos
restringido ninguno de sus movimientos. Para que (6.181) tenga solucin nica tenemos
que aplicar las condiciones de contorno. Podemos llegar al mismo resultado (6.180) por
medio del teorema de los trabajos virtuales, ver Chaves & Mnguez(2010) pg. 296.
Considerando el mismo ejemplo, ver Figura 6.63 y con las siguientes condiciones de
contorno descritas en la Figura 6.64.

w = 0

Nodo 1: 1
w1 = 0

deflexin
x

1
q

w 0

Nodo 2: 2
w2 0

Figura 6.64: Viga empotrada sometida a una carga uniforme.

Universidad de Castilla- La Mancha


Ciudad Real - Espaa

Draft

Por: Eduardo W. V. Chaves (2014)

6 ELASTICIDAD LINEAL

573

Aplicando las condiciones de contorno a la ecuacin (6.180) resulta que:


1

0
1
0

0
0
12 EI y

L3
6 EI y
L2

w 0
1
0
y1 qL
=
L2 w2 2
2

4 EI y y 2 qL

12
L

0
0
6 EI y

(6.183)

Al resolver el sistema anterior obtenemos que:


0
w1 0

4
y1 qL
= 8 EI
y
w2
qL3
y2
6 EI y

(6.184)

El momento en el nodo 1 viene dado por M y1 = EI y w1 . Utilizando la ecuacin (6.164)


obtenemos:
6x
4
6x 2
12 x 6
12 x 6
w = w1 3 2 + w2 3 + 2 y1 2 y 2 2
L
L
L
L
L
L
L
L
6
2
w( x = 0) = w1 = w2 2 y 2
L
L
w1 =

qL4 6 qL3 2

8 EI y L2 6 EI y L

2
5 qL
w1 =
6 2 EI y

Luego, el momento queda:


2
2
5 qL
5 qL
M y1 = EI y w1 = EI y
=
6 2
6 2 EI y

=
Si comparamos con el valor exacto M yexacto
1
16,6% .

(6.185)

qL2
, verificamos que hay un error de
2

Esfuerzos en el Elemento Viga


Una vez obtenido los desplazamientos (6.184) llega el momento de obtener los esfuerzos
en el elemento de viga. A travs de los desplazamientos, calculamos las fuerzas internas
segn la expresin (6.181), i.e.:

{f }= [Ke ] {u }
(e)

(1)

(e)

(6.186)

resultando:

Universidad de Castilla- La Mancha


Ciudad Real - Espaa

Draft

Por: Eduardo W. V. Chaves (2014)

MECNICA DEL MEDIO CONTINUO: PROBLEMAS RESUELTOS

574

{f }
(e)

12 EI
y

3
L

6 EI y
2

= L
12 EI y
L3

6 EI y
L2

6 EI y

12 EI y

L
4 EI y

L
6 EI y

L
6 EI y

L2
12 EI y

L2
2 EI y

L3
6 EI y

L2

6 EI y

L2 0

2 EI y 0
4
L qL

6 EI y 8 EI y
3

L2 qL

4 EI y 6 EI y
L

qL

2
5qL2

12
=
qL
2
2
qL
12

(6.187)

Si estuviramos tratando con elemento finito tradicional, los esfuerzos en el elemento de


viga seran el proporcionado por { f (e ) } , ver Figura 6.65(a). A medida que refinamos el
error disminuye. A travs del Anlisis de Estructuras, la solucin exacta de este ejemplo
viene dada por la Figura 6.65(c), y verifiquemos que las reacciones en las extremidades
del elemento viga vienen dados por:

{R }= {f }+ {~f
(e)

(e )

M y1 =

a) {f (e ) }

(e)

5qL2
12

} = {f

{~ }

} {f }

(6.188)

(e)

M y2 =

qL
Fz 1 =
2

b) f ( e ) = {f ( e ) }

(e)

Fz 2 =

qL2
12

qL
2

+
M y1 =

qL2
12

M y2 =

qL
Fz 1 =
2

Fz 2 =

qL2
12

qL
2

=
c) Solucin exacta

M y1 =

qL2
2

M y2 = 0

Fz 1 = qL

Fz 2 = 0

Figura 6.65: Reacciones en el elemento viga.


~

El vector { f ( e ) } = { f (e ) } , ver ecuacin (6.182), en el Anlisis de Estructuras, recibe el


nombre de acciones de extremidad para miembros restringidos. Es decir, la solucin
proporcionada por la Figura 6.65(b) son las reacciones que surgen si la viga estuviera
empotrada en las dos extremidades, Gere&Weaver (1965), ver Figura 6.66. Y estas
Universidad de Castilla- La Mancha
Ciudad Real - Espaa

Draft

Por: Eduardo W. V. Chaves (2014)

6 ELASTICIDAD LINEAL

575

reacciones son las mismas obtenidas para el vector de fuerzas nodales equivalentes, ver
ecuacin (6.182).
Fz 1 =

qL
2

Fz 2 =

qL
2

q
M y1 =

qL
12

M y2 =

qL2
12

Figura 6.66: Reacciones en viga biempotrada.


Solucin Analtica (Integracin Directa)
En esta sub-seccin vamos obtener la solucin analtica del Ejemplo 1, para ello
utilizaremos la Integracin Directa. Consideraremos el momento flector, ver Figura 6.67:
M y ( x) =

qx 2
qL2
+ qLx
2
2
x
2

qx

My <0
z

0 xL

Fz 1 = qL
q

M y1 =

qL2
2

x
M y ( x) =

qL2
x
+ qLx qx
2
2

Figura 6.67: Viga empotrada.


Recordar que la ecuacin diferencial de la deflexin de la viga, ver ecuacin (6.101), viene
dada por:
M y = EI y w EI y

d 2w
dx 2

EI y

d 2w
dx 2

qx 2
qL2
+ qLx
2
2

A travs de integracin directa podemos obtener que:


EI y

d 2w
dx 2

qL2
qx 2
qLx +
2
2

integrando

EI y

qLx 2 qx 3
dw qL2
=
x
+
+ C1
2
2
6
dx

La constante de integracin C1 obtenemos a travs de la condicin de contorno


dw
w( x = 0) = 0 con lo cual obtenemos que C1 = 0 . Con lo cual la pendiente de la viga
dx

viene definida por:

q
dw
w =
3L2 x 3Lx 2 + x 3
6 EI y
dx

Universidad de Castilla- La Mancha


Ciudad Real - Espaa

Draft

)
Por: Eduardo W. V. Chaves (2014)

MECNICA DEL MEDIO CONTINUO: PROBLEMAS RESUELTOS

576

Integrando una vez ms obtenemos la deflexin de la viga w( x) :

q
dw
=
3L2 x 3Lx 2 + x 3
dx 6 EI y

integrando

w =

3L2 x 2 3Lx 3 x 4

2 3 + 4

q
6 EI y

+ C2

donde la constante de integracin C 2 se obtiene en w( x = 0) = 0 C 2 = 0 . Con lo cual, la


deflexin de la viga viene definida por:
w( x) =

qx 2
6 L2 4 Lx + x 2
24 EI y

Para x = L los valores de la deflexin y de la rotacin quedan:


deflexin: w( x = L) =
rotacin:

qx 2
qL2
qL4
6 L2 4 Lx + x 2 =
6 L2 4 L2 + L2 =
24 EI y
24 EI y
8 EI y

q
q
qL3
dw
w( x = L) =
3L2 x 3Lx 2 + x 3 =
3L3 3L3 + L3 =
dx
6 EI y
6 EI y
6 EI y

w( x = L ) =

w( x = L) =

qL3
6 EI y

qL4
8 EI y

Figura 6.68: Desplazamientos de una viga empotrada.


ELEMENTO FINITO DE VIGA SOBRE BASE ELSTICA
Consideremos ahora que la viga est sobre una base elstica, ver Figura 6.69.
z
y
q<0
x
w( x) < 0
Kf

Figura 6.69: Viga sobre base elstica.

Universidad de Castilla- La Mancha


Ciudad Real - Espaa

Draft

Por: Eduardo W. V. Chaves (2014)

6 ELASTICIDAD LINEAL

577

En esta situacin, la energa potencial total del sistema viene dada por:
= U int U ext = (U viga + U muelle ) U ext

(6.189)

Segn el principio del valor estacionario de la energa potencial total hay que cumplir que:
d

=0

d {u (e ) }

d {u (e ) }

(U

viga

+ U muelle U ext = 0

dU viga
d {u (e ) }

dU muelle
d {u ( e ) }

dU ext
d{u ( e ) }

=0

(6.190)
o an
dU viga
(e )

d {u }

dU muelle
(e)

d {u }

dU ext
(e)

d {u }

=0

[ke ]{u }+ dU
d {u
(1)

(e)

muelle
(e )

{ }

= f

(e)

(6.191)

donde [ke (1) ] es la misma matriz de rigidez dada en (6.182). Luego, solo falta determinar el
trmino

dU muelle
d {u (e ) }

. Para un muele, ver Figura 6.70, la energa de deformacin viene dada

por:
L

U muelle =

2K

w 2 dx

(6.192)

F
F

energa almacenada

1
1
wF = wK f w
2
2

w
Kf
1

Kf

Figura 6.70: Elemento tipo muelle.


Considerando que el coeficiente de muelle K f es constante dentro del elemento, la
expresin (6.192) queda:
U muelle =

Kf
2

w dx
2

(6.193)

Adems utilizando la ecuacin (6.166) obtenemos que:


U muelle =

K f 13L 2 13L 2 L3 2
11L2
L3 2 9 L

w
w
w
w
w1 y1 + L
+
+

1
2
y1
y2
1 2
2 35
35
105
105
35
105
L

13L2
13L2
11L2
L3
w1 y 2
w2 y1 +
w2 y 2
y1 y 2
210
210
105
70

(6.194)

Universidad de Castilla- La Mancha


Ciudad Real - Espaa

Draft

Por: Eduardo W. V. Chaves (2014)

MECNICA DEL MEDIO CONTINUO: PROBLEMAS RESUELTOS

578

Luego

U muelle K f 26 L
9L
11L2
13L2
=
+

+
y2
w
w

1
2
y1
w1
2 35
35
105
210

3
2
2
3

U muelle K f 2 L
11L
13L
L
=
y1
y2
w1
w2

2 105
105
210
70
y1

2
2

U muelle K f 26 L
9L
13L
11L
=
y1 +
y2
w2 +
w1

w2
2 35
35
210
105

3
2
2
3

U muelle K f 2 L
13L
11L
L
=
y2 +
y1
w1 +
w2

2 105
210
105
70
y 2

(6.195)
(6.196)
(6.197)
(6.198)

Reestructurando las expresiones anteriores en forma de matriz, obtenemos que:


11L
13
35
210

L
L2
11

dU muelle
105
= K f L 210
9
13L
d {u (e ) }

420
70
L2
13L
420
140
dU muelle
= Ke ( 2 ) {u (e ) }
(e )
d {u }

9
70
13L
420
13
35
11L
210

13L
420 w
1
L2

140 y1

11L w2

210
L2 y 2
105

(6.199)

donde

[Ke ]
( 2)

13
35

11L
= K f L 210
9

70
13L
420

11L
210
L2
105
13L
420
L2
140

9
70
13L
420
13
35
11L
210

13L
420

L2
140
11L

210
L2
105

(6.200)

Ejemplo 6.39
Consideremos una viga de longitud L donde actan los esfuerzos esquematizados en la
Figura 6.71. Aplicar el Principio del trabajo virtual complementario a la viga, (ver Ejemplo
5.16 NOTA 4).
z, w

Mz

y, v

My
Qz

Qy
N

M x MT

x, u

Figura 6.71: Esfuerzos internos en un elemento tipo barra.

Universidad de Castilla- La Mancha


Ciudad Real - Espaa

Draft

Por: Eduardo W. V. Chaves (2014)

6 ELASTICIDAD LINEAL

579

OBS.: Considerar un problema esttico sin fuerzas msicas, y que la viga est sometida
nicamente a fuerzas concentradas.
Solucin:
En el Ejemplo 5.16 NOTA 4 hemos demostrado que:

r r
&r& r
t u* dS ur + (b u ) udV =

S ur

: dV

Principio del trabajo virtual


complementario

V
14243

1444442444443

Trabajo interno
virtual complementario total

Trabajo externo virtual complementario total

(6.201)

con n = t * en S . Considerando un caso esttico sin fuerzas msicas y considerando


que las nicas acciones externas son fuerzas concentradas, el principio del trabajo virtual
complementario viene dado por:
r
loc r loc
F
u43
142

Trabajo externo virtual


complementario total
(Fuerzas concentradas)

Principio del trabajo virtual


complementario (Caso esttico,
sin fuerzas msicas y fuerzas
concentradas)

: dV

V
14243

Trabajo interno
virtual complementario total

(6.202)

El trabajo interno:

Wint = : dV = ij ij dV = ( 11 11 + 22 22 + 33 33 + 2 12 12 + 2 23 23 + 213 13 )dV


V

= ( x x + y y + z z + xy xy + yz yz + xz xz )dV
V

En el ejemplo anterior hemos obtenido las tensiones en funcin de los esfuerzos (ver
Ejemplo 6.38):
x = (x1) + (x2) + (x3) :

(x1) (x1) dV

x( 2) (x2 ) dV =

x(1)

(x1)
dV =
E
My
Iy

( 3) ( 3)
x x dV

= Mz

My
EI y

N N
N
dAdx = N
dx
A EA A
EA
0

My

zdAdx = M y
0

EI y2 A

z 2 dAdx = M y
0

My
EI y

dx

Mz
dx
EI z

La componente xz ( z ) = G xz est asociada con el esfuerzo cortante Q z , (ver ecuacin


(6.97)), donde G es el mdulo de elasticidad transversal. En la ecuacin (6.108) hemos
obtenido que:

( z, y)dy =

Q
Qz
zdA = z z
Iy A
Iy

donde

z = zdA

(6.203)

Luego, la energa de deforma asociada con xz ( z ) = G xz viene dada por:

Universidad de Castilla- La Mancha


Ciudad Real - Espaa

Draft

Por: Eduardo W. V. Chaves (2014)

MECNICA DEL MEDIO CONTINUO: PROBLEMAS RESUELTOS

580

xz
1
1
dV =
dAdx =
G
GA
G
0
0

dydz dx

2
L
L
L

Q z z
Q
1 Q z z Q z z

dz dx = Q z
dz dx = Q z z z dx
=

G I y I y
G Iy
GA

0
0
0

dV =

2
xz

(6.204)

donde z es el factor de forma de la seccin y viene dado por:

z = A z
Iy

dz

(6.205)

Anlogamente podemos obtener que:

xy xy dV

= xy
V

xy
G

dV = Q y

yQy
GA

dx

donde

y
y = A
I
A z

dA

(6.206)

Por ltimo, el trabajo interno realizado por el esfuerzo momento torsor:

(r ) (r )dV =

dV =

L
L
MT
M
1 M T M T
2

r
r
dV
=
M
r
dA
dx
=
M T T dx

T
2

G J J
GJ
GJ A
0
0

(6.207)
donde J = r 2 dA es el momento polar de inercia. Para una seccin rectangular podemos
A

obtener una relacin equivalente:


L

M
0

MT
dx
GJ Eq

(6.208)

donde J Eq es el momento de inercia a torsin equivalente.


Con lo cual el trabajo interno queda:

Wint = ( x x + y y + z z + xy xy + yz yz + xz xz )dV
V

L
My
yQy
M
MT
Q
N
= N
+My
+ M z z + Qz z z + Q y
+ MT
EA
EI y
EI z
GA
GA
GJ Eq
0

dx

Con lo cual el Principio del Trabajo Virtual Complementario queda:


r
loc r loc
F
u43
142

Trabajo externo virtual


complementario total
(Fuerzas concentradas)

: dV

V
14243

Trabajo interno
virtual complementario total

L
r
yQy
My
Q
M
MT
N
loc r loc
F u = N
+My
+ M z z + Qz z z + Q y
+ MT
EA
EI y
EI z
GA
GA
GJ Eq
0

Universidad de Castilla- La Mancha


Ciudad Real - Espaa

Draft

dx

Por: Eduardo W. V. Chaves (2014)

6 ELASTICIDAD LINEAL

581

NOTA: Como ejemplo de aplicacin, vamos aplicar el principio del trabajo virtual
complementario para obtener la deflexin en el extremo libre del voladizo, ver Figura 6.72.
My >0
z
q
x
x

M y ( x) =

qx 2
2

Figura 6.72: Viga empotrada.


En esta situacin el principio del trabajo virtual complementario queda:
L
r
My
r
dx
F loc u loc = M y
EI
y
0

Como deseamos obtener la deflexin de la viga en x = 0 , vamos aplicar una carga


concentrada virtual en este punto de valor unitario F = 1 . Los diagramas de momentos
flectores para el campo real y virtual se presentan en la Figura 6.73.
REAL

M y ( x) =

VIRTUAL

qx 2
2

M y ( x) = x
F =1

Figura 6.73: Viga empotrada.


Con eso podemos concluir que:
r
My
r
F loc uloc = M y
dx
EI y
0

qx 2
L
My
q
2
= My
x 3 dx
dx = x
dx =
2 EI y 0
EI y
EI y
0
0
L

F U(1)

Teniendo en cuenta que F = 1 y resolviendo la integral podemos obtener que:


Universidad de Castilla- La Mancha
Ciudad Real - Espaa

Draft

Por: Eduardo W. V. Chaves (2014)

MECNICA DEL MEDIO CONTINUO: PROBLEMAS RESUELTOS

582

U(1) =

q
2 EI y

x 3 dx =

qL4
8 EI y

Que coincide con el resultado presentado en la Figura 6.68.


Ejemplo 6.40
Considrese una barra de seccin cuadrada de lado a y longitud L . Las constantes
elsticas del material se suponen conocidas ( E y = 0,25 ). Se pide:
a) Para el caso de carga de la Figura 6.74(a), calcular la energa almacenada (densidad de
energa de deformacin) en la barra durante la deformacin y la energa total de
deformacin;
b) Determinar la energa almacenada en el cambio de volumen y la correspondiente al
cambio de forma;
c) Mismo apartado a) para el caso de la Figura 6.74(b).

P
a

b)

a)

seccin

Figura 6.74:
Solucin:
Considerando un caso unidimensional:
x = E x x =

x
E

(6.209)

Sabemos que la densidad de energa de deformacin (por unidad de volumen) viene dada
por:
1
2

1
2

1
2

e = : unidimensi
onal

e = x x = x

x 1 P2
=
E 2 EA 2

(6.210)

Luego, la energa total U viene dada por:

ex Volumen = L A

P2
2 EA 2

U=

P2L
2 EA

(6.211)

La energa de deformacin (por unidad de volumen) tambin puede ser expresada por:

e =

1
1
I 2
II dev
6(3 + 2 )
2
144244
3 1424
3

Universidad de Castilla- La Mancha


Ciudad Real - Espaa

vol

Draft

(6.212)

forma

Por: Eduardo W. V. Chaves (2014)

6 ELASTICIDAD LINEAL

583

Considerando:
x
ij = 0
0

0 0
P
0 0 I = x =
A
0 0

(6.213)

Clculo de II dev :
1
I2

II dev = (3 II I 2 ) = = x
3
3
3

(6.214)

Luego, la densidad de energa de deformacin asociada al cambio de volumen:

e vol =

1
(1 2 ) 2 (1 2 ) 2
I 2 =
I =
x
6(3 + 2 )
6E
6E

(6.215)

(1 2 ) P 2
(por unidad de volumen)
6 E A2

(6.216)

e vol =

Densidad de energa de deformacin asociada al cambio de forma:

(6.217)

(1 + ) x
(1 + ) P 2
(por unidad de volumen)
=
E
3
3E A 2

(6.218)

e forma =
e forma =

2
(1 + ) x
1
1 2(1 + )
II dev =
II dev =

E
E
2
2
3
2

Comprobacin:
(1 2 ) P 2 (1 + ) P 2
+
=
6 E A2
3E A 2
P2
[(1 2 ) + 2(1 + )] =
=
6 EA2
P2
[1 2 + 2 + 2 ] =
=
6 EA2
P2
=
= e
2 EA2

e vol + e forma =

En el caso de solo a flexin, y adems teniendo las siguientes relaciones:


y =
y =

M y
I

12 M y
a4

donde

I=

a4
12

y = E y y =

y
E

Densidad de energa de deformacin:


1

1 12 M y y 1 12 M y 12 M y 72 M 2 y 2
=
=
E 2 a 4
Ea 4
Ea8

e = y y = 4
2
2 a

Universidad de Castilla- La Mancha


Ciudad Real - Espaa

Draft

(6.219)

Por: Eduardo W. V. Chaves (2014)

584

MECNICA DEL MEDIO CONTINUO: PROBLEMAS RESUELTOS

Universidad de Castilla- La Mancha


Ciudad Real - Espaa

Draft

Por: Eduardo W. V. Chaves (2014)


Bibliografia

Bibliografa Bsica
ASARO, R.J. & LUBARDA, V.A. (2006). Mechanics of solids and materials. Cambridge University
Press, New York, USA.
BATRA, R. C. (2006). Elements of Continuum Mechanics. John Wiley & Sons Ltd., United
Kingdom.
CASANOVA, J.C. (1993). Ejercicios de elasticidad. Editorial UPV.
CHADWICK, P. (1976). Continuum mechanics concise theory and problems. George Allen & Unwin
Ltd.Great Britain.
CHAVES, E.W.V. (2009). Mecnica del medio continuo: Modelos Constitutivos. CIMNE, Barcelona,
Espaa.
GOICOLEA, J.M. Mecnica del medio continuo web: http://w3.mecanica.upm.es/mmc-ig/
HOLZAPFEL, G.A. (2000). Nonlinear solid mechanics. John Wiley & Sons Ltd. England.
MASE, G.E. (1977). Teora y problemas de mecnica del medio continuo. McGraw-Hill, USA.
OLIVER, X. & AGELET DE SARACBAR, C. (2000). Mecnica de medios continuos para ingenieros.
Ediciones UPC, Barcelona, Espaa.
OLIVER, X. & AGELET DE SARACBAR, C. (2000). Cuestiones y problemas de mecnica de medios
continuos. Ediciones UPC, Barcelona, Espaa.
ORTIZ BERROCAL, L. (1985). Elasticidad. E.T.S. de Ingenieros Industriales. Litoprint. U.P.
Madrid.
PARKER, D.F. (2003). Fields, Flows and Waves: An introduction to continuum models. SpringerVerlag London, UK.
TAYLOR, J.R. (2005). Classical Mechanics. University Science Books. USA.

586

MECNICA DEL MEDIO CONTINUO: PROBLEMAS RESUELTOS

Bibliografa Complementaria
ALEXANDER, H. (1968). A constitutive relation for rubber-like material. Int. J. Eng. Sci., Vol.
6, pp. 549-563.
ANTMAN, S.S. (1995). Nonlinear Problems of Elasticity. Springer-Verlag, New York.
ARRUDA, E.M. & BOYCE, M.C. (1993). A three-dimensional constitutive model for the large
stretch behavior of rubber elastic materials. J. Mech. Phys. Solids, Vol. 41, N. 2, pp. 389412.
ASARO, R.J. & LUBARDA, V.A. (2006). Mechanics of solids and materials. Cambridge University
Press, New York, USA.
BAAR, Y. & WEICHERT (2000). Nonlinear continuum mechanics of solids: fundamental concepts and
perspectives. Springer Verlag. Berlin.
BATRA, R. C. (2006). Elements of Continuum Mechanics. John Wiley & Sons Ltd., United
Kingdom.
BAANT, Z.P. & KIM, S.-S. (1979). Plastic-Fracturing Theory for Concrete. J. Engng. mech.
Div. ASCE, 105, 407.
BAANT, Z.P. & PLANAS, J. (1997). Fracture and size effect in concrete and other quasibrittle
materials, CRC Press LLC, USA.
BEER, F.P. & JOHNSTON, E.R: (1987). Vector Mechanics for Engineers: Dynamics. Seventh
Edition. 2 Volumes. McGraw-Hill Science/Engineering/Math; 4 edition.
BEER, F.P.; JOHNSTON, E.R & CLAUSEN, W.E. (2004). Instructor's and Solutions Manual to
Accompany Vector Mechanics for Engineers - Dynamics. Seventh Edition. 2 Volumes.
McGraw Hill Higher Education; Seventh edition (2004)
BIGONI, D. (2000). Bifurcation and instability of non-associative elastoplastic solids. CISM
Lecture Notes. Material Instabilities in elastic an plastic Solids, H. Petryk (IPPT, Warsaw)
Coordinator.
BIOT, M.A. (1954). Theory of stress-strain relations in anisotropic viscoelasticity and
relaxation phenomena. Jour. Appl. Phys., pp. 1385-1391.
BIOT, M.A. (1955). Variational principles in irreversible thermodynamics with application to
viscoelasticity. The Physical Reviwe 97, pp. 1463-1469.
BIOT, M.A. (1956). Thermoelasticity and irreversible thermodynamics. Jour. Appl. Phys., pp.
240-253.
BLATZ, P.J. & KO, W.L. (1962). Application of finite elasticity theory to the deformation of
rubbery material. Transactions of the Society of Rheology, Vol. VI, pp. 223-251.
BONET, J. & WOOD, R.D. (1997). Nonlinear continuum mechanics for finite element
analysis. Cambridge University Press, USA.
BUECHE, F. (1960). Molecular basis for the Mullins effect. J. Appl. Poly. Sci., 4(10), 107-114.
BUECHE, F. (1961). Mullins effect and rubber-filler interaction. J. Appl. Poly. Sci., 5(15), 271281.
CAROL, I. & WILLAM, K. (1997) Application of analytical solutions in elasto-plasticity to
locaization analysis of damage models. In Owen,. D.R.J., Oate, E., and Hinton, E.
(Eds.), Computational Plasticity (COMPLAS V), pp. 714-719, Barcelona. Pineridge Press.

BIBLIOGRAFA

587

CAROL, I.; RIZZI, E. & WILLAM, K. (1998) On the formulation of isotropic and anisotropic
damage. Computational Modelling of Concrete Structures. de borst, Biani, Mang & Meschke
(eds.). Balkema, Rotterdam, ISBN 9054109467. pp. 183-192.
CASANOVA, J.C. (1993). Ejercicios de elasticidad. Editorial UPV.
CERVERA RUIZ, M. & BLANCO DAZ, E. (2001). Mecnica de Estructuras Libro 1 Resistencia de
materiales. Edicions UPC, Barcelona. Eapaa.
CHABOCHE, J.L. (1979). Le concept de contrainte effective appliqu llasticit et la
viscoplasticit en presence dun endommagement anitrope. Colloque EUROMECH
115, Grenoble Edition du CNRS.
CHADWICK, P. (1976). Continuum mechanics concise theory and problems. George Allen & Unwin
Ltd.Great Britain.
CHANDRASEKHARAIAH, D.S. & DEBNATH, L. (1994). Continuum mechanics. Academic Press,
San Diego (CA, U.S.A.).
CHAVES, E.W.V.(2007). Mecnica del Medio Continuo: Conceptos Bsicos. CIMNE (Centro
Internacional de Mtodos Numricos en la Ingeniera)-Barcelona. ISBN: 978-8496736-38-2. (3 edicin).
CHAVES, E.W.V.(2009). Mecnica del Medio Continuo: Modelos Constitutivos. CIMNE (Centro
Internacional de Mtodos Numricos en la Ingeniera)-Barcelona. ISBN: 978-8496736-68-9. (2 edicin).
CHAVES, E.W.V. & MNGUEZ, R. (2009). Mecnica Computacional en la Ingeniera con Aplicaciones
en MATLAB. Editorial UCLM, ISBN:978-84-692-8273-1. (in Spanish).
CHEN, A. & CHEN, W.F.(1975). Constitutive relations for concrete. J. Eng. Mech.-ASCE,
101:465-481.
CHEN, W.F. & HAN, D.J.(1988). Plasticity for Structural Engineers. Springer-Verlag New Yor
Inc.
CHEN, W.F. (1982). Plasticity in reinforced concrete. McGraw-Hill, Inc. USA.
CHUNG, T.J. & KIM, J.Y. (1984). Two-dimensional, combined-mode heat transfer by
conduction, convective and radiation in emitting, Absorbing, and scattering media.
ASME Trans. J. Heat Transfer 106, pp.:448-452.
CHUNG, T.J. (1996). Applied continuum mechanics, Cambridge University Press.
COLEMAN, B.D. & GURTIN, M.E. (1967). Thermodynamics with internal sate variables. J.
Chem. Phys. 47, pp. 597-613.
COLEMAN, B.D. (1964). Thermodynamics of materials with memory. Arch. Rat. Mech. Anal.
17, pp. 1-46.
CRISFIELD, M.A. (1997). Non-Linear Finite Element Analysis of Solids and Structures, volume 1,2.
John Wiley and Sons, New York, USA.
CRISTENSEN, R. M. (1982). Theory of Viscoelasticity, second edition. Dover Publications, Inc.,
New York, USA.
DESAI, C.S. & SIRIWARDANE, H.J. (1984). Constitutive laws for engineering materials with emphasis
on geological materials. Prentice-Hall, Inc.USA.
DAZ DEL VALLE, J. (1984). Mecnica de los Medios Continuos I. Servicio de publicaciones
E.T.S. de Ingenieros de Caminos, C. y P. Santander.

588

MECNICA DEL MEDIO CONTINUO: PROBLEMAS RESUELTOS

DOBLAR, M. & ALARCN, M. (1983). Elementos de Plasticidad. Sevicio de publicaciones de la


E.T.S.I. Industriales. Madrid.
DRAGON, A. & MRZ, Z. (1979). A Continuum Model for Plastic-Brittle Behavior of Rock
and Concrete. Int. J. Engng. Sci., 17, 121.
DRUCKER, D.C. (1950). Stress-strain relations in the plastic range A survey of the theory
and experiment. Report to the Office of Naval Research, under contact N7-onr-358,
Division of Applied Mathematics, Brown University, Providence, RI.
FARIA, R. & OLIVER, X. (1993). A rate dependent plastic-damage constitutive model for
large scale computations in concrete structures. Monograph CIMNE N17, International
Center for Numerical Methods in Engineering, Barcelona, Spain.
FELIPPA,C.A. (2002). Introduction to Finite Element Methods. Course Notes, see World Wide
Web.
FINDLEY, W.N.; LAI, J.S. & ONARAN, K.(1989). Creep and relaxation of nonlinear viscoelastic
materials: with an introduction to linear viscoelasticity. Dover Publications, Inc. NY.
FUNG, Y.C. (1965). Foundations of solids mechanics. Prentice Hall Inc., New Jersey.
FUNG, Y.C. (1977). A first course in continuum mechanics. Prentice Hall Inc., New Jersey.
GENT, A.N. (1996). A new constitutive relation for rubber. Rubber Chem. Technol., Vol. 69.
pp. 59-61.
GERE, J.M. & WEAVER JR., W. (1965). Analysis of Framed Structures. Van Nostrand Reinhold,
U.S.
GREEN, A.E. & NAGHDI, P.M. (1965). A general theory of an elasto-plastic continuum.
Arch. Rat. Mech. Anal., 18.
GREEN, A.E. & NAGHDI, P.M. (1971). Some remarks on elastic-plastic deformation at finite
strain. Int. J. Engng. Sci., Vol. 9, pp. 1219-1229.
GURTIN, M. E. (1996). An introduction to continuum mechanics. NY: Academic Press, Inc.
GURTIN, M.E. & FRANCIS, E.C. (1981). Simple rate-independent model for damage. J.
Spacecraft, 18(3) pages: 285-286.
HAUPT, P. (2002). Continuum mechanics and theory of materials. Springer-Verlag, Gemany.
HILL, R. (1950). The mathematical theory of plasticity. Oxford University Press.
HILL, R. (1959). Some Basic Principles in the Mechanics of Solids without a natural Time. J.
Mech. Phys. Solids, 7, 209.
HOLZAPFEL, G.A. (2000). Nonlinear solid mechanics. John Wiley & Sons Ltd. England.
IORDACHE, M.-M. (1996). Failure Analysis of Classical and Micropolar Elastoplastic Materials.
Ph.D. Dissertation, University of Colorado at Bolder.
JIRSEK, M. (1998). Finite elements with embedded cracks. LSC Internal Report 98/01,
April.
JU, J.W. (1989). Energy-based coupled elastoplastic damage models at finite strains. Journal
of Engineering Mechanics, Vol.115, No. 11, pp-2507-2525.
KACHANOV, L. M. (1986). Introduction to Continuum, Damage Mechanics. Nijhoff, Dordrecht,
The Netherlands.
KOITER, W.T. (1953). Stress-strain relations, uniqueness and variational theorems for
elastic-plastic material with singular yield surface. Q. Appl. Math. 11, 350-54.

BIBLIOGRAFA

589

LAI, W.M.; RUBIN, D. & KREMPL, E.(1978). Introduction to Continuum Mechanics. Pergamon
Press.
LAIER, J.E.; BAREIRO, J.C., (1983). Complemento de resistncia dos materiais. Publicao 073/92
So Carlos - USP - EESC.
LANCZOS, C. (1970). The variational principles of mechanics. Dover Publications, Inc., New
York.
LEE, E.H. (1969). Elastic-Plastic deformation at finite strains. Journal of Applied Mechanics,
Vol. 36, pp. 1-6.
LEE, E.H. (1981). Some comments on elastic-plastic analysis. Int. J. Solids Structures, Vol. 17,
pp. 859-872.
LEMAITRE, J. & CHABOCHE, J.-L. (1990). Mechanics of Solids materials. Cambridge University
Press, Cambridge.
LINDER, C. (2003). An arbritary lagrangian-Eulerian finite element formulation for dynamics and finite
strain plasticity models. Masters Thesis. University Stuttgart.
LOVE, A.E.H. (1944). A treatise on the Mathematical Theory of Elasticity. Cambridge University
Press, London.
LU, S.C.H. & PISTER, K.S. (1975). Descomposition of deformation and representation of
the free energy function for isotropic solids, Int. J. of Solids and Structures, 11, pages: 927934.
LUBARDA, V.A. & BENSON D.J. (2001). On the partitioning of the rate of deformation
gradient in phenomenological plasticity. Int. J. of Solids and Structures, 38 pages: 68056814.
LUBARDA, V.A. & KRAJCINOVIC, D. (1995). Some fundamental issues in rate theory of
damage-elastoplasticity. Int. J. of Plasticity, Vol. 11, N 7, pp. 763-797.
LUBARDA, V.A. (2004). Constitutive theories based on the multiplicative decomposition of
deformation gradient: Thermoelasticity, elastoplasticity, and biomechanics. Appl. Mech.
Rev., Vol. 57, no 2, March.
LUBLINER, J. (1990). Plasticity Theory. Macmillan Publishing Company, New York.
MALVERN, L.E. (1969). Introduction to the mechanics of a continuous medium. Prentice-Hall, Inc.
Englewood Cliffs, New Jersey.
MARSDEN, J. E. & HUGHES, T. J.R. (1983). Mathematical foundations of elasticity. Dover
Publications, Inc., New York.
MASE, G.E. (1977). Mecnica del Medio Continuo. McGraw-Hill, USA.
MAZARS, J. & LEMAITRE, J. (1984). Application of continuous damage mechanics to strain
and fracture behavior of concrete. Advances of Fracture Mechanics to Cementitious
Composites, NATO Advanced Research Workshop, 4-7 September 1984, Northwestern
University (Edited by S.P. Shah), pp. 375-388.
MAZARS, J. (1982). Mechanical damage and fracture of concrete structures. Advances in
Fracture Research (Fracture 81), Vol. 4, pp. 1499-1506. Pergamon Press, Oxford.
MENDELSON, A. (1968). Plasticity: Theory and application. New York, Robert E. Krieger
Publishing Company.
MOONEY, M. (1940). A theory of large elastic deformation. Journal of Applied Physics, Vol.
11, pp. 582-92.

590

MECNICA DEL MEDIO CONTINUO: PROBLEMAS RESUELTOS

MORMAN, K.N. (1986). The generalized strain measure with application to


nonhomegeneous deformation in rubber-like solids. J. Appl. Mech., Vol. 53, pp. 726728.
MORZ, Z. (1967). On the description of anisotropic work hardening. J. Mech. Phys. Solids,
15, 163.
MULLINS, L. (1969). Softening of rubber by deformation. Rubber Chemistry and Tecnology, 42,
339-351.
NAGHDI, P.M. (1960). Stress-strain relations in plasticity and thermoplasticity. in E.H. Lee
& P. Symonds, eds., Plasticity. Pergamon, Oxford, pp. 121-69.
NEMAT-NASSER, S. (1982). On finite deformation elasto-plasticity. Int. J. Solids Structures,
Vol. 18, pp. 857-872.
NOWACKI, W. (1967). Problems of thermoelasticity. Progress in Thermoelasticity, VIIIth
European Mechanics Colloquium, Warszawa.
OGDEN, R.W. (1984). Non-linear elastic deformations. Dover Publications, Inc., New York.
OLIVER, J. & AGELET DE SARACBAR, C. (2000). Mecnica de medios continuos para ingenieros.
Ediciones UPC, Barcelona, Espaa.
OLIVER, X. & AGELET DE SARACBAR, C. (2000). Cuestiones y problemas de mecnica de medios
continuos. Ediciones UPC, Barcelona, Espaa.
OLIVER, J. (2000). On the discrete constitutive models induced by strong discontinuity
kinematics and continuum constitutive equations. Int. J. Solids Struct., 37:7207-7229.
OLIVER, J. (2002). Topics on Failure Mechanics. Monograph CIMNE N 68, International
Center for Numerical Methods in Engineering, Barcelona, Spain.
OLIVER, J.O. ; CERVERA, M. ; OLLER, S. & LUBLINER, J. (1990). Isotropic damage models
and smeared crack analysis of concrete, SCI-C Second Int. Conf. On Computer Aided Design
of Concrete Structure, Zell am See, Austria. Pg 945 957.
OLLER, S. (1988). Un modelo de dao continuo para materiales friccionales, Ph.D. thesis.
Universidad Politcnica de Catalua, Barcelona, Espaa.
OLLER, S. (2001). Fractura mecnica. Un enfoque global. CIMNE, Barcelona, Espaa.
OLLER, S.; OATE, E.; OLIVER, J. & LUBLINER, J. (1990). Finite element non-linear analysis
of concrete structures using a plastic-damage model. Engineering Fracture Mechanics Vol.
35, pp. 219-231.
OATE, E. (1992). Clculo de estructuras por el mtodo de elementos finitos anlisis esttico lineal.
Centro Internacional de Mtodos Numricos en Ingeniera. Barcelona- Espaa.
ORTIZ BERROCAL, L. (1985). Elasticidad. E.T.S. de Ingenieros Industriales. Litoprint. U.P.
Madrid.
ORTIZ, M. (1985). A Constitutive theory for inelastic behavior of concrete. Mech. Mater.,
Vol. 4, 67.
PABST, W. (2005). The linear theory of thermoelasticity from the viewpoint of rational
thermomechanics. Ceramics - Silikty, 49 (4) 242-251.
PARKER, D.F. (2003). Fields, Flows and Waves: An introduction to continuum models. SpringerVerlag London, UK.
PILKEY, W. & WUNDERLICH, W. (1992). Mechanics of structures. Variational and computational
methods. CRC Press, Inc., Florida, USA.

BIBLIOGRAFA

591

POWERS, J.M. (2004). On the necessity of positive semi-definite conductivity and Onsager
reciprocity in modeling heat conduction in anisotropic media. Journal of Heat Transfer,
Vol.126, pp. 670-675.
PRAGER, W. (1945). Strain hardening under combined stress. J. Appl. Phys. 16, 837-40.
PRAGER, W. (1955). The theory of plasticity: A survey of recent achievements. Proc. Inst.
Mech. Eng. 169:41.
PRANDTL, L. (1924). Spannungverteilung in plastischen korpen, in Proceedings of the First
International. Congress of Applied Mechanics, Delft, The Netherlands, Vol. 43.
RABOTNOV, Y.N. (1963). On the equations of state for creep. Progress in Applied Mechanics,
Prager Anniversary Volume, page 307, New York, MacMillan.
RANKINE, W.J.M. (1951). Laws of the elasticity of solids bodies. Cambridge Dublin Math.
J. 6, 41-80.
REUSS, A. (1930). Berrucksichtingung der elastischen formanderungen in der
plastizitatstheorie, Z. Angew. Math. Mech. 10, 266.
ROMANO, A.; LANCELLOTA, R. & MARASCO, A. (2006). Continuum Mechanics using
Mathematica: fundamentals, applications, and scientific computing. Birkhauser Boston. USA.
RUNESSON, K. & MROZ, Z. (1989). A note on nonassociated plastic flow rules. Int. J.
Plasticity, 5, 639-658.
RUNESSON, K. & OTTOSEN, N. (1991). Discontinuity bifurcation of elastic-plastic solutions
at plane stress and plane strain. Int. J. Plasticity, 7:99-121.
SANSOUR, C. (1998). Large strain deformations of elastic shell. Constitutive modeling and
finite element analysis. Comp. Mech. Appl. Mech. Engrg. 161, pp1-18.
SANSOUR, C.; FEIH, S. & WAGNER, W. (2003). On the performance of enhanced strain
finite elements in large strain deformations of elastic shells. Comparison of two classes
of constitutive models for rubber materials. Report Universitat Karlsruhe, Institut fr
Baustatik
SECHLER, E. (1952). Elasticity in Engineering. John Willey & Sons, Inc. new York.
ILHAV, M. (1997). The mechanics and thermodynamics of continuous media. SpringerVerlag, Germany.
SIMO, J. & HUGHES, T.J.R. (1998). Computational Inelasticity. Springer-Verlag, New York.
SIMO, J.C. & JU, J.W. (1987a). Strain and Stress Based Continuum damage Models I.
Formulation. International Journal Solids Structures, Vol. 23, pp. 821-840.
SIMO, J.C. & JU, J.W. (1987b). Strain and Stress Based Continuum damage Models II.
Computational aspects. International Journal Solids Structures, Vol. 23, pp. 841-869.
SIMO, J.C. (1988a). A framework for finite strain elastoplasticity based on maximum plastic
dissipation: Part I. Computer Methods in Applied Mechanics and Engineering, Vol. 66, pp. 199219.
SIMO, J.C. (1988b). A framework for finite strain elastoplasticity based on maximum plastic
dissipation: Part II. Computer Methods in Applied Mechanics and Engineering, Vol. 68, pp. 131.
SIMO, J.C. (1992). Algorithms for static and dynamic multiplicative plasticity that preserve
the classical return mapping schemes of the infinitesimal theory, Computer Methods in
Applied Mechanics and Engineering, Vol. 99, pp. 61-112.
SOKOLNIKOFF, I.S. (1956). Mathematic theory of elasticity. New York, McGraw-Hill.

592

MECNICA DEL MEDIO CONTINUO: PROBLEMAS RESUELTOS

SOUZA NETO, E.A.; PERI, D. & OWEN, D.R.J. (1998). A phenomenological threedimensional rate-independent continuum damage model for highly filled polymers:
Formulation and computational aspects. J. Mech. Phys. Solids. 42(10), pp. 1533-1550.
SOUZA NETO, E.A.; PERI, D. & OWEN, D.R.J. (1998). Continuum Modelling and
Numerical Simulation of Material Damage at Finite Strains, Archives of Computational
Methods in Engineering, Vol.5,4, pp. 311-384.
SPENCER, A.J.M. (1980). Continuum Mechanics, Longmans, Hong-Kong.
TAYLOR, J.R. (2005). Classical Mechanics. University Science Books. USA.
TIMOSHENKO, S. & GOODIER, J.N. (1951). Theory of elasticity, 2nd edition, McGraw-Hill.
TRELOAR, L.R.G. (1944). Stress-strain data for vulcanized rubber under various types of
deformation. Proc. of the Faraday Soc, 40:59-70.
TRELOAR, L.R.G. (1975). The physics of rubber elasticity. Clarendon Press, Oxford.
TRESCA, H. (1864). Mmire sur lEcoulement des corps solids soumis a de fortes pressions
comptes rendua academie de sciences. Paris, France, Vol.59, p.754..
TRUESDELL, C.A. & NOLL, W. (1965). The non-linear field theories of mechanics, in
Handuch der Physik, Vol. III/3, S. Flgge (Ed.), Springer-Verlag, Berlin.
UGURAL, A.C. (1981). Stress in Plates and Shells. McGraw Hill.
VON MISES, R. (1930). ber die bisherigen Anstze in der lassischen MEchanik der
Kontinua. in Proceedings of the Third Intenational Congress for Applied Mechanics.
Vol.2, pp1-9.
VUJOEVI, L. & LUBARDA, V.A. (2002). Finite-Strain thermoelasticity based on
multiplicative decomposition of deformation gradient. Theoretical and Applied Mechanics,
vol. 28-29, pp. 379-399, Belgrade.
WILLAM, K. (2000). Constitutive models for materials: Encyclopedia of Physical Science & Technology,
3rd edition. Academic Press.
YEOH, O.H. (1993). Some forms of the strain energy function for rubber. Rubber Chem.
Technol., Vol. 66, pp. 754-71.
ZIEGLER, H. (1959). A modification of Pragers hardening rule. Q. Appl. Math. 17, 55-64.
ZIENKIEWICZ, O.C. & TAYLOR, R.L. (1994a). El mtodo de los elementos finitos. Volumen 1:
Formulacin bsica y problemas lineales. CIMNE, Barcelona, 4 edicin.
ZIENKIEWICZ, O.C. & TAYLOR, R.L. (1994b). El mtodo de los elementos finitos. Volumen 2:
Mecnica de slidos y fluidos. Dinmica y no linealidad. CIMNE, Barcelona, 4 edicin.

S-ar putea să vă placă și